Usmle step 2 second edition 072208 part 1

Page 1

CLINICAL REVIEW FOR THE USMLE STEP 2

CLINICAL REVIEW FOR THE USMLE STEP 2 SECOND EDITION


ii

USMLE STEP 2

www.ClinicalReview.com


Clinical Review for the USMLE Step 2

iii

CLINICAL REVIEW FOR THE USMLE STEP 2 SECOND EDITION

SURGISPHERE CORPORATION United States of America 2008


iv

USMLE STEP 2 Clinical Review for the USMLE Step 2 Copyright © 2008 by Surgisphere Corporation. All Rights Reserved

No part of this book may be utilized or reproduced in any form or by any means, mechanical or electronic, including photocopying, recording, electronic storage, virtual or actual without written permission by the Surgisphere Corporation. You may not alter or remove any notice of copyright or ownership from this content. The Surgisphere Corporation is the owner of various trademarks used within this publication. Images contained within are believed to be in the public domain; if a particular image is not referenced or used appropriately, please contact us so that the appropriate arrangements can be made. Cover image courtesy of Dr. Arthur W. Toga, Laboratory of Neuro Imaging at UCLA. USMLE is a trademark of the Federation of State Medical Boards and National Board of Medical Examiners®. Inquiries should be addressed to:

Surgisphere Corporation 3604 Witherspoon Blvd Suite 111 Durham, North Carolina, USA 27707 www.Surgisphere.com

Second Edition

Cataloging-In-Publication Data Surgisphere Corporation Clinical Review for the USMLE Step 2 690 p. cm. 1. Medicine. 2. Surgery I. Surgisphere Corporation II. Clinical Review for the USMLE Step 2 ISBN 9780980210378 C6168 617 SUR 2008901178

Although information herein is based on the author’s extensive experience and knowledge, it is not intended to be a substitute for the services of a qualified healthcare professional.

01 / 10 9 8 7 6 5 4 3 2 Second Edition © Surgisphere Corporation 2008. All Rights Reserved. First published in 2003. PRINTED IN THE UNITED STATES OF AMERICA

www.ClinicalReview.com


Sections

v

SECTIONS SECTIONS CLINICAL REVIEW FOR THE USMLE STEP 2 .......................... I SECTIONS .......................................................................... V TABLE OF CONTENTS ....................................................... VII TABLE OF FIGURES ........................................................ XVII ABBREVIATIONS ........................................................... XXV DEDICATION ................................................................ XXIX IMAGE CREDITS ........................................................... XXIX SYMBOLS ...................................................................... XXX FOREWORD ...................................................................... 2 INTRODUCTION ................................................................ 4 Overview of Topics ........................................................... 4 Organization of Text ......................................................... 4 Study Plan ......................................................................... 4 Feedback .......................................................................... 5 CONCEPTS IN BASIC SCIENCE ............................................. 8 Pharmacology ................................................................... 8 Infection and Antibiotics ................................................ 11 Perioperative Management ........................................... 27 Fever and Sepsis ............................................................. 28 Practice Questions .......................................................... 33 BIOSTATISTICS AND EPIDEMIOLOGY................................ 38 Epidemiology .................................................................. 44 Preventive Medicine....................................................... 45 Immunology.................................................................... 48 Practice Questions .......................................................... 64 NORMAL DEVELOPMENT ................................................ 66 Theories of Development ............................................... 66 Life Cycle......................................................................... 68 Ego .................................................................................. 75 Conditioning ................................................................... 80 Developmental Disorders ............................................... 82 PSYCHIATRY .................................................................... 92 Psychotic Disorders ........................................................ 92 Mood Disorders .............................................................. 97 Anxiety Disorders ......................................................... 101 Cognitive Disorders ...................................................... 108 Dissociative Disorders .................................................. 112 Somatoform Disorders ................................................. 114 Malingering and Factitious Disorders ........................... 115

Personality Disorders ................................................... 116 Substance Abuse Disorders .......................................... 120 Antipsychotics .............................................................. 135 Antidepressants ........................................................... 139 Mood Stabilizers .......................................................... 143 Anxiolytics .................................................................... 146 Other Medications ....................................................... 149 Major Adverse Drug Effects ......................................... 152 MEDICAL ETHICS ........................................................... 162 Introduction ................................................................. 162 General Concepts ......................................................... 162 HEMATOLOGY AND ONCOLOGY .................................... 170 Basic Science ................................................................ 170 Microcytic Anemia ....................................................... 173 Macrocytic Anemia ...................................................... 176 Normocytic Anemia ..................................................... 177 Specific Anemias .......................................................... 180 Other Red Blood Cell Conditions .................................. 186 Platelets and Coagulation ............................................ 188 Oncology ...................................................................... 192 B-Cell Lymphomas ........................................................ 199 T-Cell Lymphomas ........................................................ 201 Practice Questions ....................................................... 203 EMERGENCY MEDICINE ................................................. 208 Toxicology .................................................................... 208 Other Injuries ............................................................... 210 Practice Questions ....................................................... 214 NERVOUS SYSTEM ........................................................ 220 General Concepts ......................................................... 220 Central Disturbances .................................................... 227 Cerebrovascular Disease .............................................. 229 Headache ..................................................................... 235 Sensory Disturbances ................................................... 239 Infectious Diseases ....................................................... 241 Neurodegenerative Disorders ...................................... 246 Epilepsy ........................................................................ 251 Cancer .......................................................................... 253 Practice Questions ....................................................... 256

Â


USMLE STEP 2 vi RESPIRATORY SYSTEM ................................................... 260 Basic Science ................................................................ 260 Upper Respiratory Infections and Illnesses .................. 263 Lower Respiratory Infections ....................................... 267 Obstructive Lung Disease ............................................. 273 Restrictive Lung Disease ............................................... 279 Pulmonary Disease ....................................................... 282 Lung Cancer .................................................................. 287 Mediastinal Disease ..................................................... 289 Practice Questions ....................................................... 290 CARDIOVASCULAR SYSTEM ........................................... 294 Basic Science ................................................................ 294 Studies and Procedures................................................ 297 Congenital Heart Defects ........................................ 301 Coronary Heart Disease................................................ 302 Congestive Heart Failure (CHF) .................................... 313 Valvular Heart Disease ................................................. 317 Cardiomyopathy ........................................................... 325 Myocarditis .................................................................. 329 Pericardial Disease ....................................................... 330 Endocardial Disease ..................................................... 333 Arrhythmia ................................................................... 336 Aortic Diseases ............................................................. 342 Common Cardiovascular Medications ......................... 344 Vascular Disorders........................................................ 345 Practice Questions ....................................................... 351 GASTROINTESTINAL SYSTEM ......................................... 358 Basic Science ................................................................ 358 Esophagus .................................................................... 359 Stomach ....................................................................... 369 Small Intestine .............................................................. 378 Large Intestine.............................................................. 384 Hernias ......................................................................... 402 Practice Questions ....................................................... 404 HEPATOPANCREATOBILIARY SYSTEM ............................ 412 Basic Science ................................................................ 412 Hepatic Disease ............................................................ 420 Pancreatic Disorders .................................................... 428 Spleen ........................................................................... 431 Biliary Disease .............................................................. 431 Cancer and Transplantation ......................................... 434 Practice Questions ....................................................... 436

ENDOCRINE SYSTEM ...................................................... 444 Basic Science ................................................................ 444 Hypothalamus-Pituitary Axis ....................................... 446 Thyroid Gland .............................................................. 451 Parathyroid Gland ........................................................ 458 Adrenal Gland .............................................................. 461 Pancreas....................................................................... 467 Metabolic Disorders..................................................... 472 Cancer .......................................................................... 481 Practice Questions ....................................................... 482 GENITOURINARY SYSTEM .............................................. 490 Basic Science ................................................................ 490 Fluids and Electrolytes ................................................. 496 Acid-Base Disorders ..................................................... 504 Renal Failure ................................................................ 508 Glomerular and Nephrotic Disease.............................. 512 Genitourinary Infections .............................................. 526 Sexually-Transmitted Diseases .................................... 531 Practice Questions ....................................................... 539 OBSTETRICS AND GYNECOLOGY ..................................... 542 Basic Science ................................................................ 542 Women’s Issues ........................................................... 543 Pregnancy and Prenatal Care....................................... 544 Physiology of the Fetus ................................................ 550 Screening Tests in Pregnancy ...................................... 551 Normal and Abnormal Labor and Delivery .................. 555 Monitoring During Labor ............................................. 556 Cancer in Women ........................................................ 556 Practice Questions ....................................................... 562 SKIN AND SOFT TISSUE .................................................. 568 Introduction ................................................................. 568 Practice Questions ....................................................... 583 MUSCULOSKELETAL SYSTEM .......................................... 588 Basic Science ................................................................ 588 Anatomic Disorders ..................................................... 588 Metabolic Bone Diseases ............................................. 592 Infectious Bone Diseases ............................................. 595 Autoimmune and Inflammatory Disease ..................... 599 Cancer .......................................................................... 616 Practice Questions ....................................................... 617 HIGH-YIELD TOPICS........................................................ 622 INDEX ............................................................................ 632

www.ClinicalReview.com


TABLE OF CONTENTS TABLE OF CONTENTS CLINICAL REVIEW FOR THE USMLE STEP 2 .......................... I SECTIONS .......................................................................... V TABLE OF CONTENTS ....................................................... VII TABLE OF FIGURES ........................................................ XVII ABBREVIATIONS ........................................................... XXV DEDICATION ................................................................ XXIX IMAGE CREDITS ........................................................... XXIX SYMBOLS ...................................................................... XXX FOREWORD ...................................................................... 2 INTRODUCTION ................................................................ 4 Overview of Topics ........................................................... 4 Organization of Text ......................................................... 4 Study Plan ......................................................................... 4 Feedback .......................................................................... 5 CONCEPTS IN BASIC SCIENCE ............................................. 8 Pharmacology ................................................................... 8 Pharmacokinetics

................................................. 8

Pharmacodynamics .............................................. 9 Efficacy and Potency ................................................ 10 Drug Development................................................... 10 Infection and Antibiotics ................................................ 11 Surgical Prophylaxis

............................................ 11

Wound Contamination

....................................... 11

Common Antibiotics ........................................... 12 Common Infections.................................................. 17 Perioperative Management ........................................... 27 Indicators of Morbidity and Mortality ................ 27 Fever and Sepsis ............................................................. 28 Fever

................................................................... 28

Systemic Inflammatory Response Syndrome Sepsis Trisomy

..... 29

.................................................................. 29 ............................................................... 30

Growth Factors Hereditary Tumors

................................................... 30 ............................................. 31

Oncogenes / Carcinogens

................................... 32

Tumor Markers ................................................... 32 Practice Questions .......................................................... 33 BIOSTATISTICS AND EPIDEMIOLOGY................................ 38 Biostatistics .............................................................. 38 Study Design ............................................................ 43

Epidemiology.................................................................. 44 Preventive Medicine ...................................................... 45 Use of Tests .............................................................. 45 Routine Screening .................................................... 45 Cancer Screening ...................................................... 46 Exposure ................................................................... 47 Immunology ................................................................... 48 Immunization ........................................................... 48 Travel ........................................................................ 50 Anatomy of the Immune System.............................. 51 Regulation and Activation of Immunologic Pathways ............................................................................ 53 Cells of the Immune System ..................................... 54 Cytokines and Growth Factors ................................. 55 Complement ....................................................... 57 Immunologic Disorders ............................................ 58 Transplant Rejection ........................................... 60 Findings in Disease ................................................... 61 Practice Questions ......................................................... 64 NORMAL DEVELOPMENT ................................................ 66 Theories of Development............................................... 66 Freud’s Stages .......................................................... 66 Oedipal Period .......................................................... 66 Erikson’s Stages ........................................................ 67 Piaget’s Stages .......................................................... 67 Progression of Stages ............................................... 68 Life Cycle ........................................................................ 68 The Neonate

....................................................... 68

Infancy ................................................................. 69 The Toddler .............................................................. 70 Preschoolers ............................................................. 71 School-Age................................................................ 72 Teenagers ................................................................. 73 Early Adulthood ........................................................ 74 Middle Age ............................................................... 74 The Elderly ................................................................ 74 Ego ................................................................................. 75 Ego Integrity vs. Despair ........................................... 75 Ego Defenses ............................................................ 76 Mature Ego Defenses ............................................... 76 Immature Ego Defenses ........................................... 77 Conditioning ................................................................... 80 Introduction.............................................................. 80


viii

USMLE STEP 2

Classical Conditioning ............................................... 80 Operant Conditioning ............................................... 81 Reinforcement .......................................................... 81 Reinforcement Schedules ......................................... 82 Developmental Disorders............................................... 82 Introduction .............................................................. 82 Interview Strategy .................................................... 83 Psychological Tests ................................................... 83 Developmental Disorders ......................................... 83 PSYCHIATRY .................................................................... 92 Psychotic Disorders ........................................................ 92 Psychosis................................................................... 92 Schizophrenia ...................................................... 92 Schizophreniform Disorder....................................... 94 Brief Psychotic Disorder ........................................... 95 Schizoaffective Disorder ........................................... 95 Delusional Disorder .................................................. 96 Mood Disorders.............................................................. 97 Mood and Affect....................................................... 97 Subtypes ................................................................... 97 Unipolar Mood Disorders Bipolar I Disorder

.................................... 98

.............................................. 100

Bipolar II Disorder ............................................. 101 Anxiety Disorders ......................................................... 101 Anxiety Disorders .............................................. 101 Subtypes ................................................................. 101 Panic Disorder

................................................... 102

Specific Phobia

.................................................. 104

Social Phobia

..................................................... 105

Generalized Anxiety Disorder

........................... 105

Posttraumatic Stress Disorder

........................... 106

Obsessive-Compulsive Disorder ........................ 107 Cognitive Disorders ...................................................... 108 Illusions ................................................................... 108 Delusions ................................................................ 108 Hallucinations ......................................................... 108 Delirium .................................................................. 109 Dementia ................................................................ 110 Amnestic Disorders................................................. 111 Dissociative Disorders .................................................. 112 Definition ................................................................ 112 Amnesia .................................................................. 112 Ganser Syndrome ................................................... 112 Dissociative Fugue .................................................. 113 Dissociative Identity Disorder................................. 113

Depersonalization Disorder ................................... 114 Somatoform Disorders................................................. 114 Presentation ........................................................... 114 Somatization Disorder............................................ 114 Other Somatoform Disorders................................. 115 Malingering and Factitious Disorders .......................... 115 Malingering ....................................................... 115 Factitious Disorders ............................................... 116 Personality Disorders ................................................... 116 Classification of Personality Disorders ................... 116 Cluster A Personality Disorders .............................. 117 Cluster B Personality Disorders .............................. 117 Cluster C Personality Disorders .............................. 119 Treatment .............................................................. 120 Substance Abuse Disorders ......................................... 120 Substance Abuse and Dependence ........................ 120 Alcohol-Induced Disorders ..................................... 121 Anxiolytic, Sedative, and Hypnotic Substance Abuse ............................................................................... 124 Opioid Abuse .......................................................... 126 Amphetamine Abuse ............................................. 127 Other Substance Abuse Disorders ......................... 129 Antipsychotics .............................................................. 135 Indications .............................................................. 135 Mechanism of Action ............................................. 135 Side Effects ............................................................. 136 Therapy and Drug Reactions .................................. 136 Administration and Side Effects ............................. 138 Antidepressants ........................................................... 139 Indication ............................................................... 139 Mechanism of Action ............................................. 139 Therapy and Drug Reactions .................................. 140 Side Effects ............................................................. 141 Mood Stabilizers .......................................................... 143 Indications .............................................................. 143 Mechanisms of Action............................................ 144 Other Mood Stabilizers .......................................... 146 Anxiolytics .................................................................... 146 Indications .............................................................. 147 Mechanisms of Action............................................ 147 Therapy and Drug Reactions .................................. 147 Other Medications ....................................................... 149 Psychostimulants ................................................... 149 Major Adverse Drug Effects ......................................... 152 Extrapyramidal Symptoms ..................................... 152 Dystonia ................................................................. 152 Akinesia .................................................................. 153 www.ClinicalReview.com


Table of Contents Akathisia ................................................................ 153 Tardive Dyskinesia ................................................. 154 Neuroleptic Malignant Syndrome.......................... 155 Serotonin Syndrome .............................................. 156 Antidepressants – SSRIs

.................................... 157

Antidepressants – TCAs .................................... 157 Antidepressants – MAOIs ...................................... 157

ix

Anemia of Chronic Disease ............................... 177 Sideroblastic Anemia .............................................. 178 Aplastic Anemia ................................................ 179 Specific Anemias .......................................................... 180 Fanconi Anemia ...................................................... 180

Antipsychotics – Typical

................................... 158

Hemolytic Anemia ............................................. 180 Cold Hemolytic Anemia .......................................... 181 Paroxysmal Hemolytic Anemia (PHA)..................... 182 Paroxysmal Nocturnal Hemoglobinuria ................. 183

Antipsychotics - –typical

................................... 158

Transfusion Reactions

Antidepressants – Atypical

............................... 158

Anxiolytics – Typical

......................................... 158

Anxiolytics – Atypical

........................................ 159

Barbiturates

...................................................... 159

Mood Stabilizers – Typical

................................ 159

Mood Stabilizers – Atypical

.............................. 159

Psychostimulants

.............................................. 159

MEDICAL ETHICS ........................................................... 162 Introduction .................................................................. 162 General Concepts ......................................................... 162 Autonomy ......................................................... 162 Nonmaleficence ..................................................... 162 Beneficence ........................................................... 162 Ethical Decision Making ......................................... 163 Illness in Minors ..................................................... 163 Noncompliance ...................................................... 163 Demanding Patients............................................... 163 Angry Patients........................................................ 163 Patient-Physician Relationships Informed Consent

............................................. 164

Hereditary Spherocytosis (HS) .......................... 184 Glucose-6-Phosphatase Dehydrogenase Deficiency (G6PD) ............................................................... 185 Other Red Blood Cell Conditions .................................. 186 Polycythemia Vera (PV) .......................................... 186 Porphyria ................................................................ 187 Platelets and Coagulation ............................................ 188 Immune Thrombocytopenic Purpura (ITP) Von Willebrand Disease

....... 188

.................................... 189

Hemophilia A ..................................................... 189 Hemophilia B .......................................................... 190 Vitamin K Deficiency............................................... 191 Malaria ................................................................... 191 Oncology ...................................................................... 192 Acute Myelocytic Leukemia (AML)

................... 192

Acute Lymphocytic Leukemia (ALL) .................. 193 Chronic Myeloid Leukemia (CML) .......................... 194 Chronic Lymphocytic Leukemia (CLL) ................ 195 Monoclonal Gammopathy of Uncertain Significance ................................................................................ 196

......................................... 165

Multiple Myeloma (MM)

.................................. 196

.................................................. 166

Hodgkin Lymphoma (HL)

................................... 197

Advanced Directives Confidentiality

........................ 164

....................................... 184

HEMATOLOGY AND ONCOLOGY .................................... 170 Basic Science................................................................. 170 Pharmacology ........................................................ 170 Microcytic Anemia ........................................................ 173 Iron-Deficiency Anemia

.................................... 173

Sickle Cell Anemia (SCA)

................................... 173

Alpha-Thalassemia ........................................... 174 Beta-Thalassemia ................................................... 175 Macrocytic Anemia ....................................................... 176 Megaloblastic Anemia ...................................... 176 Normocytic Anemia ...................................................... 177

Non-Hodgkin Lymphoma .................................. 198 B-Cell Lymphomas ........................................................ 199 Precursor B- and T-Cell Lymphomas Peripheral B-Cell Lymphomas

................. 199

........................... 200

Plasma Cell Disorders ........................................ 201 T-Cell Lymphomas ........................................................ 201 Peripheral T-Cell Lymphomas

........................... 201

Hodgkin Lymphoma .......................................... 202 Chemotherapy Regimens ....................................... 202 Practice Questions ....................................................... 203 EMERGENCY MEDICINE ................................................. 208


x

USMLE STEP 2 Toxicology .................................................................... 208 Other Injuries ............................................................... 210 Burns....................................................................... 210 Heatstroke .............................................................. 210 Hypothermia........................................................... 211 Radiation Toxicity ................................................... 211 Electrocution .......................................................... 211 Drowning ................................................................ 212 Anaphylaxis............................................................. 212 Practice Questions ....................................................... 214

NERVOUS SYSTEM ......................................................... 220 General Concepts ......................................................... 220 Brachial Plexus ........................................................ 220 Reflexes ............................................................. 220 Nerve Regeneration ............................................... 221 Cerebral Perfusion Pressure .............................. 221 Pharmacology ......................................................... 221 Central Disturbances .................................................... 227 Spinal Cord Compression .................................. 227 Syringomyelia ......................................................... 227 Subacute Combined Degeneration (SACD)............. 228 Anterior Spinal Artery Infarction ............................ 228 Cerebrovascular Disease .............................................. 229 Epidemiology .......................................................... 229 Etiology ................................................................... 229 Pathophysiology of Ischemic Stroke ....................... 229 Pathophysiology of Hemorrhagic Stroke ................ 230 Types of Stroke ....................................................... 230 Presentation ........................................................... 230 Anterior Cerebral Artery ......................................... 231 Middle Cerebral Artery ........................................... 231 Posterior Cerebral Artery ....................................... 231 Vertebrobasilar Artery............................................ 231 Posterior Inferior Cerebellar Artery........................ 232 Anterior Inferior Cerebellar Artery ......................... 232 Lacunar Infarcts ...................................................... 232 Diagnosis Through Imaging .................................... 233 Glasgow Coma Scale (GCS) ..................................... 233 General Treatment ................................................. 234 Treatment of Stroke .......................................... 234 Treatment of TIAs ................................................... 234 Complications ......................................................... 234 Epidural Hemorrhage

........................................ 235

Subdural Hemorrhage ....................................... 235 Subarachnoid Hemorrhage .................................... 235

Headache ..................................................................... 235 Introduction ........................................................... 235 Migraine Headache

........................................... 236

Tension Headache

............................................ 236

Cluster Headache .............................................. 237 Giant Cell Arteritis (GCA)........................................ 237 Normal Pressure Hydrocephalus (NPH) ................. 238 Sensory Disturbances .................................................. 239 Acute Blindness ...................................................... 239 Hearing Loss ........................................................... 239 Vertigo.................................................................... 240 Infectious Diseases ...................................................... 241 Rabies ..................................................................... 241 Bacterial Meningitis

.......................................... 242

Viral Meningitis ................................................. 243 Fungal Meningitis ................................................... 243 Viral Encephalitis .................................................... 244 Brain Abscess ......................................................... 245 Botulism ................................................................. 245 Neurodegenerative Disorders ..................................... 246 Alzheimer Disease (AD)

.................................... 246

Parkinson Disease (PD)

..................................... 247

Huntington Disease (HD) .................................. 248 Sleep Disorders ...................................................... 249 Epilepsy ........................................................................ 251 Etiology and Pathophysiology ................................ 251 Presentation – Tonic-Clonic Seizures Presentation – Absence Seizures Presentation – Atonic Seizures

................ 251

...................... 251 ......................... 251

Presentation – Myoclonic Seizures

................... 251

Presentation – Partial Seizures ......................... 251 Presentation – Status Epilepticus and Preictal Symptoms ......................................................... 252 Diagnosis ................................................................ 252 Treatment ......................................................... 252 Complications......................................................... 252 Cancer .......................................................................... 253 Tumors ................................................................... 254 Parotid Tumor ................................................... 255 Practice Questions ....................................................... 256 RESPIRATORY SYSTEM ................................................... 260 Basic Science ................................................................ 260 Anatomy ................................................................. 260 Physiology .............................................................. 260 www.ClinicalReview.com


Table of Contents

Pharmacology ........................................................ 263 Upper Respiratory Infections and Illnesses .................. 263 Postnasal Drip ........................................................ 263 Otitis Media ...................................................... 264 Viral Pharyngitis ..................................................... 265 Bacterial Pharyngitis

......................................... 266

Acute and Chronic Cough ................................. 267 Sinusitis .................................................................. 267 Lower Respiratory Infections........................................ 267 Pertussis ................................................................. 267 Influenza

........................................................... 268

Pneumonia

....................................................... 269

Tuberculosis

..................................................... 270

Bronchitis .......................................................... 271 Lung Abscess .......................................................... 272 Obstructive Lung Disease ............................................. 273 Asthma

............................................................. 273

Chronic Obstructive Pulmonary Disease (COPD) ............................................................................... 276 Bronchiectasis ........................................................ 277 Sleep Apnea ...................................................... 278 Restrictive Lung Disease ............................................... 279 Idiopathic Pulmonary Fibrosis (IPF)

.................. 279

Sarcoidosis ........................................................ 279 Pneumoconiosis ..................................................... 280 Asbestosis .............................................................. 281 Silicosis ................................................................... 281 Coal Miner’s Lung .................................................. 281 Farmer’s Lung ........................................................ 282 Pulmonary Disease ....................................................... 282 Adult Respiratory Distress Syndrome (ARDS) Atelectasis

... 282

........................................................ 283

Spontaneous Pneumothorax

............................ 283

Tension Pneumothorax .................................... 284 Pleural Effusion ...................................................... 285 Pulmonary Embolism (PE) ................................ 285 Lung Cancer .................................................................. 287 Epidemiology ......................................................... 287 Etiology .................................................................. 287 Presentation .......................................................... 287 Diagnosis

.......................................................... 288

Features and Treatment of SCLC

...................... 288

Features and Treatment of NSCLC ................... 288 Prognosis ............................................................... 288

xi Screening ................................................................ 288 Complications ......................................................... 288 Mediastinal Disease ..................................................... 289 Mediastinitis ........................................................... 289 Pneumomediastinum (Mediastinal Emphysema) .. 290 Practice Questions ....................................................... 290 CARDIOVASCULAR SYSTEM ........................................... 294 Basic Science ................................................................ 294 Anatomy ................................................................. 294 Physiology ......................................................... 295 Pharmacology ......................................................... 296 Studies and Procedures ............................................... 297 Electrocardiogram .................................................. 297 Echocardiogram ..................................................... 299 Treadmill Stress Echocardiogram (TSE) .................. 300 Thallium Imaging .................................................... 300 Cardiac CatheTErization ......................................... 300 Causes of Chest Pain .............................................. 301 Congenital Heart Defects ........................................ 301 Coronary Heart Disease ............................................... 302 Introduction............................................................ 302 Risk Factors for CHD .......................................... 302 Dyslipidemia ........................................................... 303 Hypertension

..................................................... 304

Malignant Hypertension Angina

................................... 306

............................................................... 306

Myocardial Infarction (MI) ................................ 308 Congestive Heart Failure (CHF) .................................... 313 Etiology ................................................................... 313 Pathophysiology ..................................................... 314 Cardiovascular Compensation ................................ 314 Renal Compensation .............................................. 314 Presentation – RHF ................................................. 315 Presentation – LHF ................................................. 315 Diagnosis

........................................................... 315

Treatment of Diastolic Dysfunction

.................. 316

Treatment of Systolic Dysfunction .................... 316 Complications ......................................................... 316 Valvular Heart Disease ................................................. 317 Introduction............................................................ 317 Mitral Stenosis

.................................................. 318

Mitral Regurgitation (MR) Mitral Valve Prolapse (MVP) Aortic Stenosis (AS)

................................. 319 ............................. 320

........................................... 320


xii

USMLE STEP 2 Aortic Regurgitation (AR) .................................. 322 Tricuspid Stenosis (TS) ............................................ 323 Tricuspid Regurgitation (TR) ................................... 324

Rheumatic Fever (RF) ........................................ 325 Cardiomyopathy ........................................................... 325 Introduction ............................................................ 325 Dilated Cardiomyopathy (DCM)

........................ 326

Hypertrophic Cardiomyopathy (HCM)

.............. 327

Restrictive Cardiomyopathy (RCM) ................... 328 Myocarditis .................................................................. 329 Etiology ................................................................... 329 Pathophysiology ..................................................... 329 Presentation and Diagnosis .................................... 329 Treatment ............................................................... 329 Pericardial Disease ....................................................... 330 Introduction ............................................................ 330 Pericarditis

........................................................ 330

Pericardial Tamponade ..................................... 331 Constrictive Pericarditis .......................................... 332 Endocardial Disease ..................................................... 333 Introduction ............................................................ 333 Infective Endocarditis ........................................ 333 Libman-Sacks Endocarditis (LSE) ............................ 334 Nonbacterial Thrombotic Endocarditis (NBTE)....... 335 Arrhythmia ................................................................... 336 Introduction ............................................................ 336 Premature Contractions ......................................... 336 Sinus Bradycardia

.............................................. 336

Sinus Tachycardia .............................................. 337 Paroxysmal Atrial Tachycardia (PAT) ...................... 337 Atrial Flutter

...................................................... 338

Atrial Fibrillation

................................................ 338

Atrioventricular (AV) Block

................................ 339

Ventricular Arrhythmias .................................... 340 Wolff-Parkinson-White Syndrome (WPW) ............. 341 Torsade de Pointes ................................................. 342 Aortic Diseases ............................................................. 342 Introduction ............................................................ 342 Aortic Dissection

............................................... 342

Abdominal Aortic Aneurysm (AAA) ................... 343 Common Cardiovascular Medications ......................... 344 Nitrates ................................................................... 344 Beta-Blockers .......................................................... 344 Aspirin..................................................................... 344 Heparin ................................................................... 345

Streptokinase and Alteplase .................................. 345 ACE-Inhibitors ........................................................ 345 Digoxin ................................................................... 345 Vascular Disorders ....................................................... 345 Introduction ........................................................... 345 Churg-Strauss Disease ...................................... 346 Takayasu Arteritis................................................... 346 Wegener Granulomatosis ...................................... 347 Henoch-Schonlein Purpura .................................... 347 Polyarteritis Nodosa ............................................... 348 Thrombotic Thrombocytopenic Purpura (TTP)

. 349

Avascular Necrosis ............................................ 350 Practice Questions ....................................................... 351 GASTROINTESTINAL SYSTEM ......................................... 358 Basic Science ................................................................ 358 Pharmacology ........................................................ 358 Esophagus .................................................................... 359 Esophageal Disorders ............................................. 359 Anatomic Esophageal Defects................................ 364 Esophageal Cancer ............................................ 367 Stomach ....................................................................... 369 Anatomy

........................................................... 369

Gastritis and Ulcer Disease ............................... 371 Other Gastric Disorders.......................................... 376 Small Intestine ............................................................. 378 Introduction ........................................................... 378 Malabsorption Disorders ....................................... 378 Large Intestine ............................................................. 384 Introduction ........................................................... 384 Diarrhea and Constipation ..................................... 385 Irritable and Inflammatory Bowel Disease............. 389 Diverticular Disease ............................................... 393 Gastrointestinal Bleeding ....................................... 395 Cancers ................................................................... 397 Hernias ......................................................................... 402 Femoral Hernia

................................................. 402

Inguinal Hernia ................................................. 403 Complications of Hernia Repair ............................. 403 Practice Questions ....................................................... 404 HEPATOPANCREATOBILIARY SYSTEM ............................ 412 Basic Science ................................................................ 412 Epidemiology.......................................................... 412 Anatomy Physiology

........................................................... 412 ......................................................... 413 www.ClinicalReview.com


Table of Contents

Biochemistry ..................................................... 414 Pharmacology ........................................................ 415 Bile Acid Resins

................................................. 415

HMG-CoA Reductase Inhibitors Fibrates

........................ 415

............................................................. 415

Other Agents .................................................... 416 Osmotic Agents ...................................................... 416 Pathology ............................................................... 417 Liver Function Tests (LFTs) ..................................... 418 Diagnostic Studies .................................................. 419 Outpatient Procedures .......................................... 419 Serum-Ascites Albumin Gradient (SAAG) .............. 419 Hepatic Disease ............................................................ 420 Cirrhosis

............................................................ 420

Primary Biliary Cirrhosis ................................... 421 Gilbert Disease ....................................................... 422 Crigler-Najjar Disease ............................................ 422 Dubin-Johnson Disease .......................................... 423 Rotor Disease ......................................................... 423 Hemochromatosis.................................................. 423 Wilson Disease ....................................................... 423 Alpha-1-Antitrypsin Deficiency

......................... 423

Hepatitis ........................................................... 424 Pancreatic Disorders..................................................... 428 Introduction ........................................................... 428 Acute Pancreatitis

............................................ 428

Chronic Pancreatitis ......................................... 430 Spleen ........................................................................... 431 Splenectomy ..................................................... 431 Biliary Disease............................................................... 431 Cholelithiasis

.................................................... 431

Choledocholithiasis Gallstone Ileus Cholecystitis

.......................................... 432

.................................................. 432 ..................................................... 432

Ascending Cholangitis ....................................... 433 Primary Sclerosing Cholangitis (PSC) ..................... 434 Cancer and Transplantation ......................................... 434 Pancreatic Adenocarcinoma

............................. 434

Hepatic Adenoma ............................................. 435 Hepatic Hemangioma ............................................ 435 Hepatic Angiosarcoma ........................................... 435 Hepatocellular Carcinoma ................................ 435 Practice Questions ........................................................ 436

xiii ENDOCRINE SYSTEM ..................................................... 444 Basic Science ................................................................ 444 Pharmacology ......................................................... 444 Hypothalamus-Pituitary Axis ........................................ 446 Introduction............................................................ 446 Hyperprolactinemia .......................................... 446 Acromegaly............................................................. 447 Hypopituitarism ...................................................... 448 Empty Sella Syndrome............................................ 448 Diabetes Insipidus

............................................. 449

Secretion of Inappropriate Antidiuretic Hormone ................................................................................ 450 Pituitary Tumors ..................................................... 450 Thyroid Gland ............................................................... 451 Introduction............................................................ 451 Hyperthyroidism ............................................... 451 Subacute Thyroiditis ............................................... 453 Thyroid Storm

................................................... 453

Hypothyroidism ................................................. 454 Myxedema Coma ................................................... 455 Papillary Thyroid Cancer

................................... 455

Follicular Thyroid Cancer .................................. 456 Medullary Thyroid Cancer ...................................... 457 Anaplastic Thyroid Cancer ...................................... 457 Parathyroid Gland ........................................................ 458 Introduction............................................................ 458 Primary Hyperparathyroidism

........................... 458

Secondary Hyperparathyroidism ...................... 459 Tertiary Hyperparathyroidism ................................ 459 Hypoparathyroidism............................................... 460 Pseudohypoparathyroidism (PHP) ......................... 461 Adrenal Gland .............................................................. 461 Introduction............................................................ 461 Cushing Syndrome ............................................ 461 Pseudo-Cushing Syndrome..................................... 462 Congenital Adrenal Hyperplasia ............................. 463 Conn Syndrome

................................................. 464

Hypoaldosteronism

........................................... 465

Adrenal Insufficiency

......................................... 465

Pheochromocytoma .......................................... 466 Pancreas ....................................................................... 467 Introduction............................................................ 467 Type 1 Diabetes Mellitus

................................... 468

Type 2 Diabetes Mellitus

................................... 468

Â


USMLE STEP 2

xiv

Diabetic Ketoacidosis (DKA) .............................. 470 Hyperosmolar Hyperglycemic Nonketotic Coma ... 470 Hypoglycemia .................................................... 471 Metabolic Disorders ..................................................... 472 Obesity

.............................................................. 472

Anorexia Nervosa

.............................................. 473

Bulimia Nervosa ................................................ 474 Malnutrition ........................................................... 475 Vitamin Deficits ................................................. 476 Cancer .......................................................................... 481

Respiratory Alkalosis ......................................... 506 Renal Tubular Acidosis (RTA) ................................. 507 Renal Failure ................................................................ 508 Acute Renal Failure (ARF)

................................. 508

Acute Tubular Necrosis (ATN) ........................... 509 Allergic Interstitial Nephritis (AIN) ......................... 510 Other Causes of Intrinsic Renal Failure .................. 510 Uremic Syndrome

............................................. 511

End Stage Renal Disease (ESRD) ....................... 511 Glomerular and Nephrotic Disease.............................. 512

MEN 1

................................................................ 481

Acute Pyelonephritis

......................................... 512

MEN 2

................................................................ 481

Perinephric Abscess

.......................................... 513

Nephrotic Syndrome

......................................... 514

MEN 3 ................................................................ 481 Presentation and Diagnosis .................................... 482 Treatment ............................................................... 482 Practice Questions ....................................................... 482 GENITOURINARY SYSTEM .............................................. 490 Basic Science ................................................................ 490 Normal Electrolytes ................................................ 490 Anion Gap .......................................................... 490 Fractional Excretion of Sodium............................... 491 Body Water............................................................. 491 Serum Osmolarity ................................................... 491 Arterial Blood Gas (ABG)

................................... 492

Dialysis .............................................................. 493 Pharmacology ......................................................... 494 Fluids and Electrolytes ................................................. 496 Hyponatremia

................................................... 496

Hypernatremia

.................................................. 497

Hypokalemia

..................................................... 497

Hyperkalemia .................................................... 498 Hypocalcemia ......................................................... 499 Hypercalcemia ................................................... 500 Cancer-Induced Hypercalcemia .............................. 501 Hypophosphatemia ................................................ 501 Hyperphosphatemia ............................................... 502 Hypomagnesemia ................................................... 503 Hypermagnesemia.................................................. 504 Acid-Base Disorders ..................................................... 504 Anion Gap Metabolic Acidosis

.......................... 504

Non-Anion Gap Metabolic Acidosis

................... 505

Acute Glomerulonephritis

................................ 514

Poststreptococcal Glomerulonephritis (PSGN)

. 515

Rapidly Progressive Glomerulonephritis (RPGN) ............................................................................... 516 Membranoproliferative Glomerulonephritis (MPGN) ............................................................................... 517 IgA Nephropathy .................................................... 518 Goodpasture Syndrome ......................................... 519 Alport Syndrome .................................................... 519 Adult Polycystic Kidney Disease Nephrolithiasis

........................ 520

.................................................. 521

Incontinence ..................................................... 522 Erectile Disorders (ED) ........................................... 523 Benign Prostatic Hyperplasia (BPH)

.................. 524

Prostate Cancer ................................................ 525 Genitourinary Infections .............................................. 526 Urinary Tract Infections .................................... 526 Urethritis ................................................................ 527 Vaginitis.................................................................. 528 Pelvic Inflammatory Disease ............................. 529 Balanitis .................................................................. 529 Prostatitis ............................................................... 530 Proctitis and Anusitis.............................................. 530 Sexually-Transmitted Diseases .................................... 531 Syphilis ................................................................... 531 Chancroid ............................................................... 532 Lymphogranuloma Venereum ............................... 533 Granuloma Inguinale.............................................. 533

Metabolic Alkalosis

........................................... 505

Genital Herpes .................................................. 534 Genital Warts ......................................................... 535

Respiratory Acidosis

.......................................... 506

Gonorrhea

......................................................... 536 www.ClinicalReview.com


Table of Contents HIV and AIDS ..................................................... 536 Practice Questions ........................................................ 539 OBSTETRICS AND GYNECOLOGY .................................... 542 Basic Science................................................................. 542 Pharmacology ........................................................ 542 Women’s Issues ............................................................ 543 Introduction ........................................................... 543 Domestic Violence ................................................. 543 Pregnancy and Prenatal Care ....................................... 544 B-hCG ..................................................................... 544 Ultrasound ............................................................. 544 Term Pregnancy ..................................................... 544 Development of Pregnancy ................................... 544 Blood Tests............................................................. 545 Special Tests........................................................... 545 Triple Screen .......................................................... 546 Estrogen ................................................................. 546 Progesterone ......................................................... 546 Maternal Physiology .............................................. 546 Antepartum Complications .................................... 547 Physiology of the Fetus................................................. 550 Placental Physiology .............................................. 550 Fetal Physiology ..................................................... 550 Fetal Assessment ................................................... 551 Screening Tests in Pregnancy ....................................... 551 Introduction ........................................................... 551 Chromosomal Defects ........................................... 551 Down Syndrome (Trisomy 21) .......................... 551 Edward Syndrome (Trisomy 18)............................. 552 Patau Syndrome (Trisomy 13) ............................... 552 Turner Syndrome (45, XO) ..................................... 552 Klinefelter Syndrome (47, Y) .................................. 552 Genetic Screening Tests ......................................... 552 Developmental Defects .................................... 554 Diagnostic Tests ..................................................... 554 Normal and Abnormal Labor and Delivery ................... 555 Rupture of Membranes ......................................... 555 First Stage of Labor ................................................ 555 Second Stage of Labor ........................................... 555 Third Stage of Labor ............................................... 556 Monitoring During Labor .............................................. 556 Fetal Heart Rate ..................................................... 556 Decelerations and Variability ................................. 556 Cancer in Women ......................................................... 556 Breast Cancer

................................................... 556

Endometrial Cancer

.......................................... 558

xv

Ovarian Cancer

.................................................. 558

Cervical Cancer

.................................................. 559

Breast Cancer

.................................................... 560

Ovarian Disease

................................................ 561

Cervical, Uterine, and Vaginal Disease ........ 562 Practice Questions ....................................................... 562 SKIN AND SOFT TISSUE .................................................. 568 Introduction ................................................................. 568 Impetigo

............................................................ 568

Erysipelas

.......................................................... 568

Cellulitis ............................................................. 569 Staphylococcus Scalded Skin Syndrome ................. 569 Toxic Shock Syndrome ...................................... 570 Candidiasis.............................................................. 570 Tinea Versicolor ................................................ 571 Scabies .................................................................... 571 Molluscum Contagiosum .................................. 571 Lyme Disease .......................................................... 572 Herpes Zoster .................................................... 572 Bacillary Angiomatosis ........................................... 573 Cat Scratch Disease ................................................ 573 Malakoplakia .......................................................... 574 Necrotizing Fasciitis

.......................................... 574

Rocky Mountain Spotted Fever ......................... 575 Scarlet Fever ........................................................... 575 Actinomycosis......................................................... 576 Aspergillosis ............................................................ 576 Coccidioidomycosis ................................................ 576 Sporotrichosis ......................................................... 577 Tinea Corporis

................................................... 577

Tinea Pedis ........................................................ 578 Lice ......................................................................... 578 Varicella Zoster Virus ........................................ 578 Hand-Foot-Mouth Disease ..................................... 579 Rubella .............................................................. 579 Measles .................................................................. 580 Meningococcemia ............................................. 580 Disseminated Gonococcal Infection ....................... 581 Sjögren SLE

.............................................................. 581

..................................................................... 582

Scleroderma

...................................................... 582

Autoimmune Disorders ..................................... 583 Practice Questions ....................................................... 583


USMLE STEP 2 xvi MUSCULOSKELETAL SYSTEM.......................................... 588 Basic Science ................................................................ 588 Erythrocyte Sedimentation Rate (ESR) ................... 588 Antinuclear Antibodies ........................................... 588 Pharmacology ......................................................... 588 Anatomic Disorders ...................................................... 588 Orthopedic Emergencies ........................................ 588 Pulseless Dislocations ............................................. 588 Compartment Syndrome ................................... 589 Septic Arthritis ........................................................ 589 Developmental Dysplasia of the Hip Legg-Calve-Perthes Disease

................. 589

.............................. 590

Slipped Capital Femoral Epiphysis ..................... 590 Distal Radius Fracture ............................................. 590 Meniscal Injuries..................................................... 590 DeQuervain’s Tenosynovitis ................................... 591 Flexor Tenosynovitis .......................................... 591 Sensory Deficits of the Lower Extremity ................ 591 Low Back Pain .................................................... 591 Metabolic Bone Diseases ............................................. 592 Osteoporosis ..................................................... 592 Paget Disease ......................................................... 594 Osteomalacia .......................................................... 594 Infectious Bone Diseases.............................................. 595 Osteomyelitis

Costochondritis

................................................. 596

Lumbar Disc Herniation .................................... 596 Epidural Abscess .................................................... 597 Cauda Equina Syndrome

................................... 597

Osteoarthritis (OA) ........................................... 598 Autoimmune and Inflammatory Disease ..................... 599 Rheumatoid Arthritis (RA)

................................ 599

Systemic Lupus Erythematosus ........................ 600 Drug-Induced Lupus ............................................... 601 Antiphospholipid Antibody Syndrome ................... 602 Scleroderma

...................................................... 602

Sjögren Syndrome ............................................. 603 Spondyloarthropathies .......................................... 604 Crystalline Arthropathies ....................................... 607 Other Arthritis and Myopathy................................ 609 Cancer .......................................................................... 616 Spinal Metastasis ................................................... 616 Practice Questions ....................................................... 617 HIGH-YIELD TOPICS........................................................ 622 INDEX............................................................................ 632

.................................................... 595

www.ClinicalReview.com


Table of Figures

xvii

TABLE OF FIGURES TABLE OF FIGURES TABLE 1 SURGICAL PROPHYLAXIS................................................. 11 TABLE 2 CLASSIFICATION OF WOUND CONTAMINATION .................. 12 TABLE 3 ANTIMICROBIALS: PENICILLIN.......................................... 12 TABLE 4 ANTIMICROBIALS: FIRST GENERATION CEPHALOSPORINS ...... 13 TABLE 5 ANTIMICROBIALS: SECOND GENERATION CEPHALOSPORINS .. 13 TABLE 6 ANTIMICROBIALS: THIRD GENERATION CEPHALOSPORINS ..... 13 TABLE 7 ANTIMICROBIALS: MONOBACTAMS AND CARBAPENEMS ...... 14 TABLE 8 ANTIMICROBIALS: AMINOGLYCOSIDES .............................. 14 TABLE 9 ANTIMICROBIALS: TETRACYCLINES ................................... 14 TABLE 10 ANTIMICROBIALS: MACROLIDES .................................... 15 TABLE 11 ANTIMICROBIALS: FLUOROQUINOLONES ......................... 15 TABLE 12 ANTIMICROBIALS: SULFONAMIDES AND TRIMETHOPRIMS ... 15 TABLE 13 ANTIMICROBIALS: OTHER DRUGS .................................. 16 TABLE 14 ANTIFUNGALS ............................................................ 16 TABLE 15 ANTIVIRALS ............................................................... 17 TABLE 16 ANTIPARASITICS ......................................................... 17 TABLE 17 GENERAL ANTIBIOTICS................................................. 18 TABLE 18 SOFT TISSUE INFECTIONS ............................................. 18 TABLE 19 MUSCULOSKELETAL INFECTIONS .................................... 19 TABLE 20 PNEUMONIA ............................................................. 20 TABLE 21 GI INFECTIONS ........................................................... 20 TABLE 22 GENITOURINARY INFECTIONS ........................................ 21 TABLE 23 CNS INFECTIONS ........................................................ 21 TABLE 24 HEAD AND NECK INFECTIONS ........................................ 22 TABLE 25 STDS ....................................................................... 22 TABLE 26 FUNGAL INFECTIONS ................................................... 23 TABLE 27 GRAM POSITIVE COCCI: STAPHYLOCOCCUS ...................... 23 TABLE 28 GRAM POSITIVE COCCI: STREPTOCOCCUS........................ 23 TABLE 29 GRAM NEGATIVE COCCI: NEISSERIA ............................... 24 TABLE 30 GRAM POSITIVE RODS: NON-SPORE FORMERS ................ 24 TABLE 31 GRAM POSITIVE RODS: SPORE FORMERS ........................ 24 TABLE 32 GRAM NEGATIVE RODS: AEROBES ................................. 25 TABLE 33 GRAM NEGATIVE RODS: FACULTATIVE ANAEROBES........... 25 TABLE 34 GRAM NEGATIVE RODS: ENTEROBACTERIACEAE ............... 25 TABLE 35 GRAM NEGATIVE RODS: ANAEROBES ............................. 26 TABLE 36 ATYPICAL BACTERIA .................................................... 26 TABLE 37 CUTANEOUS FUNGAL INFECTIONS .................................. 26 TABLE 38 SYSTEMIC FUNGAL INFECTIONS ..................................... 26 TABLE 39 CAUSES OF OSTEOMYELITIS .......................................... 27 TABLE 40 CAUSES OF PHARYNGITIS ............................................. 27 TABLE 41 CAUSES OF PNEUMONIA .............................................. 27 TABLE 42 CAUSES OF URINARY TRACT INFECTIONS ......................... 27

TABLE 43 FEVER--POSTOPERATIVE ............................................... 28 TABLE 44 SYSTEMIC INFLAMMATORY RESPONSE SYNDROME (SIRS) ... 29 TABLE 45 SEPSIS....................................................................... 30 TABLE 46 PREVENTIVE MEDICINE ................................................ 45 TABLE 47 ROUTINE SCREENING SCHEDULE .................................... 46 TABLE 48 CANCER SCREENING .................................................... 47 TABLE 49 EXPOSURE ................................................................. 48 TABLE 50 IMMUNIZATION .......................................................... 51 TABLE 51 THEORIES OF DEVELOPMENT ......................................... 66 TABLE 52 FREUD’S STAGES ......................................................... 67 TABLE 53 ERIKSON’S STAGES ...................................................... 67 TABLE 54 PIAGET’S STAGES ........................................................ 68 TABLE 55 INTERRUPTERS OF DEVELOPMENT .................................. 68 TABLE 56 NEONATAL CHARACTERISTICS ........................................ 69 TABLE 57 DURATION OF NEONATAL REFLEXES ............................... 69 TABLE 58 INFANT CHARACTERISTICS ............................................. 69 TABLE 59 DEVELOPMENT OF INFANT SKILLS ................................... 70 TABLE 60 INFANT MOTOR SKILLS................................................. 70 TABLE 61 TODDLER LANGUAGE DEVELOPMENT .............................. 71 TABLE 62 TODDLER MOTOR SKILLS DEVELOPMENT ......................... 71 TABLE 63 PRESCHOOLERS LANGUAGE DEVELOPMENT ...................... 72 TABLE 64 PRESCHOOLERS MOTOR DEVELOPMENT .......................... 72 TABLE 65 SCHOOL-AGE MOTOR SKILLS DEVELOPMENT ................... 73 TABLE 66 SHAPE DRAWING ABILITY ............................................. 73 TABLE 67 TEENAGE DEVELOPMENT .............................................. 74 TABLE 68 RISK FACTORS FOR TEENAGE PREGNANCY ........................ 74 TABLE 69 CONCERNS OF EARLY ADULTHOOD ................................. 74 TABLE 70 CONCERNS OF MIDDLE AGE .......................................... 74 TABLE 71 ADULT DEVELOPMENT ................................................. 75 TABLE 72 CONCERNS OF THE ELDERLY .......................................... 75 TABLE 73 ERIKSON’S STAGES ...................................................... 75 TABLE 74 EGO DEFENSES ........................................................... 76 TABLE 75 MATURE EGO DEFENSES .............................................. 77 TABLE 76 IMMATURE EGO DEFENSES ........................................... 80 TABLE 77 CLASSICAL CONDITIONING ............................................ 81 TABLE 78 OPERANT CONDITIONING ............................................. 81 TABLE 79 REINFORCEMENT ........................................................ 82 TABLE 80 REINFORCEMENT SCHEDULES ........................................ 82 TABLE 81 COMMONLY USED CHILD DIAGNOSTIC TESTS ................... 83 TABLE 82 DIAGNOSABLE DEVELOPMENTAL DISORDERS .................... 83 TABLE 83 COMMON CAUSES OF MENTAL RETARDATION .................. 84 TABLE 84 DIAGNOSIS OF MENTAL RETARDATION ............................ 85


USMLE STEP 2 xviii TABLE 85 TREATMENT OF MENTAL RETARDATION .......................... 85 TABLE 86 DIAGNOSIS OF AUTISM – DSM 299.00 .......................... 86 TABLE 87 TREATMENT OF AUTISM ............................................... 86 TABLE 88 DIAGNOSIS OF RETT DISORDER-DSM 299.80.................. 86 TABLE 89 DIAGNOSIS OF ADHD .................................................. 87 TABLE 90 TREATMENT OF ADHD ................................................ 88 TABLE 91 DIAGNOSIS OF CONDUCT DISORDERS .............................. 88 TABLE 92 TREATMENT OF CONDUCT DISORDERS............................. 88 TABLE 93 DIAGNOSIS OF OPPOSITIONAL DEFIANT DISORDER – DSM 313.81 .......................................................................... 89 TABLE 94 SYMPTOMS OF TOURETTE DISORDER ............................... 89 TABLE 95 DIAGNOSIS OF TOURETTE DISORDER – DSM 307.23 ........ 89 TABLE 96 TREATMENT OF TOURETTE DISORDER ............................. 89 TABLE 97 FEATURES OF PSYCHOSIS .............................................. 92 TABLE 98 EPIDEMIOLOGY OF SCHIZOPHRENIA ................................ 93 TABLE 99 POSITIVE AND NEGATIVE SYMPTOMS OF SCHIZOPHRENIA .... 93 TABLE 100 DIAGNOSIS OF SCHIZOPHRENIA .................................... 94 TABLE 101 TREATMENT OF SCHIZOPHRENIA .................................. 94 TABLE 102 DIAGNOSIS OF SCHIZOPHRENIFORM DISORDER ............... 95 TABLE 103 DIAGNOSIS OF BRIEF PSYCHOTIC DISORDER .................... 95 TABLE 104 CHARACTERISTICS OF POSTPARTUM PSYCHOTIC DISORDER 95 TABLE 105 DIAGNOSIS OF SCHIZOAFFECTIVE DISORDER ................... 96 TABLE 106 TREATMENT OF SCHIZOAFFECTIVE DISORDER .................. 96 TABLE 107 DIAGNOSIS OF DELUSIONAL DISORDER .......................... 96 TABLE 108 TREATMENT OF DELUSIONAL DISORDER......................... 97 TABLE 109 MOOD VERSUS AFFECT .............................................. 97 TABLE 110 CATEGORIES OF MOOD DISORDERS .............................. 97 TABLE 111 EPIDEMIOLOGY OF MAJOR DEPRESSION ......................... 98 TABLE 112 RISK FACTORS FOR MAJOR DEPRESSION ........................ 98 TABLE 113 SYMPTOMS OF MAJOR DEPRESSION ............................. 99 TABLE 114 PHARMACOLOGICAL TREATMENT FOR MDD .................. 99 TABLE 115 DIAGNOSIS OF BIPOLAR I DISORDER ............................ 100 TABLE 116 TREATMENT OF BIPOLAR I DISORDER .......................... 101 TABLE 117 BIPOLAR II DISORDER ............................................... 101 TABLE 118 ANXIETY DISORDERS ................................................ 102 TABLE 119 TRIGGERS OF PANIC DISORDER .................................. 102 TABLE 120 SYMPTOMS OF PANIC DISORDER ................................ 103 TABLE 121 DIAGNOSIS OF PANIC DISORDER................................. 103 TABLE 122 TREATMENT OF PANIC DISORDER ............................... 104 TABLE 123 DIAGNOSIS OF SPECIFIC PHOBIA ................................. 104 TABLE 124 TREATMENT OF SPECIFIC PHOBIAS .............................. 104 TABLE 125 DIAGNOSIS OF SOCIAL PHOBIA ................................... 105 TABLE 126 TREATMENT OF SOCIAL PHOBIA ................................. 105 TABLE 127 DIAGNOSIS OF GENERAL ANXIETY DISORDER ................ 106 TABLE 128 TREATMENT OF GENERAL ANXIETY DISORDER ............... 106 TABLE 129 PRESENTATION OF POSTTRAUMATIC STRESS DISORDER .. 107 TABLE 130 DIAGNOSIS OF POSTTRAUMATIC STRESS DISORDER ........ 107

TABLE 131 TREATMENT OF POSTTRAUMATIC STRESS DISORDER ...... 107 TABLE 132 DIAGNOSIS OF OBSESSIVE COMPULSIVE DISORDER ........ 108 TABLE 133 TREATMENT OF OBSESSIVE-COMPULSIVE DISORDER ...... 108 TABLE 134 DELUSION VS. ILLUSION VS. HALLUCINATION ................ 109 TABLE 135 DIAGNOSIS OF DELIRIUM-DSM 293.0 ....................... 109 TABLE 136 TREATMENT OF DELIRIUM ........................................ 110 TABLE 137 DELIRIUM VS. DEMENTIA ......................................... 110 TABLE 138 DIAGNOSIS OF AMNESTIC DISORDERS – DSM 294.0 (SPECIFIC), 294.8 (NOT OTHERWISE SPECIFIED) .................... 111 TABLE 139 SYMPTOMS OF DISSOCIATIVE DISORDER ...................... 112 TABLE 140 TYPES OF AMNESIA ................................................. 112 TABLE 141 DIAGNOSIS OF DISSOCIATIVE DISORDER – DSM 300.15 113 TABLE 142 DIAGNOSIS OF DISSOCIATIVE FUGUE – DSM 300.13 .... 113 TABLE 143 TREATMENT OF DISSOCIATIVE FUGUE ......................... 113 TABLE 144 DIAGNOSIS OF DISSOCIATIVE IDENTITY DISORDER-DSM 300.14 ........................................................................ 114 TABLE 145 DIAGNOSIS OF DEPERSONALIZATION DISORDER-DSM 300.6 ................................................................................... 114 TABLE 146 SYMPTOMS OF SOMATOFORM DISORDER .................... 114 TABLE 147 DIAGNOSIS OF SOMATOFORM DISORDER-DSM 300.81 115 TABLE 148 OTHER SOMATOFORM DISORDERS ............................. 115 TABLE 149 MALINGERING ....................................................... 116 TABLE 150 FACTITIOUS DISORDERS VS. MALINGERING ................. 116 TABLE 151 PERSONALITY DISORDER TYPES .................................. 117 TABLE 152 DIAGNOSIS OF PARANOID PERSONALITY DISORDER ....... 117 TABLE 153 DIAGNOSIS OF SCHIZOID PERSONALITY DISORDER ......... 117 TABLE 154 DIAGNOSIS OF SCHIZOTYPAL PERSONALITY DISORDER .... 117 TABLE 155 DIAGNOSIS OF ANTISOCIAL PERSONALITY DISORDER ...... 118 TABLE 156 DIAGNOSIS OF BORDERLINE PERSONALITY DISORDER ..... 118 TABLE 157 DIAGNOSIS OF HISTRIONIC PERSONALITY DISORDER ...... 118 TABLE 158 DIAGNOSIS OF NARCISSISTIC PERSONALITY DISORDER .... 119 TABLE 159 DIAGNOSIS OF AVOIDANT PERSONALITY DISORDER ....... 119 TABLE 160 DIAGNOSIS OF DEPENDENT PERSONALITY DISORDER ..... 119 TABLE 161 DIAGNOSIS OF OBSESSIVE-COMPULSIVE PERSONALITY DISORDER ..................................................................... 120 TABLE 162 TREATMENT OF PERSONALITY DISORDERS.................... 120 TABLE 163 DIAGNOSIS OF SUBSTANCE ABUSE.............................. 121 TABLE 164 DIAGNOSIS OF SUBSTANCE DEPENDENCE..................... 121 TABLE 165 SIGNS AND SYMPTOMS OF ALCOHOL DEPENDENCE/INTOXICATION ............................................ 122 TABLE 166 DIAGNOSIS OF ALCOHOL DEPENDENCE-DSM 303.90 ... 122 TABLE 167 SYMPTOMS OF ALCOHOL WITHDRAWAL-DSM 291.81 . 123 TABLE 168 SYMPTOMS OF DELIRIUM TREMENS ........................... 124 TABLE 169 TREATMENT OF DELIRIUM TREMENS .......................... 124 TABLE 170 TREATMENT OF ALCOHOLISM .................................... 124 TABLE 171 TREATMENT OF ANXIOLYTIC, SEDATIVE, AND HYPNOTIC DRUG ABUSE ................................................................. 125 www.ClinicalReview.com


Table of Figures

TABLE 172 WITHDRAWAL OF ANXIOLYTIC, SEDATIVE, AND HYPNOTIC DRUGS ......................................................................... 125 TABLE 173 DIAGNOSIS OF ANXIOLYTIC, SEDATIVE, AND HYPNOTIC DRUG ABUSE-DSM 292.89 ..................................................... 125 TABLE 174 SYMPTOMS OF OPIOID ABUSE .................................. 126 TABLE 175 DIAGNOSIS OF OPIOID ABUSE-DSM 305.50 .............. 126 TABLE 176 SYMPTOMS OF OPIOID ABUSE WITHDRAWAL .............. 127 TABLE 177 TREATMENT OF OPIOID ABUSE ................................. 127 TABLE 178 SYMPTOMS OF AMPHETAMINE ABUSE ........................ 128 TABLE 179 DIAGNOSIS OF AMPHETAMINE ABUSE-DSM 305.70.... 128 TABLE 180 WITHDRAWAL SYMPTOMS OF AMPHETAMINE ABUSE.... 128 TABLE 181 TREATMENT OF AMPHETAMINE ABUSE ....................... 129 TABLE 182 SYMPTOMS OF MARIJUANA ABUSE ............................ 129 TABLE 183 TREATMENT OF MARIJUANA ABUSE ........................... 130 TABLE 184 DIAGNOSIS OF MARIJUANA ABUSE-DSM 305.20 ........ 130 TABLE 185 SYMPTOMS OF NICOTINE ABUSE ............................... 130 TABLE 186 WITHDRAWAL SYMPTOMS OF NICOTINE ABUSE ........... 130 TABLE 187 TREATMENT OF NICOTINE ABUSE .............................. 130 TABLE 188 DIAGNOSIS OF NICOTINE ABUSE-DSM 305.1 ............. 131 TABLE 189 SYMPTOMS OF CAFFEINE ABUSE................................ 131 TABLE 190 WITHDRAWAL SYMPTOMS OF CAFFEINE ABUSE ........... 131 TABLE 191 DIAGNOSIS OF CAFFEINE ABUSE-DSM 305.90 ........... 132 TABLE 192 SYMPTOMS OF PCP ABUSE ...................................... 132 TABLE 193 WITHDRAWAL SYMPTOMS OF PCP ............................ 133 TABLE 194 DIAGNOSIS OF PCP ABUSE-DSM 305.90 .................. 133 TABLE 195 SYMPTOMS OF LSD ABUSE ...................................... 133 TABLE 196 DIAGNOSIS OF METHAMPHETAMINE ABUSE (MDMA, ECSTASY)-DSM 292.89 ................................................. 134 TABLE 197 WITHDRAWAL SYMPTOMS OF METHAMPHETAMINE-DSM 292.0.......................................................................... 134 TABLE 198 SYMPTOMS OF GAMMA-HYDROXYBUTYRATE (GHB) ABUSE ................................................................................... 134 TABLE 199 ANTIPSYCHOTIC MEDICATIONS.................................. 135 TABLE 200 INDICATIONS FOR ANTIPSYCHOTIC MEDICATION USE ..... 135 TABLE 201 MECHANISM OF ACTION OF ANTIPSYCHOTICS .............. 136 TABLE 202 THERAPY USING TYPICAL ANTIPSYCHOTIC MEDICATIONS 137 TABLE 203 SIDE EFFECTS OF TYPICAL ANTIPSYCHOTIC MEDICATIONS 137 TABLE 204 THERAPY USING ATYPICAL ANTIPSYCHOTIC MEDICATIONS ................................................................................... 137 TABLE 205 SPECIFIC SIDE EFFECTS OF ATYPICAL ANTIPSYCHOTIC MEDICATIONS ............................................................... 137 TABLE 206 GENERAL SIDE EFFECTS OF TYPICAL ANTIPSYCHOTIC DRUGS ................................................................................... 138 TABLE 207 GENERAL SIDE EFFECTS OF ATYPICAL LOW POTENCY ANTIPSYCHOTIC DRUGS ................................................... 138 TABLE 208 ANTIDEPRESSANT MEDICATIONS ............................... 139 TABLE 209 ATYPICAL ANTIDEPRESSANT MEDICATIONS .................. 139

xix TABLE 210 INDICATIONS FOR ANTIDEPRESSANT MEDICATION USE ... 139 TABLE 211 MECHANISM OF ACTION OF ANTIDEPRESSANTS ............ 140 TABLE 212 THERAPY USING ANTIDEPRESSANT MEDICATIONS ......... 140 TABLE 213 SIDE EFFECTS OF SSRI, MAOI, AND TCA THERAPY ....... 141 TABLE 214 SIDE EFFECTS OF ATYPICAL ANTIDEPRESSANTS .............. 142 TABLE 215 TYPICAL MOOD STABILIZER MEDICATIONS ................... 143 TABLE 216 ATYPICAL MOOD STABILIZER MEDICATIONS ................. 143 TABLE 217 INDICATIONS FOR MOOD STABILIZERS ......................... 143 TABLE 218 MECHANISM OF ACTION OF MOOD STABILIZERS ........... 144 TABLE 219 THERAPY USING MOOD STABILIZER MEDICATIONS ........ 144 TABLE 220 SIDE EFFECTS OF LITHIUM CARBONATE ........................ 145 TABLE 221 SIDE EFFECTS OF VALPROATE .................................... 145 TABLE 222 SIDE EFFECTS OF CARBAMAZEPINE ............................. 146 TABLE 223 SIDE EFFECTS OF LAMOTRIGINE.................................. 146 TABLE 224 BENZODIAZEPINES ................................................... 146 TABLE 225 INDICATIONS FOR USE OF BENZODIAZEPINES ................ 147 TABLE 226 THERAPY USING BENZODIAZEPINES ............................ 148 TABLE 227 SIDE EFFECTS OF BENZODIAZEPINES ............................ 148 TABLE 228 PSYCHOSTIMULANTS ................................................ 149 TABLE 229 INDICATIONS FOR USE OF PSYCHOSTIMULANTS ............. 149 TABLE 230 SIDE EFFECTS OF PSYCHOSTIMULANTS ......................... 149 TABLE 231 ANTICHOLINERGICS ................................................. 149 TABLE 232 THERAPY USING ANTICHOLINERGICS ........................... 150 TABLE 233 SIDE EFFECTS OF ANTICHOLINERGICS .......................... 150 TABLE 234 INDICATIONS FOR USE OF BETA-BLOCKERS ................... 150 TABLE 235 SIDE EFFECTS OF USE OF BETA-BLOCKERS .................... 150 TABLE 236 CLONIDINE ............................................................ 151 TABLE 237 COGNITIVE ENHANCERS ........................................... 151 TABLE 238 THYROID HORMONES .............................................. 151 TABLE 239 RISK FACTORS FOR EPS ............................................ 152 TABLE 240 CLASSIFICATION OF EPS ........................................... 152 TABLE 241 TREATMENT OF EPS ................................................ 152 TABLE 242 TREATMENT OF DYSTONIA ........................................ 153 TABLE 243 SYMPTOMS OF SEVERE AKINESIA................................ 153 TABLE 244 TREATMENT OF AKINESIA ......................................... 153 TABLE 245 SYMPTOMS OF SEVERE AKATHISIA .............................. 154 TABLE 246 TREATMENT OF AKATHISIA........................................ 154 TABLE 247 TREATMENT OF TARDIVE DYSKINESIA .......................... 154 TABLE 248 CLASSIFICATION OF DYSTONIC REACTIONS ................... 154 TABLE 249 CAUSES OF NMS .................................................... 155 TABLE 250 SYMPTOMS OF NMS ............................................... 155 TABLE 251 TREATMENT OF NMS .............................................. 156 TABLE 252 SYMPTOMS OF SEROTONIN SYNDROME ....................... 156 TABLE 253 MINOR AUTONOMY ................................................ 162 TABLE 254 CONCEPTS TO KEEP IN MIND WHEN MAKING ETHICAL DECISIONS ..................................................................... 163 TABLE 255 INFORMED CONSENT ............................................... 164


xx

USMLE STEP 2

TABLE 256 EXCEPTIONS TO INFORMED CONSENT .......................... 165 TABLE 257 PATIENT COMPETENCY ............................................. 165 TABLE 258 ADVANCED DIRECTIVES ............................................ 166 TABLE 259 EXCEPTIONS TO CONFIDENTIALITY .............................. 167 TABLE 260 IRON DEFICIENCY ANEMIA ........................................ 173 TABLE 261 SICKLE CELL ANEMIA (SCA) ...................................... 174 TABLE 262 ALPHA-THALASSEMIA ............................................... 175 TABLE 263 BETA-THALASSEMIA ................................................. 176 TABLE 264 MEGALOBLASTIC ANEMIA ......................................... 177 TABLE 265 ANEMIA OF CHRONIC DISEASE (ACD) ......................... 178 TABLE 266 SIDEROBLASTIC ANEMIA ........................................... 179 TABLE 267 APLASTIC ANEMIA ................................................... 179 TABLE 268 FANCONI ANEMIA ................................................... 180 TABLE 269 HEMOLYTIC ANEMIA................................................ 181 TABLE 270 COLD HEMOLYTIC ANEMIA ....................................... 182 TABLE 271 PAROXYSMAL HEMOLYTIC ANEMIA (PHA) ................... 183 TABLE 272 PAROXYSMAL NOCTURNAL HEMOGLOBINURIA (PNH) ... 183 TABLE 273 TRANSFUSION REACTION .......................................... 184 TABLE 274 HEREDITARY SPHEROCYTOSIS (HS) ............................. 185 TABLE 275 GLUCOSE-6-PHOSPHATASE DEHYDROGENASE DEFICIENCY (G6PD) ........................................................................ 186 TABLE 276 POLYCYTHEMIA VERA (PV) ....................................... 186 TABLE 277 PORPHYRIA ............................................................ 188 TABLE 278 IMMUNE THROMBOCYTOPENIC PURPURA (ITP) ............ 189 TABLE 279 VON WILLEBRAND DISEASE (VWD) ............................ 189 TABLE 280 HEMOPHILIA A ....................................................... 190 TABLE 281 HEMOPHILIA B ....................................................... 191 TABLE 282 VITAMIN K DEFICIENCY ............................................ 191 TABLE 283 MALARIA............................................................... 192 TABLE 284 ACUTE MYELOCYTIC LEUKEMIA (AML) ....................... 193 TABLE 285 ACUTE LYMPHOCYTIC LEUKEMIA (ALL) ....................... 194 TABLE 286 CHRONIC MYELOID LEUKEMIA (CML) ......................... 195 TABLE 287 CHRONIC LYMPHOCYTIC LEUKEMIA (CLL) .................... 196 TABLE 288 MONOCLONAL GAMMOPATHY OF UNCERTAIN SIGNIFICANCE (MGUS)....................................................................... 196 TABLE 289 MULTIPLE MYELOMA (MM)..................................... 197 TABLE 290 HODGKIN LYMPHOMA (HL) ...................................... 198 TABLE 291 NON-HODGKIN LYMPHOMA (NHL) ............................ 199 TABLE 292 TOXICOLOGY .......................................................... 208 TABLE 293 OTHER INJURIES...................................................... 213 TABLE 294 SPINAL CORD COMPRESSION ..................................... 227 TABLE 295 SYRINGOMYELIA...................................................... 228 TABLE 296 SUBACUTE COMBINED DEGENERATION (SACD) ............ 228 TABLE 297 ANTERIOR SPINAL ARTERY INFARCTION ....................... 228 TABLE 298 STROKE ETIOLOGY ................................................... 229 TABLE 299 TYPES OF STROKES .................................................. 230 TABLE 300 PRESENTATION OF STROKES ...................................... 231

TABLE 301 CLINICAL DIAGNOSIS OF STROKE ................................ 232 TABLE 302 IMAGING TESTS ...................................................... 233 TABLE 303 GLASGOW COMA SCALE (GCS) ................................. 233 TABLE 304 TREATMENT OF STROKE ........................................... 234 TABLE 305 COMPLICATIONS OF STROKE ..................................... 235 TABLE 306 CAUSES OF HEADACHES ........................................... 236 TABLE 307 MIGRAINE HEADACHES ............................................ 236 TABLE 308 TENSION HEADACHES .............................................. 237 TABLE 309 CLUSTER HEADACHES .............................................. 237 TABLE 310 GIANT CELL ARTERITIS (GCA) ................................... 238 TABLE 311 NORMAL PRESSURE HYDROCEPHALUS (NPH) .............. 238 TABLE 312 ACUTE BLINDNESS .................................................. 239 TABLE 313 CONDUCTIVE HEARING LOSS ..................................... 240 TABLE 314 SENSORINEURAL HEARING LOSS ................................ 240 TABLE 315 VERTIGO ............................................................... 241 TABLE 316 RABIES ................................................................. 242 TABLE 317 BACTERIAL MENINGITIS ........................................... 243 TABLE 318 VIRAL MENINGITIS .................................................. 243 TABLE 319 FUNGAL MENINGITIS ............................................... 244 TABLE 320 VIRAL ENCEPHALITIS ................................................ 244 TABLE 321 BRAIN ABSCESS ...................................................... 245 TABLE 322 BOTULISM ............................................................. 246 TABLE 323 ALZHEIMER DISEASE (AD) ........................................ 247 TABLE 324 PARKINSON DISEASE (PD) ........................................ 248 TABLE 325 HUNTINGTON DISEASE (HD) .................................... 249 TABLE 326 STAGES OF SLEEP & THEIR ASSOCIATED WAVE FORMS .. 249 TABLE 327 SLEEP DISORDER TREATMENTS .................................. 250 TABLE 328 DYSSOMNIAS ......................................................... 250 TABLE 329 PARASOMNIAS ....................................................... 250 TABLE 330 NARCOLEPSY ......................................................... 251 TABLE 331 EPILEPTIC SEIZURES ................................................. 252 TABLE 332 TUMOR TYPES ........................................................ 255 TABLE 333 BRAIN TUMORS ...................................................... 255 TABLE 334 OXYGEN-HEMOGLOBIN DISSOCIATION CURVE FACTORS . 261 TABLE 335 PULMONARY FUNCTION TESTS .................................. 261 TABLE 336 CAUSES OF HYPOXIA ............................................... 263 TABLE 337 POSTNASAL DRIP .................................................... 264 TABLE 338 OTITIS MEDIA ........................................................ 265 TABLE 339 VIRAL PHARYNGITIS................................................. 266 TABLE 340 BACTERIAL PHARYNGITIS .......................................... 266 TABLE 341 ACUTE AND CHRONIC COUGH ................................... 267 TABLE 342 SINUSITIS .............................................................. 267 TABLE 343 PERTUSSIS ............................................................. 268 TABLE 344 PNEUMONIA .......................................................... 270 TABLE 345 TUBERCULOSIS ....................................................... 271 TABLE 346 BRONCHITIS........................................................... 272 TABLE 347 LUNG ABSCESS ....................................................... 273 www.ClinicalReview.com


Table of Figures

TABLE 348 ASTHMA ............................................................... 275 TABLE 349 CHRONIC OBSTRUCTIVE PULMONARY DISEASE (COPD) . 277 TABLE 350 BRONCHIECTASIS .................................................... 278 TABLE 351 SLEEP APNEA ......................................................... 278 TABLE 352 IDIOPATHIC PULMONARY FIBROSIS (IPF)..................... 279 TABLE 353 SARCOIDOSIS ......................................................... 280 TABLE 354 PNEUMOCONIOSIS .................................................. 280 TABLE 355 ASBESTOSIS ........................................................... 281 TABLE 356 SILICOSIS .............................................................. 281 TABLE 357 COAL MINER’S LUNG .............................................. 281 TABLE 358 FARMER’S LUNG .................................................... 282 TABLE 359 ADULT RESPIRATORY DISTRESS SYNDROME (ARDS)...... 283 TABLE 360 ATELECTASIS.......................................................... 283 TABLE 361 SPONTANEOUS PNEUMOTHORAX .............................. 284 TABLE 362 TENSION PNEUMOTHORAX ....................................... 284 TABLE 363 PLEURAL EFFUSION ................................................. 285 TABLE 364 PULMONARY EMBOLISM (PE) ................................... 287 TABLE 365 LUNG CANCER ....................................................... 289 TABLE 366 MEDIASTINITIS ....................................................... 289 TABLE 367 PNEUMOMEDIASTINUM ........................................... 290 TABLE 368 PATHOLOGICAL HEART SOUNDS ................................ 295 TABLE 369 ELECTROCARDIOGRAM (EKG) DETECTED ABNORMALITIES ................................................................................... 299 TABLE 370 CARDIAC TESTS ...................................................... 300 TABLE 371 CAUSES OF CHEST PAIN ........................................... 301 TABLE 372 CORONARY HEART DISEASE ...................................... 303 TABLE 373 DYSLIPIDEMIA ........................................................ 304 TABLE 374 HYPERTENSION ...................................................... 306 TABLE 375 MALIGNANT HYPERTENSION..................................... 306 TABLE 376 ANGINA................................................................ 308 TABLE 377 PATHOPHYSIOLOGY OF AN MI................................... 309 TABLE 378 EKG FINDINGS....................................................... 309 TABLE 379 DIAGNOSIS WITH CARDIAC ENZYMES .......................... 310 TABLE 380 MYOCARDIAL INFARCTION (MI) ................................ 311 TABLE 381 STAGING OF MI ..................................................... 311 TABLE 382 COMPLICATIONS OF MI ........................................... 313 TABLE 383 POSTINFARCT MANAGEMENT ................................... 313 TABLE 384 CONGESTIVE HEART FAILURE (CHF) .......................... 315 TABLE 385 TREATMENT OF DYSFUNCTIONS................................. 316 TABLE 386 PULMONARY EDEMA (APE) ..................................... 317 TABLE 387 PAROXYSMAL NOCTURNAL DYSPNEA (PND) ............... 317 TABLE 388 MITRAL STENOSIS (MS) .......................................... 318 TABLE 389 MITRAL REGURGITATION (MR) ................................ 319 TABLE 390 MITRAL VALVE PROLAPSE (MVP) ............................. 320 TABLE 391 AORTIC STENOSIS (AS) ............................................ 322 TABLE 392 AORTIC REGURGITATION (AR) .................................. 323 TABLE 393 TRICUSPID STENOSIS (TS) ........................................ 323

xxi TABLE 394 TRICUSPID REGURGITATION ...................................... 324 TABLE 395 SUMMARY DIAGNOSES OF MURMURS ........................ 324 TABLE 396 RHEUMATIC FEVER (RF) ........................................... 325 TABLE 397 DILATED CARDIOMYOPATHY (DCM)........................... 326 TABLE 398 HYPERTROPHIC CARDIOMYOPATHY (HCM).................. 328 TABLE 399 RESTRICTIVE CARDIOMYOPATHY (RCM) ...................... 329 TABLE 400 MYOCARDITIS ........................................................ 330 TABLE 401 PERICARDIAL DISEASE .............................................. 330 TABLE 402 PERICARDITIS ......................................................... 331 TABLE 403 PERICARDIAL TAMPONADE........................................ 332 TABLE 404 CONSTRICTIVE PERICARDITIS ..................................... 333 TABLE 405 ENDOCARDITIS ....................................................... 334 TABLE 406 LIBMAN-SACKS ENDOCARDITIS (LSE).......................... 335 TABLE 407 NONBACTERIAL THROMBOTIC ENDOCARDITIS (NBTE) ... 336 TABLE 408 SINUS BRADYCARDIA ............................................... 337 TABLE 409 SINUS TACHYCARDIA................................................ 337 TABLE 410 PAROXYSMAL ATRIAL TACHYCARDIA ........................... 337 TABLE 411 ATRIAL FLUTTER ..................................................... 338 TABLE 412 ATRIAL FIBRILLATION ............................................... 338 TABLE 413 ATRIOVENTRICULAR BLOCK ....................................... 340 TABLE 414 VENTRICULAR ARRHYTHMIAS .................................... 341 TABLE 415 WOLF-PARKINSON-WHITE SYNDROME (WPW) ........... 341 TABLE 416 TORSADE DE POINTES .............................................. 342 TABLE 417 AORTIC DISSECTION ................................................ 343 TABLE 418 ABDOMINAL AORTIC ANEURYSM (AAA) ..................... 344 TABLE 419 CHURG-STRAUSS DISEASE ........................................ 346 TABLE 420 TAKAYASU ARTERITIS ............................................... 346 TABLE 421 WEGENER GRANULOMATOSIS ................................... 347 TABLE 422 HENOCH-SCHONLEIN PURPURA (HSP)........................ 348 TABLE 423 POLYARTERITIS NODOSA (PAN) ................................ 349 TABLE 424 THROMBOTIC THROMBOCYTOPENIC PURPURA (TTP) .... 350 TABLE 425 AVASCULAR NECROSIS (AVN) ................................... 350 TABLE 426 ACHALASIA ............................................................ 360 TABLE 427 DIFFUSE ESOPHAGEAL SPASM ................................... 361 TABLE 428 NUTCRACKER ESOPHAGUS ........................................ 361 TABLE 429 INFECTIOUS ESOPHAGITIS ......................................... 362 TABLE 430 GASTROESOPHAGEAL REFLUX DISEASE ........................ 364 TABLE 431 ESOPHAGEAL OBSTRUCTIONS .................................... 364 TABLE 432 ZENKER DIVERTICULUM ............................................ 365 TABLE 433 ESOPHAGEAL PERFORATION ...................................... 366 TABLE 434 BOERHAAVE SYNDROME........................................... 366 TABLE 435 MALLORY-WEISS SYNDROME .................................... 367 TABLE 436 BARRETT ESOPHAGUS .............................................. 367 TABLE 437 ESOPHAGEAL CARCINOMA ........................................ 368 TABLE 438 SCLERODERMA ....................................................... 369 TABLE 439 GASTRITIS.............................................................. 372 TABLE 440 PEPTIC ULCER DISEASE............................................. 373


xxii

USMLE STEP 2

TABLE 441 GASTRIC PEPTIC ULCER ............................................ 374 TABLE 442 CURLING AND CUSHING ULCERS................................. 374 TABLE 443 DUODENAL ULCER................................................... 375 TABLE 444 ZOLLINGER-ELLISON SYNDROME ................................ 376 TABLE 445 GASTROPARESIS ...................................................... 377 TABLE 446 DUMPING SYNDROME.............................................. 377 TABLE 447 MALABSORPTION SYNDROMES .................................. 380 TABLE 448 CELIAC SPRUE......................................................... 381 TABLE 449 PROTEIN LOSING ENTEROPATHY................................. 381 TABLE 450 TROPICAL SPRUE ..................................................... 382 TABLE 451 WHIPPLE DISEASE ................................................... 383 TABLE 452 MÉNÉTRIER DISEASE ................................................ 383 TABLE 453 DIARRHEA ............................................................. 385 TABLE 454 INFECTIOUS DIARRHEA ............................................. 386 TABLE 455 MEDICATION-INDUCED DIARRHEA.............................. 386 TABLE 456 INFLAMMATORY DIARRHEA ....................................... 386 TABLE 457 OSMOTIC DIARRHEA ................................................ 387 TABLE 458 CONSTIPATION ....................................................... 387 TABLE 459 PSEUDOMEMBRANOUS COLITIS ................................. 388 TABLE 460 PSEUDOMEMBRANOUS COLITIS ................................. 389 TABLE 461 IRRITABLE BOWEL SYNDROME ................................... 389 TABLE 462 CROHN DISEASE...................................................... 391 TABLE 463 ULCERATIVE COLITIS ................................................ 392 TABLE 464 COMPLICATIONS ..................................................... 393 TABLE 465 DIVERTICULOSIS ...................................................... 394 TABLE 466 DIVERTICULITIS ....................................................... 395 TABLE 467 UPPER GASTROINTESTINAL BLEEDING ......................... 396 TABLE 468 LOWER GASTROINTESTINAL BLEEDING......................... 397 TABLE 469 MESENTERIC ISCHEMIA ............................................ 397 TABLE 470 CARCINOID SYNDROME ............................................ 399 TABLE 471 COLORECTAL CANCER .............................................. 399 TABLE 472 OTHER COLONIC SYNDROMES ................................... 400 TABLE 473 COLON CANCER ...................................................... 401 TABLE 474 STAGING OF COLON CANCER—DUKE’S STAGES ............ 401 TABLE 475 TREATMENT OF COLON CANCER................................. 402 TABLE 476 SCREENING OF COLON CANCER .................................. 402 TABLE 477 PRESENTATION OF HEPATOBILIARY DISEASE ................. 418 TABLE 478 DIAGNOSTIC STUDIES ............................................... 420 TABLE 479 CIRRHOSIS ............................................................. 421 TABLE 480 PRIMARY BILIARY CIRRHOSIS (PBC) ............................ 422 TABLE 481 INHERITED HYPERBILIRUBINEMIA OVERVIEW ................ 422 TABLE 482 INHERITED SYNDROMES............................................ 424 TABLE 483 VIRAL HEPATITIS ..................................................... 424 TABLE 484 HEPATITIS A (HAV)................................................. 425 TABLE 485 HEPATITIS B (HBV) ................................................. 426 TABLE 486 HEPATITIS C VIRUS (HCV) ........................................ 427 TABLE 487 HEPATITIS D VIRUS (HDV) ....................................... 427

TABLE 488 HEPATITIS E VIRUS (HEV) ........................................ 427 TABLE 489 HEPATITIS G VIRUS (HGV) ....................................... 428 TABLE 490 ACUTE PANCREATITIS .............................................. 430 TABLE 491 CHRONIC PANCREATITIS ........................................... 431 TABLE 492 CHOLELITHIASIS ...................................................... 432 TABLE 493 CHOLECYSTITIS ....................................................... 433 TABLE 494 ASCENDING CHOLANGITIS ........................................ 434 TABLE 495 PRIMARY SCLEROSING CHOLANGITIS (PSC) ................. 434 TABLE 496 HEPATOCELLULAR CARCINOMA ................................. 436 TABLE 497 HYPERPROLACTINEMIA ............................................ 447 TABLE 498 ACROMEGALY ........................................................ 447 TABLE 499 HYPOPITUITARISM .................................................. 448 TABLE 500 EMPTY SELLA SYNDROME (ESS) ................................ 449 TABLE 501 DIABETES INSIPIDUS (DI).......................................... 449 TABLE 502 SECRETION OF INAPPROPRIATE ANTIDIURETIC HORMONE (SIADH) ...................................................................... 450 TABLE 503 PITUITARY TUMORS ................................................ 451 TABLE 504 HYPERTHYROIDISM ................................................. 452 TABLE 505 SUBACUTE THYROIDITIS ........................................... 453 TABLE 506 THYROID STORM (THYROTOXIC CRISIS) ....................... 453 TABLE 507 HYPOTHYROIDISM................................................... 454 TABLE 508 MYXEDEMA COMA ................................................. 455 TABLE 509 PAPILLARY THYROID CANCER (PTC) ........................... 456 TABLE 510 FOLLICULAR THYROID CANCER (FTC) ......................... 456 TABLE 511 MEDULLARY THYROID CANCER (MTC) ....................... 457 TABLE 512 ANAPLASTIC THYROID CANCER (ATC)......................... 458 TABLE 513 PRIMARY HYPERPARATHYROIDISM ............................. 459 TABLE 514 SECONDARY HYPERPARATHYROIDISM ......................... 459 TABLE 515 TERTIARY HYPERPARATHYROIDISM ............................. 460 TABLE 516 HYPOPARATHYROIDISM............................................ 460 ABLE 517 PSEUDOHYPOPARATHYROIDISM (PHP) ......................... 461 TABLE 518 CUSHING SYNDROME .............................................. 462 TABLE 519 PSEUDOCUSHING SYNDROME ................................... 462 TABLE 520 CONGENITAL ADRENAL HYPERPLASIA (CAH)................ 464 TABLE 521 CONN SYNDROME .................................................. 464 TABLE 522 HYPOALDOSTERONISM............................................. 465 TABLE 523 ADRENAL INSUFFICIENCY .......................................... 466 TABLE 524 PHEOCHROMOCYTOMA............................................ 466 TABLE 525 TYPE I DIABETES MELLITUS ....................................... 468 TABLE 526 TYPE 2 DIABETES MELLITUS ...................................... 469 TABLE 527 DIABETIC KETOSIS (DKA) ......................................... 470 TABLE 528 HYPEROSMOLAR HYPERGLYCEMIC NONKETOTIC COMA (HHNC) ....................................................................... 471 TABLE 529 HYPOGLYCEMIA ...................................................... 472 TABLE 530 OBESITY ................................................................ 473 TABLE 531 ANOREXIA NERVOSA ............................................... 474 TABLE 532 BULIMIA ............................................................... 475 www.ClinicalReview.com


Table of Figures

TABLE 533 MALNUTRITION ..................................................... 475 TABLE 534 VITAMIN DEFICIENCY .............................................. 479 TABLE 535 VITAMIN TOXICITY .................................................. 480 TABLE 536 MULTIPLE ENDOCRINE NEOPLASM (MEN).................. 482 TABLE 537 NORMAL ELECTROLYTE RANGE ................................. 490 TABLE 538 ANION GAP ........................................................... 491 TABLE 539 FRACTIONAL EXCRETION OF SODIUM .......................... 491 TABLE 540 BODY WATER ........................................................ 491 TABLE 541 SERUM OSMOLARITY .............................................. 492 TABLE 542 ARTERIAL BLOOD GAS (ABG) ................................... 493 TABLE 543 DIALYSIS ............................................................... 494 TABLE 544 HYPONATREMIA ..................................................... 496 TABLE 545 HYPERNATREMIA .................................................... 497 TABLE 546 HYPOKALEMIA ....................................................... 498 TABLE 547 HYPERKALEMIA ...................................................... 499 TABLE 548 HYPOCALCEMIA ..................................................... 500 TABLE 549 HYPERCALCEMIA .................................................... 501 TABLE 550 CANCER-INDUCED HYPERCALCEMIA ........................... 501 TABLE 551 HYPOPHOSPHATEMIA .............................................. 502 TABLE 552 HYPERPHOSPHATEMIA ............................................. 503 TABLE 553 HYPOMAGNESEMIA ................................................ 503 TABLE 554 HYPERMAGNESEMIA ............................................... 504 TABLE 555 ANION GAP METABOLIC ACIDOSIS ............................. 505 TABLE 556 NON- ANION GAP METABOLIC ACIDOSIS .................... 505 TABLE 557 METABOLIC ALKALOSIS ............................................ 506 TABLE 558 RESPIRATORY ACIDOSIS ........................................... 506 TABLE 559 RESPIRATORY ALKALOSIS.......................................... 507 TABLE 560 RENAL TUBULE ACIDOSIS (RTA)................................ 507 TABLE 561 RENAL FAILURE ...................................................... 508 TABLE 562 ACUTE TUBULAR NECROSIS (ATN) ............................ 509 TABLE 563 ALLERGIC INTERSTITIAL NEPHRITIS (AIN) .................... 510 TABLE 564 OTHER CAUSES OF INTRINSIC RENAL FAILURE .............. 510 TABLE 565 UREMIC SYNDROME ............................................... 511 TABLE 566 END STAGE RENAL DISEASE (ESRD) .......................... 512 TABLE 567 ACUTE PYELONEPHRITIS ........................................... 513 TABLE 568 PERINEPHRIC ABSCESS ............................................. 513 TABLE 569 NEPHROTIC SYNDROME ........................................... 514 TABLE 570 ACUTE GLOMERULONEPHRITIS .................................. 515 TABLE 571 POSTSTREPTOCOCCAL GLOMERULONEPHRITIS (PSGN) .. 516 TABLE 572 RAPIDLY PROGRESSING GLOMERULONEPHRITIS (RPGN) 517 TABLE 573 MEMBRANOPROLIFERATIVE GLOMERULONEPHRITIS (MPGN) ................................................................................... 518 TABLE 574 IGA NEUROPATHY .................................................. 518 TABLE 575 GOODPASTURE SYNDROME ...................................... 519 TABLE 576 ALPORT SYNDROME ................................................ 520 TABLE 577 ADULT POLYCYSTIC KIDNEY DISEASE (APKD)............... 521 TABLE 578 NEPHROLITHIASIS ................................................... 522

xxiii TABLE 579 INCONTINENCE ....................................................... 523 TABLE 580 ERECTILE DISORDERS (ED) ........................................ 524 TABLE 581 BENIGN PROSTATE HYPERTROPHY (BPH) .................... 524 TABLE 582 PROSTATE CANCER .................................................. 525 TABLE 583 GENITOURINARY INFECTION ...................................... 527 TABLE 584 URETHRITIS............................................................ 528 TABLE 585 VAGINITIS .............................................................. 528 TABLE 586 PELVIC INFLAMMATORY DISEASE (PID) ....................... 529 TABLE 587 BALANITIS ............................................................. 530 TABLE 588 PROSTATITIS .......................................................... 530 TABLE 589 PROCTITIS AND ANUSITIS .......................................... 531 TABLE 590 SYPHILIS ................................................................ 532 TABLE 591 CHANCROID ........................................................... 532 TABLE 592 LYMPHOGRANULOMA VENEREUM (LGV) .................... 533 TABLE 593 GRANULOMA INGUINALE .......................................... 534 TABLE 594 GENITAL HERPES .................................................... 535 TABLE 595 GENITAL WARTS ..................................................... 535 TABLE 596 GONORRHEA .......................................................... 536 TABLE 597 HIV AND AIDS ....................................................... 538 TABLE 598 HORMONE REPLACEMENT THERAPY (HRT).................. 543 TABLE 599 DOMESTIC VIOLENCE ............................................... 544 TABLE 600 BREAST CANCER ..................................................... 557 TABLE 601 ENDOMETRIAL CANCER ............................................ 558 TABLE 602 OVARIAN CANCER ................................................... 559 TABLE 603 CERVICAL CANCER ................................................... 560 TABLE 604 OSTEOPOROSIS ....................................................... 593 TABLE 605 PAGET’S DISEASE .................................................... 594 TABLE 606 OSTEOMALACIA ...................................................... 595 TABLE 607 OSTEOMYELITIS ...................................................... 595 TABLE 608 COSTOCHONDRITIS .................................................. 596 TABLE 609 LUMBAR DISC HERNIATION ....................................... 596 TABLE 610 EPIDURAL ABSCESS .................................................. 597 TABLE 611 CAUDA EQUINA SYNDROME (CES) ............................. 597 TABLE 612 OSTEOARTHRITIS (OA) ............................................ 598 TABLE 613 RHEUMATOID ARTHRITIS (RA) .................................. 600 TABLE 614 SYSTEMIC LUPUS ERYTHEMATOSUS (SLE) .................... 601 TABLE 615 DRUG-INDUCED LUPUS ............................................ 602 TABLE 616 ANTIPHOSPHOLIPID ANTIBODY SYNDROME (APA) ........ 602 TABLE 617 SCLERODERMA ....................................................... 603 TABLE 618 SJÖGREN SYNDROME (SS) ........................................ 604 TABLE 619 ANKYLOSING SPONDYLITIS (AS) ................................. 605 TABLE 620 REACTIVE ARTHRITIS ................................................ 606 TABLE 621 PSORIATIC ARTHRITIS ............................................... 606 TABLE 622 ENTEROPATHIC ARTHROPATHY .................................. 607 TABLE 623 GOUT ................................................................... 608 TABLE 624 CALCIUM PYROPHOSPHATE DEPOSITION DISEASE (CPDD) ................................................................................... 609


xxiv

USMLE STEP 2

TABLE 625 GONOCOCCAL SEPTIC ARTHRITIS ................................ 610 TABLE 626 NONGONOCOCCAL SEPTIC ARTHRITIS .......................... 611 TABLE 627 GAS GANGRENE...................................................... 611 TABLE 628 CARPAL TUNNEL SYNDROME (CTS) ............................ 612 TABLE 629 POLYMYOSITIS ........................................................ 613 TABLE 630 BEHÇET’S SYNDROME .............................................. 613

TABLE 631 MULTIPLE SCLEROSIS (MS) ...................................... 614 TABLE 632 MYASTHENIA GRAVIS (MG) ..................................... 615 TABLE 633 GUILLAIN-BARRÉ SYNDROME (GBS) .......................... 616 TABLE 634 SPINAL METASTASIS ................................................ 617

www.ClinicalReview.com


Abbreviations

xxv

ABBREVIATIONS ABBREVIATIONS 5-ASA 5-FU 5-HIAA 5-HIAA 5-HT AAA AAFP ABC ABE ABG ABO ABVD

5-aminosalicylic acid 5-fluorouracil 5-hydroxyindolacetic acid 5-hydroxyindolacetic acid 5-hydroxytryptamine Abdominal aortic aneurysm American Academy of Family Physicians Airway, breathing, circulation Acute bacterial endocarditis Arterial blood gas Blood types Adriamycin, bleomycin, vinblastine, decarbazine ACA Anterior cerebral artery ACD Anemia of chronic disease ACE Angiotensin converting factor ACE Angiotensin-converting enzyme AChR Acetylcholine receptor ACLS Advanced cardiac life support ACS American Cancer Society or Acute Chest Syndrome ACTH Adrenocorticotrophic hormone AD Autosomal dominant / Alzheimer disease AdenoCA Adenocarcinoma ADH Antidiuretic hormone AFB Acid-fast bacilli AFP Alpha fetoprotein AG Anion gap AHO Albright’s hereditary osteodystrophy AI Adrenal insufficiency AICA Anterior inferior cerebellar artery AIDS Acquired immunodeficiency syndrome AIHA Autoimmune hemolytic anemia AIN Allergic interstitial nephritis ALL Acute lymphocytic leukemia ALT Alanine transaminase AMA American Medical Association or Antimitochondrial antibodies AMI Acute myocardial infarction AML Acute myelocytic leukemia ANA Antinuclear antibodies ANCA Antineutrophil cytoplasmic antibodies ANF Atrial natriuretic factor Anti-SM Antibodies to striated muscle Anti-TPO Antithyroid peroxidase AOM Acute otitis media APA Antiphospholipid antibody APAP p-aminophenol APC Adenomatosis polyposis coli APE Acute pulmonary edema APKD Adult polycystic kidney disease APR Acute phase reactant AR Autosomal recessive or Aortic regurgitation ARDS Adult respiratory distress syndrome ARF Acute renal failure AS Aortic stenosis or Ankylosing spondylitis ASA Aminosalicylic acid, Aspirin ASAM American Society of Addiction Medicine ASD Atrial septal defect ASLO Antistreptolysin-O

AST ATC ATG ATIII ATN ATP AUB AV node AV AVM AVN AXR BAL BCG BCx BMD BMI BMR BMS BMT BP BPH BPM BPPV BRBPR BUN BV CABG CAD CAGE CAH C-ANCA CAPD CaDPTA CBC CCK CD CEA CES CHF CHOP CI CK CK-MB CLD CLL CML CMV CNS CO COPD CPAP CPK CPM CPPD CR CRAO

Aspartate transaminase Anaplastic thyroid cancer Antithymocyte globulin Antithrombin III Acute tubular necrosis Adenosine triphosphate Abnormal uterine bleeding Atrioventricular node Arteriovenous Arteriovenous malformation Avascular necrosis Abdominal x-ray Bronchoalveolar lavage or Dimercaprol Bacillus Calmette-Guerin Blood cultures Bone mineral deficiency Body mass index Basal metabolic rate Bone marrow suppression Bone marrow transplant Blood pressure Benign prostate hypertrophy Beats per minute Benign paroxysmal positional vertigo Bright red blood per rectum Blood urea nitrogen Bacterial vaginosis Coronary artery bypass graft Coronary artery disease Cut down, Annoy others, Guilty feelings, Eye-opener in the morning Congenital adrenal hyperplasia Cytoplasmic ANCA Chronic ambulatory peritoneal dialysis Calcium trisodium Complete blood count Cholecystokinin Crohn disease Carcinoembryonic antigen Cauda equina syndrome Congestive heart failure Cyclophosphamide, hydroxydaunomycin, vincristine, prednisone Cardiac index Creatine kinase MB fraction of CK Clear liquid diet Chronic lymphocytic leukemia Chronic myeloid leukemia Cytomegalovirus Central nervous system Cardiac output Chronic obstructive pulmonary disease Continuous positive air pressure Creatine phosphokinase Central pontine myelinolysis Calcium pyrophosphate deposition disease Creatinine Central retinal artery occlusion

CREST

Calcinosis, Raynaud phenomenon, esophageal dysmotility, sclerodactyly, telangiectasia CRF Chronic renal failure or Corticotropinreleasing factor CRH Corticotrophin releasing hormone CRP C-reactive protein CSA Cyclosporine A CSF Cerebrospinal fluid CT Computed tomography CTD Connective tissue disease CVA Cerebrovascular accident CVD Collagen vascular disease or Cerebrovascular disease CVP Cyclophosphamide, vincristine, prednisone CXR Chest x-ray DB Decibel DBP Diastolic blood pressure DCIS Ductal carcinoma in situ DCM Dilated cardiomyopathy DCP Des-gamma-carboxy prothrombin DEET Insect repellant (N, N-diethyl-mtoluamide) DES Diffuse esophageal spasm DEXA Dual-energy x-ray absorptiometry DI Diabetes insipidus DIC Disseminated intravascular coagulation DIP Distal interphalangeal joint DKA Diabetic ketoacidosis DLCO Diffusability of carbon monoxide DM Diabetes mellitus DMARD Disease-modifying antirheumatic drug DMD Duchenne muscular dystrophy DMPS Dimerval DMSA Succimer DNA Deoxyribonucleic acid DOE Dyspnea on exertion DRE Digital rectal exam dsDNA Double stranded DNA DSE Dobutamine stress echocardiogram DT Delirium tremens DTR Deep tendon reflex DUB Dysfunctional uterine bleeding DVT Deep vein thrombosis EBV Epstein-Barr virus ECF Extracellular fluid ECG,EKG Electrocardiogram ECM Extracellular matrix ED Erectile dysfunction EDTA Edetate calcium disodium EDV End diastolic volume EF Ejection fraction EGD Esophagogastroduodenoscopy EHEC Enterohemorrhagic E.coli EIEC Enteroinvasive E. coli ELISA Enzyme-linked immunosorbent assay EMG Electromyography EOM Extraocular muscles EPO Erythropoietin

Â


xxvi ERCP

USMLE STEP 2

Endoscopic retrograde cholangiopancreatography ERV Expiratory reserve volume ESR Erythrocyte sedimentation rate ESRD End stage renal disease ESS Empty sella syndrome ESV End systolic volume ET Endotracheal tube ETEC Enterotoxic Escherichia coli ESWL Extracorporeal shock-wave lithotripsy F-1-P Fructose-1-phosphate FA Fatty acids FAP Familial adenomatous polyposis FEF Forced expiratory flow FENA Fractional excretion of sodium FEV Forced expiratory volume FFP Fresh frozen plasma FHH Familial hypocalciuric hypercalcemia FMF Familial Mediterranean fever FNAB Fine needle aspiration biopsy FOBT Fecal occult blood test FRC Functional reserve capacity FSH Follicle stimulating hormone FTA-ABS Fluorescent treponemal antibody absorption test FTC Follicular thyroid cancer FUO Fever of unknown origin FVC Forced vital capacity G6PD Glucose-6-phosphate dehydrogenase GABA Gamma amino butyric acid GAS Group A streptococcus GBM Glomerular basement membrane GBS Guillain- Barré syndrome GCA Giant cell arteritis GCS Glasgow coma scale GCT Glucose challenge test GERD Gastroesophageal reflux disease GETA General endotracheal anesthesia GFR Glomerular filtration rate GGT Gamma glutamyl transpeptidase GHB Gamma-Hydroxybutyrate GI Gastrointestinal system GIP Gastric inhibitory peptide GM-CSF Granulocyte-macrophage colonystimulating factor GPI Glycosyl-phosphatidylinositol GRP Gastrin releasing peptide GSD Glycogen storage disease H2 Histamine 2 receptor HACEK Haemophilus aphrophilus, Actinobacillus actinomycetemcomitans, Cardiobacterium hominis, Eikenella corrodens, Kingella kinkae HAV Hepatitis A virus HbA Adult hemoglobin HbA1C Hemoglobin A1C HBcAbIgG Hepatitis B core antibody IgG HBcAbIgM Hepatitis B core antibody IgM HBcAg Hepatitis B core antigen HbcAgIgM Hepatitis B core antigen IgM HBeAb Hepatitis B “e” antibody HBeAg Hepatitis B “e” antigen HBIG Hepatitis B immunoglobulin HbS Sickle cell hemoglobin HBsAb Hepatitis B surface antibody HBsAg Hepatitis B surface antigen HBV Hepatitis B virus HCC Hepatocellular carcinoma

HCM HCT HCTZ HCV HCVAb HD HDL HDN HDV HEV HGPRT HGV HHNC HHV HIDA HIPPA HIT HIV HJR HL HLA HNPCC HPV HR HRT HS HSIL HSP HSV HTN HUS IBD IBW IC ICA ICD ICF ICP IDDM IDL IgA IGF IgG IHSS IM IMA IML INH IPF IRV ITP IV IVC IVDA IVF IVIG IVP JVD JVP KS LA LAD LC

Hypertrophic cardiomyopathy Hematocrit Hydrochlorothiazide Hepatitis C virus Hepatitis C virus antibody Hemodialysis High density lipoprotein Hemorrhagic disease of newborns Hepatitis D virus Hepatitis E virus Hypoxanthine-guanine phosphoribosyl transferase Hepatitis G virus Hyperosmolar hyperglycemic nonketotic coma Human herpes virus 8 Hepatobiliary iminodiacetic acid Health information privacy protection act Heparin induced thrombocytopenia Human immunodeficiency virus Hepatojugular reflex Hodgkin lymphoma Human lymphocyte antigen Hereditary nonpolyposis colon cancer Human papilloma virus Heart rate Hormone replacement therapy Hereditary spherocytosis High-grade squamous intraepithelial lesion Henoch-Schonlein purpura Herpes simplex virus Hypertension Hemolytic uremic syndrome Inflammatory bowel disease Ideal body weight Inspiratory capacity or Immunocompromised Internal carotid artery Implanted cardioverter and defibrillator Intracellular fluid Intracranial pressure Insulin-dependent diabetes mellitus Intermediate density lipoprotein Immunoglobulin A Insulin-like growth factor Immunoglobulin G Idiopathic hypertrophic subaortic stenosis Intramuscular Inferior mesenteric artery Intermediolateral column Isoniazid Idiopathic pulmonary fibrosis Inspiratory reserve volume Immune thrombocytopenic purpura Intraventricular or intravenous Inferior vena cava IV drug abuser Intravenous fluids Intravenous immune globulin Intravenous pyelogram Jugular vein distension Jugular venous pulse Kaposi sarcoma Left atrium Left anterior descending branch of LCA Locus ceruleus

LCA LCIS LCX LDH LDL LE LEEP LES LFT LGA LH LHF LLQ LLSB LMWH LOC LP LPL LSD LSE LSES LSIL LUQ LUSB LV LVF LVH LWN Mab MAC MALT MAO MAP MCA MCD MCHC MCP MCV MDI Mdma MDS MEN MG MGUS MI MM MMR MMSE MODS MOPP MPGN MPTP MR MRA MRI MS MTC MUGA NAC NBTE NC NFT

Left coronary artery Lobular cancer in situ Left circumflex branch of LCA Lactate dehydrogenase Low density lipoprotein Lower extremity Loop electrosurgical excision procedure Lower esophageal sphincter Liver function test Large for gestational age Lutenizing hormone Left-sided CHF Left lower quadrant Lower left sternal border Lower molecular weight heparin Loss of consciousness Lumbar puncture Lipoprotein lipase Lysergic acid diethylamide Libman-Sacks endocarditis Low socioeconomic status Low-grade squamous intraepithelial lesion Left upper quadrant Left upper sternal border Left ventricle Left ventricular function Left ventricle hypertrophy Lown-Ganong-Levine syndrome Monoclonal antibody Mycobacterium avium complex Mucosa-associated lymphoid tissue Monoamine oxidase Mean arterial pressure Middle cerebral artery Minimal change disease Mean corpuscular hemoglobin concentration Metacarpophalangeal joint Mean corpuscular volume Metered dose inhaler 3,4-methylene dioxymethamphetamine Myelodysplastic syndrome Multiple endocrine dysplasia Myasthenia gravis Monoclonal gammopathy of uncertain significance Myocardial infarction Multiple myeloma Measles, mumps, rubella vaccination Mini mental status exam Multiple organ dysfunction syndrome Mechlorethamine, vincristine, procarbazine, prednisone Membranoproliferative glomerulonephritis 1-methyl-4-phenyl-1, 2, 3, 6tetrahydropyridine Mitral regurgitation Magnetic resonance angiography Magnetic resonance imaging Multiple sclerosis or Mitral stenosis Medullary thyroid cancer Multigated acquisition scan N-acetyl cysteine Nonbacterial thrombotic endocarditis Nasal cannula Neurofibrillary tangle

www.ClinicalReview.com


Abbreviations NG NH NHL NIDDM NIH NK NMDA NMJ NPH NPO NS NSAID NSCLC NSR OA OME OSA OT PAC PAN P-ANCA PAS PAT PBC PCA PCO2 PCP PCR PCWP PD PE PEEP PEG PET PFO PFT PGE PGL PHA PHP PICA PIF PIGA PIP PLD PMI PML PMN PND PNH PO PO2 POC PP PPD PR PRN PS PSA PSC PSGN PT PTC PTCA

Nasogastric tube Nursing home Non-Hodgkin lymphoma Non-insulin dependent diabetes mellitus National Institute of Health Natural killer cells N-methyl-D-aspartate Neuromuscular junction Normal pressure hydrocephalus Nil per os Normal saline Nonsteroidal anti-inflammatory drug Non-small cell lung cancer Normal sinus rhythm Osteoarthritis Otitis media with effusion Obstructive sleep apnea Occupational therapy Premature atrial contraction or Pulmonary artery catheter Polyarteritis nodosa Perinuclear ANCA Periodic acid shift Paroxysmal atrial tachycardia Primary biliary cirrhosis Posterior cerebral artery Carbon dioxide pressure Pneumocystis carinii pneumonia Polymerase chain reaction Pulmonary capillary wedge pressure Parkinson disease Pulmonary embolism or Plasma exchange Positive end expiratory pressure Polyethylene Glycol Positron emission tomography Patent foramen ovale Pulmonary function test Prostaglandin E Persistent generalized lymphadenopathy Paroxysmal hemolytic anemia Pseudohypoparathyroidism Posterior inferior cerebellar artery Prolactin inhibiting factor Phosphatidylinositol glycan class A gene Proximal interphalangeal joint Partial lipid dystrophy Point of maximal impulse Progressive multifocal leukoencephalopathy Polymorphonuclear lymphocyte Paroxysmal nocturnal dyspnea Paroxysmal nocturnal hemoglobinuria Per oral Oxygen pressure Products of conception Pulse pressure Purified protein derivative Pulmonary regurgitation As needed Pulmonary stenosis Prostate specific antigen Primary sclerosing cholangitis Poststreptococcal glomerulonephritis Prothrombin time or Physical therapy Percutaneous transhepatic catheter or papillary thyroid cancer Percutaneous transluminal coronary angioplasty

PTH PTHrP PTT PTU PUD PV PVC PZ RA RAA RADT RAH RAS RASTs RBBB RBC RCA RCM RF Rh RHD RHF RhIg RIND RLD RLQ RNA RNS RPGN RPR RS RSI RTA RTI RUQ RV RVH S2 SA SAAG SACD SBE SBP SC SCA SCC SCD SCID SCLC SERMS SES SG SI SIADH SIRS SLE SLN SMA SNS SOB SP SPECT SS SS-A/Ro

Parathyroid hormone Parathyroid hormone related peptide Partial thromboplastin time Propylthiouracil Peptic ulcer disease Polycythemia vera Premature ventricular contraction Peripheral zone Rheumatoid arthritis Renin-angiotensin-aldosterone Rapid antigen detection test Right atrial hypertrophy Renal artery stenosis Radioallergosorbent assay tests Right bundle branch block Red blood cell Right coronary artery Restrictive cardiomyopathy Rheumatoid factor or Rheumatic fever Rhesus group Rheumatic heart disease Right-sided CHF Rho immune globulin Reversible ischemic neurologic deficit Restrictive lung defect Right lower quadrant Ribonucleic acid Repetitive nerve stimulation Rapidly progressive glomerulonephritis Rapid plasma reagin Reiter’s syndrome or Reed-Sternberg cells Repetitive stress injury Renal tube acidosis Respiratory tract infection Right upper quadrant Residual volume Right ventricular hypertrophy Secondary heart sound Sinoatrial node Serum-ascites albumin gradient Subacute combined degeneration Subacute bacterial endocarditis Systolic blood pressure Subcutaneously Sickle cell anemia Squamous cell carcinoma Sequential compressive device Severe combined immunodeficiency disease Small cell lung cancer Selective estrogen-receptor modulators Socioeconomic status Specific gravity Small intestine Secretion of inappropriate antidiuretic hormone Systemic inflammatory response syndrome Systemic lupus erythematosus Sublingual nitroglycerin Superior mesenteric artery Sympathetic nervous system Shortness of breath Senile plaques Single photon emission CT Sjögren syndrome Antibodies to antigen A

xxvii

SS-B-La SSRI StAR STd STD Ste SV SVC SVR SVT T3 T4 TAHBSO

Antibodies to antigen B Selective serotonin reuptake inhibitor Steroidogenic acute regulatory deficiency EKG ST depression Sexually transmitted disease EKG ST elevation Stroke volume Superior vena cava Systemic vascular resistance Supraventricular tachycardia Triiodothyronine Tetraiodothyronine (thyroxine) Total abdominal hysterectomy with bilateral salpingo-oophorectomy TAS Transabdominal sonography TB Tuberculosis TCA Tricyclic antidepressant TEE Transesophageal echocardiogram THC delta-9-tetrahydrocannabinol THRH Thyrotropin-releasing hormone TIA Transient ischemic attack TIPS Transjugular intrahepatic portacaval shunt TLC Total lung capacity TMP-SMX Trimethoprim-sulfamethoxazole TNF Tissue necrosis factor TnI Troponin I TnT Troponin T ToF Tetralogy of Fallot ToV Transposition of vessels TPN Total parenteral nutrition TR Tricuspid regurgitation TRALI Transfusion-related acute lung injury TRUS Transrectal ultrasound TS Tricuspid stenosis TSE Treadmill stress echocardiogram TSH Thyroid stimulating hormone TTE Transthoracic echocardiogram TTP Thrombocytic purpura TURP Transurethral resection of the prostate TVS Transvaginal sonography TxC Type and cross TZ Transitional zone UA Urine analysis UC Ulcerative colitis UDCA Ursodeoxycholic acid UE Upper extremity URI Upper respiratory infection US Ultrasound UTI Urinary tract infection V/Q Ventilation-perfusion VC Vital capacity VF Ventricular fibrillation VHL Von-Hippel-Landau disease VIP Vasoactive intestinal peptide VLDL Very low density lipoprotein VSD Ventricular septal defect VT Ventricular tachycardia vWD Von Willebrand disease vWF Von Willebrand factor VZV Varicella zoster virus WNL Within normal limits WPW Wolf-Parkinson-White syndrome ZE Zollinger-Ellison syndrome ZnDTPA Pentetate zinc trisodium


xxviii

USMLE STEP 2

www.ClinicalReview.com


Dedication

xxix

DEDICATION DEDICATION

For my parents.

IMAGE CREDITS Dedication: © Xyzproject, Reproduced With Permission Foreword: © MedicalRF.com, Reproduced With Permission Introduction: © Sebastian Kaulitzki, Reproduced With Permission Nervous System: © MedicalRF.com, Reproduced With Permission Respiratory System: © Sebastian Kaulitzki, Reproduced With Permission Cardiovascular System: © MedicalRF.com, Reproduced With Permission Gastrointestinal System: © MedicalRF.com, Reproduced With Permission Hepatopancreatobiliary System: © MedicalRF.com, Reproduced With Permission

Endocrine System: © MedicalRF.com, Reproduced With Permission Genitourinary System: © MedicalRF.com, Reproduced With Permission Obstetrics and Gynecology: © MedicalRF.com, Reproduced With Permission Skin and Soft Tissue: © MedicalRF.com, Reproduced With Permission Musculoskeletal System: © MedicalRF.com, Reproduced With Permission High-Yield: © MedicalRF.com, Reproduced With Permission Index: © MedicalRF.com, Reproduced With Permission Other images within this textbook are adapted from the public domain. If an image is not cited appropriately, we would be happy to make the proper arrangements.


xxx

USMLE STEP 2

SYMBOLS SYMBOLS

Essential topic for the modern practice of surgery. Commonly tested on both written and oral exams.

High-yield information that frequently appears on exams.

Practice question covering a commonly tested concept.

Detailed question explanation and ancillary information.

The drug of choice for a particular disease process.

Important property of a particular medication.

www.ClinicalReview.com


Symbols

1

Â


2

USMLE STEP 2

FOREWORD FOREWORD

T

he United States Medical Licensing Examination is taken throughout the year by medical students and residents around the country. It is used by residency programs as one measure of the caliber of potential candidates and is required to become a physician eligible to practice medicine.1 Poor performance on the USMLE is likely to adversely affect your application for residency, may limit the opportunities available for you in career selection, and jeopardizes your medical career. This textbook and its accompanying online course were created to improve the quality of medical education and thereby help to improve the quality of care our patients receive. While there can be no substitute for studying the reference textbooks in the field, there are numerous moments during medical school and residency to quickly review and understand high-yield topics. The focus of this program is to provide a high-yield review of topics commonly tested on the USMLE.

This material is derived from the major medical reference textbooks and covers topics that have appeared on recent USMLE examinations. The overall organization of this textbook reflects the content outline produced by the FSMB and NBME to ensure that every subject area is given due consideration. Students and residents who have used this program in recent years have reported scores in the top 1% across the world. The online program available last year was used by medical students from nearly every medical school around the world, and residents from nearly every residency program across the nation.2 Based on their feedback and our proven approach to education, this is our solution for preparing for the USMLE.

1

Residency programs use the USMLE score as one element in selecting residents during the match. Poor scores mitigate the opportunity for competitive specialties and for joining top-tier residency programs. Poor performance may even prevent the ability to become a certified medical practitioner.

2

Students around the world and residents around the United States used this review program from 2006-2008 to prepare for the USMLE. www.ClinicalReview.com


Foreword

3

CHAPTER CONTENTS Overview of Topics................................................................................................4 Organization of Text .............................................................................................4 Study Plan .............................................................................................................4 Feedback ...............................................................................................................5


USMLE STEP 2

4

INTRODUCTION

INTRODUCTION OVERVIEW OF TOPICS

T

he United States Medical Licensing Examination Step 2 must be successfully passed in order to complete medical school and be eligible for a residency program. This examination tests your ability to apply basic medical and clinical topics to the practice of medicine. It covers a broad variety of topics from every major field of medicine with an emphasis on diagnosis and management. The Step 2 examination seeks to measure your diagnostic acumen, ability to appropriately manage common medical diseases, and prudently care for patients you are likely to see in a variety of settings.

ORGANIZATION OF TEXT This textbook and the Comprehensive Review Course faithfully follow the content outline for the USMLE Step 2 Examination. This course covers essential topics in medicine that you are expected to master. High-yield topics are covered with an emphasis on understanding the key information. Every attempt is made to highlight material that is particularly important for the practice of medicine, and therefore more likely to appear on the boards. Basic science material key to doing well on the USMLE Step 2 are covered in Volume I of this series. Major medical disciplines are covered using a systems-based approach in this volume. Appropriate emphasis is placed on basic science topics and clinical medicine topics, and this series is formulated specifically for the basic science and clinical topics that you are expected to master for the Step 2. Following most sections are pertinent questions to help you further your knowledge in additional areas while practicing what you have learned.

STUDY PLAN How you study for the USMLE depends on how much time you have available. Motivated students and residents who prepare in advance will benefit the most from the online review program and the enormous body of information available on the website. Regardless of how much time you have available, this entire review program is designed to be done piecemeal using the short periods of time you have available on call nights and down time. Each topic is designed to be covered in just a few minutes, and many students report making good progress on the question bank by doing a few questions over lunch each day. Those who are taking call report making good progress on the high-yield PowerPoint reviews. Based on student feedback, our recommendation is to keep the pocket version of the book with you at all times. It is a handy book to go through whenever a few minutes are available. This larger book is seen more and more in the hospital as the exam draws closer, and most residents recommend spending an hour every evening months in advance to review this book. While each individual study plan will be different, there are a few trends that we have noticed with high scorers: 1) Use this book in conjunction with the online questions and your existing reference books 2) Do several dozen questions and read through several topics every night starting at least a month in advance 3) Review the high-yield PowerPoint presentations at least a week in advance of your exam 4) Take note of your strengths and weaknesses and focus on them accordingly 5) Complete the entire Comprehensive Review Course www.ClinicalReview.com


Introduction

5

We recommend going through this resource in its entirety. Topics that have appeared on recent exams or more than once are marked by a star. In addition, topics that the reviewers have deemed to be particularly worthwhile are also highlighted. It is important to reiterate that the purpose of this book is to help you become a better doctor – your score on this exam should reflect your mastery of basic science and clinical medicine. Students that are more pressed for time may want to take advantage of the High-Yield Topic section at the back of the textbook. This section highlights the most frequently tested material and is a good review of key facts. Organizing your study around this is a good way to start, but no comprehensive study plan is complete without a thorough review of practice questions and a chance to reinforce what you have learned. Take advantage of our Comprehensive Review Course to maximize your score and truly get closer to your potential.

FEEDBACK We are always searching for ways to improve our products. If you have any comments or suggestions, we would love to hear from you. Contact us at Support@ClinicalReview.com any time.

Â


6

USMLE STEP 2

www.ClinicalReview.com


Introduction

7

CHAPTER CONTENTS Pharmacology .......................................................................................................8 Infection and Antibiotics .....................................................................................11 Perioperative Management ................................................................................27 Fever and Sepsis .................................................................................................28 Practice Questions ..............................................................................................33

Â


USMLE STEP 2

8

CONCEPTS IN BASIC SCIENCE CONCEPTS IN BASIC SCIENCE PHARMACOLOGY PHARMACOKINETICS The volume of distribution ( ) quantifies the amount of drug found in the body High lipid solubility means high volume of distribution. This is similar with drugs that have a low plasma protein binding and high tissue binding affinities.

VD = ratio of total drug to plasma concentration in body

Clearance refers to the rate of elimination of a drug from the body as compared to its plasma concentration

Half-life o o

The time it takes to clear the body of ½ of the current amount of drug Approximately 4 t½ are required to achieve 95% clearance

0.7

0.7

/ Zero-order elimination o Proceeds linearly with a fixed amount of drug eliminated per unit time o Typically due to saturation of the elimination enzymes First-order elimination o Proceeds as exponential decay with fixed ratio of drug eliminated per unit time o Enzymes are functioning below saturation levels

Zero-order: Fixed amount of drug is cleared First-order: Fixed ratio of drug is cleared

www.ClinicalReview.com


Concepts in Basic Science

9

Loading dose o The amount of drug that may be given initially to reach a target plasma concentration more quickly

Maintenance dose o

The amount of drug that is given to achieve a steady-state concentration between drug plasma concentration and drug elimination

PHARMACODYNAMICS Competitive antagonist o If the left graph is the agonist, the right graph is the agonist plus the competitive antagonist o The right graph also reflects the new potency of the left graph following tolerance Partial agonist o A partial agonist decreases the efficacy of a particular medication Noncompetitive antagonist o A noncompetitive antagonist would be similar to the partial agonist in the graph to the right The therapeutic index of a particular drug is shown in the graph on the right o At ED50 for the drug, half of all patients will respond o As the dose is increased, more and more patients will enter the range of the lethal dose and risk succumbing to the lethal side effects of the drug


10

USMLE STEP 2

EFFICACY AND POTENCY Efficacy o Two drugs that reach the same maximum desired effect are equally efficacious o Heroin and morphine have approximately the same efficacy in that both can lead to 100% analgesia. Codeine is less efficacious than either of these drugs. Potency o Potency refers to the minimum dose needed to achieve a desirable therapeutic effect. o Heroin is more potent than morphine in that a lower dose is needed to achieve 100% analgesia.

DRUG DEVELOPMENT Involves hundreds of millions of dollars in capital with dozens of potential drug candidates After extensive testing, a few prototypic drugs are chosen for further analysis Based on early trials in patients, the number of potential drugs is narrowed Approval is submitted for the best drugs, a process that takes nearly 10 years

www.ClinicalReview.com


Concepts in Basic Science

11

INFECTION AND ANTIBIOTICS SURGICAL PROPHYLAXIS Use of cefazolin or cefuroxime is preferred for most clean general surgery procedures. Upper GI procedures or hepatobiliary procedures receive ciprofloxacin or ceftizoxime with metronidazole. Large bowel resections are prepared with erythromycin with ceftizoxime, bisacodyl, and a clear liquid diet (CLD). GoLytely is often used. Appendectomy receives ceftizoxime or cefotetan with the addition of metronidazole. Cefazolin is commonly used in numerous other types of procedures. Vancomycin is used with procedures dealing with the pancreas or kidneys. Antibiotics must be given Preoperative antibiotics must be at least 30 minutes prior to incision for maximal effectiveness. Most given 30 minutes prior to practitioners will continue prophylactic antibiotics for at least 24 hours postincision. Continue for 24 hours surgery. Of note, the most common hospital-acquired infection and the most following surgery. common infection in patients with diabetes is a urinary tract infection. TABLE 1 SURGICAL PROPHYLAXIS Prophylaxis Surgery Type

Prophylaxis

General Surgery

Cefazolin or cefuroxime.

Upper GI procedures or hepatobiliary

Ciprofloxacin or ceftizoxime with metronidazole.

Large bowel resections

Erythromycin with ceftizoxime, bisacodyl, CLD. GoLytely is often used.

Appendectomy

Ceftizoxime or cefotetan with the addition of metronidazole.

Pancreas or kidneys

Vancomycin

WOUND CONTAMINATION ETIOLOGY The rate of postoperative wound infections is highly dependent on a number of factors inherent to both the patient and the operation. The most important is the type of operation and how contaminated the field becomes as a result of surgery. Adequate skin preparation is essential in order to limit the size of the initial bacterial inoculation into the incisional plane. Wound infections tend to become more common with operations over two hours long and with extensive tissue damage, such as following coagulation of dermal or fascial planes instead of cutting through them. Seromas and hematomas dramatically increase the risk of infection. Patient factors include the elderly, the presence of chronic illness, cardiovascular instability, malnutrition (especially an albumin < 2.5), and the presence of immunosuppressants. As discussed below, the most common cause of postoperative wound infection in non-gastrointestinal An albumin < 2.5 is the largest and non-biliary procedures is due to skin flora: Staphylococcus and Streptococcus perioperative risk factor. spp. With GI and biliary procedures, gram negative and anaerobic organisms are more common.

Â


USMLE STEP 2

12

TABLE 2 CLASSIFICATION OF WOUND CONTAMINATION Classification of Wound Contamination Clean

1.5% infection rate

An elective operation with an incision made under ideal conditions. No entry into gastrointestinal tract or respiratory tract.

CleanContaminated

8% infection rate

An elective operation with an incision made under ideal conditions. Controlled entry into the gastrointestinal tract or respiratory tract with no spillage of contents and no infectious process present (i.e. abscess).

Contaminated

18% infection rate

Emergent operations with entry into the gastrointestinal tract or urinary tract with gross spillage or presence of infection. Trauma operations.

Dirty

40% infection rate

Entry into a perforated viscus or wounds with frank pus, old trauma wounds, or grossly contaminated wounds. Open fractures are an example.

TREATMENT Contaminated wounds are those that have a bacterial count less than 100,000 and intact host responses to infection. Contaminated wounds are typically debrided thoroughly during surgery with satisfactory hemostasis to prevent the formation of a hematoma. Tissues should be approximated and no open spaces should be left (i.e. Halstead principles for wound management). Copious irrigation is used to ensure the removal of all foreign bodies and detritus. Pulse irrigation is more effective at reducing the rate of infection than other methods. Once thoroughly prepared, the tissue should be closed in layers. If these criteria cannot be met, the fascia should be closed and the overlying tissues permitted to close secondarily. The use of a wound V.A.C. device has shown to reduce the time needed for wound closure. The rate of infection of such open wounds is less than 15%.

COMMON ANTIBIOTICS ANTIMICROBIALS PENICILLINS TABLE 3 ANTIMICROBIALS: PENICILLIN DRUG

INDICATIONS

MECHANISM OF ACTION

Penicillin G

GPC (Streptococcus, meningococcus, enterococcus) GPR

Amoxicillin

GPC, GPR, GNR E. coli, H. influenzae, L. monocytogenes Extended spectrum. Pseudomonas. GNR

Inhibits crosslinking of cell wall leading to bacterial lysis Æ bactericidal Inhibits crosslinking of cell wall leading to bacterial lysis Inhibits crosslinking of cell wall leading to bacterial lysis

Piperacillin

COMPLICATIONS

CONTRAINDICATIONS

NOTES

Hypersensitivity (anaphylaxis), hemolytic anemia

Do not use in bacteria with beta-lactamases or w/ known hypersensitivity (common rxn is skin rash,

Especially effective against Neisseria spp., C. perfringens, Fusobacteria, and Treponema

As above plus pseudomembranous colitis. Seizures at high doses. Hypersensitivity, hemolytic anemia, platelet dysfunction.

Do not use if hypersensitivity to penicillins.

Use with clavulanic acid or sulbactam to block beta-lactamases. Wide spectrum. Oral.

Do not use if hypersensitivity to penicillins. Sodium loading.

Resistance to penicillins comes from beta-lactamases that cleave the penicillins to make them impotent. Penicillins bind to PBPs with subsequent inhibition of the transpeptidase step leading to cell lysis. It will not affect organisms that do not have a cell wall. Penicillins are cleared renally, which can be slowed with the administration of probenecid www.ClinicalReview.com


Concepts in Basic Science

13

CEPHALOSPORINS – FIRST GENERATION TABLE 4 ANTIMICROBIALS: FIRST GENERATION CEPHALOSPORINS DRUG

Cefazolin

INDICATIONS

MECHANISM OF ACTION

GPC E. coli K. pneumoniae P. mirabilis

Inhibit cell wall synthesis by preventing crosslinking.

COMPLICATIONS Hypersensitivity (rash). 10% penicillin cross hypersensitivity

Bactericidal.

CONTRAINDICATIONS Aminoglycosides (nephrotoxicity). Alcohol (disulfiram reaction)

NOTES Less susceptible to beta-lactamases.

Resistance to cephalosporins comes from beta-lactamases that cleave the penicillins to make them impotent. Cephalosporins have a hexagonal ring with two functional groups. Penicillins have a pentagonal ring with one functional group. Both are susceptible to beta-lactamases.

CEPHALOSPORINS – SECOND GENERATION TABLE 5 ANTIMICROBIALS: SECOND GENERATION CEPHALOSPORINS DRUG Cefoxitin

INDICATIONS GPC, E. coli Enterobacter H. influenzae K. pneumoniae Neisseria spp. P. mirabilis Serratia spp.

MECHANISM OF ACTION Inhibit cell wall synthesis by preventing crosslinking.

COMPLICATIONS Hypersensitivity (rash).

CONTRAINDICATIONS

NOTES

Avoid with aminoglycosides (nephrotoxicity).

Less susceptible to betalactamases.

Avoid with alcohol (disulfiram reaction)

Broader range compared to first generation.

Bactericidal.

CEPHALOSPORINS – THIRD GENERATION TABLE 6 ANTIMICROBIALS: THIRD GENERATION CEPHALOSPORINS DRUG

Ceftriaxone

INDICATIONS Gram-negatives. Meningitis Resistant organisms Serious infections Broad range. Low activity against gram positives.

MECHANISM OF ACTION Inhibit cell wall synthesis by preventing crosslinking. Bactericidal.

COMPLICATIONS

Hypersensitivity (rash).

CONTRAINDICATIONS

NOTES

Avoid with aminoglycosides (nephrotoxicity).

Cross blood-brain barrier.

Avoid with alcohol (disulfiram reaction)

Ceftazidime is especially good against Pseudomonas.

Broadest range.


USMLE STEP 2

14

MONOBACTAMS / CARBAPENEMS TABLE 7 ANTIMICROBIALS: MONOBACTAMS AND CARBAPENEMS DRUG Aztreonam

Imipenem

INDICATIONS GNR Klebsiella spp. Pseudomonas spp. Serratia spp. GPC, GNR Anaerobes Enterobacter

MECHANISM OF ACTION Prevents synthesis of cell wall. Bactericidal. Improved activity with aminoglycosides Beta-lactamase resistant cell wall synthesis inhibitor. Bactericidal.

COMPLICATIONS

CONTRAINDICATIONS

NOTES

GI Sx.

No effect against anaerobes or gram-positive bacteria.

GI Sx. Rash. CNS toxicity leading to seizures at high doses. Nephrotoxic. Eosinophilia.

Renal disease.

No cross-sensitivity with penicillins. Especially recommended in renal disease. Imipenem is always given with cilastatin to avoid inactivation in kidney. Synthetic.

AMINOGLYCOSIDES TABLE 8 ANTIMICROBIALS: AMINOGLYCOSIDES DRUG

Gentamicin

INDICATIONS Severe gramnegative infections. Aerobes only. Pseudomonas.

MECHANISM OF ACTION

COMPLICATIONS Nephrotoxic.

30S inhibitor. Bacteriostatic. Cause mRNA misreading by preventing formation of initiation complex Æ bactericidal.

Ototoxic. Cause NMJ blockade after surgery. Possible superinfection.

CONTRAINDICATIONS Do not use with cephalosporins. Do not use with loop diuretics. Renal clearance – avoid in renal disease.

NOTES Require O2 for use Æ no effect against anaerobes. Given IV.

R-factor resistance leads to drug inactivation and decreased uptake. May also be modified through acetylation, adenylation, or phosphorylation of the compound. Excreted unchanged by kidney.

TETRACYCLINES TABLE 9 ANTIMICROBIALS: TETRACYCLINES DRUG

INDICATIONS

Doxycycline

B. burgdorferi Chlamydia Mycoplasma Propionibacterium Rickettsia Tularemia Ureaplasma V. cholerae

MECHANISM OF ACTION 30S inhibitor prevents tRNA attachment. Bacteriostatic.

COMPLICATIONS GI Sx. Tooth discoloration and stunted growth in children.

CONTRAINDICATIONS Avoid in renal patients. Avoid in children.

NOTES Do not use in CNS infections.

Fanconi syndrome. Photosensitivity (common). Possible superinfection.

R-factor resistance leads to drug inactivation and decreased uptake. Also has increased removal from cell. Avoid dairy foods, iron-containing preparations, and antacids with use of tetracyclines.

www.ClinicalReview.com


Concepts in Basic Science

15

MACROLIDES TABLE 10 ANTIMICROBIALS: MACROLIDES DRUG

Azithromycin

INDICATIONS

MECHANISM OF ACTION

Pneumonia and URTI. GPC Mycoplasma Legionella Chlamydia Neisseria

COMPLICATIONS

50S inhibitor prevents translocation and inhibits protein synthesis. Bacteriostatic.

GI Sx (common). Hepatitis. Eosinophilia. Skin rashes.

CONTRAINDICATIONS

NOTES

Avoid in hepatic patients. Macrolides are excreted in bile.

Resistance by rRNA methylation leading to prevention of binding to 50S unit.

FLUOROQUINOLONES TABLE 11 ANTIMICROBIALS: FLUOROQUINOLONES DRUG Ciprofloxacin

INDICATIONS GNR Pseudomonas Neisseria spp. Gram-positives (MRSA) UTI, TB GI infections

MECHANISM OF ACTION Prevents action of topoisomerase II. Bactericidal.

COMPLICATIONS GI Sx. Tendon rupture. Theophylline levels increase in plasma.

CONTRAINDICATIONS Avoid in pregnancy and children due to cartilage damage.

NOTES Very potent drugs. Broad spectrum except against anaerobes and some GPC. Renal secretion.

Avoid in renal patients.

Resistance by change in DNA gyrase. Drug penetration may also change. No plasmid-resistance.

SULFONAMIDES / TRIMETHOPRIMS TABLE 12 ANTIMICROBIALS: SULFONAMIDES AND TRIMETHOPRIMS DRUG

Sulfamethoxazole

Trimethoprim

INDICATIONS Gram-positives Gramnegatives Nocardia Chlamydia Recurrent otitis media UTI UTI Prostatitis Shigella Salmonella P. carinii Nocardiosis HIB

MECHANISM OF ACTION

COMPLICATIONS

CONTRAINDICATIONS

Allergic reactions. Inhibits DHP synthase by PABA metabolites. Bacteriostatic.

Prevents the use of DHFR in bacteria. Bacteriostatic. Additive with SMX.

Nephrotoxicity, kernicterus, changes volume of other medications. Rash, anemia, crystalluria.

G6PD deficiency, avoid in infants, use with care with other drugs. Avoid in pregnancy.

NOTES

Combined with TMP. Resembles PABA. Penetrate CNS. Give with folate to avoid anemia.

Megaloblastic anemia, pancytopenia. BMS.

Used for recurrent UTIs. TMP-SMX used for P. carinii pneumonia.

Resistance by modification of DHP synthase, increased synthesis of PABA, or decreased uptake of drug.


USMLE STEP 2

16

OTHER ANTIMICROBIALS TABLE 13 ANTIMICROBIALS: OTHER DRUGS DRUG

INDICATIONS

Chloramphenicol

Meningitis

Metronidazole

Protozoa. G. vaginalis. Anaerobes. Bacteroides. Clostridium spp. Serious infections Gram-positive bacteria resistant to other agents S. aureus / MRSA / PRSP C. difficile Anaerobic infections. B. fragilis. C. perfringens.

Vancomycin

Clindamycin

MECHANISM OF ACTION 50S inhibitor to decrease tRNA binding to A site. Bacteriostatic. Formation of toxic products. Bactericidal.

Prevents cell wall formation through D-ala Dala binding and sequestration.

50S inhibitor. Bacteriostatic.

COMPLICATIONS

CONTRAINDICATIONS

NOTES

Aplastic anemia, gray baby syndrome.

Avoid in pregnancy and infants (low UDPglucuronyl transferase).

Crosses BBB.

GI sx

Avoid with alcohol due to disulfiram reaction.

Used for numerous STDs.

Nephrotoxic. Ototoxic (deafness). DVTs. Red man syndrome.

Renal disease. Renally cleared agent.

Avoid red man syndrome with antihistamines and gradual administration.

Renal and hepatic clearance.

PO. Treat pseudomembranous colitis with flagyl. A lincosamide.

Pseudomembranous colitis

Resistance to vancomycin comes from mutation of D-ala D-ala to D-ala D-lac. Plasmid-mediated. R-factor resistance for chloramphenicol leads to drug inactivation by acetyltransferase inactivation and decreased uptake.

ANTIFUNGALS ANTIFUNGALS TABLE 14 ANTIFUNGALS DRUG

Amphotericin B

Nystatin

Fluconazole

INDICATIONS

MECHANISM OF ACTION

Systemic infections. Meningitis Cryptococcus Aspergillus Histoplasma Candida Mucor

Sequesters ergosterol and has detergent-like effect on cell wall. Fungicidal at high doses.

Candida

Sequesters ergosterol, disrupts cell wall.

Systemic infections. Cryptococcal meningitis. Candida.

Block formation of ergosterol by lanosterol. Fungistatic.

COMPLICATIONS

CONTRAINDICATIONS

F/C. Hypotension. Nephrotoxicity.

Avoid in renal disease.

Arrhythmias.

NOTES

Does not cross BBB. Must be given intrathecally for meningitis. Given slowly via IV.

Oral candidiasis. Topical only.

Gynecomastia, hepatitis, F/C.

Avoid in hepatic disease.

Good absorption and CNS penetration.

www.ClinicalReview.com


Concepts in Basic Science

17

ANTIVIRALS ANTIVIRALS TABLE 15 ANTIVIRALS DRUG Acyclovir

INDICATIONS MECHANISM OF ACTION HSV Inhibits viral DNA VZV polymerase, guanine EBV analog.

Ganciclovir

CMV in IC EBV in IC

Inhibits viral DNA polymerase.

COMPLICATIONS Tremor, nephrotoxicity.

CONTRAINDICATIONS Avoid in renal patients.

NOTES Also treated with vidarabine (adenosine analog)

Pancytopenia, especially with WBCs.

Use with care due to BMS.

Like ACV.

Phosphorylation by the virus of DNA polymerase permits DNA polymerase binding and subsequent inhibition of DNA synthesis.

ANTIPARASITICS ANTIPARASITICS TABLE 16 ANTIPARASITICS DRUG

Metronidazole

INDICATIONS Giardia, Entaboeba histolytica, Gardnerella vaginalis, Trichomonas

MECHANISM OF ACTION Targets DNA.

COMPLICATIONS Disulfiram-like reaction.

CONTRAINDICATIONS

NOTES Does not eliminate cysts.

COMMON INFECTIONS GENERAL CONCEPTS Staphylococcus aureus is best treated with nafcillin or cloxacillin. An alternative to penicillin is cefazolin in methicillin-sensitive species (MSSA). For methicillin resistant species (MRSA), vancomycin is used. An alternative regimen is TMP-SMX or minocycline with rifampin. Coagulase-negative organisms are treated with vancomycin or nafcillin, if sensitive; alternative treatment includes cefazolin or clindamycin. GAS and GBS are treated with clindamycin, with backups including benzylpenicillin and cefazolin. Enterococci infections are treated with ampicillin and gentamicin, and vancomycin is the alternative treatment. E. coli and Proteus spp. are treated first with ampicillin, then cefazolin, gentamicin, or levofloxacin, if the first treatment fails. Potent exotoxins are made generally by gram positive organisms. These include S. aureus, C. perfringens, E. coli, and V. cholerae. C. botulinum generally has the most potent exotoxin, with 1mg sufficient to kill thousands of people. Bacterial lipopolysaccharides (LPS) are formed from the outer membrane of gram negative bacteria. This endotoxin leads to septic shock when released in significant amounts. The O-antigen is the major source of antigenicity and consists of polysaccharides. LPS induces septic shock by activating TNF and IL1, leading to cascade activation of IL6, IL8, GM-CSF, and other immunomodulators.


18

USMLE STEP 2

TABLE 17 GENERAL ANTIBIOTICS General Antibiotics S. Aureus

Nafcillin or cloxacillin. An alternative to penicillin is cefazolin for MSSA.

MRSA

Vancomycin. An alternative regimen is TMP-SMX or minocycline with rifampin.

Coagulase -organisms

Vancomycin or nafcillin. Alternative treatment includes cefazolin or clindamycin.

GAS & GBS

Clindamycin, with backups including benzylpenicillin and cefazolin.

Enterococci infections

Ampicillin and gentamicin, and vancomycin is the alternative treatment.

E. coli & Proteus spp.

Ampicillin, then cefazolin, gentamicin, or levofloxacin, if the first treatment fails.

SOFT TISSUE INFECTIONS Cellulitis is the infection of skin and underlying tissues commonly following local trauma. Cellulitis presents with local pain and swelling, numerous constitutional symptoms, and lymphadenopathy. The affected region is red and warm. Hypotension is common. Areas of necrosis may be found in the area. Common causes include S. pyogenes, S. aureus, S. pneumoniae, and more exotic causes in IC patients. Cellulitis by Streptococcus or Staphylococcus is treated with nafcillin or clindamycin. Penicillin G is the choice for documented Streptococcal infection. Decubitus ulcers are treated with amoxicillin and clavulanate, ampicillin and sulbactam, clindamycin and ciprofloxacin, ceftizoxime, or piperacillin and tazobactam with tobramycin. Necrotizing fasciitis is treated with penicillin with clindamycin and aminoglycosides, ampicillin with sulbactam and aminoglycosides, or imipenem; aggressive surgical debridement is also required. Vancomycin is used with resistant organisms and in patients with numerous risk factors for more severe disease. TABLE 18 SOFT TISSUE INFECTIONS Soft Tissue Infections Cellulitis by Streptococcus or Staphylococcus

Nafcillin or clindamycin. Penicillin G is the choice for documented Streptococcal infection.

Decubitus ulcers

Amoxicillin and clavulanate, ampicillin and sulbactam, clindamycin and ciprofloxacin, ceftizoxime, or piperacillin and tazobactam with tobramycin.

Necrotizing fasciitis

Clindamycin and aminoglycosides, ampicillin with sulbactam and aminoglycosides, or imipenem

TOXIC SHOCK SYNDROME ETIOLOGY AND PATHOPHYSIOLOGY Toxic shock syndrome (TSS) is erythema leading to systemic manifestations of shock due to S. aureus. Superantigen known as TSS toxin-1 (TSST-1) leading to cytokine release throughout the body leads to the diffuse injury and systemic symptoms.

PRESENTATION AND DIAGNOSIS TSS presents with symptoms include fever, hypotension, organ involvement, and distal extremity desquamation. Erythema is present with a scarlatiniform eruption. The tongue is cherry red, and hyperemia of mucous membranes is common. Several organs may be involved leading to serious systemwide damage and complications. Diagnosis is made by culture. www.ClinicalReview.com


Concepts in Basic Science

19

TREATMENT Supportive therapy is used with TSS, including IVF, pressors, antibiotics, and draining the affected regions. Silver sulfadiazine cream is contraindicated; mupirocin ointment is used instead. Standard antistaphylococcus antibiotics are used as previously discussed.

TOXIC EPIDERMAL NECROLYSIS / STEVENS JOHNSON SYNDROME Causes of toxic epidermal necrolysis include dilantin and bactrim. Biopsy of the skin will indicate nondisjunction of the dermal and epidermal interface. Treatment includes steroid therapy and stopping the offending agent.

BITES Following a human bite, an incision and drainage should be done. Augmentin should be given to the patient. Tetanus vaccination should also be verified and IgG should be considered. Human bites lead to infection with S. aureus and Eikenella corrodens. Wounds are typically left open following irrigation and debridement. Bites from animals are treated in a similar fashion. Cat bites are more likely to be infected than dog bites due to the presence of Pasteurella multocida. Bites from most animals (and humans) can be safely treated with Augmentin or Bactrim. A bite from a brown recluse spider can be treated with dapsone.

TETANUS Tetanus is caused by neurotoxin release by C. tetani, leading to a 30% mortality rate if not treated early. Tetanus infection must be especially considered in the presence of dirty wounds contaminated by soil or feces, puncture wounds, burns, and frostbite. For clean, minor wounds, children under 7 should be treated with a DPT (Diphtheriae, Pertussis, and Tetanus) vaccination as prophylaxis. Children over 7 can be treated with the Td (tetanus toxoid) vaccination for similar wounds. Adults should receive a Td if the most recent vaccination is over 10 years ago. All dirty or major wounds receive a Td unless the full immunization schedule has been followed and the most recent vaccination is less than 5 years ago.

MUSCULOSKELETAL INFECTIONS Osteomyelitis is best treated with nafcillin and rifampin or nafcillin with an aminoglycoside. Diabetic foot ulcers are treated with piperacillin and tazobactam or ampicillin and sulbactam with an aminoglycoside. Gonococcal septic arthritis is treated with ceftriaxone, or ciprofloxacin with doxycycline. GAS or S. aureus infection is treated with nafcillin and gentamicin, or nafcillin with ciprofloxacin. Prosthetic joints automatically get vancomycin with rifampin and a cephalosporin. TABLE 19 MUSCULOSKELETAL INFECTIONS Musculoskeletal Infections Osteomyelitis

Nafcillin and rifampin or nafcillin with an aminoglycoside.

Diabetic foot ulcers

Tazobactam or ampicillin and sulbactam with an aminoglycoside.

Gonococcal septic arthritis

Ceftriaxone, or ciprofloxacin with doxycycline.

GAS or S. aureus

Nafcillin and gentamicin, or nafcillin with ciprofloxacin.

Prosthetic joints

Vancomycin with rifampin and a cephalosporin.

Â


20

USMLE STEP 2

PNEUMONIA Community acquired pneumonia, with Mycoplasma or Chlamydia as the likely source, is treated with ceftriaxone or cefuroxime and erythromycin, clarithromycin, or azithromycin. Quinolones can also be used. In ICU patients, ampicillin and sulbactam or piperacillin and tazobactam are used. With aspiration, ciprofloxacin with clindamycin or metronidazole is used or ampicillin and sulbactam. Bronchiectasis is treated in a similar fashion. Hospital acquired pneumonia with E. coli, Enterobacter, Pseudomonas, or Klebsiella is treated with piperacillin and tazobactam with tobramycin, or clindamycin and ciprofloxacin, or ceftazidime, or imipenem with an aminoglycoside. MSSA is treated with nafcillin ; MRSA is treated with vancomycin. TABLE 20 PNEUMONIA Pneumonia Community acquired Mycoplasma or Chlamydia

Ceftriaxone or cefuroxime and erythromycin, clarithromycin,, or azithromycin. Quinolones can also be used.

ICU patients

Ampicillin and sulbactam or piperacillin and tazobactam are used.

Aspiration & Bronchiectasis

Ciprofloxacin with clindamycin or metronidazole is used or ampicillin and sulbactam

Hospital acquired-- E. coli, Enterobacter, Pseudomonas, or Klebsiella

Piperacillin and tazobactam with tobramycin, or clindamycin and ciprofloxacin, or ceftazidime, or imipenem with an aminoglycoside. Infection with Staphylococcus aureus is treated with nafcillin if MSSA; otherwise, vancomycin is used for MRSA.

GASTROINTESTINAL INFECTIONS Cholecystitis and cholangitis have a risk of bacterial infection by Enterococci, and cefotetan, piperacillin and tazobactam, ampicillin and sulbactam are the preferred agents. Gentamicin is commonly added to this cocktail. Diverticulitis may have superimposed infection by anaerobes or Enterococci, and any of the above regimens or clindamycin with ciprofloxacin can be used. Ceftazidime is also effective. Peritonitis or abscess formation is treated with any of the above agents, imipenem, or ceftizoxime. TABLE 21 GI INFECTIONS GI Infections Enterococci

Cefotetan, piperacillin and tazobactam, ampicillin and sulbactam are the preferred agents. Gentamicin is commonly added to this cocktail.

Anaerobes

Any of the above or clindamycin with ciprofloxacin. Ceftazidime is also effective.

Peritonitis or Abscess

Any of the above and imipenem, or ceftizoxime.

GENITOURINARY INFECTIONS Cystitis is treated with TMP-SMX, cephalexin, or nitrofurantoin. Pyelonephritis with E. coli or Proteus is treated with TMP-SMX, aminoglycosides, ceftriaxone, or ciprofloxacin. Gram-negative pyelonephritis, or infection with Pseudomonas or Enterococci is treated with ampicillin and gentamicin, piperacillin and tazobactam with tobramycin, or ciprofloxacin with ampicillin. Prostatitis is treated with ceftriaxone, or ciprofloxacin with doxycycline if the infectious agent is Neisseria gonorrhea or Chlamydia. Enterobacteriaceae infection is treated with TMP-SMX or ciprofloxacin. Chronic prostatitis is treated in a similar manner.

www.ClinicalReview.com


Concepts in Basic Science

21

TABLE 22 GENITOURINARY INFECTIONS Genitourinary Infections Cystitis

TMP-SMX, cephalexin, or nitrofurantoin.

Pyelonephritis with E. coli or Proteus

TMP-SMX, aminoglycosides, ceftriaxone, or ciprofloxacin.

Gram-negative pyelonephritis, or infection with Pseudomonas or Enterococci

Ampicillin and gentamicin, piperacillin and tazobactam with tobramycin, or ciprofloxacin with ampicillin.

Prostatitis Enterobacteriaceae

Neisseria gonorrhea or Chlamydia--- ceftriaxone, or ciprofloxacin with doxycycline TMP-SMX or ciprofloxacin.

CNS INFECTIONS Meningitis is commonly treated with a third generation cephalosporin such as ceftriaxone. Ampicillin is often added if the patient is in a high-risk category. Following neurosurgery, vancomycin and ceftazidime are used. Immunosuppressed patients receive ampicillin and ceftriaxone. Recall that Listeria monocytogenes, fungal, mycobacteria, and community acquired infections are especially common in this group. For encephalitis, ACV is likely the only successful regimen. Brain abscesses are treated with ceftriaxone and metronidazole with penicillin G. TABLE 23 CNS INFECTIONS CNS Infections Meningitis

Third generation cephalosporin such as ceftriaxone. Ampicillin is often added if the patient is in a high-risk category.

Following neurosurgery

Vancomycin and ceftazidime.

Immunosuppressed patients

Ampicillin and ceftriaxone.

Encephalitis

ACV

Brain abscesses

Ceftriaxone and metronidazole with penicillin G.

HEAD AND NECK INFECTIONS Acute sinusitis is commonly the result of S. pneumoniae (TMP-SMX), H. influenzae (cefuroxime), or Moraxella catarrhalis (amoxicillin and clavulanate). Chronic sinusitis receives amoxicillin with clavulanate. Pharyngitis with GAS (exudative pharyngitis) is treated with penicillin. Ulcerative pharyngitis due to HSV or Coxsackie virus is treated with ACV. Membranous pharyngitis due to EBV or Diphtheria receives either no treatment in the former case, or erythromycin for the latter cause. Epiglottitis due to GAS or H. influenzae receives cefuroxime or ceftriaxone. Orbital cellulitis can be treated with either cefuroxime or ampicillin with sulbactam. Acute mastoiditis is treated with either dicloxacillin or cefuroxime; cefuroxime is the preferred agent with S. pyogenes and S. aureus. Chronic mastoiditis is treated with tobramycin with piperacillin and tazobactam. All cases of mastoiditis will likely require surgery.

Â


22

USMLE STEP 2

TABLE 24 HEAD AND NECK INFECTIONS Head and Neck Infections Acute sinusitis

S. pneumoniae ---TMP-SMX. H. influenzae ---cefuroxime. Moraxella catarrhalis ---amoxicillin and clavulanate.

Chronic sinusitis

Amoxicillin with clavulanate.

Pharyngitis with GAS

Penicillin

Epiglottitis due to GAS or H. influenzae

Cefuroxime or ceftriaxone.

Acute Mastoiditis

Dicloxacillin or cefuroxime; cefuroxime is the preferred agent with S. pyogenes and S. aureus. Surgery.

Chronic Mastoiditis

Tobramycin with piperacillin and tazobactam. Surgery.

SEXUALLY TRANSMITTED DISEASES Urethritis, cervicitis, and prostatitis due to N. gonorrhea or Chlamydia are treated with ceftriaxone, ciprofloxacin with doxycycline, or ciprofloxacin with azithromycin. Disseminated gonococcal infection is treated with ceftriaxone followed by ciprofloxacin. PID is treated with ceftriaxone with doxycycline or cefotetan with doxycycline. HSV receives ACV or VCV. Haemophilus ducreyi leading to chancroid is treated with ceftriaxone, erythromycin, or azithromycin. LGV due to Chlamydia is treated with doxycycline. Syphilis due to Treponema pallidum is treated with benzathine penicillin or penicillin G if neurosyphilis is present. TABLE 25 STDS STDs STD

Treatment

Urethritis, cervicitis, and prostatitis due to N. gonorrhea or Chlamydia

Ceftriaxone, ciprofloxacin with doxycycline, or ciprofloxacin with azithromycin

Disseminated gonococcal infection

Ceftriaxone followed by Cipro.

PID

Ceftriaxone with doxycycline or cefotetan with doxycycline.

HSV

ACV or VCV.

Haemophilus ducreyi

Ceftriaxone, erythromycin, or azithromycin.

LGV due to Chlamydia

Doxycyline

FUNGAL INFECTIONS Blastomycosis and histoplasmosis are treated with itraconazole or amphotericin B. Cryptococcus is treated with amphotericin B with 5-flucytosine (5-FC). Superficial Candida albicans infection is treated with fluconazole, 5-FC, miconazole, clotrimazole, or ketoconazole. Fluconazole is used for esophageal infections. Invasive or disseminated Candida infection is treated with amphotericin B or fluconazole. Other fungal infections are commonly treated with amphotericin B, especially if they are lifethreatening or rapidly progressing.

www.ClinicalReview.com


Concepts in Basic Science

23

TABLE 26 FUNGAL INFECTIONS Fungal Infections Blastomycosis and histoplasmosis

Itraconazole or amphotericin B

Cryptococcus

Amphotericin B with 5-FC

Superficial Candida albicans

Fluconazole, 5-FC, miconazole, clotrimazole, or ketoconazole.

Esophageal infections

Fluconazole

Other fungal infections

Amphotericin B

COMMON ORGANISMS BACTERIA GRAM POSITIVE COCCI STAPHYLOCOCCUS TABLE 27 GRAM POSITIVE COCCI: STAPHYLOCOCCUS ETIOLOGY Staphylococcus aureus

FEATURES

Exotoxin

PATHOPHYSIOLOGY

PRESENTATION

Superantigen with IL-1/IL-2 synthesis. Toxic shock syndrome from TSS toxin-1 that leads to cytokine release. Scaled skin syndrome from exfoliative toxin release – exotoxins ET-A and ET-B that breakdown tight junctions. Preformed toxin.

Toxic shock syndrome – fever, hypotension, hyperemia. Scalded skin syndrome Acute bacterial endocarditis. Osteomyelitis

TREATMENT Methicillin Vancomycin Clindamycin TMP-SMX Linezolid

Staphylococcus spp. form a biofilm that protects bacteria from antibiotics.

STREPTOCOCCUS TABLE 28 GRAM POSITIVE COCCI: STREPTOCOCCUS ETIOLOGY Streptococcus pneumoniae

Streptococcus pyogenes

FEATURES IgA protease. Encapsulated.

GAS Bacitracin sensitive. Exotoxin

PATHOPHYSIOLOGY #1 cause of meningitis in children and elderly, otitis media, and pneumonia. Erythrogenic toxin (superantigen) and streptolysin O (ASO titers; hemolysin). M protein antibody. Gas production in necrotizing fasciitis (other causes include C. perfringens and Vibrio). Superantigen SSA expression may lead to systemic symptoms.

PRESENTATION Meningitis Neonatal conjunctivitis. Otitis media, Pneumonia Pharyngitis Cellulitis / Necrotizing fasciitis Impetigo / Erysipelas Scarlet fever – erythema, fever, strawberry tongue, desquamation Toxic shock syndrome Rheumatic fever – erythema marginatum, mitral valve damage Acute glomerulonephritis (PSGN)

TREATMENT Penicillin Ampicillin

Penicillin Ampicillin Clindamycin and vancomycin for necrotizing fasciitis

IgA protease permits organisms to colonize mucosal surfaces and cause infection. Encapsulated bugs that cause infection following splenectomy: S. pneumoniae, N. meningitidis, H. influenzae B., K. pneumoniae.


USMLE STEP 2

24

GRAM NEGATIVE COCCI NEISSERIA TABLE 29 GRAM NEGATIVE COCCI: NEISSERIA ETIOLOGY Neisseria meningitidis

FEATURES

PATHOPHYSIOLOGY

IgA protease. Maltose and glucose fermenter. Encapsulated

Common cause of meningitis.

PRESENTATION

TREATMENT

Meningitis Waterhouse-Friderichsen syndrome.

Vaccination available. Ceftriaxone

GRAM POSITIVE RODS NON-SPORE FORMERS TABLE 30 GRAM POSITIVE RODS: NON-SPORE FORMERS ETIOLOGY Actinomyces israelii

FEATURES

PATHOPHYSIOLOGY

Obligate anaerobe. Draining sulfur granules (yellow flecks).

PRESENTATION Oral / facial abscess. IUD infection.

TREATMENT Penicillin (SNAP)

SPORE FORMERS TABLE 31 GRAM POSITIVE RODS: SPORE FORMERS ETIOLOGY Clostridium difficile Clostridium perfringens

FEATURES Obligate anaerobe Exotoxin Obligate anaerobe.

PATHOPHYSIOLOGY

PRESENTATION

TREATMENT

Caused by clindamycin, neomycin, broad-spectrum antibiotics.

Pseudomembranous colitis – diarrhea, fever, sepsis.

Vancomycin. Metronidazole.

Lecithinase with gas production.

Myonecrosis – gas gangrene with severe infection. Food poisoning from reheated meat.

Surgical debridement and IV antibiotics

Serious exotoxin production occurs with Clostridial spp., which can lead to the development of necrotizing fasciitis as soon as six hours following an operation. The clostridial group function as gram positive rods, and are obligate anaerobes. C. perfringens in particular produces several toxins, including a necrotizing, hemolytic Lecithinase (alpha toxin), a hemolysin (theta toxin), a collagenase (kappa toxin), a hyaluronidase (mu toxin), and a deoxyribonuclease (nu toxin). An endotoxin is also produced by this potent bacterium. Rapid spread requiring surgical intervention and serial debridement may be required in the worst cases, and may lead to death even with aggressive therapy. C. difficile produces a potent exotoxin that leads to pseudomembranous colitis leading to diarrhea. Metronidazole or vancomycin are orally given with infection. Cholestyramine can be given to bind toxin. C. tetani produces a neurotoxin that leads to rigidity and muscular spasms, culminating in asphyxiation and death. Treatment is wound debridement and penicillin. C. botulinum produces a neurotoxin leading to GI symptoms, diplopia, and finally paralysis.

www.ClinicalReview.com


Concepts in Basic Science

25

GRAM NEGATIVE RODS AEROBES TABLE 32 GRAM NEGATIVE RODS: AEROBES ETIOLOGY

FEATURES Blue-green pigment. Lactose nonfermenter. Oxidase positive. Encapsulated

Pseudomonas aeruginosa

PATHOPHYSIOLOGY

Eschar formation in burn patients with secondary infection.

PRESENTATION

TREATMENT

Burn infections Pneumonia in CF Sepsis Otitis externa UTI

Ampicillin and gentamicin Ceftazidime Ciprofloxacin

FACULTATIVE ANAEROBES TABLE 33 GRAM NEGATIVE RODS: FACULTATIVE ANAEROBES ETIOLOGY Helicobacter pylori

FEATURES

PATHOPHYSIOLOGY Uses urease to breakdown mucus layer and lead to ulcer formation in stomach and duodenum

Spirals

PRESENTATION Gastritis Duodenal ulcer Gastric ulcer

TREATMENT Bismuth, metronidazole, amoxicillin, clarithromycin, omeprazole.

ENTEROBACTERIACEAE TABLE 34 GRAM NEGATIVE RODS: ENTEROBACTERIACEAE ETIOLOGY

PATHOPHYSIOLOGY

Bacteremia, lower respiratory tract infection. Most common cause of liver abscess in a patient with diverticulitis

Enterobacter

Escherichia coli

PRESENTATION

Heat-labile toxin with adenylate cyclase production through ADP ribosylation of Gs protein; heatstable works on guanylate cyclase.

Klebsiella pneumoniae

Food poisoning from undercooked meat; watery diarrhea (ETEC). Bloody diarrhea (EIEC, EHEC, EHEC O157:H7). Meningitis in elderly. Bacteremia in biliary tract (most common)

TREATMENT Imipenem, cilastatin, Meropenem, Cefepime, Ciprofloxacin.

Symptomatic Ampicillin and gentamicin Meropenem Ciprofloxacin

Pneumonia with currant jelly sputum. Cholecystitis (uncommon).

Cefotaxime, Ceftriaxone, Gentamicin, Amikacin, Piperacillin / tazobactam.

Proteus mirabilis

Ammonium magnesium phosphate stones.

Struvite stones in UTI.

Ceftriaxone, Gentamicin, Imipenem / cilastatin. Surgical stone removal.

Salmonella

Invasive. 10,000 required for disease.

Food poisoning from poultry, meat, eggs with bloody diarrhea

Permit natural course as antibiotics will worsen disease. Consider TMPSMX for systemic disease.

enteritidis

antigen – polysaccharide of endotoxin; K antigen – capsular antigen (virulence factor); H antigen – found in motile varieties Glucose fermenter


USMLE STEP 2

26

ANAEROBES TABLE 35 GRAM NEGATIVE RODS: ANAEROBES ETIOLOGY

FEATURES

PATHOPHYSIOLOGY

PRESENTATION Normal bowel flora

Bacteroides fragilis

TREATMENT Neomycin, Clindamycin.

Obligate anaerobes lack catalase and are foul smelling gas formers. B. fragilis is the most common bacterium within the intestine

ATYPICAL BACTERIA TABLE 36 ATYPICAL BACTERIA ETIOLOGY

PATHOPHYSIOLOGY

PRESENTATION

Mycobacterium

Found in the apex of the lung due to the highest PO2.

Tuberculosis – night sweats, fever, anorexia, hemoptysis. Primary: Ghon complex in lower nodes with fibrotic healing and caseating granulomas, progressive disease, bacteremia, or allergic reaction. Secondary: Fibrocaseous cavitation with secondary spread in body.

tuberculosis

TREATMENT Rifampin, Isoniazid, Streptomycin, Pyrazinamide, Ethambutol, Cycloserine

FUNGOLOGY TOPICAL INFECTIONS TABLE 37 CUTANEOUS FUNGAL INFECTIONS ETIOLOGY Candida albicans

PATHOPHYSIOLOGY

PRESENTATION

Budding yeast with pseudohyphae and germ tube formation. Water soluble toxin leads to pain.

Oral thrush. Infectious esophagitis. Diaper rash. Vaginal infection.

TREATMENT Nystatin topical therapy. Fluconazole or amphotericin B for systemic infection or vaginal infection.

SYSTEMIC INFECTIONS TABLE 38 SYSTEMIC FUNGAL INFECTIONS ETIOLOGY Aspergillus fumigatus

PRESENTATION

TREATMENT

Lung cavitary lesions with fungus balls followed by disseminated cutaneous infection. Papules, ulcers, eschars.

Amphotericin B

Coccidioidomycosis

Primary – flu-like illness and erythema nodosum common in kids and travelers. Systemic – African Americans and other groups with an HLA predisposition leading to skin, tissue, bone, and meningeal infection.

Local: fluconazole Systemic: amphotericin B

Cryptococcus neoformans

Meningitis or acute pulmonary infection in IC.

Amphotericin B

Diffuse pneumonia in IC.

Pentamidine TMP-SMX Dapsone

Pneumocystis carinii

Mold form outside body. Yeast form, in warmer temperatures, inside body.

www.ClinicalReview.com


Concepts in Basic Science

27

OSTEOMYELITIS TABLE 39 CAUSES OF OSTEOMYELITIS EPIDEMIOLOGY

MOST COMMON

OTHER CAUSES

All persons

S. aureus

Sexually-active

S. aureus

N. gonorrhea

IVDA

S. aureus

Pseudomonas

Sickle cell

Salmonella

S. aureus

PHARYNGITIS TABLE 40 CAUSES OF PHARYNGITIS EPIDEMIOLOGY

MOST COMMON

Bacterial

GAS, N. gonorrhea, HIB, Mycoplasma,

Viral

Adenovirus, Influenza, EBV, HSV

PNEUMONIA TABLE 41 CAUSES OF PNEUMONIA EPIDEMIOLOGY

MOST COMMON

OTHER CAUSES

All patients

S. pneumoniae

Klebsiella, H. influenza, E. coli, Pseudomonas

Asthma, COPD, smoking, IC

H. influenzae

Klebsiella, E. coli, Pseudomonas, Enterobacter, Serratia

Alcoholics, DM, COPD

Klebsiella

E. coli, Pseudomonas, Enterobacter, Serratia

IVDA

S. aureus

E. coli, Pseudomonas, Enterobacter, Serratia

Outbreaks

Legionella

URINARY TRACT INFECTION TABLE 42 CAUSES OF URINARY TRACT INFECTIONS EPIDEMIOLOGY

MOST COMMON

OTHER CAUSES

Most persons

E. coli

Klebsiella, S. saprophyticus

Hospital

E. coli

Proteus, Klebsiella, Serratia, Pseudomonas

PERIOPERATIVE MANAGEMENT INDICATORS OF MORBIDITY AND MORTALITY The most significant perioperative risk factor is an albumin of less than 2.5. According to the Golden criteria, the next most significant criterion is the presence of an S3 gallop.

Â


28

USMLE STEP 2

FEVER AND SEPSIS FEVER POSTOPERATIVE DAYS 1 AND 2 One of the most common postoperative sequelae is the development of a fever. The most common cause of this fever depends on when the pyrexia occurs. Fevers that occur by postoperative day two include atelectasis. Evidence is accumulating that atelectasis can be minimized or even entirely prevented through the use of incentive spirometry. The mechanism of action is unclear, but it appears as if deep inspiration has a more positive effect in preventing atelectasis rather than inducing a patient to cough. Chances of fever developing on postoperative day one or two increases in patients with ventilator support and in patients who have pain on inspiration due to surgery to the chest wall or ancillary structures.

POSTOPERATIVE DAYS 3 THROUGH 5 The most common cause of postoperative fever between postoperative days three and five is due to urinary tract infection. Prophylactic measures can be taken in patients who are more susceptible than normal for developing UTIs. This population includes patients who have indwelling Foley catheters, and who have a historical predilection for developing UTI.

POSTOPERATIVE DAYS 4 THROUGH 6 Deep vein thrombosis is the most common cause of postoperative fever on days four through six. DVT is a serious issue than can lead to sudden death due to a massive pulmonary embolism, and astute surgeons and health care providers take steps to minimize this potentially disastrous outcome. Low-dose heparin given subcutaneously and the use of sequential compressed devices (SCD) have been shown to reduce the incidence of DVT. The best way to reduce the chances of DVT occurring is to have the patient ambulate as soon as possible following surgery. The patient population most at risk of DVT is those who have undergone pelvic procedures, orthopedic surgery, or a general surgery procedure.

POSTOPERATIVE DAYS 5 THROUGH 7 Infections of the surgical wound are most common postoperative days five through seven. Prophylactic antibiotics are an important part of any surgical procedure, even in clean cases. Finally, fevers that occur after postoperative day seven are often iatrogenic, often from medications. TABLE 43 FEVER--POSTOPERATIVE Fever—Post Operative Days 1-2

Atelectasis—prevent with incentive spirometry.

Days 3-5

UTI-prophylactic measures.

Days 4-6

DVT—prevent with low-dose heparin and SCDs. Early ambulation is beneficial.

Days 5-7

Infections of surgical wounds—prevent with prophylactic antibiotics.

Days 7-beyond

Iatrogenic (often from medications).

www.ClinicalReview.com


Concepts in Basic Science

29

SYSTEMIC INFLAMMATORY RESPONSE SYNDROME ASSESSMENT Systemic inflammatory response syndrome (SIRS) is the presence of fever (not always present in the elderly), tachycardia, narrow pulse pressure, hyperpnea, and hypotension in serious cases. There is no end-organ damage, bacteremia, or significant medical support with SIRS. SIRS should be thought of as a systemwide inflammatory state leading to multiple organ dysfunction syndrome (MODS). The proinflammatory state includes numerous APRs and significant release of various cytokines. The number of organs affected determines mortality. Orders include CBC, ABG, CC1, PT, aPTT, D-dimer, fibrinogen, and LFTs. Pancultures are also collected.

MANAGEMENT SIRS is treated with admission to the ICU, close observation, and supportive therapy. Fluid repletion is done and monitoring cardiac function through a pulmonary artery catheter (PAC) is often done. SIRS can evolve into sepsis with multiple organ failure, as discussed below. TABLE 44 SYSTEMIC INFLAMMATORY RESPONSE SYNDROME (SIRS) Systemic Inflammatory Response Syndrome (SIRS) Presentation

Fever (not always present in the elderly), tachycardia, narrow pulse pressure, hyperpnea, and hypotension in serious cases. There is no end-organ damage, bacteremia, or significant medical support.

Diagnosis

CBC, ABG, CC1, PT, aPTT, D-dimer, fibrinogen, and LFTs. Pancultures are also collected.

Treatment

Admission to the ICU, close observation, and supportive therapy. Fluid repletion, and monitoring cardiac function through a (PAC) is often done.

SEPSIS ASSESSMENT Sepsis meets the criteria for SIRS in addition to a symptomatic bacteremia. Organ dysfunction may also be present. Sepsis typically has an identifiable source of infection and numerous general symptoms are often present. Fever and mental status changes are common. IV-lines should be immediately tested and changed, especially central lines. A complete physical exam should be done to identify whether other sources exist. Respiratory alkalosis is often present. Virtually any rampant infection or significant disease process can lead to SIRS and evolve into full-blown sepsis. Laboratory analysis proceeds in a manner similar to that of SIRS. CXR, US, and CT are also often used as necessary.

MANAGEMENT Supportive therapy and maintaining organ perfusion are essential to decrease morbidity and mortality. Antibiotics as discussed above are essential for most cases of sepsis. Infection by IV lines may be treated with imipenem, meropenem, cefoperazone, cefepime, or vancomycin.

Â


USMLE STEP 2

30

TABLE 45 SEPSIS Sepsis Etiology

Infection

Presentation

Sepsis meets the criteria for SIRS in addition to a symptomatic bacteremia. Organ dysfunction may also be present. Fever and mental status changes are common. Respiratory alkalosis is often present.

Treatment

IV-lines should be immediately tested and changed, especially central lines. A complete physical exam should be done to identify whether other sources exist. Laboratory analysis proceeds in a manner similar to that of SIRS. CXR, US, and CT are also often used as necessary. Supportive therapy and maintaining organ perfusion are essential to decrease morbidity and mortality. Antibiotics as discussed above are essential for most cases of sepsis. Infection by IV lines may be treated with imipenem, meropenem, cefoperazone, cefepime, or vancomycin.

TRISOMY TYPE

PRESENTATION

NOTES

Trisomy 21

MR, short stature, obesity, warm behavior?????, seizure, r/o autism rule out all that follows or just autism, premature aging, Alzheimer, microcephaly, VSD, ToF, PDA.

Down Syndrome.

Trisomy 18

Severe psychomotor retardation, growth retardation, microcephaly, microphthalmia, micrognathia, seizure, hypotonia, Arnold-Chiari, prominent occiput, short palpebral fissures, cataracts, thumb aplasia, VSD, PDA, omphalocele, overriding fingers and clenched hands.

Edward Syndrome.

Trisomy 13

PDA, VSD, ASD, dextrocardia, holoprosencephaly, microphthalmia, cleft lip, cleft palate, capillary hemangiomata, polycystic kidneys, rocker bottom feet.

Patau Syndrome.

GROWTH FACTORS TYPE

SOURCE

FUNCTION

PDGF

Platelets, endothelial cells, placenta.

Promotes CT proliferation, glial growth, and SMC development.

EGF

Submaxillary gland, Brunner’s gland.

Mesenchymal growth, glial and epithelial cell replication.

TGF-A

Transformed cells.

Normal wound healing; similar to EGF.

TGF-B

TH1 cells, NK cells.

Anti-inflammatory mediator with wound healing, inhibits macrophages and lymphocytes.

NGF

Neurons

Promotes neural cell survival and growth.

EPO

Mesangial cells.

Erythrocyte proliferation.

IGF-1

Liver

Proliferation of cells throughout the body; somatomedin C.

www.ClinicalReview.com


Concepts in Basic Science

31

HEREDITARY TUMORS TYPE

ETIOLOGY

PATHOPHYSIOLOGY

DEFECT

NOTES

Li-Fraumeni Syndrome

P53 TSR

Cell cycle regulation, apoptosis.

17p13 defect

Brain tumor, sarcoma, leukemia, breast cancer.

Familial retinoblastoma

RB1 TSR

Cell cycle regulation.

13q14

Retinoblastoma, osteogenic sarcoma.

Wilms tumor

WT1 TSR

Transcriptional regulation.

11p13

Pediatric kidney CA.

Neurofibromatosis 1

NF1 TSR

RAS inactivation promoter.

17q11

Neurofibromas, sarcomas, gliomas.

Neurofibromatosis 2

NF2 TSR

Cell membrane to cytoskeleton linker.

22q12

Vestibulocochlear schwannomas, meningiomas, astrocytomas, Ependymomas.

Familial adenomatous polyposis

APC TSR

Adhesion molecule signaling.

5q21

Colon CA.

Tuberous sclerosis 1

TSC1 TSR

9q34

Facial angiofibromas + tubers

Tuberous sclerosis 2

TSC2 TSR

GTPase activator.

16

Hamartomas throughout, rhabdomyosarcomas.

Deleted in pancreatic carcinoma

DPC4 / SMAD4 TSR

TGF-B/BMP signal transduction regulator.

18q21

Pancreatic CA, colon CA.

Deleted in colorectal carcinoma

DCC TSR

Transmembrane receptor for axonal guidance.

18q21

Colorectal CA.

Familial breast CA 1

BRCA1 TSR

Double strand breakage repair w/ Rad51.

17q21

Breast and ovarian CA; more common in women.

Familial breast CA 2

BRCA2 TSR

Double strand breakage repair.

13q12

Breast and ovarian CA; more common in men – also causes pancreatic CA and prostate CA.

Peutz-Jeghers syndrome

STK11 TSR (STK)

VEGF regulator.

19p13

Hyperpigmentation, hamartoma polyps, colorectal CA, breast CA, ovarian CA.

Hereditary nonpolyposis colorectal CA type 1

MSH2 TSR

DNA mismatch repair enzyme.

2p22

Colorectal CA.

Hereditary nonpolyposis colorectal CA type 2

MLH1 TSR

DNA mismatch repair enzyme.

3p21

Colorectal CA.

Von Hippel-Lindau syndrome

VHL TSR

Transcription elongation regulator.

3p26

Renal CA, Hemangioblastoma, pheochromocytoma.

Familial melanoma

CDKN2A TSR

Inhibits CDK4/6.

9p21

Melanoma, pancreatic CA.

Basal cell carcinoma

PTCH TSR

Hedgehog signal regulator.

9p22

Basal cell CA.

MEN1

MEN1 TSR

11q13

Parathyroid, pituitary, islet cell, carcinoid

MEN2

RET, MEN2

TK for GDNF.

10q11

Medullary thyroid, pheochromocytoma, mucosal hamartomas.

Beckwith-Wiedmann

p57, KIP2

Cell cycle regulator.

11p15

Wilms tumor, adrenocortical CA, hepatoblastoma.

Ataxia telangiectasia

Multiple, esp in ATM gene

Failure to halt cell cycle after damage to DNA.

11q22

Lymphoma, ataxia, immunodeficiency.

Bloom syndrome

BLM

DNA helicase.

15q26

Solid tumors, immunodeficiency.

Â


USMLE STEP 2

32

ONCOGENES / CARCINOGENS TYPE

TUMOR

Afltatoxin

Hepatocellular carcinoma

Vinyl chloride

Hepatocellular carcinoma

Nitrosamines

Esophageal CA and stomach CA

Asbestos

Mesothelioma, bronchogenic CA

Arsenic

SCC

CCl4

Centrilobular necrosis of liver

Aniline dyes

Bladder CA (TCC)

HTLV1

Adult T-cell leukemia

HBV

Hepatocellular carcinoma

HCV

Hepatocellular carcinoma

EBV

Burkitt’s lymphoma, nasopharyngeal CA

HPV 16, 18, 31, 33

Cervical CA

HHV8

Kaposi sarcoma, B cell lymphoma

TUMOR MARKERS TYPE

LOCATION

NOTES

AFP

Hepatocellular CA, embryonal cell tumors of the testes, yolk sac tumors, mixed germ cell tumors.

Follow after Dx.

Beta-2-microglobulin

Multiple myeloma, CLL, lymphomas.

Prognostic indicator.

B-hCG

Germ cell tumors, choriocarcinoma, mediastinal tumors.

Follow levels.

Bladder tumor antigen

Bladder CA, used w/ NMP22.

Recurrence of tumor indicator.

CA 15-3

Breast cancer.

Elevated in advanced disease.

CA 27.29

Breast cancer.

Prognostic marker.

CA 125

Ovarian CA; positive in fibroids, endometriosis, lung CA.

Possible screening test, but would miss many early CA (hence why many are not used as screening tests).

CA 72-4

Ovarian CA, stomach CA.

CA 19-9

Pancreatic CA +/- colorectal CA.

CEA better for colorectal CA; highly sensitive for pancreatic CA.

Calcitonin

Medullary thyroid CA (parafollicular C cells).

Early detection of CA.

CEA

Colorectal CA; also elevated in lung CA and breast CA.

Chromogranin A

Carcinoid, neuroblastoma, SCLC.

Estrogen receptors / progesterone receptors

Breast cancer.

Tamoxifen (raloxifene for osteoporosis).

HER-2/neu / c-erbB-2

Breast cancer.

Positive in 1 in 3 patients; prognostic indicator – use trastuzumab in patients w/ positive result.

www.ClinicalReview.com


Concepts in Basic Science

33

Neuron-specific enolase

SCLC, neuroblastoma, carcinoid.

Follow-up testing.

NMP22

Bladder CA.

Prognostic.

PSA

Prostate CA, BPH.

NOT for use as a screening test; prognostic; not 100%.

Prostate acid phosphatase

Prostate cancer.

PSA is more sensitive.

S-100

Melanoma, Histiocytosis X.

Metastatic disease.

PRACTICE QUESTIONS A patient is taking medications that are eliminated via first order kinetics. Which of the following best describes first order kinetics for drug elimination? A. B. C. D. E.

Elimination cannot be increased by inducing liver function enzymes Half life of the drug is a fixed amount and not related to rate of elimination Rate of elimination is proportional to the concentration of drug in the body Rate of elimination occurs at a certain level independent of drug concentration Volume of distribution is directly related to plasma concentration

The best answer is Rate of elimination is proportional to the concentration of drug in the body. First order kinetics for drug elimination refers to a rate of elimination of drug that is directly proportional to the concentration of drug in the body. As a result, a constant fraction of drug is eliminated per unit of time, and is dependent only on drug concentration. A fixed amount is not eliminated (i.e. 10 mg / min) as in zero order kinetics. Therefore, drug half life is equal to the rate of elimination. Other relationships: loading dose is equal to volume of distribution times the desired steady state plasma concentration; volume of distribution is equal to dose divided by plasma concentration.

A patient on vancomycin is found to have a high trough level. What is the next best step? A. Decrease the dose and increase the interval B. Decrease the drug dose C. Increase the drug dose D. Increase the interval of dosing E. Skip the next dose The best answer is Increase the interval of dosing. A high vancomycin trough level should lead to an increase in interval, i.e. from 12 hours to 24 hours. A high peak should lead to a decrease in dose.

Â


USMLE STEP 2

34

An ICU patient being treated for multifactorial sepsis develops erythema multiforme. What is the most likely antibiotic leading to this allergic reaction? A. Augmentin B. Cefoxitin C. Fluconazole D. Isoniazid E. Trimethoprim-sulfamethoxazole The best answer is Trimethoprim-sulfamethoxazole. Bactrim has been associated with both frequent mild allergic reactions and serious adverse effects including Stevens-Johnson syndrome (and its precursor, erythema multiforme), myelosuppression, mydriasis, agranulocytosis, as well as severe liver damage (cholestatic hepatosis, hepatitis, liver necrosis, fulminant liver failure). Due to displacement of bilirubin from albumin there is an increased risk of kernicterus in the newborn during the last 6 weeks of pregnancy. Also renal impairment up to acute renal failure and anuria has been reported. These side-effects are seen especially in the elderly and may be fatal. The folic acid is likely not the best option for the treatment of some adverse effects with associated with TMP-SMX, a better treatment is probably administration of folinic acid.

What is the most common bacterium in the colon? A. B. C. D. E.

B. fragilis E. coli Peptostreptococcus Staphylococcus Streptococcus

The best answer is B. fragilis. The most common colonic bacteria is the bacteroides species, specifically B. fragilis. The next most common bacterium is E. coli. The most common bacterium of the skin is staphylococcus.

A 27 year old recently postpartum female develops significant right upper quadrant pain. She recalls taking antibiotics for an unrelated condition recently. Ultrasound of the right upper quadrant detects sludge within the gallbladder, but no pericholecystic fluid, no significant stones, and no obstruction. Which of the following antibiotics can lead to sludge formation within the gallbladder? A. B. C. D. E.

Cefoxitin Ceftriaxone Ciprofloxacin Erythromycin Metronidazole

The best answer is Ceftriaxone. One of the side effects of ceftriaxone, a treatment for gonorrhea, is the formation of biliary sludge. Other causes of sludge formation within the gallbladder include pregnancy, fasting after a gastrointestinal surgery, trauma, TPN, and organ transplantation. Biliary sludge formation is a reversible process.

www.ClinicalReview.com


Concepts in Basic Science

35

Which of the following antibiotics is bactericidal? A. B. C. D. E.

Chloramphenicol Ciprofloxacin Clindamycin Doxycycline Sulfamethoxazole

The best answer is Ciprofloxacin. The bacteriostatic antibiotics include tetracyclines, clindamycin, sulfa drugs, trimethoprim, chloramphenicol, dapsone, INH, and macrolides. The bactericidal antibiotics include fluoroquinolones, rifampin, and aminoglycosides.

Which of the following is not an accepted method of decreased the incidence of surgical site infection? A. B. C. D. E.

100% FiO2 during the operation 100% FiO2 for six hours in the PACU Decreased temperature in the operating room Preoperative antibiotics Preservation of the sterile field

The best answer is Decreased temperature in the operating room. Decreased body temperature during an operation leads to a dramatic increase in the incidence of infection. The other options are necessary means of minimizing the incidence of surgical site infections.

Â


36

USMLE STEP 2

www.ClinicalReview.com


Concepts in Basic Science

37

CHAPTER CONTENTS Biostatistics .........................................................................................................38 Epidemiology ......................................................................................................44 Preventive Medicine ...........................................................................................45 Immunology ........................................................................................................48 Practice Questions ..............................................................................................64

Â


38

USMLE STEP 2

BIOSTATISTICS AND EPIDEMIOLOGY BIOSTATISTICS AND EPIDEMIOLOGY BIOSTATISTICS STATISTICAL TESTING Sample size = n o Sample size is the total number of patients enrolled in a particular study Standard deviation:

∑ Standard error of the mean:

√ PREVALENCE AND INCIDENCE Prevalence o

The number of cases of a particular disease at a point in time

o

Example: there are 2,000 cases of influenza at this moment in the United States

Prevalence: number of cases at a point in time Incidence: new cases over a period of time

Incidence o

The number of new cases of a particular disease within a span of time

o

There were 3,200 cases of diabetes reported between January and June of last year

SENSITIVITY Sensitivity: Screening test, rules out disease Specificity: Confirmatory test, rules in disease

Once a very sensitive test identified a patient as not having a disease, she or he is effectively ruled out A very sensitive test has a low rate of false negatives. Hence, you can trust a true negative result, indicating that if a test result is negative, the patient does not have a disease and is effectively ruled out. This permits sensitive tests to be used on a population as a screening test. Positive results are confirmed with a specific test.

www.ClinicalReview.com


BIOSTATISTICS AND EPIDEMIOLOGY

39

SPECIFICITY

A specific test is used to confirm a positive test result obtained from an initial screening test. A specific test has a very low rate of false positives, so a true positive is considered to be trustworthy. If a patient obtains a positive result on a specific test, it is considered to be indicative of having a disease (i.e. ruling in).

Disease -

+

TP

FP

-

FN

TN

Test

+

POSITIVE PREDICTIVE VALUE

Positive Predictive Value: Chance of disease w/ positive result

A positive predictive value is used to determine the chance of having a disease given a positive test result A test with a very low false positive rate will have a more meaningful positive result Remember that the positive predictive value is always used in conjunction with the pretest probability to determine the chance the patient truly has a disease

NEGATIVE PREDICTIVE VALUE

The negative predictive value determines the chance of not having a particular illness given a negative test result The negative predictive value is also used in conjunction with pretest probability and clinical suspicion


40

USMLE STEP 2

ODDS RATIO

The odds ratio is used in retrospective studies to determine the particular effect of a risk factor on disease The odds ratio is calculated using a table similar to that for sensitivity and specificity

RELATIVE RISK

Relative risk is used to determine the chance of a particular exposure leading to disease It is used in cohort or prospective studies

ATTRIBUTABLE RISK

A variation of relative risk, attributable or absolute risk is used to determine the actual number of cases that can be attributed to a particular cause

GAUSSIAN DISTRIBUTION A bell-shaped distribution with three distinct standard deviations o First standard deviation = 68% of all values o Second standard deviation = 95.5% of all values o Third standard deviation = 99.7% of all values

MEAN, MEDIAN, AND MODE Mean = Median = Mode o Mean = average of all values o Median = middle value when all values are in ascending order o Mode = most common value

Mean: average value Median: middle value Mode: most common value

www.ClinicalReview.com


BIOSTATISTICS AND EPIDEMIOLOGY

41

OTHER DISTRIBUTIONS Left-skewed distribution o Mean < Median < Mode Right-skewed distribution o Mode < Median < Mean Bimodal distribution o Two modes, 1 median, 1 mean Best descriptors of nonparametric distribution—median and quartiles


USMLE STEP 2

42

HYPOTHESIS TESTING Null hypothesis o Assumes that a particular treatment has no effect o This is the default assumption Alternative hypothesis o Assumes that a particular treatment has the desired effect o This is the assumption that is being tested o The assumption is accepted if it meets a particular statistical criterion i.e. p<0.05 on T-test

TYPE I ERROR

Null hypothesis: treatment has no effect Alternative hypothesis: treatment has effect Type I error: treatment falsely believed to have effect (repeat study) Type II error: treatment falsely believed NOT to have effect (increase sample size / power)

Accepting that a particular treatment has a desired effect when it really does not Occurs when you reject the null hypothesis Repeating a study is important to determine whether a type I error has occurred Decreasing the p value in very important studies is also an important step, as this decreases the likelihood of making a type I error o The p-value is an estimate of the chance of making a type I error o Less than 5% is typically considered allowable – p<0.05

TYPE II ERROR Deciding that a particular treatment has no effect when it really does Occurs when you falsely accept the null hypothesis Avoided by increasing the power of a study

POWER Power is directly related to sample size: Increasing sample size increases power Power is also dependent on measurement fidelity o An improved ability to measure differences increases power of a study Lack of power may lead to false acceptance of the null hypothesis (type II error) o The study was unable to find a difference when one really existed o The power of a study is determined by the need to estimate expected differences

CORRELATION X-axis has one variable and the Y-axis has another variable mapped. A best-fit line is drawn. A correlation coefficient of 0 means that there is no direct or inverse correlation between the two variables. However, the two variables might still be otherwise related. A correlation coefficient between 0 and 1 means a direct correlation exists – as one goes up, the other variable generally goes up. This does not mean that one variable is causative, the other dependent. A negative correlation implies an inverse relationship.

www.ClinicalReview.com


BIOSTATISTICS AND EPIDEMIOLOGY

43

STATISTICAL TESTS T-test o

Used to determine whether two quantified variables are sufficiently different based on their distributions and standard deviations Calculates a p value, which can be used to accept or reject the null hypothesis

o ANOVA o Used to determine whether three or more variables are T-test: Two group analysis for significance different ANOVA: Test for significance o Similar to T-test except with more than 2 groups with more than 2 groups Chi-squared o Used to determine whether any number of groups are different, but does so using qualitative terms The difference between T-tests and ANOVA vs. chi-squared is that chi-squared tests use categorical outcomes such as “yes” and “no”

PRECISION AND ACCURACY Precision o The ability to reproduce a test result o A finely calibrated machine will always return the same value o Reproducibility and reliability are key elements. Low susceptibility to random variation is also important. Accuracy o The ability to obtain a result close to the Truth o A test that always positively diagnosis people with a disease would have very high accuracy o Validity is the key element.

STUDY DESIGN A study must be designed to yield a practical outcome within a reasonable span of time with a minimum susceptibility to confounding Randomization, blinding, minimizing bias, using placebos, internal controls, and a wide sample size should be used whenever possible

CASE-CONTROL TRIAL Retrospective trial that takes a group of patients with known disease and looks into the past to determine the effect of various measured risk factors Susceptible to recall bias as patients with knowledge of their disease are likely to recall being subjected to a particular risk (i.e. high tension power lines) Very useful study when attempting to analyze risk factors leading to a rare disease

COHORT STUDY Prospective study that uses two groups of patients – one with a known exposure to a risk factor, and a control group known not to be exposed to the risk factor Sometimes requires a long span of time, during which loss of patients is likely to occur Useful when examining the effect of various risk factors on the development of disease


USMLE STEP 2

44

CLINICAL TRIAL Randomized – minimizes selection bias Double-blind – minimizes investigator and patient bias (measurement bias) Multi-centered – reduces confounding from a narrow sample Placebo control – helps ensure the trial is double-blind and reduces measurement bias Crossover control – patient receives treatment for half of the trial, then placebo for the remainder Also reduces measurement bias The “best” study design in that the outcomes can typically be trusted if all tenets of study design are faithfully followed The major detriment is high cost In order for a randomized study to be properly evaluated, the sample size must be known

META-ANALYSIS Uses advanced statistical methods to compare similar published studies by pooling their numbers together and attempting to see whether this increased power is sufficient to identify differences Limitations are that it relies on work done by other investigators, which may not always be accurate or precise

CASE STUDY Examining the outcome of a single patient with a disease who received a particular treatment and its outcome Useful to note interesting or odd effects of treatment, or to note an off-label use of a medication May spur more rigorous clinical studies

BIAS Late-look bias – re-examination of the collected data after the study has been unblinded Lead time bias – earlier examination of patients with a particular disease lead to earlier diagnosis, giving the false impression that the patient will live longer Measurement bias – an investigator familiar with the study does the measurement and makes a series of errors towards the conclusion s/he expects Recall bias – patients informed about their disease are more likely to recall risk factors than uninformed patients Sampling bias – the sample used in the study is not representative of the population and so conclusions may not be relevant Selection bias – lack of randomization leads to patient’s choosing their experimental group and introduces confounding.

EPIDEMIOLOGY According to the 2005 National Division of Vital Statistics, the overall top ten causes of death for both males and females are heart disease, cancer, cerebrovascular disease, chronic obstructive pulmonary disease, unintentional injuries / accidents, diabetes mellitus, influenza and pneumonia, Alzheimer disease, nephritis, nephrotic syndrome and nephrosis, and septicemia. Together, these top ten causes of death account for nearly 80% of all death in the United States of America. It is important to understand the relevance of this list. The top ten causes of death listed do not imply that these ten causes deserve the greatest amount in public health resources – quite often, some causes of death that are not ranked are of great importance to physicians and public health authorities. Some such causes that did not make the list include tracheal cancer, lung cancer, and motor vehicle accidents. Overall, death may be attributed to heart disease in about 29% of all cases. Cancer comes in at 23%, and stroke leads to about 7% of all deaths. The top three categories account for nearly 60% of all causes of death.

www.ClinicalReview.com


BIOSTATISTICS AND EPIDEMIOLOGY

45

PREVENTIVE MEDICINE Preventive medicine deals with the branch of medicine dedicated to avoiding disease and promoting sound and healthy practices for well-being. Preventive medicine is responsible for the eradication of small pox in 1977, vaccines for typhoid, diphtheria, and cholera, and a treatment for rabies in the late 1800s. Preventive medicine has three levels of prevention: primary prevention is to avert the onset of illness in healthy people; secondary prevention is to prevent the progression of a disease in people who are already afflicted; and tertiary prevention is directed towards reducing disability from a disease. Preventive medicine has fronts in routine health screening and education, cancer screening, substance abuse, domestic violence, obesity, malnutrition, immunization, and more. TABLE 46 PREVENTIVE MEDICINE Preventive Medicine Primary level

Avert onset of illness in healthy people.

Secondary level

Prevention of progression of disease in already afflicted.

Tertiary level

Reducing disability from a disease.

USE OF TESTS Screening tests are tools for the discriminating physician to use in preventive health care. However, not all screening tests are appropriate for all patients. Some screening tests reach their full efficacy only after a certain age, or in certain populations. For example, young adults have little benefit for yearly colonoscopy due to the rare incidence of colon cancer in an individual with no family history or risk factors; in fact, the risk of perforation and side effects from the procedure far outweigh the chance of detecting a malignant lesion. The best screening tests have guidelines that have been tested in randomized, double-blind, placebo-controlled, multicenter trials – a tall order for many research studies. As a result, the guidelines proposed by various institutions are based on the best available evidence and some groups frankly state that there is no preponderance of evidence indicating a good utility for certain screening tests. Realize that routine screening for disease is typically done with the best interest of the patient and population in mind – medicine is more than treating the individual, it is also improving the overall health of society.

ROUTINE SCREENING Routine screening tests should be carried out at every visit to a physician. For examination purposes, the correct answer to what a physician should do next may sometimes be to carry out an appropriate screening exam for that patient population. Generally, young adults (defined in this section as those less than 40 years of age) should receive physical exams every 3-5 years, blood pressure and weight measurements every 2 years, cholesterol measured every 5 years, glaucoma screens at least once, and a Pap smear in women every 1-3 years. The screening guidelines in middle-aged adults (defined here as those between 40 and 65) should continue with the screening recommendations for young adults and start having fasting blood sugars measured every 3 years, mammography yearly for women, and colon cancer screening every 5 years. Vision screening should increase in frequency to every 3 years. Older adults (those over 65) should receive yearly physical exams, periodic hearing tests, and vision screening yearly. A digital rectal exam should be done yearly after age 50 with tests for occult bleeding. The United States Preventive Services Task Force (USPSTF) comes out with yearly screening guidelines.


46

USMLE STEP 2

TABLE 47 ROUTINE SCREENING SCHEDULE Routine Screening Schedule Young adults (Under 40)

Physical exams every 3-5 years,; BP and weight every 2 years; cholesterol levels every 5 years; glaucoma and pap tests every 1-3 years.

Middle age (40-65)

Same as young adults plus fasting blood sugars every 3 years, mammography yearly in women, colon cancer screen every 5 years, vision every 3 years Digital, yearly rectal exam after 50.

Older adults (over 65)

Yearly physicals, periodic hearing tests, yearly vision test.

CANCER SCREENING BREAST CANCER Breast cancer is the second most common cause of death in women. Screening tests recommend self breast exams every month about a week after the onset of menses. Annual exams by a physician should be made at least once a year, but even this examination is controversial up until the age of 40 or 50. The American Cancer Society recommends clinical breast exams every three years, then yearly after the age of 40. Mammograms are recommended every 1-2 years from ages 40-49, then yearly. The data to conduct mammograms in the 40s is currently controversial. Mammograms should not be conducted earlier than age 35 due to the density of the breast shadow and limited clinical utility of this test in younger women. However, a large multicenter trial with 40,000 women completed in 2005 suggests that mammograms using digital imaging might be appropriate for younger women. It is believed that as digital imaging solutions replace current x-ray film technology, more and more women would be eligible for the diagnostic benefits of mammography.

CERVICAL CANCER Screening for cervical cancer is done with Papanicolaou (Pap) smears. Cervical cancer is commonly associated with certain human papillomavirus (HPV) types. Screening is generally recommended with yearly Pap smears starting at the age of 18 or when sexual activity begins, whichever is sooner. Pap smears may be performed every 3 years following 3 normal smears one year apart. Pap smears typically continue until the age of 70 in any woman who has a cervix.

COLORECTAL CANCER Colorectal cancer screens are somewhat controversial. Most groups recommend fecal occult blood testing (FOBT) with digital rectal exams (DRE) every year after the age of 50. Flexible sigmoidoscopy should be done every 5 years or colonoscopy every 10 years. Some physicians recommend conducting colonoscopy in lieu of sigmoidoscopy, and some studies have discounted the benefits of FOBT. DRE should be done in all patients as up to 50% of all colon cancers may lay within reach of the clinician’s finger.

OVARIAN CANCER Ovarian cancer is the fourth most common cause of death in women. Risk factors include positive family history and having a pregnancy later in life. There are no screening tests.

PROSTATE CANCER Adenocarcinoma of the prostate is the third leading cause of death in men. Most groups prefer to individualize the screening process, but generally recommend yearly DRE and prostate specific antigen (PSA) tests in individuals over the age of 50. www.ClinicalReview.com


BIOSTATISTICS AND EPIDEMIOLOGY

47

Significant elevations in PSA should be tested with a transrectal ultrasound (TRUS) of the prostate gland to confirm the clinical suspicion. Due to the increased risk of prostate cancer in African American males, some clinicians may choose to begin screening for prostate cancer earlier in this population.

SKIN CANCER Screening for skin cancer has little backing in the medical literature. However, the ACS recommends physical exams of the skin every 3 years until the age of 40, then yearly. Individuals with particular risk factors should always get personalized schedules for screening exams.

TESTICULAR CANCER Testicular cancer screening tests also have little supporting evidence in the literature. The ACS recommends testicular exams with every physical exam for cancer screening. TABLE 48 CANCER SCREENING Cancer Screening Breast

Self exam every month, clinical breast exam every 3 years until 40. Mammograms every 1-2 years between 40-49.

Cervical

Yearly Pap smear starting at 18 or when sexually active. May be performed every 3 years following 3 normal smears one year apart. Typically continue until 70.

Colorectal

FOBT with DRE every year after 50. Sigmoidoscopy every 5 years or colonoscopy every 10 years.

Ovarian

No screening tests. Family history important as is late in life pregnancy.

Prostate

DRE and PSA yearly after 50. African American males earlier than 50.

Skin

Physical exam of skin every 3 years until age 40, then yearly.

Testicular

With every physical exam.

EXPOSURE WARM WEATHER Heat exhaustion occurs from dehydration and hyponatremia but with a normal core body temperature; treatment should center on fluid resuscitation and electrolyte replenishment with the patient resting in a cool environment. Heat stroke is more significant with a loss of hypothalamic control over temperature management and CNS dysfunction. Heat stroke should be managed as an emergency to prevent multiorgan failure, rhabdomyolysis, and death. The standard of care with heat stroke is to immediately cool the patient, if their body temperature is over 40 degrees Celsius.

COLD WEATHER Long periods of exposure to the cold can lead to frostbite, especially on commonly uncovered surfaces like the ears, nose, and extremities. Frostbite occurs due to diminished blood flow to a particular region as the body attempts to conserve body heat; treatment of frostbite is to warm the body with warm water, give the Td vaccine, and consider giving antibiotics to prevent infection. Surgical debridement of affected regions is sometimes necessary. Hypothermia is another complaint that can lead to depressed body temperatures. It presents with shivering, CNS effects, and arrhythmia including an upward J point deflection

Â


USMLE STEP 2 48 known as the Osborne wave. Hypothermia is treated with immediately warming the patient and warm IV fluids. Warm-fluid lavage may also be necessary. Death is not declared until the patient has reached room temperature and remains without a pulse. TABLE 49 EXPOSURE Exposure Heat exhaustion

Due to dehydration and hyponatremia. Replace fluids and electrolytes in cool area.

Heat stroke

Loss of hypothalamic control and CNS dysfunction. Immediately cool.

Frostbite

Diminished blood flow to periphery. Warm with warm water, Td.

Hypothermia

Shivering, CNS effects, arrhythmia. Treat with warm IV fluids.

IMMUNOLOGY IMMUNIZATION ADULT IMMUNIZATIONS Adult immunizations are an important part of preventive health care, as nearly 100,000 adults die yearly from preventable infections. Immunizations should be given at specified intervals to meet standards developed by the USPSTF. Tetanus vaccines in the form of Td should be given every 10 years, and should be repeated in the face of an infected wound. The two dose hepatitis A vaccination should be given to all adults at risk. Hepatitis B vaccinations should be given as the three dose series especially in susceptible individuals. The pneumococcal polysaccharide vaccine should be given as one dose prior to the age of 65 to those at risk, then repeated after the age of 65 to all adults. The influenza vaccine should be given annually to all at-risk individuals prior to the age of 50, then yearly in all adults. In patients without sufficient documented infection in childhood or those who have never been infected, the mumps, measles and rubella (MMR) vaccination and varicella vaccination (VZV) should be given. The polysaccharide meningococcal vaccination should be given to all at-risk individuals.

IMMUNIZATIONS IN PREGNANCY Pregnant patients should all be brought up-to-date with regard to the Td and influenza vaccines. Pneumococcal, HAV, and HBV vaccinations are optional. The MMR and VZV vaccinations are contraindicated in pregnancy due to the risk of harm to the fetus.

OTHER SPECIAL GROUPS Patients with diabetes, heart disease, lung disease, and liver disease should all be given the Td, influenza, and pneumococcal vaccinations. The remaining vaccinations should be given as indicated. In immunodeficient patients, those with an immunologic disease such as lymphoma or leukemia, cancer patients, CSF leaks, or in patients with cochlear implants, the MMR and VZV vaccines are contraindicated. Patients with end stage renal disease and HIV-positive patients should be given the Td, influenza, pneumococcal, and HBV vaccinations. In patients with HIV, the VZV vaccine is contraindicated.

www.ClinicalReview.com


BIOSTATISTICS AND EPIDEMIOLOGY

49


50

USMLE STEP 2

TRAVEL VACCINATIONS Patients preparing to travel to foreign countries should start receiving counseling and be prepared for vaccinations and prophylactic regimens at least one month prior to the start of their journey. Patients traveling to most developing countries require vaccination for HAV due to the risk of acquiring this infection from contaminated drinking water or food. The immunoglobulin against HAV may also be given if the travel is slated to begin sooner than four weeks. HBV vaccinations are recommended for all health care workers, and also those who plan to be intimately involved with the local population of a foreign country – whether to aid the impoverished, have sexual relations, or receive medical attention. Intramuscular vaccinations for rabies should be given to all travelers to Central America and Asia. Vaccinations for typhoid fever should be given in the live attenuated form to all travelers to developing countries, due to the risk of infection from contaminated food and water. The polysaccharide version should be given to HIV positive patients. A polio vaccine should be given to all travelers to developing countries. Finally, the meningitis vaccine with the polysaccharide version should be given to all individuals traveling to Africa, northern India, and Nepal. Pilgrims going to Mecca should also be fully immunized per Saudi Arabian regulations. Patients lacking a spleen should also receive the meningococcus vaccine.

www.ClinicalReview.com


BIOSTATISTICS AND EPIDEMIOLOGY

51

PROPHYLAXIS Patients traveling to countries where malaria is an issue, such as Central America, Asia, or Africa, should be given chloroquine to reduce the risk of acquiring this disease. Certain areas have strains of malaria resistant to chloroquine, which should lead to replacing chloroquine prophylaxis with mefloquine. A third agent of choice is doxycycline. Pregnant patients should receive malaria prophylaxis with proguanil and atovaquone. Patients experiencing traveler’s diarrhea can take loperamide when fever or blood is not present with the diarrhea. Severe symptoms should be treated with TMP/SMX or fluoroquinolone. TABLE 50 IMMUNIZATION Immunization

Adult

Tetanus: Td should be given every 10 years, and should be repeated in the face of an infected wound. The two dose Hepatitis A: Should be given to all adults at risk. Hepatitis B: Should be given as the three dose series especially in susceptible individuals. Pneumococcal polysaccharide vaccine: Should be given as one dose prior to the age of 65 to those at risk, then repeated after the age of 65 to all adults. Influenza: should be given annually to all at-risk individuals prior to the age of 50, then yearly in all adults. MMR: In patients without sufficient documented infection in childhood or those who have never been infected, the MMR and VZV should be given. The polysaccharide meningococcal vaccination should be given to all at-risk individuals.

Pregnancy

Pregnant patients should all be brought up-to-date with regard to the Td and influenza vaccines. Pneumococcal, HAV, and HBV vaccinations are optional. The MMR and VZV vaccinations are contraindicated in pregnancy due to the risk of harm to the fetus.

Special Groups

Diabetes, heart disease, lung disease, and liver disease: should all be given the Td, influenza, and pneumococcal vaccinations. The remaining vaccinations should be given as indicated. Immunodeficient patients: those with an immunologic disease such as lymphoma or leukemia, cancer patients, CSF leaks, or in patients with cochlear implants, the MMR and VZV vaccines are contraindicated. ESRD and HIV-positive patients: should be given the Td, influenza, pneumococcal, and HBV vaccinations. In patients with HIV, the VZV vaccine is contraindicated.

Travel

Preparing to travel to foreign countries: should start receiving counseling and be prepared for vaccinations and prophylactic regimens at least one month prior to the start of their journey. HAV: Patients traveling to most developing countries require vaccination for HAV. The immunoglobulin against HAV may also be given if the travel is slated to begin sooner than four weeks. HBV for all health care workers, and also those who plan to be intimately involved with the local population of a foreign country – whether to aid the impoverished, have sexual relations, or receive medical attention. Rabies: IM vaccinations for rabies should be given to all travelers to Central America and Asia. Typhoid Fever: should be given in the live attenuated form to all travelers to developing countries, due to the risk of infection from contaminated food and water. The polysaccharide version should be given to HIV positive patients. Polio: should be given to all travelers to developing countries. Meningitis: vaccine with the polysaccharide version should be given to all individuals traveling to Africa, northern India, and Nepal. Pilgrims going to Mecca should also be fully immunized per Saudi Arabian regulations. Patients lacking a spleen should also receive the meningococcus vaccine.

Prophylaxis

Patients traveling to countries where malaria is an issue, such as Central America, Asia, or Africa, should be given chloroquine to reduce the risk of acquiring this disease. Certain areas have strains of malaria resistant to chloroquine, which should lead to replacing chloroquine prophylaxis with mefloquine. A third agent of choice is doxycycline. Pregnant patients should receive malaria prophylaxis with proguanil and atovaquone. Patients experiencing traveler’s diarrhea can take loperamide when fever or blood is not present with the diarrhea. Severe symptoms should be treated with TMP/SMX or fluoroquinolone.

ANATOMY OF THE IMMUNE SYSTEM ANTIBODY CLASSES


52 TYPE

IgG

USMLE STEP 2 FEATURES

Monomeric 65%

NOTES

DIFFERENCES

Most abundant immunoglobulin found throughout the body. Freely crosses the placenta and provides for maternal-fetal immunity during early neonatal period. Binds to wide variety of infective organisms, activates complement (classic pathway) and through opsonization. IgG1 is the most common subtype (IgG1 – 4). IgG2 does not bind to macrophages but is a common antibody to carbohydrates (celiac sprue). IgG3 does not play a role in rheumatoid factor antigen but strongly binds to complement compared to the other IgGs. IgG4 does not fix complement (Clq) or bind to macrophages. Most versatile is IgG1 followed by IgG3. IgG has higher affinity than IgM.

IgM

Monomeric or Pentameric 10%

Does not cross placenta. Strongly fixes complement. No passive transfer reaction capability. Early antibody response. Largest immunoglobulin. Pentamers.

Primary response to antigen that often signifies early disease. Converted to IgG to give long-lasting immunity. Antigen receptor that is also located on B cell surface. Forms pentamers. High avidity and is the first immunoglobulin to be expressed. IgM has higher avidity than IgG.

IgA

IgD

IgE

IgG can cross placenta; the others cannot. Fixes complement, but IgM is more potent. No passive transfer reaction capability. Longest half-life. Long-lasting immunity. The smallest immunoglobulin.

Monomeric or dimeric 20%

IgA1 is readily broken down by bacterial IgA protease. IgA2 is not. Forms dimers. Secreted on mucus membranes to play a role in protecting mucosal surfaces. Secretory component added during excretion from epithelial cells. Also found in breast milk, saliva, tears, GI tract, and lungs.

Protective barrier on mucosal surfaces. Dimers. Uses noncovalent bonds.

Monomeric 1%

Found on the surface of B cells and in lesser amounts in serum. Low in concentration with an unclear role to date. Coexpressed with IgM in naïve B cells. Deficiency does not appear to hinder immune function.

Mostly intravascular (like IgM). Shortest half-life. Surface receptor.

Type I hypersensitivity reaction (anaphylaxis) with a gate-keeper-like function. Induces degranulation from mast cells, basophils, and eosinophils. Very high elevations typically signify parasitic infections. Heat-labile (only one). Numerous carbohydrates found in this class. Does not activate complement.

Passive transfer reaction capability. Lowest concentration in serum. Short half-life. Anaphylaxis / gatekeeper role. Heat-labile.

Monomeric 1%

Affinity – binding strength of antigen to antibody. Avidity – number of sites to which antigen can be bound. Allotype – amino acid substitutions in constant region due to random change between individuals of a species; used for paternity testing. Idiotype – Variations in the variable region within a particular class of immunoglobulins; permits recognition of wide range of epitopes. Isotype – Specific classes of immunoglobulins including IgG, IgM, IgA, IgD, and IgE. The constant region is an example of an isotype; the variable region is an example of idiotype. www.ClinicalReview.com


BIOSTATISTICS AND EPIDEMIOLOGY

53 Antibodies work directly against viruses. They work through complement fixation (opsonization) and induction of phagocytosis or cell-lysis for larger invading organisms (bacteria, fungi, parasites). Humoral response – Bacteria, viruses are targeted via antibodies secreted by B-cells. Cell-mediated response – Viruses targeted via T-cells (cytotoxic, helper, regulatory, NK).

MHC DOMAINS TYPES

MHC-I

MHC-II

MHC-III

FUNCTION

NOTES

Antigen processing (A, B, C). Binds to CD8 (cytotoxic T-cells Æ cell-mediated pathway).

Host cell infected by bacteria, virus, or transformed. Found on all nucleated cells and platelets within the body. Transmembrane alpha chain and beta-2-microglobulin. Interacts with cytotoxic T-cells (CD8+ cells) which induce apoptosis, after pairing with antibodies and inducing humoral response. Proteins are generated in cytoplasm. Loading occurs in rough endoplasmic reticulum.

Peptide loading (DM, DP, DQA, DQB, DRA, DRB) Binds to CD4 (helper Tcells Æ cell-mediated or humoral pathway).

Macrophage or phagocytic host with bacteria, viruses, or particulate matter. Found on macrophages, dendritic cells, activated T-cells, B-cells. Alpha and beta chain with proteins digested in lysosomes, with presentation to CD4+ helper T-cells (may induce a humoral or cell-mediated immune response). Mediate organ-rejection following transplantation. Loading occurs in lysosome.

Complement, cytokines.

C2, C4, TNF-alpha, and TNF-beta function.

Human leukocyte antigen genes code for the major histocompatibility complex, and these genes are associated with various diseases (discussed further down) based on linkage disequilibrium (non-Mendelian inheritance). The antigen-presenting cells (APCs) are macrophages, dendritic cells, activated T-cells, and B-cells.

REGULATION AND ACTIVATION OF IMMUNOLOGIC PATHWAYS Step 1: Bacteria invades body and replicates and it is exposed to the immune system. Step 2: Macrophage identifies target and ingests it. Step 3: Macrophage processes various proteins from target and displays it on its cell surface via MHC II molecule. o Activates helper T-cells through IL-12. Step 4: T-cell recognizes abnormal epitope and compares it to variable regions it encounters on B-cells. o Activated through costimulation of CD4/CD8 and by APC. Both B7 and CD28 are simultaneously activated in costimulation. o Produces IL-12 to activate cell-mediated pathway OR produces IL-4 to activate humoral pathway. o Cell-mediated pathway uses TH1 cells, which secrete IFN-gamma to inhibit TH2 cell production. o Humoral pathway uses TH2 cells, which secrete IL4 and IL10 to inhibit TH1 cell production. Humoral Step 5: Matching pair is found and the T-cell activates the B-cell, which transforms into a plasma cell that produces antibodies. The TH2 cell produces IL4 and IL5 to stimulate B cell conversion to plasma cells. Humoral Step 6: B-cell expands to form a clonal population of plasma cells and begins hypersecretion of a clonal population of antibodies against that epitope. Cell Step 5: TH1 cell is formed which secretes IFN-gamma and IL-2. Cell Step 6: Macrophages are activated and cytotoxic T-cells are induced to launch thecell-mediated pathway. o Endogenous activation of cytotoxic T cells is also possible through IL-4 production by cells. CD4 binds to MHC II. CD8 binds to MHC I.


54

USMLE STEP 2

CELLS OF THE IMMUNE SYSTEM SURFACE PROTEINS

FUNCTION

CD14, MHC II

Phagocytosis, antigen presentation to helper T cell (MHC II), cellmediated immunity. Form granulomas (histiocytosis X, TB, chronic granulomatous disease, sarcoidosis, rheumatic fever (Aschoff bodies), gummas in syphilis. Role in atherosclerosis (progressive plaque formation). Reservoir of HIV in HIV/AIDS.

B-cell

CD19, CD20 IgM

Humoral immune system activation. Plasma B-cells – secrete large amounts of antibody and form a clonal population. Memory B-cells – long-lasting humoral immunity with ability to rapidly form a clonal population when reactivated.

Tc-cell

CD3, CD8 TCR

Destruction of infected cells after activation by macrophages or other APCs.

Cytotoxic T-cells.

TH1-cell

CD3, CD4, CD28 TCR

Activate cell-mediated response through secretion of IFN-gamma and IL2.

Induced by IL12.

TH2-cell

CD3, CD4, CD28 TCR

Activate humoral response through secretion of IL4, IL5, and IL10.

Induced by IL4.

Regulatory Tcells

CD4, CD25

Suppress immune system activation and help avoid autoimmune diseases.

NK cell

CD16, MHC I

Major role in killing cancerous cells.

CD80, CD86

Strong T-cell activators through antigen-presentation (MHC II). Role in allergies and autoimmune disease.

MHC I

Includes all nucleated cells and platelets

CELL TYPE

Macrophage

Dendritic cells

Somatic cells

NOTES Liver – Kupffer cells. Skin – Langerhans . cells. Bone – Osteoclasts. Brain – Microglia. Spleen – Sinusoidal.

All cells in body except RBCs.

www.ClinicalReview.com


BIOSTATISTICS AND EPIDEMIOLOGY

55

CYTOKINES AND GROWTH FACTORS INTERLEUKINS TYPE

FEATURES

NOTES

IL-1

Secreted by macrophages and activates acute phase reaction.

Increases body temperature (resets hypothalamus temperature regulation) and increases adhesion factor expression by cells (promotes extravasation of immune mediators). IL-1 receptor antagonist is used for the treatment of RA. Pyrogen.

IL-2

Secreted by T-cells and stimulates Tcell response.

Used as adjunctive therapy in cancer, especially malignant melanoma and renal cell carcinoma. Activates regulatory T-cells to moderate reaction to self. Stimulates B-cells.

IL-3

Secreted by T-cells to stimulate bone marrow stem cells.

Chromosome 5, similar function as GM-CSF (see below). Myeloid role.

IL-4

Secreted by TH2 cells to induce B-cell proliferation.

Mediator of allergic reactions. Regulates humoral and adaptive immunity and promotes class-switching to IgE (and so plays a role in allergies and anaphylaxis).

IL-5

Secreted by TH2 cells to induce B-cell stimulation and production of IgA.

Mediates eosinophil activation as well.

Secreted by macrophage and activates acute phase reaction.

Very high levels following trauma, burns, and tissue damage. Stimulates osteoblasts to stimulate osteoclasts (almost all pathways are indirect like this). Estrogen inhibits IL-6 and so can theoretically be used for the treatment of osteoporosis. Induces fever. Works through gp130. Pyrogen.

IL-7

Involved in survival of lymphocytes and NK cells.

Stimulates lymphoid progenitors. Important role in immune reaction.

IL-8

Secreted by macrophages to serve as a chemotactic factor for neutrophils.

Higher risk of schizophrenia with bad prognosis in offspring of pregnant women if in high levels. Induces neutrophils to enter surrounding tissue.

IL-9

Stimulates mast cells.

IL-6

IL-10

IL-11 IL-12

Inhibits TH1 cytokine production.

Stimulates TH2 / B-cell pathway. Anti-inflammatory function to shutdown immune system (IL-2, IL-3, IFN-gamma, TNF-alpha, GM-CSF antagonist). Expressed by TH2 cells and some mast cells. Chromosome 1.

Part of the acute phase reaction. Stimulates NK cells.

Antiangiogenic activity to help suppress carcinogenic effects. Worsens autoimmune disease (stimulates TH1 response).

IL-13

Stimulates B-cell growth and development. Inhibits TH1 cells.

Stimulates class switching to IgG1 and to IgE.

IL-17

Induces cytokine production to accelerate inflammation.

IL-18

Induces IFN-gamma production.

Stimulates macrophages.

Autocrine reaction – Cytokine activates the cell that secretes the cytokine. Paracrine reaction – Cytokine works in the region of the cell that secretes the cytokine (local effects). Endocrine reaction – Cytokine works as a hormone that is carried by blood throughout the body (systemwide effects). Pleiotropy – the ability work on multiple cell types (cytokines are pleiotropic). Interleukin – a cytokine that works predominantly on white blood cells. Chemokine – a cytokine that works predominantly to attract other cells.


USMLE STEP 2

56

CHEMOKINES TYPE

FEATURES

CC chemokines

CXC chemokines

NOTES

Cysteines are adjacent

Bind to CCR receptors. Attract monocytes, T cells, granulocytes, and NK cells (RANTES, others).

Amino acid located between two cysteines

Bind to CXCR receptors CXCR receptors have a glutamic acid-leucine-arginine binding motif Attract neutrophils (IL-8, IFN-gamma, APRs).

Cytokines that induce inflammation and work as chemotactic agents. Released during inflammation, infection, and autoimmune disorders to attract neutrophils, monocytes, and leukocytes. Numerous factors work through the above CC and CXC motifs to function as chemokines. CXCR4 mediates fusion of HIV into T-cells. CCR5 mediates entry of HIV into macrophages. Lacking CCR5 reduces the risk of HIV infection but increases risk of infection by West Nile virus.

GROWTH FACTORS AND ACUTE PHASE REACTANTS TYPE GM-CSF

FEATURES

NOTES

Stimulates granulocyte proliferation and macrophage proliferation.

Granulocyte macrophage colony-stimulating factor. Synthetic form used to stimulate macrophage population following chemotherapy.

G-CSF

Produced by endothelium and macrophages to induce WBC production.

Granulocyte colony-stimulating factor. Recombinant form used for recovery following chemotherapy-induced neutropenia (filgrastim).

RANTES

T-cell, granulocyte chemotaxis (CC) and suppresses HIV.

Regulation upon activation, normal T-cell expressed, and secreted.

IFNalpha

Inhibit viral protein synthesis.

Activate NK cells.

Inhibit viral protein synthesis.

Activate NK cells.

Increase MHC I and MHC II expression by cells to increase antigen-processing.

Activate cell-mediated pathway and NK cells. Inhibits humoral pathway.

TNFalpha

Major role in inflammation and acute phase response. Released by WBCs during infection / inflammation and endothelium due to damage. Suppresses appetite, resets thermostat, and increases APRs by liver.

Also known as cachexin. Synthetic forms used for the treatment of autoimmune disorders but increased risk of TB or activation of latent infection (CMV, EBV, VZV).

TNF-beta

LT-alpha; plays nonspecific role in several viral illnesses.

LTA4

Leukotriene precursor formed by 5-LOX.

Derived from phospholipase A2.

LTB4

Neutrophil chemotaxis.

Alternative splicing of LTA4.

LTC4

Venoconstriction, bronchoconstriction, increased vascular permeability.

Alternative splicing of LTA4.

LTD4

Venoconstriction, bronchoconstriction, increased vascular permeability.

Formed by LTC4.

LTE4

Venoconstriction, bronchoconstriction, increased vascular permeability.

Formed by LTD4.

LTF4

Venoconstriction, bronchoconstriction, increased vascular permeability.

Formed by LTE4.

IFN-beta IFNgamma

www.ClinicalReview.com


BIOSTATISTICS AND EPIDEMIOLOGY

57

Granulocytes include neutrophils, eosinophils, and basophils. Adjuvant – nonspecific activator of the immune system that creates a strong immune response when given with a weak activator. Vaccines are given with adjuvants to promote an immune reaction against the components of the vaccine (immune system irritant). Anergy – lack of costimulation of T-cells leads to inactivation of otherwise selfreactive cells. Method of developing tolerance. The acute phase response leads to antibody production, a cytotoxic response, phagocytosis, complement activation, leukocytosis, fever, increased antigen processing, decreased viral or bacterial replication, and developing of adaptive immune response. Antigen drift – small mutations; medications still work. Antigen shift – large mutations leading to epidemics; medications no longer work. Active immunity – from attenuated live viruses (best), killed vaccinations, or actual infection with long-lasting immunity and numerous B-cells. Passive immunity – mother to baby (IgG), serum transfer (i.e. rabies Rx) with rapid onset but short duration. Zileuton – blocks 5-LOX inhibiting all leukotrienes. Zafirlukast and montelukast – inhibit cysteinyl leukotrienes (all but LTB4).

COMPLEMENT TYPE

FUNCTION

NOTES

C1

Binds to antigen-antibody complex.

Opsonization; inhibited by C1EI.

C2

Binds to antigen-antibody complex / activated.

Opsonization.

C3

Broken down into C3a and C3b / activated by C2/C4 or C3b,Bb.

Opsonization.

C3a

Weak neutrophil chemotaxis, anaphylaxis reaction / inflammation.

Anaphylaxis.

C3b

Combines with C2b, C3a and C4b to form C5 convertase (classic pathway) or combines with C3b, Bb, C3a to form C5 convertase.

Opsonization.

Bb

Alternative pathway.

C4

Binds to antigen-antibody complex / activated.

C5

Broken down into C5a and C5b.

C5a

Strong neutrophil chemotactic agent.

Anaphylaxis and neutrophil chemotaxis (strong).

C5b

Membrane attack complex.

Important in Neisseria infection.

C6

Membrane attack complex.

Important in Neisseria infection.

C7

Membrane attack complex.

Important in Neisseria infection.

C8

Membrane attack complex.

Important in Neisseria infection.

C9

Membrane attack complex.

Important in Neisseria infection.

DAF

Found on surface of cells to prevent complement-deposition and activation.

Decay-accelerating factor.

C1EI

Inactivates complement to prevent complement activation and cell destruction.

C1 esterase inhibitor.

Opsonization.

The alternative pathway provides a means of activating the immune system without antibodies. C3b is critical in both classic and alternative pathways.


USMLE STEP 2 58 The classic pathway is activated by IgM or IgG. Toxins, IgA, and spontaneous activation occurs in the alternative pathway.

IMMUNOLOGIC DISORDERS B-CELL DISEASES DISEASE

PATHOPHYSIOLOGY

PRESENTATION

Bruton’s agammaglobulinemia

XLR Missing Bruton tyrosine kinase leading to inability of B-cell proliferation or differentiation. Few CD19+ B-cells. No plasma cells, failure to manufacture antibodies, poorly developed germinal centers.

Sepsis, meningitis, fatal diarrhea (Giardia, C. jejuni, enterovirus, poliovirus), HSV, arthritis, autoimmune hemolytic anemia, idiopathic thrombocytopenia, dermatomyositis.

Selective immunoglobulin deficiency

Inability to switch isotypes with lack of IgA most common; occurs mostly in young.

Sinopulmonary infections, especially in young.

Common variable immunodeficiency

Variable inheritance. Reduction in IgG, IgA levels; some with IgM reduction as well. Bcell defect, lack of PKC activation, and inability to repair DNA are all possible reasons. Immature B-cells present with limited IgM secretion ability but inability to maintain antibody secretion with infections. Some patients also have a T-cell defect that leads to inability to activate B-cells.

Recurrent staph, strep infections, autoimmune disease common, lymphoid hyperplasia, granulomas, tumors, LAD, splenomegaly. Celiac sprue. Form of Bruton’s.

Decrease in helper T-cell function leading to decreased IgG and IgA production; no B-cell defect. Normal antibody responses.

Frequent otitis media, sinusitis, bronchitis. No serious illnesses and normalization of immune function by age 3 years. Increased incidence of atopy, allergies, asthma.

Transient hypogammaglobulinemia of infancy

www.ClinicalReview.com


BIOSTATISTICS AND EPIDEMIOLOGY

59

T-CELL DISEASES DISEASE DiGeorge syndrome

Chronic mucocutaneous candidiasis

PATHOPHYSIOLOGY

PRESENTATION

AR, Chr 11, ATM gene mutation with failure of phosphatidylinositol 3-kinase signaling.

Congenital heart disease (LV outflow tract obstruction, ToF), hypoparathyroidism, absent thymus, facial hypoplasia. Cerebellar ataxia (progressive), drooling, mask-like facies, decreased reflexes, telangiectasias.

T-cell dysfunction.

Impaired action against Candida leads to frequent fungal infections and possible systemwide infections; commonly affects skin and mucous membranes. Fluconazole is the drug of choice.

MIXED B- AND T-CELL DISEASES DISEASE

PATHOPHYSIOLOGY

PRESENTATION

Severe combined immunodeficiency syndrome

XLR, Combined B- and T-cell defect presenting at 3 months of age with numerous infections. Defect with JAK-STAT pathway, in some, leading to failure to activate T-cells; others with defect in adenosine deaminase and purine nucleoside phosphorylase leading to toxic cell death in lymphocytes. No CD3, CD45, MHC II, and many others.

Recurrent serious infections, diarrhea, dermatitis, FTT. Atrophic thymus, missing lymphoid tissue. Early lifethreatening infections. Infant Doe.

Wiskott-Aldrich syndrome

XLR, defect in WASP protein (needed for antibody and platelet function) Decrease in CD8 T-cells, defect in phagocytic vacuoles, chemotaxis. Elevated IgA, normal IgE, and decreased IgM.

Bacterial URI, eczema, thrombocytopenia, CA, small platelets.

AR, Chr 11, DNA repair enzyme defect leading to IgA deficiency.

Ataxia, telangiectasias, sinupulmonary infections.

Ataxia-telangiectasia

MACROPHAGE DISEASES DISEASE

PATHOPHYSIOLOGY

PRESENTATION

Chediak-Higashi disease

AR, Inability to remove debris from phagocytic vacuoles due to microtubular defect. Absent NK cells and partial albinism common. Defects in degranulation and chemotaxis.

Pyogenic infections.

Job syndrome

AD Chr 4q; others AR, Failure of IFN-gamma production leading to inability to suppress humoral immunity when cell-mediated is more applicable. This leads to excess IgE.

Atopy, asthma, eczema, allergies, rhinorrhea.

Leukocyte adhesion deficiency syndrome

Defect in adhesion protein, CD18 that prevents leukocyte adherence at sites of infection. Failure of chemotaxis and opsonization.

Severe Staph and Strep infections (GAS).

Hyper-IgM syndrome

XLR, Absence of CD40 ligand on CD4+ T-cells and absence of CD154 leading to B-cell differentiation failure. The net result is failure of class switching in B-cells. CD19 is present along with IgM and IgD, but no proliferation occurs and germinal centers are absent.

Recurrent otitis media, pneumonia, sinusitis, diarrhea, opportunistic infections (PCP, Cryptosporidium).

IL-12 receptor deficiency

Inability to target myobacteria.

TB infections.

Myeloperoxidase defect

Defect in granule enzyme.

Asymptomatic.

Â


USMLE STEP 2

60

HYPERSENSITIVITY REACTIONS TYPE

FEATURES

NOTES

DISEASES

Type I

IgE mediated response that leads to activation of mast cells and granulocytes following initial sensitization. Net result is a degranulation of histamine, proteoglycans such as heparin, serine proteases, prostaglandin D2 (vasodilation), cytokines, and leukotriene C4.

Rapid response with immediate reaction that presents with anaphylaxis, hives, wheals, flares, urticaria, and intense pruritus.

Occurs in allergic reactions (bee stings, peanut allergies, drug allergies).

Type II

IgG and IgM mediated response that leads to direct antibody-mediated cell toxicity with cell destruction; either direct phagocytosis or complement-mediated cell death.

Direct antibody-mediated destruction of self and with activation of membrane attack complex through complement pathways (classic pathway).

Autoimmune hemolytic anemia, erythroblastosis fetalis (Rh disease), Goodpasture, rheumatic fever, Graves’, bullous pemphigoid.

Type III

Indirect IgG and IgM mediated response that leads to antibody-antigen complexes. These complexes deposit in various tissues leading to inflammation and cellular destruction.

Activation of immune system with neutrophils and macrophages leading to cell damage.

Serum sickness occurs with horse serum components in some vaccines or serum transfers leading to tissue damage. Arthus reaction occurs with local antigen injection leading to complex deposition within skin and local urticaria; occurs in fungal infections of the lung

Type IV

Delayed T-cell mediated cytotoxicity leading to direct T-cell mediated cellular destruction.

T-cell effects.

TB test, contact dermatitis.

TRANSPLANT REJECTION TYPE

FINDINGS

TREATMENT

Hyperacute

Type II reaction with preformed IgM or IgG antibodies from prior sensitization leading to antibody-mediated destruction.

Remove transplant.

Acute

Type IV reaction with cytotoxic T-cell-mediated destruction.

Cyclosporin

Chronic

Type II and III reaction with antibody-antigen deposition and direct antibody cytotoxicity.

Remove transplant.

Graft-vs.-host disease

Type IV reaction with cytotoxic T-cell-mediated destruction.

Remove transplant. www.ClinicalReview.com


BIOSTATISTICS AND EPIDEMIOLOGY

61

FINDINGS IN DISEASE HLA ASSOCIATIONS DISEASE

HLA

PRESENTATION

DR3

Anorexia, muscle weakness, fatigue, decrease in blood pressure due to insufficient aldosterone production by the kidney.

B27

AS presents with back pain, loss of vertebral mobility leading to morning stiffness, and arthropathy. Inflammatory back pain affects nearly all patients, beginning bilaterally with the sacroiliac joints and progressing superiorly. A “bamboo spine” appearance is obvious on plain films and is virtually diagnostic of AS.

Behçet's syndrome

B51

Behçet’s syndrome is an autoimmune disorder that leads to oral and genital ulcers, arthritis, uveitis, DVT, and psychiatric changes.

Birdshot chorioretinopathy

A29

Uvea inflammation with white/yellow patches, in females, with loss of color vision over time. Recurrent, relapsing, remitting, progressive.

Celiac sprue

DR3, DQ2

Celiac sprue presents like other malabsorptive syndromes because there is diarrhea and steatorrhea, bloating and abdominal pain, and weight loss. Unlike other malabsorptive syndromes, there may also be seizures and ataxia.

Chronic hepatitis (autoimmune)

DR3

Jaundice, cholestasis, portal hypertension, varices, palmar erythema, asterixis, CNS encephalopathy, Dupuytren’s contracture, pruritus, scleral icterus, darkening of urine.

Dermatitis herpetiformis

DR3

Celiac sprue may present with a pruritic rash, known as dermatitis herpetiformis.

DR2

Goodpasture disease presents with hemoptysis with pulmonary involvement, chest pain, glomerulonephritis symptoms, and arthralgia.

DR3

Graves’ disease itself may present with the triad of proptosis, exophthalmos, and pretibial myxedema.

Hashimoto’s disease

DR11

Hypothyroidism presents with asymptomatic goiter (especially in Hashimoto thyroiditis) that may lead to local impingement. Weight gain commonly occurs with concomitant lethargy and decreased energy. Cold intolerance and constipation are commonly present. Dry coarse hair is typically seen along with myxedema.

Hemochromatosis

A3

Hemochromatosis is the result of an autosomal recessive defect leading to excessive iron absorption. This large iron load precipitates in various tissues leading to bronze discoloration of the skin, hepatomegaly, diabetes, restrictive heart failure, arthropathy, and generalized organ failure.

DR3

Nephrotic syndrome is the development of severe proteinuria greater than 3 grams daily. It is due to the breakdown of the glomerular filtration barrier leading to increased protein permeability.

DR2, DR3, DRB1, DRB5, DQB1

MS presents with intermittent attacks with overall progression to disability. Some patients have mostly intermittent relapses, while others progress more continuously. After a point in the disease, the deterioration progresses more rapidly and neurodegeneration occurs. Weakness and fatigue are universal, and optic nerve dysfunction may occur leading to transient blindness. Cognitive changes occur in some, ataxia in others, along with hemiparesis, depression, and psychomotor changes. Psychiatric changes occur later in life.

Myasthenia gravis

DR3

MG presents as variable weakness worsened on exertion and improved with rest. Extraocular muscles (EOM) are weak and ptosis may be present in many patients. Facial muscle weakness is obvious on physical exam, along with weakness in the bulbar muscles, extremities, respiratory muscles, and ocular muscles.

Narcolepsy

DR15, DQB1

Narcolepsy occurs when an individual suddenly falls asleep. Hallucinations may accompany narcolepsy – if the hallucinations occur just as the person is falling asleep, they are known as hypnagogic hallucinations; those that occur just as the person is waking up are known as hypnapompic hallucinations.

Addison’s disease

Ankylosing spondylitis

Goodpasture syndrome

Graves’ disease

Membranous glomerulonephritis

Multiple sclerosis


62

USMLE STEP 2 DR4, DRw6

Mucosal lesions, cutaneous lesions, flaccid blisters filled with clear fluid. Nail dystrophy.

Postpartum thyroiditis

DR4, DQB1

Urticarial lesions that become blisters. Many forms but commonly in often used areas.

Psoriasis vulgaris

CW6

Silver scaling occurs with increased cell turnover, in part that may be due to excess T cell activity and stress occurring in patients with a genetic predisposition

B27

Reactive arthritis presents with constitutional symptoms, urethritis, conjunctivitis, and arthritis. Circinate balanitis along with asymmetric arthritis, keratoderma blennorrhagica, and other ocular lesions may be present.

DR1, DR4

RA presents with morning stiffness that resolves with activity. Three distinct joints are affected with arthritis throughout the body. Swelling is typically present in the wrist, metacarpophalangeal (MCP) or PIP joint. Arthritis tends to be generally symmetrical. Subcutaneous nodules are present over joints, and titers of RF can be demonstrated.

DR3, B8, DW3

SS presents with sicca, leading to xerophthalmia, xerostomia, blepharitis, and dyspareunia (from decreased lubrication of the vagina). Dry skin is another common complaint. Parotitis and parotid hypertrophy are commonly present. Xerotrachea leading to dry cough and dyspnea occur occasionally, leading to URI; decreased salivation and lubrication can impede normal swallowing and clearance of food.

SLE

DR2, DR3

SLE presents with numerous constitutional symptoms, especially fatigue, myalgia, and arthralgia. Fever is also present in a majority of patients. Changes in weight occur in many patients. Cutaneous manifestations include a malar rash, generalized erythema, skin lesions (such as papules or plaques) leading to central scarring and atrophy, discoid lesions, alopecia, panniculitis, nephritis (leading to renal failure) or CRF, psychiatric changes, headache, pleuritic chest pain, pleural effusions, dyspnea, pulmonary HTN, nausea, dyspepsia, dysphagia, peritonitis, pericarditis, LibmanSacks endocarditis, myocarditis, CAD, and vasculitis.

Type I DM

DR3, DR4, DQ6

Presentation of IDDM is with polyuria, polydipsia, and polyphagia with symptoms of hyperglycemia. Ketoacidosis often accompanies IDDM. Patients tend to be thin and complain of numerous constitutional symptoms. Blurred vision due to hyperosmolar infiltrate into the lens is common.

DQB1, DR2, DR4

NIDDM is typically asymptomatic and much of public health efforts are focused on early detection. Obesity is a common presentation, and pregnant women may deliver an infant that is large for gestational age (LGA). Later complications caused by NIDDM are numerous, and include a significantly elevated risk of cardiovascular disease leading to MI and CVA. Retinopathy is common and may worsen with HTN.

Pemphigus vulgaris

Reactive arthritis / Reiter’s arthritis

Rheumatoid arthritis

Sjögren syndrome / sicca syndrome

Type II DM

DISEASE ASSOCIATIONS HLA

DISEASE

A29

Birdshot chorioretinopathy.

A3

Hemochromatosis.

B27

Ankylosing spondylitis, Reactive arthritis / Reiter’s arthritis.

B51

Behçet's syndrome.

B8

Sjögren syndrome.

CW6

Psoriasis vulgaris.

DQ2

Celiac sprue.

DQ6

Type I DM.

www.ClinicalReview.com


BIOSTATISTICS AND EPIDEMIOLOGY DQB1

Multiple sclerosis, Narcolepsy, Postpartum thyroiditis, Type II DM.

DR11

Hashimoto’s disease.

DR15

Narcolepsy.

DR1

Rheumatoid arthritis.

DR2

Goodpasture syndrome, SLE, Multiple sclerosis, Type II DM.

DR3

Addison’s disease, Celiac sprue, Chronic hepatitis (autoimmune), Dermatitis herpetiformis, Grave’s disease, Membranous glomerulonephritis, Multiple sclerosis, Myasthenia gravis, SLE, Sjögren syndrome, Type I DM.

DR4

Pemphigus vulgaris, Postpartum thyroiditis, Rheumatoid arthritis, Type I DM, Type II DM.

DRB1

Multiple sclerosis.

DRB5

Multiple sclerosis.

DRW6

Pemphigus vulgaris.

DW3

Sjögren syndrome.

63

AUTOANTIBODIES TYPE

DISEASE

NOTES

c-ANCA

Wegener’s granulomatosis

Cytoplasmic

Desmoglein

Pemphigus vulgaris

dsDNA

SLE

Gliadin

Celiac sprue

IgG

Rheumatoid arthritis

IgM vs. IgG

La

SLE, Sjögren

SS-B

Mitochondrial

Primary biliary cirrhosis

AMA

nAChR

Myasthenia gravis

Neutrophil

Vasculitis

pANCA

Polyarteritis nodosa

Perinuclear

Phospholipid

Antiphospholipid syndrome

APA

RNP

SLE, MCTD

Ro

SLE, Sjögren

Smooth muscle

Autoimmune hepatitis

Thyroid peroxidase

Hashimoto thyroiditis

TSH-R

Graves disease

SS-A


USMLE STEP 2

64

Lambert-Eaton syndrome

VGCC

PRACTICE QUESTIONS A physician requests a specific test that will ensure that a positive result confirms his already high clinical suspicion. Which of the following defines specificity? A. B. C. D. E.

True negatives divided by the sum of true negatives and false negatives True negatives divided by the sum of true negatives and false positives True negatives divided by the sum of true negatives and true positives True positives divided by the sum of true positives and false negatives True positives divided by the sum of true positives and false positives

The best answer is True negatives divided by the sum of true negatives and false positives. The physician is asking for a specific test that will confirm a positive test result; the correct formula for specificity is the number of true negatives divided by the sum of true negatives and false positives.

Which of the following best describes a type II error? A. B. C. D. E.

A study that does not find a difference when one really does not exist A study that does not find a difference when one really exists A study that finds a difference when one really does not exist A study that finds a difference when one really exists Making an error even with sufficiently powered studies

The best answer is A study that does not find a difference when one really exists. Lack of power may lead to false acceptance of the null hypothesis. Being unable to therefore find a difference when one really exists leads to a type II error. Increasing power via increasing sample size can help to avoid this type of error.

What is the definition of sensitivity? A. B. C. D.

FP / (FP+FN) FP / (FP+TN) TP / (TP+FN) TP / (TP+TN)

The best answer is TP / (TP+FN). Sensitivity is the proportion of true positives to the sum of true positives and false negatives (all patients with disease). A very sensitive test has a low rate of false negative. Hence, you can trust a true negative result, indicating that if a test result is negative, the patient does not have a disease and is effectively ruled out. This permits sensitive tests to be used on a population as a screening test. Positive results are confirmed with a specific test.

www.ClinicalReview.com


BIOSTATISTICS AND EPIDEMIOLOGY

65

CHAPTER CONTENTS Theories of Development ...................................................................................66 Life Cycle .............................................................................................................68 Ego ......................................................................................................................75 Conditioning ........................................................................................................80 Developmental Disorders ...................................................................................82


66

USMLE STEP 2

NORMAL DEVELOPMENT NORMAL DEVELOPMENT THEORIES OF DEVELOPMENT Child development covers the maturation of a child from birth through her Freud splits child teenage years. There are three major theories that attempt to describe the development into the oral, various stages a child goes through as she ages. Sigmund Freud breaks down the anal, oedipal, latent, and developmental process as falling into the oral period, anal period, oedipal period, intensification of sexual latency, and intensification of sexual activity. Erik Erikson breaks the stages activity periods down into a time of basic trust versus mistrust, autonomy versus shame and doubt, initiative versus guilt, industry versus inferiority, and identity versus role confusion. Finally, Jean Piaget breaks down these stages into a period of sensorimotor development, perioperational development, preoperational development, concrete operations, and formal operations. TABLE 51 THEORIES OF DEVELOPMENT Sigmund Freud • Oral period • Anal period • Oedipal period • Latency • Intensification of sexual activity

Theories of Development Erik Erikson • Basic trust versus mistrust • Autonomy versus shame and doubt • Initiative versus guilt • Industry versus inferiority • Identity versus role confusion

Jean Piaget • Sensorimotor development • Perioperational development • Preoperational development • Concrete operations • Formal operations

FREUD’S STAGES Freud’s stages are centered on the erotogenic zones. He believes that the first Oral: 0-18 months period is the oral phase, as this is the stage in which the child derives pleasure Anal: 18 months – 3 years from suckling. If the child is able to successfully meet his needs through suckling, Oedipal: 3 – 6 years Freud says he will later be able to meet the needs of other stages. The oral stage Latent: 6 – 12 years begins at birth and continues until about 18 months of age. The anal period is Intensification: 12+ years when the child is able to exert readiness and self control through activities such as toilet training. During this period of time, the conflicts in the child’s life center on self-control versus parental demands. With too much control by the parents during this time, Freud believes that the child will become stubborn and compulsive as an adult. This period spans the time from 18 months to 3 years.

OEDIPAL PERIOD The Oedipal period is when the child develops feelings for the opposite-sex parent. The child is fearful of retaliation by the opposite sex parent at this time, and resolves this conflict by identifying with the same-sex parent. This occurs during the preschool years, from 3 years to 6 years of age. The latent period is next. During this stage the child is able to translate his sexual drive into schoolwork and play. This stage occurs during the early schooling years, from 6 to 12 years of age. Finally, Freud believes that the last period occurs during the teenage years and is a time of increased sexuality.

www.ClinicalReview.com


Normal Development

67

TABLE 52 FREUD’S STAGES Freud’s Stages Birth to 18 months

Oral period

18 months to 3 years

Anal period

3 years to 6 years

Oedipal period

6 years to 12 years

Latent period

12 years to adulthood

Intensification of sexual activity

ERIKSON’S STAGES Erikson’s stages are similar in concept to Freud’s, without the focus on human sexuality. The first period is the development of basic trust versus mistrust, and occurs from birth to 18 months. It is during this time that the child learns to rely upon his parents, especially his mother as he forms a bond through suckling and intimate care. From 18 months to 3 years, the child begins to focus on autonomy versus shame and doubt, learning to take care of tasks on his own and continuing to take on additional responsibilities. Initiative versus guilt follows in the preschool years, from age 3 to 6. This is the time that the child begins to communicate more with the outside world; positive feedback at this stage is important as the child develops language abilities.

Basic trust vs. mistrust: 0 – 18 months Autonomy vs. shame and doubt: 18 months – 3 years Initiative vs. guilt: 3 – 6 years Industry vs. inferiority: 6 – 12 years Identity vs. role confusion: 12 years +

From 6 years of age to 12, the child begins to take pride in his education and takes part in various hobbies and sports. This is the development of industry versus inferiority. The teenage years hallmark the development of identity versus role confusion as the child learns who he is and how he fits into the world. Various social, physical, and emotional strengths and weaknesses are explored during these years. TABLE 53 ERIKSON’S STAGES Erikson’s Stages Birth to 18 months

Basic trust versus mistrust

18 months to 3 years

Autonomy versus shame and doubt

3 years to 6 years

Initiative versus guilt

6 years to 12 years

Industry versus inferiority

12 years to adulthood

Identity versus role confusion

PIAGET’S STAGES Piaget hallmarks the period from birth to 18 months as the sensorimotor period. During this period the child learns basic interactions with the outside world. This includes both sensory and motor stimulation, as the child initially learns through direct interactions with her environment, followed by out of sight, then out of mind, and finally learning object permanence. The next phase, spanning 18 months to 3 years, is the perioperational phase. The child’s intuition is the main source of decision-making, not logic, during this phase. From 3 years to 6 years, the preoperational phase appears and the child believes that moving objects are alive and have feelings (animism), the child is egocentric and so believes that the world exists for her, and assigns meaning to events around her that do not have a foundation in reality (artificialism).


USMLE STEP 2 68 In the school age years, from 6 to 12, the child develops concrete operations and learns about conservatism and reversibility. The child begins to realize that water poured from a large, thin glass into a short, wide glass is still the same amount of water. Reversibility occurs when the child learns that taking 3 blocks from 10 yields 7 blocks, and adding 3 blocks to 7 gives 10 blocks. During the teenage years, formal operations are developed where the adolescent learns abstract reasoning and post conventional morality. During this stage, the teen becomes aware that exceptions to rules exist and should be exercised when ethically permissible.

Sensorimotor: 0 – 18 months Perioperational: 18 months – 3 years Preoperational: 3 – 6 years Concrete: 6 – 12 years Abstract: 12+ years

TABLE 54 PIAGET’S STAGES Piaget’s Stages Birth to 18 months

Sensorimotor period

18 months to 3 years

Perioperational period

3 years to 6 years

Preoperational period

6 years to 12 years

Concrete operations

12 years to adulthood

Abstract reasoning and post conventional morality

PROGRESSION OF STAGES It is important to realize that subsequent stages cannot proceed until the stage before it is completed successfully. For example, it is difficult to develop abstract reasoning when basic logic skills are not in place. Circumstances that prevent maturation into the next stage can include brain injury, prolonged neglect, prolonged hospitalization, and a variety of developmental disorders. There are also a number of influences on child development, including genetic factors, prenatal care, socioeconomic status, gender, temperament, parental attachment, and the quality of the parental relationships. If development proceeds without significant delay or interference, the child begins to develop basic skills by a certain age. Failure to develop these abilities leads to developmental retardation with sequelae that can continue all the way through adulthood. TABLE 55 INTERRUPTERS OF DEVELOPMENT Interrupters of Development Prolonged neglect

Prenatal care

Prolonged hospitalization

Parental attachment

Developmental disorders

Temperament

Brain trauma

Genetic factors

LIFE CYCLE THE NEONATE At the time the child is born, or during the neonatal period, there are certain innate skills. The neonate is born with excellent sensory skills, including the ability to distinguish between various tastes, mimicking a parent’s expressions, responding to loud noises, following an object with his eyes, and being comforted by a voice or being picked up. Basic reflexes present at birth include the palmar grasp reflex (the child automatically holds onto an object placed in his palm), the rooting reflex, (the child turns www.ClinicalReview.com


Normal Development

69

in the direction of stroking of his cheek), the stepping reflex ( the child moves as if he is walking when held upright); the Moro reflex (the neonate extends his limbs when startled), and the Babinski reflex (the child moves his great toe upwards and fans his toes outward when the plantar surface is stroked). The palmar grasp reflex remains until 2 months of age, the rooting reflex remains until 3 months of age, and the stepping reflex, Moro reflex, and Babinski reflex remain for about 1 year. Continuation of these reflexes several months after they should have rescinded may indicate an underlying pathology. TABLE 56 NEONATAL CHARACTERISTICS Neonatal Characteristics Excellent sensory skills

Mimicking parent’s expressions

Responding to loud noises

Following objects with eyes

Comforted by voice or holding

Palmar grasp reflex

Rooting reflex

Stepping reflex

Moro reflex

Babinski reflex

TABLE 57 DURATION OF NEONATAL REFLEXES Duration of Neonatal Reflexes Palmar grasp reflex

2 months of age

Rooting reflex

3 months of age

Stepping reflex

1 year of age

Moro reflex

1 year of age

Babinski reflex

1 year of age

INFANCY During infancy, from birth to 18 months, the infant begins to spontaneously smile. Spontaneous smiling begins at approximately 1 week and disappears by 3 months. The infant begins to smile ad lib by one month, and develop a social smile (smiling at a face) by 2 months. At three months, the infant begins to use facial expressions to show underlying emotions. Laughing begins at four months. The infant begins to exhibit stranger anxiety at 7 months and is oriented to the voice of his mother and father. Stranger anxiety typically resolves in a few months. At 15 months of age, the infant develops separation anxiety when kept from his mother and this separation may cause great distress. TABLE 58 INFANT CHARACTERISTICS Infant Characteristics Stranger anxiety

Separation anxiety

Roll over

Crawl

Sit unsupported

Stand without support

Spontaneous smile

Laughing

Babbling echolalia

Walks independently

Â


70

USMLE STEP 2

TABLE 59 DEVELOPMENT OF INFANT SKILLS Development of Infant Skills Spontaneous smile

1 week to 3 months

Social smile

By 2 months

Facial expressions

By 3 months

Laughing

By 4 months

Stranger anxiety

7 months to 12 months

Separation anxiety

15 months to 18 months

MOTOR DEVELOPMENT Motor development progresses to the point where the infant is able to roll over at 3 months. Around 6 months, the infant begins to crawl, and at 7 months, the infant is able to sit unsupported. At one year of age, the infant is able to stand without support, can walk if given assistance, and says his first word. By 15 months, the child is able to walk independently. Language development begins at 3 months with babbling. The first words are usually echoes of what is spoken to the infant (echolalia). It is no wonder that the first words are typically something repeatedly said to the infant, such as mama or papa. TABLE 60 INFANT MOTOR SKILLS Infant Motor Skills Roll over

3 months

Crawl

6 months

Sit unsupported

7 months

Stand without support

12 months

Walk independently

15 months

COUNSELING BY THE PHYSICIAN During visits to the physician, guidance to the parents should center on childproofing the house, because the infant begins to move from place to place and is now at risk of numerous environmental dangers. The infant should focus on one major task at a time – for example, the infant should not walk and eat at the same time due to the concentration required in either task. Recall that during infancy, the child is going through the oral period (Freud), basic trust versus mistrust (Erikson), and sensorimotor development (Piaget).

THE TODDLER LANGUAGE DEVELOPMENT From 18 months to 3 years is the toddler period. During this time, the toddler is going through the anal period (Freud), autonomy versus shame and doubt (Erikson), and the perioperational phase (Piaget). Autonomy is manifest during this time by the child saying “no” to simple parental requests, even when the child would ordinarily say “yes.” Language continues to develop with simple two-word sentences at age 2, three-word sentences at age 3, four-word sentences at age 4, and five-word sentences at age www.ClinicalReview.com


Normal Development

71

5. Comprehended vocabulary extends to several hundred words at age 2, even though only several dozen words make up the spoken vocabulary. Some stuttering may occur at this early age, as the motor pathways continue to develop. TABLE 61 TODDLER LANGUAGE DEVELOPMENT Toddler Language Development Two-word sentences

2 years

Three-word sentences

3 years

Four-word sentences

4 years

Five-word sentences

5 years

Ability to understand a vocabulary of several hundred words

2 years

MOTOR SKILLS DEVELOPMENT Control over bowel movements is typically completed by age 3, and control over micturition is controlled by age 4. Toilet training should typically not be attempted until cues are given by the child, including curiosity about the bathroom, squatting to expel feces, and displeasure after urination or defecation. At 1 year of age, the child is able to stack three blocks, throw a ball, scribble on paper, run, and climb stairs using one foot at a time. The child moves to and fro from his mother, which is known as rapprochement. At age 2, the child can stack six blocks, kick a ball, undress himself, and use utensils. The child plays alongside other children (parallel play), and can name body parts. This is also the time that autonomy becomes most noticeable with frequent use of the word “No.” TABLE 62 TODDLER MOTOR SKILLS DEVELOPMENT Toddler Motor Skills Development Stack 3 blocks, throw a ball, scribble on paper, run, and climb stairs one foot at a time, rapprochement

1 year

Stack 6 blocks, kick a ball, undress self, use utensils, parallel play, use of “No”

2 years

Control over defecation

3 years

Control over micturition

4 years

COUNSELING BY THE PHYSICIAN Counseling by the physician during this time should include management of tantrums. Techniques that the parents should be educated on include ignoring the toddler’s outburst completely, shifting the child’s attention to something else, and using “time out” periods to teach the child that tantrums are unacceptable. The most significant task during the second year of life is resolving separation anxiety and alleviating the distress in the child caused by removing him from his mother. In children who are hospitalized at this age, their most significant concern is separation from their parents, which supersedes even their fear of bodily harm or pain. The child’s height at age 2 should be noted, as this is typically half her height will be as an adult.

PRESCHOOLERS DEVELOPMENT BY AGE 3


USMLE STEP 2 72 Preschool aged children (age 3 – 6) have significantly larger vocabularies, numbering in the thousands of words. Pronouns, age, gender, and name are articulated by age 3, prepositions are complete by age 4, and an understanding of the concept of time is complete by age 5, including the use of the future tense. At age 3, the child can stack 9 blocks, can ride a tricycle, use scissors, partially dress himself, and can climb stairs using alternate feet. At this point, the child is typically able to spend a portion of the day away from his mother, and can also identify colors. At this time there is no evidence that daily separation from his mother has long-term sequelae, and a good day care setting may be recommended by the physician.

DEVELOPMENT BY AGE 4 At age 4, the child is able to create simple drawings of people, button her clothes, and brush her teeth. She can hop on one foot and copy a cross. Social development continues at this age with interest in sexual identity, having nightmares, developing imaginary companions, and being overly concerned about disease and injury. Preschool-aged children typically can play cooperatively with other children, and have good self-expression.

DEVELOPMENT BY AGE 5 At age 5, the preschooler can draw a person in detail, play hopscotch, and copy a square. Freud’s Oedipal stage reaches its climax at this age, as the child begins to have a rivalry with the same-sex parent, and the child is unable to understand the meaning of death. The loss of friends, relatives, or pets is typically not understood to be permanent and he may expect them to return at any given moment. In addition to the Oedipus complex, the child is maturing through the initiative versus guilt phase (Erikson) and continuing to develop through the preoperational stage (Piaget). TABLE 63 PRESCHOOLERS LANGUAGE DEVELOPMENT Preschoolers Language Development Pronouns, age, gender, name

3 years

Prepositions

4 years

Time and use of future tense

5 years

TABLE 64 PRESCHOOLERS MOTOR DEVELOPMENT Preschoolers Motor Development Simple drawings of people, button clothes, brush teeth, hop on one foot, copy a cross, interest in sexual identity, nightmares, cooperative play with others

4 years

Draw a person in detail, play hopscotch, copy a square, do NOT understand the meaning of death

5 years

SCHOOL-AGE As school-aged children (age 6-12), they become more mature and cooperative and tend to start becoming more involved with people other than their parents. As they begin to develop role models, these children tend to identify with the same sex parent, and psychosocial issues tend to be dormant at this age. Children at this age have well-developed motor skills and a strong moral sense. She is able to tie shoelaces, ride a bicycle, copy a triangle, print letters, and learns to read. By age 6, the child understands that the death of others is permanent. Due to a well-developing superego, this is often considered the best time for elective surgery. During these years, the child is in Freud’s latent stage, Erikson’s industry versus inferiority stage, and Piaget’s concrete operations stage. www.ClinicalReview.com


Normal Development

73

TABLE 65 SCHOOL-AGE MOTOR SKILLS DEVELOPMENT School-Age Motor Skills Development Tie shoelaces, ride a bicycle, copy a triangle, print letters, learn to read, understanding of death; best time for elective surgery

6 years

SHAPE DRAWING ABILITY With regard to the ability to draw shapes, a three year old can draw a circle, a four year old can draw a cross and a rectangle, a five year old can draw a square, a six year old can draw a triangle, and a seven year old can draw a diamond. Note that one way to remember this progression is to note that circle, cross, rectangle, square, and triangle are in alphabetical order. TABLE 66 SHAPE DRAWING ABILITY Shape Drawing Ability Circle

3 years

Cross

4 years

Rectangle

4 ½ years

Square

5 years

Triangle

6 years

Diamond

7 years

TEENAGERS Teenagers continue to develop through Freud’s stage of intensified sexual activity, Erikson’s identity versus role confusion stage, and Piaget’s formal operations stage. Puberty begins in these years and is hallmarked by the onset of menarche in females, typically at the age of 11-12, ejaculation in males that occurs around age 13-14, and the heightening of both gender’s sexual drive. Physical development at this age, in addition to side effects such as acne, lead to a number of psychosocial issues for this age group. These issues often include a preoccupation with body image and gender roles. They now begin to deeply explore their sexuality and solidify their sense of identity.

SEXUAL DEVELOPMENT Homosexual experiences may occur at this age as role confusion develops. Some element of risk-taking behavior is typically present. This hazardous behavior may be minimized by using short-term consequences. Both the parents and physician should serve as mentors and role models as teenagers experiment with their sexuality. In their counseling role, birth control and education regarding sexually transmitted diseases should be discussed. The first age of sexual intercourse is typically around 16, but less than 1/3 of teenagers regularly use birth control. This has led to a serious problem with teenage pregnancy, leading to over one million United States pregnancies in this age group yearly. Of that number of pregnancies, nearly 400,000 end in abortion, and often require parental consent depending on state law. Risk factors for teenage pregnancy include divorced parents, lack of planning for the future, psychosocial issues leading to depression, and poor performance in school.


74

USMLE STEP 2

TABLE 67 TEENAGE DEVELOPMENT Teenage Development Menarche in females

11-12 years

Ejaculation in males

13-14 years

First age of sexual intercourse

Typically 16 years

Number of teenage pregnancies

Nearly 1,000,000 annually

TABLE 68 RISK FACTORS FOR TEENAGE PREGNANCY Risk Factors for Teenage Pregnancy Divorced parents

No planning for the future

Psychosocial issues with resultant depression

Poor performance in school

EARLY ADULTHOOD With early adulthood spanning the ages of 20 to 40, typical concerns include marriage and having children. The major developmental stage at this point is Erikson’s intimacy versus isolation stage. Nearly 75% of Americans have been married and with children by age 30, but only 25% of families have two parents living in the home. It is also at this age that a reappraisal of one’s life starts and the adult’s role in society is more clearly defined. TABLE 69 CONCERNS OF EARLY ADULTHOOD Concerns of Early Adulthood Marriage

Having children

Reappraisal of one’s role in life

MIDDLE AGE Middle adulthood, from 40 to 65, leads to more concern over power and authority. Continued productivity and success in life helps adults in middle age successfully complete Erikson’s generativity versus stagnation stage. Failures in this stage often lead to a change in profession, a change in lifestyle, divorce or infidelity, use of illicit drugs, and depression. This is what is often termed as a “midlife crisis,” and it is widely believed to be caused by one’s increased awareness of their mortality. This is typically punctuated by menopause in women and diminished strength, stamina, and sexual performance in both genders. However, it is important to note that hormone levels stay the same and there is no evidence of decreased “sexual drive”, per se. TABLE 70 CONCERNS OF MIDDLE AGE Concerns of Middle Age Power and authority

Continued productivity and success

Increased awareness of mortality (midlife crisis)

Diminished physical & sexual stamina

THE ELDERLY

www.ClinicalReview.com


Normal Development

75

LIFE EXPECTANCY The average life expectancy in the United States is 76 years, and the fastest growing age group in the population is adults over age 85. African American men have the lowest life expectancy at 66 years of age, followed by white men and black women at 74, and white women at 80. Physical changes at this age make independent living more difficult due to decreased vision and hearing, incontinence, diminished immune responses, a general decrease in body system functioning and reserve, diminished strength, and decreased bone mass. Decreased cerebral blood flow and age-related mental function changes lead to some memory lapses, but any change in intelligence should be considered pathological. There are also changes in sleep patterns leading to a loss of sleep and decreased quality. TABLE 71 ADULT DEVELOPMENT Adult Development Early adulthood

Marriage, children and reappraisal of goals

Middle age

Concern over power and authority, possible mid-life crisis

Elderly

Diminished physical capacities, pride over accomplishments. Physical and social activity important.

EGO EGO INTEGRITY VS. DESPAIR Erikson’s ego integrity versus despair stage is dominant at this age, and pride over one’s accomplishments at this age typically prevents depression, anxiety, and worsening of any underlying abuse of alcohol or drugs. The combination of continued physical and occupational activity, in addition to higher education, a strong social support system, and a beneficial family history are all contributors to increased longevity. TABLE 72 CONCERNS OF THE ELDERLY Concerns of the Elderly Decrease in mental function

Depression

Loss of independence

General decline in health

TABLE 73 ERIKSON’S STAGES Erikson’s Stages Birth to 18 months

Basic trust versus mistrust

18 months to 3 years

Autonomy versus shame and doubt

3 years to 6 years

Initiative versus guilt

6 years to 12 years

Industry versus inferiority

12 years to 20 years

Identity versus role confusion

20 years to 40 years

Intimacy versus isolation

40 years to 65 years

Generativity versus stagnation

65 years +

Ego integrity versus despair


76

USMLE STEP 2

EGO DEFENSES Ego defenses are techniques that people use to cope with stresses presented to them by their environment. These defenses are unconscious reactions to this stress. Over time, a person’s ego defenses typically mature as a function of age and experience. There are four mature ego defenses, including altruism, humor, sublimation, and suppression. There are a number of immature ego defenses, including acting out, blocking, denial, displacement, dissociation, fixation, identification, isolation of affect, passiveaggressive behavior, projection, rationalization, reaction formation, regression, repression, somatization, splitting, and undoing. TABLE 74 EGO DEFENSES Mature Ego Defenses

Immature Ego Defenses

Sublimation

Acting out

Projection

Altruism

Blocking

Rationalization

Suppression

Denial

Reaction formation

Humor

Displacement

Regression

Dissociation

Repression

Fixation

Somatization

Identification

Splitting

Isolation of affect

Transference

Passive-aggressive

Undoing

MATURE EGO DEFENSES ALTRUISM Altruism is a mature ego defense in which the person gives to others without expecting anything in return. Altruistic behavior is typically preformed to alleviate guilty feelings. Large donations during the holidays by anonymous individuals may be done to reduce guilty feelings regarding the amount of wealth they have (“share the wealth”).

HUMOR Humor is a mature ego defense that individuals use to reduce anxiety in a difficult situation. Making jokes helps to alleviate tension in an otherwise intractable situation; for example, humor is especially common in anatomy labs where it is sometimes the only way to deal with the macabre nature of what medical students have to confront.

SUBLIMATION Sublimation is the replacement of thoughts or actions that are unacceptable with a more desirable action. The new action is consistent with the person’s ethics and morals, whereas the previous action was more impulsive and less responsible. Art and literature are prime examples of sublimation, where one artist’s feelings may be transformed into an attractive painting.

www.ClinicalReview.com


Normal Development

77

SUPPRESSION Suppression is a voluntary ego mechanism and so is different from the other mature ego defenses in that it is not automatic. Suppression involves temporarily removing a thought from the consciousness in order to continue to cope with the outside world. A mature application of this is to temporarily stop thinking about various appointments that one must go to until the day after your wedding (“take it one step at a time”). Suppression should be differentiated from repression in that removing a thought from the consciousness is temporary in suppression; in repression, the thought is removed from memory and the person forgets that the thought was removed. Repression is an immature ego defense. TABLE 75 MATURE EGO DEFENSES Mature Ego Defenses Sublimation

Replacement of thoughts or actions that are unacceptable with a more desirable action

Altruism

Giving to others without expecting anything in return, typically to alleviate guilty feelings

Suppression

Temporarily removing a thought from the consciousness

Humor

Making jokes to reduce anxiety in a difficult situation

IMMATURE EGO DEFENSES ACTING OUT Immature ego defenses deal with the outside world in ways that only forestall dealing with a difficult situation. They typically do not lead to resolution of events. Acting out is an example of an immature ego defense in which a person will express undesirable feelings with equally undesirable actions. An example of acting out is throwing a temper tantrum. In this behavior a child’s frustration with a parent may lead to an immature emotional outburst. Acting out covers the real emotion (the child’s frustration) and so is different than displacement.

BLOCKING Blocking is an immature defense mechanism in which a person will transiently stop thinking about a particular thought. It is different than suppression in that the conflict is never resolved, and the person continues to stop thinking about the thought whenever it occurs. For example, a homosexual male embarrassed about being attracted to other men may block his libidinous thoughts instead of coming to terms with his sexual orientation.

DENIAL Denial is an immature ego defense in which the person contradicts a fact to avoid pain. Denial is typically the first stage in dealing with death in which a person will believe that the results of accurate medical tests are false. Denial can best be treated by explaining the situation in terms that the patient can comprehend and helping them become aware that there may be a more responsible or mature method of dealing with the situation at hand (“you can’t argue with the facts”).

DISPLACEMENT Displacement is an immature way to transfer unacceptable feelings towards one person onto another person. A person who has a bad day at work and yells at his wife when he comes home is displacing his negative feelings towards his boss onto his wife. There is no change in the unacceptable feelings – fear in one situation is expressed as fear in another situation; anger with anger.


USMLE STEP 2 78 Dealing with displacement may be helping the person to realize where the underlying feelings are coming from and helping them to deal with the adverse situation in a more constructive manner (“face the truth”).

DISSOCIATION Dissociation is a method of changing one’s identity to avoid a stressful situation. In its more extreme presentation, a stressful situation may lead to dissociative identity disorder or multiple personality disorder. A person may deal with a stressful situation by packing up his things and creating an entirely different life elsewhere. More common presentations involve changing aspects of one’s personality, actions, memory, or consciousness.

FIXATION Fixation is having a preoccupation with a certain event. An obsession with the sports channel or with cooking shows on television may be an egosyntonic method (albeit immature way) of avoiding other obligations. Dealing with fixation may be done by empowering a person with other methods of dealing with their stressful obligations.

IDENTIFICATION Identification is an immature ego defense of believing that the averse actions one person has experienced makes it acceptable for the patient to employ those same actions on another. This is common in abused children who later become the abusers. Assisting a person involves helping them realize where their negative tendencies originated from, and transforming those tendencies into more positive actions that do not harm others (also known as sublimation).

ISOLATION OF AFFECT Isolation of affect results in separating one’s feelings from an idea or event. Separating one’s feelings prevents a person from being hurt while dealing with an otherwise stressful event. This may be common in cold-blooded murders, in which the person does not demonstrate remorse when conducting a distressing and despicable action. If the process is more cognitive, isolation of affect is termed intellectualization. This is common in patients with obsessive-compulsive disorder in which the patient ruminates about actions that they cannot help conducting. Schizophrenics sometimes demonstrate isolation of affect.

PASSIVE-AGGRESSIVE Passive-aggressive ego defenses are an immature method of being unconsciously hostile towards others in order to relieve negative feelings toward them. A person may demonstrate passive-aggressive behavior towards his boss by not alerting him that his vehicle is parked in a tow-away zone. Passive-aggressive behavior is common in patients with borderline personality disorder and in children. A person behaving badly towards another is an intentional act, and is by definition not a manifestation of passiveaggressiveness.

PROJECTION Projection is attributing an unacceptable thought or impulse as being due to another source. For example, a person who is angry with another person may believe that others are angry towards him. Another example is where a man who hates dogs believes that dogs hate him when they urinate on his property. Projection is especially common in patients with paranoid delusions (“everyone is against me”). The belief that the world exists for the individual, for example, the sports fan that believes that the event is being held for her benefit, is known as introjection. Projection is often tested on the USMLE. www.ClinicalReview.com


Normal Development

79

RATIONALIZATION Rationalization is a method to relieve guilt and shame by creating a faulty logical reasoning for why the action is somehow beneficial. For example, spending money on toys instead of spending it to provide food and shelter can be rationalized by believing that one needs the toys for her survival. Rationalization is the method that obsessive-compulsives use to alleviate their remorse over their actions and thoughts.

REACTION FORMATION Reaction formation is an immature method of taking a negative impulse and doing the opposite action. A person who wants to destroy another person’s career but instead takes steps to help them gain a promotion is engaging in reaction formation. It is an immature ego defense since the underlying negative impulse is never resolved. Reaction formation may underlie the behavior of long-time hostages when they illogically take steps to protect their abuser.

REGRESSION Regression is common in stressed children, such as those hospitalized or confronted with the birth of a new sibling. Regression is returning to a previous level of functioning, such as suddenly becoming incontinent after years of successful toilet training. Regression also occurs in severely ill adults, who suddenly require all of their needs to be taken care of when they are capable of being somewhat independent.

REPRESSION Repression, as discussed above, is an immature ego defense whereby the person takes an untenable idea, forgets it, and then forgets that they forgot it. Repressed memories are common in sexually abused patients who attempt to move on with their lives by forgetting all details of the event. Repression is an immature mechanism because stimuli later in life may lead to a sudden and dramatic recall of events leading to sudden collapse of the ego. One way to deal with repression is to gradually bring memories to the forefront in an egosyntonic method, best resolved by creating a permissible and egosyntonic atmosphere.

SOMATIZATION Somatization is an immature method of having physical symptoms to escape an otherwise stressful event. For example, developing a headache to avoid a disturbing conversation is an immature ego defense. Somatization is common in somatoform disorder.

SPLITTING Splitting is the belief that things are either one way or the opposite way. There is no middle ground. For example, the belief that all things are either good or evil is splitting (“black of white, no shades of gray”). Splitting is especially common in borderline personality disorder, and in people with prejudices and stereotypes.

UNDOING AND TRANSFERANCE Undoing is an attempt to prevent an unacceptable impulse or event from occurring without taking any concrete and logical actions. An example of this is knocking on wood after saying that one has never been in a car accident in an attempt to magically avoid future car accidents. Transference is projection of one’s feelings, due to ideas or events in their life, onto another person. Patients often transfer their anger with a disease onto the physician. In this case, the physician may also be guilty of


USMLE STEP 2 80 countertransference. In this situation the physician’s resentment of the patient being angry with him and causes him to become angry with his patient. TABLE 76 IMMATURE EGO DEFENSES Immature Ego Defenses Acting out

Expressing undesirable feelings with equally undesirable actions such as a temper tantrum

Blocking

Transiently stop thinking about a particular thought

Denial

Contradicting a fact to avoid pain

Displacement

Transferring unacceptable feelings towards one person onto another person

Dissociation

Changing one’s identity to avoid a stressful situation

Fixation

Preoccupation with a certain event such as sports

Identification

Belief that adverse actions others did to them makes it acceptable to take those actions on another

Isolation of affect

Separating one’s feelings from an idea or event

Passive-aggressive

Unconsciously hostile towards others in order to relieve negative feelings toward them

Projection

Attributing an unacceptable thought or impulse as being due to another source

Rationalization

Relieve guilt and shame by creating a faulty logical reasoning for why the action is somehow beneficial

Reaction formation

Taking a negative impulse and doing the opposite action

Regression

Returning to a previous level of functioning

Repression

Forgetting an untenable idea and forgetting that the idea was forgotten

Somatization

Having physical symptoms to escape an otherwise stressful event

Splitting

Belief that things are either one way or the opposite way

Transference

Projection of one’s feelings, due to ideas or events in their life, onto another person

Undoing

Attempt to prevent an unacceptable impulse or event from occurring without taking any concrete and logical actions

CONDITIONING INTRODUCTION Conditioning is a topic that is worth a careful but brief review for the USMLE. The topics are easily tested and it is important not to confuse the various types of conditioning. Reinforcement schedules are confusing to remember, but it is important to have a clear understanding of this so you can successfully answer a simple recall-type question on the boards. Knowing the basic information and being comfortable in applying it to the exam and in clinical practice is vital.

CLASSICAL CONDITIONING Conditioning pairs a stimulus with a response. The unconditioned response is the automatic response we make to a certain stimulus. For example, we salivate when presented with a tasty meal. Salivating is the unconditioned response to the meal, which acts as the stimulus. Since we are biologically programmed to respond to food, food is the unconditioned stimulus – unconditioned because it does not require any learning on our part. Pavlov conducted these experiments in dogs, where he www.ClinicalReview.com


Normal Development

81 realized that if he paired ringing the bell with providing the dogs food that they would begin to associate the ringing of the bell with the unconditioned response, salivation. Eventually, he was able to simply ring the bell and cause the dogs to salivate. The ringing of the bell became the conditioned stimulus – conditioned because it required the development of a learned behavior in the dogs, and the salivation the conditioned response – because the bell now caused the salivation (which is not a normal response) TABLE 77 CLASSICAL CONDITIONING Classical Conditioning Unconditioned response

Automatic response to a particular stimulus

Unconditioned stimulus

An item that produces an automatic response

Conditioned stimulus

An action that replaces the unconditioned stimulus to produce the conditioned response

Conditioned response

Automatic response to a conditioned stimulus

OPERANT CONDITIONING Operant conditioning is eliciting a particular response in order for the experimental subject to receive a reward. For example, giving a dog a treat when she responds to an oral command is an example of operant conditioning – operant because the reward is operating to create a particular response. Operant conditioning is different from classical conditioning because it does not involve innate responses to innate stimuli in order to cause a change. TABLE 78 OPERANT CONDITIONING Operant Conditioning Eliciting a specific response for subject to receive reward Does not involve innate response to innate stimuli

REINFORCEMENT Operant conditioning is broken into positive and negative reinforcement, and positive and negative punishment. Positive reinforcement is giving a reward once a desired action is performed. In the example above, giving the dog a treat once it obeys a command is an example of positive reinforcement – positive because something is given, reinforcement because it strengthens the chance of a particular behavior occurring. Negative reinforcement is removing something negative if a desired behavior occurs. Read this example carefully – an example of negative reinforcement is taking away the leash of a dog that obeys a command to sit (assuming the leash serves as something the dog dislikes). The dog learns to sit (reinforcement of an action), and learns that if it obeys that command, the leash will be removed (a negative stimulus). Negative reinforcement is often confused with punishment. Negative reinforcement is taking something negative away. Positive punishment is taking a negative action to suppress an undesired behavior. For example, spanking a child when they throw a tantrum is positive punishment – positive because an action is taken (spanking), and punishment because something negative is done to avoid a behavior. Negative punishment is taking away something positive in order to stop a particular behavior. An example of negative punishment is taking away a child’s favorite toy when they are misbehaving. Negative punishment is different than negative reinforcement in that negative reinforcement involves taking away something negative (the leash), while negative punishment involves taking away something positive (the toy). Punishment is typically ineffective in removing a negative


USMLE STEP 2 82 behavior as the unwanted behavior quickly returns once the punishment stops. Long lasting changes tend to occur only with reinforcement. TABLE 79 REINFORCEMENT Reinforcement Positive reinforcement

Giving a reward once a desired action is performed

Negative reinforcement

Removing something negative once the desired action is performed

Positive punishment

Taking a negative action to suppress an undesired behavior

Negative punishment

Taking away something positive to suppress an undesired behavior

REINFORCEMENT SCHEDULES Reinforcement schedules also influence the speed at which a wanted behavior is learned or an unwanted behavior extinguished. Continuous reinforcement, or providing a reward every time a particular action is desired, leads to rapid extinction of the desired action once the reward is stopped. One way to think about this is that the person learns to associate the desired action with the reward, and that there is no reason to continue the desired action once there is no reward coming. Variable ratio reinforcement has the slowest extinction once the reward is stopped; the next most successful reinforcement schedule is fixed ratio reinforcement. Slot machines demonstrate variable ratio reinforcement due to their payoff at seemingly random times. Vending machines demonstrate continuous reinforcement since they reward the user every time money is spent. These disparate reinforcement schedules explain why people get angry when vending machines malfunction, but do not get angry when they do not win in a casino on their first attempt. To complete this section, it should be noted that money is an example of a tool that provides secondary reinforcement as accumulation of wealth through hard work assists in obtaining additional rewards. Finally, studies have demonstrated that an increase in GABA is necessary to prevent learned helplessness. For example, mice may simply accept that they will be shocked if they do not pull a lever to prevent the negative stimulus. TABLE 80 REINFORCEMENT SCHEDULES Reinforcement Schedules Variable ratio reinforcement

Giving a reward at variable times that a desired action is completed

Fixed ratio reinforcement

Giving a reward at a predetermined interval that a desired action is completed

Continuous reinforcement

Providing a reward every time a desired action is done

DEVELOPMENTAL DISORDERS INTRODUCTION There are a number of disorders that take place primarily in children. These psychiatric illnesses are traditionally placed along axis II disorders and may be present from birth or develop spontaneously during the formative years. A majority of these disorders are present throughout the lifespan, and can often lead to severe psychosocial dysfunction. It should also be noted that the majority of illnesses that occur in adults can also present in children, and the astute clinician should be wary of these developments.

www.ClinicalReview.com


Normal Development

83

INTERVIEW STRATEGY Structuring interviews with children requires investigators to be more concrete with their questions. For example, the questions posed to children should be both precise and accurate – “Do you hit your younger brother when you get mad?” is preferable to “How do you react when you get angry?” It is also important to observe children in their interactions with others and to be cognizant that this younger age group often presents with comorbid mental illnesses.

PSYCHOLOGICAL TESTS Child psychiatry often makes use of various psychological tests. The most common tests of general intelligence are the StanfordBinet Intelligence Scale and the revised Wechsler Intelligence Scale for Children (WISC-R). The latter is the most commonly used test in children who attend school. The WISC-R yields a comprehensive performance score including verbal and performance scores that can be used to gauge a child’s intelligence quotient (IQ). A number of other tests also exist to measure personality traits, behavior, motor skills, perceptual skills, etc. TABLE 81 COMMONLY USED CHILD DIAGNOSTIC TESTS Commonly Used Child Diagnostic Tests Stanford-Binet Intelligence Scale

Tests for general intelligence

Wechsler Intelligence Scale for Children (WISC-R)

Used for schoolagers. Includes verbal and performance scores which can be used to gauge Intelligence Quotient (IQ)

DEVELOPMENTAL DISORDERS A number of disorders can be diagnosed in infancy, childhood, or adolescence. These include mental retardation, learning disorders, motor skills deficits, communication disorders, pervasive developmental disorders such as autism, Rett syndrome, childhood disintegrative disorder, and Asperger’s disorder, attention-deficit hyperactivity disorders (ADHD), feeding and eating disorders of infancy, tic disorders, elimination disorders, separation anxiety disorder, selective mutism, reactive attachment disorder, and stereotypic movement disorder. TABLE 82 DIAGNOSABLE DEVELOPMENTAL DISORDERS Diagnosable Developmental Disorders Mental retardation

Learning disorder

Motor skills deficits disorder

Pervasive developmental (e.g. autism)

Rett’s syndrome

Childhood disintegrative disorder

Asperger’s disorder

Attention-deficit hyperactivity disorder

Communication disorder

Reactive attachment disorder

Separation anxiety disorder

Selective mutism

Stereotypic movement disorder

Tic disorder

Elimination disorder

Feeding and eating of infancy disorder

MENTAL RETARDATION DEFINITION


USMLE STEP 2 84 Mental retardation is defined as having an intelligence quotient of less than 70. According to the WISC-R examination, IQ can be quantified as the mental age (defined by the test) divided by the chronologic age of the child, multiplied by 100. For example, if a five-year old child scores a 10 on the WISC-R, the child’s IQ is 200. A mentally retarded child would score an IQ of less than 70. A diagnosis of mild mental retardation is made if the IQ of the child is between 50 and 70 (85% of the mentally-retarded population); moderate mental retardation has an IQ between 35 and 50 (10%); severe mental retardation has an IQ between 20 and 35, and profound mental retardation has an IQ less than 20.

EPIDEMIOLOGY Mental retardation affects approximately 2% of the population, and is more common in males. Mental retardation has positive predictors, including low socioeconomic status (SES), which can lead to mild mental retardation (but is most likely a side effect of the poorer education and poor nutrition and therefore poorer performance on intelligence tests). More severe forms of mental retardation are independent of socioeconomic status.

ETIOLOGY The most common causes of mental retardation are Down syndrome (Trisomy 21), fragile X syndrome (the most common cause of heritable mental retardation), inborn errors of metabolism, maternal diabetes, substance abuse, rubella, and perinatal or early childhood injuries. Approximately one-third of all patients do not have a clear cause of the retardation. TABLE 83 COMMON CAUSES OF MENTAL RETARDATION Common Causes of Mental Retardation Down syndrome (trisomy 21)

Fragile X syndrome

Inborn errors of metabolism

Maternal diabetes

Substance abuse

Rubella

Perinatal or childhood injuries

1/3 of cases have no clear cause

PRESENTATION Mental retardation typically manifests with physical malformations, such as the moon faces of Down syndrome. Infants are eventually identified by their inability to meet developmental milestones including delayed speech, inability to care for self, and poor social skills.

DIAGNOSIS In order for mental retardation to be diagnosed, symptoms must be present prior to age 18, and the child’s IQ must be less than 70 with congruent deficits in functioning. Mental retardation has a long differential diagnosis, but some of the more common etiologies include attention deficit hyperactivity disorder (ADHD), learning disorders, depression, and schizophrenia. Tests to rule out some of these etiologies include EEGs for seizure disorder, an MRI or CT of the brain to rule out organic brain diseases, IQ testing, and a thorough medical, neurologic, and psychiatric work-up.

www.ClinicalReview.com


Normal Development

85

TABLE 84 DIAGNOSIS OF MENTAL RETARDATION Diagnosis of Mental Retardation Mild (DSM 317)

IQ 50-70

Moderate (DSM 318.0)

IQ 35-50

Severe (DSM 318.1)

IQ 20-35

Profound (DSM 318.2)

IQ less than 20

Severity unspecified (DSM 319) Differentiate with ADHD, learning disorders, depression, and schizophrenia CT or MRI to rule out organic brain disease

PROGNOSIS The majority of children with mental retardation progress through the normal developmental milestones, but at a slower pace. These children often require supplemental resources at school and occasionally a more structured environment to help them manage their resources and stay focused on task. Children with mild mental retardation can be educated so that they can read, write, and perform basic mathematical tasks. Many of these children can live with their parents and become productive citizens in society by holding a job. Children with moderate mental retardation typically can only be trained to perform basic tasks and to perform basic self-care, thereby enabling them to live in a structured group home. Children with profound mental retardation invariably require institutionalized care starting in early life. Many of these children often have a severe inborn error of metabolism such as Tay-Sach’s disease, leading to progressive decline and early death. TABLE 85 TREATMENT OF MENTAL RETARDATION Treatment of Mental Retardation Supplemental resources at school for mildly retarded Basic task training for moderately retarded Institutionalizations for profoundly retarded

LEARNING DISORDERS EPIDEMIOLOGY AND DIAGNOSIS Learning disorders are characterized by difficulties in particular types of learning, for example, writing, and mathematics. These specific learning tasks are significantly below expectations. Learning disorders tend to occur in families, and are typically due to a neurodevelopmental defects leading to a focal cerebral deficit. Learning disorders are more common in boys than girls and affect approximately 2% of school-age children. Diagnosis is typically made by fourth or fifth grade, when children are expected to be proficient in these three academic areas listed above. Children with specific learning disorders can still possess a high IQ, which sometimes makes diagnosis more difficult as these children may learn to adapt to mask their deficiencies.

PERVASIVE DEVELOPMENTAL DISORDERS DIAGNOSIS OF AUTISM

Â


USMLE STEP 2 86 Autism is the most common pervasive developmental disorder. Autism presents with dysfunction in communication, dysfunctional social interactions, and a limited range of activities and interests. Autism has a strong genetic component with incomplete penetrance, although a number of cases can be attributed to fragile X syndrome and tuberous sclerosis. Autism is rare, and presents soon after birth. It is easily noticed in infants as they fail to form a social smile, have limited facial expressions or eye contact, and have no desire to form social bonds with others as they age. The diagnosis of autism must be made before age 3. Upon a mental status examination, language dysfunction is clearly present as abnormalities in pitch, tone, rhythm, rate, and content. TABLE 86 DIAGNOSIS OF AUTISM – DSM 299.00 Diagnosis of Autism – DSM 299.00 Failure of infant to form a social smile Limited facial expression or eye contact Have no desire to form social bonds with others their age Diagnosis must be made before the age of 3 Language dysfunction. Speech has abnormal pitch, tone, rhythm, rate, and content

PRESENTATION OF AUTISM Children with autism may have a preoccupation with a rigid schedule to help structure their day. They may repetitively conduct tasks or rituals, take part in repetitive motor patterns, and are obsessed by particular items. Up to one quarter of these children have seizures, and three quarters have moderate mental retardation. Occasionally, savant abilities may be present in this group. Autism has severe morbidity and often requires special education, structured management, and care in specialized homes. TABLE 87 TREATMENT OF AUTISM Treatment of Autism Special education Structured management Sometimes care in specialized homes

RETT DISORDER AND ASPERGER SYNDROME Rett disorder is limited to females as affected males die during development while still in utero. Rett disorder spontaneously appears at age four when affected children suddenly regress and display a worsening loss of development and mental retardation. Asperger syndrome is a milder form of autism that leads to dysfunction with social relationships, but with normal intelligence. These children have no cognitive or social retardation, but continue to take part in repetitive behavior. TABLE 88 DIAGNOSIS OF RETT DISORDER-DSM 299.80 Diagnosis of Rett Disorder – DSM 299.80 Appears at age 4 at when children regress Loss of development Mental retardation

www.ClinicalReview.com


Normal Development

87

ATTENTION-DEFICIT HYPERACTIVITY DISORDER EPIDEMIOLOGY Attention-deficit hyperactivity disorder (ADHD) is characterized by the presence of hyperactivity, impulsiveness, inattention, and easy distractibility. Attention-deficit hyperactivity disorder has a strong genetic component, and is more likely to be present in families with a history of mood disorders, learning disorders, antisocial personality disorders, and substance use disorders. Children with attention-deficit hyperactivity disorder have been shown to have an abnormal sleep pattern with decreased REM latency, increased delta latency, and particular signs on EEG.

DIAGNOSIS Attention-deficit hyperactivity disorder has a prevalence of 4% and is more common in boys. The diagnosis of attention-deficit hyperactivity disorder requires symptoms before age 7 and should be present in more than one setting. Symptoms in only one setting may suggest an environmental cause. Children with ADHD disorder tend to go to sleep late, wake up early, and take part in various unmanageable activities, running around the house, and being uncontrollable. Diagnosis is made by the presence of more than six distinct symptoms for more than six months in two distinct settings. Symptoms include inattention, hyperactivity, impulsivity, difficulty following instructions or finishing tasks, easily distracted, fidgeting, interrupting others, and not waiting for their turn. TABLE 89 DIAGNOSIS OF ADHD Diagnosis of ADHD Symptoms before age 7 Present in more than one setting Sleep late wake up early Uncontrollable Diagnosis is made by the presence of more than 6 symptoms for more than 6 months, including: Inattention

Hyperactivity

Impulsivity

Inability to concentrate

Easily distracted

Fidgeting

Interrupting others

Not waiting their turn

Unable to complete tasks

Differentiate between: Oppositional defiant disorder

Mood disorders

Anxiety disorders

Substance-induced disorders

Pervasive developmental disorders

Environmental causes

Normal childhood behavior

TREATMENT Treatment of attention-deficit hyperactivity disorder is through a combination of cognitive-behavioral therapy and psychostimulants such as methylphenidate or D-amphetamine. Long-term use of these amphetamines may lead to stunted growth and weight loss, although children appear to normalize once the medication is halted. Other side effects may include

Â


USMLE STEP 2 88 unmasking of tic disorders with the psychostimulants, insomnia, irritability, and decreased appetite. Children occasionally respond well to agents that increase norepinephrine, such as bupropion or atomexetine (norepinephrine reuptake inhibitor), and also to clonidine. TABLE 90 TREATMENT OF ADHD Treatment of ADHD CBT Stimulants such as methylphenidate Buproprion, atomexetine, clonidine

CONDUCT DISORDERS EPIDEMIOLOGY AND DIAGNOSIS Conduct disorders are characterized by a pattern of violation of societal mores and others’ personal rights. Children with conduct disorder tend to be violent towards living beings, unthinkingly destroy property, lie, cheat, steal, and break laws and rules. Conduct disorder is the equivalent of antisocial personality disorder. Predictors of conduct disorder include divorce, substance abuse by the parents, physical or sexual abuse, poor parenting, and association with delinquent peers. TABLE 91 DIAGNOSIS OF CONDUCT DISORDERS Diagnosis of Conduct Disorders Aggression to people and animals Destruction of property Deceitfulness or theft Serious violations of rules

TREATMENT Treatment varies by the situation, but may require placement in a foster home, teaching the parents how to set limits, using behavior modification therapy, and other psychosocial techniques. Medications have no effect on conduct disorder, and nearly half of these children will go on to develop antisocial personality disorder as an adult. TABLE 92 TREATMENT OF CONDUCT DISORDERS Treatment of Conduct Disorders Foster care Teaching parents to set limits Behavior modification therapy

OPPOSITIONAL DEFIANT DISORDER Oppositional defiant disorder is distinct from conduct disorder. A child who does not meet the criteria for conduct disorder characterizes oppositional defiant disorder. Children with oppositional defiant disorder tend to be difficult to deal with and often www.ClinicalReview.com


Normal Development

89 take part in disruptive behavior. These children tend to be argumentative, but typically do not violate laws or the rights of others. Both oppositional defiant and conduct disorder are comorbid with attention-deficit hyperactivity disorder and various learning disorders. TABLE 93 DIAGNOSIS OF OPPOSITIONAL DEFIANT DISORDER – DSM 313.81 Diagnosis of Oppositional Defiant Disorder – DSM 313.81 Argumentative Difficult to deal with Take part in disruptive behavior

TIC DISORDERS TOURETTE DISORDER Tourette disorder is an example of a tic disorder. It presents with multiple, involuntary motor spasms or vocalizations. It is relatively rare, affects males more than females, and tends to run in families. Children with Tourette disorder also tend to have obsessive-compulsive disorder. Coprolalia, or shouting of obscenities, is sometimes present with Tourette disorder. Patients are aware of their state, and are occasionally able to control the urges, but eventually become overwhelmed by them. The differential of Tourette disorder includes Wilson disease and early onset of Huntington disease. EEGs should also be performed to rule out a seizure disorder. Management of Tourette disorder involves the use of neuroleptics such as pimozide or haloperidol, supportive psychotherapy, and education. TABLE 94 SYMPTOMS OF TOURETTE DISORDER Symptoms of Tourette Disorder OCD Coprolalia TABLE 95 DIAGNOSIS OF TOURETTE DISORDER – DSM 307.23 Diagnosis of Tourette Disorder – DSM 307.23 Tics occur many times per day Onset before age 18 Not due to medications or general medical condition TABLE 96 TREATMENT OF TOURETTE DISORDER Treatment of Conduct Disorders Pimozide or haloperidol Supportive psychotherapy Education Behavior modification therapy


90

USMLE STEP 2

SEPARATION ANXIETY DISORDER DIAGNOSIS Separation anxiety disorder is a continuation of the normal separation anxiety into the early school-age years. This is a common diagnosis in children starting all-day schooling. Separation anxiety disorder is characterized by somatization due to fear of loss of the parental figure.

www.ClinicalReview.com


Normal Development

91

CHAPTER CONTENTS Psychotic Disorders .............................................................................................92 Mood Disorders ..................................................................................................97 Anxiety Disorders ..............................................................................................101 Cognitive Disorders ...........................................................................................108 Dissociative Disorders .......................................................................................112 Somatoform Disorders......................................................................................114 Malingering and Factitious Disorders ...............................................................115 Personality Disorders ........................................................................................116 Substance Abuse Disorders ..............................................................................120 Antipsychotics ...................................................................................................135 Antidepressants ................................................................................................139 Mood Stabilizers ...............................................................................................143 Anxiolytics .........................................................................................................146 Other Medications ............................................................................................149 Major Adverse Drug Effects ..............................................................................152

Â


92

USMLE STEP 2

PSYCHIATRY PSYCHIATRY PSYCHOTIC DISORDERS PSYCHOSIS Psychosis is impairment in the ability to distinguish what is real from what is not. Examples of psychosis include hallucinations, delusions, illusions, ideas of reference, ideas or influence, and disorganization of thought. A primary psychosis is differentiated from a mood disorder that has psychotic features in that a mood disorder is fundamentally characterized by the affective disarrangement. Some components of psychosis can include ideas of influence, in which an individual believes that some outside force or entity is controlling them; ideas of reference, in which an individual believes that people on television or on the radio are speaking directly to the person; noesis, in which a person feels a revelation has been made to them and that they are a leader; and clang associations in which words are associated based on similar sounds (homonyms). TABLE 97 FEATURES OF PSYCHOSIS Features of Psychosis Clang associations

Speech disorder. Words are associated based on similar sounds

Delusions

False belief inconsistent with reality

Disorganization of thought

Disruption of organized logical thinking process

Hallucinations

Dream-like perceptions while awake, sensation of stimuli that are not present

Ideas of influence

Unrealistic impression that one has influence over others

Ideas of reference

Impression that conversation or action of others are about you

Illusions

Misinterpretation of stimuli (contrast with hallucination above)

SCHIZOPHRENIA EPIDEMIOLOGY Schizophrenia is one of the most common psychoses, affecting nearly 1% of the population. Schizophrenia is defined as a psychotic disorder with psychosocial dysfunction lasting greater than six months. Schizophrenia tends to occur in young adults, earlier in men than in women with nearly one third of women having their first psychotic episode in their 30s. Schizophrenia is more likely to be diagnosed in the indigent, and although a number of theories have been postulated, they have yet to be scientifically proven. Several studies have also indicated that schizophrenia is more prevalent in individuals with lower socioeconomic status. One hypothesis is that patients with schizophrenia are less likely to maintain steady jobs and thus fall into a lower economic standing due to chronic joblessness.

www.ClinicalReview.com


Psychiatry

93

TABLE 98 EPIDEMIOLOGY OF SCHIZOPHRENIA Epidemiology of Schizophrenia 1% incidence More common in young adults, affects young men earlier than women More likely diagnosis in the indigent Strong genetic component

ETIOLOGY Schizophrenia has a strong genetic component, but does occur spontaneously. A neurobiological basis has been postulated, known as the dopamine hypothesis. According to this theory, schizophrenia is due to an increase in activity of the dopaminergic pathways. This is substantiated by the mechanism of action of antipsychotics, which serve to reduce dopamine levels in the brain. Further, postmortem studies have demonstrated elevated levels of dopamine receptors in certain subcortical nuclei of the brain. The dopamine hypothesis also explains why cocaine and other amphetamines lead to psychosis, as these drugs are known to work through a dopamine-mediated mechanism.

PRESENTATION Schizophrenia has both positive and negative symptoms. Positive symptoms include having: disorganized, unusual type of thinking; auditory or visual hallucinations; eccentric behavior. Negative symptoms are the absence of normal mentation and psychosocial functions. For example, amotivation, isolation, and poor hygiene are all negative symptoms. It appears that certain medications are better suited for treating positive symptoms of schizophrenia, while others are better for dealing with the negative symptoms. TABLE 99 POSITIVE AND NEGATIVE SYMPTOMS OF SCHIZOPHRENIA Positive and Negative Symptoms of Schizophrenia Disorganized or bizarre thinking

Amotivation

Auditory or visual hallucinations, delusions, and illusions

Asociality

Eccentric behavior

Poor hygiene

Alogia

Flattened affect

Dysarthria (disorganized speech, speech difficult to understand)

DIAGNOSIS The formal diagnosis of schizophrenia requires the presence of two or more of the following criteria, including delusions, hallucinations, dysarthria (disorganized speech), disorganized or catatonic motor behavior, and the presence of negative symptoms. The negative symptoms can be characterized as flattened affect, alogia (lack of words), and asociality (isolation from others). Schizophrenia, as a clinical diagnosis, also requires significant alterations in one’s psychosocial or occupational function. Finally, these symptoms must be present for more than six months.

Â


94

USMLE STEP 2

TABLE 100 DIAGNOSIS OF SCHIZOPHRENIA Diagnosis of Schizophrenia Two or more of the following: catatonic behavior, delusions, disorganized speech, hallucinations, and negative symptoms Symptoms present for at least a one month period Psychosocial dysfunction Duration of at least six months Exclusion of schizoaffective disorder, mood disorders, substance-induced disorders, and general medical conditions

TREATMENT Straightforward presentations of schizophrenia are primarily managed through medical intervention. For example, neuroleptics, also known as antipsychotic agents, are used to treat both acute presentations and for maintenance. In patients with schizophrenia that is refractory to simple medical management, a combination of agents is necessary to achieve satisfactory resolution of symptoms. These agents include psychosocial interventions, such as stable reality-oriented psychotherapy, family interventions, structured environments, and possibly electroconvulsive therapy (ECT). TABLE 101 TREATMENT OF SCHIZOPHRENIA Treatment of Schizophrenia Neuroleptics Combination of agents if refractory to simple treatment Reality-oriented psychotherapy Family intervention Living in a structured environment ECT

SCHIZOPHRENIFORM DISORDER DIAGNOSIS Schizophreniform disorder meets the criteria for schizophrenia with the exception of time-constraint. Schizophreniform disorder is the name given for the psychosis with schizophrenia-type features, but with a time course less than six months but more than one month. Patients who develop schizophreniform disorder tend to go on to develop schizophrenia, although some develop a mood disorder with psychotic features instead. Schizophreniform disorder is additionally differentiated from schizophrenia in that social withdrawal is not required for this diagnosis.

www.ClinicalReview.com


Psychiatry

95

TABLE 102 DIAGNOSIS OF SCHIZOPHRENIFORM DISORDER Diagnosis of Schizophreniform Disorder – DSM 295.40 Meets the criteria for schizophrenia Episodes last for at least one month but less than six months Good prognostic features include rapid onset of dysfunctional behavior or traits, confusion during episodes, good prior psychosocial functioning, and lack of negative symptoms

BRIEF PSYCHOTIC DISORDER DIAGNOSIS A brief psychotic disorder is characterized as a short-lived, acute derangement with features similar to schizophrenia without a triggering event. Psychotic symptoms last for at least one day, but must resolve within one month with a return to normal baseline functioning. If symptoms last for more than one month a diagnosis of schizophreniform disorder and eventually schizophrenia must be considered. TABLE 103 DIAGNOSIS OF BRIEF PSYCHOTIC DISORDER Diagnosis of Brief Psychotic Disorder – DSM 298.8 Symptoms similar to schizophrenia including bizarre speech, delusions, hallucinations, and odd behavior Duration of at least one day but less than one month Return to original state of functioning Exclusion of other conditions TABLE 104 CHARACTERISTICS OF POSTPARTUM PSYCHOTIC DISORDER Characteristics of Postpartum Psychotic Disorder Characteristics of brief psychotic disorder Onset of traits within four weeks of delivery Resolves within 2-3 months

SCHIZOAFFECTIVE DISORDER DIAGNOSIS Schizoaffective disorder is the presence of schizophrenia-like features but with an overlying mood disturbance. In order for this diagnosis to be tenable, psychotic features must persist in the absence of any mood disturbance. For example, a patient must have schizophrenia-type symptoms only for a certain period of time, and then have symptoms of both a mood disorder and schizophrenia simultaneously during a different period of time.


96

USMLE STEP 2

TABLE 105 DIAGNOSIS OF SCHIZOAFFECTIVE DISORDER Diagnosis of Schizoaffective Disorder – DSM 295.70 Uninterrupted period of illness when at sometime the following symptoms are observed: Major depressive episode Manic episode Mixed episode with delusions, hallucinations, disorganized speech Negative symptoms such as alogia and affect flattening

TREATMENT Treatment of schizoaffective disorder is centered on treating the psychosis and the mood disturbance. These patients require treatment with both an antipsychotic medication and a mood stabilizing medication. Antidepressants and electroconvulsive therapy are also used as needed in the management of schizoaffective disorder. TABLE 106 TREATMENT OF SCHIZOAFFECTIVE DISORDER Treatment of Schizoaffective Disorder Antipsychotic medication Mood stabilizing medication Antidepressants, as needed ECT, as needed

DELUSIONAL DISORDER DIAGNOSIS Delusional disorder is characterized by the presence of delusions without other psychotic features. These fixed, false beliefs are nonbizarre, and are characterized by things that could happen in real life such as being followed, being poisoned, etc. TABLE 107 DIAGNOSIS OF DELUSIONAL DISORDER Diagnosis of Delusional Disorder – DSM 297.1 Nonbizarre, fixed, false beliefs Typical onset in middle age or elderly Tend to be chronic and unremitting Paranoid personality disorder is often an overlying condition in delusional disorder. Delusions in this disorder must be present for at least one month. The patient cannot meet criteria for any other mood disorder, and no social maladjustment is necessary for diagnosis.

www.ClinicalReview.com


Psychiatry

97

TREATMENT Management of delusional disorder involves psychotherapy. No attempt to support or refute the delusion should be made; rather, the therapy should focus on creating a strong alliance with the patient. With time, the alliance may aid the patient in coming to terms with his or her fixed, false beliefs. Antipsychotics have also been used in treatment, but with few positive results. TABLE 108 TREATMENT OF DELUSIONAL DISORDER Treatment of Delusional Disorder Psychotherapy Doctor should form strong alliance with patient Antipsychotics may be used, but not very effective

MOOD DISORDERS MOOD AND AFFECT Mood is defined as one’s emotional state, and differs from affect in that the latter is the external manifestation of feelings. Affect is what others see, while mood is what the individual feels inside. TABLE 109 MOOD VERSUS AFFECT Mood Versus Affect Affect

Mood

Emotions people observe

Emotion individual feels

External

Internal

SUBTYPES Mood disorders are separated into three separate categories: unipolar mood disorders including major depressive disorder (MDD) and dysthymic disorder; bipolar mood disorders including bipolar I, bipolar II, and cyclothymic disorder; and substance-induced mood disorders or mood disorders due to a general medical condition. TABLE 110 CATEGORIES OF MOOD DISORDERS Categories of Mood Disorders Unipolar (including subtypes: MDD and Dysthymic Disorder) Bipolar (including subtypes: Bipolar I, Bipolar II, and Cyclothymic Disorder) Substance-induced (or due to a general medical condition)

Â


98

USMLE STEP 2

UNIPOLAR MOOD DISORDERS EPIDEMIOLOGY Major depressive disorder is characterized by patients who have had one or more major depressive episodes. There are profound emotional changes with a concomitant changes in sleep patterns, interest in other activities, energy, and appetite. The lifetime prevalence of depression ranges from 5 to 20%, and it is twice as common in females as compared to males. Major depression is a psychiatric disorder that is not tied to a particular socioeconomic class, which is in contrast to schizophrenia. Major depression disorder primarily affects adults between twenty and forty years of age. The rate of recurrence is 60% after one episode. TABLE 111 EPIDEMIOLOGY OF MAJOR DEPRESSION Epidemiology of Major Depression One or more major depressive episodes Changes in sleep patterns, interest in activities, energy, and appetite Twice as common in females as males Not tied to socioeconomic class (contrast with schizophrenia) Affects primarily adults 20 to 40

RISK FACTORS Risk factors for depression vary, but the highest correlation is with the actual or perceived loss of a person close to the patient. Recent theories postulate that this loss leads to a sudden shift in one’s regular egosyntonic controls and positive feedback mechanisms for self-regard, leading to a cognitive distortion and subsequent negative misperception of the external environment. TABLE 112 RISK FACTORS FOR MAJOR DEPRESSION Risk Factors for Major Depression Loss of close personal acquaintance Genetic component

ETIOLOGY Depression has a strong genetic component, and has a strong concordance in monozygotic twins. A genetic defect appears to lead to a dysfunction in normal amine neurotransmitter levels, especially in the hypothalamic-pituitary-adrenal axis.

SLEEP CHANGES Clinical manifestations of major depression vary, but disturbances in sleeping patterns are nearly universal complaints. Sleep studies in depressed individuals have noted a decrease in stage 3 and 4 of sleep (delta waves). There is an increased time spent in REM sleep (beta waves), with decreased latency of the rapid eye movement (hence, faster onset of REM sleep). Other changes in major depression include having a depressed mood for the majority of the day on a regular and consistent basis, difficulty sleeping or excessive sleeping, anhedonia (lack of pleasure), feelings of worthlessness and guilt, low energy with chronic fatigue, decreased ability to concentrate on tasks, change in appetite with a resultant change in weight, psychomotor retardation or agitation, change in libido, and suicidal tendencies. www.ClinicalReview.com


Psychiatry

99

TABLE 113 SYMPTOMS OF MAJOR DEPRESSION Symptoms of Major Depression Disturbance of sleep patterns Depressed mood for the majority of the day on a regular basis Anhedonia Feelings of worthlessness and guilt Chronic fatigue Change in appetite and resultant change in weight Psychomotor retardation or agitation Change in libido Suicidal tendencies

TREATMENT Depression is best treated with psychotherapy and medical intervention. Antidepressant medications are the mainstay of medical treatment, and the best prognosis is found in patients treated with these medications coupled with psychotherapy, especially if the depression is severe. A number of classes of antidepressants are available, and it appears that tricyclic antidepressants (TCAs), selective serotonin reuptake inhibitors (SSRIs), and monoamine oxidase inhibitors (MAOIs) work well. Atypical antidepressants include thyroid hormone, psychostimulants, and lithium carbonate. All of these have all been used with positive effect. In depression refractory to medical treatment, electroconvulsive therapy has been used with positive effect. Anxiolytics may also be used as indicated, while phototherapy presents itself as an option in patients with seasonal mood disorders. TABLE 114 PHARMACOLOGICAL TREATMENT FOR MDD Pharmacological Treatment for MDD Tricyclic antidepressants (TCA) Selective serotonin uptake inhibitors (SSRI) Monoamine oxidase inhibitors (MAOI) Thyroid hormone Psychostimulants Lithium carbonate

OTHER SYMPTOMS The symptoms of depression can be remembered with the mnemonic SIG E CAPS, or sleep changes (hypersomnia or insomnia), interest (loss thereof), guilty feelings and worthlessness, energy changes (decreased), concentration deficits, appetite changes (increased or decreased), psychomotor agitation or retardation, and suicidal ideations. Depression may present with psychotic features, such as nihilistic delusions in which there are strong feelings that one’s self or others have been destroyed. Severe such delusions are termed Cotard syndrome. Such patients may complain of many bizarre losses, including various organs within their body, status, strength, respect, possessions, and so forth. Nihilistic delusions may also involve the world becoming nothingness. Cenesthetic delusions are also possible, in which there are false beliefs that things are occurring within the body.

Â


100

USMLE STEP 2

BIPOLAR I DISORDER EPIDEMIOLOGY Bipolar I disorder is the most serious variant of the three bipolar disorders (bipolar I, bipolar II, and cyclothymic disorder). Bipolar I disorder is diagnosed in patients following a single episode of mania. These patients have major depressive episodes during their lifetime. The prevalence of bipolar I is approximately 1%, with an equal preponderance in males and females.

DIAGNOSIS Bipolar I disorder has a strong relationship to genetic heritage. Triggers for bipolar I include the typical psychosocial stressors, and disturbances in the sleep / wake cycle. The formal diagnosis of this disorder also requires several of the following criteria to be met: pressured speech, flight of ideas and / or racing thoughts, distractibility, increase in goal-directed activity and / or impulsivity, excess of pleasurable activities including hypersexuality, spending excess amounts of money, decreased need for sleep with only a few hours of sleep every night, and delusions of grandeur / inflated ego. TABLE 115 DIAGNOSIS OF BIPOLAR I DISORDER Diagnosis of Bipolar I Disorder Several of the following criteria must be met: Pressured speech Flight of ideas and/or racing thoughts Distractibility Increase in goal-directed activity and/or impulsiveness Over indulgence in pleasurable activity and hypersexuality Excess spending Decrease in sleep requirements Delusions of grandeur/inflated ego

OTHER FEATURES One of the key indicators for bipolar disorder is the attempt to treat a major depressive episode with antidepressants, leading to a rapid manic episode. In these patients, since a substance is responsible for causing the manic episode, the formal diagnosis is really substance-induced mood disorder. Other psychiatric illnesses presenting with some features of bipolar disorder include schizoaffective disorder, cluster B borderline personality disorder, and depression with agitation.

TREATMENT Bipolar I disorder is best clinically managed by antipsychotics and with benzodiazepines when patients present acutely (discussed below). Other drugs commonly used with positive effect include lithium carbonate (the most commonly used mood stabilizer when renal impairment is not present), and valproic acid (also known as depakote; the intramuscular version is known as depakene). Should these first-line agents fail, carbamazepine, gabapentin, lamotrigine, and long-acting benzodiazepines can be added to the treatment regimen. Finally, bipolar I disorder refractory to medical intervention can be treated with electroconvulsive therapy. The successful treatment of bipolar I disorder requires the use of a mood stabilizer to prevent cycling www.ClinicalReview.com


Psychiatry

101 from “highs” and “lows.” Since patients with this diagnosis are often resistant to treatment, the use of psychotherapy is key in increasing compliance with medications. The first line treatment is lithium, followed by valproate, carbamazepine, and lamotrigine. TABLE 116 TREATMENT OF BIPOLAR I DISORDER Treatment of Bipolar I Disorder First Line Antipsychotics

Second Line Antipsychotics

Benzodiazepines when patients present acutely

Carbamazepine

Lithium carbonate

Gabapentin

Valproic acid (depakene if IM use, depakote)

Lamotrigine

Mood stabilizers

Psychotherapy

Electroconvulsive therapy (when antipsychotics are ineffective))

BIPOLAR II DISORDER DIAGNOSIS Bipolar II disorder is similar to bipolar I disorder except for the absence of significant mania. Patients with bipolar II have a milder form of mania, known as hypomania. These patients may still experience profound major depressive episodes. The lifetime prevalence of bipolar II disorder is half that of bipolar I disorder, ranging at approximately 0.5%. Like bipolar I, bipolar II is a cyclic disorder that varies between “highs” and “lows.” A similar proportion of these patients commit suicide as in bipolar I disorder. The treatment of bipolar II disorder is similar to that of bipolar I. TABLE 117 BIPOLAR II DISORDER Bipolar II Disorder – DSM 296.89 Similar to bipolar I but with absence of significant mania Treatment same as for Bipolar I disorder

ANXIETY DISORDERS ANXIETY DISORDERS Anxiety is the over-reactive response to an impending challenge or event that is not congruent with the actual stress level of challenge or event. Anxiety disorders are the most common type of psychiatric illness, affecting nearly 8 percent of all people.

SUBTYPES There are various types of anxiety disorders, including panic disorder with or without agoraphobia, agoraphobia, social phobia, a specific phobia, obsessive-compulsive disorder (OCD), generalized anxiety disorder (GAD), acute stress disorder, posttraumatic stress disorder (PTSD), substance-induced anxiety disorder (SIAD), anxiety disorder due to a general medical condition, and anxiety disorder not otherwise specified (NOS).


102

USMLE STEP 2

TABLE 118 ANXIETY DISORDERS Anxiety Disorders Panic disorder with agoraphobia

General anxiety disorder (GAD)

Panic disorder without agoraphobia

Acute stress disorder

Social phobia

Posttraumatic stress disorder (PTSD)

Specific phobia

Substance-induced anxiety disorder (SIAD)

Obsessive-compulsive disorder (OCD)

Anxiety disorder due to a general medical condition

Specific phobia

Anxiety disorder not otherwise specified (NOS)

PANIC DISORDER EPIDEMIOLOGY Panic disorder has a lifetime prevalence of two to three percent, and is most common in women. It typically has an onset in young adults. Agoraphobia has a similar prevalence, demographic breakdown, and onset. Patients with only agoraphobia are less likely to seek treatment; hence, most patients with agoraphobia seen in a clinical setting are likely to also have a panic disorder.

ETIOLOGY Panic disorder has many possible pathologic bases, including excessive sensitivity to carbon dioxide (CO2), dysfunction in the locus ceruleus (LC) (controls arousal), elevated catecholamine levels in the central nervous system (CNS), and dysfunction in the gamma-amino butyric acid (GABA) receptor. The latter has been thought to be a potential mechanism mediating panic attacks due to the ability of benzodiazepines, which act through a GABA pathway, to prevent future recurrence of panic disorders. The converse has also been demonstrated: some patients can be induced to have anxiety disorder with GABA antagonists. TABLE 119 TRIGGERS OF PANIC DISORDER Triggers of Panic Disorder Excessive sensitivity to carbon dioxide Dysfunction of the locus ceruleus Elevated catecholamine levels in CNS Dysfunction of GABA receptors Any stressful or fearful situation

PRESENTATION Panic disorder is characterized by recurrent, random, unexpected attacks of disabling anxiety. These patients may occasionally experience severe dread of open places, where escape from a potentially difficult situation may be difficult. Panic disorder may occur with or without agoraphobia, and agoraphobia may occur by itself.

www.ClinicalReview.com


Psychiatry

103

SYMPTOMS Panic disorder often has a trigger, and can include the presence of being in any stressful or fearful situation. Four or more of the following symptoms are required for the formal diagnosis of panic disorder and these symptoms must reach a peak within ten minutes of the stressor: palpitations or accelerated heart rate (with or without chest pain); accelerated respiratory rate, shortness of breath, or a feeling of being choked; trembling or shaking; diaphoresis; chills or hot flashes; nausea, vomiting or acute gastrointestinal distress; dizziness or lightheadedness; depersonalization (detachment from self) or derealization (feeling of artificiality); fear of losing control or going crazy; a sense of impending doom or fear of dying; and paresthesia. TABLE 120 SYMPTOMS OF PANIC DISORDER Symptoms of Panic Disorder Palpitations or accelerated heart rate (with or without chest pain)

Chills or hot flashes

Accelerated respiratory rate

Nausea

Shortness of breath

Vomiting

Sensation of being choked

Gastrointestinal distress

Trembling or shaking

Dizziness or lightheadedness

Depersonalization (detachment from self)

Derealization (feeling of artificiality)

Fear of losing control or going crazy

Sense of impending doom or fear of dying

Paresthesia

DIAGNOSIS Episodes of panic disorder occur suddenly and without warning, peak within ten minutes, and last between five and thirty minutes. These patients must experience these episodes for at least one month with significant worry about impending attacks, brooding about the sequelae of these attacks, and manifest a change in behavior in an attempt to minimize the implications of having these episodes. When panic disorder significantly affects a person’s lifestyle, it becomes a clinical disorder. TABLE 121 DIAGNOSIS OF PANIC DISORDER Diagnosis of Panic Disorder – DSM 300.01 Without Agoraphobia) 300.21 (With Agoraphobia) Episodes of panic occur suddenly

Brooding about future attacks

Attacks peak within 10 minutes

Change in behavior to prevent attacks

Attacks last for 5 to 30 minutes

Significant affect on lifestyle

The above must be experienced for at least one month

TREATMENT Panic disorder is primarily treated with a combination of medical interventions and cognitive-behavioral therapy (CBT). Certain selective serotonin reuptake inhibitors (SSRIs), tricyclic antidepressants (TCAs), monoamine oxidase inhibitors, and benzodiazepines have been shown to have positive effect. Benzodiazepines are good for immediate relief, while SSRIs or TCAs are the drug of choice for maintenance. Cognitive-behavioral therapy uses a combination of relaxation techniques and desensitization to mitigate the onset and complications of panic disorder. Teaching a patient that the episodes of panic disorder are innocuous


USMLE STEP 2 104 can often reduce the severity of this psychiatric condition. Finally, exposure therapy can be used to teach a patient fearful of open spaces (agoraphobia) that such situations do not pose a danger. TABLE 122 TREATMENT OF PANIC DISORDER Treatment of Panic Disorder Cognitive behavioral therapy (CBT) SSRIs TCAs Exposure therapy

SPECIFIC PHOBIA DIAGNOSIS The irrational fear seen in specific phobia is out of proportion with the actual danger. Diagnosis is made when the anticipation of being in a particular situation or the presence of a particular object leads to an extreme anxiety reaction and distress and with an impairment of one’s lifestyle and interpersonal relationships. Symptoms must be present for at least six months, if the onset of this disorder is in individuals younger than eighteen. TABLE 123 DIAGNOSIS OF SPECIFIC PHOBIA Diagnosis of Specific Phobia – DSM 300.29 Intense fear of a particular item or environment Extreme anxiety reaction Impairment of lifestyle and interpersonal relationships Symptoms present for more than six months Anticipation leads to anxiety

TREATMENT Specific phobias often remits spontaneously, if the onset is in childhood. Chronic specific phobias can be treated with exposure therapy, with systematic desensitization, or flooding. No medical intervention is necessary. TABLE 124 TREATMENT OF SPECIFIC PHOBIAS Treatment of Specific Phobias Often remit spontaneously, if onset in childhood Exposure therapy Systematic desensitization Flooding No pharmacologic intervention necessary

www.ClinicalReview.com


Psychiatry

105

SOCIAL PHOBIA DIAGNOSIS As in specific phobia, a social phobia may be diagnosed in patients younger than eighteen only when it has been present for greater than six months. A generalized social phobia may be diagnosed if the patient responds with an anxiety reaction to any situation, or a limited social phobia may exist if only particular triggers exist for the anxiety reaction. TABLE 125 DIAGNOSIS OF SOCIAL PHOBIA Diagnosis of Social Phobia – DSM 300.23 Only if patient over 18 years old, unless present for greater than 6 moths Generalized, if patient responds with anxiety to any situation Limited, if patient only responds to particular trigger(s)

TREATMENT Social phobia is managed with cognitive-behavioral therapy, but medication may be required for proper social functioning. SSRIs, low-dose benzodiazepines, and beta-blockers are available. Flooding and systemic desensitization are two approaches used in cognitive-behavioral therapy with good outcomes. Individual and group psychotherapeutic approaches have also been used and a combination of all of these therapies has been shown to have the best outcome. TABLE 126 TREATMENT OF SOCIAL PHOBIA Treatment of Social Phobia Cognitive behavioral therapy SSRIs Low dose benzodiazepines Beta blockers Individual and group therapy

GENERALIZED ANXIETY DISORDER DIAGNOSIS Generalized anxiety disorder is characterized by an irrational fear of virtually every aspect of common, everyday events. Generalized anxiety disorder is diagnosed only when patients are found to worry to an excessive level with high feelings of anxiety. These patients experience excessive anxiety about every facet of their life, including excessive worry about interpersonal relationships, occupational situations, personal health, and the environment around them. This level of incongruent anxiety must be present for at least six months, and the worrying must be difficult to control. The formal diagnosis of generalized anxiety disorder also requires the presence of restlessness from the anxiety, fatigue, irritability, sleep disturbances, difficulty concentrating, and tension in the musculoskeletal system. The key to this diagnosis is that the worry has to do with everyday, normal events, not the worrying associated with experiencing another psychiatric disorder or its manifestations.

Â


106

USMLE STEP 2

TABLE 127 DIAGNOSIS OF GENERAL ANXIETY DISORDER Diagnosis of General Anxiety Disorder – DSM 293.84 Patient worries to an excessive level with high feelings of anxiety Excessive anxiety over interpersonal relationships, occupational situations, personal health, environment Worry must deal with everyday, normal events Anxiety must be present for at least 6 months Worrying must be difficult to control Symptoms include: restlessness, fatigue, irritability, sleep anomalies, difficulty in concentration, muscular tension

TREATMENT Generalized anxiety disorder is best managed with a combination of cognitive-behavioral therapy with relaxation techniques, and medical intervention through the use of benzodiazepines, non-benzodiazepine anxiolytics such as buspirone, beta-blockers, and gabapentin. Benzodiazepines are effective medications, but are limited in their use due to their habit-forming nature. Benzodiazepines are recommended for the immediate relief, while SSRIs, venlafaxine, and buspirone are better for maintenance. TABLE 128 TREATMENT OF GENERAL ANXIETY DISORDER Treatment of General Anxiety Disorder – DSM 293.84 CBT with relaxation techniques Benzodiazepines Non-benzodiazepines anxiolytics such as: buspirone, beta-blockers, gabapentin Venlafaxine SSRIs

POSTTRAUMATIC STRESS DISORDER PRESENTATION Posttraumatic stress disorder is characterized as an anxiety disorder with the patient repeatedly experiencing the original traumatic event. The patient attempts to block the recollection of this traumatic event. S/he is also in a hyperaroused state. Posttraumatic stress disorder affects nearly one percent of the population, especially women, and can occur in any age group at any time after the traumatic event. Posttraumatic stress disorder has been neurologically characterized by an increase in volume of the hippocampus.

www.ClinicalReview.com


Psychiatry

107

TABLE 129 PRESENTATION OF POSTTRAUMATIC STRESS DISORDER Presentation of Posttraumatic Stress Disorder Patient attempts to block recollection of traumatic event Patient in hyperaroused state May occur in any age group after a traumatic event Causes increased volume of the hippocampus

DIAGNOSIS Patients with posttraumatic stress disorder report experiencing or witnessing a horrific and /or potentially deadly event. These patients repeatedly experience this event through thought insertions and intrusive dreams, hallucinations, illusions, or flashbacks. TABLE 130 DIAGNOSIS OF POSTTRAUMATIC STRESS DISORDER Diagnosis of Posttraumatic Stress Disorder – DSM 309.81 Experience of horrific and/or potentially deadly event Event repeatedly experienced through intrusive dreams, insertions, hallucinations, illusions, flashbacks

TREATMENT Treatment of posttraumatic stress disorder centers on the use of psychotherapy and symptom-based psychoactive medications, including antidepressants as necessary. SSRIs, buspirone, and mood stabilizers have positive effect in the treatment of PTSD. Anxiolytics such as beta-blockers, benzodiazepines, and clonidine are recommended as necessary. TABLE 131 TREATMENT OF POSTTRAUMATIC STRESS DISORDER Treatment of Posttraumatic Stress Disorder Psychotherapy

Buspirone

Symptom(s) based psychoactive medication(s)

Mood stabilizers

Antidepressants

Beta-blockers

SSRIs

Benzodiazepines

Clonidine

OBSESSIVE-COMPULSIVE DISORDER DIAGNOSIS The obsessions of this disorder include intrusive, repeated ideas or thoughts that lead to episodes of anxiety. Compulsions of this disorder are stereotyped, purposeful mental or physical rituals that the patient does in order to neutralize the obsessive ideas he

Â


USMLE STEP 2 108 or she is experiencing. Hence, a long-standing obsession can lead to compulsions in an effort to decrease anxiety and mitigate the perceived threat of not doing the action. TABLE 132 DIAGNOSIS OF OBSESSIVE COMPULSIVE DISORDER Diagnosis of Obsessive-Compulsive Disorder – DSM 300.3 Intrusive, repeated ideas or thoughts that lead to episodes of anxiety Compulsions are stereotyped purposeful mental or physical rituals

TREATMENT The management of obsessive-compulsive disorder is best done with clomipramine and selective serotonin reuptake inhibitors. Cognitive-behavioral therapy in the form of flooding, systematic desensitization, or response prevention has also been used in some patients with positive effect. TABLE 133 TREATMENT OF OBSESSIVE-COMPULSIVE DISORDER Treatment of Obsessive-Compulsive Disorder Clomipramine SSRIs CBT, especially flooding

COGNITIVE DISORDERS ILLUSIONS A clear definition of illusions, delusions, and hallucinations is important to have a good understanding of various psychiatric illnesses. An illusion is an inaccurate interpretation of an actual outside stimulus. For example, believing that a boat is sounding a foghorn to warn of an impending crash when one hears a semi truck horn is an example of an illusion.

DELUSIONS Delusions are fixed, false beliefs that exist despite evidence to the contrary. Delusions are not an appropriate term if those beliefs are consistent with the beliefs of a particular culture or society. For example, a religious person believing in God is not a delusion; however, the belief that aliens are invading Earth is a delusion. Delusions are a malformed thought where the content of the thought itself is incorrect. Delusions are different than loose associations because the thoughts are connected to each other in a disorganized way.

HALLUCINATIONS Hallucinations are sensory perceptions in the absence of any outside stimuli. Believing that werewolves are attacking one is an example of a hallucination. There are a number of types of hallucinations. Tactile hallucinations involve feeling something in the absence of touch stimuli; they are especially common in substance abuse disorders such as delirium tremens in alcohol withdrawal, in cocaine abusers, and in amphetamine psychosis (these hallucinations are known as formication). Visual and

www.ClinicalReview.com


Psychiatry

109 auditory hallucinations such as hearing voices are especially common in schizophrenia. Olfactory hallucinations may occur as part of the aura of migraines or epilepsy. TABLE 134 DELUSION VS. ILLUSION VS. HALLUCINATION Delusion vs. Illusion vs. Hallucination Delusion – False belief inconsistent with reality Illusion – Misinterpretation of outside stimuli Hallucination – Dream-like perceptions while awake, sensation of stimuli that are not present

DELIRIUM EPIDEMIOLOGY Delirium is characterized by changes in attention and cognition due to a particular medical condition or substance. The most common substance-induced causes of delirium include alcohol or benzodiazepine withdrawal and toxicity from anticholinergic drugs. Delirium predominantly affects hospitalized, post-surgical patients, especially those over 65 and in intensive care units (ICUs). Delirium is hallmarked by a disturbance of consciousness leading to deficits in attention and arousal, an alteration in cognition and memory, relatively rapid development over a period of hours, and a distinct medical condition or substance-related trigger. In delirium, the sleep-wake cycle is disturbed, and profound psychomotor agitation may be present. There may be difficulty separating delirium from dementia, especially since dementia is a positive predictor for the onset of delirium. The key differences are that delirium is reversible, develops relatively quickly, and the presence of an obvious, immediate precipitating factor. TABLE 135 DIAGNOSIS OF DELIRIUM-DSM 293.0 Diagnosis of Delirium – DSM 293.0 Withdrawal from: Alcohol Benzodiazepines Toxicity from anticholinergic drugs Phenothiazine TCAs Hospitalized post-surgical patients (especially over 65 and in ICU)

TREATMENT Delirium carries with it a 50% chance of mortality in one year. Treatment of delirium includes mostly supportive therapy while addressing the cause of the delirium. Haloperidol has been used as an antipsychotic to reduce agitation. Benzodiazepines can also be used as needed. Brightly lit rooms, plenty of cues to help orient the patient, and the presence of health care personnel and family can help reassure a delirious patient.


110

USMLE STEP 2

TABLE 136 TREATMENT OF DELIRIUM Treatment of Delirium Supportive therapy Address cause of delirium Haloperidol can be used as an antipsychotic Benzodiazepines, as needed Environment – Brightly lit room, presence of health personnel and/or family to reassure patient

DEMENTIA DEMENTIA VS. DELIRIUM Dementia is distinguished from delirium in that dementia presents over a period of weeks to years, and so is much more gradual in onset. Dementia is stable over the short term, versus the fluctuating nature of delirium. Dementia also tends to be progressive over time, while delirium tends to improve with supportive therapy. Dementia may or may not impair attention, while delirium causes severe impairments to attention. Both delirium and dementia cause cognitive impairments, but the deficits in dementia also affect executive function of the brain. Both delirium and dementia affect the sleep / wake cycle and cause a labile affect with mood disturbances. While a specific, temporally near precipitant is typically present with delirium, the precipitant for dementia tends to be more chronic and distant, if it can be identified at all. TABLE 137 DELIRIUM VS. DEMENTIA Delirium vs. Dementia Delirium

Dementia

Fluctuating nature

Stable over short term

Improves with supportive therapy

Progressive over time

Severe impairment of attention

May or may not impair attention

Cognitive impairment, executive function unaffected

Cognitive and executive functions impaired

Precipitating cause typically present

Precipitating cause chronic & distant, if identifiable

Rapid onset (hours)

Slow onset (weeks)

EPIDEMIOLOGY Dementia is the presence of memory impairment with superimposed cognitive deficits. Dementia is due to progressive neural loss secondary to an organic brain disorder, trauma, infection, infarction, hypoxia, or other biological precursor. Dementia has a prevalence of 3% after age 65, and over 20% in patients over 85. There are multiple causes of dementia, but some of the more prominent include Alzheimer disease, cerebrovascular disease, HIV, trauma to the head, Parkinson disease, Huntington disease, Pick disease, and Creutzfeldt-Jakob disease.

www.ClinicalReview.com


Psychiatry

111

ALZHEIMER DISEASE Dementia may be due to a number of etiologies. Alzheimer disease leads to a profound loss of memory affecting skills, execution of familiar tasks, difficulty with language, disorientation to time and place, loss of judgment ability, and loss of initiative. Females are more affected than males in Alzheimer, and it is the most common cause of dementia in patients over 65. Alzheimer is genetically linked to defects on chromosomes 1, 14, 19 with ApoE, and 21 with Down syndrome. Anatomic changes include flattening of the sulci, diffuse cortical atrophy, ventricular enlargement due to cortical atrophy, and decreased parietal lobe blood flow. This global atrophy leads to a decrease in choline acetyltransferase (ChAt) activity. Up to 60% of Alzheimer patients are suicidal, and nearly 15% are successful.

HIV-DEMENTIA HIV-dementia is due to viral encephalitis and myelitis. Nearly ⅔ of cases are insidious in onset, and ⅓ are florid. HIV-dementia presents with decreased memory, loss of concentration, confusion, withdrawal and apathy, dysphoria, ataxia, and muscle weakness. HIV-dementia is often diagnosed as depression, but an accurate diagnosis is critical as death often occurs within four months of onset of symptoms. Recall that death related to HIV infection is common in young adults up to middle age, and so onset of HIV-dementia is typically much earlier than Alzheimer disease.

VASCULAR DEMENTIA Vascular dementia is due to patchy loss of neurons leading to cognitive deficits. Vascular dementia may be due to intermittent vascular occlusion due to strokes or vasospasm. Onset is usually in the 60’s and so occurs earlier than Alzheimer disease but later than HIV dementia. Hypertension is a common overlying pathology and is likely responsible for the widespread, diffuse vascular disease.

AMNESTIC DISORDERS DIAGNOSIS Amnestic disorders are deficits of only memory, and are likely the result of a general medical condition or substance use disorder. Damage is common to the mammillary bodies, fornix, and hippocampus – all-important components of memory formation. This can lead to the inability to recall old memories and / or to form new memories (retrograde and anterograde amnesia). TABLE 138 DIAGNOSIS OF AMNESTIC DISORDERS – DSM 294.0 (SPECIFIC), 294.8 (NOT OTHERWISE SPECIFIED) Diagnosis of Amnestic Disorders Damage to mammillary bodies, fornix, and hippocampus Inability to recall memory (retrograde) or to form new ones (anterograde) Hypertension is a common overlying pathology Generally result from general medical condition or substance abuse


112

USMLE STEP 2

DISSOCIATIVE DISORDERS DEFINITION Dissociative disorders are the failure to integrate mental functions, leading to a loss of memory of personal identifying information, fragmentation of personality, and altered reality perception. In dissociative amnesia, a patient is unable to recall his or her identity and other personal information.

AMNESIA Various forms exist. Localized amnesia is characterized by the inability to recall personal information over a certain period of time, typically following head trauma. Selective amnesia is where particular types of information are lost. Generalized amnesia is where the information is lost for the remainder of the lifespan. In continuous amnesia, information from a particular point of time to the present cannot be recalled. Particular categories of information are lost in systematized amnesia.

GANSER SYNDROME One particular type of dissociative disorder is known as Ganser syndrome, a psychiatric illness that primarily occurs in inmates. These patients are characterized by giving approximate answers to questions, having chronic disorientation, amnesia, and perceptual abnormalities. TABLE 139 SYMPTOMS OF DISSOCIATIVE DISORDER Symptoms of Dissociative Disorder Failure to integrate mental functions Loss of memory of personal identifying information Altered reality perception TABLE 140 TYPES OF AMNESIA Types of Amnesia Dissociative amnesia

Unable to recall identity and other personal information

Localized amnesia

Inability to recall personal information over certain period of time, often due to head trauma

Selective amnesia

Particular types of information are lost

Generalized amnesia

Information is lost for the rest of lifespan

Continuous amnesia

Information from a particular point of time to the present cannot be recalled

Systematized amnesia

Particular categories of information lost

www.ClinicalReview.com


Psychiatry

113

TABLE 141 DIAGNOSIS OF DISSOCIATIVE DISORDER – DSM 300.15 Diagnosis of Dissociative Disorder – DSM 300.15 NOT 2 or more distinct personalities Derealization States of dissociation due to subjected periods of prolonged and coercive persuasion (e.g. “brainwashing”) Dissociative trance disorder Ganser syndrome – Inmates giving approximate answers to questions, having chronic disorientation, amnesia, and perceptual abnormalities. Loss of consciousness, stupor, or coma NOT due to medical condition

DISSOCIATIVE FUGUE DIAGNOSIS Dissociative fugue occurs in people who suddenly travel to a new place, create an entirely new identity and life, and continue to live as if nothing has transpired. These patients have an amnesia about their past, and have their cognitive and intellectual abilities completely intact. Dissociative fugues often remit over time without treatment or intervention. TABLE 142 DIAGNOSIS OF DISSOCIATIVE FUGUE – DSM 300.13 Diagnosis of Dissociative Fugue – DSM 300.13 People who suddenly travel to new place Creation of entirely new identity and life Amnesia about past Cognitive and intellectual abilities intact Remits over time TABLE 143 TREATMENT OF DISSOCIATIVE FUGUE Treatment of Dissociative Fugue Usually remit over time without treatment or intervention

DISSOCIATIVE IDENTITY DISORDER DIAGNOSIS Dissociative identity disorder, formerly known as multiple personality disorder, is the presence of several separate and distinct personalities that have autonomous control over a person’s behavior. Patients with dissociative identity disorder often complain of losing time, as each personality is unaware of others. Surveys of patients with this disorder indicate seven distinct personalities on average. Many of these patients are highly suggestible. Many report severe physical or sexual abuse during childhood, while


USMLE STEP 2 114 others report abuse in a cult. To complicate this issue, these memories may not necessarily be true, but the individual with the disorder perceives them as real. TABLE 144 DIAGNOSIS OF DISSOCIATIVE IDENTITY DISORDER-DSM 300.14 Diagnosis of Dissociative Identity Disorder-DSM 300.14 Presence of 2 or more distinct identities or personality states At least two identities or personality states recurrently take over person’s behavior Inability to recall personal information that is too extensive to be explained by forgetfulness Not due to medication or substance abuse

DEPERSONALIZATION DISORDER DIAGNOSIS Depersonalization disorder is a feeling of being detached from the world around the person, as if the person were an onlooker. TABLE 145 DIAGNOSIS OF DEPERSONALIZATION DISORDER-DSM 300.6 Diagnosis of Depersonalization Disorder-DSM 300.6 Persistent feeling of being detached During depersonalization experience, reality testing remains intact Causes clinically significant distress or impairment in social or occupational areas Doesn’t occur exclusively during the course of another mental disorder or physiological effects of medications or drugs

SOMATOFORM DISORDERS PRESENTATION Somatoform disorders are those where individuals present with physical symptoms of a particular ailment without an identifiable medical cause. Somatoform disorders are distinct in that the symptoms produced by the patient are not intentional. TABLE 146 SYMPTOMS OF SOMATOFORM DISORDER Symptoms of Somatoform Disorder Physical symptoms of an ailment without an identifiable cause Symptoms produced by patient are not intentional

SOMATIZATION DISORDER DIAGNOSIS

www.ClinicalReview.com


Psychiatry

115 Somatization disorder is one diagnosable somatoform disorder in which an individual has multiple medical complaints with no particular identifiable medical illness. Somatization disorder can only be diagnosed when the patient presents with symptoms in four different parts of the body or parts of the body with different functions. For example, the patient has two distinct gastrointestinal complaints, one sexual complaint, and one neurologic complaint. The last three criteria cannot include pain. Further, a portion of these symptoms must have started prior to age 30 and have been present for several years. Many of these symptoms are refractory to treatment, and occasionally, medical and surgical interventions may actually be iatrogenic causes of some of these complaints. Somatization disorder is more common in females, and there appears to be a genetic cause for it, especially in families with antisocial personality disorder and substance abuse disorder. TABLE 147 DIAGNOSIS OF SOMATOFORM DISORDER-DSM 300.81 Diagnosis of Somatoform Disorder-DSM 300.81 Multiple medical complaints with no identifiable medical cause No particular mental illness Can only be diagnosed when patient presents with pain in 4 different part of the body or parts with different functions Part of symptoms must have started before age 30 and been present several years

OTHER SOMATOFORM DISORDERS DIAGNOSIS Other somatoform disorders include undifferentiated somatoform disorder, which is a less severe form of somatization disorder with fewer complaints and a shorter duration. Conversion disorder is hallmarked by complaints with sensory and motor function not due to a medical cause. Pain disorder involves an exaggerated pain response not congruent with any present medical illness. Hypochondriasis is the fixed, false belief that the individual has a medical illness, but this is most likely a misinterpretation of a normal bodily function. Body dysmorphic disorder is an exaggerated perceived belief that a particular body part has a defect. Pseudocyesis is the development of physical signs of pregnancy and the false belief that one is pregnant in the face of negative laboratory tests and studies. TABLE 148 OTHER SOMATOFORM DISORDERS Other Somatoform Disorders Conversion disorder-DSM 300.11

Complaints with sensory or motor function NOT due to medical cause

Pain disorder-DSM 307.80

Exaggerated pain response not congruent with present medical condition

Hypochondriasis-DSM 300.7

Fixed, false belief that one has a medical illness. Usually misinterpretation of normal bodily function

Body dysmorphic disorder-DSM 300.7

Exaggerated perceived belief that body part has defect

MALINGERING AND FACTITIOUS DISORDERS MALINGERING PRIMARY, SECONDARY, AND TERTIARY GAIN

Â


USMLE STEP 2 116 Malingering is a psychiatric illness in which a person invents a disorder in order to receive a specific benefit. Symptoms are intentionally produced with the motivation to receive a specific gain. For example, a person may create the symptoms of the flu in order to avoid going to school. This is known as secondary gain because having the symptom (the flu) leads to a specific reward for the patient (avoiding school). Primary gain is the benefit to the patient’s ego by having the symptom. Tertiary gain is the benefit to the caretaker due to the patient having the symptom. An example of primary gain is the increase in self-esteem by having the flu; an example of tertiary gain is the satisfaction the physician gains by diagnosing a rare disorder. Primary gain is not seen in malingering by definition as the gain is sought out intentionally. Tertiary gain may or may not be present depending on the caretaker. TABLE 149 MALINGERING Malingering - DSM V65.2 Primary

Gain to patients ego

Secondary

Symptom leads to specific reward

Tertiary

For benefit of caretaker

FACTITIOUS DISORDERS PRESENTATION AND DIAGNOSIS Factitious disorder differs from somatization disorder in that the individual with a factitious disorder consciously creates a sign or symptom of a medical illness to have a primary or secondary gain. Factitious disorders are also different from malingering, in which an individual creates an illness to obtain gains that are different from those obtained by being in a sick role (secondary gains). Repeated admission to the hospital and willingness to undergo invasive procedures is termed Munchausen’s syndrome. Parents who create symptoms of an illness in a child is termed Munchausen’s syndrome by proxy. TABLE 150 FACTITIOUS DISORDERS VS. MALINGERING Factitious Disorders vs. Malingering Factitious

Gains from created illness

Malingering

Gains different from those gained by created illness

PERSONALITY DISORDERS CLASSIFICATION OF PERSONALITY DISORDERS Personality disorders are axis II disorders in the five axis evaluation of psychiatric disorders. Personality disorders can be characterized by one of three major types. Cluster A personality disorders include paranoid, schizoid, and schizotypal personality disorders, and can best be characterized as odd or eccentric in nature. Cluster B personality disorders include antisocial, borderline, histrionic, and narcissistic personality disorders, and can be characterized as overly dramatic or emotional in nature. Finally, cluster C personality disorders include avoidant, dependent, and obsessive-compulsive personality disorders, and tend to be anxious or fearful in nature. The dysfunction of personality disorders can be recalled with the mnemonic MEDIC, or maladaptive, enduring, deviation from cultural norms, inflexible, and causing psychosocial impairment.

www.ClinicalReview.com


Psychiatry

117

TABLE 151 PERSONALITY DISORDER TYPES Personality Disorder Types Cluster A

Paranoid, schizoid, and schizotypal

Cluster B

Antisocial, borderline, histrionic, narcissistic

Cluster C

Avoidant, dependent, obsessive-compulsive

CLUSTER A PERSONALITY DISORDERS PARANOID PERSONALITY DISORDER TABLE 152 DIAGNOSIS OF PARANOID PERSONALITY DISORDER Diagnosis of Paranoid Personality Disorder —DSM 301.0 Inherently distrustful High level of suspicion Requires separation from paranoia associated with psychotic disorders

SCHIZOID PERSONALITY DISORDER TABLE 153 DIAGNOSIS OF SCHIZOID PERSONALITY DISORDER Diagnosis of Schizoid Personality Disorder –DSM 301.20 Tend to avoid other people, emotionally distant and withdrawn Tend to be related to others in the family with schizophrenia or schizotypal personality disorder Detachment from the world Must be differentiated from avoidant personality disorder, social phobia, and schizophrenia

SCHIZOTYPAL PERSONALITY DISORDER TABLE 154 DIAGNOSIS OF SCHIZOTYPAL PERSONALITY DISORDER Diagnosis of Schizotypal Personality Disorder –DSM 301.22 Especially tied to patients who have families with a history of schizophrenia Avoid others Characterized by eccentric thoughts, bizarre affects, odd perceptions, incongruent beliefs Distrustful to the point of paranoia

CLUSTER B PERSONALITY DISORDERS ANTISOCIAL PERSONALITY DISORDER


118

USMLE STEP 2

TABLE 155 DIAGNOSIS OF ANTISOCIAL PERSONALITY DISORDER Diagnosis of Antisocial Personality Disorder – DSM 301.7 Disregard rules Lie, cheat and exploit others Impulsive Aggressive No remorse for illegal or moral actions Must be differentiated from bipolar and substance abuse disorders

BORDERLINE PERSONALITY DISORDER TABLE 156 DIAGNOSIS OF BORDERLINE PERSONALITY DISORDER Diagnosis of Borderline Personality Disorder –DSM 301.83 High levels of fear, anger, and worry about being abandoned their spouse Shifting idealization and devaluation of others Unpredictable changes in interpersonal relationships Highly reactive and volatile toward others Respond with anger and panic or depression to unexpected stressors Exhibit risk taking behavior Tend to be suicidal or feign suicide

HISTRIONIC PERSONALITY DISORDER TABLE 157 DIAGNOSIS OF HISTRIONIC PERSONALITY DISORDER Diagnosis of Histrionic Personality Disorder – DSM 301.50 Sexually seductive and socially inappropriate with regard to their hypersexuality Exaggerated emotional outbursts Unpredictable changes in interpersonal relationships Theatrical attention-seeking behavior Must be differentiated from somatization disorder

www.ClinicalReview.com


Psychiatry

119

NARCISSISTIC PERSONALITY DISORDER TABLE 158 DIAGNOSIS OF NARCISSISTIC PERSONALITY DISORDER Diagnosis of Narcissistic Personality Disorder – DSM 301.81 Highly egocentric and arrogant (reaction formation to protect them from a fragile ego and low self-esteem) Demand attention and special treatment Little concern for others Differentiate from bipolar disorder with grandiose features

CLUSTER C PERSONALITY DISORDERS AVOIDANT PERSONALITY DISORDER TABLE 159 DIAGNOSIS OF AVOIDANT PERSONALITY DISORDER Diagnosis of Avoidant Personality Disorder – DSM 301.82 Hypersensitive to criticism Feelings of extreme inadequacy to the point where there is no interaction with others due to fear of being singled out Extreme fear of humiliation and rejection by others Differentiate between avoidant personality and generalized social phobia

DEPENDENT PERSONALITY DISORDER TABLE 160 DIAGNOSIS OF DEPENDENT PERSONALITY DISORDER Diagnosis of Dependent Personality Disorder – DSM 301.6 Unable to function without the assistance from another Strong desire to be taken care of by others Cling to people due to fear of separation Tend to be highly submissive Differentiate from borderline personality disorder


120

USMLE STEP 2

OBSESSIVE-COMPULSIVE PERSONALITY DISORDER TABLE 161 DIAGNOSIS OF OBSESSIVE-COMPULSIVE PERSONALITY DISORDER Diagnosis of Obsessive-Compulsive Personality Disorder -DSM 301.4 Adhere to rigid schedules Rigid in interpersonal relationships Quick to judge Devoted to their work Avoid intimate relationships

TREATMENT TABLE 162 TREATMENT OF PERSONALITY DISORDERS Treatment of Personality Disorders Long term psychotherapy CBT Changes in life and interpersonal relationships Dialectical behavior therapy (DBT) is effective with borderline personality disorder Medication to treat symptoms of personality disorder Mood stabilizers to treat impulsiveness Benzodiazepines to treat anxiety Beta-blockers to treat anxiety SSRIs for depression, eating disturbances, and OCD Antipsychotics for psychotic conditions

SUBSTANCE ABUSE DISORDERS SUBSTANCE ABUSE AND DEPENDENCE SUBSTANCE ABUSE DIAGNOSIS Substance abuse is less severe than substance dependence (abuse before dependence). Substance abuse is defined as a malformed pattern of ingestion of a particular substance that leads to impairment in one’s psychosocial obligations. At least one of the following conditions must be fulfilled in order to meet the requirements of diagnosis, including a failure to meet obligations or expectations at school, work, or home; repeated use of a substance in dangerous situations; repeated infractions with the law; or repeated use despite worsening social or interpersonal problems due to the negative effects of the substance.

www.ClinicalReview.com


Psychiatry

121

TABLE 163 DIAGNOSIS OF SUBSTANCE ABUSE Diagnosis of Substance Abuse One or more of the following: Failure to met school, home or work obligations/expectations Repeated use of substance in dangerous situations Repeated infractions with the law Repeated use despite worsening social or interpersonal problems due to negative effects of substance

SUBSTANCE DEPENDENCE DIAGNOSIS Substance dependence is defined with regard to its clinical impairment, and requires at least three of the following criteria within a one year period for diagnosis: tolerance, withdrawal, repeated use, failed efforts to cut down on use, large amount of time spent trying to obtain the substance, decrease in important social or occupational activities, use despite awareness of psychosocial impairments, use despite physical impairments, and excessive or unintended use of the compound. TABLE 164 DIAGNOSIS OF SUBSTANCE DEPENDENCE Diagnosis of Substance Dependence Three or more of the following within one year Tolerance

Withdrawal

Repeated use

Failed efforts to cut down on use

Repeated infractions with the law

Large amount of time spent trying to obtain substance

Decrease in social or occupational activities

Use despite awareness of psychosocial impairment

Use despite physical impairment

Excessive or unintended use of substance

ALCOHOL-INDUCED DISORDERS PRESENTATION Alcohol intoxication is characterized by uncoordinated motor activity, slurred speech, imbalance, nystagmus, defect in concentration or memory, stupor or coma, and maladaptive or inappropriate behavior or functioning. Alcohol intoxication is quantified by serum tests such as a blood alcohol level (BAL). Other lab tests in severe, chronic alcohol abuse include elevated high-density lipoprotein (HDL), decreased low-density lipoprotein (LDL), elevated mean red blood cell volume (MCV), and elevated liver enzymes. Alcohol intoxication presents with loss of inhibition over one’s better judgment. Abusers are more prone to emotional lability, lose the ability to enunciate during speech, have difficulty walking and maintaining balance, can lose consciousness with heavy abuse, and in severe cases, fall into a coma.

Â


122

USMLE STEP 2

TABLE 165 SIGNS AND SYMPTOMS OF ALCOHOL DEPENDENCE/INTOXICATION Signs and Symptoms of Alcohol Dependence/Intoxication Uncoordinated motor activity

Slurred speech

Imbalance

Nystagmus

Defect in concentration or memory

Stupor or coma

Maladaptive or inappropriate behavior or functioning

EPIDEMIOLOGY AND DIAGNOSIS Alcohol dependence is a clinical disorder characterized by tolerance and withdrawal symptoms. The lifetime prevalence of alcoholism (alcohol dependence) is nearly 14%, with males disproportionately affected compared to females. There appears to be a genetic basis for alcoholism, especially in males. Alcoholics also tend to have a family history of antisocial personality disorder and depression. TABLE 166 DIAGNOSIS OF ALCOHOL DEPENDENCE-DSM 303.90 Diagnosis of Alcohol Dependence – DSM 303.90 Tolerance

Excessive or unintended use of substance

Withdrawal symptoms

Use despite physical impairment

Repeated use

Failed efforts to cut down on use

Repeated infractions with the law

Large amount of time spent trying to obtain substance

Decrease in social or occupational activities

Use despite awareness of psychosocial impairment

Palmar erythema

Painless hepatomegaly

Acne rosacea

Cirrhosis

Jaundice

Ascites

Atrophy of testicles

Gynecomastia

Dupuytren’s’s contracture

Wernicke-Korsakoff syndrome

PRESENTATION Alcoholics tend to strongly deny their illness, and the importance of obtaining collateral information cannot be overstated. Physical symptoms sometimes indicate the presence of this disorder, including palmar erythema, painless hepatomegaly, and acne rosacea. Advanced alcoholism includes signs of cirrhosis of the liver, jaundice, ascites, atrophy of the testicles, gynecomastia, and Dupuytren’s contracture. Sequelae of each of these clinical symptoms are also present depending on the stage of the alcoholism. There is also an increased incidence of cancer, pneumonia, heart disease, and hypertension. Alcoholism can also be gauged by the CAGE questions, or asking whether the individual has felt a need to cut down on their drinking, whether www.ClinicalReview.com


Psychiatry

123 they have ever been annoyed by the criticism of others regarding their drinking, whether they feel guilty about their drinking, and whether they require an eye-opener or early morning drink. More than one positive answer makes alcoholism likely. WERNICKE-KORSAKOFF SYNDROME Wernicke-Korsakoff syndrome may also develop in alcoholics due to the destruction of the mammillary bodies. This syndrome develops due to a deficiency in thiamine. The initial stage of this disorder, known as the Wernicke stage, is characterized by nystagmus, ataxia, and mental confusion. Wernicke’s encephalopathy may be reversed with prompt administration of thiamine. However, failure to do so leads to progression to Wernicke-Korsakoff syndrome with Korsakoff’s psychosis, presenting with anterograde amnesia and confabulations. Korsakoff’s psychosis is frequently irreversible, and can be complicated with hallucinations, dementia, neuropathy, depression, and a greater tendency towards suicide. Alcohol-dependent patients also tend to have signs of old rib fractures on X-ray. WITHDRAWAL The symptoms of alcohol withdrawal include the development of a fine tremor, increased heart rate, increased blood pressure, nausea and vomiting, seizures, anxiety, and in severe cases, hallucinations and delirium tremens. Tremors may peak at approximately one day after cessation of alcohol. Such withdrawal can last nearly a week if untreated, and may be accompanied by nausea and vomiting, headache, tachycardia, and high blood pressure. These withdrawal symptoms, if minor, can be treated with benzodiazepines including oxazepam and chlordiazepoxide. These medications should be titrated to match the withdrawal symptoms and tapered over several days. SEVERE WITHDRAWAL More severe symptoms of withdrawal include seizures that can begin about half a day after cessation. These seizures can precede delirium tremens, and should be treated with benzodiazepines and prophylactic phenytoin, in high-risk patients. Strong auditory hallucinations may also be present in alcoholic hallucinosis, with an onset within two days after cessation. A neuroleptic such as haloperidol may be administered to treat this etiology. TABLE 167 SYMPTOMS OF ALCOHOL WITHDRAWAL-DSM 291.81 Symptoms of Alcohol Withdrawal-DSM-291.81 Fine tremor

Increased heart rate

Increased blood pressure

Nausea and vomiting

Seizures

Anxiety

Hallucinations

Delirium tremens

Seizures

DELIRIUM TREMENS Delirium tremens is serious sequelae of alcohol withdrawal, and can be fatal in 15-20% of patients, if untreated. Delirium tremens is characterized by confusion, disorientation, agitation, perceptual disturbances such as hallucinations, hyperarousal of the autonomic nervous system (ANS), and a mild fever. It typically begins several days after cessation of alcohol, and can occur in one in twenty hospitalized patients with alcohol dependence. Treatment includes benzodiazepines and supportive therapy and may


USMLE STEP 2 124 require admission to an intensive care unit (ICU), if there is instability in the blood pressures or other problems of the autonomic nervous system. The duration of delirium tremens is typically three days, but can last up to one month. TABLE 168 SYMPTOMS OF DELIRIUM TREMENS Symptoms of Delirium Tremens Confusion

Disorientation

Agitation

Hallucinations

Hyperarousal of ANS

Mild fever

TABLE 169 TREATMENT OF DELIRIUM TREMENS Treatment of Delirium Tremens Benzodiazepine Supportive care

REHABILITATION Rehabilitation of alcoholism requires admission to a twelve-step program such as alcoholics’ anonymous, completion of the entire program, and continued avoidance of all alcohol. Side effects of chronic alcohol use include depression and anxiety, both of which should be treated if they continue two to four weeks after cessation of alcohol abuse. Disulfiram can be used in some patients to maintain avoidance of alcohol as this enzyme inhibits aldehyde dehydrogenase; this causes conditioned avoidance. Consumption of alcohol after blocking this enzyme leads to rapid accumulation of acetaldehyde in the bloodstream and subsequent nausea, vomiting, flushing, palpitations, and hypotension. Naltrexone is another medication that can be used to maintain avoidance. This opiate antagonist reduces alcohol intake and frequency of intake. Naltrexone can be taken even if the patient continues alcohol. Naltrexone’s mechanism of action may be mediated by reducing the positive reinforcement of alcohol. Overall, alcohol rehabilitation has a fifty percent failure rate. TABLE 170 TREATMENT OF ALCOHOLISM Treatment of Alcoholism 12 step program Continued avoidance of alcohol (Disulfiram can be used – causes conditioned avoidance; Naltrexone is opioid antagonist) Possible treatment for depression and anxiety

ANXIOLYTIC, SEDATIVE, AND HYPNOTIC SUBSTANCE ABUSE ABUSE AND WITHDRAWAL SYMPTOMS Barbiturates and benzodiazepines are examples of drugs that fall into the anxiolytic, sedative, and hypnotic category. These drugs are cross-tolerant with alcohol, and can form a co-dependence. Abuse of these drugs can lead to symptoms similar to alcohol intoxication and withdrawal. It is distinguished from alcohol intoxication by the absence of alcohol in the serum or a urine toxicology screen. Withdrawal symptoms are similar to that of alcohol, and specifically include anxiety, apprehension, www.ClinicalReview.com


Psychiatry

125

restlessness, tremors, nausea and vomiting, weakness and fatigue, hyperreflexia, diaphoresis, seizures, and orthostatic hypotension. Three or more of these symptoms must be present for the diagnosis of substance dependence disorder. Visual and somatic hallucinations, disorientation, and confusion can begin several days after cessation of the drug. TREATMENT Management involves detoxification, administration of benzodiazepines or barbiturates with a controlled taper to minimize withdrawal symptoms, and symptomatic management. Barbiturates tend to be more dangerous than benzodiazepines, and withdrawal can lead to dangerously high fevers (hyperpyrexia) and death. Diazepam or phenobarbital is often used in withdrawal management. ALCOHOL AND BENZODIAZEPINE ABUSE In patients who abuse both alcohol and benzodiazepines or barbiturates, a pentobarbital challenge test is required to quantify the tolerance to these compounds and quantify the amount of medication required for a controlled taper. Patients should also be referred to a twelve step program for further rehabilitation. TABLE 171 TREATMENT OF ANXIOLYTIC, SEDATIVE, AND HYPNOTIC DRUG ABUSE Treatment of Anxiolytic, Sedative and Hypnotic Drug Abuse Detoxification Administer benzodiazepines (diazepam) or barbiturates (phenobarbital), with controlled taper Symptomatic management TABLE 172 WITHDRAWAL OF ANXIOLYTIC, SEDATIVE, AND HYPNOTIC DRUGS Withdrawal of Anxiolytic, Sedative and Hypnotic Drugs Two or more of the following: Sweating

Pulse rate over 100

Increased hand tremor

Insomnia

Nausea or vomiting

Transient tactile, auditory, or visual hallucinations

Psychomotor agitation

Anxiety

Grand mal seizures

TABLE 173 DIAGNOSIS OF ANXIOLYTIC, SEDATIVE, AND HYPNOTIC DRUG ABUSE-DSM 292.89 Diagnosis of Anxiolytic, Sedative and Hypnotic Drug Abuse-DSM 292.89 Three or more of the withdrawal symptoms (see table 124) Clinically significant maladaptive behavior or psychological changes that occur shortly after drug abuse Recent use of sedative, hypnotic, or anxiolytic

Â


126

USMLE STEP 2

OPIOID ABUSE EPIDEMIOLOGY ANDPRESENTATION Morphine, codeine, heroin, meperidine, and hydromorphone are examples of substances that fall under opioid abuse disorders. Heroin is the only one of these substances that is completely illegal in the United States. Lifetime prevalence of abuse is less than 1%, though this number is increasing. Opioids are often taken intravenously, leading to intense pleasure and a diffuse orgasm. After these symptoms occur, general well being follows then subsequent psychomotor retardation, impaired concentration, sleepiness, and fatigue. Upon use, the pupils constrict (miosis), there is respiratory depression, slurred speech, bradycardia, hypotension, and hypothermia. Nausea, vomiting, and constipation can all be present in opiate use TABLE 174 SYMPTOMS OF OPIOID ABUSE Symptoms of Opioid Abuse Pupil constriction (miosis)

Respiratory depression

Slurred speech

Bradycardia

Hypotension

Hypothermia

Nausea/vomiting

Constipation

DEPENDENCE Use of opiates more than three times a day is typically a hallmark of dependent behavior. TABLE 175 DIAGNOSIS OF OPIOID ABUSE-DSM 305.50 Diagnosis of Opioid Abuse-DSM 305.50 Use of opiates more than three times a day One or more of the following: Failure to met school, home or work obligations/expectations Repeated use of substance in dangerous situations Repeated infractions with the law Repeated use despite worsening social or interpersonal problems due to negative effects of substance

WITHDRAWAL Withdrawal begins within half a day after cessation, and signs and symptoms include dysphoria, rhinorrhea, lacrimation, diaphoresis, mydriasis, piloerection, hypertension, tachycardia, fever, diarrhea, insomnia, and yawning. Severe symptoms include nausea and vomiting, seizure especially in meperidine withdrawal, muscle aches, abdominal cramps, hot and cold flashes, and severe anxiety.

www.ClinicalReview.com


Psychiatry

127

TABLE 176 SYMPTOMS OF OPIOID ABUSE WITHDRAWAL Symptoms of Opioid Abuse Withdrawal Dysphoria

Lacrimation

Rhinorrhea

Tachycardia

Hypertension

Mydriasis

Fever

Diarrhea

Diaphoresis

Piloerection

Yawning

Insomnia

COMORBID ILLNESSES Comorbid psychiatric illnesses often exist in opioid abusers, including antisocial or borderline personality disorder, and mood disorders. These patients tend to commit crimes in order to finance their drug habits, and also have a high mortality rate due to the intercurrent intravenously-transmitted illnesses, accidental overdoses, and tendency towards suicide. TREATMENT Opioid abusers should be withdrawn by methadone administration, a mu opiate receptor partial agonist. Methadone causes few positive or negative effects, and so is an ideal substance to take the place of the powerful stimulatory and withdrawal effects of opiates. Clonidine can also be used to treat withdrawal through its alpha-two receptor agonist abilities. Clonidine is effective at treating the acute autonomic dysfunction of withdrawal, but has little effect on managing the cravings of the opioid addict. Additional medications include treating abdominal cramps with dicyclomine, nausea with promethazine, and muscle aches with quinine. The combination of medical intervention and rehabilitation programs such as a twelve-step program is necessary, along with long-term administration of methadone. TABLE 177 TREATMENT OF OPIOID ABUSE Treatment of Opioid Abuse Dicyclomine

Quinine

Clonidine

Promethazine

Methadone (long term)

12 step program

AMPHETAMINE ABUSE PRESENTATION Cocaine and other amphetamines are commonly abused drugs. These drugs cause stimulation of the central nervous system and tend to drive the sympathetic nervous system. Cocaine can be either snorted in the powder form, or smoked in the crack form. Cocaine has a rapid onset, a short half-life, and a number of side effects. Intoxication with cocaine or other amphetamines includes dysfunctional behavioral changes such as hypervigilance or euphoria, mydriasis, nausea and vomiting, weight loss, confusion, seizures, dyskinesia, coma, muscular weakness, respiratory depression, chest pain, cardiac dysrhythmias, sudden cardiac death, either tachycardia or bradycardia, either hypertension or hypotension, either diaphoresis or chills, and either psychomotor retardation or agitation.

Â


128

USMLE STEP 2

TABLE 178 SYMPTOMS OF AMPHETAMINE ABUSE Symptoms of Amphetamine Abuse Hypervigilance or euphoria

Mydriasis

Nausea and vomiting

Weight loss

Confusion

Seizures

Dyskinesia

Coma

Muscular weakness

Respiratory depression

Chest pain

Cardiac dysrhythmias

Tachycardia or bradycardia

Sudden cardiac death

Hypertension or hypotension

Diaphoresis or chills

TACTILE HALLUCINATIONS Cocaine is one of the few drugs that can cause a tactile hallucination on intoxication, such as a feeling that bugs are crawling all over a person. A transient psychosis with visual hallucinations and paranoia can also occur during intoxication. TABLE 179 DIAGNOSIS OF AMPHETAMINE ABUSE-DSM 305.70 Diagnosis of Amphetamine Abuse-DSM 305.70 Tactile hallucinations on intoxications Transient psychosis with visual hallucinations Paranoia

WITHDRAWAL Withdrawal symptoms can be severe but nonfatal, and include fatigue, nightmares, depression, headache, diaphoresis, muscle cramps, and hunger. Withdrawal typically lasts several days. TABLE 180 WITHDRAWAL SYMPTOMS OF AMPHETAMINE ABUSE Withdrawal Symptoms of Amphetamine Abuse Fatigue

Nightmares

Depression

Headache

Confusion

Seizures

Diaphoresis

Muscle cramps

Hunger

Nonfatal

www.ClinicalReview.com


Psychiatry

129

TREATMENT Amphetamines are available through prescription for the treatment of obesity, attention-deficit hyperactivity disorder, and narcolepsy. Management of amphetamine abuse is typically supportive therapy, as the withdrawal is self-limited and not hazardous. Antipsychotics can be used for extreme anxiety, as indicated. A twelve-step rehabilitation program is required for proper treatment. TABLE 181 TREATMENT OF AMPHETAMINE ABUSE Treatment of Amphetamine Abuse Supportive therapy Antipsychotics 12 step program

OTHER SUBSTANCE ABUSE DISORDERS MARIJUANA ABUSE The abuse of marijuana or hashish (cannabis) is common throughout the world. Intoxication causes euphoria, impaired judgment, poor concentration, and decreased memory. Side effects include permanent memory effects, permanent intellectual impairment, testicular degeneration, delirium, and psychosis. Marijuana is a commonly abused drug that leads to a number of side effects. Marijuana leads to a profound feeling of pleasure, but abuse can also lead to anxiety and delusions or hallucinations. One of the effects of abuse is the feeling that time has slowed. ETIOLOGY AND PRESENTATION Marijuana leads to impaired judgment and withdrawal from society (“stoned”). Appetite is increased and dry mouth with bronchitis are common physical symptoms. The effects of marijuana can best be summarized as follows: respiratory effects, amotivation, and increased risk of mental illnesses. Marijuana increases serotonin through the function of an inhibitory G protein. This is the mechanism by which a dreamlike state is created and gives the sensation that time has slowed. The vomiting that can occur with marijuana is best treated with dronabinol. TABLE 182 SYMPTOMS OF MARIJUANA ABUSE Symptoms of Marijuana Abuse Euphoria

Poor judgment

Appetite increase

Poor concentration

Decreased memory

Dry mouth with bronchitis

Permanent memory loss

Permanent intellectual impairment

Testicular degeneration

Delirium

Chest pain

Cardiac dysrhythmias

Tachycardia or bradycardia

Sudden cardiac death

Hypertension or hypotension

Psychomotor agitation or retardation

Tactile or visual hallucinations

Paranoia

Side effects may include:


130

USMLE STEP 2

TABLE 183 TREATMENT OF MARIJUANA ABUSE Treatment of Marijuana Abuse Dronabinol for vomiting TABLE 184 DIAGNOSIS OF MARIJUANA ABUSE-DSM 305.20 Symptoms of Marijuana Abuse Recent use of cannabis

Symptoms not due to general medical condition

Two or more of the following: Conjunctiva injection

Xerostomia

Tachycardia

Increased appetite

NICOTINE ABUSE Nicotine abuse leads to symptoms similar to caffeine abuse, including restlessness, difficulty sleeping, anxiety, and in severe cases, cardiac arrhythmias. Withdrawal from nicotine is similar to caffeine-withdrawal, including headaches, anxiety, and weight gain. Nicotine withdrawal also leads to a strong craving and elevated heart rate. Treatment for nicotine abuse is with bupropion, an antidepressant that reduces the craving for nicotine. TABLE 185 SYMPTOMS OF NICOTINE ABUSE Symptoms of Nicotine Abuse Restlessness Difficulty sleeping Anxiety Cardiac arrhythmias, in severe cases TABLE 186 WITHDRAWAL SYMPTOMS OF NICOTINE ABUSE Withdrawal Symptoms of Nicotine Abuse Headaches Anxiety Weight gain Elevated heart rate Strong cravings TABLE 187 TREATMENT OF NICOTINE ABUSE Treatment of Nicotine Abuse Buproprion

www.ClinicalReview.com


Psychiatry

131

TABLE 188 DIAGNOSIS OF NICOTINE ABUSE-DSM 305.1 Diagnosis of Nicotine Abuse-305.1 Daily use for at least several weeks Stopping or cut back of nicotine use causes at least 4 of the following: Dysphoric or depressed mood

Insomnia

Irritability, frustration, or anger

Anxiety

Decreased heart rate

Increased appetite or weight gain

Difficulty concentrating

Restlessness

Significant distress in social, occupational or other important areas of functioning Not due to general medical condition or mental disorder

CAFFEINE ABUSE Caffeine abuse is common. Smaller doses of caffeine lead to restlessness and insomnia. Increasing doses can lead to a diuresis and muscle twitching. In susceptible individuals, potentially fatal cardiac arrhythmias can develop. Caffeine leads to an increase in cAMP release in neurons. Caffeine withdrawal is a common cause of headaches seen in a clinical setting. Withdrawal from caffeine also leads to fatigue, depression, and weight gain. TABLE 189 SYMPTOMS OF CAFFEINE ABUSE Symptoms of Caffeine Abuse Restlessness Insomnia Diuresis Muscle twitching TABLE 190 WITHDRAWAL SYMPTOMS OF CAFFEINE ABUSE Withdrawal Symptoms Caffeine Abuse Headaches Anxiety Weight gain Elevated heart rate Strong cravings

Â


132

USMLE STEP 2

TABLE 191 DIAGNOSIS OF CAFFEINE ABUSE-DSM 305.90 Diagnosis of Caffeine Abuse-DSM 305.90 Daily use for at least several weeks Stopping or cut back of caffeine use causes at least 4 of the following: Dysphoric or depressed mood

Insomnia

Irritability, frustration, or anger

Anxiety

Decreased heart rate

Increased appetite or weight gain

Difficulty concentrating

Restlessness

Significant distress in social, occupational or other important areas of functioning Not due to general medical condition or mental disorder

PCP ABUSE PCP (phencyclidine) abuse has been on the decline for sometime due to psychotic effects it induces in many individuals. PCP is one of the few drugs that lead to aggressive and belligerent behavior culminating in homicide or suicide. Individuals intoxicated with PCP tend to be very impulsive and act like they are “drunk”. There is a severe psychomotor anxiety, psychosis, and delirium. The activity of the autonomic nervous system is increased with elevated heart rate. Central nervous system symptoms of PCP abuse include ataxia, vertical and horizontal nystagmus, and fever. TABLE 192 SYMPTOMS OF PCP ABUSE Symptoms of PCP Abuse Impulsive Subject acts “drunk” Severe psychomotor anxiety Psychosis Delirium

PCP WITHDRAWAL PCP withdrawal is unique in that it can lead to a repeat of the symptoms seen in PCP abuse. This occurs because the drug may be reabsorbed by the GI tract. This reabsorption over time can lead to episodes of sudden homicidal violence. Abuse is on the decline as people who abuse these drugs are more interested in the sense of euphoria than going to jail due to homicide.

www.ClinicalReview.com


Psychiatry

133

TABLE 193 WITHDRAWAL SYMPTOMS OF PCP Withdrawal Symptoms of PCP Abuse Repeat of symptoms of PCP abuse Sudden homicidal violence TABLE 194 DIAGNOSIS OF PCP ABUSE-DSM 305.90 Diagnosis of PCP Abuse-DSM 305.90 Recent use of PCP Maladaptive behavior after recent PCP use Within an hour of use, 2 or more of the following: Vertical or horizontal nystagmus

Hypertension or tachycardia

Numbness or diminished response to pain

Ataxia

Dysarthria

Muscle rigidity

Seizures

Coma

Hyperacusis

LSD ABUSE LSD is different than PCP in that LSD breeds a cross tolerance with ecstasy and MDMA, but PCP does not. LSD abuse is like PCP abuse in that it leads to significant anxiety and delusions. LSD also causes profound visual hallucinations and flashbacks. Mydriasis is common. LSD is a partial postsynaptic serotonin agonist, and it is through this mechanism that flashbacks and synesthesias may occur. LSD can also lead to convulsions. There are no significant withdrawal symptoms of LSD. LSD abuse is similar to the effects of ecstasy and MDMA, but the latter two substances can induce a panic psychosis. Abuse is best treated with diazepam. MDMA leads to increased impulsiveness through destroyed serotonin receptors and can lead to memory gaps. TABLE 195 SYMPTOMS OF LSD ABUSE Symptoms of LSD Abuse Significant anxiety and delusions Mydriasis Flashbacks Synesthesias

ECSTASY ABUSE Ecstasy is frequently abused in dance clubs, and has a stimulant and euphoric effect. Ecstasy, also known as MDMA, can enhance one’s desire for intimacy. Long-term effects include loss of serotonin axons in the brain. Methamphetamine is a powerful amphetamine that can lead to destruction of dopamine neurons.


134

USMLE STEP 2

TABLE 196 DIAGNOSIS OF METHAMPHETAMINE ABUSE (MDMA, ECSTASY)-DSM 292.89 Diagnosis of Methamphetamine Abuse (MDMA, Ecstasy) - DSM 292.89 Euphoria or affective blunting

Changes in sociability

Tachycardia or bradycardia

Hypervigilance

Pupil dilation

Interpersonal sensitivity

Anxiety

Tension

High or low blood pressure

Anger

Nausea or vomiting

Stereotyped behaviors

Impaired judgment

Weight loss

Impaired social/occupational functioning

Muscular weakness

Chest pain

Respiratory depression

Confusion

Seizures

Coma

TABLE 197 WITHDRAWAL SYMPTOMS OF METHAMPHETAMINE-DSM 292.0 Withdrawal Symptoms of Methamphetamine-DSM 292.0 Cessation or reduction of amphetamines Dysphoric mood Two or more of the following: Fatigue

Unpleasant vivid dreams

Insomnia or hypersomnia

Increased appetite

Psychomotor retardation

Agitation

GHB ABUSE Gamma-hydroxybutyrate (GHB) is a steroid drug that promotes an increase in muscle mass; overdoses can cause highs, but can lead to death due to respiratory arrest. The abuse of anabolic steroids can lead to skin atrophy, acne, spontaneous bruising, hypokalemia, cardiomyopathy, osteoporosis, hypertension, diabetes, emotional lability and depression, gynecomastia, testicular atrophy, and alopecia. Abuse is especially common in body builders, and their negative effects occasionally make headlines due to sudden cardiac death in some athletes (not to be confused with idiopathic hypertrophic cardiomyopathy and Brugada syndrome, both of which belong on the differential diagnosis in this case). TABLE 198 SYMPTOMS OF GAMMA-HYDROXYBUTYRATE (GHB) ABUSE Symptoms of Gamma-Hydroxybutyrate (GHB) Abuse Can lead to death due to respiratory arrest Skin atrophy Acne Spontaneous bruising Hypokalemia Cardiomyopathy

www.ClinicalReview.com


Psychiatry

135

KETAMINE ABUSE Ketamine is a dissociative anesthetic that can cause hallucinations. Rohypnol is a benzodiazepine that can cause sedation and amnesia, and is frequently used by sexual predators.

ANTIPSYCHOTICS TABLE 199 ANTIPSYCHOTIC MEDICATIONS Antipsychotic Medications Typical

Atypical

Haloperidol

Risperdone

Fluphenazine

Ziprasidone

Perphenazine

Aripiprazole

Trifluoperazine

Olanzapine

Thioridazine

Clozapine

Chlorpromazine

Quetiapine Clozaril

INDICATIONS Antipsychotic medications have a number of uses in treating various types of psychiatric illness. Typical antipsychotics, also known as neuroleptics, and atypical antipsychotics are the two major classes of antipsychotics. Antipsychotics are the drugs of choice for the treatment of psychotic symptoms such as hallucinations, delusions, and bizarre behavior. These psychotic symptoms can belong to any category of psychiatric illness so long as they are positive symptoms. Antipsychotics are generally less effective in treating negative symptoms such as amotivation, akinesia, isolation, and blunting of affect with the exception of clozapine. Antipsychotics have also been used in the treatment of various types of behavioral dysregulation as that seen in Alzheimer’s, mental retardation, Tourette syndrome, and in patients with personality disorders. TABLE 200 INDICATIONS FOR ANTIPSYCHOTIC MEDICATION USE Indications for Antipsychotic Medication Use Delusions

Note: Only clozapine works for negative symptoms

Bizarre behavior Hallucinations Alzheimer’s Mental retardation Tourette syndrome

MECHANISM OF ACTION DOPAMINERGIC HYPOTHESIS


USMLE STEP 2 136 Antipsychotics are thought to act via a dopaminergic pathway, as that explained previously with regard to the dopamine hypothesis of schizophrenia. According to this hypothesis, antipsychotics block the dopamine 2 (D2) receptor, leading to a decrease in dopamine-mediated positive symptoms of psychosis. This is consistent with the dopamine hypothesis of schizophrenia, which states that the positive symptoms of schizophrenia are due in part to an excessive number of dopamine receptors leading to heightened stimulation of dopamine pathways. This theory is supported by the induction of psychosis with dopamine agonists such as amphetamines. MECHANISM OF ACTION Blockade of the dopamine pathways occurs in the ventral tegmental area and substantia nigra, both of which project to the basal ganglia, frontal cortex, and limbic regions of the brain. Reduction of the psychotic symptoms is thought to occur by the blockade in the cortical and limbic regions. However, the notable side effect of extrapyramidal symptoms (EPS) is thought to occur to unintentional blockade of D2 receptors in the basal ganglia. TABLE 201 MECHANISM OF ACTION OF ANTIPSYCHOTICS Mechanism of Action of Antipsychotics Block dopamine receptors (D2), but may also block D1-D5, histamine and serotonin receptors Decrease dopamine, thus decreases positive symptoms of psychosis Affects ventral tegmental area and substantia nigra with resultant affects on basal ganglia, frontal cortex, and limbic system.

SIDE EFFECTS Antipsychotics have a number of side effects related to their additional, still incompletely characterized functions elsewhere in the brain. Other antipsychotics, especially the atypical antipsychotics, have been found to work through a mechanism in addition to the dopamine blockade. Newer medications have the additional effect of blockading serotonin 5-hydroxytryptamine-2 (5HT2) receptors. Atypical antipsychotics are categorized as such based on their fewer extrapyramidal side effects, and their putative function through 5HT2 blockade. It is thought that blockade of these serotinergic pathways has a protective effect against extrapyramidal symptoms and even contribute to their antipsychotic effects. Of the atypical antipsychotics, risperidone is an example of a medication that preferentially blocks the D2 receptors. Clozapine, on the other hand, is an example of an atypical antipsychotic that primarily blocks the D4 receptor. THERAPY AND DRUG REACTIONS TYPICAL ANTIPSYCHOTICS The typical antipsychotics are thioridazine, chlorpromazine, perphenazine, trifluoperazine, thiothixene, haloperidol, and fluphenazine. Haloperidol and fluphenazine are the most potent of the typical antipsychotics, while thioridazine and chlorpromazine are the least efficacious. Thioridazine and chlorpromazine are highly sedating, while haloperidol and thiothixene are the least sedating. Thioridazine and chlorpromazine cause orthostatic hypotension, while the other drugs have a lower incidence of this. Thioridazine is highly anticholinergic. EPS symptoms are least prevalent with thioridazine and chlorpromazine. Thioridazine also causes a retinopathy and fatal cardiac events, and so is used only for schizophrenia refractory to other medications. A prevalence of anticholinergic effects can be partially reversed with the peripherally acting cholinergic stimulant bethanechol.

www.ClinicalReview.com


Psychiatry

137

TABLE 202 THERAPY USING TYPICAL ANTIPSYCHOTIC MEDICATIONS Therapy Using Typical Antipsychotic Medications Most sedating

Least sedating

Most potent

Thioridazine

Haloperidol

Haloperidol

Chlorpromazine

Thiothixene

Fluphenazine

TABLE 203 SIDE EFFECTS OF TYPICAL ANTIPSYCHOTIC MEDICATIONS Side Effects of Typical Antipsychotic Medications Thioridazine – orthostatic hypotension, retinopathy, fatal cardiac events, highly anticholinergic, increased pigmentation of retina

ATYPICAL ANTIPSYCHOTICS The atypical antipsychotics include clozapine, quetiapine, ziprasidone, aripiprazole, olanzapine, and risperidone. Risperidone is the most potent, while quetiapine and clozapine are the least potent. Clozapine and quetiapine are highly sedating, and also cause orthostatic hypotension. Risperidone also causes orthostatic hypotension. Clozapine has anticholinergic effects, while risperidone has few anticholinergic events. All of the atypical antipsychotics rarely cause EPS. Clozapine is noted for causing an agranulocytosis and also myocarditis. Ziprasidone causes prolongation of the QT interval. Risperidone has also been known to cause hyperprolactinemia, which can lead to a decreased sex drive and amenorrhea. TABLE 204 THERAPY USING ATYPICAL ANTIPSYCHOTIC MEDICATIONS Therapy Using Atypical Antipsychotic Medications Most Sedating

Most Potent

Clozapine

Risperdone

Quetiapine

CHOICE OF MEDICATION All antipsychotics are considered to be highly effective in the management of positive symptoms of psychosis. The choice of medication should be based on prior responses, known side effects and the ability of the patient to tolerate them, and whether the form of the medication is suitable for the patient. Of all the antipsychotic medications, clozapine is an effective choice when the other antipsychotics have minimal effect on the psychosis. Clozapine is an ideal candidate for psychiatric illnesses with positive psychotic features that are refractory to other medical treatment as this medication works primarily through blockade of D4 receptors instead of D2 receptors. In patients who require intramuscular (IM) injections of drugs, haloperidol and fluphenazine are suitable candidates. TABLE 205 SPECIFIC SIDE EFFECTS OF ATYPICAL ANTIPSYCHOTIC MEDICATIONS Specific Side Effects of Atypical Antipsychotic Medications Clozapine

Lowering seizure threshold, in addition to its lethal side effect of agranulocytosis, EKG changes, hyperlipidemia

Olanzapine

EKG changes, weight gain, hyperlipidemia

Quetiapine

Cataract formation

Risperidone

Hyperprolactinemia, decreased sex drive, amenorrhea

Â


138

USMLE STEP 2

ADMINISTRATION AND SIDE EFFECTS The administration of antipsychotics requires careful and close monitoring of patients for a number of side effects. The most serious neurologic drug reactions can lead to neuroleptic malignant syndrome (NMS), extrapyramidal symptoms (EPS), and akathisia or restlessness. Patients on clozapine need to be monitored for agranulocytosis by having weekly white blood cell (WBC) counts. Generally, monitoring serum levels of these antipsychotics has little value due to their numerous metabolites. Haloperidol and clozapine are the only exceptions to this rule. LOW-POTENCY ANTICHOLINERGIC SIDE EFFECTS The low-potency antipsychotics are noted for having the worst anticholinergic effects, including dry mouth, blurred vision, constipation, and urinary retention. However, these low-potency drugs with the elevated anticholinergic effects counter the extrapyramidal symptoms seen with antipsychotics. The elderly, patients with organic brain disease, and patients on other anticholinergic medications may experience delirium. SIDE EFFECTS – SEIZURE THRESHOLD The low-potency antipsychotics are also notable for lowering the seizure threshold. Clozapine is also noted for lowering this seizure threshold, in addition to its lethal side effect of agranulocytosis. Orthostatic hypotension is common with the low-potency antipsychotics, in addition to risperidone. This orthostatic hypotension is primarily due to blockade of alpha-receptors. SIDE EFFECTS – CARDIAC, RETINA, WEIGHT Cardiac side effects are found with the low-potency antipsychotics, especially thioridazine, ziprasidone, and risperidone. These side effects primarily manifest as long QT syndrome with the risk of torsade de pointes. Clozapine and olanzapine can also lead to EKG changes. Metabolic effects such as weight gain and the onset of diabetes are noted drug reactions of olanzapine and clozapine. Hyperlipidemia is also a noted side effect of olanzapine and clozapine. Pigmentation of the skin and eyes can occur with the typical antipsychotics. Thioridazine is especially noted for causing increased pigmentation of the retina at high doses. Quetiapine may lead to increased cataract formation. Ziprasidone is weight neutral. TABLE 206 GENERAL SIDE EFFECTS OF TYPICAL ANTIPSYCHOTIC DRUGS General Side Effects of Antipsychotic Medications Neuroleptic malignant syndrome (NMS) Extrapyramidal syndrome (EPS) Quetiapine

Cataract formation

Risperidone

Hyperprolactinemia, decreased sex drive, amenorrhea

TABLE 207 GENERAL SIDE EFFECTS OF ATYPICAL LOW POTENCY ANTIPSYCHOTIC DRUGS General Side Effects of Atypical Low Potency Antipsychotic drugs Dry mouth

Blurred vision

Urinary retention

Constipation

Long QT syndrome with the risk of torsade de pointes

Delirium in those with organic brain disease, the elderly, and others on anticholinergic medications

Lower seizure threshold

Orthostatic hypotension

www.ClinicalReview.com


Psychiatry

139

ANTIDEPRESSANTS TABLE 208 ANTIDEPRESSANT MEDICATIONS Antidepressant Medications SSRIs

TCAs

MAOIs

Fluoxetine

Nortriptyline

Tranylcypromine

Sertraline

Imipramine

Phenelzine

Paroxetine

Desipramine

Fluvoxamine

Clomipramine

Citalopram Escitalopram TABLE 209 ATYPICAL ANTIDEPRESSANT MEDICATIONS Atypical Antidepressant Medications Buproprion

Nefazodone

Mirtazapine

Trazodone

Venlafaxine

INDICATION Antidepressants are potent drugs used to treat symptoms of major depression, anxiety disorders, obsessive-compulsive disorder, and other conditions. Antidepressants are divided into SSRIs, TCAs, MAOIs, and miscellaneous compounds with similar effects. All of the antidepressants can be used for the treatment of major depression and dysthymia. In addition, all of these medications, in addition to high doses of benzodiazepines, can be used to treat panic disorder. OCD responds well to clomipramine and to high doses of SSRIs. Obsessions in particular respond well, although as long as three months of therapy may be required in order to evaluate the medication. Further, purging bulimia nervosa is amenable to treatment by any of the antidepressants, especially SSRIs due to their fewer side effects. TABLE 210 INDICATIONS FOR ANTIDEPRESSANT MEDICATION USE Indications for Antidepressant Medication Use Major depression Anxiety disorders Obsessive-compulsive disorder Dysthymia Mental retardation Tourette syndrome

MECHANISM OF ACTION BLOCKADE OF NEUROTRANSMITTER UPTAKE

Â


USMLE STEP 2 140 Antidepressants work chiefly through effects on monoamine neurotransmitters such as serotonin, norepinephrine, and dopamine. Dopamine is released by the ventral brainstem, norepinephrine by the locus ceruleus, and serotonin by the raphe nuclei. SSRIs function by inhibiting reuptake of serotonin, thereby increasing the length of time serotonin is present in the synapticcleft. This leads to a potentiation of the serotinergic pathway. TCAs block reuptake of serotonin and norepinephrine. MAOIs block monoamine oxidase, the enzyme that breaks down each of the monoamine neurotransmitters, thereby leading to elevated levels of norepinephrine, dopamine, and serotonin. Each of these classes of medications also has long-term effects that are presently under investigation. TABLE 211 MECHANISM OF ACTION OF ANTIDEPRESSANTS Mechanism of Action of Antidepressants SSRIs work by inhibiting reuptake of serotonin – increasing time of serotonin in synapse TCA blocks reuptake of serotonin and norepinephrine MAOIs block monoamine oxidase (the enzyme which breaks down monoamine neurotransmitters)

THERAPY AND DRUG REACTIONS OVERVIEW The SSRIs include fluoxetine, sertraline, paroxetine, fluvoxamine, citalopram, and escitalopram. The TCAs include nortriptyline, imipramine, desipramine, and clomipramine. The MAOIs include tranylcypromine and phenelzine. Other antidepressants include bupropion, nefazodone, venlafaxine, mirtazapine, and trazodone. FIRST LINE MEDICATIONS The first line of medications that should be used for the treatment of depression should include venlafaxine and TCAs in order to cause remission of the depression. These medications are the drugs of choice due to their effect on the noradrenergic and serotinergic systems. A combination of bupropion and SSRIs may also be used to obtain this effect. The best-tolerated medications include venlafaxine, bupropion, mirtazapine, nefazodone, and SSRIs, which are primarily considered to be the first line medications for the treatment of major depression. These medications have fewer side effects such as orthostatic hypotension, anticholinergic effects, and sedation. Patients with cardiac conduction abnormalities, along with any condition that may be worsened with anticholinergic stimulation (such as glaucoma, prostatic hypertrophy, or constipation) should be initially started on venlafaxine, bupropion, mirtazapine, nefazodone, or SSRIs. SSRIs are typically the best choice in patients with cerebrovascular or cardiovascular defects. TABLE 212 THERAPY USING ANTIDEPRESSANT MEDICATIONS Therapy Using Antidepressant Medications First line of treatment

Venlafaxine, bupropion and SSRIs

Best tolerated medications

Venlafaxine, bupropion, mirtazapine, nefazodone, and SSRIs

Second line of treatment

TCAs (nortriptyline, desipramine have least side effects)

Depression refractory to standard treatment

Lithium carbonate. T3, methylphenidate ECT

www.ClinicalReview.com


Psychiatry

141

SECOND LINE MEDICATIONS The next line of medications is the TCAs, of which nortriptyline and desipramine have the least side effects. TCAs can be used as first line agents in younger patients, especially if cost is an issue. MAOIs should be used selectively due to the dietary restrictions and risk of orthostatic hypotension. They are the drugs of choice in patients who have failed other therapies, or in patients who have a seizure disorder – in such patients, both MAOIs and SSRIs can be used, as they do not affect the seizure threshold. Patients with atypical depression or social phobias respond best to MAOIs or SSRIs. Finally, high dose SSRIs, especially fluvoxamine and the TCA clomipramine are the drugs of choice for the treatment of OCD. ATYPICAL ANTIDEPRESSANTS (THIRD LINE) Approximately half of all patients with major depression will respond after six weeks of use of a single antidepressant medication. Therapy should continue for at least six months, with recurrent or chronic major depression treated on a lifelong basis as needed. Depression refractory to standard treatment can be treated with increasing doses of medications, adding lithium or T3, adding a psychostimulant such as methylphenidate, changing the medication, adding a second medication, or using ECT. Patients on TCAs require serum level monitoring to determine proper dosing regimens. An antidepressant such as bupropion may be appropriate in patients presenting with depression and a history of childhood ADHD. SIDE EFFECTS SSRIS AND MAOIS TABLE 213 SIDE EFFECTS OF SSRI, MAOI, AND TCA THERAPY Side Effects of SSRI, MAOI, and TCA Therapy SSRI Therapy Nausea

Headache

Akathisia

Insomnia

Sedation

Lack of orgasm or ejaculation

Cannot be combined with MAOIs due to development of fatal serotonin syndrome MAOI Therapy Hyperadrenergic crisis due to excessive tyramine ingestion Severe hypertension with subsequent myocardial infarction or stroke, Dose-related orthostatic hypotension Certain foods containing tyramine must be avoided Over-the-counter medications must also be avoided, especially pain and cold medicines Tranylcypromine can cause insomnia and agitation Phenelzine can cause daytime sleepiness TCA Therapy Orthostatic hypotension Anticholinergic effects including constipation, dry mouth, blurred vision, difficulty urinating, hallucinations, seizures, akathisia, agitation, delirium, sexual dysfunction, cardiac toxicity, sinus & ventricular tachycardia, arrhythmias

Â


USMLE STEP 2 142 Side effects of SSRIs include nausea, headache, akathisia, insomnia or sedation, and lack of orgasm and ejaculation. SSRIs cannot be combined with MAOIs due to the development of a fatal serotonin syndrome. Side effects of MAOIs include hyperadrenergic crisis from excessive tyramine ingestion, severe hypertension with subsequent myocardial infarction or stroke, and a dose-related orthostatic hypotension. Certain foods containing tyramine must be avoided when on MAOIs, including cured meats, fish, beer, red wine, and all types of cheese except cream cheese and cottage cheese, and overripe fruits. Over-the-counter medications must also be avoided, especially pain and cold medicines. The development of malignant hypertension can be treated with intravenous phentolamine, an alpha blocker, or nitroprusside infusion. The MAOI tranylcypromine can cause insomnia and agitation, while phenelzine can cause daytime sleepiness. TCAS TCAs can cause orthostatic hypotension, anticholinergic effects, and sexual dysfunction. Cardiac toxicity can also occur. Elderly patients in particular are susceptible to the orthostatic hypotension that can occur. The anticholinergic effects appear to be centered on constipation, dry mouth, blurred vision, and difficulty with urination. More severe effects include hallucinations, seizures, akathisia, agitation, and delirium. Sinus tachycardia, ventricular tachycardia, ventricular fibrillation, and other arrhythmias are all possible with TCAs, and these medications should be avoided in patients with notable heart disease. Sexual dysfunction centers on impotence in males and decreased arousal in women. The major side effects of TCAs can be remembered with the mnemonic Tri-C’s, i.e. convulsions, cardiac arrhythmias, and coma. ATYPICAL ANTIDEPRESSANTS Venlafaxine is a serotonin and noradrenergic reuptake inhibitor often used for the treatment and induction of remission of major depression. Venlafaxine has fewer side effects than other antidepressants, and is a good first line agent. Trazodone is often given with an SSRI to induce sleep. A side effect of trazodone is priapism. Nefazodone has less sedation and a smaller incidence of priapism. OTHER ANTIDEPRESSANTS Buproprion is effective in smoking cessation and in ADHD, but can lower the seizure threshold. Mirtazapine is highly sedating. TABLE 214 SIDE EFFECTS OF ATYPICAL ANTIDEPRESSANTS Side Effects of Atypical Antidepressants Trazodone

Priapism

Buproprion

Can lower the seizure threshold

Mirtazapine

Highly sedating

PHOTOTHERAPY INDICATIONS The use of bright lights to treat seasonal major depression is highly effective. Phototherapy is also useful in delayed sleep phase syndromes, with early morning bright light therapy more effective than evening light. Phototherapy has been known to induce mania in certain individuals.

www.ClinicalReview.com


Psychiatry

143

ELECTROCONVULSIVE THERAPY INDICATIONS AND SIDE EFFECTS ECT is a highly effective treatment for major depression refractory to other medications. Anesthetics and paralytics are used to block memory formation and minimize the motor component of seizure activity during ECT sessions. Side effects include temporary short-term memory loss and confusion. Bilateral ECT is more effective than unilateral therapy, but has more side effects.

MOOD STABILIZERS TABLE 215 TYPICAL MOOD STABILIZER MEDICATIONS Typical Mood Stabilizer Medications Lithium carbonate Valproate Carbamazepine TABLE 216 ATYPICAL MOOD STABILIZER MEDICATIONS Atypical Mood Stabilizer Medications Calcium-channel blockers Benzodiazepines Antipsychotics Lamotrigine Gabapentin

INDICATIONS Mood stabilizers include lithium, valproate, and carbamazepine. Calcium-channel blockers, benzodiazepines, and antipsychotics have also been used to manage bipolar disorder refractory to other treatment. Mood stabilizers are the drugs of choice for mania. They are used for long-term maintenance of bipolar disorder. Valproate and carbamazepine are also used in patients with a seizure component. Impulsive behaviors can also be controlled with the mood stabilizers. The latter two medications are especially effective in managing rapid cyclers. Lithium can be used to further augment antidepressant medications. TABLE 217 INDICATIONS FOR MOOD STABILIZERS Indications for Mood Stabilizers Mania Long-term maintenance of bipolar disorder Mood symptoms with a seizure component Impulsive behavior Effective in managing rapid cyclers

Â


144

USMLE STEP 2

MECHANISMS OF ACTION CAMP AND G-PROTEIN EFFECTS The mechanism of action of lithium is through modification of cyclic adenosine monophosphate (cAMP) and G-proteins. Lithium works as an ion and travels through certain ion channels. Through these effects, lithium is known to affect serotonin, norepinephrine (both through G-protein mediated effects) and GABA metabolism. GABA POTENTIATION Valproate functions by supplementing the role of GABA through increased GABA synthesis, decreased breakdown of GABA, and increased potentiation at the synapse. SODIUM CHANNEL BLOCKADE Carbamazepine functions by blocking sodium channels following depolarization, thereby preventing repolarization of the neuron and aborting future synaptic potentials. Carbamazepine also reduces the release of neurotransmitter into the synaptic cleft, thereby preventing the firing of the postsynaptic neuron. TABLE 218 MECHANISM OF ACTION OF MOOD STABILIZERS Mechanism of Action of Mood Stabilizers Lithium

Modification of cAMP and G-Proteins, also travels through certain ion channels

Valproate

Increases GABA synthesis, decreases breakdown of GABA, increases potentiation at synapse

Carbamazepine, lamotrigine

Blocks sodium channels, reduces release of NTS

LITHIUM INDICATIONS Lithium is the first line treatment as a mood stabilizer for bipolar disorder that does not rapidly cycle. Lithium may be used in any patient who does not have a compromise of their renal function, as it can quickly reach toxic levels if it is not readily cleared. Lithium is also used as an adjunct to antidepressant treatment in major depression. The use of lithium requires frequent monitoring, and also regular blood tests to monitor thyroid stimulating hormone (TSH), creatinine, and other thyroid and renal function tests. TABLE 219 THERAPY USING MOOD STABILIZER MEDICATIONS Therapy Using Mood Stabilizer Medications Lithium

First line of treatment for bipolar treatment that doesn’t rapidly cycle

Valproate

Drug of choice for bipolar rapid cyclers; can control impulsivity with carbamazepine

Carbamazepine

Second line medication for mania

Lamotrigine

Mood stabilizer and antidepressant

Gabapentin

Inhibits seizure disorders, mood stabilizer, and anxiolytic for bipolar disorder. Cannot be used alone www.ClinicalReview.com


Psychiatry

145

SIDE EFFECTS The side effects of lithium include tremor, gastrointestinal upset, polyuria, some deficits in memory, worsening of acne, and increase in weight. Thyroid function tests must be regularly conducted as lithium interferes with hormone production, leading to hypothyroidism. As lithium has a narrow therapeutic index, it must be regularly monitored to ensure that it does not rise to toxic levels. It is for this reason that lithium is contraindicated in patients with renal disease. If lithium rises to toxic levels, its side effects can lead to ataxia, confusion, sinus arrest, coma, and death. TABLE 220 SIDE EFFECTS OF LITHIUM CARBONATE Side effects of Lithium Carbonate Cannot be used in patients with: Tremor

Memory deficits

GI upset

Increases existing acne problems

Polyuria

Increased weight

Compromised renal function

VALPROATE INDICATIONS Valproate is a medication often used in the treatment of bipolar disorder, and as maintenance. Valproate is the drug of choice for rapid cyclers and for mixed bipolar disorder. It has little function against depression or the depressive elements of bipolar disorder. Valproate is used along with carbamazepine to control impulsivity. SIDE EFFECTS Side effects of valproate are many, with the most serious being thrombocytopenia, impaired platelet activity, fulminant hepatic or pancreatic failure, and agranulocytosis. Other side effects are similar to lithium, with tremor, gastrointestinal distress, and some sedation. TABLE 221 SIDE EFFECTS OF VALPROATE Side Effects of Valproate Thrombocytopenia

Impaired platelet activity

Hepatic or pancreatic failure

Agranulocytosis

Similar side effects as lithium with tremor, GI distress, and some sedation

CARBAMAZEPINE INDICATIONS AND SIDE EFFECTS Carbamazepine is a second-line medication for mania. It is often used to treat acute mania and is effective in rapid cyclers and mixed forms of mania. Side effects include those similar to lithium, also nausea, rash, and leucopenia. Toxic doses can lead to

Â


USMLE STEP 2 146 depression of the respiratory system, heart block, autonomic dysfunction, and coma. Carbamazepine has also been known tocause agranulocytosis, pancytopenia, and occasionally, aplastic anemia. Carbamazepine levels must be regularly monitored. TABLE 222 SIDE EFFECTS OF CARBAMAZEPINE Side Effects of Carbamazepine Similar to lithium

Agranulocytosis

Nausea

Depression of respiratory system

Rash

Leucopenia

Heart block

Autonomic dysfunction

Coma

Pancytopenia

Occasionally aplastic anemia

OTHER MOOD STABILIZERS LAMOTRIGINE Lamotrigine is a new mood stabilizer originally used as an anticonvulsant. Lamotrigine inhibits sodium channels and thereby stabilizes neuronal membranes. Lamotrigine functions as both a mood stabilizer and antidepressant, and is a good drug to use after a patient fails with the traditional mood stabilizers (lithium, valproate, and carbamazepine). Lamotrigine can cause StevensJohnson syndrome, so its dosage should be gradually increased. Physicians should also watch for ataxia, blurred vision, dizziness, nausea and vomiting, and rash. TABLE 223 SIDE EFFECTS OF LAMOTRIGINE Side Effects of Lamotrigine Stevens-Johnson syndrome

Rash

Ataxia

Blurred vision

Dizziness

Nausea/vomiting

GABAPENTIN Gabapentin has been used to inhibit seizure disorders and functions as a mood stabilizer and anxiolytic for bipolar disorder. Gabapentin cannot be used alone, but is a good adjunct with the traditional mood stabilizers.

ANXIOLYTICS TABLE 224 BENZODIAZEPINES Benzodiazepines Alprazolam

Chlordiazepoxide

Clonazepam

Diazepam

Flurazepam

Lorazepam

Oxazepam

Temazepam

Triazolam

www.ClinicalReview.com


Psychiatry

147

INDICATIONS BENZODIAZEPINES Benzodiazepines are the primary treatment for anxiety disorders and for anxiety features of other psychiatric disorders. Benzodiazepines have also been used for the short-term treatment of insomnia, alcohol withdrawal, mania, catatonia, and dementia. TABLE 225 INDICATIONS FOR USE OF BENZODIAZEPINES Indications for Use of Benzodiazepines Anxiety disorders

Anxiety features of other psychiatric disorders

Insomnia

Alcohol withdrawal

Mania

Catatonia

Dementia

BUSPIRONE Buspirone is a non-benzodiazepine that has been used for the treatment of generalized anxiety disorder. MECHANISMS OF ACTION GABA-A AGONISM AND 5HT1A / D2 ANTAGONISM Benzodiazepines function as an agonist to GABA-A receptors through a chloride ion channel-mediated effect and have immediate action. On the other hand, buspirone functions through positive serotonergic effects at the 5HT1A receptor and through D2 receptor antagonism. Buspirone is slow in onset and is useful only for the treatment of generalized anxiety disorder. THERAPY AND DRUG REACTIONS EFFICACY AND METABOLISM The benzodiazepines include alprazolam, chlordiazepoxide, clonazepam, diazepam, flurazepam, lorazepam, oxazepam, temazepam, and triazolam. The high-potency benzodiazepines include alprazolam, clonazepam, triazolam, and also lorazepam. The fastest onset is with diazepam, flurazepam, and triazolam. The slowest onset is with oxazepam. Metabolism of all the benzodiazepines is through oxidation except for lorazepam, oxazepam, and temazepam, which are metabolized through conjugation. Alprazolam, chlordiazepoxide, diazepam, flurazepam, and triazolam all have active metabolites. INDICATIONS – ANXIETY, DETOXIFICATION, INSOMNIA, CATATONIA Alprazolam and clonazepam are high-potency benzodiazepines used for the treatment of panic disorder. Alprazolam, clonazepam, diazepam, and lorazepam are indicated for the treatment of anxiety. Alcohol detoxification can be conducted by chlordiazepoxide and oxazepam. Insomnia is best treated with flurazepam, triazolam, diazepam, and temazepam. Finally, catatonia can be treated by lorazepam.

Â


148

USMLE STEP 2

TABLE 226 THERAPY USING BENZODIAZEPINES Therapy Using Benzodiazepines High potency

Alprazolam, clonazepam, triazolam, temazepam

Fastest onset

Diazepam, flurazepam, triazolam

Slowest onset

Oxazepam

Panic disorder

Alprazolam, clonazepam

Anxiety

Alprazolam, clonazepam, diazepam, lorazepam

Alcohol detoxification

Chlordiazepoxide, oxazepam

Insomnia

Flurazepam, triazolam, diazepam, temazepam

Catatonia

Lorazepam

INDICATIONS – PANIC DISORDER Benzodiazepines with a rapid rate of onset can lead to “highs” and are more addictive. The benzodiazepines that are metabolized through oxidation require a highly functioning liver and the presence of cirrhosis or other liver dysfunction can lead to toxic accumulation of benzodiazepines and their metabolites. Medications with longer half-lives, such as clonazepam, are favored for the treatment of panic disorder due to a longer duration of therapeutic effect once the medication has reached a therapeutic level. INDICATIONS – GENERALIZED ANXIETY DISORDER Buspirone is not as effective as benzodiazepines for the treatment of generalized anxiety disorder and its long onset of effect make it a difficult drug to use in patients with severe symptoms. However, buspirone is an attractive option in patients who are addicted to benzodiazepines. No routine monitoring is required of either benzodiazepines or buspirone. SIDE EFFECTS Side effects of benzodiazepines include sleepiness, a paradoxical disinhibition especially in the elderly, carbon dioxide retention to a lethal extent in patients with intercurrent respiratory ailments, and death through overdose when combined with alcohol and barbiturates, as these compounds also work through the same GABA-A receptor. Buspirone can cause nausea, a paradoxical anxiety, dizziness, and a severe withdrawal syndrome if abused. TABLE 227 SIDE EFFECTS OF BENZODIAZEPINES Side Effects of Benzodiazepines Sleepiness

Paradoxical disinhibition, especially in the elderly

Potentially lethal carbon dioxide retention

Overdose death when combined with alcohol and barbiturates

Atypical anxiolytic Buspirone can cause nausea, paradoxical anxiety, dizziness, and severe withdrawal syndrome

www.ClinicalReview.com


Psychiatry

149

OTHER MEDICATIONS TABLE 228 PSYCHOSTIMULANTS Psychostimulants Dextroamphetamine Methylphenidate Pemoline

PSYCHOSTIMULANTS INDICATIONS AND SIDE EFFECTS Psychostimulants have been used for ADHD, narcolepsy, and for depression refractory to other medical intervention. These medications alter neurotransmitter release. Abuse can lead to tolerance and side effects present in other amphetamines. The most commonly used psychostimulants are dextroamphetamine, methylphenidate, and pemoline. Pemoline in particular is tied to an increased rate of hepatic failure. TABLE 229 INDICATIONS FOR USE OF PSYCHOSTIMULANTS Indications for Use of Psychostimulants ADHD Depression in the elderly Depression refractory to other medical intervention Narcolepsy TABLE 230 SIDE EFFECTS OF PSYCHOSTIMULANTS Side Effects of Psychostimulants Pemoline

Hepatic failure

ANTICHOLINERGICS CLASSIFICATION TABLE 231 ANTICHOLINERGICS Anticholinergics Typical anticholinergics:

Benztropine, trihexyphenidyl

Atypical anticholinergic:

Diphenhydramine (antihistamine)

Treatment for:

Depression refractory to other medical intervention, and narcolepsy

Â


USMLE STEP 2 150 INDICATIONS AND SIDE EFFECTS Anticholinergics have been used for the treatment of iatrogenic Parkinsonism and iatrogenic dystonic reactions. Benztropine and trihexyphenidyl are the most commonly used anticholinergics. Diphenhydramine has also been used, although this particular drug is an antihistamine. Anticholinergics and diphenhydramine are muscarinic receptor antagonists. Side effects are generally excessive anticholinergic activity, such as urinary retention, constipation, blurry vision, sedation, and delirium in elderly. TABLE 232 THERAPY USING ANTICHOLINERGICS Therapy Using Anticholinergics Iatrogenic Parkinsonism Iatrogenic dystonic reactions TABLE 233 SIDE EFFECTS OF ANTICHOLINERGICS Side Effects of Anticholinergics Urinary retention

Constipation

Sedation

Delirium

Blurred vision

BETA-BLOCKERS INDICATIONS AND SIDE EFFECTS Beta-blockers are used for the treatment of akathisia, performance anxiety, impulsivity, and lithium-induced tremors. Betablockers alter catecholamine function by diminishing central nervous system arousal, tachycardia, tremor, diaphoresis, and hyperventilation. Side effects are excessive decrease in the sympathetic nervous system, and also masking of diabetes. Depression-like symptoms may also be present in overdose. TABLE 234 INDICATIONS FOR USE OF BETA-BLOCKERS Indications for Use of Beta-Blockers Akathisia

Performance anxiety

Impulsivity

Lithium-induced tremors

TABLE 235 SIDE EFFECTS OF USE OF BETA-BLOCKERS Side Effects of Use of Beta-Blockers Excessive decrease in sympathetic nervous system Masks diabetes Overdose may cause depression-like symptoms

CLONIDINE INDICATIONS AND SIDE EFFECTS

www.ClinicalReview.com


Psychiatry

151

Clonidine has been used as a treatment of impulsivity, for Tourette syndrome, and opiate withdrawal. Clonidine is an alpha2 adrenergic agonist that blocks the release of norepinephrine. It also has been used as an antihypertensive agent. Side effects include overdose with consequential hypotension, dizziness, and sedation. TABLE 236 CLONIDINE Clonidine Treatment for: Impulsivity

Tourette syndrome

Opiate withdrawal

Antihypertensive

Side effects: Hypotension

Dizziness

Sedation

COGNITIVE ENHANCERS INDICATIONS AND SIDE EFFECTS Donepezil and tacrine have been used for memory loss in Alzheimer disease. These medications reversibly inhibit acetylcholinesterase (AChE) and supplement the loss of cholinergic neurons in Alzheimer disease. Side effects include gastrointestinal upset, bradycardia, and increased gastric acid secretion. Tacrine has been shown to increase liver function enzymes. TABLE 237 COGNITIVE ENHANCERS Cognitive Enhancers Donepezil and tacrine

Treatment for: Memory loss in Alzheimer’s disease (AD)

Side effects: GI upset

Bradycardia

Increased gastric secretion

Tacrine increases liver function enzymes

THYROID HORMONES INDICATIONS AND SIDE EFFECTS Thyroid hormones are used to augment therapy for major depression and for the treatment of rapid cycling bipolar disorder. Hyperthyroidism is the only major side effect. TABLE 238 THYROID HORMONES Thyroid Hormones Augment treatment for:

Major depression and rapid cycling bipolar disorder

Side effects:

Hyperthyroidism


152

USMLE STEP 2

MAJOR ADVERSE DRUG EFFECTS EXTRAPYRAMIDAL SYMPTOMS RISK FACTORS AND ONSET Neuroleptic-induced Parkinsonism has classic symptoms of Parkinsonism, including lead-pipe rigidity, akinesia, and a fine tremor in the muscles of the head, face, and limbs. Risk factors for extrapyramidal symptoms include high-potency or high dose antipsychotics, being elderly, and having a history of extrapyramidal symptoms (EPS). Onset is typically within a few weeks of starting treatment. Extrapyramidal symptoms can be controlled by anticholinergics as mentioned above, or by lowering or changing the antipsychotic medication. TABLE 239 RISK FACTORS FOR EPS Risk Factors for EPS High potency or high dose of antipsychotics

Old age

History of EPS

CLASSIFICATION Extrapyramidal symptoms that occur within four hours are classified as acute dystonia; after four days are akinesia; after four weeks are akathisia; and after four months are tardive dyskinesia. Treat acute dystonia with benztropine or diphenhydramine, prevent it with benztropine. Treat akathisia with propranolol and decrease neuroleptics; consider benzodiazepines and anticholinergics instead. Treat dyskinesia with benztropine or amantadine, and decrease the dosage of neuroleptics. Treat tardive dyskinesia by stopping neuroleptics. Administering anticholinergics or decreasing neuroleptics may initially worsen tardive dyskinesia. TABLE 240 CLASSIFICATION OF EPS Classification of EPS Occurrence within 4 hours

Acute dystonia

Occurrence after 4 days

Akinesia

Occurrence after 4 weeks

Akathisia

Occurrence after 4 months

Tardive dyskinesia

TABLE 241 TREATMENT OF EPS Treatment of EPS Acute dystonia

Benztropine, diphenhydramine, prevent with benztropine

Akinesia

Benztropine, amantadine, decrease dosage of neuroleptics

Akathisia

Propranolol and decrease neuroleptics; consider benzodiazepines and anticholinergics

Tardive dyskinesia

Discontinuation of neuroleptics

DYSTONIA

www.ClinicalReview.com


Psychiatry

153

SYMPTOMS AND NSET Dystonic reactions are muscle spasms that can affect any muscle in the body, but predominantly center on the muscles of the head and neck. There is a broad range of spasms, including some increased muscle tension to severe tetany with resultant compromise of the airway. Spasms, posturing, dysarthria, and impaired bodily functions are all characteristics of more severe dystonic reactions. Dystonic reactions are most likely to occur after the use of high-potency or high dose antipsychotics, and are more common in younger men. Onset is usually within a few days of starting therapy. Treatment is generally supportive care. Intravenous benztropine (anticholinergic) or diphenhydramine (antihistamine with anticholinergic effects) may be indicated, and discontinuation of the offending agent is usually normal practice, if the symptoms cannot be controlled. TABLE 242 TREATMENT OF DYSTONIA Treatment of Dystonia Supportive care Intravenous benztropine or diphenhydramine Discontinue offending drug

AKINESIA SYMPTOMS AND ONSET Akinesia typically occurs within 4 days of use of neuroleptic agents. Presentation of akinesia is with difficulty beginning or continuing a motion. There is a marked slowing of voluntary movements, a retardation of initiating movements such as reaching for objects, an inability to repeat stereotyped motions such as clapping hands, difficulty carrying out a sequence of motions, masked facies similar to Parkinsonism, loss of spontaneous gait motions such as swinging of the arms, a shuffling gait, and decreased dexterity. Akinesia is treated by lowering the dose of antipsychotics, changing the medication, or using clozaril as a substitute. TABLE 243 SYMPTOMS OF SEVERE AKINESIA Symptoms of Severe Akinesia Loss of voluntary movements

Loss of initiation of motion

Cannot do repetitive motions

Masked facies

Loss of spontaneous gait

Decreased dexterity

TABLE 244 TREATMENT OF AKINESIA Treatment of Akinesia Reduce dose of offending medication Change offending medication Use clozaril as a substitute

AKATHISIA SYMPTOMS AND ONSET

Â


USMLE STEP 2 154 Akathisia is restlessness with anxiety or agitation. Akathisia is a side effect of increasing doses of antipsychotics and SSRIs. Akathisia, when severe, can lead to dysphoria, aggression, and suicide. Cases of akathisia tend to occur within the first few weeks of increasing the dose of a medication. It may be treated by reducing the dose of the offending medication, administering beta blockers such as propranolol, benzodiazepines such as lorazepam, and possibly anticholinergics as mentioned above. Akathisia should not be confused with worsening psychosis. TABLE 245 SYMPTOMS OF SEVERE AKATHISIA Symptoms of Severe Akathisia Dysphoria

Aggression

Suicide

TABLE 246 TREATMENT OF AKATHISIA Treatment of Akathisia Reduce dose of offending medication Beta-blockers such as propranolol Lorazepam

TARDIVE DYSKINESIA SYMPTOMS AND ONSET Tardive dyskinesia (TD) develops over a long period of time of use of antipsychotics. It is more likely to occur in the elderly, females, African Americans, and in patients with mood disorders. Tardive dyskinesia is characterized by choreaform motions, as in Huntington disease, especially of the head and neck muscles. Tardive dyskinesia requires lowering the dose of antipsychotics or changing the medication, and using clozaril instead. Clozaril is an atypical antipsychotic that works through a different mechanism and avoids tardive dyskinesia entirely. Tardive dyskinesia may be permanent. TABLE 247 TREATMENT OF TARDIVE DYSKINESIA Treatment of Tardive Dyskinesia Lowering dose of antipsychotics or changing medication and using clozaril instead TABLE 248 CLASSIFICATION OF DYSTONIC REACTIONS Classification of Dystonic Reactions Acute Dystonia

Akathisia

Akinesia

Tardive Dyskinesia

Time Course

4 hours

4 weeks

4 days

4 months

Minor Signs

Muscle spasms of the head and neck

Anxiety and agitation

Loss of spontaneous motions

Major Signs

Airway compromise

Dysphoria, aggression, suicide

Masked facies, immobility

Choreaform motions. Permanent.

Treatment

Benztropine, diphenhydramine

Propranolol, benzodiazepines, lorazepam

Clozaril as a substitute

Clozaril as a substitute

www.ClinicalReview.com


Psychiatry

155

NEUROLEPTIC MALIGNANT SYNDROME CAUSES AND SYMPTOMS Neuroleptic malignant syndrome (NMS) is a medical emergency that requires prompt intervention. It is a complication of highdose or high-potency antipsychotics, an effect of rapid dose increases, intramuscular injection of antipsychotics, occurs in patients with agitation or dehydration, and in patients with a history of neuroleptic malignant syndrome. Onset is typically within a few weeks, but it can occur at any point during treatment. Neuroleptic malignant syndrome is characterized by gradual onset of autonomic, motor, behavioral, and laboratory abnormalities. Autonomic dysfunction includes tachycardia and development of cardiac arrhythmias, hypertension or hypotension, diaphoresis, and fever leading to hyperthermia / hyperpyrexia. Motor dysfunction is characterized by rigidity or dystonia, akinesia, mutism, and dysphagia. Behavioral abnormalities include agitation, incontinence, delirium, seizures, and coma. Laboratory tests typically indicate a rise in the creatine kinase (CK), abnormal liver function tests (LFTs), and an increase in white blood cells (WBCs). TABLE 249 CAUSES OF NMS Causes of NMS High does or high potency antipsychotics Rapid dose increase Intramuscular injection of antipsychotics Patients with agitation or dehydration Patients with history of NMS TABLE 250 SYMPTOMS OF NMS Symptoms of NMS Gradual onset of autonomic, motor, behavioral and laboratory abnormalities Tachycardia

Cardiac arrhythmias

Hypertension or hypotension

Diaphoresis

Fever leading to hyperthermia / hyperpyrexia

Muscle rigidity

Dystonia and akinesia

Increase in white blood cells

Mutism

Dysphagia

Rise in creatine kinase

Abnormal liver function test

TREATMENT Treatment of neuroleptic malignant syndrome must be prompt to avoid the more serious sequelae of this disorder. It is typically supportive, but includes discontinuation of the offending agent, use of dantrolene to relax the muscles and minimize myonecrosis, bromocriptine as a dopamine agonist to reverse the dopamine block from the antipsychotics, and symptomatic management. Neuroleptic malignant syndrome develops as a result of antipsychotic medications; patients on both MAOIs and antipsychotics may also develop serotonin syndrome, and distinguishing between the two can be difficult.

Â


156

USMLE STEP 2

TABLE 251 TREATMENT OF NMS Treatment of NMS Must be prompt and supportive Discontinue offending medicine Dantrolene Bromocriptine (dopamine agonist) Symptomatic management

SEROTONIN SYNDROME CAUSES AND SYMPTOMS Serotonin syndrome may occur with the combination of SSRIs and MAOIs, with MAOIs plus antipsychotics, MAOIs plus meperidine or dextromethorphan, and MAOIs plus other opiates. Serotonin syndrome can be life threatening, with autonomic, motor, and behavioral effects similar to neuroleptic malignant syndrome. Specifically, the autonomic effects include tachycardia, hypertension, diaphoresis, fever leading to hyperpyrexia; the motor effects can lead to shivering, myoclonus, tremor, hyperreflexia, and oculomotor abnormalities; and the behavioral abnormalities can include restlessness, agitation, delirium, and coma. TABLE 252 SYMPTOMS OF SEROTONIN SYNDROME Symptoms of Serotonin Syndrome Tachycardia

Hypertension

Diaphoresis

Fever leading to hyperpyrexia

Shivering

Cyclones

Tremor

Hyperreflexia

Oculomotor abnormalities

Restlessness

Agitation

Delirium and coma

DIFFERENTIAL DIAGNOSIS Serotonin syndrome can be differentiated from neuroleptic malignant syndrome in that serotonin syndrome does not produce the muscular rigidities seen in neuroleptic malignant syndrome. There is also no dystonic reaction in serotonin syndrome – the muscular effects tend to lead to increased muscle activity via tremor, shivering, and increased reflexes. TREATMENT Treatment for serotonin syndrome is typically supportive with discontinuation of the offending agents.

www.ClinicalReview.com


Psychiatry

157

ANTIDEPRESSANTS – SSRIS DRUG Citalopram

MECHANISM OF ACTION

INDICATIONS Major depression

COMPLICATIONS

Selective serotonin reuptake inhibitor

Fluoxetine

Major depression

Selective serotonin reuptake inhibitor

Sertraline

Major depression

Selective serotonin reuptake inhibitor

CONTRAINDICATIONS Avoid in patients taking MAOIs

CNS stimulation, anorexia, N/V, insomnia, sexual dysfunction, and serotonin syndrome with MAOI

Avoid in patients taking MAOIs Avoid in patients taking MAOIs and disulfiram

ANTIDEPRESSANTS – TCAS DRUG Clomipramine

Imipramine

Nortriptyline

INDICATIONS

MECHANISM OF ACTION

COMPLICATIONS

OCD Major depression Enuresis Cataplexy Major depression

CONTRAINDICATIONS Avoid in patients taking MAOIs

Block NE and serotonin reuptake

Convulsions, CV toxicity, coma, CNS depression, hallucinations in elderly (a ticholinergic effect), hyperthermia

Avoid in patients taking MAOIs, renal or hepatic failure, or after MI Avoid in patients taking MAOIs, or after MI. Avoid in children and pregnant women.

Major depression

ANTIDEPRESSANTS – MAOIS DRUG

INDICATIONS

Phenelzine

Major depression

Tranylcypromine

Major depression

MECHANISM OF ACTION

MAOA and MAOB inhibition with increase in NE and catecholamines

COMPLICATIONS

Hypertensive crisis with tyramine or meperidine, CNS stimulation, hepatotoxicity, postural hypotension

CONTRAINDICATIONS Avoid in CHF, liver disease, with tyramine, with dopamine, with amphetamines, with guanethidine, and in pheochromocytoma Avoid with alcohol, amphetamines, antihypertensives, diuretics, cheese, tyramine


USMLE STEP 2

158

ANTIDEPRESSANTS – ATYPICAL DRUG

INDICATIONS

MECHANISM OF ACTION

COMPLICATIONS

CONTRAINDICATIONS

Bupropion

Nicotine addiction ADHD, depression.

Unknown

CNS and CV stimulation, xerostomia, worsening of psychosis.

Seizure disorder, MAOI

Trazodone

Major depression

Inhibits serotonin reuptake.

Priapism, sedation, postural hypotension.

MI

Venlafaxine

Major depression GAD

Inhibits serotonin and dopamine reuptake.

Anxiety, insomnia.

MAOI, HTN. Monitor BP.

ANTIPSYCHOTICS – TYPICAL DRUG

INDICATIONS

COMPLICATIONS

CONTRAINDICATIONS

Chlorpromazine

Schizophrenia

Coma, CNS depressants, pregnancy.

Fluphenazine

Schizophrenia

May obscure Reye syndrome, vomiting.

Coma, CNS, BMS, high extrapyramidal toxicity

Haloperidol

Schizophrenia Delirium Tourette disorder

TD, NMS. Care with lithium.

Coma, CNS depression, Parkinson.

Thioridazine

Schizophrenia

TD, NMS. Heart disease.

Coma, CNS depression.

TD, NMS, Schizophrenia

ANTIPSYCHOTICS - –TYPICAL DRUG

INDICATIONS

MECHANISM OF ACTION

COMPLICATIONS

Clozapine

Schizophrenia

Dopamine antagonist

Quetiapine

Schizophrenia

Antagonist at multiple sites

TD, NMS

Risperidone

Schizophrenia

Dopamine and serotonin antagonist

NMS, TD, hyperglycemia

Agranulocytosis,

CONTRAINDICATIONS BMS.

MPD, epilepsy, CNS depression, BMS.

ANXIOLYTICS – TYPICAL DRUG

Alprazolam

Chlordiazepoxide

INDICATIONS Anxiety disorder Panic disorder DTs Anxiety disorder

MECHANISM OF ACTION

COMPLICATIONS

Seizure GABAA receptor agonist that leads to increased frequency of chloride channel opening to decrease synaptic activity CNS depressant.

www.ClinicalReview.com


Psychiatry

159

ANXIOLYTICS – ATYPICAL DRUG

INDICATIONS

MECHANISM OF ACTION

GAD Anxiety disorder

Buspirone

5HT1A and D2 antagonist

COMPLICATIONS

NOTES Low addiction potential. No interaction with ETOH.

CNS depressant.

No rebound insomnia. NIghtmares, headache, GI Sx, dizziness,

Anxiolytic

Zolpidem

No tolerance with use. No anticonvulsant or muscle-relaxing properties.

BARBITURATES DRUG

INDICATIONS Anxiolytic Hypnotic Sedative

Phenobarbital

MECHANISM OF ACTION Facilitates GABAA and increase length of chloride channel opening to decrease neuronal firing

COMPLICATIONS

CONTRAINDICATIONS

Induce tolerance Low therapeutic index Respiratory depression Rarely used

Severe withdrawal

MOOD STABILIZERS – TYPICAL DRUG

INDICATIONS

Carbamazepine

Lithium carbonate

Valproate

MECHANISM OF ACTION

Bipolar I disorder

Blocks Na+ channels to reduce NT release

Major depression Bipolar I disorder

Modifies cAMP and Gproteins to affect serotonin, norepinephrine, and GABA metabolism.

Bipolar I disorder

Increases GABA synthesis, decreases GABA metabolism

COMPLICATIONS

CONTRAINDICATIONS BMS, hypersensitivity to TCAs or MAOIs, pregnancy, hematologic reactions.

Agranulocytosis.

Renal disease, CV disease, pregnancy, young children. Hepatotoxicity, pancreatitis, somnolence, TCP.

Hepatic disease.

MOOD STABILIZERS – ATYPICAL DRUG

INDICATIONS

MECHANISM OF ACTION

Bipolar I disorderSome childhood disorders Partial seizures

Lamotrigine

Sodium ion channels

COMPLICATIONS Rash,

SJS, multiorgan failure.

PSYCHOSTIMULANTS DRUG

INDICATIONS

CONTRAINDICATIONS

Dextroamphetamine

Depression

CV disease, HTN, hyperthyroidism, glaucoma.

Methylphenidate

Depression

Anxiety, glaucoma, tics, Tourette.


160

USMLE STEP 2

www.ClinicalReview.com


Psychiatry

161

CHAPTER CONTENTS Introduction ......................................................................................................162 Autonomy .........................................................................................................162 Nonmaleficence ................................................................................................162 Beneficence.......................................................................................................162 Ethical Decision-Making ....................................................................................163 Illness in Minors ................................................................................................163 Noncompliance .................................................................................................163 Demanding Patients..........................................................................................163 Angry Patients ...................................................................................................163 Patient-Physician Relationships ........................................................................164 Informed Consent .............................................................................................164 Advanced Directives .........................................................................................165 Confidentiality...................................................................................................166

Â


162

USMLE STEP 2

MEDICAL ETHICS MEDICAL ETHICS INTRODUCTION A number of concepts dealing with medical ethics are tested. You can expect up to a dozen questions asking you to use your clinical decision making capability to arrive at the best course of action available to the physician in response to a particular case scenario. Concepts on autonomy, informed consent, and advanced directives are virtually guaranteed to be tested, as these are encountered on a regular basis in clinical practice. The physician’s response to a particular ethical situation is also common on the exam. This section on medical ethics will start with a definition of common terms, followed by discussions of informed consent, advanced directives, confidentiality, and malpractice. The last section delineates the appropriate response of a physician to various hypothetical patient case presentations.

GENERAL CONCEPTS AUTONOMY Autonomy is defined as the recognition that patients are individuals with their own preferences, and that the physician should make efforts to honor the patient’s right to request or refuse medical care whenever possible. Autonomy is tested on the USMLE by testing the limits of when it is appropriate to listen to a patient and follow their requests or when ancillary measures must be taken, such as receiving parental permission or a court order in order to take action. For example, minors (those less than eighteen years of age) have autonomy in treatment for sexually transmitted diseases. No parental consent is required, and minors have the right to refuse care for their illness. Minors also have autonomy in treatment for substance abuse, birth control, and prenatal care. However, parental permission must be obtained before conducting an abortion. TABLE 253 MINOR AUTONOMY Minor Autonomy Sexually transmitted disease

Substance abuse

Birth control

Prenatal care

NONMALEFICENCE Nonmaleficence is more colloquially known as “do no harm.” This is obviously a relative consideration, as certain therapies for cancer may cause a number of dangerous treatment-related side effects, but the therapy should proceed as scheduled as not treating the patient at all is almost certainly going to lead to death.

BENEFICENCE Beneficence is a related concept in which physicians are required to act in the patient’s best interest when no contradictory decision otherwise exists. This is the default action a physician should always take in the absence of other information. In this manner, the physician is acting as a fiduciary for the patient. Patient autonomy supersedes beneficence in almost every situation

www.ClinicalReview.com


Medical Ethics

163 and it is very difficult ethically and legally to institute positive treatment for a patient against her will. The few exceptions to this are when the patient is a danger to themselves or to others.

ETHICAL DECISION MAKING The USMLE may present you with a number of situations where you are required to use your clinical judgment to decide what the appropriate response would be. The question writers follow very specific guidelines regarding the appropriate ethical and moral conduct of a physician. Keeping the concepts of autonomy, informed consent, decision-making capacity, nonmaleficence, beneficence, confidentiality, and malpractice in mind should clarify what the appropriate decision should be in a particular case. TABLE 254 CONCEPTS TO KEEP IN MIND WHEN MAKING ETHICAL DECISIONS Concepts to Keep in Mind When Making Ethical Decisions Autonomy

Informed consent

Decision-making capacity

Beneficence

Confidentiality

Malpractice

Nonmaleficence

ILLNESS IN MINORS Discussing illnesses with children should always be done after a thorough discussion with the parent. The physician must first understand what the parent has told the child, and inform the child in accordance with the parent’s wishes. Although minors are autonomous with regard to treatment for STDs, birth control, prenatal care, and treatment for substance abuse, they are not permitted to have an abortion without permission of the parent except in an emergency.

NONCOMPLIANCE In situations where the patient is noncompliant with his medical care, such as not taking a necessary prescription, the best course of action is to identify the basic issue that prevents compliance. In response to most case-scenarios, the best way to proceed is to work to improve the trust in the physician-patient relationship. In situations where the patient is attempting to be compliant but has difficulty with maintaining the proper schedule, written directions should be provided to the patient. If appropriate, and within the patient’s wishes, additional help may be solicited with family members and other health caretakers.

DEMANDING PATIENTS Patients may occasionally make uncomfortable demands on the physician. Patients who request unnecessary procedures that are not medically indicated, or refuse to undertake a necessary procedure deserve to have an honest discussion with the physician. Understanding the patient’s thinking is the key to resolving these types of problems. A common course of action with patient’s requiring unnecessary cosmetic surgery is to alleviate the underlying psychological insecurity issues. Unnecessary procedures should not be performed in the interest of nonmaleficence. In this same vein, providing euthanasia to any patient is contraindicated. Physician-assisted suicide is not medically acceptable. However, in the interest of beneficence, the physician may provide strong pain relieving medications when medically indicated, even though their administration will coincidentally lead to a decreased life span.

ANGRY PATIENTS Patients also get angry for a number of reasons in a health care setting. Anger over the amount of time spent waiting to see the doctor should be handled by apologizing to the patient for the wait. The physician is not obligated (and should not) attempt to explain the reasons for the delay. A patient upset with another physician or the level of care she has received elsewhere should


USMLE STEP 2 164 be handled carefully. It is not professionally appropriate to discuss this situation with the patient; instead, the patient should be encouraged to speak directly with the individual that caused his grievance. In cases where the patient is upset with a member of your staff, the patient should be told that you will personally speak with that individual and address the patient’s concerns.

PATIENT-PHYSICIAN RELATIONSHIPS Finally, patients may state that they find you attractive and that they are interested in a romantic relationship with you. While standards on this vary from place to place (and are often left to the discretion of the physician but never advisable from a legal standpoint), for the purposes of the USMLE, romantic relationships are never appropriate between a physician and patient. In this situation, direct, close-ended questions should be used and the presence of a chaperone may be indicated. Another situation in which a chaperone is indicated is when a male physician does a breast or pelvic exam on a female.

INFORMED CONSENT MEDICOLEGAL STANDARD Informed consent is the practice of obtaining permission in order to perform an invasive procedure. Informed consent is traditionally in the written form and can serve as a legal document. Most procedures require informed consent except when the procedure must be performed on an emergency basis and consent cannot be acquired or is impractical (treat the latter situation carefully). In order for informed consent to be successfully obtained, the patient must be educated in a clear and simple manner regarding the benefits of the procedure, the risks of the procedure, alternatives to the procedure, and the consequences of not having any procedure performed. The legal standards are further met by obtaining this consent with a discussion of the pertinent information, establishing written consent stating the patient’s agreement to the procedure offered, and gaining this consent in a manner free from coercion. TABLE 255 INFORMED CONSENT Informed Consent Patient must be educated in a clear and simple manner about: Benefits and risks of procedure. Alternatives to procedure. Consequences of not having procedure. Pertinent information. Obtaining written consent without coercion.

EXCEPTIONS TO INFORMED CONSENT Informed consent is not practically obtained in a number of circumstances, and familiarity with the exceptions to this rule are likely to be tested. Informed consent does not need to be obtained in cases where emergency treatment is required to alleviate a life-threatening condition and there is no time to discuss the benefits and alternatives; in cases where therapeutic privilege is instituted, in which information must be withheld from the patient to prevent even greater harm than would occur with the therapy (but make this decision very carefully, as therapeutic privilege is often successfully challenged in a court); when the patient lacks the cognitive or physical ability to competently make an informed decision; and in cases where the patient signs a waiver to obtaining informed consent. It is important to note that family members cannot require that a physician not inform the patient regarding their illness. In this case, the appropriate course of action is to discuss the situation in a private room with the www.ClinicalReview.com


Medical Ethics

165 family members first, determine whether informing the patient would create even greater harm, and if it does not, then moving forward to inform the patient regarding their illness. TABLE 256 EXCEPTIONS TO INFORMED CONSENT Exceptions to Informed Consent Informed consent is NOT needed when: Required to alleviate a life-threatening condition and there is no time to discuss the benefits and alternatives. Where therapeutic privilege is instituted, in which information must be withheld from the patient to prevent even greater harm than would occur with the therapy (note caution in text above). When the patient lacks the cognitive or physical ability to competently make an informed decision. Where the patient signs a waiver to obtaining informed consent.

COMPETENCY A patient’s competence is established through their sound decision-making capability. Elements of a good decision-making ability include informing the patient regarding the intervention, the patient’s decision does not repeatedly change over time, the decision appears to be consistent with other choices the patient would make, the patient does not have any psychosocial issues that complicate their ability to make an informed decision, and if the patient communicates a clear choice to the physician. TABLE 257 PATIENT COMPETENCY Patient Competency Patient competency is established: Through the patient’s sound decision-making capability. When patient is informed regarding the intervention. When patient’s decisions do not repeatedly change over time. When the decision appears to be consistent with other choices patient would make. When the patient communicate a clear choice to the physician.

ADVANCED DIRECTIVES ORAL ADVANCED DIRECTIVES Advanced directives are a guide to providing treatment to a patient when she is otherwise unable to make a choice due to an intervening medical illness. In this case, a patient’s prior oral directives take precedence, especially if the patient was able to make an informed, clear choice and communicate this lucidly to the physician ahead of time. The decision is considered valid if it is repeated over time.

WRITTEN ADVANCED DIRECTIVES Written advanced directives are used in cases where an oral directive was not established. There are two major types of written advanced directives, including living wills and durable power of attorney. Living wills direct the physician to employ only certain


USMLE STEP 2 166 life-saving measures and dictate if and when they can be withheld or withdrawn. Living wills take precedence only when there is no prior oral directive. Living wills are not flexible and are being replaced with durable power of attorney, in many cases.

DURABLE POWER OF ATTORNEY Durable power of attorney is the designation of a surrogate, usually a spouse or other family member, who makes the judgment on behalf of the patient regarding medical decisions. The legality of durable power of attorney is predicated on the designated person making a decision similar to what the patient would choose if she were competent. A durable power of attorney incorporates elements of a living will in that the patient can designate that certain measures be taken in various clinical situations. Written advanced directives can be revoked at any time by the patient. TABLE 258 ADVANCED DIRECTIVES Advanced Directives Oral advanced directives are valid when: Patient made an informed clear choice and communicated this lucidly to the physician ahead of time. Choice is repeated over time. Written advanced directives In cases where an oral directive was not established Living will: directs the physician to employ only certain life-saving measures and dictates if and when they can be withheld or withdrawn. Living wills take precedence only when there is no prior oral directive. Living wills are not flexible are being replaced with durable power of attorney Durable power of attorney: surrogate makes the judgment on behalf of the patient regarding medical decisions. The designated person must make a decision similar to what the patient would choose if s/he were competent. A durable power of attorney incorporates elements of a living will. The patient can designate that certain measures be taken in various clinical situations. Written advanced directives can be revoked at any time by the patient.

CONFIDENTIALITY HIPPA GUIDELINES As discussed previously, information flow between patients, family members, and health care team members should follow HIPPA guidelines. The health care issues involved with a patient should not be discussed with anybody except health care personnel immediately involved with care of that patient. With permission from the patient, information can be provided to family members. According to HIPPA guidelines, this discussion should take place in a private place that protects the privacy of the information; hence, discussions regarding specific patients should be undertaken only if their privacy can be protected. Identifying information should be avoided, and discussion in public places should be limited.

HIPPA PROVISIONS Confidentiality is an important cornerstone to providing medical care in a secure environment while protecting the privacy of patients. Recent regulations instituted via the Health Information Privacy Protection Act (HIPPA) dictate when and how confidentiality should operate. Generally, disclosing information about the patient to anybody except members of the health care team directly involved with the patient is disallowed. Information can be disclosed only on a need to know basis, although family

www.ClinicalReview.com


Medical Ethics

167 members can be informed regarding the patient’s condition if it is compatible with what the physician believes the patient would desire. Waivers to confidentiality can be signed by the patient.

EXCEPTIONS TO CONFIDENTIALITY Confidentiality also has its own exceptions. Informing others regarding a patient’s condition is allowed in situations where the potential harm to others is serious, as can occur if the patient is a danger to society. Confidentiality is also waived in situations where the patient is a danger to himself and the physician may make decisions to protect the patient from himself or others as deemed medically necessary. Physicians are required by law to inform people who may be harmed by the patient, thus breaching confidentiality. Other exceptions to confidentiality include informing health officials and people at risk for infectious diseases, situations involving abuse of children or elders, patients who are at risk of harming others due to inability to operate a motor vehicle or operate in a safe manner in their vocation, and in patients prone to suicide or homicide. TABLE 259 EXCEPTIONS TO CONFIDENTIALITY Exceptions to Confidentiality When the potential harm to others is serious, as when a patient is a danger to society. When the patient is a danger to himself or others. When others are at risk of an infectious disease. In situations involving child or elderly abuse When patients are at risk of harming others due to inability of operate a motor vehicle or work in a safe manner at their jobs. In patients prone to suicide or homicide.

Â


168

USMLE STEP 2

www.ClinicalReview.com


Medical Ethics

169

CHAPTER CONTENTS Basic Science .....................................................................................................170 Microcytic Anemia ............................................................................................173 Macrocytic Anemia ...........................................................................................176 Normocytic Anemia ..........................................................................................177 Specific Anemias ...............................................................................................180 Other Red Blood Cell Conditions ......................................................................186 Platelets and Coagulation .................................................................................188 Oncology ...........................................................................................................192 B-Cell Lymphomas ............................................................................................199 T-Cell Lymphomas .............................................................................................201 Practice Questions ............................................................................................203

Â


USMLE STEP 2

170

HEMATOLOGY AND ONCOLOGY HEMATOLOGY AND ONCOLOGY BASIC SCIENCE PHARMACOLOGY ANTICOAGULANTS DRUG Heparin

Warfarin

Abciximab

INDICATIONS

MECHANISM OF ACTION

Anticoagulation Atrial fibrillation

Antithrombin III potentiation (similar to heparin sulfate proteoglycans on endothelial cells) leading to prevention of clot formation

Anticoagulation Atrial fibrillation Anticoagulation in PTCA

COMPLICATIONS HIT

Inhibits epoxide reductase in liver to prevent formation of factors II, VII, IX, X, protein C, and protein S

Interacts with numerous medications (P-450) GI bleed Limb necrosis

Prevents platelet aggregation and thrombus formation

TCP

THROMBOLYTICS DRUG Streptokinase

tPA

Clopidogrel

INDICATIONS Fibrinolytic

Fibrinolytic

Hypercoagulability

MECHANISM OF ACTION

COMPLICATIONS

NOTES

Cleaves plasminogen to produce plasmin and breakdown clots

Allergic reaction on subsequent doses Bleeding

Usually given only once following MI

Serine protease forms plasmin to breakdown clots

Bleeding

Used in MI and stroke within the first six hours of symptom onset

Prevents platelet aggregation through ADP receptor blockade on platelets

Neutropenia TTP Hemorrhage

Also known as Plavix, currently in court cases due to possibility of causing MI and stroke Similar to ticlodipine

Intrinsic urokinase elevation in the body correlates to malignancy (facilitates metastasis)

www.ClinicalReview.com


Hematology and Oncology

171

S-PHASE METABOLITES DRUG

Methotrexate

5-Fluorouracil

INDICATIONS

MECHANISM OF ACTION

COMPLICATIONS

ALL Psoriasis RA Crohn disease Abortifacient

Antimetabolite that inhibits folic acid (DHFR competition)

Pancytopenia Bruising Pulmonary fibrosis BMS (reverses with leucovorin)

Colorectal cancer

Antimetabolite, pyrimidine analog that leads to cell cycle arrest

BMS (does not reverse with leucovorin) Photosensitivity

Other drugs in this category include azacitidine, capecitabine, doxifluridine, and gemcitabine

Methotrexate

ALKYLATING AGENTS DRUG

Cyclophosphamide

INDICATIONS

Lymphoma Leukemias Solid tumors SLE

MECHANISM OF ACTION

Cross-links guanine nucleotides

COMPLICATIONS Hemorrhagic cystitis through production of acrolein byproduct N/V BMS Alopecia Lethargy Carcinogenic (TCC)

Cyclophosphamide

Â


USMLE STEP 2

172

CYTOTOXIC ANTIBIOTICS DRUG

Doxorubicin

INDICATIONS Uterine CA Breast CA Ovarian CA Hodgkin lymphoma Lung CA Sarcoma

MECHANISM OF ACTION

Intercalates with DNA through topoisomerase II inhibition

COMPLICATIONS N/V BMS Alopecia CHF with cumulative dosing

CONTRAINDICATIONS

Previous chemotherapeutic myelosuppression

Doxorubicin

OTHER CELL CYCLE AGENTS DRUG Vincristine

Cisplatin

INDICATIONS

MECHANISM OF ACTION

COMPLICATIONS

NHL Hodgkin lymphoma ALL

Binds to tubulin dimers to disassemble microtubules used in the mitotic spindle

Sarcoma Small cell lung CA Ovarian CA Lymphoma Germ cell tumors

Cross-links DNA and leads to apoptosis

Peripheral neuropathy Constipation

Nephrotoxic, neurotoxic, N/V, ototoxic, alopecia

IMMUNOSUPPRESSANTS DRUG Cyclosporine

Tacrolimus

INDICATIONS

MECHANISM OF ACTION

COMPLICATIONS

Organ transplantation Psoriasis RA

Binds to cyclophilin in T-cells to inhibit calcineurin and thereby decrease the formation of interleukin-2 and other lymphokines

Gum hyperplasia, seizure, PUD, pancreatitis, N/V/D/F, dyspnea, pruritus, HTN, multiple organ damage

Atopic dermatitis Refractory uveitis Bone marrow transplantation

Binds to cyclophilin in T-cells to inhibit calcineurin and thereby decrease the formation of interleukin-2 and other lymphokines

Gum hyperplasia, seizure, PUD, pancreatitis, N/V/D/F, dyspnea, pruritus, HTN, multiple organ damage

www.ClinicalReview.com


Hematology and Oncology

173

MICROCYTIC ANEMIA IRON-DEFICIENCY ANEMIA ETIOLOGY AND PATHOPHYSIOLOGY Iron-deficiency anemia is due to decreased iron stores following poor intake, excess loss, or poor absorption. Iron-deficiency anemia is the most commonly encountered anemia in general practice. It is most common in the reproductive years of women and in pregnant women. The most common cause in men is the result of an occult GI bleed. Other causes include alveolar hemorrhage, nosocomial loss, CRF treated with hemodialysis, following surgery, and various types of hemolysis.

PRESENTATION AND DIAGNOSIS Iron-deficiency anemia presents with constitutional symptoms, exertional dyspnea, anorexia, melena, hematochezia, and / or hemoptysis, depending on the particular cause of blood loss. Objective signs include glossitis, angular stomatitis, koilonychias, pallor; iron-deficiency anemia can also be entirely subclinical. Anisocytosis and increased RDW are early signs of this disorder, and MCV indicates a hypochromic microcytic anemia. These results, combined with low ferritin, are diagnostic for iron-deficiency anemia.

TREATMENT Treatment involves replacing iron stores and correcting any underlying etiology. Ferrous sulfate is the agent of choice to replenish iron stores in the body. TABLE 260 IRON DEFICIENCY ANEMIA Iron Deficiency Anemia Etiology

Poor intake, excess loss, or poor absorption.

Presentation

Constitutional symptoms, exertional dyspnea, anorexia, melena, hematochezia, and / or hemoptysis

Diagnosis

Glossitis, angular stomatitis, koilonychias, pallor; iron-deficiency anemia. Anisocytosis and increased RDW are early signs of this disorder, and MCV indicates a hypochromic microcytic anemia, low ferritin.

Treatment

Replacing iron stores and correcting any underlying etiology.

SICKLE CELL ANEMIA (SCA) ETIOLOGY AND PATHOPHYSIOLOGY Sickle cell anemia (SCA) is a commonly inherited disease that is associated with significant morbidity and decreased lifespan. An autosomal recessive defect in the beta chain of the adult hemoglobin (HbA) leads to the sickle cell hemoglobin (HbS). Due to defects in RBC deformability, obstruction of blood vessels leads to sickle cell pain crises and organ damage, in addition to anemia.

PRESENTATION AND DIAGNOSIS SCA presents with constitutional symptoms and anemia. Painful crises occur intermittently due to vessel blockade (and possibly hand-foot syndrome) cause swelling and pain in the distal upper and lower extremities. Stroke is common, along with TIAs and

Â


USMLE STEP 2 174 RIND. Priapism may also occur with SCA. Acute presentations can include acute chest syndrome (ACS), which may present with severe chest pain due to blockade within the pulmonary vasculature. Chronic SCA can present with growth retardation, hepatomegaly, splenomegaly, pallor, jaundice, cardiomegaly with an SEM, skin ulceration, and cholelithiasis. A proliferative retinopathy is often present as well. Serious complications may also occur similar to an infection. Diagnosis of SCA is made by hemoglobin studies.

TREATMENT SCA is definitively cured only by bone marrow transplantation. In most individuals, treatment centers on avoiding pain crises, giving prophylaxis for infection, and reducing the symptoms and damage from SCA. Fluid repletion is commonly the first step in any acute presentation. NSAIDs are the first line of treatment for pain management followed by hydroxyurea (by some clinicians). Penicillin is often given to avoid pneumonia, which is common in SCA, and folic acid supplements. TABLE 261 SICKLE CELL ANEMIA (SCA) Sickle Cell Anemia (SCA) Etiology

Autosomal recessive defect in the beta chain of the HbA.

Presentation

Constitutional symptoms and anemia. Painful crises occur intermittently. Hand-foot syndrome may occur. Stroke is common, along with TIAs and RIND. Acute presentations can include ACS. Chronic SCA can present with growth retardation, hepatomegaly, splenomegaly, pallor, jaundice, cardiomegaly with an SEM, skin ulceration, and cholelithiasis. A proliferative retinopathy is often present as well.

Diagnosis

Made by hemoglobin studies.

Treatment

Definitively cured only by bone marrow transplantation. In most individuals, treatment centers on avoiding pain crises, giving prophylaxis for infection, and reducing the symptoms and damage. Fluid repletion is commonly the first step in any acute presentation. NSAIDs are the first line of treatment for pain. Penicillin, folic acid, hydroxyurea.

ALPHA-THALASSEMIA ETIOLOGY AND PATHOPHYSIOLOGY Alpha-thalassemia is the result of a hereditary defect in hemoglobin synthesis leading to an excess of beta-globins and the formation of hemoglobin H tetramers. Alpha-thalassemia affects Africans, Asians, Mediterranean’s, and Europeans. A cis deletion is one in which two alpha genes are lost on the same chromosome; cis loss has a greater potential to lead to more serious disease in offspring. Deletion of just one gene is asymptomatic and clinically insignificant. A cis deletion or trans deletion leads to alphathalassemia. Loss of three alpha hemoglobin genes leads to chronic hemolytic anemia. Loss of all four alpha hemoglobin genes leads to death in utero due to hydrops fetalis.

PRESENTATION AND DIAGNOSIS Alpha-thalassemia has variable penetrance and can lead to a hemolytic anemia. Multiple blood transfusions are necessary in some individuals, leading to hemochromatosis and widespread organ damage. Pallor is evident with significant anemia. Jaundice and hepatosplenomegaly are also typically present, along with folic acid deficiency. Ulceration and a predisposition to infection are common. A hypochromic, microcytic hemolytic anemia is typically present. Pigment gallstones from the hemolytic anemia may also develop. Hemoglobin electrophoresis demonstrates HbH.

www.ClinicalReview.com


Hematology and Oncology

175

TREATMENT Treatment for alpha-thalassemia involves supportive therapy, rapid resolution of infections, transfusion, if symptomatic severe anemia hemoglobin titers occur below 7 g / dL, treatment of hemochromatosis, and in severe cases, a bone marrow transplantation – the only definitive cure. Hemoglobin H disease, also known as Bart’s hemoglobin, requires blood transfusions for survival. Alpha-thalassemia minor leads to subclinical disease. TABLE 262 ALPHA-THALASSEMIA Alpha-thalassemia Etiology

Chromosomal deletion.

Presentation

Variable penetrance and can lead to a hemolytic anemia. Pallor is evident with significant anemia. Jaundice and hepatosplenomegaly are typically present, along with folic acid deficiency. Ulceration and a predisposition to infection are common. Pigment gallstones from the hemolytic anemia may also develop. A hypochromic, microcytic hemolytic anemia is also typically present.

Diagnosis

Hemoglobin electrophoresis.

Treatment

Supportive therapy, rapid resolution of infections, transfusion if symptomatic severe anemia, treatment of hemochromatosis, and in severe cases, a bone marrow transplantation. Hemoglobin H disease requires blood transfusions for survival. Alpha-thalassemia minor leads to subclinical disease.

BETA-THALASSEMIA ETIOLOGY AND PATHOPHYSIOLOGY Beta-thalassemia presents with an excess of alpha-globins leading to alpha-globin tetramers. Deletions and substitutions within the genetic framework account for most cases of beta-thalassemia and spontaneous disease is possible. Beta-thalassemia deals with a maternal and paternal allele; as a result, damage to one allele leads to reduced formation of the beta-globin. Damage to both alleles leads to formation of only alpha chains. Beta-thalassemia is especially common in Italians, Greeks, and Southeast Asians.

PRESENTATION AND DIAGNOSIS Beta-thalassemia presents as a congenital condition that leads to severe jaundice and anemia early in life. Constitutional symptoms, symptoms of anemia, and hypersplenism are common. Gallstones may also be present. Diagnostic workup proceeds in the same fashion as alpha-thalassemia. Beta-thalassemia major leads to early splenomegaly and symptoms of anemia, and typically requires transfusions to maintain hemoglobin above 3-5 g / dL. Thalassemia intermedia leads to a hemoglobin titer between 6 and 9 g / dL. Thalassemia minor leads to clinically insignificant disease.

TREATMENT Treatment for beta-thalassemia includes transfusion for symptomatic, severe anemia and supportive therapy is a must. Like alpha-thalassemia, splenectomy may be necessary to decrease the number of required transfusions. Early treatment for infections is required. Bone marrow transplantation is curative.


176

USMLE STEP 2

TABLE 263 BETA-THALASSEMIA Beta-thalassemia Etiology

Genetic deletions and substitutions.

Presentation

Severe jaundice and anemia early in life. Constitutional symptoms, symptoms of anemia, and hypersplenism are common. Gallstones may also be present.

Diagnosis

Workup proceeds in the same fashion as alpha-thalassemia.

Treatment

Transfusion for symptomatic, severe anemia. Supportive therapy. Splenectomy may be necessary. Early treatment for infections is required. Bone marrow transplantation is curative.

MACROCYTIC ANEMIA MEGALOBLASTIC ANEMIA ETIOLOGY Megaloblastic anemia is the presence of immature erythroblasts with an increase in MCV. Megaloblastic anemia is commonly found in vitamin B12 deficiency or folate deficiency. The former may be due to an autoimmune defect leading to pernicious anemia, poor intake, or malabsorption due to ileal disease. The latter may be due to alcoholism, poor intake, chronic hemolytic anemia, or various malabsorption syndromes. Various chemotherapy drugs may also lead to megaloblastic anemia. Megaloblastic anemia is more common with increasing age. African American women and the elderly are most at risk.

PATHOPHYSIOLOGY – FOLATE DEFICIENCY Folate deficiency may be the result of poor dietary intake, increased demand, as in pregnancy, or increased demand, as in chronic hemolytic anemias. Any number of hemolytic anemias may lead to an increased requirement in folate levels. Malabsorption syndromes such as Crohn disease or other enteropathies can lead to failure to properly absorb folate. Antagonists to folate such as methotrexate, or those agents that affect metabolism such as alcohol, sulfasalazine, triamterene, TMP-SMX, barbiturates, and nitric oxide can impede the use of folate by RBCs. Exposure to heavy metals or toxins such as arsenic or chlordane can also lead to retardation of folate utilization.

PATHOPHYSIOLOGY – VITAMIN B12 DEFICIENCY Vitamin B12 deficiency may be a result of many of the causes that lead to folate deficiency. In addition, pernicious anemia due to antibodies against parietal cells may lead to diminished amounts of intrinsic factor required for binding and absorption of vitamin B12. Lack of animal protein in strict vegetarian diets can also lead to deficiencies in vitamin B12. Diverticulosis and bacterial overgrowth, or infection by D. latum also lead to decreased vitamin B12 available for absorption.

PRESENTATION – FOLATE DEFICIENCY Folate-deficiency megaloblastic anemia presents with pallor and glossitis. There are no neurologic deficits.

www.ClinicalReview.com


Hematology and Oncology

177

PRESENTATION – VITAMIN B12 DEFICIENCY Vitamin B12-deficiency megaloblastic anemia presents with pallor, glossitis, and a peripheral sensory neuropathy that advances to a loss of deep tendon reflexes (DTR). Confusion and memory loss may be present. Delirium and dementia may occur in later stages.

DIAGNOSIS Diagnosis of megaloblastic anemia is made by increases in MCV, identified decrease in folate and / or vitamin B12, and the presence of an anemia. Hypersegmented neutrophils may be present in pernicious anemia. A Schilling test may be done to test the ability to absorb vitamin B12. Methylmalonic acid is normal in folate-deficiency megaloblastic anemia but positive in vitamin B12-deficiency megaloblastic anemia. Antibodies to intrinsic factor can be demonstrated for pernicious anemia.

TREATMENT Treatment for megaloblastic anemia is to replace the vitamin deficiency. Care should be taken in reversing a vitamin B12 deficiency by folate. Folate, however, will not prevent further progression of the neurologic symptoms from a vitamin B12 deficiency. Transfusion is occasionally undertaken in severe anemia, but pulmonary edema may develop. Vitamin replacement is the standard of care. TABLE 264 MEGALOBLASTIC ANEMIA Megaloblastic Anemia Etiology

Vitamin B12 deficiency due to an autoimmune defect leading to pernicious anemia, poor intake, or malabsorption due to ileal disease. Folate deficiency due to alcoholism, poor intake, chronic hemolytic anemia, or various malabsorption syndromes.

Presentation

B12-- pallor, glossitis, and a peripheral sensory neuropathy that advances to loss of DTR. Confusion and memory loss may be present. Delirium and dementia may occur in later stages. Folate-- pallor and glossitis, no neurologic deficiencies.

Diagnosis

Increase in MCV, identified decrease in folate and / or vitamin B12, and the presence of an anemia. Hypersegmented neutrophils, Schilling test, antibodies to intrinsic factor.

Treatment

Replace the vitamin deficiency, possibly transfusion.

NORMOCYTIC ANEMIA ANEMIA OF CHRONIC DISEASE ETIOLOGY AND PATHOPHYSIOLOGY The most common cause of a normocytic anemia is anemia of chronic disease (ACD); ACD is also the second most common type of anemia. While ACD tends to be a normocytic normochromic anemia, some presentations may have a microcytic anemia. ACD is due to decreased bone marrow production of erythrocytes after longstanding chronic disease, itself the result of a combination of erythropoietin resistance, decreased production, and decreased RBC half-life. ACD may also be due to chronic inflammation, cancer, and systemic diseases.

Â


178

USMLE STEP 2

PRESENTATION AND DIAGNOSIS ACD tends to develop with a moderate- or low-grade anemia and is typically subclinical in presentation. More severe cases may present with symptoms of anemia.

TREATMENT Treatment of the primary disease is the only way to resolve ACD. Blood transfusions are rarely required. TABLE 265 ANEMIA OF CHRONIC DISEASE (ACD) Anemia of Chronic Disease (ACD) Etiology

Decreased bone marrow production of erythrocytes after longstanding chronic disease. It may also be the result of chronic inflammation, cancer, and systemic diseases.

Presentation/Diagnosis

Moderate- or low-grade anemia, which is typically subclinical. More severe cases may present with symptoms of anemia.

Treatment

Treatment of the primary disease.

SIDEROBLASTIC ANEMIA ETIOLOGY AND PATHOPHYSIOLOGY Sideroblastic anemia presents with ring sideroblasts in the bone marrow, decreased heme synthesis, and a normocytic anemia. The general cause is a disruption in the normal metabolism of mitochondria leading to decreased ATP available for consumption by the RBC. This injury may be the result of numerous etiologies, including hereditary mechanisms such as a congenital X-linked disorder, an autosomal dominant disorder, an autosomal recessive disorder, and inherited mitochondrial cytopathy; acquired etiologies include myelodysplastic syndrome (MDS), certain drugs such as alcohol, INH, chloramphenicol, and cycloserine, toxins such as lead and zinc, and nutritional deficiencies such as lack of copper or pyridoxine. MDS is a dysfunction in hematopoietic stem cell function that affects all three cell lines (myeloid precursors, erythroid precursors, and megakaryocytes) leading to ineffective erythropoiesis, marrow dysfunction, and eventually, the development of acute leukemia. Various drugs lead to sideroblastic anemia through blockade of the heme biosynthetic pathway; INH in particular has been implicated in numerous cases of sideroblastic anemia.

PRESENTATION AND DIAGNOSIS Sideroblastic anemia presents with moderate or severe anemia with the typical symptoms of anemia – including constitutional symptoms such as fatigue, dizziness, and decreased exercise tolerance. A history identifying a particular etiology may also be present, such as alcoholism or toxin exposure. Basophilic stippling is evident on examination of a peripheral blood smear; hypochromia and microcytosis may be present in some cases instead of the typical normocytic anemia.

TREATMENT Treatment of sideroblastic anemia involves reversing those etiologies that are amenable to treatment. Blood transfusions are often required in other cases. Pyridoxine is sometimes effective. Monitoring iron load and using deferoxamine to avoid hemochromatosis is the standard of care in any situation involving chronic transfusions.

www.ClinicalReview.com


Hematology and Oncology

179

TABLE 266 SIDEROBLASTIC ANEMIA Sideroblastic Anemia Presentation

Moderate or severe anemia with the typical symptoms of anemia – including constitutional symptoms such as fatigue, dizziness, and decreased exercise tolerance.

Diagnosis

A history identifying a particular etiology. Basophilic stippling is evident on examination of a peripheral blood smear; hypochromia and microcytosis may be present in some cases.

Treatment

Reversing those etiologies that are amenable to treatment. Blood transfusions are often required. Pyridoxine is sometimes effective. Monitoring iron load and using deferoxamine.

APLASTIC ANEMIA ETIOLOGY AND PATHOPHYSIOLOGY Aplastic anemia is the development of pancytopenia as a result of an acquired or familial syndrome that leads to bone marrow failure. Acquired disease may be the result of exposure to viral infections such as EBV, HIV, HBV, or dengue fever, mycobacterial infection, autoimmune disease, toxic chemical exposure including benzene, chemotherapeutic agents, arsenic, and estrogens, and exposure to ionizing radiation. Many other medications can also lead to aplastic anemia, but they are a rare complication. Insecticides and gold compounds in particular can lead to serious illness. Aplastic anemia affects fewer than 1 patient in 100,000, and generally peaks in the elderly. Transient causes include vitamin B12 or folate deficiency and infection by agents such as parvovirus B19.

PRESENTATION AND DIAGNOSIS Aplastic anemia presents with abnormalities in bleeding, epistaxis, fever, pharyngitis, easy bruising, and various constitutional symptoms. Signs of anemia are present, along with oral ulcerations and retinal hemorrhage. Severe aplastic anemia has anemia with reticulocytopenia, thrombocytopenia, neutropenia, and a hypocellular bone marrow. Pancytopenia is found on CBC, and bone marrow biopsy confirms the diagnosis. A dry bone marrow tap is more typical of an infiltrative marrow disease, not aplastic anemia.

TREATMENT Therapy for aplastic anemia revolves around correcting any reversible causes of the anemia and consideration of a bone marrow transplantation. Immunosuppressive therapy may be needed in patients who cannot receive a transplantation; cyclosporine and antithymocyte globulin are used in this case. Infection prophylaxis is important for good care. TABLE 267 APLASTIC ANEMIA Aplastic Anemia Etiology

Acquired or familial syndrome that leads to bone marrow failure. Acquired disease may be the result of exposure to viral infections, mycobacterial infection, autoimmune disease, and exposure to ionizing radiation. Insecticides and gold compounds in particular can lead to serious illness.

Presentation

Abnormalities in bleeding, epistaxis, fever, pharyngitis, easy bruising, and various constitutional symptoms. Signs of anemia are present, along with oral ulcerations and retinal hemorrhage.

Diagnosis

CBC, and bone marrow biopsy.

Treatment

Correcting any reversible causes of the anemia and consideration of a bone marrow transplantation. Infection prophylaxis is important for good care.

Â


180

USMLE STEP 2

SPECIFIC ANEMIAS FANCONI ANEMIA ETIOLOGY AND PATHOPHYSIOLOGY Fanconi anemia is an inherited failure of the bone marrow leading to pancytopenia and aplastic anemia. This autosomal recessive disease occurs due to mutations in several different genes that code for proteins responsible for DNA repair; the outcome is a series of birth defects, bone marrow dysfunction, and carcinogenesis. About 1 in 300 persons are carriers, but only about 1 in over 300,000 persons is affected. Ashkenazi Jews are more affected by Fanconi anemia. Morbidity in this disorder is due to bone marrow failure, the development of leukemia, and carcinogenesis.

PRESENTATION AND DIAGNOSIS Fanconi anemia presents with growth retardation, anatomic defects in the genitourinary system, and radial ray anomalies. CafĂŠ au lait spots are present, along with petechiae and numerous constitutional symptoms. Symptoms of thrombocytopenia and pancytopenia are predominant over time. Short stature is nearly universal, along with profound defects of the head, face, and remaining skeleton. Conductive deafness and congenital cardiac defects are typically present. Diagnosis is made by chromosomal analysis and the presence of marrow failure. Imaging studies confirm the skeletal defects.

TREATMENT Symptomatic and supportive therapies are the standard of care for patients with Fanconi anemia. Stem cell transplantation may prevent aplastic anemia, MDS, and leukemia. Surgical intervention is often necessary to repair limb defects. TABLE 268 FANCONI ANEMIA Fanconi Anemia Etiology

Autosomal recessive disease occurs due to mutations in several different genes

Presentation

Growth retardation, anatomic defects in the genitourinary system and radial ray anomalies. CafĂŠ au lait spots, along with petechiae and numerous constitutional symptoms. Symptoms of thrombocytopenia and pancytopenia predominant over time. Short stature is nearly universal, along with profound defects of the head, face, and remaining skeleton. Conductive deafness and congenital cardiac defects are typically present.

Diagnosis

Chromosomal analysis and the presence of marrow failure. Imaging studies confirm the skeletal defects.

Treatment

Symptomatic and supportive therapy. Stem cell transplantation, surgical intervention.

HEMOLYTIC ANEMIA ETIOLOGY AND PATHOPHYSIOLOGY Hemolytic anemia leads to early destruction of RBCs and presents with anemia when the bone marrow cannot compensate for the loss of RBCs. Numerous causes exist, but major ones include G6PD deficiency, hereditary spherocytosis, sickle cell anemia, DIC, HUS, TTP, prosthetic valves, and PNH. It is present in about 1 in 20 anemias and leads to symptoms only with severe anemia.

www.ClinicalReview.com


Hematology and Oncology

181

PRESENTATION AND DIAGNOSIS Hemolytic anemia presents with symptoms of anemia. Tachycardia, dyspnea, and weakness are typically present in severe cases. Bilirubin pigmented stones may lead to cholelithiasis. Repeated transfusions may lead to hemochromatosis. A history of use of certain medications, such as penicillin, quinine, or L-dopa may be responsible for an immune reaction leading to hemolytic anemia. Favism is especially common in the Mediterranean type of G6PD. Pallor, jaundice, splenomegaly, leg ulcers, and other symptoms of anemia may be present on physical exam. Diagnosis is made by peripheral blood smear and standard tests for anemia.

TREATMENT Treatment for hemolytic anemia is similar to that for any other type of anemia – transfusions with symptomatic, severe anemia, avoiding triggers that worsen the anemia, and treating reversible causes. TABLE 269 HEMOLYTIC ANEMIA Hemolytic Anemia Etiology

G6PD deficiency, hereditary spherocytosis, sickle cell anemia, DIC, HUS, TTP, prosthetic valves, and PNH.

Presentation

Symptoms of anemia. Tachycardia, dyspnea, and weakness, cholelithiasis. A history of use of certain medications, favism. Pallor, jaundice, splenomegaly, leg ulcers may be present on physical exam.

Diagnosis

Diagnosis is made by peripheral blood smear and standard tests for anemia.

Treatment

Transfusions with symptomatic, severe anemia, avoiding triggers that worsen the anemia, and treating reversible causes.

COLD HEMOLYTIC ANEMIA ETIOLOGY AND PATHOPHYSIOLOGY Cold agglutinin hemolytic anemia occurs due to IgM antibodies that induce a complement-mediated lysis of RBCs. The IgM antibodies may be due to a clonal expansion of B cells, or following infection by Mycoplasma pneumoniae, EBV, influenza, and HIV. Agglutination of the IgM antibodies and subsequent hemolysis occurs especially in colder temperatures, hence the name of this disease. Low titers of IgM are found in otherwise healthy persons; elevations may occur with infection, autoimmune disease, or periods of significant stress. Only 1 in 300,000 persons are affected.

PRESENTATION AND DIAGNOSIS Cold agglutinin hemolytic anemia presents with Raynaud’s phenomenon with significant symptoms during cold weather. Constitutional symptoms are typically present, along with symptoms from any concurrent infection. Dark urine may also be present with cold weather due to significant peripheral hemolysis. Pallor is typically present, but splenomegaly and jaundice are absent in this disease. It is differentiated from warm hemolytic anemia (discussed above) because this disorder is present only with cold temperatures. Diagnosis is made in a manner similar to that of other hemolytic anemias and with cold agglutination studies.


182

USMLE STEP 2

TREATMENT Treatment is to avoid the cold and to properly protect the patient against cold weather. Otherwise, standard therapy for anemia is undertaken but rarely necessary as this anemia is typically mild in nature. Plasmapheresis may be intermittently necessary to remove excess IgM. TABLE 270 COLD HEMOLYTIC ANEMIA Cold Hemolytic Anemia Etiology

IgM antibodies that induce a complement-mediated lysis of RBCs. The IgM antibodies may be due to a clonal expansion of B cells, or following infection by Mycoplasma pneumoniae, EBV, influenza, and HIV.

Presentation

Raynaud’s phenomenon with significant symptoms during cold weather. Constitutional symptoms, along with symptoms from any concurrent infection. Dark urine, pallor.

Diagnosis

Similar to that of other hemolytic anemias. Cold agglutination studies can be performed.

Treatment

Avoid the cold, protect the patient against cold weather. Plasmapheresis may be intermittently necessary.

PAROXYSMAL HEMOLYTIC ANEMIA (PHA) ETIOLOGY AND PATHOPHYSIOLOGY Paroxysmal hemolytic anemia (PHA) is also known as autoimmune hemolytic anemia (AIHA), a type of hemoglobinuria that occurs due to an autoimmune disorder leading to significant intravascular hemolysis and anemia. It is typically present during periods of significant stress such as infection. PHA is distinct from other hemolytic anemias in that it is highly symptomatic and also has a proteinuria with a Bence Jones-type of polypeptide. PHA is due to low temperatures leading to binding of a polyclonal IgG to RBCs inducing intravascular hemolysis. Postviral infections may induce the formation of this antibody and subsequent disease.

PRESENTATION AND DIAGNOSIS PHA presents with significant symptoms of anemia that develop insidiously after cold exposure. Significant lower extremity pain and severe constitutional symptoms are acutely present. Renal failure can occur due to the significant level of hemolysis taking place. A concurrent viral infection may be present. Diagnosis is made by clinical history and confirmation through an anemia workup. LDH and unconjugated bilirubin are high, and free hemoglobin may be found in the plasma – a testament to the rapid intravascular hemolysis taking place. A positive D-L antibody test also indicates cold temperature-mediated lysis of RBCs after the temperature rises.

TREATMENT Treatment is similar to that of cold agglutinin hemolytic anemia. Rapid treatment of infections is necessary. Hydration is necessary to avoid renal failure and damage. Folic acid supplements are recommended, as they are in virtually all anemias.

www.ClinicalReview.com


Hematology and Oncology

183

TABLE 271 PAROXYSMAL HEMOLYTIC ANEMIA (PHA) Paroxysmal Hemolytic Anemia (PHA) Presentation

Significant symptoms of anemia that develop insidiously after cold exposure. Lower extremity pain and severe constitutional symptoms are acutely present. Renal failure can occur. A concurrent viral infection may be present.

Diagnosis

Clinical history and confirmation through an anemia workup. LDH and unconjugated bilirubin are high, and free hemoglobin may be found in the plasma.

Treatment

Similar to that of cold agglutinin hemolytic anemia. Rapid treatment of infection and hydration are necessary. Folic acid supplements are recommended.

PAROXYSMAL NOCTURNAL HEMOGLOBINURIA ETIOLOGY AND PATHOPHYSIOLOGY Paroxysmal nocturnal hemoglobinuria (PNH) is the formation of dark urine due to hemolysis that occurs over time. Hemolytic anemia is present due to lack of deformability in the RBC membrane. Thrombosis of large vessels may occur in this disorder. Hemopoietic deficiencies may also be present and lead to pancytopenia or aplastic anemia. This triad distinguishes PNH from other hemolytic anemias. The underlying defect is due to inability to manufacture glycosyl-phosphatidylinositol (GPI), an anchor that latches proteins onto the cell membrane. The defect is in the phosphatidylinositol glycan class A (PIGA) gene, among defects found on other genes.

PRESENTATION AND DIAGNOSIS PNH presents with dark urine early in the morning, but hemolysis continues throughout the day. Venous thrombosis may present with abdominal pain, hepatomegaly, ascites (as in Budd-Chiari syndrome), and symptoms of aplastic anemia. Painful skin nodules may be present, and severe headaches may occur as a result of the venous thrombosis. A predisposition to infection may also occur. Diagnosis is made by identifying the genetic defects and demonstrating the pathophysiology of PNH. Acidified serum lysis and the Ham test are diagnostic, along with a complement lysis sensitivity test.

TREATMENT Stem cell transplantation is curative, but rarely an option. Glucocorticoids are sometimes used to alleviate the symptoms of PNH. Other standard principles of anemia therapy are used with PNH. As in other cases of bone marrow failure and aplastic anemia, antithymocyte globulin (ATG) has been used with success. TABLE 272 PAROXYSMAL NOCTURNAL HEMOGLOBINURIA (PNH) Paroxysmal Nocturnal Hemoglobinuria (PNH) Presentation

Presents with dark urine early in the morning. Venous thrombosis may present with abdominal pain, hepatomegaly, ascites, and symptoms of aplastic anemia. Painful skin nodules and severe headaches may occur.

Differential diagnosis

Hemopoietic deficiencies may also be present and lead to pancytopenia or aplastic anemia. This triad distinguishes PNH from other hemolytic anemias.

Diagnosis

Identifying the genetic defects and demonstrating the pathophysiology. Acidified serum lysis, Ham and test complement lysis sensitivity tests.

Treatment

Glucocorticoids, standard principles of anemia therapy. ATG has been used with success.

Â


184

USMLE STEP 2

TRANSFUSION REACTIONS ETIOLOGY AND PATHOPHYSIOLOGY Transfusion reactions occur due to immune reactions against donated blood that does not match that of the recipient. Serious reactions can lead to death, and rapid identification and reversal are necessary. Acute reactions may be due to immune-mediated reactions with various antibodies; those antibodies against the major blood groups (AB) can lead to death through notable intravascular hemolysis. Non-ABO antibodies result in extravascular reactions and are milder. Nonimmune reactions occur with damage to the donated RBCs leading to hemoglobinuria and hemoglobinemia. The presence of various cytokines in the donated blood can precipitate additional APRs and lead to constitutional symptoms. Immune reactions can present as severe anaphylaxis and lead to shock and death. Fatal reactions affect 4 persons in a million; nonfatal immune reactions affect 1 in 10,000 persons. Allergic reactions occur in 1 in 300 persons, and anaphylaxis may occur in 1 in 50,000 persons. Multiparous women are more likely to have symptoms than other groups.

PRESENTATION AND DIAGNOSIS Transfusion reactions present with the aforementioned symptoms. Early signs include fever, dropping BP, flushing, anxiety, and wheezing. Later signs include DIC. In nonhemolytic reactions, only fever is present along with mild constitutional symptoms and hypotension. Allergic reactions may present with a maculopapular rash and pruritus. Anaphylactic reactions may present with dyspnea, wheezing, anxiety, bronchospasm, and hypotension. In transfusion-related acute lung injury (TRALI), SOB, hypoxia, and orthopnea with cardiac decompensation may be present. Diagnosis is made by workups for anemia and a direct Coombs test.

TREATMENT Transfusion reactions are treated by stopping the transfusion and careful observation. Prophylaxis against renal failure and DIC are necessary. Diuresis may be necessary. Acetaminophen is used for fever, diphenhydramine for mild allergic reactions, and epinephrine for anaphylactic reactions. Severe symptoms may require admission to ICU and supportive therapy. A workup for sepsis may be necessary. TABLE 273 TRANSFUSION REACTION Transfusion Reaction

Presentation

Allergic reactions, anaphylaxis, shock. Early signs include fever, dropping BP, flushing, anxiety, and wheezing. Later signs include DIC. In nonhemolytic reactions, only fever is present along with mild constitutional symptoms and hypotension. Allergic reactions may present with a maculopapular rash and pruritus. Anaphylactic reactions may present with dyspnea, wheezing, anxiety, bronchospasm, and hypotension. In TRALI, SOB, hypoxia, and orthopnea with cardiac decompensation may be present.

Diagnosis

Diagnosis is made by workups for anemia and a direct Coombs test.

Treatment

Stopping the transfusion and careful observation. Prophylaxis against renal failure and DIC. Diuresis may be necessary. Acetaminophen is used for fever, diphenhydramine for mild allergic reactions, and epinephrine for anaphylactic reactions. A workup for sepsis may be necessary.

HEREDITARY SPHEROCYTOSIS (HS) ETIOLOGY AND PATHOPHYSIOLOGY www.ClinicalReview.com


Hematology and Oncology

185 Hereditary spherocytosis (HS) is a familial disorder that can lead to severe hemolytic anemia. Loss of the RBC membrane leads to decreased integrity and an increased risk of hemolysis. These spherical RBCs are sequestered by the spleen, leading to marked splenomegaly in severe cases. HS may be due to defects in spectrin, ankyrin, band 3, and protein 4.2. Northern Europeans are affected the most by this usually autosomal trait. Morbidity and mortality are determined by the severity of the anemia.

PRESENTATION AND DIAGNOSIS HS presents with anemia, jaundice, and splenomegaly. The nature of the anemia varies by individual, but it can be very severe and lead to significant mortality. Reticulocytosis and increased mean corpuscular hemoglobin concentration (MCHC) are common. Peripheral blood smears identify spherocytes, and hyperbilirubinemia may be detected. The osmotic fragility test may also be used to identify RBCs susceptible to hemolysis – a sensitive but not specific test of acquired deficiencies in the RBC skeleton.

TREATMENT Treatment of HS involves antibiotic and vaccination prophylaxis to avoid infections prior to splenectomy. Cholecystectomy may be required with severe cholelithiasis. Splenectomy is curative, but care should be taken to avoid infection by encapsulated organisms such as S. pneumoniae, H. influenzae, and Neisseria spp. TABLE 274 HEREDITARY SPHEROCYTOSIS (HS) Hereditary Spherocytosis (HS) Etiology

Autosomal disorder.

Presentation

Anemia, jaundice, and splenomegaly. Reticulocytosis and increased MCHC are common.

Diagnosis

Peripheral blood smears identify spherocytes, and hyperbilirubinemia may be detected. The osmotic fragility test may also be used.

Treatment

Antibiotic and vaccination prophylaxis prior to splenectomy. Cholecystectomy may be required. Splenectomy is curative.

GLUCOSE-6-PHOSPHATASE DEHYDROGENASE DEFICIENCY (G6PD) ETIOLOGY AND PATHOPHYSIOLOGY Glucose-6-phosphatase dehydrogenase (G6PD) deficiency is an X-linked disorder that affects nearly half a billion people around the world. Its protection against malaria is the likely reason G6PD deficiency is so prevalent. Defects in the ability to oxidize certain reactions lead to excess glutathione, which in turn leads to free radical formation and premature damage to RBCs.

PRESENTATION AND DIAGNOSIS G6PD presents with neonatal jaundice and acute hemolytic anemia. Drug-induced hemolysis or consumption of fava beans leading to hemolytic anemia is common. Jaundice and splenomegaly is found on physical exam. Most patients are entirely asymptomatic. Diagnosis is confirmed by measuring the activity of G6PD enzyme.

Â


186

USMLE STEP 2

TREATMENT Alleviating symptoms by discontinuing the offending agent is the only treatment necessary, in addition to avoiding fava beans and giving supportive therapy in acute exacerbations. TABLE 275 GLUCOSE-6-PHOSPHATASE DEHYDROGENASE DEFICIENCY (G6PD) Glucose-6-Phosphatase Dehydrogenase Deficiency (G6PD) Etiology

X-linked disorder.

Presentation

Neonatal jaundice and acute hemolytic anemia. Drug-induced hemolysis or consumption of fava beans leading to hemolytic anemia is common. Jaundice and splenomegaly.

Diagnosis

Measure the activity of G6PD enzyme.

Treatment

Discontinue the offending agent, in addition to avoiding fava beans and giving supportive therapy in acute exacerbations.

OTHER RED BLOOD CELL CONDITIONS POLYCYTHEMIA VERA (PV) ETIOLOGY AND PATHOPHYSIOLOGY Polycythemia vera (PV) is the development of a neoplastic marrow that leads to uncontrolled erythrocytosis, myelocytosis, and megakaryocytosis. The end result is thrombosis and bleeding diatheses. PV is rare.

PRESENTATION AND DIAGNOSIS PV begins with symptoms of hyperviscosity in the vessels leading to thrombosis, headache, tinnitus, visual defects, angina, and claudication. Bleeding diatheses occur, and marked splenomegaly is present. Excess degranulation of mast cells and basophils can lead to pruritus. A red complexion is common, along with HTN. All hematopoietic cells are increased in number. Over half a million platelets are typically found in a CBC, along with elevated WBCs.

TREATMENT PV is treated by therapeutic phlebotomy to reduce hematocrit (Hct) to 40%. Hydroxyurea may be necessary for myelosuppression. Platelet aggregation can be inhibited with anagrelide. Splenectomy is helpful. TABLE 276 POLYCYTHEMIA VERA (PV) Polycythemia Vera (PV) Etiology

Neoplastic marrow.

Presentation

Symptoms of hyperviscosity in the vessels leading to thrombosis, headache, tinnitus, visual defects, angina, and claudication. Bleeding diatheses occur, and marked splenomegaly is present. Pruritus. A red complexion is common, along with HTN.

Diagnosis

CBC

Treatment

Therapeutic phlebotomy. Hydroxyurea, anagrelide. Splenectomy is helpful. www.ClinicalReview.com


Hematology and Oncology

187

PORPHYRIA ETIOLOGY AND PATHOPHYSIOLOGY Porphyria refers to six distinct disorders that are defects in metabolic enzymes leading to neuropsychiatric manifestations, abdominal pain, and accumulation of porphyrins in tissues. Porphyria is categorized as porphyria cutanea tarda, acute intermittent porphyria, erythropoietic protoporphyria, variegate porphyria, hereditary coproporphyria, and congenital erythropoietic porphyria. Specific enzyme defects lead to each of these specific porphyrias. Porphyria cutanea tarda may be acquired through HCV, exposure to hydrocarbons, alcohol abuse, and use of estrogens. Risk of developing porphyria is tied to liver disease and factors that affect the function of the liver. Porphyria cutanea tarda is the most common porphyria, and affects 1 in 10,000 persons. The others are significantly rarer. Congenital porphyrias present early in life, while porphyria cutanea tarda and acute intermittent porphyria present in young adults. Caucasians are affected more than other groups.

PRESENTATION AND DIAGNOSIS Acute intermittent porphyria may present with constitutional symptoms, dysuria, incontinence, fever, extremity pain or pain in the torso, loss of the sensorium, seizures, disorientation, depression, hallucinations, and paranoia. Delta-aminolevulinic acid dehydratase deficiency porphyria presents similar to acute intermittent porphyria. Porphyria cutanea tarda presents with photosensitivity, bullous vesicles followed by crusting, blistering of the skin, red urine on standing, hypertrichosis, pigmentation changes on the skin, acanthosis, and onycholysis. Erythropoietic protoporphyria presents with early onset of photosensitivity even with room lights, gallstones, hepatic disease, and acanthosis. Variegate porphyria presents similar to acute intermittent porphyria but with photosensitivity and lesions like porphyria cutanea tarda. Hereditary coproporphyria presents like variegate porphyria. Congenital erythropoietic porphyria is similar to erythropoietic porphyria except with a much earlier presentation. Alopecia and ocular abnormalities may be seen in this version. Acute intermittent porphyria is associated with HTN, CRF, hepatocellular carcinoma, and depression. Porphyria cutanea tarda is associated with a more infectious etiology, including HIV, HCV, hemochromatosis, and hepatocellular carcinoma. Diagnosis is made by examining metabolites in the urine, examining the venous blood by a total porphyrin test, and quantitative tests for porphyrins.

TREATMENT Treating the porphyrias requires avoiding precipitating factors such as alcohol, estrogen, various medications such as NSAIDs and sulfonylureas, and iron supplements. Phlebotomy may be required to avoid hemochromatosis. Chloroquine may be necessary. Protection against the sun is necessary, and antioxidant therapy may be useful. Glucose is administered with neuropsychiatric manifestations, gabapentin for seizures, and hematin with severe symptoms. SSRIs are administered for depression. Avoiding the precipitating factors is the key for long term control.

Â


188

USMLE STEP 2

TABLE 277 PORPHYRIA Porphyria Etiology

Specific enzyme defects

Presentation

Acute intermittent-- constitutional symptoms, dysuria, incontinence, fever, extremity torso pain, loss of the sensorium, seizures, disorientation, depression, hallucinations, and paranoia. Delta-aminolevulinic acid dehydratase deficiency-- similar to acute intermittent. Porphyria cutanea tarda--photosensitivity, bullous vesicles followed by crusting, blistering of the skin, red urine on standing, hypertrichosis, pigmentation changes on the skin, acanthosis, and onycholysis. Erythropoietic--early onset of photosensitivity, gallstones, hepatic disease, and acanthosis. Variegate-- similar to acute intermittent porphyria but with photosensitivity and lesions like porphyria cutanea tarda. Hereditary coproporphyria--presents like variegate porphyria. Congenital erythropoietic-- is similar to erythropoietic porphyria except with a much earlier presentation. Alopecia and ocular abnormalities may be seen in this version.

Diagnosis

Examine metabolites in the urine, examine the venous blood by a total porphyrin test, and quantitative tests for porphyrins.

Treatment

Avoiding precipitating factors, various medications. Phlebotomy may be required. Chloroquine may be necessary. Protection against the sun is necessary, and antioxidant therapy may be useful. Glucose is administered with neuropsychiatric manifestations, gabapentin for seizures, and hematin with severe symptoms. SSRIs are administered for depression.

PLATELETS AND COAGULATION IMMUNE THROMBOCYTOPENIC PURPURA (ITP) ETIOLOGY AND PATHOPHYSIOLOGY Also known as idiopathic thrombocytopenic purpura (ITP), this bleeding diathesis presents with thrombocytopenia, purpura, and petechiae along with a predisposition towards hemorrhage. The presence of autoantibodies against platelets leads to decreased platelet longevity due to macrophage phagocytosis in the spleen. The antibody appears to be against a GPI anchor. ITP is a relatively rare disorder due to its subclinical nature in most patients. Concurrent SLE, AML, CML, or MDS may be present; the disorder may also follow infection by EBV, VZV, CMV, rubella virus, HAV, HBV, HCV, HIV, or a generic URI. Medications that can lead to sensitization include quinine, cephalothins, rifampicin, gold salts, NSAIDs, HTN medications, diuretics, and abciximab. Heparin-induced thrombocytopenia (HIT) may also lead to ITP.

PRESENTATION AND DIAGNOSIS ITP may lead to morbidity through intracranial hemorrhage or bleeding in other parts of the body. Petechiae and ecchymoses are typically present. Neurologic exam may be positive for findings, and a hemopericardium may be identified in some individuals. Diagnosis is confirmed by CBC and large platelets found in peripheral blood smears. Antiplatelet antibodies may also be present. A positive Coombs test is common. Marrow biopsy is normal.

TREATMENT ITP is treated with corticosteroids, IVIG or Rho immune globulin (RhIG), and platelet transfusions, if severe bleeding is present. Splenectomy results in remission.

www.ClinicalReview.com


Hematology and Oncology

189

TABLE 278 IMMUNE THROMBOCYTOPENIC PURPURA (ITP) Immune Thrombocytopenic Purpura (ITP) Etiology

Idiopathic

Presentation

Intracranial hemorrhage or bleeding in other parts of the body. Petechiae and ecchymoses. Neurologic exam may be positive for findings, and a hemopericardium may be identified.

Diagnosis

CBC and large platelets found on peripheral blood smear. Antiplatelet antibodies may also be present. A positive Coombs test is common.

Treatment

Corticosteroids, IVIG or RhIG, and platelet transfusions if severe bleeding is present. Splenectomy results in remission.

VON WILLEBRAND DISEASE ETIOLOGY AND PATHOPHYSIOLOGY Von Willebrand disease (vWD) is a bleeding diathesis that prevents hemostasis in response to vascular damage. Adhesion to platelets is impaired and stabilization of various coagulation factors never occurs. vWD is rather rare and morbidity varies. vWD is an inherited autosomal condition with onset at a young age; females especially present at onset of menarche.

PRESENTATION AND DIAGNOSIS vWD presents with bleeding diatheses leading to epistaxis, easy bruising, and hematoma formation. Significant menstrual bleeding is possible. GI bleeds do not occur as frequently. A deficiency in von Willebrand factor (vWF) is diagnostic. This activity of vWF can be measured using a ristocetin activity test, while the presence of vWF can be determined with an antigen test. A PTT is increased in vWD, while PT is normal. The most severe vWF disease occurs in type III disease.

TREATMENT Treatment of type I vWD involves DDAVP which leads to a rise in vWF due to release from storage vesicles. Type II vWD is treated with DDAVP as well, but concentrates with factor VIII and vWF may be necessary prior to surgery. Treatment of type III vWD involves vWF-containing factor VIII concentrates. Platelet transfusions are also therapeutic. TABLE 279 VON WILLEBRAND DISEASE (VWD) Von Willebrand Disease (vWD) Etiology

Autosomal disorder.

Presentation

Bleeding diatheses leading to epistaxis, easy bruising, and hematoma formation

Diagnosis

Ristocetin activity test, while the presence of vWF can be determined with an antigen test. PTT is increased, while PT is normal.

Treatment

Type I vWD involves DDAVP. Type II vWD is treated with DDAVP as well, but concentrates with factor VIII and vWF may be necessary prior to surgery. Type III vWD-- vWF-containing factor VIII concentrates. Platelet transfusions.

HEMOPHILIA A

Â


190

USMLE STEP 2

ETIOLOGY AND PATHOPHYSIOLOGY Hemophilia A is an X-linked recessive disorder with factor VIII leading to a bleeding diathesis through disruption of the normal coagulation cascade. Spontaneous bleeding can occur, and significant bleeding with trauma is possible. Incidence is 1 in 5,000 persons but the lifespan is normal if no blood-borne illnesses are transmitted through transfusion. Males are significantly more affected; females tend to be carriers.

PRESENTATION AND DIAGNOSIS Hemorrhage occurs frequently with hemarthrosis, CNS complaints, GI bleeds, genitourinary bleeds such as hematuria, epistaxis, hemoptysis, compartment syndromes from hematomas, and contusions. Physical exam elicits signs of hemorrhage including tachycardia, tachypnea, hypotension, and orthostatic hypotension. Lab studies indicate a normal PT, elevated aPTT, normal platelet count, and deficits in factor VIII levels.

TREATMENT Hemophilia A is treated with factor VIII infusions (recombinant factor VIII is preferred). Oral hemorrhage may be treated with a combination of factor VIII and epsilon aminocaproic acid to reduce fibrinolysis. DDAVP may increase levels of factor VIII. TABLE 280 HEMOPHILIA A Hemophilia A Etiology

X-linked recessive disorder in factor VIII.

Presentation

Hemorrhage occurs frequently with hemarthrosis, CNS complaints, GI bleeds, genitourinary bleeds, epistaxis, hemoptysis, compartment syndromes from hematomas, and contusions.

Diagnosis

Tachycardia, tachypnea, hypotension, and orthostatic hypotension. Lab studies indicate a normal PT, elevated PTT, normal platelet count, and deficits in factor VIII.

Treatment

Factor VIII infusions. Oral hemorrhage may be treated with a combination of factor VIII and epsilon aminocaproic acid. DDAVP may increase levels of factor VIII.

HEMOPHILIA B ETIOLOGY AND PATHOPHYSIOLOGY Hemophilia B is an X-linked recessive disorder that leads to defects in factor IX and subsequent hemorrhage. 1 in 5,000 persons are affected and lifespan is normal in the absence of blood-borne illness from transfusion.

PRESENTATION AND DIAGNOSIS Hemophilia B presents like hemophilia A. Hemoglobin, PT, and platelets are normal. aPTT is increased. Factor IX is decreased significantly.

TREATMENT Hemophilia B is treated with recombinant factor IX infusion; epsilon aminocaproic acid is used with oral bleeds to prevent fibrinolysis. www.ClinicalReview.com


Hematology and Oncology

191

TABLE 281 HEMOPHILIA B Hemophilia B Etiology

X-linked recessive disorder in factor IX.

Presentation

Like hemophilia A.

Diagnosis

Hemoglobin, PT, and platelets are normal. aPTT is increased. Factor IX is decreased significantly.

Treatment

Recombinant factor IX infusion; epsilon aminocaproic acid is used with oral bleeds.

VITAMIN K DEFICIENCY ETIOLOGY AND PATHOPHYSIOLOGY Vitamin K is a lipid soluble vitamin that is essential for the formation of clotting factors. It is produced by colonic bacteria. Terminal ileum disease prevents normal vitamin K production and absorption. It presents as hemorrhagic disease of newborns (HDN) in infants; in adults, it presents as a bleeding diathesis. Other causes include parenchymal liver disease such as cirrhosis, in which case vitamin K supplements have little effect (fresh frozen plasma [FFP] is required), malabsorption syndromes, biliary disease, cholestyramine, coumadin, and various other medications (INH, rifampin, barbiturates, and others), lupus anticoagulant, DIC, polycythemia vera, cystic fibrosis, and leukemia.

PRESENTATION AND DIAGNOSIS Vitamin K deficiency, if severe enough, presents as complaints of significant hemorrhage following mild trauma. Ecchymoses, petechiae, hematomas, and oozing of blood are common. GI bleeds, hematuria, menorrhagia, epistaxis, and mucosal bleeds occur frequently. PT and aPTT are elevated. Des-gamma-carboxy prothrombin (DCP) is present in the absence of vitamin K.

TREATMENT Treatment for vitamin K deficiency involves correcting the cause of the underlying deficit and providing vitamin K supplements. FFP is necessary in severe disease. Subcutaneous injections of phylloquinone (vitamin K 1) can be given; menadione (vitamin K 3) can be given orally in malabsorption syndromes. Phytonadione can also be directly injected in severe disease. Green leafy vegetables and oils provide a good source of vitamin K. TABLE 282 VITAMIN K DEFICIENCY Vitamin K Deficiency Etiology

Terminal ileum disease. Other causes include parenchymal liver disease, malabsorption syndromes, biliary disease, cholestyramine, coumadin, and various other medications, lupus anticoagulant, DIC, polycythemia vera, cystic fibrosis, & leukemia.

Presentation

Significant hemorrhage following mild trauma. Ecchymoses, petechiae, hematomas, and oozing of blood. GI bleeds, hematuria, menorrhagia, epistaxis, and mucosal bleeds.

Diagnosis

PT and aPTT are elevated. DCP is present in the absence of vitamin K.

Treatment

Correct the cause of the underlying deficit and providing vitamin K supplements. FFP is necessary in severe disease.

MALARIA

Â


192

USMLE STEP 2

ETIOLOGY AND PATHOPHYSIOLOGY Malaria is the result of infection by Plasmodium protozoa carried by the Anopheles mosquito leading to potentially serious illness. Plasmodium species that can cause infection include P. falciparum, P. vivax, P. ovale, and P. malariae. P. falciparum can be fatal. P. falciparum and P. vivax are commonly responsible for new infections. Infection begins after a mosquito transmits the infection to a human host. The liver phase begins first with transmission to hepatocytes. Blood borne illness occurs a few weeks later with merozoites developing into trophozoites then schizonts inside RBCs. The erythrocyte phase begins, and merozoites are released a short time later when the RBC undergoes lysis. Fever occurs, and additional RBCs are infected. The immune system now steps in to control the infection, but death can occur with P. falciparum before a sufficient host reaction develops. Gametocytes can form during this process and lead to repeated infections. Nearly 3 million deaths occur annually throughout the world due to malaria; malaria is not endemic to the United States.

PRESENTATION AND DIAGNOSIS Constitutional symptoms can be quite severe with malaria and a flu-like illness may develop. P. falciparum in particular can induce coma due to cerebral infection leading to seizure. Severe anemia is also possible, along with renal failure due to blockade of vessels supplying the renal cortex. Pulmonary edema is also present in some cases. P. falciparum and, more rarely, the other causes of malaria can also cause death due to splenic rupture. Diagnosis is made by blood smears and identification of the parasites. A rapid dipstick test for P. falciparum exists.

TREATMENT Treatment for malaria includes adequate prophylaxis to avoid infection. Atovaquone / proguanil are commonly prescribed for prophylaxis, but it is not for use by pregnant women or children. Doxycycline is also often used, but the same contraindications apply. Mefloquine is safe for use by pregnant women; contraindications include patients with depression or other psychiatric illnesses, a history of seizures, and those with cardiac conduction abnormalities. Primaquine is used in certain situations, but G6PD deficiency is an absolute contraindication, along with pregnant women. Chloroquine, hydroxychloroquine, insect repellants (such as N,N-diethyl-m-toluamide [DEET]) are also used. TABLE 283 MALARIA Malaria Etiology

Infection by Plasmodium protozoa.

Presentation

Flu-like illness may develop, coma, seizure, severe anemia is also possible, renal failure. Pulmonary edema & splenic rupture are possible.

Diagnosis

Diagnosis is made by blood smears and identification of the parasites. A rapid dipstick test for P. falciparum also exists.

Treatment

Atovaquone / proguanil & doxycycline are commonly prescribed for prophylaxis. Mefloquine is safe for use by pregnant women. Chloroquine, hydroxychloroquine, & insect repellant are also used.

ONCOLOGY ACUTE MYELOCYTIC LEUKEMIA (AML) ETIOLOGY AND PATHOPHYSIOLOGY

www.ClinicalReview.com


Hematology and Oncology

193

Acute myelogenous leukemia (AML) is a malignant bone marrow disease leading to arrest of cells in the marrow leading to anemia, thrombocytopenia, and neutropenia. Cell proliferation occurs secondarily, leading to sequestration in the liver and spleen. AML kills some 7,000 people annually in the US, and it is common in Caucasians males more than in other groups. The risk of contracting AML increases with age, especially those over 65. Causes of AML include hematologic disorders such as MDS, myelofibrosis, aplastic anemia, PNH, PV, and CML, a congenital predisposition such as in Down syndrome or Fanconi anemia, familial syndromes such as Li-Fraumeni syndrome, exposure to certain environmental toxins such as benzene, and exposure to various chemotherapeutic agents, especially alkylating compounds and topoisomerase-II inhibitors.

PRESENTATION AND DIAGNOSIS AML presents with symptoms of bone marrow failure, including signs and symptoms of anemia, neutropenia, and thrombocytopenia. WBC counts are paradoxically increased even with the decrease in neutrophils due to the increased risk of infection and numerous URIs. Infiltration of various organs by leukemic cells can also lead to splenomegaly, gum infiltration causing gingivitis, altered mental status, and bone pain. DIC is a common presentation leading to elevated PT, and is especially common in M3 AML. Bone marrow biopsy with flow cytometry is diagnostic and permits subtype categorization.

CLASSIFICATION AML is classified into one of 8 subtypes. M0 is undifferentiated leukemia, M1 is myeloblastic without differentiation, M2 is differentiated myeloblastic, M3 is promyelocytic, M4 is myelomonocytic, M5 is monoblastic, M6 is erythroleukemia, and M7 is megakaryoblastic.

TREATMENT Treatment for AML involves infusion of daunorubicin, idarrubicin, or mitoxantrone with arabinosylcytosine with monitoring of organ function including heart, liver, and kidneys. Bone marrow transplantation (BMT) is used by many centers. M3 AML (acute promyelocytic leukemia) presents with a pancytopenia and is treated with chemotherapy and steroids. Relapses of AML have a terrible prognosis. Supportive care and antibiotic prophylaxis is standard of care with all AML. Growth factors such as GM-CSF are also occasionally used. TABLE 284 ACUTE MYELOCYTIC LEUKEMIA (AML) Acute Myelocytic Leukemia (AML) Etiology

Hematologic disorders, myelofibrosis, aplastic anemia, PNH, PV, and CML, a congenital predisposition & familial syndromes, exposure to certain environmental toxins, and exposure to various chemotherapeutic agents.

Presentation

Symptoms of bone marrow failure, including signs and symptoms of anemia, neutropenia, and thrombocytopenia. Splenomegaly, gingivitis, altered mental status, and bone pain. DIC is a common presentation.

Diagnosis

Bone marrow biopsy with flow cytometry.

Classification

M0-undifferentiated leukemia, M1-myeloblastic without differentiation, M2- differentiated myeloblastic, M3- promyelocytic, M4- myelomonocytic, M5- monoblastic, M6- erythroleukemia, and M7-megakaryoblastic.

Treatment

Daunorubicin, idarrubicin, or mitoxantrone with arabinosylcytosine with monitoring of organ function including heart, liver, and kidneys. BMT is used by many centers. M3 AML presents with a pancytopenia and is treated with chemotherapy and steroids. Supportive care and antibiotic prophylaxis is standard of care with all AML. Growth factors such as GM-CSF are also occasionally used.

ACUTE LYMPHOCYTIC LEUKEMIA (ALL)

Â


194

USMLE STEP 2

ETIOLOGY AND PATHOPHYSIOLOGY Acute lymphoblastic leukemia (ALL) is the replacement of the bone marrow by early lymphoid precursors similar to B and T cell precursors. As with AML, anemia, thrombocytopenia, and neutropenia occur. ALL presents most commonly in children.

PRESENTATION AND DIAGNOSIS ALL presents with symptoms of marrow failure and secondary disease from leukemic infiltrate. Splenomegaly is common and a Tcell mass leading to mediastinal impingement can occur. Very high WBC counts are possible leading to leukostasis, but this is less common than in AML. Anemia and infections from the neutropenia complete the presentation of ALL. Diagnosis is made in the same manner as AML with the addition of a CXR to identify any thymic masses. ALL typically has the Philadelphia chromosome [t(22;9)].

TREATMENT Few adults with ALL are cured. Treatment involves a regimen similar to that of AML including induction agents followed by consolidation agents. CNS prophylaxis is necessary with AML to avoid relapse, and intrathecal chemotherapy with cyclophosphamide, vincristine, doxorubicin, and dexamethasone. B-cell ALL also receives methotrexate and cyclophosphamide. BMT appears to be beneficial in some patients. As with AML, supportive therapy and infection prophylaxis complete the standard of care. A neutropenic diet is recommended in which only cooked foods are eaten and no fresh vegetables or fruits are consumed. TABLE 285 ACUTE LYMPHOCYTIC LEUKEMIA (ALL) Acute Lymphocytic Leukemia (ALL) Etiology

Replacement of the bone marrow by early lymphoid precursors.

Presentation

Marrow failure and secondary disease from leukemic infiltrate. Splenomegaly is common and a T-cell mass leading to mediastinal impingement can occur. Anemia and infections.

Diagnosis

Diagnosis is made in the same manner as AML with the addition of a CXR. ALL typically has the Philadelphia chromosome.

Treatment

Similar to that of AML including induction agents followed by consolidation agents. CNS prophylaxis is necessary with AML to avoid relapse, and intrathecal chemotherapy. Supportive therapy and infection prophylaxis complete the standard of care. B-cell ALL also receives methotrexate and cyclophosphamide. BMT appears to be beneficial in some patients. A neutropenic diet is recommended in which only cooked foods are eaten and no fresh vegetables or fruits are consumed.

CHRONIC MYELOID LEUKEMIA (CML) ETIOLOGY AND PATHOPHYSIOLOGY Chronic myelogenous leukemia (CML) is a granulocytic cell proliferation and subsequent differentiation leading to flooding of the vasculature with numerous granulocytes and precursors of granulocytes. The Philadelphia chromosome is common in CML leading to a bcr-abl fusion gene product. Most patients have a chronic form that leads to a blast crises after several years. CML accounts for 20% of all leukemias in adults, and is especially common in late middle-age. Risk factors include radiation exposure and benzene exposure.

www.ClinicalReview.com


Hematology and Oncology

195

PRESENTATION AND DIAGNOSIS CML presents as a chronic splenomegaly, leukocytosis, thrombocytopenia, neutropenia and anemia. Within a few years a blast crisis may occur, leading to leukemic infiltrates and worsening of the anemia and thrombocytopenia. A basophilia may occur along with profound splenomegaly. The blast phase has a poor prognosis. Prior to the blast crisis, CML is typically asymptomatic and discovered only incidentally. A low-grade fever may be present. An elevated WBC count around 50,000 is present on CBC and can reach over 100,000 in the blast crisis. Myeloid precursors found in a peripheral blood smear clinch the diagnosis. A hypercellular marrow with mild fibrosis is seen on marrow biopsy.

TREATMENT The chronic phase of CML can be easily controlled with hydroxyurea, interferon alpha, or busulfan for many years. Protein tyrosine kinase inhibitors against bcr-abl are a relatively new form of treatment; imatinib mesylate can be used for this purpose and leads to a high rate of response with a subsequently good prognosis. Interferon alpha is another medication often used in CML therapy. BMT is used in younger patients. Splenectomy can be palliative. TABLE 286 CHRONIC MYELOID LEUKEMIA (CML) Chronic Myeloid Leukemia (CML) Etiology

Philadelphia chromosome is common.

Presentation

Chronic splenomegaly, leukocytosis, thrombocytopenia, neutropenia and anemia. Within a few years a blast crisis may occur. A basophilia may occur. Prior to the blast crisis, CML is typically asymptomatic and discovered only incidentally. A low-grade fever may be present. An elevated WBC.

Diagnosis

Myeloid precursors found in a peripheral blood smear clinch the diagnosis. A hypercellular marrow with mild fibrosis is seen on marrow biopsy.

Treatment

Hydroxyurea, interferon alpha, or busulfan for many years. Protein tyrosine kinase inhibitors against bcr-abl are a relatively new form of treatment; imatinib mesylate can be used for this purpose and leads to a high rate of response with a subsequently good prognosis. BMT is used in younger patients. Splenectomy can be palliative.

CHRONIC LYMPHOCYTIC LEUKEMIA (CLL) ETIOLOGY AND PATHOPHYSIOLOGY Chronic lymphocytic leukemia (CLL) is a replacement of the marrow by an incompetent monoclonal lymphocytic population of cells leading to the production of large amounts of autoantibodies. Morbidity and mortality are very high.

PRESENTATION AND DIAGNOSIS CLL presents acutely with numerous infections such as pneumonia and reactivation of VZV, Lymphadenopathy, splenomegaly, sequelae of thrombocytopenia, and numerous constitutional symptoms. Hepatomegaly, pallor, and petechiae are common. Lymphocytes number around 5,000 and a lymphocytosis is present in the peripheral blood smear. Flow cytometry is used to confirm the diagnosis. The specific type of lymphocytosis is confirmed by examination of surface immunoglobulins.

TREATMENT Therapy for CLL involves treatment of symptomatic disease with prednisone, nucleoside analogues such as fludarabine, cladribine, and pentostatin, and combination agents using drugs similar to those for other leukemias. Monoclonal antibodies such as

Â


USMLE STEP 2 196 rituximab also appear to be successful in controlling this disease. Infection prophylaxis is essential due to the significant hypogammaglobulinemia and numerous autoantibodies present. Splenectomy is beneficial. TABLE 287 CHRONIC LYMPHOCYTIC LEUKEMIA (CLL) Chronic Lymphocytic Leukemia (CLL) Etiology

Replacement of the marrow by an incompetent monoclonal lymphocytic population of cells leading to the production of large amounts of autoantibodies.

Presentation

Numerous infections, lymphadenopathy, splenomegaly, sequelae of thrombocytopenia, and numerous constitutional symptoms. Hepatomegaly, pallor, and petechiae are common.

Diagnosis

Flow cytometry is used to confirm the diagnosis. The specific type of lymphocytosis is confirmed by examination of surface immunoglobulins.

Treatment

Treatment of symptomatic disease with prednisone, nucleoside analogues, and combination agents using drugs similar to those for other leukemias. Monoclonal antibodies. Infection prophylaxis is essential. Splenectomy is beneficial.

MONOCLONAL GAMMOPATHY OF UNCERTAIN SIGNIFICANCE ETIOLOGY AND PATHOPHYSIOLOGY Monoclonal gammopathy of uncertain significance (MGUS) is a plasma cell dyscrasia that leads to the production of M-protein, but not to the damaging levels as in multiple myeloma (MM). MGUS is typically an incidental finding which generally merits careful observation. MGUS is a risk factor for future MM development. Increasing age is a risk factor. Nearly 3% of those over 70 have MGUS. Evolution of MGUS to MM can occur in up to 15% of patients within 10 years.

PRESENTATION AND DIAGNOSIS MGUS is a subclinical disorder discovered incidentally by the presence of an M-protein, plasma cells that constitute less than 15% of the marrow, and inconsequential Bence-Jones protein in the urine. No lytic bone lesions are present, and there is no marrow disturbance.

TREATMENT MGUS requires no specific treatment other than annual serum protein electrophoresis to identify whether MM develops. TABLE 288 MONOCLONAL GAMMOPATHY OF UNCERTAIN SIGNIFICANCE (MGUS) Monoclonal Gammopathy of Uncertain Significance (MGUS) Etiology

Plasma cell dyscrasia.

Presentation and Diagnosis

Subclinical disorder discovered incidentally by the presence of an M-protein, plasma cells and inconsequential Bence-Jones protein in the urine. No lytic bone lesions are present, and there is no marrow disturbance.

Treatment

No specific treatment other than annual serum protein electrophoresis to identify whether MM develops.

MULTIPLE MYELOMA (MM)

www.ClinicalReview.com


Hematology and Oncology

197

ETIOLOGY AND PATHOPHYSIOLOGY MM is a serious malignancy that leads to excessive monoclonal protein production by plasma cells causing systemwide effects and eventually death. There are more cases in African American males than in other groups for a total incidence of 4 cases per 100,000 persons. There are numerous sequelae to MM. HLA-Cw2 and HLA-Cw5 are suspected of playing a role in a genetic susceptibility to developing MM.

PRESENTATION AND DIAGNOSIS MM leads to lytic bone lesions contributing to bone pain and susceptibility to fractures. A typical pattern is seen on plain films of the skull and other bones. These lytic bone lesions can contribute to hypercalcemia. Cord compression can also occur. Bleeding diatheses may occur from the thrombocytopenia, and increased tumor volume may lead to a hyperviscosity syndrome with epistaxis, CVA, MI, and easy bruising. The leukopenia may contribute to an increased risk of infection. Neurologic symptoms are also common, such as sensory loss, muscle weakness, and carpal tunnel syndrome. Amyloidosis occurs in a subset of patients with MM and presents with macroglossia and peripalpebral purpura. Protein electrophoresis is used to identify the Bence-Jones proteins. The presence of a beta-2 microglobulin indicates a poorer prognosis. A complete skeletal scan is done with MM to identify impending fractures and sites of bone lesions.

TREATMENT Radiation therapy is used to mitigate pain and is highly successful for MM management. Bisphosphonates are added to minimize the impact of lytic bone lesions and reduce the risk of fractures. Corticosteroids minimize the symptoms of spinal cord compression. Plasmapheresis and hydration are used with renal failure. Autologous BMT is preferred following ablation of the bone marrow. Two sequential BMTs appear to be more beneficial in these patients. Melphalan and prednisone are the two most commonly used chemotherapeutic agents for MM. TABLE 289 MULTIPLE MYELOMA (MM) Multiple Myeloma (MM) Etiology

Excessive monoclonal protein production by plasma cells.

Presentation

Bone pain and susceptibility to fractures. Lytic bone lesions, hypercalcemia. Cord compression. Bleeding diatheses, and increased tumor volume may lead to a hyperviscosity syndrome with epistaxis, CVA, MI, and easy bruising, leukopenia. Neurologic symptoms are also common.

Diagnosis

Protein electrophoresis is used to identify the Bence-Jones proteins, complete skeletal scan.

Treatment

Radiation therapy, bisphosphonates, corticosteroids. Plasmapheresis and hydration are used with renal failure. Autologous BMT is preferred following ablation of the bone marrow. Two sequential BMTs appear to be more beneficial in these patients. Melphalan and prednisone are the two most commonly used chemotherapeutic agents.

HODGKIN LYMPHOMA (HL) ETIOLOGY AND PATHOPHYSIOLOGY Hodgkin lymphoma (HL) is a distinct malignant lymphoma with a clonal B-cell population proliferating as Reed-Sternberg (RS) cells. These cells propagate the effects of HL while numerous inflammatory cells lead to local insult and injury. HL accounts for fewer

Â


USMLE STEP 2 198 than 1% of all cancers and has a decent 5 year survival rate, if caught early. It is more common in Caucasian males. It has been postulated that infection by EBV is a predecessor to HL.

PRESENTATION AND DIAGNOSIS HL presents with supradiaphragmatic lymphadenopathy (typically seen in the neck and axilla), numerous constitutional symptoms, chest pain, intermittent fever, and pruritus. Hepatosplenomegaly is typically evident on physical exam. On laboratory workup, ESR and LDH are elevated. CBC typically indicates an anemia of chronic disease. CT scans are used to identify the extent of disease, and flow cytometry is the key for diagnosis.

TREATMENT HL is treated with radiation therapy and chemotherapy. The MOPP or ABVD regimen is often used; MOPP stands for the trade names of mechlorethamine, vincristine, procarbazine, and prednisone and ABVD stands for adriamycin, bleomycin, vinblastine, and dicarbazine. Other regimens are also used for HL therapy. BMT is sometimes also used. PET scans are used to assess the success of the therapy. TABLE 290 HODGKIN LYMPHOMA (HL) Hodgkin Lymphoma (HL) Etiology

Clonal B-cell population proliferating as RS cells.

Presentation

Supradiaphragmatic lymphadenopathy (typically seen in the neck and axilla), numerous constitutional symptoms, chest pain, intermittent fever, and pruritus. Hepatosplenomegaly is typically evident on physical exam.

Diagnosis

ESR and LDH are elevated. CBC typically indicates an anemia of chronic disease. CT scans are used to identify the extent of disease, and flow cytometry is the key for diagnosis.

Treatment

Radiation therapy and chemotherapy. MOPP or ABVD regimen is often used. BMT is sometimes also used. PET scans are used to assess the success of the therapy.

NON-HODGKIN LYMPHOMA ETIOLOGY AND PATHOPHYSIOLOGY (NHL) Non-Hodgkin lymphoma (NHL) is a lymphoid tumor with several distinct presentations. It may be due to malignant expansion of B cells, T cells, natural killer (NK) cells, or macrophages, but the majority are due to B cell expansion. NHL leads to over 25,000 deaths a year with over 50,000 new cases annually. Patients tend to be middle-aged adults at the time of diagnosis. Causes of NHL include a chromosomal translocation that predisposes to the patient to the lymphoma, history of infection by EBV, human Tcell leukemia virus (HTLV), HCV, and herpesvirus 8 (HHV 8), exposure to certain environmental toxins or chemotherapeutic agents, various congenital causes (severe combined immunodeficiency disease [SCID]), a state of chronic inflammation, and H. pylori infection.

PRESENTATION AND DIAGNOSIS NHL presents as a painless peripheral adenopathy, extension to the bone marrow and subsequent pancytopenia, multiple constitutional symptoms, and extranodal manifestations. Bowel obstruction may occur, along with significant growth of the lymphoma leading to cranial nerve impingement. Hepatosplenomegaly is common, and more advanced disease may also present www.ClinicalReview.com


Hematology and Oncology

199

with testicular enlargement, skin lesions, and a mediastinal mass. Workup includes a CBC to detect the extent of the pancytopenia and lymphocytosis, elevation of LDH (which is tied to prognosis), and screen for involvement of other organs through various enzyme function tests. Imaging studies help to identify the extent of the tumor.

TREATMENT Early stage NHL is treated with radiation therapy, but chemotherapy is sometimes used in high-risk patients. More advanced stages have a combination of radiotherapy and chemotherapy with the CHOP regimen (trade names for cyclophosphamide, hydroxydaunomycin, vincristine, and prednisone) or CVP (cyclophosphamide, vincristine, and prednisone). Monoclonal antibodies such as rituximab have been used with success. More aggressive tumors are treated with high dose chemotherapy, radiotherapy, followed by BMT. TABLE 291 NON-HODGKIN LYMPHOMA (NHL) Non-Hodgkin Lymphoma (NHL) Etiology

Due to malignant expansion of B cells, T cells, NK cells, or macrophages, but the majority are due to B cell expansion.

Presentation

Painless peripheral adenopathy, extension to the bone marrow and subsequent pancytopenia, multiple constitutional symptoms, and extranodal manifestations. Bowel obstruction may occur, along with significant growth of the lymphoma leading to cranial nerve impingement. Hepatosplenomegaly is common, and more advanced disease may also present with testicular enlargement, skin lesions, and a mediastinal mass.

Diagnosis

CBC, elevation of LDH, and screen for involvement of other organs through various enzyme function tests. Imaging studies help to identify the extent of the tumor.

Treatment

Radiation therapy, chemotherapy (CHOP regimen or CVP) is sometimes used in high-risk patients. Monoclonal antibodies. A More aggressive tumor is treated with high dose chemotherapy, radiotherapy, followed by BMT.

B-CELL LYMPHOMAS PRECURSOR B- AND T-CELL LYMPHOMAS TYPE

Precursor B-cell / T-cell ALL

Lymphoblastic lymphoma

ETIOLOGY

PRESENTATION

NOTES

t(9:22), t(4:11), t(2:8), t(8:14), negative myeloperoxidase stain, positive TdT bcr-abl p190 type.

Anemia, pallor, fever, pneumonia, thrombocytopenia with petechiae, DIC, HSM, LAD, skin infiltration, increased WBC count but decrease in neutrophil count.

Develops into CLL.

LDH, TdT???, flow cytometry.

Most commonly affects T cells (80%). Similar to ALL, but abnormal cells typically found in thymus or lymph nodes. May lead to marrow and renal effects. LAD, mediastinitis, anorexia, night sweats, high fever.

Aggressive. Rare in adults. Common in children (1 in 3 lymphomas??).

Increase in risk of development of these tumors comes from chromosomal translocations, environmental factors such as plastics, pesticides, paint, rubber, and synthetics, organophosphates, benzene, wood preservatives, chemotherapy recipients, EBV (Burkitt lymphoma), HTLV-1 (adult T-cell leukemia / lymphoma), HHV8 (HIV infection, lymphoma in body cavity + Kaposi sarcoma), immunodeficiency states, chronic infections, CTD???, celiac sprue, IBD, and H. pylori.

Â


200

USMLE STEP 2

PERIPHERAL B-CELL LYMPHOMAS TYPE

ETIOLOGY

PRESENTATION

NOTES

t(14:19)(q32:q13.1) bcl-3, bcl-2 CD20+, CD3-, CD10-, CD5+, CD23+

HSM, petechiae, pallor, bleeding, fatigue, anemia, LAD, pneumonia, HSV, VZV.

Indolent Consider splenectomy. Rx chlorambucil, fludarabine, alemtuzumab.

B-cell prolymphocytic leukemia

B-cell disease only.

CLL variant, mostly immature cells leading to splenomegaly, high WBC count. No LAD.

Aggressive

Lymphoplasmacytic lymphoma

t(9:14)(p13:q32) PAX-5

Also known as Waldenstrรถm macroglobulinemia (IgM). Hyperviscosity syndrome ร DVT/PE/MI/CVA.

Indolent. CHOP.

t(11:14)(q13:q32) bcl-2 CD20+, CD3-, CD10-, CD5+, CD23-, CD43+, PRAD1+

5% of adult NHL. Occurs in older adult males. Found in lymph nodes, spleen, marrow, and GI tract.

Aggressive. 5 year survival. Treat with CHOP.

t(14:18)(q32:q21) t(3:22)(q27:q11) bcl-2 CD20+, CD3-, CD10+, CD5-

Small cleaved cells and large centroblast cells. Small cleaved cells dominate. Lymphocytosis in some. Para-trabecular lymphoid aggregates in bone marrow. Involves spleen and liver.

Aggressive

B-cell MALToma

t(1:14)(p22:q32) t(11:18)(q21:q21) bcl-10 CD20+, CD3-, CD10-, CD5-, CD23-

Found in GI tract, lungs, exocrine glands, endocrine glands, and lacrimal glands. Associated with chronic H. pylori disease and autoimmune disease.

Indolent Treat with eradication of H. pylori and excise tumor. Evolves to diffuse large B cell lymphoma.

Nodal / splenic marginal zone lymphoma

CD20+, CD3-, CD10-, CD5-, CD23-

Splenomegaly, no LAD, some marrow involvement.

Indolent Treat with splenectomy. Similar Rx as CLL.

Hairy cell leukemia

Hairlike projections. CD19+, CD20+, CD22+, CD21-. Bly-7. TRAP??? staining.

Clonal B cell penetration into bone marrow with HSM, low-grade fever, and atypical mycobacterial infections. Bone marrow failure.

Indolent Rx cladribine, pentostatin, IFN.

Diffuse large B-cell lymphoma

t(14:18)(q32:q21) t(3:22)(q27:q11) bcl-2

Generalized form of more indolent NHL. Large cell size, diffuse growth, oval nucleus with margination of chromatin, several multilobulated nucleolus. Basophilic cytoplasm with Reed-Sternberg-like inclusions.

Aggressive. Treat with CHOP +/- radiation.

Burkitt lymphoma

8q24 translocation t(8:14)(q24:q32) t(2:8)(p11-12:q24) t(8:22)(q24:q11) c-myc CD20+, CD3-, CD10+, CD5-, TDT-

Found mostly in children. Uniform nuclear size and contour. Risk of CNS involvement. Diffuse lymphoid infiltrate, cells approximately 15 um in diameter with several nucleoli. High mitotic index. Starry sky pattern.

Aggressive

B-cell CLL / Small lymphocytic lymphoma

Mantle cell lymphoma

Follicular lymphoma

The general presentation of these tumors is typically with LAD, fever, night sweats, weight loss, fatigue, and mass effects on other organ systems. A familial pattern is typically present. Most patients are over 50 at presentation; only high-grade lymphoblastic and small noncleaved cell lymphomas are found in children and young adults. Males are more likely to be affected than women. Caucasians are more affected than other races. NHL causes 5% of all cancer-related deaths. It is the leading cause of cancer death in young adult males.

www.ClinicalReview.com


Hematology and Oncology

201

PLASMA CELL DISORDERS TYPE

ETIOLOGY

PRESENTATION

Chromosome 13, 14q, 19p, 9q, 1q changes. IL6 present.

Solid mass of plasma cells in a particular location. May occur anywhere. Mostly in head, neck, or bone.

M protein. Related to IL6, IL1, IGF-1, FGF

Common in elderly, few symptoms or signs of disease. May rarely progress to multiple myeloma.

Multiple myeloma

HLA-Cw5, HLA-Cw2, agricultural work, petrochemicals, laxatives, hair dye, radiation.

Direct tubular injury, amyloidosis, plasmacytoma, renal failure, spinal cord compression, pathologic fractures due to lytic bone lesions, hypercalcemia, and mass effects. Anemia. Bence-Jones protein (IgA lambda), beta-2microglobulin, CRP.

Amyloidosis

HHV8, idiopathic.

Lambda class predominant Ig protein deposits leading to edema, neuropathy, macroglossia, CHF, nephrotic syndrome, purpura, diastolic dysfunction, ecchymoses, arrhythmia, and HSM.

Plasmacytoma Monoclonal gammopathy of undetermined significance

T-CELL LYMPHOMAS PERIPHERAL T-CELL LYMPHOMAS TYPE T-cell CLL / prolymphocytic

ETIOLOGY

PRESENTATION

lymphoma

Rare form of CLL. See notes for Bcell CLL.

Anorexia, fatigue, fever.

T-cell granular lymphocytic leukemia

CD2+, CD16+, CD56+ OR CD2+, CD3+, CD8+

Neutropenia, history of RA. Anemia, splenomegaly. Felty syndrome is related.

Mycosis fungoides

CD4+, CD7-, CLA+

Migration to skin, skin rash with pruritus early, followed by plaques, patches, erythema, then tumors and erythroderma. LAD and visceral involvement late with IC.

Peripheral T-cell lymphoma, NOS

CD20-, CD3+

Hepatosplenic gamma/delta T-cell lymphoma

V-delta gene. CD4-, CD8-

Very rare. Plaques, nodules on extremities, HSM, infiltration in various organs. On test?? Or eliminate.

Angioimmunoblastic T-cell lymphoma

CD2+, CD3+, CD4+, CD5+, CD7-; mature cells; LMP1 deletion; EBV association.

High fever, LAD, HSM, hemolytic anemia, polyclonal hypergammaglobulinemia. Skin infestations with morbilliform rash.

Extranodal T-cell/NK-cell lymphoma

Progression from low-grade cutaneous lymphoma with TCR rearrangement.

Enteropathy-type intestinal T-cell lymphoma

Jejunum lymphoma in celiac disease.

Adult T-cell lymphoma/leukemia, HTLV-1 Anaplastic large cell lymphoma Aggressive NK-cell leukemia

Retrovirus – human T-cell leukemia virus type 1.

Skin lesions, LAD, HSM, lytic bone lesions, hypercalcemia, CNS demyelination Æ lower limb spasticity, weakness

CD20-, CD3+, CD30+, CD15-, EMA+, ALK+, Ki-1, t(2:5)

Systemic or cutaneous forms. LAD.

CD3-, CD56+ (NCAM), strong EBV association.

High, unstable, spiking fevers. Common in nasopharynx.


USMLE STEP 2 202 LDH is a marker for bulky or aggressive disease. When it is positive, the prognosis is generally poor. HTLV-2 leads to atypical hairy cell leukemia, large granular lymphocytic leukemia, and lymphoma in HIV/AIDS.

HODGKIN LYMPHOMA TYPE

ETIOLOGY

HISTOLOGIC FEATURES

NOTES

Nodular sclerosis

CD30+, CD15+

Nodular pattern with thickened capsule. Lacunar-type ReedSternberg cell with multilobulated nucleus.

60-80% Treat with radiation and/or chemotherapy (BEACOPP).

Mixed cellularity

CD30+, CD15+

Diffuse infiltrate, large, bilobate Reed-Sternberg cells, eosinophilic inclusion bodies.

15-30%

Lymphocytedominant

CD30+, CD15+

Numerous lacunar-type cells with lymphocytic infiltrate.

5%

Lymphocyte depletion

CD30+, CD15+

Diffuse, hypocellular infiltrate with numerous Reed-Sternberg cells. Occurs in older patients and HIV+ patients. Confused with anaplastic large-cell tumors.

1%

Presentation for all: LAD, anorexia, fever, night sweats, chest pain, mass effects in chest, pruritus, back pain, HSM, SVC syndrome, Guillain-BarrĂŠ syndrome, leukoencephalopathy. Very rare. Bcl2 present, related to EBV (present in 50%of biopsies).

CHEMOTHERAPY REGIMENS DRUG

MEMBERS

CHOP

Cyclophosphamide Doxorubicin Vincristine Prednisone

BACOP

Bleomycin Doxorubicin Cyclophosphamide Vincristine Prednisone

MOPP

Prednisone Methotrexate Leucovorin Doxorubicin Cyclophosphamide Etoposide

NOTES

May include TMP-SMX +/- methotrexate (MACOP-B).

www.ClinicalReview.com


Hematology and Oncology

203

PRACTICE QUESTIONS A 61 year old male with von Willebrand disease presents with signs and symptoms of acute appendicitis. Which of the following is recommended to minimize the risk of bleeding in this patient? A. B. C. D. E. F.

Cryoprecipitate Fresh frozen plasma Packed red blood cells Platelet transfusion Vasopressin Whole blood transfusion

The best answer is Cryoprecipitate. Von Willebrand disease leads to a functional deficiency in platelet adherence to collagen. A prolonged bleeding time is the effect, and may be disastrous in surgery. Either cryoprecipitate or DDAVP can be given to help reverse this defect.

A 44 year old male diagnosed with hemophilia A is a candidate for a liver transplantation due to longstanding cryptogenic cirrhosis. Prior to undergoing this procedure, which of the following is the best infusion to minimize intraoperative bleeding? A. B. C. D. E.

Factor IX Factor VIII Fresh frozen plasma Packed red blood cells Platelet infusion

The best answer is Factor VIII. Recombinant factor VIII is the best treatment for patient with hemophilia A who require an operative intervention in order to minimize intraoperative bleeding. Factor VIII deficiency is manifest as an increase in PTT.

What is the half-life of albumin? A. B. C. D. E.

12 hours 2 days 2 months 3 weeks 4 hours

The best answer is 3 weeks. Albumin has a half-life of 21 days. Pre-albumin should be used as a measure of nutritional repletion as it has a half-life of 2 days.

Â


USMLE STEP 2

204

Which of the following is a major contributor to blood oxygen content leading to perfusion of peripheral tissues? A. B. C. D. E.

Heart rate Hemoglobin concentration and %SO2 Hemoglobin concentration and PaO2 PaO2 PAO2

The best answer is Hemoglobin concentration and %SO2. The greatest contributor to oxygenation is hemoglobin concentration and percent of oxygen saturation within the blood. The initial treatment to improve peripheral oxygenation is to increase oxygen saturation to 100%. Further increases lead to an increase in dissolved oxygen within the blood, but this has a negligible effect at low oxygen flow levels. The next step is to give blood transfusions as clinically indicated in order to improve oxygenation. The relevant equation is 1.37 x oxygen saturation x hemoglobin concentration + 0.003 x PaO2.

Which of the following leads to a decrease in the affinity of hemoglobin to oxygen? A. B. C. D. E.

Decrease in hemoglobin concentration Decreased 2,3 DPG levels Decreased CO2 levels Increased acidity Platelet transfusion

The best answer is Increased acidity. The affinity of hemoglobin to oxygen is decreased by a drop in pH, increase in 2,3 DPG levels, and an increase in CO2.

Which of the following has the greatest chance of being transmitted from one person to another in the course of a blood transfusion? A. B. C. D. E.

CMV HBV HCV HIV HTLV

The best answer is CMV. Cytomegalovirus is the most common infection transmitted from one person to another in the course of a blood transfusion, and routine screening is not done due to the high rates of endemic infection. Over 20% of patients are positive by age 20, and 70% are positive by age 70. White blood cells are the carrier for infection, and for this reason, washing blood when giving it to immunocompromised patients is recommended. The risk of contracting hepatitis A, B, or C is up to 1 in 30,000. The risk of HTLV is up to 1 in 250,000. The risk of HIV is over 1 in a million.

www.ClinicalReview.com


Hematology and Oncology

205

A 32 year old patient is found to have von Willebrand disease as part of a coagulation workup after experiencing excess bleeding following removal of his appendix. Which of the following best describes the source of vWF? A. B. C. D. E.

Endothelium Fibroblasts Macrophages Platelets White blood cells

The best answer is Endothelium. vWF is released by the endothelium. vWF serves as a carrier for factor 8 and assists in the binding of glycoprotein Ib on inactivated platelets and GP IIb/IIa on activated platelets. There are three times of deficits: type 1 is the most common and is the mildest – treatment involves DDAVP only; type 2 is somewhat worse, and type 3 is the most severe as it is a homozygous defect – type II and type III are treated with cryoprecipitate and factor IIX. Diagnosis is made by increased bleeding time and PTT; PT is normal. Of note, DDAVP is also used as an adjunct to reversing a coagulopathy in patients with ESRD and CRF.

Which of the following is one of the changes that occurs to packed red blood cells after prolonged storage? A. B. C. D. E.

Breakdown of 2,3 DPG Decrease in osmotic fragility Decrease in potassium Increase in calcium More basic pH

The best answer is Breakdown of 2,3 DPG. Stored blood tends to have very little 2,3 DPG, an increase in osmotic fragility, a decrease in calcium due to binding with citrate, an increase in potassium, and a more acidic pH. On a different note, platelet aggregation and vasoconstriction can be induced by thromboxane A2; production of TxA2 is inhibited via downregulation of prostaglandin conversion by aspirin. Other mediators that can cause platelet aggregation include serotonin and ADP. Release of platelet granules can occur via platelet factor 4. The administration of mismatched blood during an operation will present as bleeding.

A patient receiving coumadin has monitoring of his INR until it becomes therapeutic. Which of the following factors is NOT affected by coumadin? A. B. C. D. E.

Factor II Factor IX Factor V Factor VII Factor X

The best answer is Factor IX. Warfarin leads to a prolongation in PT / INR, which affects factors II, V, VII, and X. Fibrinogen is also affected. Liver disease also leads to similar effects. Warfarin administration can be reversed with the administration of vitamin K, which leads to gamma-carboxylation and increased concentrations of factors produced by the liver. Warfarin may be more active with the concomitant administration of alcohol, metronidazole, amiodarone, and cimetidine. Warfarin may be inhibited with rifampin, phenytoin, and barbiturates.


USMLE STEP 2

206

A 22 year old female who receives coumadin due to atrial fibrillation develops peripheral skin necrosis. Which of the following factor deficits can this be attributed to? A. B. C. D. E.

Factor II Factor IX Factor X Protein C Protein S

The best answer is Protein C. This patient has a deficiency in protein C, leading to warfarin-induced skin necrosis. A heparin to coumadin bridge is essential in preventing the onset of this hypercoagulable state. Protein C deficiency can occur in up to 5% of the population. Factor V Leiden deficiency is a related defect that prevents protein C from binding to factor V, leading to hypercoagulability. In the formation and breakdown of clots, there are several important mediators. One of them is urokinase, which is responsible for the breakdown of fibrin through activation of plasminogen. In the event of an overdose of this clot-breaking medication, amino-caproic acid and cryoprecipitate can be given. Heparin can lead to thrombocytopenia, which presents as white clots and is due to antibodies against the heparin. Platelet aggregation is the end result, leading to vascular thrombosis. Reversal of heparin with protamine may lead to hypotension.

A 36 year old female suffers significant blood loss during a lysis of adhesions when the surgeon inadvertently transects a major vessel. Once the patient is stabilized, which of the following changes is the body making to compensate for isovolemic anemia? A. B. C. D. E.

Decrease in cardiac output Increase in 2,3 DPG Increase in blood viscosity Increase in dissolved oxygen Increase in peripheral resistance

The best answer is Increase in 2,3 DPG. The body will seek to increase 2,3 DPG to promote greater unloading of oxygen along the oxygen-hemoglobin dissociation curve. Other effects will be a decrease in pH, increase in temperature, an increase in cardiac output to improve peripheral oxygen delivery, decreased blood viscosity to decrease resistance to flow, and decrease peripheral resistance to improve flow.

www.ClinicalReview.com


Hematology and Oncology

207

CHAPTER CONTENTS Toxicology .........................................................................................................208 Other Injuries ....................................................................................................210 Practice Questions ............................................................................................214

Â


208

USMLE STEP 2

EMERGENCY MEDICINE EMERGENCY MEDICINE TOXICOLOGY TABLE 292 TOXICOLOGY Toxicology Toxic Agent Acetaminophen

Presentation

Treatment

Fulminant hepatic failure and renal failure.

NAC

Amphetamine

Increased arousal and significant sympathomimetic effects. Mental status changes, dyskinesia, agitation, formication, chest pain, dry mouth, diarrhea, diaphoresis, HTN, and mydriasis .

Patient restraint and supportive therapy after protecting the airway. Activated charcoal and benzodiazepines can be used. Using neuroleptics to manage psychosis along with dantrolene to relax the skeletal muscles and avoid hyperthermia Phentolamine is used to reduce cardiac activity, and nitroglycerin is given to reduce pain.

Anticholinergics

Skin flushing, dry skin, mydriasis, and changes in mental status. Tachycardia, diminished bowel sounds, ileus, HTN, and myoclonus are also present. Both anticholinergics and certain antihistamines can exert these effects.

ACLS, naloxone, benzodiazepines, activated charcoal, IVF, and the antidote, and physostigmine salicylate are all given.

Antidepressant

TCAs lead to progressive decreases in CNS orientation and eventually coma. Seizures, hypotension, cardiac conduction abnormalities, hypoventilation, and various anticholinergic effects occur.

Treatment is sodium bicarbonate followed by IVF. Vasopressors may be used with severe hypotension. Activated charcoal is also used, along with epinephrine and benzodiazepines for seizure management.

Arsenic

Bloody diarrhea, vomiting, dehydration, QT prolongation, and hepatorenal damage.

Stabilize the patient and use BAL, DMSA, and DMPS.

Barbiturate

Respiratory and cardiac depression and decreased CNS function. Mental status changes and psychiatric changes are common. Decreased bowel sounds are found on exam. Bullous lesions may also be present.

Treat with ABCs, IVF ,ET intubation, administer activated charcoal, and alkalinize the urine with sodium bicarbonate. As in many instances of toxic substance consumption, hemodialysis and exchange transfusions may be required in the most serious cases.

Benzodiazepine

Similar to that of barbiturate poisoning and may be worsened with concomitant ETOH abuse. Confusion, drowsiness, blurred vision, hallucinations, ataxia, hypotonia, amnesia, respiratory depression, and coma.

ABCs, O2, glucose, naloxone, flumazenil to deactivate the benzodiazepine, and activated charcoal.

Beta-Blocker

Arrhythmia, hypotension, bradycardia, renal failure, dilated cardiomyopathy, seizure, bronchospasm, and coma.

ABCs, IVF, gastric lavage, activated charcoal, hemodialysis, close cardiac management including pacing, and atropine, epinephrine, dopamine, and isoproterenol for hypotension are the standard of care. Glucagon, calcium chloride, magnesium sulfate, and insulin are also frequently used to maintain function.

Carbon Monoxide

Constitutional symptoms, lethargy, DOE, depression, incontinence, and changes in memory. Tachycardia, hyperthermia, tachypnea, cherry red skin, flame retinal hemorrhages with red retinal veins, and psychiatric manifestations are common on exam.

Nonrebreather masks with continuous 100% oxygen, intubation, and hyperbaric oxygen.

www.ClinicalReview.com


Emergency Medicine

209

Caustic Agents

Strong acids or alkali leads to coagulation necrosis especially in the pharynx, esophagus, stomach, and SI. Presentation includes dyspnea, dysphagia, odynophagia, chest pain, abdominal pain, nausea and vomiting, airway obstruction, drooling, peritonitis, and hematemesis.

Diluting alkaline ingestions with water or milk is sometimes done. Emesis is contraindicated. Maintain the ABCs and attempt gastric lavage. Large volume liquid ingestion is often used. Do not use acids with base ingestions or vice versa to avoid heat production and additional damage. Antibiotics are necessary due to extensive tissue injury. Antihistamines are also administered to avoid esophageal damage from stomach acid.

Cocaine

Cardiac arrhythmia, MI, stroke, subarachnoid hemorrhage, malignant hyperthermia, and sudden death. Rhabdomyolysis can also occur. Numerous respiratory, cardiac, neurologic, gastrointestinal, and renal ailments may occur. Mesenteric ischemia is relatively common. CARO and psychiatric manifestations also occur, with significant constitutional symptoms in withdrawal. Systemwide depression is a sign of late stage intoxication.

ABCs, O2, IVF, naloxone, benzodiazepines for seizures, patient restraint, insulin and glucose, epinephrine with caution in cardiac arrest, beta-blockers, and pressor as necessary to avoid cardiovascular collapse. Polyethylene glycol and activated charcoal are also used. Give vitamin B1 before glucose.

Cyanide

A sense of impending doom, numerous constitutional symptoms, neurologic symptoms, SOB, nausea, vomiting, mydriasis, pulmonary edema, cardiac conduction changes, and coma are apparent. Carboxyhemoglobin is present along with methemoglobin.

ABCs, oxygen, sodium bicarbonate, antidotes to cyanide (sodium nitrite, sodium thiosulfate, and hydroxocobalamin), anticonvulsants, and vasopressors are used as indicated. Amyl nitrate is another antidote that is sometimes used.

Digitalis

Palpitations, syncope, dyspnea, CNS changes, yellow-green vision, photophobia, halos, scotomas, GI symptoms, and cardiac conduction changes are present. Toxicity may be the result of drug interactions (see text above).

Provide O2, IVF, and monitor for cardiac functioning. Activated charcoal and antibodies to digitalis are used. Repair any changes in electrolytes and use magnesium sulfate to maintain rhythm.

Ethylene Glycol

Kussmaul respirations due to the severe metabolic acidosis, altered mental status, and formation of oxalate crystals in the kidneys and throughout the body. Serum osmolal gap is present.

IVF, bicarbonate, pyridoxine, thiamine, and ethanol. 4methylpyrazole (4-MP, also known as fomepizole) has also been used with benefit. Dialysis is also used

GHB

GHB, the date-rape drug, presents with CNS depression, seizure disorder, bradycardia, hypotension, and some hypothermia.

Supportive therapy and maintaining the airway. Gastric lavage is sometimes used. Atropine is used to treat the bradycardia.

Hallucinogen

LSD, PCP, ketamine, mescaline, MDMA, and THC, among other psychoactive compounds, presents with hallucinations, psychosis, flashbacks, self-injurious behavior, agitation, abdominal symptoms, and diaphoresis.

Reassure the patient, sedate with benzodiazepines, and restraints are the initial steps. Haloperidol and benzodiazepines for psychosis, ventilatory assistance, and nitroglycerin for elevations in BP.

Heavy Metals

Lead exposure leads to GI dysfunction, encephalopathy, seizures, and anemia. Arsenic toxicity presents similar to lead toxicity, but a syndrome similar to Guillain-BarrĂŠ syndrome may develop. Mercury toxicity presents with tremor, hematochezia, esophagitis, acrodynia, stomatitis, salivation, and gingivitis.

Lead toxicity is treated PEG, chelation therapy with edetate calcium disodium (EDTA), dimercaprol (BAL), and DMSA, and supportive therapy. Arsenic toxicity is treated with fluid rehydration, PEG, and chelation with BAL, DMSA, or penicillamine. Mercury toxicity is treated with activated charcoal and chelation therapy with BAL, DMSA, and penicillamine. Succimer is another chelating agent similar to BAL.

Iron

GI tract corrosion and subsequent diarrhea and hematemesis. Cell death also occurs and affects the heart, kidney, and lung. Metabolic acidosis and hyperglycemia is common. Coagulopathy can develop with sufficient toxicity along with hepatic dysfunction and hypoglycemia.

Iron toxicity is treated with IVF, O2, and chelation with deferoxamine.

MDMA

Increased activity of the SNS leading to HTN, hyperthermia, and tachycardia. Serotonin syndrome may also develop leading to further breakdown in thermoregulation and rhabdomyolysis. DIC and hepatotoxicity may occur. Hyponatremia also occurs, leading to seizures. Long term psychiatric dysfunction also occurs.

Symptomatic and supportive management to avoid the lethal sequelae of toxicity. Thiamine and glucose are given as necessary. Benzodiazepines are used for seizure control and fluids given to minimize the effects of rhabdomyolysis. Icing the patient helps reduce the very high temperatures that can develop.

Â


210

USMLE STEP 2

Narcotics

Abuse presents as depression of the CNS and respiratory centers, miosis, euphoria, and seizures. Cardiac dysfunction may occur along with orthostatic hypotension.

Naloxone and restraints. Methadone, atropine, and various fentanyl derivatives may be necessary for refractory intoxication. Activated charcoal is also useful.

NSAID

Changes in respiration, S3 gallop, water retention with HTN, mental status changes, tremors, seizures, ulcers, hepatorenal failure, and general malaise.

ABCs, activated charcoal, hemodialysis, and supportive therapy.

Salicylate

Significant catabolism leading to metabolic acidosis, respiratory alkalosis, aciduria, delirium, respiratory arrest, aspiration pneumonitis, ototoxicity with tinnitus or deafness, hypotension, U waves and flat T waves with a QT prolongation, CNS depression, GI distress, renal failure, diaphoresis, and dehydration.

Treated in a manner similar to other types of poisoning. Urine alkalization may be done with sodium bicarbonate. Hemodialysis is the best method of reducing toxic levels of salicylates.

OTHER INJURIES BURNS ASSESSMENT First-degree burns present with reddening of the skin and local pain, such as with sunburn. Second-degree burns lead to blistering and swelling. Peeling of the skin may also occur. Third-degree burns lead to black charred skin or white skin with little pain due to destruction of nerve endings. Severe third-degree burns may lead to avascular necrosis of an entire extremity.

MANAGEMENT Treatment of burns is to cool the affected area and rinse with soap and water. Applying silver sulfadiazine to the affected area and covering with gauze assists in avoiding infection. Debridement of the affected area should be undertaken with sterile instruments. Chemical injury should immediately elicit rinsing the region until the chemical is sufficiently diluted. Oral antibiotics are often used. Pain control with NSAIDs is important.

HEATSTROKE ASSESSMENT Heatstroke is the result of uncontrolled heat buildup in the body leading to end-organ damage. Several thousand people die every year from heatstroke, and mortality is high. Heatstroke evolves after symptoms of heat exhaustion occur, which include constitutional symptoms, headache, dizziness, irritability, myalgia, and muscle cramps. Sudden onset of CNS symptoms and psychosis herald heatstroke; diaphoresis is as common as anhidrosis. Orthostatic hypotension is present along with piloerection and tachycardia. With heat exhaustion, the body temperature is less than 41oC. With heatstroke, the temperature is typically more than 41oC (106oF). DIC may be present.

MANAGEMENT Treatment involves cooling the patient with tepid water and large fans after ABCs and IVF.

www.ClinicalReview.com


Emergency Medicine

211

HYPOTHERMIA ASSESSMENT Hypothermia is a drop in the core body temperature below 35oC leading to a destabilization in the ability of the body to produce and maintain a normal body temperature. Hypothermia presents with chills, dyspnea, nausea, dizziness, stiffness, weakness, shivering, and ataxia. More severe hypothermia has J waves on EKG, slow reflexes, cold diuresis, delirium, cessation of shivering, and eventually, ventricular fibrillation, hypotension, pulmonary edema, and coma.

MANAGEMENT Treatment of hypothermia is to start ACLS, O2, NG tube, warm hydration, external warming, and core rewarming.

RADIATION TOXICITY ASSESSMENT Ionizing radiation is made up of alpha and beta particles, and gamma rays. Radiation poisoning presents with a prodrome of general malaise, followed by organ failure. Erythema and desquamation of skin can occur. Burns may indicate severe radiation. CNS symptoms include confusion and seizures. Cardiovascular collapse may also occur.

MANAGEMENT Management includes decontamination, washing the skin with soap and water after removing clothing, and giving potassium iodide to minimize thyroid gland damage. EDTA and penicillamine are given to treat poisoning with radioactive lead. Plutonium poisoning and other causes can be chelated with pentetate calcium trisodium (CaDTPA) and pentetate zinc trisodium (ZnDTPA).

ELECTROCUTION ASSESSMENT Nearly 1000 deaths occur yearly from electrical shock. Electrocution can lead to direct tissue damage, coagulation necrosis through heat production, and mechanical injury that results following the burst of energy. AC is more dangerous than DC due to tetany, respiratory arrest, and ventricular fibrillation.

MANAGEMENT Fluid resuscitation, urinary catheterization, bicarbonate to forestall acidosis, mannitol to cause an osmotic diuresis, acetazolamide to alkalinize the urine, and fasciotomy to visualize the extent of injury are typically done.

Â


212

USMLE STEP 2

DROWNING ASSESSMENT Death from drowning occurs due to suffocation. Drowning from fresh water causes water to enter the bloodstream and lyse RBCs. Salt water leads to a hypertonic solute in the lungs and passage of intravascular fluids into the lung. Rapid submerging into very cold water can lead to survival for over an hour underwater with no damage, a phenomenon referred to as the mammalian dive reflex. Consciousness is lost within 3 minutes, and brain damage occurs shortly thereafter.

MANAGEMENT First aid involves the Heimlich maneuver, followed by resuscitation and warming the person. Warm IVF, NG tube, bronchodilators to avoid bronchospasm, antibiotics to avoid respiratory ailments, and steroids are used. Shock management is essential.

ANAPHYLAXIS ETIOLOGY AND PATHOPHYSIOLOGY Anaphylaxis is an allergic reaction with numerous systemic and dermatologic signs. Increased mucous membrane permeability, bronchial smooth muscle tone, and capillary permeability occur as a result of histamine, leukotriene C4, prostaglandin D2, and tryptase. Anaphylaxis is mediated by IgE that leads to a rapid series of effects involving other immune cell mediators. The most commonly implicated agents include antibiotics, IV contrast, bee stings, and various foods. Several hundred deaths occur annually.

PRESENTATION AND DIAGNOSIS Anaphylaxis presents with angioedema, urticaria, erythema, and pruritus as skin manifestations. Cough and nasal congestion with rhinorrhea is present, along with dyspnea, chest pain, and hypotension. The eyes may becoming itchy and start tearing. GI distress can occur. Severe cases may lead to circulatory collapse or respiratory arrest.

TREATMENT Treatment for anaphylaxis involves maintaining the ABCs, providing O2, IVF, and giving epinephrine for systemic manifestations. Corticosteroids are also used. Beta-agonists are helpful with respiratory symptoms. Antihistamines may also be used to aid with maintaining BP.

www.ClinicalReview.com


Emergency Medicine

213

TABLE 293 OTHER INJURIES Other Injuries Other Injury

Etiology

Anaphylaxis

An allergic reaction with numerous systemic and dermatologic signs. Increased mucous membrane permeability, bronchial smooth muscle tone, and capillary permeability occur.

Burns

First-degree burns present with reddening of the skin and local pain, such as with sunburn. Second-degree burns lead to blistering and swelling. Peeling of the skin may also occur. Third-degree burns lead to black charred skin or white skin with little pain due to destruction of nerve endings. Severe third-degree burns may lead to avascular necrosis of an entire extremity.

Assessment

Management

Angioedema, urticaria, erythema, and pruritus as skin manifestations. Cough and nasal congestion with rhinorrhea is present, along with dyspnea, chest pain, and hypotension. The eyes may becoming itchy and start tearing. GI distress can occur. Severe cases may lead to circulatory collapse or respiratory arrest.

Maintaining the ABCs, providing O2, IVF, and giving epinephrine for systemic manifestations. Corticosteroids are also used. Beta-agonists are helpful with respiratory symptoms. Antihistamines may also be used to aid with maintaining BP.

Cool the affected area and rinse with soap and water. Apply silver sulfadiazine to the affected area and cover with gauze assists in avoiding infection. Debridement of the affected area should be undertaken with sterile instruments. Chemical injury should immediately elicit rinsing the region until the chemical is sufficiently diluted. Oral antibiotics are often used. Pain control with NSAIDs is important.

Death from drowning occurs due to suffocation.

Drowning from fresh water causes water to enter the bloodstream and lyse RBCs. Salt water leads to a hypertonic solute in the lungs and passage of intravascular fluids into the lung. Rapid submerging into very cold water can lead to survival for over an hour underwater with no damage. Consciousness is lost within 3 minutes, and brain damage occurs shortly thereafter.

Heimlich maneuver, followed by resuscitation and warming the person. Warm IVF, NG tube, bronchodilators to avoid bronchospasm, antibiotics to avoid respiratory ailments, and steroids are used. Shock management is essential.

Electrical shock.

Direct tissue damage, coagulation necrosis through heat production, and mechanical injury that results following the burst of energy. AC is more dangerous than DC due to tetany, respiratory arrest, and ventricular fibrillation.

Fluid resuscitation, urinary catheterization, bicarbonate to forestall acidosis, mannitol to cause an osmotic diuresis, acetazolamide to alkalinize the urine, and fasciotomy to visualize the extent of injury are typically done.

Heat Stroke

The result of uncontrolled heat buildup in the body leading to endorgan damage.

After symptoms of heat exhaustion occur, which include constitutional symptoms, headache, dizziness, irritability, myalgia, and muscle cramps. Sudden onset of CNS symptoms and psychosis herald heatstroke; diaphoresis is as common as anhidrosis. Orthostatic hypotension is present along with piloerection and tachycardia. The body temperature is typically more than 41oC (106oF). DIC may be present.

Cooling the patient with tepid water and large fans after ABCs and IVF.

Hypothermia

A drop in the core body temperature below 35oC leading to a destabilization in the ability of the body to produce and maintain a normal body temperature.

Chills, dyspnea, nausea, dizziness, stiffness, weakness, shivering, and ataxia. More severe hypothermia has J waves on EKG, slow reflexes, cold diuresis, delirium, cessation of shivering, and eventually, ventricular fibrillation, hypotension, pulmonary edema, and coma.

Start ACLS, O2, NG tube, warm hydration, external warming, and core rewarming.

Drowning

Electrocution

Â


USMLE STEP 2

214

Radiation Toxicity

Ionizing radiation (alpha and beta particles, and gamma rays).

A prodrome of general malaise, followed by organ failure. Erythema and desquamation of skin can occur. Burns may indicate severe radiation. CNS symptoms include confusion and seizures. Cardiovascular collapse may also occur.

Decontamination, washing the skin with soap and water after removing clothing, and giving potassium iodide to minimize thyroid gland damage. EDTA and penicillamine are given to treat poisoning with radioactive lead. Plutonium poisoning and other causes can be chelated with CaDTPA and ZnDTPA.

PRACTICE QUESTIONS Due to a defect in the sarcoplasmic reticulum, a patient develops malignant hyperthermia following administration of succinylcholine during induction of anesthesia. Which of the following is the best therapy? A. B. C. D. E.

Dantrolene, insulin, and bicarbonate Nipride drip, bicarbonate Nitroglycerin, diazepam Dantrolene Hydralazine, plasmapheresis

The best answer is Dantrolene, insulin, and bicarbonate. The best treatment for malignant hyperthermia caused by succinylcholine or lidocaine is to stop the anesthesia, give dantrolene, followed by insulin and bicarbonate to mitigate the hyperkalemia and acidosis that will follow. This will forestall any further damage and also avoid arrhythmias from the hyperkalemia. Succinylcholine is a competitive agonist that competes with acetylcholine.

A patient who is undergoing a bedside wound debridement suddenly experiences perioral numbness and tinnitus in his ear, followed by seizures and cardiac arrest. What is the most likely etiology? A. B. C. D. E.

Lidocaine toxicity Pulmonary embolism Massive cardiovascular accident Toxic hypokalemia Toxic hypercalcemia

The best answer is Lidocaine toxicity. This patient likely received an overdose of lidocaine leading to perioral paresthesias and tinnitus in his ear. Seizures may follow with sufficient medication, followed by ventricular fibrillation and cardiac arrest. No more than 4-5 mg per kg should be used if lidocaine is given without epinephrine. Up to 7 mg per kg can be used with epinephrine.

www.ClinicalReview.com


Emergency Medicine

215 An ICU patient who has significant respiratory compromise is intubated using succinylcholine. During intubation, the patient becomes asystolic. What is the immediate treatment required? A. B. C. D. E.

Calcium gluconate, insulin, and epinephrine Vasopressin IV KCl with epinephrine Cardioversion Etomidate

The best answer is Calcium gluconate, insulin, and epinephrine. This patient likely developed hyperkalemia due to succinylcholine administration, leading to arrhythmia that degenerated into asystole. Treatment includes bringing the potassium levels down with calcium gluconate, insulin, bicarbonate, and glucose, and treatment with epinephrine. As a result of this complication, succinylcholine is contraindicated in any high potassium state such as burn injury, renal disease, crush injuries to the muscles, and skeletal muscle disorders such as myasthenia gravis.

Which of the following paralytic agents is best used in a patient with liver and kidney failure? A. B. C. D. E.

Atracurium Propofol Etomidate Succinylcholine Ketamine

The best answer is Atracurium. The drug of choice for muscle paralysis in patients with combined liver and kidney failure is atracurium as it undergoes breakdown by Hoffman elimination. Propofol is an anesthetic agent that leads to amnesia and sedation, with a side effect of hypotension. Etomidate may lead to adrenal insufficiency. Succinylcholine can lead to arrhythmias and is contraindicated in kidney failure. Ketamine is a smooth muscle relaxant and dissociative amnesic agent used to induce anesthesia. Halothane can cause arrhythmias.

Which of the following is a contraindication to spinal anesthesia? A. B. C. D. E.

Liver surgery Kidney surgery Vasculopathy INR of 1.5 Hypertension

The best answer is Liver surgery. Contraindications to spinal anesthesia include coagulopathy with an INR greater than 1.5, sepsis, increased intracranial pressure, and hepatic surgery.

Â


USMLE STEP 2

216

An 84 year old male who has a myocardial infarction is transferred to the ICU. A Swan-Ganz catheter is floated and a significant elevation in PCWP is noted. What is the underlying cause? A. B. C. D. E.

Increased EDV, increased SVR, and decreased CO Increased EDV Increased SVR Decreased CO Increased EDV and increased SVR

The best answer is Increased EDV, increased SVR, and decreased CO. This patient has an increase in end diastolic volume due to cardiac hypocontractility, leading to an increase in systemic vascular resistance. The hypocontractility directly leads to a drop in cardiac output.

A 76 year old male who has a myocardial infarction is brought to the ICU in cardiogenic shock. Which of the following is the best initial course of therapy? A. B. C. D. E.

Dobutamine Norepinephrine Milrinone Vasopressin Epinephrine

The best answer is Dobutamine. The best initial vasopressor is dobutamine due to its improvement in cardiac contractility without significantly increasing myocardial oxygen consumption. The result is a mitigation in ischemic damage due to failure to increase oxygenation. Hypertrophic subaortic stenosis should be treated with ACE inhibitors, beta blockers, and a possible balloon pump placement to decrease myocardial oxygen demand. Pressors that increase oxygen demand should be avoided in hypertrophic subaortic stenosis.

An ICU patient who has been improving over a period of days cannot be weaned from his ventilator. His tube feeds were recently started and is thought to be the cause of his difficulty. Which of the following is the most likely reason for failure to extubate? A. B. C. D. E.

Overfeeding Hypoglycemia Starvation Refeeding syndrome TPN toxicity

The best answer is Overfeeding. Overfeeding leads to increased carbon dioxide production, leading to greater demands on the lungs to remove this from the body. The increased carbon dioxide production is the result of increased nutrient metabolism. Overworking the lungs leads to continued dependence on the ventilator. The treatment is to reduce carbon dioxide production by changing the composition of the tube feeds and re-evaluating intake.

www.ClinicalReview.com


Emergency Medicine

217

Which of the following is the fastest anesthetic agent that also leads to the most rapid clearing? A. B. C. D. E.

Propofol Ketamine Midazolam Diazepam Lorazepam

The best answer is Propofol. Propofol is a rapid anesthetic agent with rapid onset and rapid clearing. It is cleared quickly through hepatic metabolism. It is therefore contraindicated in patients with significant hepatic disease.

Â


218

USMLE STEP 2

www.ClinicalReview.com


Emergency Medicine

219

CHAPTER CONTENTS General Concepts..............................................................................................220 Central Disturbances.........................................................................................227 Cerebrovascular Disease ...................................................................................229 Headache ..........................................................................................................235 Sensory Disturbances........................................................................................239 Infectious Diseases ...........................................................................................241 Neurodegenerative Disorders ..........................................................................246 Epilepsy .............................................................................................................251 Cancer ...............................................................................................................253 Practice Questions ............................................................................................256

Â


USMLE STEP 2

220

NERVOUS SYSTEM NERVOUS SYSTEM GENERAL CONCEPTS BRACHIAL PLEXUS

C5 – Shoulder abduction, extension, and external rotation; some elbow flexion C6 – Elbow flexion, forearm pronation and supination, some wrist extension C7 – Diffuse loss of function in the extremity without complete paralysis of a specific muscle group, consistently supplies the latissimus dorsi C8 – Finger extensors, finger flexors, wrist flexors, hand intrinsics o

The ulnar nerve is responsible for innervations to the intrinsic muscles of the hand. Transection of this nerve is not repaired primarily.

T1 – Hand intrinsics

REFLEXES Biceps – C5,C6 (MC nerve) Triceps – C6, C7 (radial nerve) Knee Jerk – L3, L4 (femoral and common peroneal nerves) Ankle Jerk – L5, S1 (tibial nerve) www.ClinicalReview.com


NERVOUS SYSTEM

221

NERVE REGENERATION The rate of peripheral nerve regeneration following injury is approximately 1cm per month.

CEREBRAL PERFUSION PRESSURE Cerebral perfusion pressure is the difference between the mean arterial pressure and intracranial pressure. Normal perfusion pressure is between 80 and 100 mmHg. A decrease in cerebral perfusion pressure occurs with an increase in intracerebral pressure or a drop in systemic pressure. A drop below 60mmHg can lead to significant ischemia.

PHARMACOLOGY CHOLINERGICS – DIRECT AGONISTS DRUG Bethanechol

Pilocarpine

INDICATIONS

MECHANISM OF ACTION

COMPLICATIONS

Urinary retention Ileus

Stimulates bladder and intestinal smooth muscle

Intestinal hypermotility

Closed angle glaucoma

Ciliary muscle contraction, opening of meshwork, increased aqueous humor outflow

Miosis

CHOLINERGICS – INDIRECT AGONISTS DRUG

INDICATIONS

MECHANISM OF ACTION

Edrophonium

Diagnosis of myasthenia gravis

Increase acetylcholine

Neostigmine

Ileus, urinary retention, myasthenia gravis, reverses NMJ blockade

Increase acetylcholine

ANTICHOLINERGICS – MUSCARINIC ANTAGONISTS DRUG

INDICATIONS

MECHANISM OF ACTION

COMPLICATIONS

Atropine

Pupillary dilation Decrease acid secretion in PUD Decrease urinary urgency Decrease GI motility Reduce airway secretions

Inhibits parasympathetic muscarinic receptor

Tachycardia, T, xerostomia, dry skin, mydriasis with cycloplegia, constipation

Ipratropium

Bronchodilator

Inhibits vagally-mediated reflexes

Epistaxis, nasal irritation

EPS: often due to neuroleptic induced iatrogenic Parkinsonism with classic features. Acute dystonia: Within 4 hours. Rx = benztropine, diphenhydramine. Akinesia: Within 4 days. Rx = neuroleptic reduction. Akathisia: Within 4 weeks. Rx = propranolol and neuroleptic reduction. Tardive dyskinesia: Within 4 months. Rx = stop neuroleptics.


USMLE STEP 2

222

ANTICHOLINERGICS – NICOTINIC ANTAGONISTS DRUG

INDICATIONS

MECHANISM OF ACTION

Succinylcholine

Muscle paralysis Mechanical ventilation

Rapid onset and short duration with decrease in excitatory potential below threshold. Initial stage with prolonged depolarization leading to fasciculations and muscle pain. Second stage with repolarization but blockade of receptors.

Atracurium

Muscle paralysis Mechanical ventilation

Nicotinic receptor blockade

Nondepolarizing blockade can be reversed with neostigmine, edrophonium, and cholinesterase inhibitors.

ADRENERGIC AGONISTS – CATECHOLAMINES DRUG

INDICATIONS

MECHANISM OF ACTION

Shock with renal protection CHF

D1, D2 β1

Shock

α1, α2, β1, β2

Epinephrine

Open angle glaucoma Acute asthma Anaphylactic shock Increase local anesthetic duration

α1, α2, β1, β2 Increases aqueous humor outflow

Isoproterenol

Bronchodilator Cardiac stimulant

β1, β2, α1

Dopamine Norepinephrine

Dobutamine Milrinone

CHF Increase cardiac contractility

COMPLICATIONS Nausea, HTN, arrhythmia.

Mydriasis

Flushing, angina, arrhythmia

β1 Phosphodieserase inhibitor

Norepinephrine

www.ClinicalReview.com


NERVOUS SYSTEM

223

ADRENERGIC AGONISTS – NON-CATECHOLAMINES DRUG

INDICATIONS

MECHANISM OF ACTION

Alpha-methyldopa

HTN

CNS

Clonidine

PTSD Opioid withdrawal ADHD Tourette syndrome HTN

α2, CNS

Ephedrine

Asthma Nasal decongestant Urinary incontinence

α, β

Albuterol

Bronchospasm Premature labor

β2

COMPLICATIONS Sedation, hemolytic anemia, liver disorders

ALPHA ADRENERGIC ANTAGONISTS DRUG

INDICATIONS

MECHANISM OF ACTION

Pheochromocytoma Peripheral vascular disease

α1, α2

Terazosin

HTN, BPH

α1

Reserpine

Huntington’s disease

Blocks dopamine carrier

Phenoxybenzamine

COMPLICATIONS Orthostatic hypotension Reflex tachycardia Orthostatic hypotension on first dose, dizziness, syncope, and HA

BETA ADRENERGIC ANTAGONISTS DRUG Propranolol Metoprolol

Labetalol

MECHANISM OF ACTION

INDICATIONS HTN Angina pectoris MI SVT CHF Glaucoma

Akathisia, HTN, angina, migraine, IHSS

β1

HTN Pheochromocytoma and HTN

β1, β2

malignant

α1, β1, β2

COMPLICATIONS Sedation, hyperlipidemia Sedation, hyperlipidemia Sedation

Also used for social phobia, akathisia, impulsivity, and performance anxiety through PNS inhibition. Latanoprost is also used for closed angle glaucoma through increased aqueous humor outflow, S/E is darkening of iris


USMLE STEP 2

224

SEROTONINERGIC AGENTS DRUG

INDICATIONS Migraines Cluster headaches

Sumatriptan

Ondansetron

Treatment of N/V in chemotherapy

MECHANISM OF ACTION

COMPLICATIONS

CONTRAINDICATIONS

5-HT1D agonist leading to acute vasoconstriction.

Distal paresthesia, ACS

CAD, variant (Prinzemetal) angina

5-HT3 receptor antagonist to reduce vagus nerve activity and reduce serotonin receptor activity in CTZ

Dizziness, generally rare

Hepatic disease (P-450)

Ergotamines and caffeine also lead to acute vasoconstriction and can be used acutely for migraines. Analgesics are prescribed for mild migraines. The symptomatic phase of migraines is the significant intracranial vasodilation that follows the initial vasoconstriction.

ANTICONVULSANTS DRUG Carbamazepine

INDICATIONS

MECHANISM OF ACTION

Partial seizures Tonic-clonic seizures

Blocks post tetanic potentiation

COMPLICATIONS

Diplopia, aplastic anemia, hepatotoxicity, P-450 induction

Ethosuximide

Absence seizures

GI Sx, urticaria, SJS

Gabapentin

Partial seizures Tonic-clonic seizures

Sedation,

Lamotrigine

Partial seizures Tonic-clonic seizures

Rash,

Phenobarbital

Partial seizures Tonic-clonic seizures Status seizures

ataxia

SJS

Sedation, P-450 induction

Phenytoin

Partial seizures Tonic-clonic seizures Status epilepticus

Topiramate

Partial seizures

Sedation, nephrolithiasis, weight loss

Valproate

Partial seizures Tonic-clonic Absence Myoclonic

GI Sx, hepatotoxicity, NTDs, alopecia, tremor, pancreatitis

Blocks sodium channels

Nystagmus, diplopia, ataxia, gingival hyperplasia, hirsutism, anemia, teratogenic, malignant hyperthermia

Partial seizures include both simple and complex partial seizures.

www.ClinicalReview.com


NERVOUS SYSTEM

225

ALZHEIMER DISEASE DRUG

INDICATIONS

MECHANISM OF ACTION

COMPLICATIONS

CONTRAINDICATIONS

Donepezil

Alzheimer disease

Inhibits AChe

Used to minimize memory loss

Hypersensitivity.

Tacrine

Alzheimer disease

Inhibits AChe

Used to minimize memory loss

Hypersensitivity, asthma, ↑LFTs, GI disease, CV disease.

PARKINSON DISEASE DRUG Amantidine

INDICATIONS PD

Levodopa Carbidopa Selegiline

Dyskinesia PD

PD

MECHANISM OF ACTION

COMPLICATIONS

Enhances dopamine release

Suicide

Replaces dopamine

Arrhythmia

Inhibits dopamine decarboxylase in periphery

Dyskinesia

Decreases conversion of dopamine to norepinephrine Enhances levodopa effects

PD

Nausea, dizziness

ANESTHETICS – INHALED DRUG

MECHANISM OF ACTION

INDICATIONS

Sevoflurane

Nitrous oxide Desflurane

COMPLICATIONS

NOTES

Anesthesia

Bradycardia, respiratory depression, increased ICP

Rapid onset

Anesthesia

Minimal

Lowest potency, combined with other agents

Anesthesia

Airway irritation, coughing, respiratory depression, increased ICP

Most rapid onset

ANESTHETICS – INTRAVENOUS DRUG

INDICATIONS

Midazolam

Endoscopy

Ketamine

Short procedures

Propofol

Etomidate

MECHANISM OF ACTION

Dissociative anesthetic

Short procedures Amnesia, sedation.

COMPLICATIONS

NOTES

Respiratory depression, amnesia

Benzodiazepine; reverse w/ flumazenil

↑CV, hallucinations, ↑ICP

Arylcyclohexylamine

Hypotension

No cumulative effects, strict aseptic technique mustbe maintained.

Uncontrolled skeletal muscle activity


USMLE STEP 2

226

ANESTHETICS – LOCAL DRUG

INDICATIONS

Procaine

Local anesthesia

Lidocaine

Local anesthesia

Bupivacaine

Local anesthesia

COMPLICATIONS

NOTES

CNS stimulation, CV stimulation, HTN CNS stimulation, HTN CV toxicity

Greater amounts needed in infected tissue (acidic tissue) Smaller fibers affected first, so pain is lost first, then T, touch, and finally P Give with epinephrine to increase local effects No allergic cross reactivity between esters and amides. Long duration.

ANALGESICS – OPIOID DRUG

INDICATIONS

Morphine

Analgesia

Fentanyl

Analgesia

Dextromethorphan

Cough suppression

MECHANISM OF ACTION Mu opioid agonists Net effect is change in neural activity along various pathways, especially pain pathways

COMPLICATIONS Substance abuse and dependence, respiratory depression, miosis, CNS depression, constipation

ANALGESICS – NSAIDS DRUG

Naproxen

Indomethacin

Acetaminophen

INDICATIONS

MECHANISM OF ACTION

COMPLICATIONS

CONTRAINDICATIONS

Analgesia Antiinflammatory Antipyretic

Reversible inhibition of COX-1 and COX-2

Nephrotoxicity, aplastic anemia, PUD

PUD. Renal patients

Analgesia Antiinflammatory Antipyretic

Reversible inhibition of COX-1 and COX-2

Nephrotoxicity, aplastic anemia, PUD

PUD. Renal patients

Reversible inhibition of COX in CNS

Hepatic necrosis in OD, glutathione depletion

Hepatitis.

Avoids PUD. Nephrotoxic. Possible stroke risk.

PUD. Renal patients.

PUD, Reye syndrome, tinnitus, reflex acidosis from reflex hypoventilation due to initial hyperventilation (late stage poisoning)

ETOH, do NOT use in children especially with VZV or influenza infection

Analgesic Antipyretic

Rofecoxib

RA, OA

Aspirin

Analgesic Antipyretic Antiinflammatory Antiplatelet aggregation

COX-2 inhibition

Irreversible COX-1 and COx-2 inhibition inhibits prostaglandin formation

www.ClinicalReview.com


NERVOUS SYSTEM

227

CENTRAL DISTURBANCES SPINAL CORD COMPRESSION ETIOLOGY Compression of the spinal cord is a neurologic emergency that requires prompt medical attention to avoid lifelong sequelae. Common causes include tumor spread to the back, disk herniation, abscess, and hematoma formation. The most common site of compression is the thoracic spinal cord.

PRESENTATION AND DIAGNOSIS Cord compression presents as sudden onset of neurologic symptoms distal to the site of spinal cord compression, with symptoms including sudden weakness in lower extremities, incontinence, sexual dysfunction, and sensory changes. Pain is common in nearly all patients. Diagnosis is made by careful history and physical exam, including a full neurologic exam that identifies focal deficits. Plain films are typically diagnostic, but MRI and CT myelogram are all used for additional information.

TREATMENT Cord compression is treated with immediate steroid therapy, including high doses of dexamethasone. The key to this treatment is to reduce any inflammation and prevent further injury. Cancer-induced compression should be treated with chemotherapy or radiation therapy, while surgical options are available for disk herniation, hematoma, and abscesses. Outcome depends on the promptness of medical therapy, but the majority of patients are able to regain their baseline function, if they had good function previously. TABLE 294 SPINAL CORD COMPRESSION Spinal Cord Compression Etiology

Tumor spread to the back, disk herniation, abscess, and hematoma formation.

Presentation

Sudden onset of neurologic symptoms distal to the site of spinal cord compression, sudden weakness in lower extremities, incontinence, sexual dysfunction, and sensory changes. Pain.

Diagnosis

Full neurologic exam that identifies focal deficits. Plain films, MRI and CT.

Treatment

Steroid therapy. Reduce any inflammation and prevent further injury. Cancer-induced compression should be treated with chemotherapy or radiation therapy, while surgical options are available for disk herniation, hematoma, and abscesses.

SYRINGOMYELIA ASSESSMENT AND MANAGEMENT Syringomyelia is divided into a communicating cavitation or noncommunicating cavitation of the spinal cord. Communicating syringomyelia is associated with Arnold-Chiari malformation, and noncommunicating syringomyelia is typically secondary to spinal cord trauma. Neurologic symptoms include sensory deficits and some lower motor neuron defects. The cervical spinal cord is most commonly affected. Treatment involves surgery.

Â


228

USMLE STEP 2

TABLE 295 SYRINGOMYELIA Syringomyelia Communicating cavitation

Associated with Arnold-Chiari malformation.

Noncommunicating cavitation

Secondary to spinal trauma.

Diagnosis

Sensory deficits, lower motor neuron defects.

Treatment

Surgery

SUBACUTE COMBINED DEGENERATION (SACD) ASSESSMENT AND MANAGEMENT Subacute combined degeneration is due to vitamin B12 deficiency, itself commonly the result of intrinsic factor deficiency, decreased intake, terminal ileum disorders, Diphyllobothrium latum infection, or atrophic gastritis. In addition to the megaloblastic anemia that develops, vitamin B12 deficiency leads to weakness of the extremities with paresthesia. Ataxia is common. Plantar extension and hyperreflexia are found on physical exam, along with deficits in vibration sensation and proprioception. Treatment is to administer vitamin B12. TABLE 296 SUBACUTE COMBINED DEGENERATION (SACD) Subacute Combined Degeneration (SACD) Etiology

Vitamin B12 deficiency, intrinsic factor deficiency, decreased intake, terminal ileum disorder, Diphyllobothrium latum infection, atrophic gastritis.

Diagnosis

Megaloblastic anemia, weakness of extremities, paresthesia, ataxia, plantar extension & hyperreflexia.

Treatment

Administer vitamin B12.

ANTERIOR SPINAL ARTERY INFARCTION ASSESSMENT AND MANAGEMENT Anterior spinal artery infarction is rare and leads to flaccid paralysis followed by spastic paresis through ischemic damage to the spinal cord. Vibration and proprioception remain intact, but pain and temperature are lost. TABLE 297 ANTERIOR SPINAL ARTERY INFARCTION Anterior Spinal Artery Infarction Etiology

Trauma, dissecting aortic aneurysm, aortography, polyarteritis nodosa, hypertensive crisis

Presentation

Flaccid paralysis followed by spastic paresis, loops of pain and temperature.

Diagnosis

Imaging studies

Treatment

Symptomatic

www.ClinicalReview.com


NERVOUS SYSTEM

229

CEREBROVASCULAR DISEASE EPIDEMIOLOGY Cerebrovascular accidents (CVA) affect more than 400,000 patients a year with a rapid increase projected over the next 50 years. Stroke is the third leading cause of death; overall, it is the second leading cause of death worldwide. Men are at more risk than women, and up to Âź of all strokes affect individuals under the age of 65.

ETIOLOGY Cerebrovascular disease (CVD) and cerebrovascular accidents present with acute focal neurologic deficits commonly due to loss of circulation to a portion of the brain. Also known as a stroke, there are numerous types of CVAs. Broadly, CVA is categorized as either hemorrhagic or ischemic. Ischemic strokes are commonly secondary to embolism from elsewhere in the body or intracranial thrombosis. Disruption of the blood flow leads to neuronal death and infarction of the brain. Common sources of the embolism include valvular or mural thrombi, carotid circulation, and occasionally, the right heart in the presence of a right to left shunt. About 1/5 of all strokes lead to lacunar infarcts (which involve the subcortical cerebrum and brainstem). Lacunar infarcts are most common in patients with DM and HTN. Lacunar infarcts lead to either a pure sensory deficit, a pure motor deficit, or a hemiparetic stroke with ataxia. Sources of thrombus formation include the branch points within the circle of Willis and near the internal carotid artery (ICA). Stenosis, atherosclerosis, and platelet defects are common causes of arterial blockade; other causes are hypercoagulable states, polycythemia, and sickle cell anemia. Overall, other causes of stroke include vascular dissection, hypotension, and excessive hemorrhage. Risk factors that increase CVA include increasing age, HTN, smoking, CHD, LVH, atrial fibrillation, hypertriglyceridemia, oral contraceptive use, pregnancy, and hypercoagulable states. Thrombotic strokes are typically slower in onset, while embolic strokes are sudden in onset. TABLE 298 STROKE ETIOLOGY Stroke Etiology Ischemic

Commonly secondary to embolism from elsewhere in the body, thrombus, stenosis, platelet defect.

Lacunar infarct

DM, HTN.

Hemorrhagic

Excessive bleeding, vascular dissection.

Risk factors

HTN, smoking, CHD, LVH, atrial fibrillation, hypertriglyceridemia, oral contraceptive use, pregnancy, and hypercoagulable states.

PATHOPHYSIOLOGY OF ISCHEMIC STROKE The brain is highly sensitive to disruption in blood supply. An ischemic cascade begins almost immediately following loss of perfusion and eventually leads to irreversible infarction. The area of the brain that still receives some transient blood flow is a region of reversible ischemia; this region forms a sort of penumbra (think of the sun during a full eclipse) around the area of the stroke. The result of ischemia is a failure of membrane transport, large calcium influx, large quantities of neurotransmitter release with additional calcium influx, and local oxidative and ischemic injury. Large amounts of inflammatory mediators are created and free radical injury occurs. Degradation of the cell membrane occurs, and necrosis and apoptosis take place. The region that has infarcted has little chance of restoration. Current medical efforts attempt to save the region of the ischemic penumbra through limitation of toxic free radical formation, reducing the duration of ischemia, and protecting the neurons from additional insults. Reperfusion must take place within 3 hours to avoid permanent damage to the penumbra. Short of removing the arterial blockade, little can be done to reverse the changes in the region of apoptosis and necrosis.

Â


230

USMLE STEP 2

PATHOPHYSIOLOGY OF HEMORRHAGIC STROKE Hemorrhagic stroke is categorized as either subarachnoid bleeds or intracerebral bleeds. Subarachnoid bleeds most commonly occur as the result of head trauma, AV malformations, and aneurysms. Intracerebral bleeds may occur in HTN, bleeding diatheses, and amyloidosis. Hypoperfusion may also lead to stroke and affect especially the parasagittal strips of the cortex (a region known as the watershed area where the end points of the circulation to the brain are located). Hemorrhagic stroke leads to direct injury to neurons by the toxic effects of blood. Compressive injury and electrolyte imbalances worsen the injury. Hemorrhagic strokes worsen with rising in the morning and evolve over a period of minutes.

TYPES OF STROKE There are several types of CVA that may occur. The term “stroke” refers to the presence of infarcted tissue in the brain. A “stroke in evolution” is progressive, ongoing injury. A “completed stroke” has caused irreparable harm to a certain portion of the brain and has been stable in its course for a few days. There is a type of stroke that spontaneously resolves with no damage; this type is known as a “transient ischemic attack” (TIA) and typically resolves within 30 minutes to 24 hours. Repeated TIAs are referred to as “crescendo TIAs.”. A TIA that lasts more than 24 hours but less than 3 weeks and has progressive improvement is known as a reversible ischemic neurologic deficit (RIND). TIAs and RINDs are highly indicative of a future stroke potential. TABLE 299 TYPES OF STROKES Types of Strokes Stroke in evolution

Progressive, ongoing injury.

Completed stroke

Irreparable harm to particular region of brain & has been stable for a few days.

Transient ischemic attack

Stroke that resolves within 30 minutes to 24 hours.

Reversible ischemic neurologic deficit

TIA that lasts 24 hours but less than 3 weeks.

PRESENTATION CVAs present as an acute neurologic deficit or an altered state of consciousness. Numerous constitutional symptoms are typically present in addition to one or more of the following: abrupt onset of paresis, visual deficits, vestibular or hearing deficits, aphasia, dysarthria, and ataxia. Physical exam may uncover cardiac or vasculature abnormalities or evidence of trauma that can be used to pinpoint the cause of the stroke. A full neurologic exam is required and together with diagnostic testing, can be used to identify the precise site of neurologic injury.

www.ClinicalReview.com


NERVOUS SYSTEM

231

TABLE 300 PRESENTATION OF STROKES Presentation of Strokes Abrupt onset of paresis

Visual deficits

Vestibular or hearing deficits

Aphasia

Ataxia

Dysarthria

ANTERIOR CEREBRAL ARTERY There are four major stroke syndromes that can occur depending on the anatomic location of the ischemic injury and the particular artery that is affected. Strokes from occlusion of the anterior cerebral artery (ACA) present with changes in mental status, impaired judgment, apraxia, and weakness of the contralateral lower extremities. Other symptoms commonly include incontinence and personality or behavioral changes. The region that is affected is mostly the frontal lobe.

MIDDLE CEREBRAL ARTERY Middle cerebral artery (MCA) strokes lead to contralateral hemiparesis and hemiplegia with sensory loss. Ipsilateral hemianopsia presents with a gaze preference ipsilateral to the side of injury. Agnosia and aphasia may occur especially if the dominant hemisphere is affected. Upper extremity deficits are typically prominent.

POSTERIOR CEREBRAL ARTERY The posterior cerebral artery (PCA) may occlude and present as changes in vision via a homonymous hemianopsia or blindness, agnosia, defects in memory, and altered mental status. The PCA can also lead to Weber syndrome through cranial nerve (CN) III palsy (leading to contralateral hemiplegia, or contralateral ataxia [known as Benedikt syndrome]).

VERTEBROBASILAR ARTERY Strokes of the vertebrobasilar artery present with a number of diverse deficits and are difficult to diagnose. Symptoms include vestibular effects (such as vertigo or nystagmus), visual effects (including diplopia or field deficits), motor defects (such as dysarthria or dysphagia), ataxia, syncope, and loss of pain and temperature sensations on the ipsilateral face and contralateral body. Other stroke syndromes typically present with ipsilateral symptoms. Occlusion of the paramedian branches can lead to locked-in-syndrome with only intact eye movements but complete quadriparesis.

Â


232

USMLE STEP 2

POSTERIOR INFERIOR CEREBELLAR ARTERY Blockage of the posterior inferior cerebellar artery (PICA) can lead to Wallenberg syndrome, presenting as ipsilateral loss of sensation to the face and contralateral paresthesias of the body. Horner syndrome occasionally accompanies PCA and PICA stroke. Horner syndrome includes hemianhidrosis, unilateral effects, ptosis, miosis, and enophthalmos.

ANTERIOR INFERIOR CEREBELLAR ARTERY Occlusion of the cerebellar arteries presents with vertigo, nystagmus, nausea, and vomiting. Ataxia of an extremity is also common. Occlusion of the anterior inferior cerebellar artery (AICA) presents with gaze palsy, deafness, tinnitus, and weakness of the ipsilateral face.

LACUNAR INFARCTS Infarctions affecting the lenticulostriate arteries branching from the MCA are most commonly affected in patients with HTN. A pure sensory stroke may occur with loss of sensory information in one half of the body. A pure motor stroke may occur with loss of motor ability in the face or one of the extremities. Ataxic hemiparesis may also result as a combined motor and sensory loss leading to ataxia. Dysarthria with retardation of normal motor and sensory transmission may occur leading to weakness and impediments in normal motion of one of the extremities or face. TABLE 301 CLINICAL DIAGNOSIS OF STROKE Clinical Diagnosis of Stroke Anterior cerebral artery

Changes in mental status, impaired judgment, apraxia, and weakness of the contralateral lower extremities, incontinence & personality or behavioral changes.

Middle cerebral artery

Contralateral hemiparesis and hemiplegia with sensory loss. Ipsilateral hemianopsia. Agnosia and aphasia may occur, especially if the dominant hemisphere is affected. Upper extremity deficits are typically prominent.

Posterior cerebral artery

Changes in vision, agnosia, defects in memory, and altered mental status. Weber & Benedikt syndromes.

Vertebrobasilar artery

Vestibular effects, visual effects, motor defects, and loss of pain and temperature on the ipsilateral face and contralateral body.

PICA

Wallenberg syndrome, Horner syndrome.

AICA

Vertigo, nystagmus, nausea, and vomiting. Ataxia of an extremity, gaze palsy, deafness, tinnitus, and weakness of the ipsilateral face.

Lacunar infarcts

Sensory stroke may occur with loss of sensory information in half of the body. A pure motor stroke may occur with loss of motor ability in the face or one of the extremities. Ataxic hemiparesis may also result as a combined motor and sensory loss leading to ataxia. Dysarthria with retardation of normal motor and sensory transmission may occur leading to weakness and impediments in normal motion of one of the extremities or face.

www.ClinicalReview.com


NERVOUS SYSTEM

233

DIAGNOSIS THROUGH IMAGING Noncontrast CT is the initial preferred test and is mandatory for distinguishing the various types of stroke and identifying the particular location of injury. Patients with acute ischemic stroke may entirely bypass this diagnostic study and be taken for immediate therapy. CT has normal findings in the first 6 hours; however, edema over this time leads to changes in the form of a hypodense region. Lumbar puncture should be done in all patients suspected of having a subarachnoid hemorrhage, as CT changes are sometimes nonspecific in this particular etiology. Carotid duplex scanning is done in patients who may have stenosis of the carotid artery, leading to possible endarterectomy in some patients. Echocardiography and other diagnostic studies are also used if particular causes of stroke are suspected. MRI is useful in patients that have a cerebellar or lacunar defect. Angiography is the definitive study that precisely identifies even subtle occlusion. TABLE 302 IMAGING TESTS Imaging Tests Noncontrast CT

Distinguishes various types of strokes.

Lumbar puncture

Patients suspected of having subarachnoid hemorrhage.

Carotid duplex scanning

Patients who may have stenosis of the carotid artery.

Echocardiography

If particular causes of stroke are suspected.

Angiography

Precisely identifies occlusions.

GLASGOW COMA SCALE (GCS) The Glasgow coma scale is used for rapid neurologic assessment following acute head injury or stroke. It is closely tied to outcome and is often used to dictate therapy in certain instances. The GCS ranges between 3 and 15, with 3 being the worst. Three responses are gauged, including eye response, verbal response, and motor response. The eye responses range from 1-4, and include no eye opening (1 point), eye response to pain (2), eye response to verbal command (3), and spontaneous eye response (4). No verbal response gets 1 point, incomprehensive sounds (2), inappropriate words (3), confused (4), and oriented gets 5 points. Motor responses range from no response (1), extension to pain (2), flexion to pain (3), withdrawal from pain (4), localizing pain (5), and obeying commands (6). A range of 13 or more correlates with mild or nonexistent brain injury. Between 9 and 12 is considered a moderate injury, and less than 9 is a severe injury. TABLE 303 GLASGOW COMA SCALE (GCS) Glasgow Coma Scale (GCS) Eye response

No eye opening (1 point), eye response to pain (2), eye response to verbal command (3), and spontaneous eye response (4).

Verbal response

No verbal response gets 1 point, incomprehensive sounds (2), inappropriate words (3), confused (4), and oriented gets 5 points.

Motor response

No response (1), extension to pain (2), flexion to pain (3), withdrawal from pain (4), localizing pain (5), and obeying commands (6).

Interpretation

>13 mild or nonexistent brain injury; 12-9 moderate injury; <9 is severe.

Â


234

USMLE STEP 2

GENERAL TREATMENT Treatment of stroke depends on the particular type and severity of stroke. Basic emergency management includes establishing airway, breathing, and circulation (ABCs), especially with a GCS of less than 9 or dropping GCS scores. Lidocaine, pancuronium, succinylcholine, and oral endotracheal intubation may be necessary with increased intracranial pressure (ICP). Hyperventilation is the key to decreasing ICP and cerebral blood flow. Hydration status should be assessed and overhydration prevented. Lowering BP is necessary with HTN, and commonly used agents include nitroprusside and labetalol. Antipyretics should be used with fever, and cerebral edema prevented.

TREATMENT OF STROKE Use of calcium-channel blockers such as lubeluzole may be beneficial very early in the evolution of stroke to avoid calcium influx. Free-radical scavengers such as tirilazad and citicoline and stabilizers of neuronal membranes such as citicoline are useful later in the ischemic cascade. Antibodies against leukocyte adhesion molecules, such as enlimomab, may serve a neuroprotective role. Anticoagulation with heparin may have some protection in progressive stroke and especially with occlusion affecting the vertebrobasilar artery. However, anticoagulation for stroke has up to a 4% risk of hemorrhage. Contraindications for anticoagulation include concomitant HTN, bleeding diatheses, and intracranial hemorrhage. Tissue-plasminogen activators (t-PA) can be used to restore cerebral blood flow and help resolve an evolving neurologic defect. However, the use of tPAs such as streptokinase, urokinase, or alteplase can increase mortality in some groups through increased intracranial bleed (which can lead to death in up to half of all patients). Overall, 1 in 8 patients had full recovery with t-PA treatment, 1 in 17 had intracranial bleeds, and 1 in 40 died from complications of therapy. Of all the medications available for t-PA, only alteplase is recommended and approved for therapy; streptokinase is not recommended. t-PA therapy should be given within 3 hours in order to be effective.

TREATMENT OF TIAS TIAs should be treated with antiplatelet agents including aspirin and clopidogrel. Anticoagulation may be necessary with heparin and warfarin. Carotid endarterectomy should be considered if the carotid artery is implicated as a causative agent. TABLE 304 TREATMENT OF STROKE Treatment of Stroke

Emergency

ABCs. Lidocaine, pancuronium, succinylcholine, and oral endotracheal intubation may be necessary with increased ICP. Hyperventilation is the key to decreasing ICP and cerebral blood flow. Overhydration should be prevented. Lowering BP with HTN, antipyretics should be used with fever, and cerebral edema prevented.

Early in evolution

Calcium-channel blockers.

Later in evolution

Free-radical scavengers, antibodies against leukocyte adhesion molecules, anticoagulation.

Treatment of TIA

Antiplatelet agents, possibly carotid endarterectomy.

COMPLICATIONS There are several complications to stroke. Several syndromes may develop. Anosognosia may occur with the inability to identify parts of the body as belonging to the individual. Aphasia may occur with defects in spoken or written language comprehension or expression. Broca aphasia may occur with inability to express speech or write. Wernicke aphasia may occur through impaired comprehension but copious amounts of nonsensical speech. Apraxia can occur with the inability to repeat learned motor

www.ClinicalReview.com


NERVOUS SYSTEM

235 motions. Dysarthria is the inability to speak, and dysphagia is difficulty with swallowing. As discussed above, numerous stroke syndromes can occur including Horner syndrome, Wallenberg syndrome, Weber syndrome, and Benedikt syndrome. TABLE 305 COMPLICATIONS OF STROKE Complications of Stroke Anosognosia

Aphasias—Broca & Wernicke

Apraxia

Dysarthria

Horner syndrome

Wallenberg syndrome

Weber Syndrome

Benedikt syndrome

EPIDURAL HEMORRHAGE Etiology: Temporal bone fracture Pathophysiology: Middle meningeal artery rupture (branch of maxillary artery) Presentation: Lucid interval followed by rapid deterioration Imaging tests: Mass does not cross suture lines Management: Treated surgically with immediate evacuation

Epidural hemorrhage: Lucid interval, needs surgical evacuation Subdural hemorrhage: Progressive symptoms. Do not evacuate surgically

SUBDURAL HEMORRHAGE Etiology: Elderly, alcoholics, trauma Pathophysiology: Bridging vein rupture with venous bleeding Presentation: Delayed symptoms followed by progressive deterioration Clinical imaging: Crosses suture lines Management: Typically not treated surgically due to continuing bleeding A subdural hematoma is associated with a high mortality rate due to the underlying brain contusion that likely accompanies this presentation.

SUBARACHNOID HEMORRHAGE Etiology: APKD, Ehlers-Danlos, Marfan Pathophysiology: Berry aneurysm rupture, which may occur at the Circle of Willis bifurcation point especially at the anterior communicating artery. Presentation: Worst HA of the patient’s life

HEADACHE INTRODUCTION Headaches are a common complaint among many patients, but the causes of headache are vast and of varying importance. Primary causes of headache include migraines, tension headaches, and cluster headaches. Secondary causes include intracranial hemorrhage, tumor, meningitis, temporal arteritis, and glaucoma. The top ten causes of headache include various headache syndromes such as migraine, cluster headaches, and tension headaches, subarachnoid hemorrhage, meningitis, HTN, masses, temporal arteritis, trigeminal neuralgia, brain abscess, pseudotumor cerebri, and subdural hematoma. Worrisome headaches occur when they are of new onset in a patient who has no prior history of headaches. Extremely severe headache may herald an intracranial hemorrhage and requires rapid diagnosis and treatment. Many of the secondary causes of headache are discussed elsewhere in this section. Primary headaches and giant cell arteritis are discussed below.


236

USMLE STEP 2

TABLE 306 CAUSES OF HEADACHES Causes of Headaches Primary

Migraines, tension and cluster headaches.

Secondary

Intracranial hemorrhage, tumor, meningitis, temporal arteritis, glaucoma.

MIGRAINE HEADACHE ETIOLOGY AND PATHOPHYSIOLOGY Migraine headaches are thought to occur as a result of vasodilation following cerebral vasoconstriction. More than 1 in 10 persons are affected, women more than men. The majority of patients have a positive family history.

PRESENTATION AND DIAGNOSIS Migraines may present with or without an aura that precedes the actual onset of the headache. Auras are focal neurologic findings that include photophobia, sonophobia, tinnitus, bright lights, ataxia, weakness, or any other focal, stereotyped, repeated occurrence that precedes the migraine. This type of migraine is known as a classic migraine. Migraines without auras, known as common migraines, present with photophobia and sonophobia along with symptoms of the headache. The headache itself leads to nausea and vomiting, and general constitutional symptoms. A prodrome that occurs before the onset of the migraine may occur several days prior to migraine and may include lethargy, depression, edema, or food cravings. Migraines are ongoing for several hours, are unilateral, and present as a severe, throbbing pain.

TREATMENT Migraine headaches are treated by avoiding triggers, reducing stress, and resting in a dark, quiet environment. NSAIDs are commonly used, along with ergotamines. Sumatriptan, a 5-HT receptor agonist, is popular among patients with migraines; however, in sufficient doses, this class of drugs may cause vasospasm. Metoclopramide may be given as an antiemetic. Migraine prophylaxis includes calcium channel blockers, beta blockers, and SSRIs. TABLE 307 MIGRAINE HEADACHES Migraine Headaches Etiology

Result of vasodilation following cerebral vasoconstriction.

Presentation

Nausea and vomiting, general constitutional symptoms and severe throbbing pain.

Treatment

Avoiding triggers, reducing stress, and resting in a dark, quiet environment. NSAIDs, ergotamines, sumatriptan, antiemetic. Migraine prophylaxis includes calcium channel blockers, beta blockers, and SSRIs.

TENSION HEADACHE ASSESSMENT Tension headaches occur following significant stress, and typically present as head and neck stiffness. Tension headaches have no prodrome, no aura, and are bilateral. They worsen over the course of the day. Tension headaches are described as a band-like or vise-like pain that surrounds the head or neck region. www.ClinicalReview.com


NERVOUS SYSTEM

237

MANAGEMENT Tension headaches are alleviated by NSAIDs. If severe, narcotics can be used. Amitriptyline, a tricyclic antidepressant, has been used as tension headache prophylaxis. TABLE 308 TENSION HEADACHES Tension Headaches Etiology

Occur following significant stress.

Presentation

Head and neck stiffness, band-like pain that surrounds head.

Treatment

NSAIDs, narcotics, amitriptyline as prophylaxis.

CLUSTER HEADACHE ASSESSMENT Cluster headaches are severe, periorbital pains that significantly worsen immediately after onset. They tend to occur in clusters, and the unrelenting, severe facial pain that occurs is highly debilitating. Cluster headaches may last up to an hour and a half, occur several times a day, and last for up to a month or two. Associated symptoms include rhinorrhea, injected conjunctiva, lacrimation, ptosis, miosis, facial sweating, and nausea. There are no obvious triggers. The pain is so severe that patients may even commit suicide.

MANAGEMENT Treatment for cluster headaches is to use high-flow oxygen, lidocaine, ergotamine, sumatriptan, and antiemetics for symptomatic control. Verapamil, methysergide, prednisone, and indomethacin are all effective in some patients for prophylaxis. TABLE 309 CLUSTER HEADACHES Cluster Headaches Presentation

Severe, periorbital pains that significantly worsen immediately after onset. They tend to occur in clusters, and the unrelenting, severe facial pain that occurs is highly debilitating. They may last up to an hour and a half, occur several times a day, and last for up to two months.

Etiology

None known.

Treatment

Oxygen, ergotamine, lidocaine, sumatriptan, and antiemetics for symptomatic control. Verapamil, methysergide, prednisone, or indomethacin for prophylaxis.

GIANT CELL ARTERITIS (GCA) ETIOLOGY AND PATHOPHYSIOLOGY Giant cell arteritis, also known as temporal arteritis, is an immune-mediated, idiopathic inflammation of the temporal artery. Giant multinucleated cells are found in the region, and the majority of patients suffer from polymyalgia rheumatica. GCA affects mostly women and patients over 60 years of age.

Â


238

USMLE STEP 2

PRESENTATION AND DIAGNOSIS GCA presents with a unilateral headache pain along the course of the temporal artery. Palpation of the arteries yields thickened, tender arteries. Claudication can occur leading to ischemic changes in the masseter, temporalis, and tongue. Ipsilateral loss of vision may also occur. A pulsatile flow may be apparent in the temporal artery. Diagnosis is made by an elevated ESR and CRP, and confirmed by biopsy of the temporal artery.

TREATMENT Corticosteroids are used to control the inflammation and reduce symptoms of temporal arteritis. Corticosteroids also protect the vision. NSAIDs are used for pain control. TABLE 310 GIANT CELL ARTERITIS (GCA) Giant Cell Arteritis (GCA) Etiology

Immune-mediated, idiopathic inflammation of the temporal artery.

Presentation

Unilateral headache pain along the course of the temporal artery, claudication, ipsilateral vision loss.

Diagnosis

Palpation of the arteries yields thickened, tender arteries, elevated ESR and CRP, biopsy of temporal artery.

Treatment

Corticosteroids, NSAIDs.

NORMAL PRESSURE HYDROCEPHALUS (NPH) ASSESSMENT Normal pressure hydrocephalus (NPH) is increased CSF but no increase in ICP. NPH is commonly the result of subarachnoid hemorrhage, meningitis, tumor, or trauma. NPH presents with a triad of gait disturbance, incontinence, and dementia. Treating the gait disturbance is often successful, but the dementia usually is refractory to medical management. Diagnosis of NPH is made by ruling out other etiologies by a normal LP, detecting ventricular hypertrophy on CT or MRI, and finding improvement in symptoms after a therapeutic LP.

MANAGEMENT Treatment for NPH involves therapeutic CSF removal, placing a ventriculoperitoneal shunt to remove excess CSF, and possibly removing a portion of the choroid plexus. TABLE 311 NORMAL PRESSURE HYDROCEPHALUS (NPH) Normal Pressure Hydrocephalus (NPH) Presentation

Increased CSF but no increase in ICP, gait disturbance, incontinence, & dementia.

Diagnosis

LP to rule out other causes, CT, MRI, reduced symptoms after LP.

Treatment

Therapeutic CSF removal, placing a ventriculoperitoneal shunt, possibly removing a portion of the choroid plexus.

www.ClinicalReview.com


NERVOUS SYSTEM

239

SENSORY DISTURBANCES ACUTE BLINDNESS ASSESSMENT AND MANAGEMENT There are a number of causes for sudden loss of vision. Although many of these are discussed in more detail in the Clinical Review of Ophthalmology, a brief discussion of some of these etiologies is pertinent here. Central retinal artery occlusion (CRAO) is a sudden, painless loss of vision due to ischemia to the retina. A red spot is present on the fovea. Treatment of CRAO is to remove the embolism to avoid permanent blindness. Retinal detachment is another potentially treatable cause of blindness, and commonly presents with flashes of light and floaters. Optic neuritis is a painful loss of vision on one side due to inflammation of the optic nerve leading to demyelination. Many of these patients experience this as their first manifestation of MS. Optic neuritis typically resolves with steroid therapy. Finally, vitreous hemorrhage is due to bleeding into the vitreous humor and leads to progressive blindness. Therapy is to coagulate the bleeding or perform a vitrectomy. TABLE 312 ACUTE BLINDNESS Acute Blindness Etiology

CRAO, retinal detachment, optic neuritis, vitreous humor hemorrhage.

Diagnosis

LP to rule out other causes, CT, MRI, reduced symptoms after LP

Presentation

CRAO—Sudden painless loss of vision. Retinal detachment—flashes of light and floaters. Optic neuritis—ipsilateral painful loss of vision. Vitreous humor hemorrhage—blurred vision.

Treatment

CRAO—remove embolism. Retinal detachment—photocoagulation. Optic neuritis—steroid therapy. Vitreous humor hemorrhage—Coagulate bleeding or vitrectomy.

HEARING LOSS CONDUCTIVE HEARING LOSS Hearing loss may be due to either a problem with conduction or with the vestibulocochlear nerve (CN VIII). Loss of hearing due to conductive abnormalities are often attributed to impaction of excessive amounts of cerumen, external auditory canal swelling and blockade, perforation of the tympanic membrane (30 dB), fluid within the middle ear (40 dB), cholesteatoma, otosclerosis, and abnormalities within the malleus, incus, and stapes. Otosclerosis is proliferation of the temporal bone leading to impingement of the footplate of the stapes and thus the inability of the ossicular chain to transmit mechanical waves in the fluid of the inner ear. Stapedectomy is the preferred treatment for otosclerosis; a prosthesis is then placed to permit transmission of sound.


240

USMLE STEP 2

TABLE 313 CONDUCTIVE HEARING LOSS Conductive Hearing Loss Etiology

Impaction of cerumen, external auditory canal swelling and blockade, perforation of the tympanic membrane, fluid within the middle ear, cholesteatoma, otosclerosis, and abnormalities within the ossicles.

Presentation

Hearing loss

Diagnosis

Audiometry

Treatment

Treat underlying cause; if otosclerosis, stapedectomy and prosthesis.

SENSORINEURAL HEARING LOSS Sudden loss of hearing can occur for a variety of poorly understood reasons. This condition presents an otolaryngology emergency that requires prompt treatment to help these patients regain their hearing. Sudden loss of hearing can be treated with high dose steroids that are tapered over a period of weeks. Antivirals have also been shown to have some positive effect. The combination of prednisone 60 mg over a three week taper in addition to famcyclovir 500 mg TID for 10 days has been shown to lead to recovery in 67% of all patients that experience this condition. Other types of sensorineural hearing loss may be due to damage to the hair cells of the cochlea responsible for transducing the mechanical sound vibrations into the electrical signal of the nervous system. Common causes include exposure to loud noises on a regular basis, unfavorable congenital or genetic traits, age-related changes (known as presbycusis), infection, inflammation, and tumor growth. Age-related changes tend to be symmetrical. Presbycusis may also lead to difficulties with speech discrimination. Asymmetric hearing losses may be due to acoustic neuromas of the vestibulocochlear nerve. In such patients, there is typically poor speech discrimination, tinnitus, disequilibrium, and other symptoms. Diagnosis of acoustic neuroma is preferably made through gadolinium MRI. TABLE 314 SENSORINEURAL HEARING LOSS Sensorineural Hearing Loss Etiology

Virus, loud noises, congenital, genetic, age-related, tumor growth.

Presentation

Typically poor speech discrimination, tinnitus, disequilibrium, other symptoms.

Diagnosis

Audiometry. Acoustic neuroma is preferably made through gadolinium MRI.

Treatment

High dose steroids that are tapered over a period of weeks, antivirals.

VERTIGO EPIDEMIOLOGY Vestibular disorders affect people of all ages. The incidence of vestibular dysfunction increases with age, and as many as onethird of those between 65-75 years of age report significant problems with dizziness and imbalance. Some of the more serious vestibular diseases, such as Ménière’s disease, affect 16 out of 100,000 people every year. More serious disorders can lead to imbalance, nausea, confusion and disorientation, headaches, and vertigo (which is not to be confused with dizziness).

www.ClinicalReview.com


NERVOUS SYSTEM

241

ETIOLOGY Vestibular disorders center on the sensations of vertigo, dizziness, lightheadedness, syncope, and ataxia. When dealing with patients with a chief complaint of dizziness, it is important to elucidate the precise nature of the complaint in more descriptive and precise terms. The more serious disorders can lead to imbalance, nausea, disorientation, headaches, acute and chronic vertigo, and even death. A more thorough account of vestibular disorders is presented in the Clinical Review of Otolaryngology.

PATHOPHYSIOLOGY While spinning in a circle and attempting to walk around is relatively benign, dizziness becomes significantly more disruptive with age causing falls and, therefore, one of the leading causes of death in the elderly. This situation is likely exacerbated by various medications consumed by this population that have side effects that further disrupt normal vestibular function. Differentiating central vertigo from peripheral vertigo is important. Central vertigo is gradual in onset and occurs with defects in other sensory functions. A vertical nystagmus is often present. Causes of central vertigo can be from acoustic neuroma, hemorrhage, or ischemia. Multiple sclerosis may also present with central vertigo. Peripheral vertigo presents suddenly with hearing loss and tinnitus. There is typically a horizontal nystagmus. Causes of peripheral vertigo include benign paroxysmal positional vertigo (BPPV), Ménière’s disease, labyrinthitis, and antibiotic ototoxicity.

TREATMENT Treatment of vestibular disorders should be tailored to the particular etiology. Ménière disease is typically treated with low-salt diet and diuretics, followed by labyrinthectomy, if the disorder is refractory to medical management. BPPV is treated with the Dix-Hallpike maneuver. Labyrinthitis is treated with meclizine and diazepam. TABLE 315 VERTIGO Vertigo Diagnosis

Vertigo, dizziness, lightheadedness, syncope, and ataxia.

Etiology

Central vertigo—acoustic neuroma, hemorrhage, ischemia, MS. Peripheral vertigo—BPPV, Ménière’s disease, labyrinthitis, and antibiotic ototoxicity.

Presentation

Central vertigo—gradual in onset and occurs with defects in other sensory functions. A vertical nystagmus is often present. Peripheral vertigo— presents suddenly with hearing loss and tinnitus, horizontal nystagmus.

Treatment

Tailored to particular etiology. Ménière disease is treated with low-salt diet & diuretics, possibly followed by labyrinthectomy. BPPV is treated with the Dix-Hallpike maneuver. Labyrinthitis is treated with meclizine and diazepam.

INFECTIOUS DISEASES RABIES ETIOLOGY AND PATHOPHYSIOLOGY Rabies is the progressive and lethal infection that ravages the nervous system. Rabies is exceedingly rare, but common causes include bites from raccoons, skunks, feral dogs, foxes, and exposure to bats. Corneal transplants may also lead to rabies in rare


USMLE STEP 2 242 instances. Replication of the rabies virus occurs in the region of the bite, followed by transmission to the CNS, peripheral nerves, and salivary glands.

PRESENTATION AND DIAGNOSIS The prodromal phase in rabies infection leads to irritability, hydrophobia, GI symptoms, pain, and aerophobia. The excitation phase leads to hyperactivity with disorientation and seizures. Lethargy appears later with cardiac and respiratory decompensation, shortly followed by death. Diagnosis is confirmed by the presence of Negri bodies and positive antibody tests with polymerase chain reaction (PCR).

TREATMENT Rabies is treated with rabies immunoglobulin and a human diploid cell rabies vaccine. Isolation is required and prophylactic vaccination is used with high-risk populations. TABLE 316 RABIES Rabies Etiology

Rabies virus from bites of mammals

Presentation

Prodromal phase--irritability, hydrophobia, GI symptoms, pain, & aerophobia. Excitation phase--hyperactivity with disorientation and seizures. Lethargy appears later with cardiac and respiratory decompensation.

Diagnosis

Presence of Negri bodies and positive antibody tests with PCR.

Treatment

Rabies immunoglobulin and a human diploid cell rabies vaccine.

BACTERIAL MENINGITIS ETIOLOGY AND PATHOPHYSIOLOGY Bacterial meningitis is commonly the result of infection of the pia and arachnoid membranes by Streptococcus pneumoniae (over half of all cases), Neisseria meningitidis, Listeria monocytogenes, gram-negative rods, Haemophilus influenzae, and group B streptococcus (GBS). Young adults are likely to have meningitis from N. meningitidis; IC patients are more likely to suffer from L. monocytogenes; neonates are likely to acquire GBS perinatally. Infection may occur through the bloodstream, direct invasion, or from otitis or sinusitis.

PRESENTATION AND DIAGNOSIS Meningitis presents with headache, neck stiffness, fever, and photophobia. Mental status changes leading to increasing Glasgow coma scale scores is common. Seizure is present in some patients. Kernig’s sign is typically positive with extension of the knee and thigh leading to pain in the back. Brudzinski’s sign is positive demonstrating neck flexion and leads to knee and hip flexion. CBC indicates a leukocytosis with PMNs. Blood cultures are positive in about half of all individuals. Lumbar puncture indicates increased neutrophils, increased protein, low glucose, and high opening pressure. CSF culture is more sensitive than blood culture. The presence of numerous monocytes indicates infection by L. monocytogenes.

www.ClinicalReview.com


NERVOUS SYSTEM

243

TREATMENT Treatment of bacterial meningitis is empirical and typically includes vancomycin plus cefotaxime or ceftriaxone. This is also the standard treatment for Streptococcus pneumoniae. If N. meningitidis is the identified cause, treatment can proceed with penicillin G or ceftriaxone. L. monocytogenes infection is treated with ampicillin and gentamicin. TABLE 317 BACTERIAL MENINGITIS Bacterial Meningitis Etiology

Infection of the pia and arachnoid membranes by Streptococcus pneumonia, Neisseria meningitidis, Listeria monocytogenes, gram-negative rods, Haemophilus influenzae, and GBS.

Presentation

Headache, neck stiffness, fever, and photophobia. Seizure is present in some patients.

Diagnosis

Increasing GCS scores. Kernig’s and Brudzinski’s signs are typically positive. CBC indicates a leukocytosis with PMNs. Blood cultures are positive in about half of all individuals. LP indicates increased neutrophils, increased protein, low glucose, and high opening pressure. CSF culture.

Treatment

Vancomycin plus cefotaxime or ceftriaxone. If N. meningitidis is the identified cause, treatment can proceed with penicillin G or ceftriaxone. L. monocytogenes infection is treated with ampicillin and gentamicin.

VIRAL MENINGITIS ASSESSMENT AND MANAGEMENT Viral meningitis, also referred to as aseptic meningitis, is more common than bacterial meningitis. Viral meningitis is typically benign with signs and symptoms similar to that of bacterial meningitis. Lumbar puncture (LP) typically indicates high WBCs with mostly lymphocytes, normal protein and glucose, and a normal or high opening pressure. Treatment is primarily supportive. TABLE 318 VIRAL MENINGITIS Viral Meningitis Etiology

Virus

Presentation

Similar to bacterial meningitis.

Diagnosis

LP with high WBCs, normal protein and glucose, normal or high opening pressure.

Treatment

Supportive

FUNGAL MENINGITIS ASSESSMENT AND TREATMENT Fungal meningitis presents with a high WBC count of mostly lymphocytes on LP. Protein is high, glucose is low, and opening pressure is high. Treatment of fungal meningitis is with appropriate antifungals.


244

USMLE STEP 2

TABLE 319 FUNGAL MENINGITIS Fungal Meningitis Etiology

Fungi

Presentation

High WBC count of mostly lymphocytes on LP.

Diagnosis

LP with high WBCs, protein is high, glucose low, opening pressure high.

Treatment

Antifungals

VIRAL ENCEPHALITIS ETIOLOGY AND PATHOPHYSIOLOGY Viral encephalitis is typically more of an aseptic meningoencephalitis that generally occurs in teenagers and young adults. Viruses interact with various cell surface molecules to propagate their infection; for example, the rabies virus takes advantage of acetylcholine receptors. Infection may occur through retrograde neuronal infection or spread through the vasculature. Unlike viral meningitis, viral encephalitis may lead to significant destruction and neuronal cell death and even be fatal. Nearly 1 in 10,000 persons are affected annually. Causes include HSV, VZV, influenza virus, enterovirus, rabies virus, lymphocytic choriomeningitis virus (LCV), lassa fever, mumps, measles, nipah virus, eastern equine virus (EEV), western equine virus (WEV), St. Louis encephalitis, west Nile virus, Dengue fever, Colorado tick fever, and a whole host of more rare causes.

PRESENTATION AND DIAGNOSIS Viral encephalitis presents with acute fever, headache, neck stiffness, focal neurologic findings, seizures, stupor and coma. CNS changes are common. HSV is the most common infectious cause. Lymphocytes dominate the CBC. Viral cultures and antibody tests can be attempted by collecting the CSF or even a brain biopsy in some cases. Low density lesions may be found on CT scan with HSV infection, especially around the temporal lobe. More details about these infections can be found in the Clinical Review of Neurology.

TREATMENT Treatment for viral encephalitis includes supportive management after protecting the airway. Reducing ICP and seizure prophylaxis is important. Ribavirin, ACV, and other antivirals are sometimes effective. TABLE 320 VIRAL ENCEPHALITIS Viral Encephalitis Etiology

Typically more of an aseptic meningoencephalitis that generally occurs in teenagers and young adults. Infection may occur through retrograde neuronal infection or spread through the vasculature.

Presentation

Acute fever, headache, neck stiffness, focal neurologic findings, seizures, stupor and coma. CNS changes are common.

Diagnosis

Lymphocytes dominate the CBC. Viral cultures and antibody tests can be attempted by collecting the CSF or even a brain biopsy in some cases. Low density lesions may be found on CT scan with HSV infection, especially around the temporal lobe.

Treatment

Supportive management after protecting the airway. Reducing ICP and seizure prophylaxis is important. Ribavirin, ACV, and other antivirals are sometimes effective. www.ClinicalReview.com


NERVOUS SYSTEM

245

BRAIN ABSCESS ASSESSMENT A brain abscess may develop insidiously or following trauma, surgery, or with spread from a nearby infection such as sinusitis or otitis media. The incidence has increased with AIDS, and mortality is very high if the abscess ruptures. Common causes are S. aureus, S. intermedius, Bacteroides, Prevotella, Fusobacterium, Enterobacteriaceae, Pseudomonas, and other infectious agents. Brain abscess is typically present for only a few weeks prior to diagnosis. Symptoms of neurologic impingement occur, including headache, neurologic changes and deficits, fever, seizures, nuchal rigidity, and papilledema. Rupture may lead to rapid decompensation. Abscess in the cerebellum may lead to defects in motor balance and subsequent ataxia and nystagmus. Abscess in the brainstem may lead to auditory and facial nerve defects. Frontal abscess may lead to mental status changes. Temporal lobe abscess may lead to visual defects. CT is the preferred test to determine the extent of disease.

MANAGEMENT Surgical excision with long-term antibiotic use is the standard of care following precise planning of the surgical approach with the aid of CT. Penicillin is a good choice against streptococci and staphylococci. Metronidazole is used against gram-negative bacilli. Ceftazidime is used against Pseudomonas. A third generation cephalosporin with metronidazole is commonly used with otitis, mastoiditis, and sinusitis. Dental infections get penicillin and metronidazole. Vancomycin plus a cephalosporin is used following trauma or surgery. TABLE 321 BRAIN ABSCESS Brain Abscess Etiology

May develop insidiously or following trauma, surgery, or with spread from a nearby infection such as sinusitis or otitis media. Common causes are S. aureus, S. intermedius, Bacteroides, Prevotella, Fusobacterium, Enterobacteriaceae, Pseudomonas, and other infectious agents.

Presentation

Symptoms of neurologic impingement occur, including headache, neurologic changes and deficits, fever, seizures, nuchal rigidity, and papilledema. Rupture may lead to rapid decompensation. Abscess in the cerebellum may lead to defects in motor balance and subsequent ataxia and nystagmus. Abscess in the brainstem may lead to auditory and facial nerve defects. Frontal abscess may lead to mental status changes. Temporal lobe abscess may lead to visual defects. CT is the preferred test to determine the extent of disease.

Diagnosis

CT

Treatment

Surgical excision with long-term antibiotic use is the standard of care following precise planning of the surgical approach with the aid of CT.

BOTULISM ASSESSMENT Botulism occurs due to ingestion of toxins formed by Clostridium botulinum leading to flaccid paralysis. It commonly occurs from poor preparation and canning of foods, and contamination of open wounds. It presents after about one day and its effects are due to blockade of acetylcholine release in peripheral nerves. Symptoms include dry mouth, diplopia, dysphagia, dysarthria, weakness of the extremities, and weakness of respiratory muscles. Constitutional symptoms are typically present, including nausea and vomiting, diarrhea, and abdominal cramping. Diagnosis is by serology to detect the toxin. Honey carries botulinum

Â


USMLE STEP 2 246 spores and can lead to botulism in infants through replication in their relatively immature GI tract (doubtful see recent literature). The toxin may be inactivated after ten minutes of boiling, but destroying the spores requires several hours of boiling.

MANAGEMENT Botulism is treated by administering the antitoxin and decreasing absorption by inducing vomiting or diarrhea. Penicillin and antitoxin are given if the focus is a contaminated wound. Ventilator support may be necessary to avoid respiratory failure. TABLE 322 BOTULISM Botulism Etiology

Clostridium Botulinum from poorly prepared food, contamination of open wounds.

Presentation

Dry mouth, diplopia, dysphagia, dysarthria, weakness of extremities and respiratory muscles, nausea, vomiting, diarrhea, abdominal cramps.

Diagnosis

Serology to detect toxin

Treatment

Antitoxin, ventilation

NEURODEGENERATIVE DISORDERS ALZHEIMER DISEASE (AD) ETIOLOGY AND PATHOPHYSIOLOGY Alzheimer disease results in dementia, cognitive deficits, and behavioral changes in more than 5 million people in the US. The elderly are the fastest growing population in the US, and the prevalence of AD is expected to increase significantly. The lifetime risk is on the order of 1 in 3 individuals, with increasing risk with age. AD is the leading cause of death after cancer and cerebrovascular disease, and the primary cause of death is intercurrent illness. AD increases with age and affects both men and women equally. AD is tied to the development of neurofibrillary tangles (NFTs), senile plaques (SPs), and cerebrocortical atrophy in the temporal lobe. NFTs and SPs occur in other neurodegenerative conditions and in normal aging, but AD is distinct in that the temporal lobe has a preponderance of these two anatomic defects with neuronal loss and synaptic degeneration. While NFTs and SPs are not pathognomonic for AD, their coexistence in sufficient numbers in the temporal lobe is diagnostic. Later stages of AD have NFTs and SPs in other regions of the brain. Causes include genetic risk factors such as APP on chromosome 21, presenilin I on chromosome 14, presenilin II on chromosome 1, and various other markers on chromosomes 12 and 19; advancing age-related changes, and head injury.

PRESENTATION AND DIAGNOSIS AD presents with progressive memory deficits, cognitive impairment, and personality changes leading to dementia and loss of higher order brain functions over time. Delirium is not present, which would signify an entirely separate etiology. A mini mental status examination (MMSE) and a language examination make up the repertoire of physical exam tools available to the clinician. CT and MRI are used to diagnose the cerebral atrophy and rule out other causes of brain damage. EEG has specific findings in AD.

www.ClinicalReview.com


NERVOUS SYSTEM

247

TREATMENT Treatment for AD is limited and there is no cure or therapy that slows its progression. Alleviating psychiatric and psychologic factors (such as anxiety) help somewhat, along with behavioral therapy and cognitive therapy. Managing any concurrent psychiatric ailments is important (see the Clinical Review of Psychiatry for a more thorough discussion of these agents). Anticonvulsants such as gabapentin in particular may be useful. Acetylcholinesterase (AChE) inhibitors such as donepezil, tacrine, and rivastigmine may assist in the treatment of AD by avoiding ACh depletion in the cerebral cortex and hippocampus. N-methylD-aspartate (NMDA) antagonists such as memantine may assist in various neurodegenerative conditions by preventing overstimulation of glutamate receptors. Depression is commonly found in AD, and the use of antidepressants is the key to alleviating some of the symptoms of AD. Free-radical scavengers are also occasionally used. TABLE 323 ALZHEIMER DISEASE (AD) Alzheimer Disease (AD) Etiology

APP on chromosome 21, presenilin I on chromosome 14, presenilin II on chromosome 1, and various other markers on chromosomes 12 and 19; advancing age-related changes, and head injury.

Presentation

Progressive memory deficits, cognitive impairment, and personality changes leading to dementia and loss of higher order brain functions over time.

Diagnosis

MMSE and a language examination, CT and MRI, EEG has specific findings.

Treatment

Alleviating psychiatric and psychologic factors, gabapentin, AChE inhibitors, NMDA antagonists, antidepressants, free-radical scavengers.

PARKINSON DISEASE (PD) ETIOLOGY AND PATHOPHYSIOLOGY Parkinson disease affects 1 out of 500 individuals. It is more common in men and PD increases with age, especially after the age of 60. PD is the result of loss of pigmented dopaminergic neurons from the substantia nigra (SN) especially in the ventral lateral SN. Lewy bodies (which are dense core bodies found within neurons throughout the cortex, nucleus basalis, locus ceruleus (LC), intermediolateral (IML) column, and the SN) are also found. Lewy bodies are non-specific findings in PD. Defects in the output of the basal ganglia-thalamocortical motor circuit as the transmissions pass through the SN lead to abnormalities in motor suppression. Failure to activate the direct pathway and inhibit the indirect pathway in the striatum leads to the pathognomonic features of PD, discussed below. Causes include genetic predisposition, exposure to certain pesticides or industrial toxins, and use of 1-methyl-4-phenyl-1, 2, 3, 6-tetrahydropyridine (MPTP). Oxidative damage leading to lipid peroxidation has also been implicated.

PRESENTATION AND DIAGNOSIS PD is predominantly a clinical diagnosis made by the presence of an asymmetric resting tremor, bradykinesia, rigidity, gait abnormalities, and constitutional symptoms. Tremor and motor derangements progress with axial flexor, dysphagia, and autonomic dysfunction. The three most important signs in diagnosing PD include a resting tremor, rigidity, and bradykinesia. Postural instability arises later in the disease. The rigidity takes the form of either a lead-pipe rigidity or cogwheel rigidity. Loss of spontaneous movements and slowness of motion characterizes bradykinesia. Loss of righting reflexes leads to postural instability. Dementia occurs late in the disease and affects about 1/3 of patients. Imaging studies are used only to exclude other etiologies. Positron emission tomography (PET) and single photon emission CT (SPECT) are useful for diagnosis.

Â


248

USMLE STEP 2

TREATMENT Treatment of PD centers on supportive therapy to reduce symptoms and retain baseline functioning for as long as possible. Selegiline is used as a neuroprotective agent against MPTP toxicity. Tocopherol, or vitamin E, has also been used with some benefit and increases the time before levodopa is necessary. The use of levodopa becomes necessary in reducing the symptoms of Parkinsonism, but some studies suggest that long-acting dopamine agonists such as bromocriptine, pergolide, pramipexole, ropinirole, and cabergoline are preferred in certain situations. Stereotaxic surgery to stimulate portions of the thalamus has been found to be beneficial in some groups of patients. Pallidotomy can be done to reduce dyskinesia. Newer therapeutic options may offer transplantation of dopaminergic cells to the affected region. TABLE 324 PARKINSON DISEASE (PD) Parkinson Disease (PD) Etiology

Genetic predisposition, exposure to certain pesticides or industrial toxins, and use MPTP. Oxidative damage leading to lipid peroxidation has also been implicated.

Presentation

Resting tremor, rigidity, and bradykinesia.

Diagnosis

PET, SPECT.

Treatment

Supportive therapy, selegiline, vitamin E, levodopa, some studies suggestion that long-acting dopamine agonists such are preferred in certain situations. Stereotaxic surgery, pallidotomy.

HUNTINGTON DISEASE (HD) ETIOLOGY AND PATHOPHYSIOLOGY Huntington disease (HD) is a progressive, autosomal dominant disorder that leads to neuron loss within the cortex and basal ganglia. Atrophy of the caudate and putamen nuclei occurs with atrophy of other related structures over time. The basic cause is a triplet CAG expansion in a protein product that leads to derangements in synaptic vesicles, microtubules, and mitochondria with concomitant mutant protein accumulation and the development of cellular inclusions. Disease occurs with neuronal cell death from excessive inclusion. Other causes of HD include increased excitotoxicity, oxidative stress, impaired neuronal energy metabolism, and apoptosis. HD affects 5 out of 100,000 people and is more common in certain European populations. It manifests itself in the 50s with death several decades later, primarily from pneumonia or cardiovascular disease. HD in younger patients reflects genetic inheritance with anticipation.

PRESENTATION AND DIAGNOSIS HD presents with chorea, a movement disorder that begins with increased fidgeting followed by flailing of the extremities in a phenomenon known as hemiballism. Chorea progresses to dystonia, bradykinesia, rigidity, postural instability, and finally, an akinetic-rigid state. Clonus and spasticity are present later in disease. Ocular abnormalities and tic disorders are common early on. Early onset of HD (known as the Westphal variant) leads to early dementia and the development of a seizure disorder. Impairments in cognition proceed at variable rates, and depression is common. Changes in personality can occur. Imaging tests are used to rule out other disorders. Genetic testing is used to confirm the diagnosis by identifying multiple CAG repeats.

TREATMENT Treatment involves reducing the chorea with agents such as tetrabenazine, various benzodiazepines, or anticonvulsants. Levodopa can be used to modify the bradykinesia and rigidity. Depression should be treated early with selective serotonin www.ClinicalReview.com


NERVOUS SYSTEM

249 reuptake inhibitors (SSRIs). Antipsychotics may be used to modify psychotic symptoms and irritability. Personality changes should be addressed with medications as appropriate. Surgical ablation and cell transplant options are currently being studied. TABLE 325 HUNTINGTON DISEASE (HD) Huntington Disease (HD) Etiology

Autosomal dominant disorder.

Presentation

Chorea, dystonia, bradykinesia, rigidity, postural instability, and finally, an akinetic-rigid state. Clonus and spasticity are present later in disease. Ocular abnormalities and tic disorders are common early on. Early onset of HD leads to early dementia and the development of a seizure disorder. Impairments in cognition proceed at variable rates, depression is common.

Diagnosis

Imaging tests are used to rule out other disorders. Genetic testing is used to confirm the diagnosis.

Treatment

Tetrabenazine, benzodiazepines, or anticonvulsants. Levodopa can be used to modify the bradykinesia and rigidity. Depression should be treated with SSRIs. Antipsychotics may be used to modify psychotic symptoms and irritability.

SLEEP DISORDERS NORMAL SLEEP STAGES Sleep is separated into four distinct stages and REM sleep. Stage 1 is typically only 5% of the sleep cycle, and is a period of light sleep. Stage 2 of sleep is about 45% of the time, and is a deeper sleep. The deepest sleep occurs in stages 3 and 4, during which one-quarter of the sleep cycle is spent. Stages 3 and 4 of sleep are the period during which night terrors, sleepwalking (somnambulism), and bed-wetting occurs. REM sleep consumes one-quarter of the sleep cycle, and is the period during which dreaming occurs. There is a loss of muscle tone during REM sleep, increased processing by the brain of the daily events, memory retention, erections, and elevated use of oxygen by the brain. REM sleep occurs every 90 minutes, increases in duration during the night, and decreases with age. The extraocular movements that occur in REM sleep are due to action potentials emanating from the paramedian pontine reticular formation, also known as the conjugate gaze center.

NORMAL SLEEP WAVEFORMS Waveforms vary by the period of sleep or wakefulness. When a person is awake with their eyes open, beta waves, or high frequency, low amplitude waves, predominate. A person who is awake with their eyes closed has mostly alpha waves. Stage 1 of sleep is characterized by theta waves. Stage 2 of sleep has sleep spindles and K-complexes. Stages 3 and 4 of sleep have delta waves, which are the lowest frequency but highest amplitude waves found on an EEG. REM sleep has beta waves. The mnemonic BATS Drink Blood can be used to remember the waveforms seen in wakefulness and sleep. TABLE 326 STAGES OF SLEEP & THEIR ASSOCIATED WAVE FORMS Stages of Sleep & Their Associated Wave Forms Stage 1, Light sleep

Theta waves

Stage 2, Deeper sleep

Sleep spindles and K complexes

Stage 3, Deepest sleep

Delta waves

Stage 4, Deepest sleep

Delta waves

REM, Rapid eye movement and dreaming

Beta waves

Â


250

USMLE STEP 2

PATHOPHYSIOLOGY IN SLEEP STAGES Sleep is initiated by the serotonergic raphe nuclei. Acetylcholine aids in initiating the onset of REM sleep. Elevated levels of norepinephrine reduce REM sleep. Night terrors and sleepwalking seen in stages 3 and 4 can be reduced through the use of benzodiazepines. Bed-wetting, known as enuresis, can be reduced with imipramine. TABLE 327 SLEEP DISORDER TREATMENTS Sleep Disorder Treatments Night terrors & sleep walking

Benzodiazepines

Bedwetting

Imipramine

PATHOPHYSIOLOGY OF SLEEP DISORDERS Sleep disorders are separated into dyssomnias and parasomnias. Dyssomnias are primary sleep disorders that are characterized by a difficulty in initiating or maintaining sleep. This leads to difficulty in feeling rested after sleep, and can lead to excessive sleepiness. The five types of dyssomnias are insomnia, hypersomnia, narcolepsy, breathing-related sleep disorder, and circadian rhythm sleep disorder. TABLE 328 DYSSOMNIAS Sleep Disorder Insomnia

Hypersomnia

Narcolepsy

Breathing-related sleep disorders

Circadian rhythm sleep disorder

PATHOPHYSIOLOGY OF PARASOMNIAS Parasomnias are disorders that occur during sleep that can lead to arousal from sleep. Parasomnias include excessive nightmares, sleep terrors, and sleep walking. Secondary sleep disorders can also occur, and are attributable to other psychiatric illnesses, a general medical condition, or a substance-induced sleep disorder. TABLE 329 PARASOMNIAS Parasomnia Nightmares

Sleep terrors

Sleep walking

Related to another mental disorders

General medical condition

Substance-induced

PATHOPHYSIOLOGY OF NARCOLEPSY Narcolepsy occurs when an individual suddenly falls asleep. Hallucinations may accompany narcolepsy – if the hallucinations occur just as the person is falling asleep, they are known as hypnagogic hallucinations; those that occur just as the person is waking up are known as hypnapompic hallucinations. A person who experiences an episode of narcolepsy while standing, and then falls to the floor, experiences cataplexy. Narcolepsy is best treated with stimulants. www.ClinicalReview.com


NERVOUS SYSTEM

251

TABLE 330 NARCOLEPSY Narcolepsy Hypnagogic

Hallucination occurs when patient falling asleep

Hypnapompic

Hallucinations occur as person is waking up

Cataplexy

Narcolepsy occurs while standing

Treatment

Stimulants

EPILEPSY ETIOLOGY AND PATHOPHYSIOLOGY Epilepsy is the presence of recurrent seizures, or abnormal bursts of CNS activity leading to abnormalities in motor behavior, autonomic activity, and changes in consciousness. There are two types of seizures, generalized and partial seizures. Generalized seizures are broken down into tonic-clonic, or grand mal seizures, absence or petit mal seizures, atonic seizures, and myoclonic seizures. Common causes of seizure include vascular defects, cerebral infection, penetrating trauma to the head, autoimmune disorders, metabolic derangements, neoplasm, psychiatric causes, and idiopathic causes.

PRESENTATION – TONIC-CLONIC SEIZURES Tonic-clonic seizures begin with a loss of consciousness (LOC) and loss of postural control, followed by the tonic phase. The tonic phase is characterized by muscle rigidity throughout the body. The clonic phase follows, with rhythmic contractions of the extremities. Incontinence is often a feature of tonic-clonic seizures. EEG changes reflect the abnormal activity taking place.

PRESENTATION – ABSENCE SEIZURES Absence seizures are brief disruptions of consciousness leading to the individual appearing as if they are not paying attention or concentrating on the task at hand. There are occasionally some motor cues, including lip-smacking, chewing, or partial loss of motor tone. Bilaterally synchronous slow wave activity is seen on EEG.

PRESENTATION – ATONIC SEIZURES Atonic seizures are brief losses of consciousness and postural tone, leading to a sudden fall to the floor in standing individuals. This type of seizure is similar to the cataplexy of narcolepsy.

PRESENTATION – MYOCLONIC SEIZURES Myoclonic seizures are muscle contractions without a superimposed loss of consciousness. Myoclonic seizures are more common in neurodegenerative disorders.

PRESENTATION – PARTIAL SEIZURES Partial seizures are broken down into simple and complex partial seizures. Simple partial seizures have a straightforward sensory, motor, or autonomic abnormality during the seizure. The nature of the defect is dependent on the particular part of the brain that has the epileptiform focus. Complex partial seizures meet the criteria for simple partial seizures, except there is a


USMLE STEP 2 252 superimposed disturbance in cognition. This type of seizure activity is the most common type of seizure disorder found in adults. Dream-like sensations may be present during the seizure.

PRESENTATION – STATUS EPILEPTICUS AND PREICTAL SYMPTOMS Seizures can present with preictal symptoms such as auras or sensations. Postictal symptoms typically include delirium, amnesia, or focal paralysis (known as Todd paralysis). Status epilepticus presents with continuous seizures.

DIAGNOSIS Diagnosis of seizure activity is made by electroencephalogram. Derangements in electrolytes and other causes of seizures must be identified and appropriately treated. CT scan or MRI is used to identify any anatomic defects.

TREATMENT Seizure treatment includes securing the ABCs. Reversible causes should be treated immediately. The initial drugs of choice are benzodiazepines, including lorazepam or diazepam. If potentiating GABA function does not inhibit the epileptic focus, treatment with phenytoin or fosphenytoin is initiated to inhibit sodium-dependent action potentials. Phenobarbital, a barbiturate, is added. Finally, if seizures continue with this complex regimen, midazolam or propofol can be added to induce anesthesia. First-time seizures are treated only with a clear family history, abnormal EEG, or abnormal neurologic exam. Tonic-clonic seizures are primarily treated with valproic acid, followed by lamotrigine to increase GABA availability or decreasing glutamate release, respectively. Absence seizures are treated with ethosuximide or valproic acid. Myoclonic and atonic seizures are primarily treated with valproic acid. Partial seizures are all treated with carbamazepine, phenytoin, valproic acid, or lamotrigine.

COMPLICATIONS Side effects of seizure treatment are numerous. Phenytoin may lead to ataxia, dizziness, diplopia, hirsutism, and rash. Phenobarbital can lead to ataxia and rash. Valproic acid may cause ataxia, hepatotoxicity, thrombocytopenia, GI irritation, or hyponatremia. Lamotrigine is implicated in causing Stevens-Johnson syndrome, in addition to rash and ataxia. TABLE 331 EPILEPTIC SEIZURES Sleep Disorder Treatments

Types

Tonic-clonic—LOC, muscle rigidity throughout the body, incontinence, EEG changes, rhythmic contraction of extremities (Grand mal). Absence—Brief disruption of consciousness, person appears as if not paying attention, bilateral slow wave EEG (Petit mal). Atonic—Brief LOC. Similar to cataplexy. Myoclonic-- muscle contractions without a superimposed LOC. Simple partial--straightforward sensory, motor, or autonomic abnormality during the seizure. The nature of the defect is dependent on the particular part of the brain that has the epileptiform focus. Complex partial--criteria for simple partial seizures, except there is a superimposed disturbance in cognition. Dream-like sensations may be present during the seizure. Status epilepticus—Continuous seizure.

Presentation

Preictal symptoms such as auras or sensations. Postictal symptoms typically include delirium, amnesia, or focal paralysis.

Diagnosis

EEG. Derangements in electrolytes and other causes of seizures must be identified and treated. CT or MRI is used to identify any anatomic defects.

www.ClinicalReview.com


NERVOUS SYSTEM

253

Treatment

Secure ABCs. The initial drugs of choice are benzodiazepines, treatment with phenytoin or fosphenytoin, phenobarbital, a barbiturate. Finally, if seizures continue with this complex regimen, midazolam or propofol can be added to induce anesthesia. Tonic-clonic seizures are primarily treated with valproic acid, followed by lamotrigine. Absence seizures are treated with ethosuximide or valproic acid. Myoclonic and atonic seizures are primarily treated with valproic acid. Partial seizures are all treated with carbamazepine, phenytoin, valproic acid, or lamotrigine.

Complications

Phenytoin--ataxia, dizziness, diplopia, hirsutism, rash. Phenobarbital--to ataxia and rash. Valproic acid--ataxia, hepatotoxicity, thrombocytopenia, GI irritation, or hyponatremia. Lamotrigine--Stevens-Johnson syndrome, in addition to rash and ataxia.

CANCER TYPES

ETIOLOGY

PRESENTATION

NOTES

Glioblastoma multiforme

Palisading tumor cells with necrosis, hemorrhage, rapid growth, bilateral.

Elderly most affected; most common primary tumor; malignant, often fatal; found in cerebral hemispheres.

Meningioma

Psammoma bodies.

Women most affected especially at cerebral hemispheres; benign tumor outside of brain; treat with resection.

Palisading tumor cells; typically vestibulocochlear schwannoma; if bilateral, consider NF-2 (Chr 22).

Later in life or hereditary.

Oligodendroglioma

Fried egg appearance, calcifications.

Slow growing, indolent.

Pituitary adenoma

Bitemporal hemianopsia, typically secretes prolactin (amenorrhea in women with gynecomastia).

Rathke’s pouch.

Astrocytoma

Diffuse with Rosenthal fibers.

Posterior fossa, children, benign.

Medulloblastoma

Hydrocephalus, mass effects, rosettes.

Children most affected, especially at cerebellum; very malignant but radiosensitive.

Ependymoma

Rosettes with rod-shaped inclusions.

Most common spinal cord tumor with metastasis to vertebral body; 4th ventricle.

Hemangioblastoma

Foam cells, vascularity, polycythemia from EPO production.

Cerebellar, VHL syndrome (WT-1 w/ aniridia).

Craniopharyngioma

Calcification in remnant of Rathke’s pouch – no hormone production.

Benign, childhood.

Unilateral or bilateral retinal tumor development.

Occurs in children, commonly in cerebral hemispheres.

Schwannoma

NF-2 or spontaneous

Neuroblastoma

N-myc oncogene

Retinoblastoma

Rb gene deletion

Very young children most affected; most common eye tumor of children.

Chordomas are embryonal tumors found near the sacrum; empty sella syndrome – missing pituitary on CT but functional.


USMLE STEP 2 254 Arnold-Chiari malformation: small posterior fossa, cerebellum malformation, vermis herniation, hydrocephalus – presents with stridor, no gag reflex, dysphagia, spastic quadriparesis, nystagmus, syncope, progressive LOF, pneumonia, GERD, and more severe form is quickly fatal. Congenital condition. Dandy-Walker malformation: large posterior fossa, no vermis with CSF foramen atresia – presents with ataxia, syringomyelia, microcephaly, spina bifida, cardiac anomalies.

TUMORS EPIDEMIOLOGY AND ETIOLOGY Brain tumors kill over 13,000 patients a year, but metastatic tumors to the brain occur in more than 80,000 additional cases. Metastasis occurs most commonly from the breast, lung, and melanomas. Brain tumors are the most common solid tumor in children. The most common tumors include gliomas, meningiomas, and schwannomas. Brain tumors in adults are commonly the result of exposure to radiation or HIV infection. Genetic transmission also occurs, but most tumors tend to occur earlier in age. A thorough discussion of brain tumors can be found in the Clinical Review of Neurology, and an examination of pediatric brain tumors can be found in the Clinical Review of Pediatrics.

PATHOPHYSIOLOGY – ASTROCYTOMA Astrocytomas are the most common brain tumor in adults. Most astrocytomas are high grade tumors with a poor prognosis. A particular type of astrocytoma is a glioblastoma multiforme (GBM), an aggressive and often fatal tumor.

PATHOPHYSIOLOGY – OLIGODENDROGLIOMA Oligodendrogliomas are calcifying tumors that carry a somewhat better prognosis than astrocytomas. Many of these tumors are benign.

PATHOPHYSIOLOGY – MENINGIOMA Meningiomas are a common mesodermal tumor that presents as a cranial nerve palsy. These tumors are typically slow growing.

PATHOPHYSIOLOGY – SCHWANNOMA Schwannomas are common cranial nerve tumors that most commonly presents as an acoustic neuroma – a title that is inaccurate considering that it neither directly affects the acoustic nerve nor is a neuroma. These tumors are located on the vestibular nerve.

PATHOPHYSIOLOGY – LYMPHOMA CNS lymphomas occur due to derangements in B cell function and lead to headaches, visual defects, and personality changes. Chemotherapy and radiation are commonly used for treatment.

PATHOPHYSIOLOGY – EPENDYMOMA Ependymomas are typically found in the spinal canal and have an excellent prognosis if found early.

www.ClinicalReview.com


NERVOUS SYSTEM

255

TABLE 332 TUMOR TYPES Tumor Types Astrocytoma

High grade tumor, poor prognosis. Tumor composed of astrocytes.

Oligodendroglioma

Calcifying tumors that carry a somewhat better prognosis than astrocytomas. Many of these tumors are benign.

Meningioma

Common mesodermal tumor that presents as a cranial nerve palsy. These tumors are typically slow growing.

Schwannoma

Common cranial nerve tumors that most commonly presents as an acoustic neuroma. These tumors are located on the vestibular nerve.

Lymphoma

Derangements in B cell function and lead to headaches, visual defects, and personality changes.

Ependymoma

Found in the spinal canal and have an excellent prognosis if found early.

PRESENTATION AND DIAGNOSIS Brain tumors present with headache in most patients. The headache is present on awakening and disappears within an hour or so. Headaches with brain tumors can rouse a patient from sleep, and typically worsen when lying supine. A common cause of a new headache in older patients is a brain tumor. Nausea and vomiting are typically present, along with focal neurologic changes including vision loss, weakness, and seizures. Diagnosis is made by CT, MRI, and confirmed by biopsy.

TREATMENT Treatment for brain tumors consists of decreased ICP with steroids, followed by surgical resection, chemotherapy, and radiation. TABLE 333 BRAIN TUMORS Brain Tumors Etiology

Occurs most commonly from the breast, lung, and melanomas. The most common childhood tumors include gliomas, meningiomas, and schwannomas. Brain tumors in adults are commonly the result of exposure to radiation or HIV infection. Genetic transmission also occurs.

Presentation

Headache is present on awakening and disappears within an hour or so. Headaches can rouse a patient from sleep, and typically worsen when lying supine. Nausea and vomiting are typically present, along with focal neurologic changes including vision loss, weakness, and seizures.

Diagnosis

Diagnosis is made by CT, MRI, and confirmed by biopsy.

Treatment

Decrease ICP with steroids, followed by surgical resection, chemotherapy, and radiation.

PAROTID TUMOR The most common benign parotid tumor is a pleomorphic adenoma, which can be treated with a superficial parotidectomy. A mobile parotid tumor (typically a low grade acinar tumor) can also be treated with a superficial parotidectomy with sparing of the facial nerve. This is also the treatment of choice when an indeterminate fine needle aspiration results on cytology.80% of parotid tumors are benign in nature. The most common malignant tumor of the parotid is a mucoepidermoid tumor, which may elicit a total parotidectomy. Every attempt is made to spare the facial nerve unless tumor is it is invaded by tumor.

Â


USMLE STEP 2

256

PRACTICE QUESTIONS Which of the following is the most common site of an intervertebral disc herniation? A. B. C. D. E.

L2-L3 L3-L4 L4-L5 L5-S1 T12-L1

The best answer is L4-L5. L4-L5 is the most common site of an intervertebral disc herniation, which most commonly presents as a posterolateral herniation. Chronic back pain is common, and loss of big toe dorsiflexion may be a presenting sign with nerve impingement. Foot drop is another presenting sign, along with pain on a straight leg raise. Herniation along L3-L4 leads to a weak knee jerk reflex. Herniation at L5-S1 prevents the patient from standing on their tiptoes, weak ankle jerk, and weak plantar flexion. Treatment is typically with NSAIDs. Lifestyle alterations and physical findings with symptoms over 1-2 months are treated with surgery.

A 32 year old patient involved in an ATV accident presents with a displaced skull fracture of approximately 2 cm. What is the next best course of action? A. B. C. D. E.

Antibiotics only Drain placement and observation Observation Seizure prophylaxis Washout, primary closure, and antibiotics

The best answer is Washout, primary closure, and antibiotics. As this patient has a displaced skull fracture greater than the width of the bone, they should be taken to the OR for a washout and primary closure. Antibiotics are necessary. Skull fractures without significant displacement can be treated with observation only. Seizure prophylaxis is typically required in all circumstances.

Which of the following nerves can be injured leading to the inability to make high pitched sounds? A. B. C. D. E.

External superior laryngeal nerve Hypoglossal nerve Internal superior laryngeal nerve Recurrent laryngeal nerve Vagus nerve

The best answer is External superior laryngeal nerve. The external superior laryngeal nerve travels with the vagus and superior thyroid artery and supplies the cricothyroid muscle. Damage to this nerve prevents the formation of high pitched sounds. The internal superior laryngeal nerve is a sensory nerve that also controls secretions from the larynx.

www.ClinicalReview.com


NERVOUS SYSTEM

257

A 59 year old female who recently had a modified radical mastectomy with axillary lymph node dissection reports difficulty with getting herself out of a sitting position. She has difficulty pulling on things with her arm. Which of the following nerves is most likely to have been injured? A. B. C. D. E.

Intercostobrachial nerve Lateral pectoral nerve Long thoracic nerve Posterior cord of the brachial plexus Thoracodorsal nerve

The best answer is Thoracodorsal nerve. The thoracodorsal nerve supplies the latissimus dorsi, which is responsible for arm and shoulder adduction, a muscle maneuver used to help people get out of sitting positions. The long thoracic nerve leads to winged scapula. The intercostobrachial nerve is a purely sensory nerve to the medial aspect of the upper arm and chest. Injury to the pectoral nerves is uncommon. The posterior cord of the brachial plexus should never be injured in an axillary dissection.

Which of the following is not one of the changes that occur in neurogenic shock? A. B. C. D. E.

Decreased CO Decreased PCWP Decreased SVO2 Decreased SVR Increased CVP

The best answer is Increased CVP. In neurogenic shock, injury to the central nervous system and spinal cord may present as warm extremities with hypotension and bradycardia. Fluid resuscitation is required. Presenting signs include a decrease in CVP, CO, SVO2, SVR, and PCWP. Early treatment of spinal shock is pressors.

Â


258

USMLE STEP 2

www.ClinicalReview.com


NERVOUS SYSTEM

259

CHAPTER CONTENTS Basic Science .....................................................................................................260 Upper Respiratory Infections and Illnesses ......................................................263 Lower Respiratory Infections ............................................................................267 Obstructive Lung Disease..................................................................................273 Restrictive Lung Disease ...................................................................................279 Pulmonary Disease ...........................................................................................282 Lung Cancer ......................................................................................................287 Mediastinal Disease ..........................................................................................289 Practice Questions ............................................................................................290

Â


260

USMLE STEP 2

RESPIRATORY SYSTEM RESPIRATORY SYSTEM BASIC SCIENCE ANATOMY LUNG ALVEOLI Type I alveoli form the structure of the alveolar wall. Type II alveoli secrete surfactant to decrease the surface tension of water and permit gas exchange. Type III alveoli are immune cells that destroy foreign invaders.

ESOPHAGUS The layers of the esophagus include the innermost mucosa (which includes the lamina propria), muscularis mucosa, submucosa, muscularis propria (between the inner circular and outer longitudinal muscle layers), then the adventitia. The submucosal plexus is also known as Meissner plexus; the muscular propria plexus is also known as Auerbach plexus.

PHYSIOLOGY OXYGEN-HEMOGLOBIN DISSOCIATION CURVE

www.ClinicalReview.com


Respiratory System

261

TABLE 334 OXYGEN-HEMOGLOBIN DISSOCIATION CURVE FACTORS Variable

Right Shift

Left Shift

Temperature

High

Low

2,3 DPG

High

Low

PCO2

High

Low

PCO

Low

High

pH

Acidosis

Alkalosis

Hemoglobin

Adult hemoglobin

Fetal hemoglobin

PULMONARY FUNCTION TESTS Pulmonary function tests (PFTs) are a mainstay for diagnosis of various pulmonary etiologies. PFTs measure different aspects of lung volume, including the total lung capacity (TLC), residual volume (RV), forced expiratory volume over 1 second (FEV1), forced vital capacity (FVC), and midmaximal forced expiratory flow (FEF). The permeability of the alveoli is tested with the diffusability of carbon monoxide (DLCO). Bronchial hyperreactivity is tested with a methacholine challenge test, and is contraindicated in asthmatics. TABLE 335 PULMONARY FUNCTION TESTS Pulmonary Function Tests Total lung capacity

Residual volume

Force expiratory volume

Forced vital capacity

Midmaximal forced expiratory flow

Diffusability of carbon monoxide

Metacholine challenge The TLC is the entire volume of air that the lung can retain with maximal inspiration. RV is the remaining volume after maximum expiration, and is a volume of air that is always present. The vital capacity (VC) is the difference in volume between the TLC and RV. The functional reserve capacity (FRC) is the volume that remains after a normal breath, in addition to the RV. The inspiratory capacity (IC) is the maximum inspiratory volume that can be taken in with a normal breath. The IC and FRC add up to the VC, and, as stated above, the VC plus the RV equal the TLC. With a normal breath, the inspiration is known as the tidal volume (VT). The additional volume that is available for inspiration is known as the inspiratory reserve volume (IRV), while the additional volume available for expiration is the expiratory reserve volume. The IRV plus VT plus ERV equal the VC. With a normal breath, the ERV is equal to the FRC.

TLC = RV + VC = RV + FRC + IC = RV + ERV + VT + IRV VC = FRC + IC = ERV + VT + IRV

FRC = ERV (normal breath)

Expiratory flow rate =

FEV1 = FEF FVC

The diffusing capacity of the alveoli (DLCO) is generally decreased with interstitial lung disease and emphysema.

Â


262

USMLE STEP 2 FEV1 > 0.8L required prior to lung surgery. FEV1 > 2L prior to pneumonectomy.

Pulmonary function tests (PFTs) are a mainstay for diagnosis of various pulmonary etiologies. PFTs measure different aspects of lung volume, including the total lung capacity (TLC), residual volume (RV), forced expiratory volume over 1 second (FEV1), forced vital capacity (FVC), and midmaximal forced expiratory flow (FEF). The permeability of the alveoli is tested with the diffusability of carbon monoxide (DLCO). Bronchial hyperreactivity is tested with a methacholine challenge test, and is contraindicated in asthmatics. Pulmonary vascular resistance increases with hypoxia. PFTs are required before lung surgery and must have a predicted post-op FEV1 greater than 0.8. FEV1 must be at least 2L prior to a pneumonectomy, 1L prior to a lobectomy, and 0.6L prior to a wedge resection.

GAS EXCHANGE AND HYPOXIA Disorders with gas exchange can lead to impaired oxygen delivery to vital tissues and present with symptoms of hypoxia. Oxygen transport relies on sufficient cardiac output and hemoglobin of sufficient saturation and quantity. In patients with poor CO or Hgb, giving the patient 100% oxygen does little to improve the oxygenation. Calculating oxygenation can be done with the alveolar-arteriolar gradient:

PAO2 = 150 -

PaCO 2 0.8

A - a gradient = PAO2 - Pa O2 Faulty gas exchange can manifest as hypoxia. Causes of hypoxia can be attributed to poor ventilation-perfusion (V-Q) mismatch leading to poorly ventilated alveoli or ventilation of poorly perfused alveoli; a right to left cardiac shunt that moves poorly oxygenated blood into the systemic circulation; anemia that leads to decreased oxygen carrying capacity in the blood and subsequent poor oxygenation; poor perfusion from MI or shock; increased oxygen demand with poor increases in supply; impairment in oxygen delivery due to cyanide or carbon monoxide poisoning; and low inspired oxygen at higher altitudes.

www.ClinicalReview.com


Respiratory System

263

TABLE 336 CAUSES OF HYPOXIA Causes of Hypoxia Poor ventilation-perfusion mismatch; and low inspired oxygen at higher altitudes.

A right to left cardiac shunt that moves poorly oxygenated blood into the systemic circulation.

Anemia that leads to decreased oxygen carrying capacity.

Increased oxygen demand with poor increases in supply.

Impairment in oxygen delivery due to CN or CO poisoning.

Poor perfusion from MI or shock.

Low inspired oxygen at higher altitudes.

PHARMACOLOGY CROMOLYN DRUG Cromolyn sodium

INDICATIONS Asthma prophylaxis

MECHANISM OF ACTION

NOTES

Stabilizes mast cell membranes to prevent histamine release

Used only for prophylaxis. No effect in active disease.

CORTICOSTEROIDS DRUG

INDICATIONS

MECHANISM OF ACTION

COMPLICATIONS

NOTES

Asthma prophylaxis

Prednisone

Immunosuppression Nephrotic syndrome Crohn disease Organ rejection Autoimmune disease

Suppresses cytokine production and decreases TNF-α action

Dependency Addisonian crisis if sharp cessation Hyperglycemia

First line for chronic asthma

ANTILEUKOTRIENES DRUG

Zileuton

INDICATIONS

Asthma prophylaxis

MECHANISM OF ACTION

Inhibits 5-lipoxygenase to inhibit leukotriene synthesis

UPPER RESPIRATORY INFECTIONS AND ILLNESSES POSTNASAL DRIP ASSESSMENT AND MANAGEMENT


USMLE STEP 2 264 The most common cause of cough is due to postnasal drip, commonly the result of a URI, sinusitis, rhinitis, or allergic rhinitis. Pertussis, COPD, asthma, CHF, pneumonia, aspiration, and PE are other causes of postnasal drip. Presentation is with a feeling of nasal discharge into the back of the throat with drainage into the hypopharynx and subsequent elicitation of the coughing reflex. Treatment is to cure the underlying etiology, whether with antihistamines and decongestants for the common cold, or with corticosteroids and antihistamines if allergy-induced. TABLE 337 POSTNASAL DRIP Postnasal Drip Cause

URI, sinusitis, rhinitis, allergic rhinitis, pertussis, COPD, asthma, CHF, pneumonia, aspiration, & PE.

Presentation

Feeling of nasal discharge into the back of the throat with drainage into the hypopharynx and subsequent coughing reflex.

Treatment

Antihistamines and decongestants for the common cold, or corticosteroids and antihistamines if allergyinduced.

OTITIS MEDIA ETIOLOGY AND PATHOPHYSIOLOGY Otitis media is a middle ear infection often due to a dysfunction of the Eustachian tube. Obstruction of this passage leads to negative pressure in the middle ear, which can cause fluid to migrate from the nasopharynx into the middle ear cavity. This environment is conducive to growth of the normal flora of the nasopharynx, especially Streptococcus pneumoniae, Haemophilus influenzae, and Moraxella catarrhalis. Otitis media resolves completely in 50% of patients within 14 days. By the ten-week mark, nearly 90% of all patients have had complete resolution.

PRESENTATION AND DIAGNOSIS Otitis media with effusion (OME) in adults, especially when unilateral, indicates that a thorough work-up for a nasopharyngeal carcinoma and other disease processes in this region. Nasopharyngeal carcinoma, if not properly diagnosed, may eventually metastasize to cervical lymph nodes, cause osteomyelitis, and lead to various neuropathies. Endoscopic exam of the nasopharynx is required. Acute otitis media (AOM) can lead to mastoiditis and meningitis if not treated quickly. These serious complications are rare due to typically prompt treatment with antibiotics. Meningitis is most commonly due to blood borne transmission of Haemophilus influenzae. Other complications include rupture of the tympanic membrane due to elevated pressure from fluid collection.

TREATMENT Treatment of otitis media from any of these three causes is primarily with amoxicillin or trimethoprim and sulfamethoxazole for a duration of ten days. A beta-lactamase may also be added to the regimen as dictated by resistance, especially by Streptococcus pneumoniae. A culture may be required for further treatment. Children, however, tend to cycle through repeated otitis media infections, a condition known as recurrent otitis media. Such patients often require small tubes to be placed through the tympanic membrane to help equalize the pressure between the middle ear and the outside atmosphere. Equalizing the pressure helps to prevent transudation of the contaminated fluid of the nasopharynx into the middle ear cavity. The tubes tend to fall out without medical intervention. With developmental changes of the eustachian, future bouts of otitis media are less likely. In the event of a perforation, treatment with a 0.3% solution of ofloxacin is indicated. Spontaneous repair of the perforation is common, but in some cases, chronic otitis media may result if the hole is not repaired. www.ClinicalReview.com


Respiratory System

265

TABLE 338 OTITIS MEDIA Otitis Media Etiology

Dysfunction of the Eustachian tube due to infection.

Presentation

Pain in middle ear.

Treatment

Primarily with amoxicillin or TMP/SMX for a duration of ten days. A beta-lactamase may also be added to the regimen as dictated by resistance, tubes in some children.

VIRAL PHARYNGITIS ETIOLOGY AND PATHOPHYSIOLOGY The majority of pharyngitis is viral in nature and is often referred to as the common cold. Viral pharyngitis is commonly the result of infection by rhinovirus, adenovirus, EBV, HSV, influenza virus, parainfluenza virus, coronavirus, enterovirus, RSV, CMV, or HIV infection leading to inflammation in the oropharynx, hypopharynx, and tonsils. Viral pharyngitis is extremely common, and the typical person will experience several infections yearly.

PRESENTATION AND DIAGNOSIS Presentation of viral pharyngitis is with a sore throat. Nasal discharge is common, leading to significant post nasal drip leading to a cough. A thick yellow discharge may occur with viral pharyngitis and a cough is nonproductive. A low grade fever is sometimes present. Adenovirus is particularly common in children and military personnel and may also present with conjunctivitis. EBV occurs more in young adults, along with HSV. The latter may lead to gingivostomatitis and decreased PO intake. Influenza can lead to pharyngitis in the winter months. Enteroviruses may present with significant odynophagia and a rash. RSV affects those with pulmonary conditions or at extremes of age. CMV tends to affect sexually active adults. Pharyngeal edema and erythema are typically present, and symptoms of nasal irritation may also be found on exam. Diagnosis of viral pharyngitis is made by CBC, which identifies a somewhat elevated WBC count, an occasional lymphocytosis, and negative rapid streptococcal antigen test and bacterial culture of the throat. Viral cultures are rarely done.

TREATMENT Antibiotics are entirely unnecessary for viral infections, except as prophylaxis against bacterial infections in certain high risk individuals. Oral hydration, analgesics for throat irritation and pain, antipyretics, and relief from coughing are the standard of care. Influenza may be treated with amantadine or rimantadine within 2 days of onset; there is little efficacy against influenza B with these two agents. Ribavirin is also useful. Zanamivir and Oseltamivir have been shown to decrease the duration of infection with influenza. EBV is treated with acyclovir (ACV), ganciclovir (GCV), and interferon alpha. HSV is treated with ACV, famcyclovir (FCV), and valacyclovir (VCV).

Â


266

USMLE STEP 2

TABLE 339 VIRAL PHARYNGITIS Viral Pharyngitis Etiology

Commonly the result of infection by rhinovirus, adenovirus, EBV, HSV, influenza virus, parainfluenza virus, coronavirus, enterovirus, RSV, CMV, or HIV infection.

Presentation

Sore throat, nasal discharge, significant post nasal drip leading to cough. A thick yellow discharge may occur with an unproductive cough. A low grade fever is sometimes present.

Diagnosis

Pharyngeal edema and erythema, and symptoms of nasal irritation may also be found.

Treatment

Prophylaxis against bacterial infections in certain high risk individuals. Oral hydration, analgesics for throat irritation and pain, antipyretics, and relief from coughing, antivirals.

BACTERIAL PHARYNGITIS ETIOLOGY AND PATHOPHYSIOLOGY Bacterial pharyngitis makes up less than a quarter of all cases of pharyngitis, and occurs most during the winter months and early spring. The most common cause of bacterial pharyngitis is group A streptococcus (GAS), leading to nearly 10 million diagnosed cases of infection. Rapid identification and appropriate treatment of GAS pharyngitis is important in a clinical setting to reduce morbidity and mortality. Related sequelae can include rheumatic fever with subsequent valvular damage and cardiac dysfunction, poststreptococcal glomerulonephritis (PSGN) that may lead to hematuria, proteinuria, and end stage renal disease (ESRD), and related upper respiratory tract infections such as otitis media, abscess formation, and sinusitis.

PRESENTATION AND DIAGNOSIS GAS pharyngitis presents with constitutional symptoms, which include fever, general malaise, chills, headache, nausea, vomiting, and abdominal pain. Sore throat is universal. However, there is rarely a related rhinorrhea or nasal discharge as seen with viral pharyngitis. As a result, post nasal drip does not occur and there is rarely a cough due to irritation. Conjunctivitis is also not present. Significant erythema and swelling of the pharynx occurs and patchy exudates may be apparent on physical exam. Diagnosis must be confirmed by a rapid antigen detection test (RADT), and further compared to a throat culture. A throat culture also serves to minimize the chance of a false negative from the RADT; the RADT is useful because immediate results can be obtained and treatment started promptly. There is no need for serum antibody tests with routine bacterial pharyngitis.

TREATMENT Treatment of bacterial pharyngitis from GAS is with penicillin for ten days. Supportive therapy, as mentioned above for viral pharyngitis, may also be necessary. Delaying treatment may lead to RF; all diagnosed cases should be treated immediately. TABLE 340 BACTERIAL PHARYNGITIS Bacterial Pharyngitis Etiology

GAS

Presentation

Fever, general malaise, chills, headache, nausea, vomiting, and abdominal pain. Sore throat is universal. Significant erythema and swelling of the pharynx occurs and patchy exudates may be apparent.

Diagnosis

RADT and throat culture.

Treatment

Penicillin for ten days. www.ClinicalReview.com


Respiratory System

267

ACUTE AND CHRONIC COUGH ASSESSMENT AND MANAGEMENT Acute cough is present for less than 3 weeks and is most commonly due to postnasal drip. Chronic cough lasts more than 3 weeks and is commonly due to postnasal drip, asthma, GERD, bronchitis, bronchiectasis, pertussis, cancer, ACE inhibitors, and psychogenic causes. Treatment is to cure the underlying etiology. TABLE 341 ACUTE AND CHRONIC COUGH Acute and Chronic Cough Acute

Present for less than 3 weeks and is most commonly due to postnasal drip.

Chronic

Lasts more than 3 weeks and is commonly due to postnasal drip, asthma, GERD, bronchitis, bronchiectasis, pertussis, cancer, ACE inhibitors, and psychogenic causes.

Treatment

Cure underlying etiology.

SINUSITIS ASSESSMENT AND MANAGEMENT Sinusitis is a common cause of postnasal drip and cough, and is most commonly due to bacterial URI with maxillary sinus obstruction, long-standing inflammation, allergic reactions, foreign bodies, cystic fibrosis, and asthma. Sinusitis presents with nasal discharge, fever, and tenderness with external palpation of the sinuses. CT is sensitive and is used only in severe cases. Treatment of bacterial sinusitis is with amoxicillin or TMP-SMX. Complications include extension into nearby anatomic structures leading to neurologic sequelae or ophthalmologic complications. Additional discussion of sinusitis and other related ear, nose, and throat disorders can be found in the Clinical Review of Otolaryngology. TABLE 342 SINUSITIS Sinusitis Etiology

Bacterial URI with maxillary sinus obstruction, long-standing inflammation, allergic reactions, foreign bodies, cystic fibrosis, and asthma.

Presentation

Nasal discharge, fever, and tenderness with external palpation of the sinuses.

Diagnosis

CT in severe cases.

Treatment

Amoxicillin or TMP-SMX.

LOWER RESPIRATORY INFECTIONS PERTUSSIS ETIOLOGY AND PATHOPHYSIOLOGY Pertussis is the development of whooping cough in children, and a common cause of cough in adults due to Bordetella pertussis. Although pertussis is increasingly rare in children, due to aggressive vaccination, it is common in adults and teenagers as the

Â


USMLE STEP 2 268 vaccination wanes. Vaccines are given to children every two months for the first six months of life, then repeated at 18 months and between 4-6 years.

PRESENTATION AND DIAGNOSIS Pertussis begins in the catarrhal stage that lasts between 1 and 2 weeks. This stage is hallmarked by a mild URI. The paroxysmal stage lasts up to a month and includes a prolonged cough that worsens at night. The convalescent stage has gradual improvement. Diagnosis of pertussis infection is made by nasopharyngeal swabs and culture. Pertussis is a reportable infection in many states, and the public health department must be informed.

TREATMENT Treatment of pertussis includes erythromycin, which is effective if started within a few days of illness identification. If erythromycin is no longer an option, supportive therapy is recommended. Infected adults should be kept away from young children who have not yet had the vaccination. TABLE 343 PERTUSSIS Sinusitis Etiology

Bacterial URI with maxillary sinus obstruction, long-standing inflammation, allergic reactions, foreign bodies, cystic fibrosis, and asthma.

Presentation

Nasal discharge, fever, and tenderness with external palpation of the sinuses.

Diagnosis

CT in severe cases.

Treatment

Amoxicillin or TMP-SMX.

INFLUENZA EPIDEMIOLOGY Influenza is the result of a viral infection that can cause significant morbidity and mortality, especially at the extremes of age. Epidemics due to infection by the influenza A and B viruses tend to occur in the winter months, but the extent of the morbidity and mortality often depends on the particular makeup of the viral architecture. Infection often affects millions in an epidemic, and tens of thousands may die. By gender, pregnant women late in the pregnancy are at the highest risk of developing complications. Extremes of age are also a risk factor for morbidity – the elderly and the very young are the most susceptible. Flu pandemics are rare, and occur a few times every century. Recent major pandemics occurred between 1918-1919, and again in 1957. These “superinfections” affected millions of people with higher rates of morbidity and mortality due to a change in the architecture of the capsid of the influenza virus.

ETIOLOGY Influenza is a viral illness transmitted by the family of viruses known as orthomyxoviridae (single-stranded RNA viruses). Influenza types A, B, and C have been identified and they are structurally and biologically similar to each other. However, their precise genetic makeup varies. Influenza is distinct from viral infections that can cause the common cold (these viruses include rhinovirus, coronavirus, respiratory syncytial virus, and others).

www.ClinicalReview.com


Respiratory System

269

PATHOPHYSIOLOGY Influenza A and B are the most common viral subtypes that cause disease in humans. Influenza A has also been found to infect animals, and transmission from an animal host to humans may lead to outbreak of pandemics. In addition to humans, influenza A can affect birds, horses, pigs, seals, and other animals. Influenza A contains ten distinct proteins that compose its outer viral coat, and eight internal single stranded RNA segments. Influenza B differs by having eleven outer viral coat proteins. The surface proteins responsible for antigen recognition by the body include hemagglutinin and neuraminidase, and particular variations in influenza viruses are characterized by the composition and types of these surface proteins. A particularly subtype of influenza A virus, known as avian influenza, containing hemagglutinin type 5 and neuraminidase type 1, is responsible for the death of millions of birds in southeast Asia. It has resulted in the death of several dozen human beings, and currently appears to be transmissible from animal to human, but not between humans. An avian influenza of subtype H9N2 has also been described.

PRESENTATION Influenza presents as an abrupt onset of fever up to 104o, severe sore throat, myalgia, headache, rhinitis, weakness, and in some, an acute encephalopathy. Constitutional symptoms are severe and debilitating. Hypoxia and fever can lead to a tachycardia, and pharyngitis may be severe enough to lead to dyspnea and odynophagia. Nasal discharge is typically minimal or nonexistent. Wheezing and ronchi may be apparent on physical exam. It should be noted that influenza may present with only minimal symptoms in some patients.

DIAGNOSIS Viral cultures can be done to identify the particular subtype of influenza infecting the patient, but such tests hold a more academic interest, as nearly a week may pass before the results are available. Direct immunofluorescence may be used along with serologic cultures to more rapidly identify the particular subtype of influenza. Several rapid tests are also available, but they are expensive and difficult to obtain. CXR should be done in high-risk patients to rule out concomitant pneumonia.

TREATMENT Influenza is treated by avoiding the disease altogether – all high-risk individuals should receive the annual flu vaccine. Amantadine and rimantadine are effective against influenza A, but viral resistance can rapidly develop against these two medications. Oseltamivir and Zanamivir can be used to reduce the duration of illness, but as with amantadine and rimantadine, these agents should be given early in disease. Supportive therapy is necessary in more severe disease.

PNEUMONIA ETIOLOGY AND PATHOPHYSIOLOGY Pneumonia is commonly the result of infection by community-acquired sources such as Streptococcus pneumoniae or Haemophilus influenzae, or hospital acquired sources such as Pseudomonas aeruginosa, Staphylococcus aureus, or enteric organisms. Atypical sources of pneumonia include Chlamydia pneumoniae, Legionella pneumoniae, and Mycoplasma pneumoniae. A common cause of pneumonia in alcoholics, the elderly, and with blood products includes Klebsiella, especially if currant jelly sputum is present. Patients with chronic bronchitis are likely to be infected by Haemophilus influenzae. Immunocompromised (IC) patients are likely to be infected by Pneumocystis carinii pneumonia (PCP). Streptococcus pneumoniae presents with rust-colored sputum. It is also the likely cause of a complicated pneumonia arising after influenza.

Â


270

USMLE STEP 2

ETIOLOGY IN IMMUNOCOMPROMISED STATES HIV patients and others with significant immunocompromise are likely to be infected by a number of atypical sources. With CD4 counts above 200, Mycobacterium tuberculosis should be suspected. CD4 counts dropping below 200 should begin a search for, and prophylactic treatment against Pneumocystis carinii, Histoplasma capsulatum, and Cryptococcus neoformans. If the CD4 count continues to drop below 50, cytomegalovirus and MAC should be suspected. Neutropenic hosts should be suspected of having Pseudomonas aeruginosa or enteric bacteria. Those with splenectomy may be infected with encapsulated bacteria. Patients who abuse steroids may be infected by Mycobacterium tuberculosis or Nocardia. Alcoholics, in addition to being infected with Klebsiella, may also have Streptococcus pneumoniae or Haemophilus influenzae.

PRESENTATION AND DIAGNOSIS Pneumonia generally presents with fever, productive cough, and pleuritic chest pain. Pneumonia is equally likely to present with atypical symptoms, such as dry cough, constitutional symptoms, and generalized GI symptoms. On physical exam, rales and tactile fremitus are typically present. Pectoriloquy is also present with egophony. Diagnosis is made by telltale CXR signs including a lobular or segmental infiltrate (upper lobe if TB or Klebsiella), cavities (small in TB, large in many others), and bilateral infiltrates (in PCP and TB). Gram stains are done on sputum samples. Encapsulated organisms may be detected, including Streptococcus pneumoniae, Haemophilus influenzae, Klebsiella, and Neisseria meningitidis. An idiopathic eosinophilic pneumonia can occasionally occur, known as Loeffler pneumonia. Elevated LDH is indicative of PCP. No bacteria on smear may be indicative of Legionella and Mycoplasma spp.

TREATMENT Inpatient treatment is necessary for patients at extremes of age, residents of nursing homes (NHs), those with chronic conditions, any change in mental status, hypotension, tachypnea, tachycardia, a PaO2 < 60, and those with pleural effusions. Treatment consists of penicillins, cephalosporins, and quinolones. Atypical pneumonias get coverage with erythromycin; hospital-acquired pneumonias are covered for P. aeruginosa; and pneumonia in IC patients gets PCP coverage. TABLE 344 PNEUMONIA Pneumonia Etiology

Commonly the result Streptococcus pneumoniae or Haemophilus influenzae, or hospital acquired sources.

Presentation

Fever, productive cough, and pleuritic chest pain. It is equally likely to present with dry cough, constitutional symptoms, and generalized GI symptoms.

Diagnosis

CXR signs including a lobular or segmental infiltrate, cavities, and bilateral infiltrates. Gram stains are done on sputum samples.

Treatment

Penicillins, cephalosporins, and quinolones. Atypical pneumonias get coverage with erythromycin.

TUBERCULOSIS ETIOLOGY AND PATHOPHYSIOLOGY Tuberculosis is the most common cause of death in the world. It is transmitted by Mycobacterium tuberculosis and the most common manifestation is with pulmonary tuberculosis. The majority of those infected are likely to be free of disease; however, about 5% develop TB within 2 years, and another 5% develop TB sometime in the course of their life. Patients most at risk include IC patients, HIV patients, IV drug users, persons from outside the US, prisoners, the homeless, and those who live in poorer areas www.ClinicalReview.com


Respiratory System

271

of the US. People most likely to develop active TB once infected include those with HIV, IC, IV drug users, and those with abnormal CXR.

PRESENTATION AND DIAGNOSIS TB presents with night sweats, hemoptysis, and constitutional symptoms. A positive PPD strongly raises the clinical suspicion of TB. The presence of an infiltrate on CXR, sputum positive for acid-fast bacilli (AFB), and a positive culture clinch the diagnosis. The PPD is used as a screening test in those at risk, and typically yields a positive reaction within several days. The PPD is read within 2 or 3 days of placement, and the level of induration is measured. A positive result may indicate TB, but it may also be positive due to a vaccination with the Bacillus Calmette-Guérin (BCG) regimen or infection by another Mycobacterium spp. False negatives are common in up to ¼ of all patients; a two step PPD is used in those with a high clinical suspicion of TB. Induration greater than 15 mm is considered positive in all patients. If more than 10 mm, it is considered positive in those at high risk of TB; if it is greater than 5 mm, only those patients with HIV, in close contact with others, and those with abnormal CXR are considered positive.

TREATMENT Prophylaxis of TB is done with INH for twelve months. Prevention with INH is contraindicated for those with hepatic derangements or anyone over 35. Therapy for active TB includes six months of INH, rifampin, pyrazinamide, and ethambutol. Streptomycin may be substituted for ethambutol. Nine months of therapy are used for those patients who cannot use pyrazinamide. After treatment, for about 10 days and three negative AFBs, the patient is no longer considered to be infectious.

COMPLICATIONS Complications of treatment are numerous but generally preventable. INH is a bactericidal agent that can lead to a peripheral neuropathy if not given with pyridoxine. Seizures can occur with overdose. Hepatitis is also possible. Rifampin is another bactericidal agent that can lead to increased LFTs; rifampin discolors contacts and can cause a reddening of the urine, sweat, tears, and stool. Ethambutol is more of a bacteriostatic agent and can lead to loss of color vision and optic neuritis. TABLE 345 TUBERCULOSIS Tuberculosis Etiology

Mycobacterium tuberculosis

Presentation

Night sweats, hemoptysis, and constitutional symptoms.

Diagnosis

A positive culture confirms the diagnosis. Positive PPD. The presence of an infiltrate on CXR, sputum positive AFB.

Treatment

6 months of INH, rifampin, pyrazinamide, and ethambutol. Streptomycin may be substituted for ethambutol. 9 months of therapy are used for those patients who cannot use pyrazinamide.

Complications

Peripheral neuropathy, seizures, hepatitis.

BRONCHITIS ETIOLOGY AND PATHOPHYSIOLOGY Bronchitis is the result of an inflammation from a variety of causes including bacteria, viruses, parasites, or toxic injury from smoking or toxins. Local irritation leads to edema and hyperemia with a boggy mucous membrane. Mucus production increases


USMLE STEP 2 272 significantly and leads to cough. Excess production for several months over a two year period leads to the diagnosis of chronic bronchitis, discussed earlier. Common bacterial causes include Mycoplasma spp., Chlamydia pneumoniae, Streptococcus pneumoniae, Moraxella catarrhalis, and Haemophilus influenzae. Viral infections include influenza, parainfluenza, adenovirus, rhinovirus, and RSV.

PRESENTATION AND DIAGNOSIS Bronchitis presents with constitutional symptoms such as fever and productive cough. Sore throat and rhinorrhea are common. Muscle aches and fatigue are also common. A low-grade fever may also be present. Sputum cultures are often obtained and a plain film performed.

TREATMENT Symptomatic treatment is the key to handling this disease. Antitussive medication such as guaifenesin with dextromethorphan is commonly used. Antibiotics that are occasionally used include telithromycin, erythromycin, clarithromycin, azithromycin, tetracycline, cefditoren, TMP-SMX, amoxicillin, ciprofloxacin, and others. TABLE 346 BRONCHITIS Bronchitis Etiology

Inflammation from a variety of causes including bacteria, viruses, parasites, or toxic injury from smoking or toxins

Presentation

Constitutional symptoms such as fever and productive cough. Sore throat, rhinorrhea, muscle aches and fatigue are also common. A low-grade fever may also be present.

Diagnosis

Sputum cultures are often obtained and a plain film performed

Treatment

Symptomatic treatment. Antitussive medication, antibiotics.

LUNG ABSCESS ASSESSMENT A lung abscess is the formation of an infectious cavity with subsequent pulmonary damage. Infectious sources such as Staphylococcus, Streptococcus, Klebsiella, HIB, Actinomyces, Nocardia, and other sources. Morbidity drastically increases with rupture or developing in high-risk patients, such as those who are immunocompromised. Lung abscess presents with fever, productive cough, night sweats, and weight loss with a foul smelling sputum. Hemoptysis is sometimes present. Blood culture and sputum culture are often collected, and a bronchoscopy may be done. Plain films and CT are also done to precisely identify the location of the defect.

MANAGEMENT Treatment involves clindamycin and specific coverage against identified agents.

www.ClinicalReview.com


Respiratory System

273

TABLE 347 LUNG ABSCESS Lung Abscess Etiology

Formation of an infectious cavity with subsequent pulmonary damage. Sources include Staphylococcus, Streptococcus, Klebsiella, HIB, Actinomyces, Nocardia.

Presentation

Fever, productive cough, night sweats, and weight loss with a foul smelling sputum. Hemoptysis is sometimes present.

Diagnosis

Blood culture, sputum culture and bronchoscopy may be done. Plain films and CT are also done to precisely identify the location of the defect.

Treatment

Clindamycin and specific coverage against identified agents.

OBSTRUCTIVE LUNG DISEASE ASTHMA ETIOLOGY Asthma is reversible airway hyperreactivity that leads to bronchoconstriction, inflammation, and increased secretion. Asthma is typically a lower airway disease with the formation of mucus plugs and edema. It typically occurs intermittently with interposed periods of normal airway, and is most common in children. Asthma usually spontaneously remits over time and 80% of adults who had asthma growing up are symptom-free. About half of all causes of asthma deal with non-allergenic causes, including exposure to cold, certain inhalants, exercise, and anxiety. This is known as intrinsic asthma, and typically heralds a poorer prognosis. On the other hand, extrinsic asthma is due to allergens that increase IgE and cause an immune reaction with increased eosinophils. Also known as atopic asthma, this form of asthma typically has a family history of sensitivity, and presents with rhinorrhea, eczema, and urticaria. Extrinsic asthma has a better prognosis than intrinsic asthma. Asthma is worsened with respiratory irritation as often occurs with infection, aspirin, and B-blockers. The typical individual who has aspirin-induced asthma often meets the criteria for Samter’s triad: nasal polyps, aspirin allergy, and asthma.

PATHOPHYSIOLOGY Changes in the airway with asthma are due to bronchial smooth muscle spasms that lead to constriction of both large and small airways. Over time, hypertrophy of these cells also leads to symptoms. In extrinsic asthma, the release of vasoactive substances by eosinophils leads to edema and secretion of copious amounts of mucus. The result is plugging of the airway and obstruction leading to hypoxia. Such mediators include histamine, bradykinin, various leukotrienes, and various prostaglandins. Mast cells, lymphocytes, and eosinophils have all been implicated as playing a vital role in the inflammatory process. With extrinsic asthma, the early phase occurs with IgE-mediated effects leading to mast cell release of histamine. The late phase occurs with cytokine release.

PRESENTATION The signs and symptoms of an asthma attack are dependent on the severity of the attack. Mild asthma typically produces a transient dyspnea with tachypnea and tachycardia. Diffuse wheezing is sometimes evident. Moderate asthma may have both inspiratory and expiratory wheezing. Severe asthma presents with no wheezing, diminished breath sounds, chest tightness, significant dyspnea, shortness of breath, intercostal retractions, and a dry, nonproductive cough. Factors that herald a poorer prognosis include cyanosis, decreased wheezing, bradycardia, decreased breathing with diminishing breath sounds, pulsus

Â


USMLE STEP 2 274 paradoxus, and diaphoresis. On physical exam, chest wall retractions may be present indicating increased respiratory effort, inability to speak normally, and wheezing may all be obvious.

DIAGNOSIS Asthma is diagnosed by CXR, PFTs, ABG, and careful history and exam. CXR is typically nonspecific and is primarily useful in ruling out other etiologies. PFTs indicate an obstructive pattern in which FEV1 is less than 80% of normal, or the ratio of FEV1: FVC is less than 65%. ABG shows an acute increase in pH and a decrease in PaCO2, while severe asthma may have a decrease in pH, decrease PaO2, and increased PaCO2. Decompensation in the asthmatic may have an ABG that shows a normal PaCO2, often a sign that intubation may be necessary due to impending respiratory failure. Allergen inhalation challenges, skin testing, and radioallergosorbent assay tests (RASTs) may be used to clinch the diagnosis of asthma, although these are not typically performed. A typical ABG in mild asthma is a pH of 7.48, PCO2 of 30, and PO2 of 60 (recall that normal is 7.4, 40, and 98 respectively). Severe asthma presents with a pH of 7.4, PCO2 of 40, and a PO2 of 55.

STAGING AND TREATMENT – MILD INTERMITTENT Asthma staging is important for proper treatment of this potentially fatal disorder. Asthma is graded by severity as mild, moderate, or severe, and by frequency of symptoms as either intermittent, or persistent. Mild intermittent asthma (stage 1) presents with symptoms no more than twice a week with night-time symptoms less than twice a month. FEV1 is 80% or better with a less than 20% variation. Mild intermittent asthma is treated with short-acting bronchodilators and severe exacerbations are treated with systemic glucocorticoids.

STAGING AND TREATMENT – MILD PERSISTENT Mild persistent asthma (stage 2) presents with daytime symptoms several times a week, but not on a daily basis. Nocturnal symptoms occur several times a month. FEV1 continues to be better than 80%, but variability is typically between 20% and 30%. Treatment of mild persistent asthma consists of daily low-dose inhaled glucocorticoids, short-acting bronchodilators as needed, and several alternative therapies that are occasionally prescribed. Inhaled cromolyn, leukotriene modifiers, nedocromil, and sustained-release theophylline are some of these alternatives.

STAGING AND TREATMENT – MODERATE PERSISTENT Stage 3 asthma is moderate persistent with daily symptoms. Nocturnal symptoms occur weekly, and FEV1 is between 60-80% of normal. Variation of FEV1 is more than 30%. Moderate persistent asthma is treated with inhaled glucocorticoids on a daily basis in combination with long-acting bronchodilators. Short-acting bronchodilators are also added to the regimen. Alternative therapies include increasing the dosage of inhaled glucocorticoids, and using a leukotriene modifier or theophylline.

STAGING AND TREATMENT – SEVERE PERSISTENT Stage 4 asthma is severe persistent asthma, also known as continuous asthma. Daytime symptoms are always present, and nocturnal symptoms are common. FEV1 is less than 60% of normal, and variation is more than 30%. Treatment consists of highdose inhaled glucocorticoids and long-acting beta-2 agonists given on a daily basis. Short-acting bronchodilators are given as necessary. Additional medications are added as needed, and include oral glucocorticoids and the regimens mentioned above.

www.ClinicalReview.com


Respiratory System

275

GENERAL TREATMENT Asthma requires prompt treatment to resolve symptoms. The hypoxia is very real and results in over 5,000 deaths a year. Glucocorticoids are the mainstay of treatment and are used to reduce inflammation. Supplemental oxygen, nebulizer treatment, IV fluids, and ventilatory support are all used in asthma treatment. Smoking cessation is important, along with modification of environmental risk factors such as dust, animals, cockroaches, mold, and pollen. Immunotherapy is occasionally used with repeated injections of allergen into the body, and is highly effective. Omalizumab, an antibody to IgE, is also used in certain cases, especially in severe asthma. TABLE 348 ASTHMA Asthma Etiology

Intrinsic asthma- Non-allergenic causes: exposure to cold, certain inhalants, exercise, and anxiety. Extrinsic asthma- Due to allergens that increase IgE and cause an immune reaction with increased eosinophils, has a family history of sensitivity, and presents with rhinorrhea, eczema, and urticaria.

Presentation

Mild asthma - transient dyspnea with tachypnea and tachycardia. Diffuse wheezing is sometimes evident. Moderate asthma -may have both inspiratory and expiratory wheezing. Severe asthma-no wheezing, diminished breath sounds, chest tightness, significant dyspnea, shortness of breath, intercostals retractions, and a dry, nonproductive cough. On physical exam, chest wall retractions may be present indicating increased respiratory effort, inability to speak normally, and wheezing.

Differential diagnosis

Reactive airway disease, CHF, aspiration of a foreign body, pneumoconiosis, COPD, emphysema, pneumonia, PAN, sarcoidosis, tracheal stenosis, sinusitis, pulmonary HTN, lung cancer, and variants of asthma.

Diagnosis

CXR, PFTs, ABG, and careful history and exam. Allergen inhalation challenges, skin testing, and RASTs may be used to clinch the diagnosis of asthma.

Mild intermittent (stage 1)

Treated with short-acting bronchodilators and severe exacerbations are treated with systemic glucocorticoids.

Mild persistent (stage 2)

Daytime symptoms several times a week, but not on a daily basis. Nocturnal symptoms occur several times a month. Daily low-dose inhaled glucocorticoids, short-acting bronchodilators as needed, and several alternative therapies are occasionally prescribed.

Moderate persistent (stage 3)

Daily symptoms. Nocturnal symptoms occur weekly. Inhaled glucocorticoids on a daily basis in combination with longacting bronchodilators. Short-acting bronchodilators are also added to the regimen.

Severe persistent (stage 4)

Daytime symptoms are always present, and nocturnal symptoms are common. High-dose inhaled glucocorticoids and long-acting beta-2 agonists given on a daily basis. Short-acting bronchodilators are given as necessary. Additional medications are added as needed, and include oral glucocorticoids.

General treatment

Glucocorticoids, supplemental oxygen, nebulizer treatment, IV fluids, and ventilatory support. Smoking cessation is important, along with modification of environmental risk factors. Immunotherapy is occasionally used with repeated injections of allergen into the body, and is highly effective, Omalizumab.

Inpatient treatment

Indications for intubation include severe dyspnea, hypoxia, tachypnea, and mental status changes indicating hypoxia. ABG is useful. Overventilation should not be done, and expiratory time should be prolonged.

TREATMENT – MEDICATIONS Beta 2 agonists are administered through nebulizers or metered dose inhalers (MDIs). Ipratropium bromide is delivered by the same mechanism and is used to decrease airway secretions. Albuterol is an example of a short-acting medication, and salmeterol is an example of a long-acting medication. Glucocorticoids are used to decrease inflammation, and can be given by MDI, oral, or IV routes. Side effects of glucocorticoids include inhibited growth, weight gain, HTN, glaucoma, cataracts, diabetes, and osteoporosis. Leukotriene modifiers such as zafirlukast are used to prevent degranulation; another example is the mast cell

Â


USMLE STEP 2 276 stabilizer cromolyn sodium and nedocromil. Theophylline is used only in limited cases due to the number of side effects this medication has, including the ability to cause seizures and arrhythmia.

INPATIENT TREATMENT Asthmatics may require intubation in order to ensure respiratory integrity. Indications for intubation include severe dyspnea, hypoxia, tachypnea, and mental status changes indicating hypoxia. ABG is useful in intubation, in that an ABG with normal PaCO2 in a patient who is laboring to breath (or has suddenly become very quiet) is an indicator for immediate intubation. Overventilation should not be done, and expiratory time should be prolonged.

CHRONIC OBSTRUCTIVE PULMONARY DISEASE (COPD) ETIOLOGY AND PATHOPHYSIOLOGY Chronic obstructive pulmonary disease (COPD) includes emphysema and chronic bronchitis. A long-standing expiratory obstruction is present with decreases in FEV1. COPD is an irreversible airway obstruction and presents with significant anatomical and functional changes. Chronic bronchitis presents with a chronic productive cough for at least three months within a two year period. Emphysema presents with airway enlargement distal to the terminal bronchioles. COPD is more common in men and carries a greater mortality in Caucasians. The most common cause of COPD is smoking, followed by air pollution, infection, allergy, and alpha-1-antitrypsin deficiency. Up to 90% of patients with COPD are smokers. Alpha-1-antitrypsin deficiency is an autosomal recessive (AR) disorder that is more common in patients of Mediterranean descent. Liver abnormalities are commonly present in this disorder. The underlying pathophysiology of COPD is increased airway resistance in bronchitis, and decreased lung recoil in emphysema. Patients with chronic bronchitis are typically known as blue bloaters and present with RHF, polycythemia, and high PCO2 with low O2 on ABG. Patients with emphysema are typically known as pink puffers with a barrel chest, anorexic appearance, and a low PCO2 with normal PO2 on ABG.

TYPES OF EMPHYSEMA Emphysema may present as centrilobular, panlobular / panacinar, or distal acinar emphysema. Centrilobular emphysema is more common with smoking, and affects the respiratory bronchioles. Panacinar emphysema is more common with alpha-1-antitrypsin deficiency, as this enzyme normally protects against degradation of the elastin located in this section of the lung. Distal acinar emphysema is more likely to lead to spontaneous pneumothorax.

PRESENTATION AND DIAGNOSIS COPD diagnosis is made after physical exam, CXR, and PFTs. Chronic bronchitis presents with ronchi and wheezes upon auscultation; emphysema presents with distant breath sounds. CXR indicates pulmonary markings with chronic bronchitis. Emphysematous changes on CXR include lung hyperinflation, flattening of the diaphragm, small heart size, and increased retrosternal space. PFTs indicate an increased TLC and RV, but a reduction in FEV1: FVC and FEF. DLCO is decreased in emphysema. As COPD is irreversible, there is typically little change in FEV1: FVC after administration of bronchodilators. PFT is the diagnostic method of choice for COPD.

TREATMENT COPD is primarily treated with oxygenation via home oxygen support and nighttime oxygen support. The goal is to maintain PaO2 over 60 and hemoglobin saturated at over 90%. Vaccinations against influenza and Streptococcus pneumoniae are mandatory, www.ClinicalReview.com


Respiratory System

277

and antibiotic treatment is given prophylactically against Haemophilus influenzae and Streptococcus pneumoniae to reduce hospitalization and acute symptoms. Beta agonists are used along with ipratropium bromide, and steroids are used with significantly poor FEV1. The first line treatment is composed of steroids, especially in acute exacerbations. Surgical options are limited, but include lung reduction to improve FEV1. Regardless of what treatment is used, smoking cessation is mandatory to slow the progression.

PROGNOSIS Survival in COPD is best predicted by FEV1, with a rapid decline indicative of increased morbidity and mortality. Dyspnea often occurs with exercise (DOE) when FEV1 drops below 50%. FEV1 below 25% often portends dyspnea at rest. TABLE 349 CHRONIC OBSTRUCTIVE PULMONARY DISEASE (COPD) Chronic Obstructive Pulmonary Disease (COPD) Diagnosis

Physical exam, CXR, and PFTs.

Presentation of bronchitis

Ronchi and wheezes upon auscultation, CXR indicates pulmonary markings.

Presentation of emphysema

Distant breath sounds, CXR include lung hyperinflation, flattening of the diaphragm, small heart size, and increased retrosternal space. PFTs indicate an increased TLC and RV, but a reduction in FEV1: FVC and FEF. DLCO is decreased, PFTs indicate an increased TLC and RV, but a reduction in FEV1: FVC and FEF. DLCO.

Diagnosis

PFT, CXR

Treatment

Oxygenation with home oxygen support and nighttime oxygen support. Vaccinations against influenza and Streptococcus pneumoniae are mandatory, and antibiotic treatment is given prophylactically against Haemophilus influenzae and Streptococcus pneumoniae. Beta agonists are used along with ipratropium bromide, and steroids are used with significantly poor FEV1. The first line treatment is composed of steroids, especially in acute exacerbations. Surgical options include lung reduction to improve FEV1. Smoking cessation is mandatory.

BRONCHIECTASIS ETIOLOGY AND PATHOPHYSIOLOGY The anatomic distortion of a conducting bronchi, often with permanent dilation due to elastic and muscular damage to the epithelium, results in bronchiectasis. This obstructive lung disease leads to dilation of the medium-sized bronchi and may be either acquired or genetic. Congenital bronchiectasis presents early in age and is due to a developmental disorder that affects the bronchi. Such disorders include cystic fibrosis (CF), ciliary dyskinesia (as in Kartagener syndrome) with concomitant situs inversus and sinusitis. Acquired disorders include early lung injury such as pneumonia, abscess, tuberculosis and other infections, and chronic lung injury with RSV or bacterial infection. Immunocompromised states leading to Mycobacterium avium complex (MAC) is another cause. Allergic bronchopulmonary aspergillosis, alpha-1-antitrypsin deficiency, autoimmune disease, and toxic gas exposure are rarer causes of bronchiectasis.

PRESENTATION AND DIAGNOSIS Bronchiectasis presents with fever, anorexia, hemoptysis, dyspnea, pleuritic chest pain, wheezing, and significant sputum production and a chronic, productive cough. A history of recurrent infections is typically present in the acquired form, while obvious physical signs of any congenital form are also present in selected patients. On physical exam, crackles, wheezing, and signs of chronic hypoxia are also evident. Diagnosis is made by CXR that shows a tram-track appearance in later stages of the

Â


USMLE STEP 2 278 disease, CT, Polycythemia and elevated WBCs on CBC, sputum analysis, and PFTs. PFTs typically indicate irreversible obstruction. CT is the preferred diagnostic test due to its very high sensitivity. The most common cause of hemoptysis is bronchiectasis.

TREATMENT Symptomatic management of bronchiectasis is the cornerstone of therapy. Smoking cessation is mandatory, along with vaccinations for influenza and pneumonia. Oxygen therapy, as with COPD, is also used as necessary. Antibiotic prophylaxis is used including amoxicillin, tetracycline, TMP-SMX, azithromycin, cephalosporins, and quinolones. Bronchodilators, physical therapy, and drainage of mucus are essential to reducing morbidity and hospitalization; special compressive devices are also available to help expel the mucus. Surgical resection is reserved for severe cases involving poor function or large hemoptysis. Lung transplantation is the ultimate option. TABLE 350 BRONCHIECTASIS Bronchiectasis

Etiology

Congenital bronchiectasis -developmental disorders-CF, ciliary dyskinesia. Acquired disorders- early lung injury such as pneumonia, abscess, tuberculosis and other infections, and chronic lung injury with RSV or bacterial infection. Immunocompromised states leading to MAC is another cause. Allergic bronchopulmonary aspergillosis, alpha-1-antitrypsin deficiency, autoimmune disease, and toxic gas exposure are rarer.

Presentation

Fever, anorexia, hemoptysis, dyspnea, pleuritic chest pain, wheezing, and significant sputum production and a chronic, productive cough. A history of recurrent infections is typically present in the acquired form, while obvious physical signs of any congenital form are also present in selected patients.

Diagnosis

CXR that shows a tram-track appearance in later stages of the disease, CT, Polycythemia and elevated WBCs on CBC, sputum analysis, and PFTs.

Treatment

Smoking cessation, vaccinations for influenza and pneumonia. Oxygen therapy. Antibiotic prophylaxis. Bronchodilators, physical therapy, and drainage of mucus. Surgical resection is reserved for severe cases involving poor function or large hemoptysis. Lung transplantation is the ultimate option.

SLEEP APNEA PATHOPHYSIOLOGY AND TREATMENT Sleep apnea is the abrupt, recurring discontinuation of airflow for more than 10 seconds that occurs multiple times per night in a sleeping patient. Decreased oxygen saturation and increased pulmonary pressure account for the high complication rate of pulmonary HTN in this group of patients. Daytime sleepiness is common due to this sleep abruption, along with HTN and cor pulmonale. Obstructive sleep apnea (OSA) is secondary to poor airway integrity leading to airway collapse; positive pressure ventilation via nasal cannula (CPAP via NC) is necessary. Central sleep apnea is due to decreased respiratory drive by the CNS. Treatment of central sleep apnea is with administration of progesterone, acetazolamide, and oxygen. Diagnosis is made by polysomnography TABLE 351 SLEEP APNEA Sleep Apnea Presentation

Abrupt, recurring discontinuation of airflow for more than 10 seconds that occurs multiple times a night Daytime sleepiness is common, along with HTN and cor pulmonale.

Diagnosis

Polysomnography.

Treatment

CPAP via NC, progesterone, oxygen, acetazolamide. www.ClinicalReview.com


Respiratory System

279

RESTRICTIVE LUNG DISEASE IDIOPATHIC PULMONARY FIBROSIS (IPF) ETIOLOGY AND PATHOPHYSIOLOGY Progressive interstitial lung disease leading to inflammation and lung fibrosis is known as idiopathic pulmonary fibrosis (IPF). In this disease, repeated injury and healing leads to a rampant fibrotic process that destroys healthy lung tissue. The precise cause is unknown, but environmental allergens, various pneumoconioses, and aspiration from GERD have all been implicated.

PRESENTATION AND DIAGNOSIS IPF presents with DOE, a nonproductive cough, and constitutional symptoms. Oxygen-dependence occurs early in the disease, and there is no remission. Progressive illness leads to worsening symptoms with eventual clubbing from hypoxia, pulmonary HTN, and RHF. Diagnosis is made by CT, which indicates pleural effusions, hilar adenopathy, densities, mediastinal lymph nodes, reticular and linear opacities, cystic air spaces, and ground-glass opacities. PFTs indicate a reduction in lung volume, and DLCO is used to determine the extent of the disease. PaO2 with ABGs classically decreases with exertion. A bronchoalveolar lavage (BAL) is occasionally performed. Biopsy is necessary to rule out other modalities. IPF is a diagnosis of exclusion.

TREATMENT Treatment of IPF involves steroids and azathioprine to decrease the progression to full lung fibrosis and respiratory collapse. However, this is a progressive disease and total lung failure is irreversible. TABLE 352 IDIOPATHIC PULMONARY FIBROSIS (IPF) Idiopathic Pulmonary Fibrosis (IPF) Etiology

Repeated injury and healing leading to a rampant fibrotic process that destroys healthy lung tissue.

Presentation

DOE, a nonproductive cough, pulmonary HTN, and RHF.

Diagnosis

CT, PFT, DLCO, biopsy. Diagnosis by exclusion.

Treatment

Steroids, azathioprine.

SARCOIDOSIS ETIOLOGY AND PATHOPHYSIOLOGY Sarcoidosis is a inflammatory disorder that affects multiple systems leading to the development of noncaseating granulomas. T cells are responsible for the development of sarcoidosis, and increased production of various cytokines with T cell propagation to various tissues leads to inflammation, B cell hyperreactivity, and fibrinogenesis. Sarcoid leads to death via respiratory failure and RHF. Sarcoid is especially common in African Americans and young adults.

Â


280

USMLE STEP 2

PRESENTATION AND DIAGNOSIS Sarcoidosis presents with a history of constitutional symptoms, pulmonary symptoms including DOE, chest pain, and cough; however, sarcoidosis can be entirely asymptomatic. Physical exam findings include exertional desaturation, bilateral hilar lymphadenopathy on CXR, and fibrosis. Erythema nodosum occasionally occurs, along with a facial rash, granulomas leading to impingement of the eyesight, and CNS changes. Diagnosis involves increased vitamin D secretion by the granulomas with hypercalcemia, elevated alkaline phosphatase from liver damage, increased ACE, documenting pulmonary changes, and PFTs that indicate decreased DLCO and a restricted pattern. Biopsy is used for confirmation.

TREATMENT Sarcoid is a difficult disease to treat. Steroids are used with some benefit, but increased relapse may occur. Methotrexate, hydroxychloroquine, cyclosporine, azathioprine, and other medications are variably useful. Minimal disease affecting the lungs is typically only observed; treatment is instituted only with chronic disease with changes in oxygenation and PFTs. TABLE 353 SARCOIDOSIS Sarcoidosis Etiology

Repeated injury and healing leading to a rampant fibrotic process that destroys healthy lung tissue.

Presentation

DOE, chest pain, cough, bilateral hilar lymphadenopathy, fibrosis.

Diagnosis

Increased vitamin D secretion with hypercalcemia, elevated alkaline phosphatase, increased ACE, and PFTs. Biopsy is used for confirmation.

Treatment

Steroids, azathioprine, methotrexate, cyclosporine, hydroxychloroquine.

PNEUMOCONIOSIS ETIOLOGY AND PATHOPHYSIOLOGY Pulmonary fibrosis stemming from toxic lung injury is known as pneumoconiosis. Repeated exposure to pulmonary irritants and toxins leads to fibrosis of the lung over many years. The disease begins with macrophages consuming the toxic compounds leading to inflammation. Early bouts of inflammation are typically resolved, but repeated toxic injury leads to rampant inflammation and fibrosis. Common causes include asbestos dust exposure, silica dust exposure, and coal dust exposure.

TREATMENT There is typically no treatment for pneumoconiosis. However, bronchodilators have been used in certain etiologies with some benefit. Standard respiratory treatments (i.e. oxygen therapy, beta-agonists) are somewhat beneficial. TABLE 354 PNEUMOCONIOSIS Pneumoconiosis Etiology

Inflammation and fibrosis.

Presentation

DOE, a nonproductive cough, pulmonary HTN, and RHF.

Diagnosis

CT, PFT, DLCO, biopsy. Diagnosis by exclusion.

Treatment

Standard respiratory treatment, bronchodilators. www.ClinicalReview.com


Respiratory System

281

ASBESTOSIS Asbestosis may occur in mining, milling, shipyard work, and repeated exposure to materials that contain high levels of asbestos (insulation, brake linings). Asbestosis presents with DOE, wheezing, cough, pleuritic chest pain, SOB, excess sputum production, clubbing, and respiratory failure. Hypoxemia is present along with a restrictive PFT. DLCO is decreased. CXR indicates pleural thickening with plaques and calcifications. Pleural effusions are typically present from the inflammation, especially in the lower lung region around the diaphragm. Asbestosis has been linked to bronchogenic adenocarcinoma. Smoking increases this risk by nearly 100 fold. TABLE 355 ASBESTOSIS Asbestosis Etiology

Asbestos

Presentation

DOE, wheezing cough, pleuritic chest pain, SOB, excess sputum, clubbing.

Diagnosis

CXR

Treatment

Standard respiratory treatment, bronchodilators.

SILICOSIS Silica dust exposure may occur in mining, quarrying, glass production, and sandblasting. It presents similar to asbestosis, while significant exposure can lead to rapid respiratory failure. Hyaline nodules are typically present. Silicosis has been associated with tuberculosis, and prophylaxis for this infection is typically mandated. TABLE 356 SILICOSIS Silicosis Etiology

Silica dust

Presentation

Similar to asbestosis, hyaline nodules.

Diagnosis

CXR

Treatment

Standard respiratory treatment, bronchodilators.

COAL MINER’S LUNG Coal dust exposure is especially common in miners. CXR signs include round densities in the upper lung. Coal miner’s lung is associated with increased IgA, IgG, C3, ANA, and RF. Massive progressive fibrosis typically results. TABLE 357 COAL MINER’S LUNG Coal Miner’s Lung Etiology

Coal dust

Presentation

Productive cough with black mucus.

Diagnosis

CXR

Treatment

Standard respiratory treatment, bronchodilators.


282

USMLE STEP 2

FARMER’S LUNG Farmer’s lung is due to exposure to actinomycete spores leading to a hypersensitivity pneumonitis. Repeated exposures lead to fibrosis, while acute exposure presents as a hypersensitivity response that typically remits with steroid use. TABLE 358 FARMER’S LUNG Farmer’s Lung Etiology

Actinomycete spores.

Presentation

Hypersensitivity pneumonitis.

Diagnosis

CXR, remission with steroids.

Treatment

Steroids.

PULMONARY DISEASE ADULT RESPIRATORY DISTRESS SYNDROME (ARDS) ETIOLOGY AND PATHOPHYSIOLOGY Adult respiratory distress syndrome is a severe diffuse alveolar injury that leads to pulmonary infiltrates, hypoxemia, and failure of normal lung function. Increased permeability of the alveoli leads to fluid infiltration into the alveoli and subsequent damage to their sensitive epithelia. Damage occurs primarily to the vascular endothelium or alveolar epithelium, depending on the nature of the infiltrate. Pulmonary edema occurs with damage mostly to type I cells. Longstanding damage leads to hypoxemia, pulmonary HTN, and in more severe cases, fibrosis with permanent and progressive pulmonary damage. Limited acute injury typically resolves.

PRESENTATION AND DIAGNOSIS ARDS presents with acute onset of dyspnea and hypoxemia following an identifiable cause (i.e. drug overdose, sepsis, acute pancreatitis, aspiration). Physical findings include tachypnea, tachycardia, DOE, ARDS: From sepsis, pancreatitis. oxygen supplementation, signs of hypoxia, and signs of the underlying etiology. Leads to bilateral pulmonary ARDS often occurs with sepsis and shock. Diagnosis is made by ABG that indicates infiltrates and PaO2 / FiO2 < respiratory alkalosis or metabolic acidosis, if ARDS occurs due to sepsis; and CXR 200. Treat underlying etiology. that indicates pulmonary infiltrates bilaterally with alveolar filling. ARDS is primarily a clinical diagnosis that is confirmed with various diagnostic tests.

TREATMENT Treatment of ARDS involves treating the underlying etiology. Respiratory support may be necessary. Fluid management is important for certain patients.

www.ClinicalReview.com


Respiratory System

283

TABLE 359 ADULT RESPIRATORY DISTRESS SYNDROME (ARDS) Adult Respiratory Distress Syndrome (ARDS) Etiology

Severe diffuse alveolar injury that leads to pulmonary infiltrates, hypoxemia, and failure of normal lung function.

Presentation

Acute onset of dyspnea and hypoxemia following an identifiable cause. Physical findings include tachypnea, tachycardia, DOE, oxygen supplementation, signs of hypoxia, and signs of the underlying etiology. ARDS often occurs with sepsis and shock.

Diagnosis

ABG that indicates respiratory alkalosis or metabolic acidosis if ARDS occurs due to sepsis; and CXR that indicates pulmonary infiltrates bilaterally with alveolar filling.

Treatment

Treating the underlying etiology. Respiratory support may be necessary. Fluid management is important for certain patients.

ATELECTASIS ASSESSMENT Collapse of a portion of the lung is one of the most common causes of postoperative fever. Poor inspiratory effort in this period, lack of sufficient coughing and lung expansion, and failure to use devices that aid in inspiration and expiration have all been implicated in the development of lung collapse. Atelectasis presents with fever, tachycardia, dyspnea, tachypnea, and hypoxemia. Deviations of the trachea or elevations of the diaphragm may be present on CXR, while more significant atelectasis may have mediastinal shifts.

MANAGEMENT Treatment of atelectasis involves incentive spirometry, inducing cough, and deep breathing in an attempt to reperfuse the affected lung regions with air. Treating any concomitant pulmonary disorders is also necessary, along with removing any obstructions or foreign bodies via bronchoscopy. TABLE 360 ATELECTASIS Atelectasis Causes

Poor inspiratory effort, lack of sufficient coughing and lung expansion, and failure to use devices that aid in inspiration and expiration.

Presentation

Fever, tachycardia, dyspnea, tachypnea, and hypoxemia. Deviations of the trachea or elevations of the diaphragm may be present on CXR, while more significant atelectasis may have mediastinal shifts.

Treatment

Incentive spirometry, inducing cough, and deep breathing. Treating any concomitant pulmonary disorders is also necessary, along with removing any obstructions or foreign bodies via bronchoscopy.

SPONTANEOUS PNEUMOTHORAX ETIOLOGY AND PATHOPHYSIOLOGY Spontaneous pneumothorax is a potentially devastating disorder that leads to air in the pleural cavity from a sudden rupture of a pleural bleb. It is most likely to occur in tall and thin people, or in those who smoke. Other causes include COPD, cystic fibrosis, pneumonia, cancer, and illicit drug abuse.

Â


284

USMLE STEP 2

PRESENTATION AND DIAGNOSIS Spontaneous pneumothorax presents with pleuritic chest pain and decreased breath sounds. Hyperresonance due to increased conduction through the air is present on the affected side. The trachea may deviate towards the side of the pneumothorax. Diagnosis includes upright expiratory CXR, where a collapsed lung may be found. EKG changes are sometimes present in the form of ST changes.

TREATMENT Treatment includes oxygen and observation. If the pneumothorax is significant, air may need to be removed via tube thoracostomy. Pleurodesis may be necessary to seal the hole. TABLE 361 SPONTANEOUS PNEUMOTHORAX Spontaneous Pneumothorax Etiology

COPD, cystic fibrosis, pneumonia, cancer, and illicit drug abuse.

Presentation

Pleuritic chest pain and decreased breath sounds.

Treatment

Oxygen and observation. If the pneumothorax is significant, air may need to be removed via tube thoracostomy. Pleurodesis may be necessary to seal the hole.

TENSION PNEUMOTHORAX ASSESSMENT

Tension pneumothorax: Tracheal deviation away from PTX. Diagnose clinically. Needle decompression followed by chest tube.

Tension pneumothorax occurs with air under pressure within the pleural space, leading to impingement upon the airway and compromise in both ventilation and perfusion of the affected lung region. It presents similarly to spontaneous pneumothorax, with the addition of tracheal deviation away from the side of the pneumothorax, hypotension, and tachycardia. Trauma to the region may also lead to the presence of blood.

MANAGEMENT Tension pneumothorax requires emergency management and is treated with decompression by needle followed by tube thoracostomy. TABLE 362 TENSION PNEUMOTHORAX Tension Pneumothorax Presentation

Similar to spontaneous pneumothorax, with the addition of tracheal deviation away from the side of the pneumothorax, hypotension, and tachycardia. Trauma to the region may also lead to the presence of blood.

Treatment

Decompression by needle followed by tube thoracostomy. www.ClinicalReview.com


Respiratory System

285

PLEURAL EFFUSION ETIOLOGY AND PATHOPHYSIOLOGY Pleural effusions are transudative or exudative fluid collections within the pleural cavity. Common causes of a transudative effusion are from increased hydrostatic pressure or decreased oncotic pressure, which occurs in CHF, cirrhosis, and nephrotic syndrome. Transudative effusions are likely to be due to systemic causes and are bilateral in nature. Pulmonary embolism is another cause of transudative effusion. Exudative transfusions contain cells and are due to local processes such as cancer, infection, and trauma. Exudative effusions tend to be unilateral.

PRESENTATION AND DIAGNOSIS Pleural effusions are diagnosed by thoracentesis and the LDH and protein titers are measured. A LDH greater than 200, a ratio of LDH between the effusion and serum greater than 0.6, and a protein effusion to serum ratio more than 0.5 are indicative of an exudative effusion. Any positive value is indicative of exudative effusion. Transudative effusions have all three numbers below their cut-off values and relative ratios. A parapneumonic effusion is considered if the leukocyte count is greater than 10,000 with a high PMN number; these are always exudative effusions due to the high cellular content. Blood in the effusion brings the differential to trauma, infection, aortic dissection, and malignancy. Low glucose in the effusion leads to the consideration of tuberculosis, empyema, rheumatoid arthritis, and malignancy. Finally, elevated amylase titers require ruling out pancreatitis, renal failure, esophageal rupture, and tumors.

TREATMENT Transudative and exudative effusions should be treated by treating the underlying cause(s). TABLE 363 PLEURAL EFFUSION Pleural Effusion Etiology

Diagnosis

Treatment

Transudative effusion: increased hydrostatic pressure or decreased oncotic pressure, which often occurs in CHF, cirrhosis, nephrotic syndrome, and pulmonary embolism Exudative effusions: Cancer, infection, and trauma, tend to be unilateral. Thoracentesis and the LDH and protein titers are measured as well as a leukocyte count. Check for blood, low glucose, and elevated amylase titers. Elevated amylase titers require ruling out pancreatitis, renal failure, esophageal rupture, and tumors. Transudative effusion and exudative effusions should be dealt with by treating the underlying cause(s).

PULMONARY EMBOLISM (PE) ETIOLOGY Pulmonary embolism is a challenging diagnosis that must be made to avoid a potentially lethal outcome. PE is commonly due to venous stasis, intimal injury, and hypercoagulability (Virchow’s triad), and often occurs in deep veins of the lower extremities. Motion of the deep vein thrombus (DVT) leads to travel through the inferior vena cava and eventually into the right atrium, followed by the right ventricle, then through the pulmonary artery into the pulmonary circulation. The larger the embolus, the more consequences of the blockade that leads to ischemia. Large emboli may become lodged at the bifurcation of the pulmonary

Â


USMLE STEP 2 286 artery or major branches thereof, leading to a saddle embolus that leads to significant ischemia throughout that region of the lung and concomitant respiratory failure. Other causes of PE include use of oral contraceptives, cancer, thrombophilias (including factor V Leiden, antithrombin III (ATIII) deficiency), protein C (PrC) deficiency, protein S (PrS) deficiency, and antiphospholipid antibody (APA). The majority of PEs is due to DVTs.

PATHOPHYSIOLOGY Respiratory effects of PE include alveolar necrosis, hypoxemia, and hyperventilation. Pulmonary infarction often occurs with untreated PE, and surfactant loss in the affected lung region often occurs, leading to a loss of lung integrity. Arterial hypoxemia is a common finding due to ventilation-perfusion (V/Q) mismatch, formation of shunts, decreased cardiac output from increased pulmonary circulation pressure, and patency of a foramen ovale (PFO). Infarction of the lung is rare due to collateral circulation from the bronchial arteries. Increased pulmonary vascular resistance leads to increased RV afterload with subsequent RVH. Sudden increases in load can lead to sudden cardiac death from excessive RV load. Hemodynamic collapse is the effect of large PEs.

PRESENTATION A large PE presents with circulatory collapse and death, and is a common cause of death in hospitalized postoperative patients, especially in the elderly. Smaller PEs can present with pulmonary infarction with pleuritic chest pain and hemoptysis. SOB, DOE, pallor, hypotension, and numerous atypical symptoms such as seizures, syncope, abdominal pain, wheezing, CNS changes, atrial fibrillation, and other signs and symptoms complicate the picture. The most common signs and symptoms include tachypnea, rales, tachycardia, S4 gallop, and accentuated P2 heart sound.

DIAGNOSIS PE diagnosis involves a host of exams and imaging studies with variable sensitivity and specificity. The diagnostic testing begins after a thorough history and physical exam are completed. ABGs are drawn, and may indicate hypoxemia, hypocapnia, and alkalosis with a decreased A-a gradient. D-dimer is a nonspecific test that indicates fibrin breakdown; it should not be used alone for diagnosis but raises the clinical suspicion if positive. CXR is normal in the acute phase, but later shows dilation of pulmonary vessels, atelectasis, pleural effusions, and an elevated diaphragm (Westermark sign). V/Q scanning provides a meaningful diagnosis in many cases; segmental perfusion defects are highly indicative of PE but many patients fall in the intermediate probability category. Additional testing is warranted, and includes ultrasound to detect a DVT, helical CT with contrast to identify a significant pulmonary embolus (smaller ones are not detected), and pulmonary angiography (the gold standard). MRI and echocardiography have been used with mixed results. Signs of a PE on EKG include ST-T wave changes, right-axis deviation, S waves in lead 1, Q waves in lead 3, and inverted T waves in lead 3 (S1-Q3-T3).

TREATMENT Any patient with a high or medium clinical suspicion with a high-probability V/Q scan, or those with confirmed results should be treated for PE. Anticoagulation therapy starting with heparin followed by warfarin is used; treatment is typically continued for at least six months. LMWH can also be substituted, especially in pregnant patients. Thrombolytic therapy is often used in hemodynamically unstable patients and must be given shortly after the event to achieve maximal effectiveness. An inferior vena cava filter is placed in patients with contraindications to anticoagulation, fragile patients who cannot tolerate another PE, and those with risk of recurrence.

www.ClinicalReview.com


Respiratory System

287

TABLE 364 PULMONARY EMBOLISM (PE) Pulmonary Embolism (PE) Etiology

Commonly due to venous stasis, intimal injury, and hypercoagulability, and often occurs in deep veins of the lower extremities. Use of oral contraceptives, cancer, thrombophilias, ATIII deficiency, PrC deficiency, PrS deficiency, and APA.

Presentation

A large PE presents with circulatory collapse and death. Smaller PEs can present with pulmonary infarction with pleuritic chest pain and hemoptysis. SOB, DOE, pallor, and hypotension. The most common signs and symptoms include tachypnea, rales, tachycardia, S4 gallop, and accentuated P2 heart sound.

Diagnosis

ABG, D-dimer test, CXR, V/Q scanning, segmental perfusion defects, ultrasound, helical CT, pulmonary angiography, EKG.

Treatment

Anticoagulation therapy for at least six months. Thrombolytic therapy must be given shortly after the event to achieve maximal effectiveness. An IVC filter is placed in patients with contraindications to anticoagulation.

LUNG CANCER EPIDEMIOLOGY The leading cause of cancer-related death is due to lung cancer, and is the number one killer in both men and women. Over 160,000 people die annually in the United States, surpassing the total number of deaths from breast cancer, prostate cancer, and colorectal cancer. Lung cancer has very high morbidity and mortality with only a fraction of patients surviving after five years. By the time tumors are found, malignant metastasis has already occurred in most people with rapid spread through lymph nodes and blood to the liver, adrenal glands, bones, and brain. The vast majority of lung cancer can be traced back to a longstanding history of smoking, and a history of smoking invariably worsens the prognosis of lung cancer. About 95% of all lung cancers are either small cell lung cancer (SCLC) or non-small cell lung cancer (NSCLC). Rarer varieties include carcinoid, lymphoma, metastatic cancers to the lung, and others. Of the NSCLCs, adenocarcinoma (adenoCA) and squamous cell carcinoma (SCC) are the most common, each making up 30% of all lung cancers. SCLC makes up another 30%.

ETIOLOGY Without question, the most significant positive predictor of lung cancer risk is a history of smoking. Further, the number of cigarettes smoked, the number of years of smoking, and the age that smoking started are specific positive predictors of risk. Other causes make up about 15% of all lung cancers, and include secondhand smoke, air pollution, asbestosis, chronic lung disease (including TB and COPD), radon exposure, exposure to heavy metals, and a prior history of lung cancer.

PRESENTATION Lung cancer presents without symptoms in up to a quarter of patients at the time of diagnosis. The remainder develop a new, progressive cough that is sometimes blood-tinged, pleuritic chest pain, SOB, wheezing, hoarseness, recurrent URIs, and systemic effects based on metastasis. Paraneoplastic

Â


USMLE STEP 2 288 syndromes can also occur with production of gastrin, ACTH, ADH, calcitonin, ANF, and PTHrP. The result can be clubbing, metastatic ossification, anemia, weakness, constitution symptoms, neural degeneration, anorexia, and hyponatremia.

DIAGNOSIS Lung cancer is diagnosed by a number of exams. A careful smoking history and complete physical exam are necessary. CXR is typically the first test, but abnormalities may include only small nodules in early cases – they may even be entirely negative due to the low sensitivity of this exam. CT and MRI are more sensitive and can detect smaller nodules, and are also useful for staging. Biopsy, sputum testing, and bronchoscopy are other methods of obtaining cells and tissue for additional analysis. Collection of fluid via thoracentesis, or open procedures such as thoracotomy is necessary to collect an adequate tissue sample. Local spread is often gauged by a mediastinoscopy.

FEATURES AND TREATMENT OF SCLC

SCLC = gastrin, ACTH, ADH, ANF, calcitonin NSCLC = PTHrP

The treatment of SCLC differs compared to NSCLC. SCLC tends to be more aggressive and rapidly growing. Histology often indicates dark nuclei with little cytoplasm. SCLC produces gastrin, ACTH, ADH, ANF, and calcitonin, which can lead to a number of secondary effects. Treatment of SCLC involves radiotherapy and chemotherapy, as the majority of these tumors respond well to this modality. Surgical resection is typically not possible. SCLC has a low 5 year survival.

FEATURES AND TREATMENT OF NSCLC NSCLC features cells with pleomorphic nuclei and large amounts of cytoplasm. NSCLC produces PTHrP. This type of tumor is less responsive to radiotherapy, and barely responsive to chemotherapy. However, surgical resection is an option in cases with limited spread. Five year survival is better for NSCLC than it is for SCLC.

PROGNOSIS Palliative care is often the final outcome of therapy when dealing with lung cancer. This is due to the high rate of recurrence, in which many lung cancers are present again within 2 years. Pain management with hospice care involves the use of high dose opioids and NSAIDs as indicated. Oxygen administration is often the key to patient comfort. The primary method of avoiding the lethal pitfalls of lung cancer is avoidance – smoking cessation is the first and most important step to reducing the risk of lung cancer. Survival with stage I NSCLC have a 70% 5 year survival; stage II offers a 30% survival, stage III offers a 20% survival, and higher stages offer only a 9 month survival period. SCLC that is treated with chemotherapy has a 10% 5 year survival; advanced SCLC has a 6 month survival period.

SCREENING Some institutions offer CT scans as a screening test in high risk patients; early data appears to indicate that CT scans can detect lung cancer earlier, and in some patients, this may lead to earlier treatments and potential surgical resection before significant metastasis occurs. Whether this translates into less morbidity and mortality over a 5 year period remains to be determined.

COMPLICATIONS Lung cancer is associated with numerous syndromes, including superior vena cava syndrome, Horner syndrome, Pancoast tumor, SIADH, Eaton-Lambert syndrome, and Trousseau syndrome. Superior vena cava syndrome occurs due to compression of the SVC www.ClinicalReview.com


Respiratory System

289

with swelling of the upper extremity, head, and neck. Cough, headache, epistaxis, and syncope are commonly associated symptoms. Horner syndrome presents with paralysis of the sympathetic nerve due to damage to the ganglion; it presents with ptosis, enophthalmos, miosis, and anhidrosis. Pancoast tumor presents with damage to the 8th cervical nerve, 1st and 2nd thoracic nerve, and damage to the ribs that leads to pain that radiates to the ipsilateral arm. SIADH presents with hyposmolality and hyponatremia. Eaton-Lambert syndrome occurs with an autoimmune reaction to nerve terminals leading to decreased release of acetylcholine (ACh). Trousseau syndrome is a hypercoagulable state that leads to venous thrombosis. TABLE 365 LUNG CANCER Lung Cancer Etiology

Smoking, smoke, air pollution, asbestosis, chronic lung disease, TB and COPD, radon exposure, exposure to heavy metals, and a prior history of lung CA.

Presentation

Without symptoms in up to a quarter of patients at the time of diagnosis. The remainder develop a new, progressive cough that is sometimes blood-tinged, pleuritic chest pain, SOB, wheezing, hoarseness, recurrent URIs, and systemic effects based on metastasis.

Diagnosis

Physical exam, CXR is typically the first test, CT and MRI, biopsy, sputum testing, and bronchoscopy. Thoracentesis, or thoracotomy, mediastinoscopy.

Treatment of SCLC

Radiotherapy, chemotherapy.

Treatment of NSCLC

Surgical resection.

Palliative care

Hospice care involves the use of high dose opioids and NSAIDs as indicated. Oxygen administration.

MEDIASTINAL DISEASE MEDIASTINITIS ASSESSMENT Inflammation of the mediastinum may be due to rupture of the esophagus, anthrax infection leading to hemorrhage (nearly 100% fatal), and infection by TB or histoplasma. Mediastinitis presents with constitutional symptoms followed by hypoxia, dyspnea, hemorrhage, and death.

MANAGEMENT Mediastinitis requires prompt medical attention. The underlying etiology is rapidly treated with antibiotics, including penicillins, fluoroquinolones, and tetracyclines and drainage as necessary. TABLE 366 MEDIASTINITIS Mediastinitis Etiology

Rupture of esophagus, anthrax, TB, histoplasma.

Presentation

Hypoxia, dyspnea, hemorrhage.

Diagnosis

CT, CXR

Treatment

Antibiotics, drainage

Â


USMLE STEP 2

290

PNEUMOMEDIASTINUM (MEDIASTINAL EMPHYSEMA) ASSESSMENT Pneumomediastinum is the result of air in the mediastinum, commonly due to rupture of the esophagus, alveolar rupture, tracheal damage, or dissection of the neck or abdomen with an air leak. It is treated by reversing the underlying etiology.

DIAGNOSIS This diseases is often found in asthmatics, those with DKA, and those with pernicious vomiting. Hammon’s sign is present in 50% of the cases. Confirmation is by radiography demonstrating gas within the mediastinal tissues. TABLE 367 PNEUMOMEDIASTINUM Pneumomediastinum (Mediastinal Emphysema) Etiology

Air in mediastinum (ruptured alveoli or esophagus, tracheal damage, surgical air leaks).

Presentation

Intense chest pain that worsens with movement

Diagnosis

Found in asthmatics, those with DKA, and pernicious vomiting. Hammon’s sign is present in 50% of cases. Confirmation by radiography demonstrating gas within the mediastinal tissues.

Treatment

Reverse underlying cause.

PRACTICE QUESTIONS A 28 year old male involved in a knife fight presents to the ER with several penetrating stab wounds to his right chest. Two chest tubes are inserted and his tension pneumothorax resolves. Over the next 4 hours, the patient has a chest tube output of 300 cc / hr. What is the next step in management? A. B. C. D. E.

Chest CT Exploratory laparotomy Observation Pulmonary resection Thoracotomy

The best answer is Thoracotomy. This patient will go to the operating room for an immediate thoracotomy. Indications following placement of a chest tube include a spontaneous output of over 1500 cc upon placement, or an output rate of 200 cc / hr over 4 hours. Any loss of vital signs or tamponade on FAST will lead to an ER thoracotomy. Left thoracoabdominal penetrating injuries require laparoscopy to exclude damage to abdominal organs. The best step in management of any questionable patient is immediate exploratory laparotomy of the abdominal cavity in hypotensive patients without other obvious primary causes.

www.ClinicalReview.com


Respiratory System

291 A 47 year old male who has had some difficulty with ventilation has an increase in his PEEP to 15. A transient decrease in urine output is noticed. Which of the following is the next step in management? A. B. C. D. E.

Aggressive fluid boluses Decrease PEEP as tolerated FENA Observation Renal ultrasound

The best answer is Decrease PEEP as tolerated. This patient has an increase in thoracic pressure from the increase in PEEP, leading to release of renin and oliguria. Treatment is to decrease PEEP as tolerated. Fluid boluses are not necessary, and observation is not the initial best step in management. PEEP serves to avoid collapse of functional alveoli, thereby increasing recruitment and FRC / VC. PaO2 thereby increases. PEEP should be avoided in COPD due to the risk of puncture. Of the alveoli, type I cells are squamous and type II are glandular. The right upper lobe has the highest oxygen concentration, and so is also the site of most bacterial abscesses.

An 18 year old male has a difficult time with sports due to signs and symptoms of thoracic outlet obstruction. Hypertrophy of the anterior scalene muscle is deemed to be the culprit and a resection is completed. Which of the following describes the anatomy around this muscle? A. B. C. D. E.

Lies anterior to the subclavian artery and brachial plexus, and anterior to the subclavian vein Lies anterior to the subclavian artery and brachial plexus, and posterior to the subclavian vein Lies lateral to the subclavian artery and brachial plexus, and medial to the subclavian vein Lies posterior to the subclavian artery and brachial plexus, and posterior to the subclavian vein Lies posterior to the subclavian artery and brachial plexus, and anterior to the subclavian vein

The best answer is Lies anterior to the subclavian artery and brachial plexus, and posterior to the subclavian vein. The anterior scalene muscle, also known as the scalenus anticus, lies deeply at the side of the neck, behind the sternocleidomastoideus. It arises from the anterior tubercles of the transverse processes of the third, fourth, fifth, and sixth cervical vertebra, and descending, almost vertically, is inserted by a narrow, flat tendon into the scalene tubercle on the inner border of the first rib, and into the ridge on the upper surface of the rib in front of the subclavian groove. It lies anterior to the subclavian artery and brachial plexus, posterior to the subclavian vein, and may lead to thoracic outlet obstruction through compression of the brachial plexus between the anterior and middle scalene muscles when it hypertrophies.

Â


USMLE STEP 2

292

A 28 year old male presents to clinic with a small, firm, but mobile anterior neck mass. What is the next best step in management? A. B. C. D. E.

Excisional biopsy Fine needle aspiration Lymph node dissection Observation Open lymph node biopsy

The best answer is Fine needle aspiration. All anterior neck masses initially undergo a fine needle aspiration as part of their workup. An excisional biopsy can be completed if additional testing is required. An open biopsy is not recommended due to the need to preserve lymph node architecture for possible future interventions. A fine needle aspiration is upwards of 90% sensitive for tumors – the most common being squamous cell carcinoma and lung cancer. Of note, a thyroglossal duct cyst is completely excised along with the hyoid bone. It presents as a mobile midline mass that is attached to the tongue. The presence of a buccal squamous cell carcinoma receives a wide resection with a selective neck dissection.

A 32 year old female presents with her third complication due to a pulmonary embolism. She has a long history of deep vein thromboses. She initially started subcutaneous heparin therapy to avoid these thromboembolic phenomenon, but has apparently failed it. What is the next step in management? A. B. C. D. E.

Coumadin and thrombectomy IVC filter IVC filter and coumadin IVC filter and heparin Lifelong coumadin

The best answer is IVC filter. Intracaval filters are the therapy of choice for recurrent pulmonary embolisms due to deep vein thrombosis within the lower extremities. Multisystem trauma, hemorrhagic cerebrovascular accidents, and failed anticoagulation therapy are also indications. The standard therapy otherwise is a heparin bridge to coumadin with a target INR of 2-2.5. This particular patient would receive an IVC filter only. There are some indications in the literature that the use of an IVC filter with coumadin or heparin may be even more beneficial.

www.ClinicalReview.com


Respiratory System

293

CHAPTER CONTENTS Basic Science .............................. 294 Studies and Procedures ............. 297 Congenital Heart Defects ........... 301 Coronary Heart Disease ............. 302 Congestive Heart Failure............ 313 Valvular Heart Disease ............... 317 Cardiomyopathy ........................ 325 Myocarditis ................................ 329

Pericardial Disease .......................... 330 Endocardial Disease......................... 333 Arrhythmia....................................... 336 Aortic Disease .................................. 342 Common Cardiovascular Malformations ................................. 344 Vascular Disorders ........................... 345 Practice Questions ........................... 351

Â


294

USMLE STEP 2

CARDIOVASCULAR SYSTEM CARDIOVASCULAR SYSTEM BASIC SCIENCE ANATOMY HEART The heart is made of four chambers, including two atria and two ventricles. The flow that occurs throughout the pulmonary vasculature and systemic vasculature is due to the action of the muscular ventricles, while the atria serve primarily to store blood during diastole. One atrium works in conjunction with a ventricle, and the two systems are separated by a septum. The atrioventricular valves control flow of blood from the atria to the ventricles. The right atrium and ventricle are separated by the tricuspid valve, and the left atrium and ventricle are separated by the bicuspid (mitral) valve. The SA node is located at the junction of the right atrium and superior vena cava.

VASCULATURE The internal mammary artery is the first branch off the subclavian artery. The left internal mammary artery has the best patency for a CABG. The subclavian artery is located between the anterior and middle scalene muscles The blood supply to the heart consists of the coronary arteries. The left coronary artery (LCA) supplies the left side of the heart, and the left anterior descending (LAD) branch of the LCA supplies the anterior wall of the left ventricle and anterior 2/3’s of the interventricular septum of the heart. The left circumflex (LCX) branch supplies the lateral wall of the left ventricle and posterior portions of the heart. The LCA arises distal to the posterior aortic sinus of Valsalva. The right coronary artery (RCA) supplies the inferior wall of the left ventricle and the SA and AV nodes of the heart. The RCA also gives off the marginal artery, which supplies the right atrium and right ventricle. The RCA arises directly from the aorta immediately distal to the anterior aortic sinus of Valsalva. SA node AV node RA RV LA Anterior LV Lateral LV Posterior LV Anterior septum Posterior septum Apex

RCA via sinus node branch RCA via sinus node branch RCA via sinus node branch and atrial branch RCA via acute marginal and RV branches LCA via circumflex; RCA and LCA directly LCA via LAD LCA via diagonal branches LCA via obtuse marginal, posterolateral, and posterior descending via RCA LCA via LAD and septal branches RCA via posterior descending (right dominant) LCA via LAD and IV branches

www.ClinicalReview.com


Cardiovascular System

295

THORACIC DUCT The thoracic duct terminates at the junction of the left subclavian vein and the left internal jugular vein.

PHYSIOLOGY HEMODYNAMICS

EF =

( EDV − ESV ) SV = EDV EDV CO = HR × SV The compensatory reaction for mild, normovolemic anemia is via increased cardiac output. Mean arterial pressure is 2/3s of the diastolic pressure plus 1/3 of the systolic pressure. MAPs should be kept above 60. Components of MAP are cardiac output and peripheral vascular resistance. Oxygen delivery is affected by cardiac output, hemoglobin, and oxygen saturation. Per Starling’s rule, cardiac output increases with left ventricular end diastolic volume. Left heart valve disorders lead to distortions in pulmonary capillary wedge pressure. A significant discordance in PCWP and central venous pressure indicate right heart failure. Myocardial oxygen consumption is proportional to ventricular wall tension.

PULMONARY ARTERY CATHETER A pulmonary artery catheter (PAC, also known as a Swan-Ganz catheter) is an excellent method for measuring pulmonary vascular resistance. Indications include hypovolemia with significant fluid management issues, and low urine output following an operation. A fixed reading on the pulmonary artery catheter with loss of respiratory variation is an indication for malposition. TABLE 368 PATHOLOGICAL HEART SOUNDS Pathological Heart Sounds Diagnosis

S1: Wide splitting. Loud S1 S2 Pulmonic valve closes before the aortic valve and is especially noticeable upon inspiration.

Etiology

S1: Splitting=RBBB; Loud S1=mitral stenosis S2: Hypertrophic cardiomyopathy, aortic stenosis, left bundle branch block.


USMLE STEP 2

296

PHARMACOLOGY CALCIUM CHANNEL BLOCKERS DRUG

MECHANISM OF ACTION

INDICATIONS

Diltiazem

HTN Angina

COMPLICATIONS

L-type calcium channel blockade

CONTRAINDICATIONS Sick sinus syndrome, 2nd & 3rd degree AV block, severe hypotension, acute MI.

↓CV, edema, dizziness

The calcium channel blockers work well in African Americans and the elderly for the control of HTN

NITRATES DRUG

MECHANISM OF ACTION

INDICATIONS

Nitroglycerin

Nitroprusside

Hydralazine

HTN Angina PE

↑GMP

COMPLICATIONS

NOTES

↑HR, HA, dizziness

Fast acting medication available in sublingual and IV formulation

Malignant HTN Angina PE

↑GMP

↑HR, HA, dizziness

Fast acting medication

Malignant HTN CHF

↑GMP

Reflex ↑HR, SLE

Causes vasodilation more then venodilation

Risk of syncope and methemoglobinemia. Treat methemoglobinemia with methylene blue.

CARDIAC GLYCOSIDES DRUG

INDICATIONS

Digoxin

CHF

MECHANISM OF ACTION Increases contractility through Na/K ATPase inhibition with a subsequent inhibition of the secondary active Na/Ca antiport. The net effect is an increase in intracellular calcium.

COMPLICATIONS N/V/D. Yellow vision. Hypokalemia leading to arrhythmia.

NOTES Positive inotrope. No change in mortality, but reduces hospital admissions.

High bioavailability with long half-life. Treat arrhythmias induced by the cardiac glycosides with K+ normalization, lidocaine, and anti-digoxin Fab.

CLASS IA ANTIARRHYTHMICS DRUG Amiodarone

Procainamide

INDICATIONS

MECHANISM OF ACTION

COMPLICATIONS

NOTES

Suppress ectopic rhythms from abnormal pacemaker cells Phase 3 and 4 effects

Decrease phase 4 depolarization and increase firing threshold State-dependent effects target abnormal pacemaker cells

AV block, ventricular arrhythmia

Sodium channel blocker

Suppress ectopic rhythms from abnormal pacemaker cells Phase 3 and 4 effects

Decrease phase 4 depolarization and increase firing threshold State-dependent effects target abnormal pacemaker cells

SLE, AV block, ventricular arrhythmia

Sodium channel blocker

www.ClinicalReview.com


Cardiovascular System

297

CLASS IB ANTIARRHYTHMICS DRUG

INDICATIONS

MECHANISM OF ACTION

Lidocaine

Ventricular arrhythmia Phase 3 and 4 effects

Decrease AP duration

COMPLICATIONS Local anesthesia CNS changes ↓CV, Proarrhythmic

NOTES Also used in digitalis toxicity Targets ischemic and depolarized tissue

CLASS IC ANTIARRHYTHMICS DRUG Flecainide

INDICATIONS

MECHANISM OF ACTION

VT and VF

COMPLICATIONS

Increase threshold of firing and decrease phase 4 depolarization

Proarrhythmic

CLASS II ANTIARRHYTHMICS DRUG Metoprolol

INDICATIONS Suppress ectopic focus, HTN

MECHANISM OF ACTION ↓cAMP with ↓Ca2+ current Leads to ↓phase 4 in AV node with ↑PR

COMPLICATIONS Proarrhythmic, impotence, asthma, CV and CNS effects, masks hypoglycemia

CLASS III ANTIARRHYTHMICS DRUG Amiodarone

INDICATIONS Last line for arrhythmia

MECHANISM OF ACTION ↑AP, ↑ERP, ↑QT, ↓IK

COMPLICATIONS Pulmonary fibrosis, hepatotoxicity, smurf skin, photodermatitis, CNS and CV effects, change in thyroid function

NOTES Long half-life, very toxic

CLASS IV ANTIARRHYTHMICS DRUG Diltiazem

INDICATIONS

SVT

MECHANISM OF ACTION

COMPLICATIONS

↓conduction velocity to ↑ERP and ↑PR due to calcium-channel effects

Flushing, constipation, CV effects

NOTES Targets AV node Ca2+ channel blocker

Generally, class IA and IC decrease phase 0 depolarization to slow conduction, class IB decrease phase 3 repolarization to slow conduction, class II suppresses phase 4 depolarization, class III prolongs phase 3 repolarization, and class IV shortens the action potential to make reaching threshold more difficult

STUDIES AND PROCEDURES ELECTROCARDIOGRAM QRS AXIS DEVIATIONS


USMLE STEP 2 298 Deflections between 180 and -90 degrees are commonly due to emphysema, hyperkalemia, transposition of leads, cardiac pacing, and ventricular tachycardia. Deflections between -90 and -30 can be normal in children and thin adults, or due to right ventricular hypertrophy (RVH), chronic lung disease, anterolateral myocardial infarction (MI), left posterior hemiblock, pulmonary embolism (PE), Wolff-Parkinson-White (WPW) syndrome, atrial septal defect (ASD), or ventricular septal defect (VSD). Deflections between 90 and 180 degrees are due to left anterior hemiblock, inferior MI, cardiac pacing, emphysema, hyperkalemia, WPW, tricuspid atresia, ostium primum ASD, and left coronary artery (LCA) contrast injection.

OTHER EKG CHANGES EKG changes may be found in left ventricular hypertrophy (LVH) with tall S waves in V1 and V2 and tall R waves in V5 and V6. Right ventricular hypertrophy (RVH) may be found with an R wave taller than the S wave in V1. Atrial hypertrophy is indicated in leads II and V1 with peaked P waves indicated right atrial hypertrophy (RAH) and notched and wide P waves in left atrial hypertrophy (LAH). Ischemic changes typically present with T wave inversions in leads I, II, and V2-V6. Myocardial injury has elevated ST (STe) segments, while an infarct will progress to Q waves. The location of anterior left ventricular injury can often be diagnosed by examining the chest leads. V1 and V2 will be abnormal in anteroseptal injury; anterior injury will be diagnosed by changes in V3 and V4; and V5 and V6 will indicate anterolateral injury. Leads II, III, and aVF indicate damage to the inferior wall (the so-called inferior leads). Lateral wall damage is obvious in leads I and aVL. And, reciprocal changes will be found in V1 and V2 with posterior wall damage.

www.ClinicalReview.com


Cardiovascular System

299

TABLE 369 ELECTROCARDIOGRAM (EKG) DETECTED ABNORMALITIES Electrocardiogram (EKG) Detected Abnormalities Short PR segments (less than .2 sec)

Wolff-Parkinson-White (WPW) syndrome, LGL syndrome, DMD, and Pompe disease.

Long PR intervals

First degree heart block.

QRS complex (not less than .12 sec)

Hyperkalemia, bundle branch blocks.

Increase in QT interval

MI, myocardial disease, hypocalcaemia, hypothyroidism, CVA, or in Romano Ward syndrome.

ST segment (flat or elevated)

MI, acute pericarditis, depressed in myocardial ischemia, digoxin toxicity, ventricular hypertrophy, acute posterior MI, PE, or LBBB.

Tall T waves

Hyperkalemia, very recent MI, and LBBB.

Flat or inverted T waves

May be idiopathic, LVH, digoxin, PE, RBBB, deficits in electrolytes.

QRS axis deviations (between 180 & -90 deg.)

Commonly due to emphysema, hyperkalemia, transposition of leads, cardiac pacing, & ventricular tachycardia.

QRS axis deviations (between -90 & -30 deg)

Can be normal in children and thin adults, or due to RVH, chronic lung disease, anterolateral MI, left posterior hemiblock, PE, WPW syndrome, ASD, or VSD.

QRS axis deviations (between 90 & 180 deg)

Left anterior hemiblock, inferior MI, cardiac pacing, emphysema, hyperkalemia, WPW, tricuspid atresia, ostium primum ASD, and LCA contrast injection.

Tall S waves in V1 & V2 & tall R waves in V5 & V6.

LVH

R wave taller than the S wave in V1

RVH

Peaked P waves

RAH

Notched P waves

LAH

T wave inversions

Ischemic changes (in leads I, II, and V2-V6).

Elevated ST segments

Myocardial injury.

Elevated Q wave

Infarct

Examine chest leads

Anterior left ventricular injury.

Abnormal V1 & V2

Anteroseptal injury.

Changes in V3 & V4

Anterior injury

Changes in V5 & V6

Anterolateral injury.

Leads II, III, & a VF

Indicate damage to the inferior wall.

Leads I and a VL

Lateral wall damage.

ECHOCARDIOGRAM Echocardiograms use sound waves in order to conduct real-time visualization of the heart anatomy and function. There are two non-stress versions, the transthoracic echocardiogram (TTE) and transesophageal echocardiogram (TEE). The TTE is preferred as the initial study as it is less invasive; it is better in thin patients as there is less tissue between the heart and the instrument. The TEE is the next test that is often utilized due to significantly more detail and the ability to assess the great vessels. Echocardiography is used for a multitude of heart diseases, including assessing wall motion abnormalities in MI, identifying

Â


USMLE STEP 2 300 ventricular function and ejection fraction (EF) in congestive heart failure (CHF), determining the severity of valvular disease, assessing pericardial effusions, identifying tears in aortic dissection, detecting increased right-sided pressure in pulmonary embolisms (PE), observing bubbles leaking through a patent foramen ovale (PFO) after administering air through a peripheral intravenous (IV) line, and identifying anatomic defects in congenital heart disease (CHD).

TREADMILL STRESS ECHOCARDIOGRAM (TSE) A TSE is used to detect the severity of coronary artery disease (CAD) and other heart disease (HD). The treadmill echocardiogram has patients walking on a treadmill that increases in difficulty to allow the patient to reach 85% of their maximum heart rate. Concurrent EKG monitoring detects ischemic changes, arrhythmias, and other signs and symptoms of heart disease. CAD is often diagnosed following EKG changes including ST depression (STd) or ST elevation (STe), and decreased blood pressure (BP). The patient must be able to walk on a treadmill in order to successfully have a diagnostic TSE. Contraindications to TSE may permit stress testing via a dobutamine stress echocardiogram (DSE).

THALLIUM IMAGING The use of radio-labeled molecules, such as thallium 201 or technetium 99, permit imaging the heart and determining proper cardiac functioning. Nuclear medicine scans (NMS) using the aforementioned molecules permit gauging myocardial perfusion and EF, and also permit an analysis of the end diastolic volume (EDV) and end systolic volume (ESV). Together with heart rate (HR), the cardiac output (CO) can be determined, along with the mean stroke volume (SV). Specific defects in cardiac function can be identified with careful analysis of NMS.

CARDIAC CATHETERIZATION Catheterization through the femoral vessels permits the diagnosis and treatment of several major heart defects and the right heart structures can be accessed and manipulated. In addition, the left heart can also be accessed. Balloon angioplasty, known as percutaneous transluminal coronary angioplasty (PTCA), can be used to dilate a stenosis in a major coronary artery and to place a stent. PTCA is often used as an emergent procedure immediately following certain types of MI and in unstable angina. Manipulation of the heart structures, such as valvuloplasty in valvular heart disease, radiofrequency ablation of ectopic foci in arrhythmia, biopsy of the heart wall in certain heart diseases, and closing patency such as an ASD, VSD, or PFO can all be done through catheterization. TABLE 370 CARDIAC TESTS Cardiac Test

Echocardiogram

Wall motion abnormalities in MI, ventricular function and EF in CHF, determining the severity of valvular disease, assessing pericardial effusions, identifying tears in aortic dissection, detecting increased right-sided pressure in pulmonary embolisms, observing bubbles leaking through a PFO, and identifying anatomic defects in CHD.

TSE

Detect the severity of CAD.

Thallium imaging

Myocardial perfusion and EF, also permit an analysis of EDV & ESV.

Cardiac catheterization

Via the femoral vein: the right heart structures can be accessed and manipulated. The left heart can also be accessed. Balloon angioplasty coronary PTCA can be used to dilate a stenosis in a major coronary artery and place a stent. PTCA can also be used as following certain types of MI and in manipulation of the heart structures, such as valvuloplasty, radiofrequency ablation of ectopic foci, biopsy of the heart wall, and closing patency.

www.ClinicalReview.com


Cardiovascular System

301

CAUSES OF CHEST PAIN Chest pain can be attributed to a number of causes, and although a particular pain syndrome may be classic for a specific etiology, some of the most serious causes of chest pain can atypically present with little or no pain. The common causes of chest pain are divided into those due to gastroesophageal reflux disease (GERD) and disorders of the esophagus,(which typically presents as a burning sensation, dysphagia, and general symptoms similar to a myocardial infarction). Pneumonia presents as pleuritic chest pain that is accompanied by cough and worsens with deep inspiration. Pulmonary embolism (PE) is associated with a coagulopathy and the presence of a deep vein thrombosis (DVT), and presents with tachypnea, tachycardia, and acute chest pain. Costochondritis is one of the more common causes of chest pain, and is a localized, sharp pain that is reproducible upon physical exam. Pericarditis presents as a chest pain that radiates to the shoulder, back, or neck and is relieved by leaning forward. Aortic dissection presents with a tearing sensation, chest pain that radiates to the back, and discordant pulses and blood pressures between arms. Finally, MI and angina present with substernal pain and pressure that radiates to the left arm, jaw, or shoulder. Other causes of chest pain that should be on the differential diagnosis include peptic ulcer disease (PUD), hiatal hernia, hepatobiliary disease, herpes zoster, pneumothorax, pulmonary hypertension, and various psychiatric conditions. TABLE 371 CAUSES OF CHEST PAIN Causes of Chest Pain GERD

Burning sensation, dysphagia, and general symptoms similar to MI.

Pneumonia

Pleuritic chest pains accompanied by cough and worsen with deep inspiration.

Pulmonary embolism

Coagulopathy & DVT, tachypnea, tachycardia, and acute chest pain.

Costochondritis

Localized, sharp pain that is reproducible upon physical exam.

Other differential diagnosis etiologies

PUD, hiatal hernia, hepatobiliary disease, herpes zoster, pneumothorax, pulmonary hypertension, and various psychiatric conditions.

CONGENITAL HEART DEFECTS TYPE

PATHOPHYSIOLOGY

PRESENTATION

NOTES

Tetralogy of Fallot

Abnormal neural crest migration leading to displacement of anterosuperior infundibular septal development.

Pulmonary stenosis, RVH, overriding aorta, and VSD leads to RÆL shunt and mixing of blood.

RÆL shunt w/ cyanosis, clubbing, boot-shaped heart, syncope.

If aorta is from RV, then known as Taussig-Bing malformation; Tet spells in ToF.

Transposition of great vessels

Aorta comes from R heart, pulmonary artery comes from L heart – requires PDA or other RÆL shunt to be compatible with life.

Defect in neural crest migration leads to septal malformation.

Truncus arteriosus

ETIOLOGY

Risk factor includes diabetes.

Single great vessel leaving heart due to failure of septum development from faulty neural crest migration.


302

USMLE STEP 2

Aortic coarctation

Infantile: Aortic stenosis proximal to end of aortic arch Adult: Aortic stenosis distal to end of aortic arch. Presents with notching of ribs, UE HTN, cerebral hemorrhage, infective endocarditis.

Infantile occurs in Turner syndrome.

Patent ductus arteriosus

Machine-like murmur.

Indomethacin closes; PGE1 keeps open.

VSD

High or low-pitched murmur (low or none = bad).

Most common heart defect – mostly in membranous portion of heart.

Loud S1 and fixed S2.

Typically a secundum type defect – less common.

ASD

Eisenmenger syndrome – Occurs later in life with long-term RVH that eventually reverses a LÆR shunt and makes it RÆL. Occurs with PHTN and requires heart-lung transplant. When shunt reverses, leads to cyanosis, hypoxia, clubbing, polycythemia.

CORONARY HEART DISEASE INTRODUCTION Coronary heart disease (CHD), also known as coronary artery disease (CAD), is a serious public health problem that affects nearly 14 million Americans. CHD is the result of years of fatty acid deposition, calcification, and subsequent plaque formation within the coronary arteries. Narrowing of the artery lumen by the plaque leads to decreased blood to the myocardium, followed by symptoms of angina and chest pain. Sudden complete blockade of the lumen, as can occur with the rupturing of the plaques, leads to signs and symptoms of an MI. Coronary heart disease is a significant cause of death in the United States.

RISK FACTORS FOR CHD There are a number of risk factors for the development and progression of CHD. Atherosclerosis is the most significant cause, and this disease may be exacerbated by a positive family history, smoking, elevated titers of low-density lipoprotein (LDL), decreased titers of high-density lipoprotein (HDL), hypertension (HTN), central obesity, diabetes mellitus (DM), elevated C-reactive protein (CRP), elevated homocysteine levels, and increase in the circulation of fibrinogen. Lack of regular exercise, poor diet, high levels of emotional stress, and a typical type A personality also contribute to the development of CHD. Anemia and carbon monoxide poisoning can significantly decrease the oxygen-carrying capacity of the blood and exacerbate CHD. A family history for CHD is considered positive if relatives have experienced a MI prior to the age of 45 in men, and 55 in women. The major causes of CHD are discussed below. CHD that presents with decreased blood supply and oxygen to the heart is often referred to as ischemic heart disease (IHD).

www.ClinicalReview.com


Cardiovascular System

303

TABLE 372 CORONARY HEART DISEASE Coronary Heart Disease Epidemiology

Affects nearly 14 million Americans, significant cause of death in America.

Etiology

The result of years of fatty acid deposition, calcification, and plaque formation within the coronary arteries. Narrowing of the artery lumen by the plaque leads to decreased blood to the myocardium, causing symptoms of angina and chest pain.

Risk factors

Atherosclerosis, a positive family history, smoking, elevated titers of LDL, decreased titers HDL, HTN, central obesity, DM, elevated CRP, elevated homocysteine levels, increase in circulation of fibrinogen. Lack of regular exercise, poor diet, high levels of emotional stress, and type A personality.

DYSLIPIDEMIA NORMAL LIPID TRANSPORT Normal lipid transport relies on a number of particles to carry lipids between various organs of the body. Derangements in this transport, or overloading certain carriers, results in deposition of this lipid into the arteries and can lead to CHD over time. Lipid transport begins with digestion of fatty acids and their breakdown within the small intestine. Digested cholesterol is transported via chylomicrons. Fatty acids are removed from chylomicrons for use in normal tissue function by lipoprotein lipase (LPL) in the liver. The remnants of the chylomicrons are converted into very low-density lipoprotein (VLDL) and secreted by the liver. VLDL carries fatty acids and cholesterol in the bloodstream to various tissues, which metabolize the VLDL to form intermediate-density lipoprotein (IDL). The IDL is metabolized to form LDL, and continues to transport the cholesterol-laden vesicles throughout the body. Once metabolism of LDL is complete, the now high-density molecules, HDL, are transported from various tissues back to the liver for further metabolism and excretion. Elevated LDL is the single most important risk factor for dyslipidemia, while elevated HDL titers (>60) indicate good metabolic function and diminished risk of dyslipidemia and subsequent CHD.

ETIOLOGY AND PATHOGENESIS Dyslipidemia is the cause of nearly half of all CHD, and most cases have a significant familial component. The types of dyslipidemias are broken down into five distinct types. Causes of isolated elevated cholesterol titers are typically due to type IIa, which includes familial hypercholesterolemia, familial defective apo B100, and polygenic hypercholesterolemia. The total cholesterol titers vary, but are often between 250 and 500 with elevated LDL but normal VLDL and chylomicron levels. Hypertriglyceridemia may occur by itself with familial hypertriglyceridemia and elevated VLDL (type IV disease), familial lipoprotein lipase deficiency or familial apo CII deficiency, both with elevated chylomicrons (type I and V disease). Diabetes with poor glucose control can also present as type I or type V disease. A combined hyperlipidemia occurs with elevations in VLDL and LDL in type IIb disease, while a dysbetalipoproteinemia with elevated VLDL and IDL occurs in type III disease. The highest cholesterol concentrations are found in type I, IV, and V disease. Very high triglyceride levels may lead to pancreatitis as a complication.

PRESENTATION Dysbetalipidemia presents with obvious signs and symptoms only in very severe disease. Severe hypercholesterolemia may present with xanthelasmas, which are yellow-colored swellings on the eyelids, and xanthomas, which are red papules on the exposed portions of the skin.

Â


304

USMLE STEP 2

DIAGNOSIS Diagnosis of dyslipidemia is made by a lipid profile after fasting overnight. Repeated testing may be required in pregnancy, recent weight loss, following surgery, and in severe illness. Testing done within the first few hours following an MI will be indicative of pre-MI lipid levels, while those performed thereafter may be the result of disease. Total cholesterol (TC) is considered normal if it falls below 200. Cholesterol below 240 is considered borderline, while greater than 240 is high. Normal HDL is between 30 and 100, although recent guidelines prefer an HDL greater than 60 in order for HDL to be cardioprotective. LDL should be around 100, with 130-159 borderline, 160-189 high, and 190 or more considered to be very high.

TREATMENT Dyslipidemia is treated with immediate changes to dietary intake and increased exercise. A Step 1 diet is tried for the first few months and includes decreasing total fat intake to less than 30% of all caloric intake, keeping saturated fats below 10%, and reducing dietary cholesterol to less than 300 mg per day. A step 2 diet is used thereafter by further decreasing the saturated fat intake to less than 7% of total caloric intake, and reducing dietary cholesterol to less than 200 mg per day. Successful implementation of a step 2 diet can reduce total cholesterol by more than 10%, and exercise can be cardioprotective by raising HDL. Failure of diet by six months typically heralds the use of medications to improve the lipid profile.

MEDICATIONS Treatment for dyslipidemia begins after the failure of diet and exercise to return a lipid profile to within normal limits (WNL). The use of 3-hydroxy-3-methylglutaryl-coenzyme A (HMG-CoA) reductase inhibitors, such as statins, reduces the conversion of acetylCoA to LDL, and increases the formation of HDL. Statins are among the most successful medications for decreasing LDLs, which can be nearly 35%, and raising HDL by nearly 10%. The use of bile acid sequestrants such as cholestyramine and colestipol are sometimes used to decrease LDL by 20%; these medications function by binding to bile acids in the intestine and preventing their reuptake. Nicotinic acid, or niacin, is used to prevent the synthesis of cholesterol and prevents the liberation of fatty acids from adipose tissue, leading to a gradual 20% decrease in LDL. Fibrates are useful for increasing HDL by up to 30%, although their success varies. Thus far, the use of vitamin E appears to have a negative effect on the lipid profile, plant stanols have been shown to decrease cholesterol absorption, calcium blockers, angiotensin-converting enzyme (ACE) inhibitors, angiotensin receptor blockers (ARBs), and alpha-adrenergic blockers have no effect on lipids, beta-receptor blockers and diuretics worsen the lipid profile, and fish oils such as omega-3 fatty acids (FA) decrease triglycerides and increase HDL. TABLE 373 DYSLIPIDEMIA Dyslipidemia Presentation

Presents with obvious signs and symptoms only in very severe disease. Severe hypercholesterolemia may present with xanthelasmas, xanthomas.

Diagnosis

Fasting lipid profile.

Treatment

Changes to dietary intake and increased exercise. Medication, if diet & exercise fail.

Medications

Statins, cholestyramine and colestipol, niacin, fish oils.

HYPERTENSION ETIOLOGY AND EPIDEMIOLOGY

www.ClinicalReview.com


Cardiovascular System

305

The formal diagnosis of hypertension is made after confirmation of an elevation in systolic blood pressure (SBP) over 140 or diastolic blood pressure (DBP) over 90. HTN is divided into an idiopathic cause (90% of all cases), and secondary HTN attributable to a specific medical condition. Secondary HTN can be due to renal artery stenosis (RAS), Cushing or Conn syndrome leading to hyperaldosteronism, pheochromocytoma, eclampsia / preeclampsia, aortic coarctation, and parenchymal disease of the kidney. HTN is broken down into four stages: stage I HTN has a SBP below 150 and a DBP below 100; stage 2 disease has SBP below 180 and DBP below 110; stage 3 disease has SBP below 210 and DBP below 120; and stage 4 disease is any SBP above 210 or DBP above 120. The highest blood pressure determines the stage. HTN affects nearly 1/3 of the adult American population.

PATHOPHYSIOLOGY Hypertension is the result of increased systemic vascular resistance (SVR) with a normal cardiac output. This results in an increase in work by the heart in order to have the same stroke volume (SV) with every heartbeat. Risk factors for HTN include a high salt diet, obesity, smoking, diabetes, family history, African American race, and male gender.

PRESENTATION Hypertension presents with few, if any symptoms unless severe. Such symptoms may include epistaxis, hematuria, blurred vision, chest pain, light-headedness, headache, and congestive heart failure (CHF).

DIAGNOSIS The diagnosis of HTN is made by testing the BP in both arms and finding symmetrically abnormal readings of at least stage 1 HTN. Ophthalmoscope exam is typically positive for papilledema, arteriovenous (AV) nicking, and punctate hemorrhages. Renal artery bruits may be present with renal disease or RAS, and hematuria may be present on urinalysis (UA). Renal disease may be tested with kidney function tests, with abnormalities in the BUN) to creatinine ratio, and elevations in serum potassium. Finally, an electrocardiograph should be done in all patients with long-standing HTN to diagnose any cardiac conduction dysfunction.

TREATMENT Hypertension requires a reduction in fatty-acid intake similar to the step 1 and step 2 diets of dyslipidemia. Weight loss and exercise are required to improve cardiac function and reduce SVR. Improved mortality has been found with the use of betareceptor blockers and thiazide diuretics. The use of ACE inhibitors is especially effective in Caucasians and diabetics. African American patients benefit more from calcium-channel blockers. Patients with dyslipidemia should avoid beta-blockers. Pregnant patients should avoid diuretics and ACE inhibitors, and should be treated with alpha-methyldopa and hydralazine. Diabetics should avoid beta-blockers, as should asthmatics and patients with vascular disease. Patients with gout should avoid diuretics, and elderly patients are most likely to benefit from diuretics, ACE inhibitors, and calcium channel blockers to reduce the isolated SBP commonly found in this group.

Â


306

USMLE STEP 2

TABLE 374 HYPERTENSION Hypertension Etiology

Increased SVR with a normal CO. This results in an increase in work by the heart in order to have the same SV with every heartbeat. Risk factors for HTN include a high salt diet, obesity, smoking, diabetes, family history, African American race, and males.

Presentation

Presents with few, if any symptoms unless severe. Severe symptoms may include epistaxis, hematuria, blurred vision, chest pain, light-headedness, headache, and CHF.

Diagnosis

Testing the BP in both arms and finding symmetrically abnormal readings of at least stage 1 HTN. Ophthalmoscope exam positive for papilledema, AV nicking, & punctate hemorrhages.

Risk factors

High salt diet, obesity, smoking, diabetes, family history, African American race, and male gender.

Treatment

Reduction in fatty-acid, weight loss and exercise, use of beta-receptor blockers and thiazide diuretics. The use of ACE inhibitors is especially effective in Caucasians and diabetics. African American patients benefit more from calcium-channel blockers.

MALIGNANT HYPERTENSION ETIOLOGY AND PRESENTATION Malignant HTN is severe BP increases that result in end-organ damage. Damage is typically found in a fundoscopic exam, but damage may also occur within the renal parenchyma, central nervous system, heart, and elsewhere with signs such as neurologic changes, papilledema, heart failure, renal failure, and cerebrovascular accident (CVA).

TREATMENT Treatment of malignant HTN is to reduce the mean arterial pressure (MAP), and IV agents should be used in emergency situations. Such agents include nitroglycerin, nitroprusside, and labetalol. Pheochromocytoma is often controlled with phentolamine, and pre-eclampsia is controlled with hydralazine or alpha-methyldopa. Magnesium sulfate is also given for pre-eclampsia. Nitroprusside is preferred with CNS changes, while nitroglycerin is preferred with coronary changes. TABLE 375 MALIGNANT HYPERTENSION Malignant Hypertension Etiology

Severe BP increase that results in end-organ damage.

Treatment

Reduce MAP, nitroglycerin, nitroprusside, labetalol.

ANGINA ETIOLOGY AND PATHOPHYSIOLOGY Angina is chest pain associated with decreased oxygen flow to the myocardium leading to temporary, reversible ischemia. There are three major types of angina – stable angina, unstable angina, and Prinzmetal angina.

www.ClinicalReview.com


Cardiovascular System

307

PRESENTATION Angina presents with a retrosternal pain that radiates to the left shoulder, arm, or jaw. Angina lasts a few minutes and is intermittent in nature. The pain is typically a heavy pressure sensation, sometimes described as a squeezing or tightening inside the chest. Angina typically does not last more than 15 minutes – if it does, the clinician should seriously consider angina evolving into an MI. Angina is precipitated by factors that affect cardiovascular stability or increase cardiac work; these factors typically include vasoconstriction due to cold or exertion, psychoactive factors such as anxiety, and eating large meals. Angina also presents with shortness of breath (SOB), nausea and vomiting (N/V), diaphoresis, and palpitations. Angina is relieved by nitroglycerin within a few minutes; resting also ameliorates the symptoms of angina. Physical signs of angina include tachycardia with or without an S4 gallop.

STABLE ANGINA Stable angina is due to ischemia of the myocardium leading to episodic pain. Stable angina is chronic in nature and does not progress over time. Stable angina occurs with predictive changes to myocardial oxygen consumption, such as with strenuous exercise. Stable angina is relieved with rest. This disease may evolve into unstable angina.

UNSTABLE ANGINA Unstable angina is any new-onset angina that progresses over time with respect to location, frequency, or severity. Unstable angina can occur at rest, and typically requires increasing amounts of medication to have relief from symptoms. Unstable angina requires treatment to avoid evolution into an MI.

PRINZMETAL ANGINA Prinzmetal angina is a rare variant of angina that is due to vasospasms of the coronary arteries. Prinzmetal presents as a chronic, intermittent chest pain unrelated to exertion, and tends to occur at predictable times in the morning. The pain wakes up the patient at night. Prinzmetal angina presents with ischemic changes and ST elevations on EKG during exacerbations. Diagnosis of Prinzmetal is done with angiography.

DIAGNOSIS Diagnosing angina proceeds by ruling out other causes of chest pain by carefully obtaining a history, completing a physical exam, and running an EKG. EKG changes may be normal, or present with ST changes (depression or elevation), and T wave inversions. An exercise stress test (treadmill stress echocardiogram, or TSE) is done to determine the type of angina that occurs and its severity. A TSE with thallium is done in patients with valvular defects, arrhythmia, young women, and those with acute ischemic changes. Other examinations available to diagnose and determine the prognosis of angina include dobutamine stress echocardiogram (DSE), persantine echocardiogram, and thallium imaging. Stress tests should not be performed on individuals with unstable angina due to the risk of evolution to MI. Patients with aortic stenosis (AS), hypertrophic subaortic stenosis, chronic obstructive pulmonary disease (COPD), congestive heart failure, aortic dissection, ischemic changes on EKG, uncontrolled HTN, and those with general contraindications to exercise should also not receive a stress echocardiogram.

TREATMENT Angina is initially treated by lifestyle and dietary modifications. Patients with stable angina should receive sublingual nitroglycerin (SLN) during exacerbations, percutaneous transluminal coronary angioplasty (PTCA) or a coronary artery bypass graft (CABG), beta blockade to reduce oxygen demand by the myocardium, and aspirin (ASA) to decrease the risk of evolution to MI. Those with

Â


USMLE STEP 2 308 unstable angina should also receive SLN, PTCA and CABG as indicated, and should also be monitored in a cardiac care unit (CCU) due to the risk of MI. Heparin or low molecular weight heparin (LMWH) should be used in conjunction with ASA to decrease the risk of thrombosis. Finally, those patients with Prinzmetal angina should receive calcium channel blockers and nitrates to reduce coronary artery spasms; beta-blockers are contraindicated in this population. TABLE 376 ANGINA Angina Etiology

Severe BP increase that results in end-organ damage.

Presentation

Retrosternal pain that radiates to the left shoulder, arm, or jaw. Lasts a few minutes and is intermittent. The pain is typically a heavy pressure sensation. Angina also presents SOB, N/V, diaphoresis, and palpitations. Physical signs of angina include tachycardia with or without an S4 gallop.

Types

Stable angina: due to ischemia of the myocardium leading to episodic pain. Chronic in nature and does not progress over time. Occurs with predictive changes to myocardial oxygen consumption, is relieved with rest. Unstable angina: new-onset angina that progresses over time with respect to location, frequency, or severity. Can occur at rest, and typically requires increasing amounts of medication to have relief from symptoms. Prinzmetal angina: rare variant of angina that is due to vasospasms of the coronary arteries. Chronic, intermittent chest pain unrelated to exertion, and tends to occur at predictable times in the morning. The pain wakes up the patient at night. Presents with ischemic changes and ST elevations on EKG during exacerbation.

Treatment

Initially treated by lifestyle and dietary modifications Stable angina: SLN during exacerbations, PTCA or a coronary artery CABG, beta blockade, and ASA. Unstable angina: SLN, PTCA and CABG as indicated, and should also be monitored CCU. Heparin or low LMWH should be used in conjunction with ASA. Prinzmetal angina: calcium channel blockers and nitrates.

MYOCARDIAL INFARCTION (MI) ETIOLOGY AND EPIDEMIOLOGY MI is the development of myocardial necrosis following prolonged ischemia. MI is typically attributed to sudden rupture of a plaque leading to coronary artery blockade, an embolism from a secondary source, shock leading to impaired cardiac perfusion, and coronary vasospasm. Hypercoagulable states such as thrombocytosis and polycythemia vera can contribute to embolism, and direct embolisms can occur from atrial myxoma and coronary thrombi. Spasms of the coronary arteries are common in cocaine abuse and in Prinzmetal angina. Finally, inflammation of the vasculature can lead to MI, as in systemic lupus erythematosus (SLE), polyarteritis nodosa (PAN), Takayasu arteritis, and Kawasaki disease. Risk factors for MI include males over 55 years of age, postmenopausal women, smokers, HTN, hyperlipidemia, DM, and atherosclerosis.

PATHOPHYSIOLOGY The typical MI is a progressive process that leads to significantly greater damage to the myocardium with the passage of time. If therapy is not immediately started after coronary vasocculsion, permanent damage to the myocytes sets in within 20 minutes. Coagulation necrosis begins within about 6 hours with the influx of PMNs. Total coagulative necrosis sets in after a day or two, and granulation tissue starts to form around 3-7 days. It is during this time that the destroyed myocytes are being cleared and fibrotic tissue is being formed. Rupture through the heart wall can occur, leading to cardiac tamponade and significant morbidity and mortality. Acute MI (AMI) can occur as a transmural lesion, which is typically accompanied with Q waves. A subendocardial infarct is confined to the innermost portion of the ventricular wall and does not present with Q waves; regardless of the cause of the MI, the subendocardial tissue of the left ventricle is the most susceptible to damage due to the tenuous oxygen supply from lack of direct arterial flow. www.ClinicalReview.com


Cardiovascular System

309

TABLE 377 PATHOPHYSIOLOGY OF AN MI Pathophysiology of an MI After 20 minutes

Permanent damage to myocytes.

After 6 hours

Coagulation necrosis with influx of PMNs.

After a day or two

Total coagulation necrosis.

After 3-7 days

Granulation tissue starts to form.

PRESENTATION The typical MI presents with symptoms of severe, retrosternal chest pain for more than 20 minutes. This severe crushing pressure or squeezing sensation leads to nausea and vomiting, diaphoresis, weakness, and anxiety. Shortness of breath develops with failure of heart function. In order to compensate for the ischemic changes taking place, tachycardia ensues with an S4 gallop. Cardiogenic shock with hypotension, jugular venous distention (JVD), and S3 gallop, and rales may develop if more than half of the myocardium has been compromised. Depending on the severity of the damage, arrhythmias and septal rupture may occur. Inferior MIs may present with bradycardia. Typically, an MI will present as pain that radiates to the left arm or shoulder, or the jaw. It is most likely to occur in the early waking hours as this is when the blood pressure can be the highest. MI may be entirely silent in diabetics, the elderly, patients with HTN, and in postsurgical patients. Psychological symptoms include a feeling of impending doom.

DIAGNOSIS WITH EKG The initial test in diagnosing an MI is the use of an EKG. In inferior wall MI, there will be an STe in leads II, III, and aVF. Anteroseptal MI has STe in leads V1-V3. Lateral wall MI has STe in V4-V6, and posterior wall MI has STd in V1 and V2. New onset of Q waves is indicative of a transmural MI, and is the most common EKG finding after an ST elevation MI (STEMI). Flattening of the ST segment typically does not progress to Q waves, and this is known as a non-ST elevation MI (NSTEMI). A new LBBB is also diagnostic of MI. Echocardiograms in order to demonstrate hypokinesia of the ventricular wall are used in patients with nondiscernable EKGs but who otherwise fit the clinical picture of MI. The EKG changes that occur immediately upon onset of MI include sharp, peaked T waves and STe, followed by Q waves, then T wave inversions (Twi) after several hours. The Q waves may remain as an indicator of prior MI, and the Twi may remain for several years. TABLE 378 EKG FINDINGS EKG Findings Inferior wall MI

STe in leads II, III, and aVF

Anteroseptal MI

STe in leads V1-V3

Lateral wall MI

STe in V4-V6

Posterior wall MI

STd in V1 and V2

Transmural MI

Q waves after an ST elevation MI (STEMI)

Â


310

USMLE STEP 2

DIAGNOSIS WITH CARDIAC ENZYMES The progression of an MI leads to a predictable rise in cardiac markers of myocardial damage. Some of these markers are specific, others sensitive, and some useful only as early markers of disease. Myoglobin is typically one of the earliest markers to become serologically positive with elevations within one hour of experiencing an MI; however, myoglobin is found in numerous tissues and so is not very specific. Rises in CPK occur within 6 hours, but this marker is similar to myoglobin in that it is not specific. Troponin T (TnT) and troponin I (TnI) rise within 3-4 hours, remain elevated for up to a week, and are highly sensitive markers for even minimal myocardial damage (so called microinfarcts). Creatine kinase, MB fraction (CK-MB) is a highly specific and sensitive marker for cardiac damage that is positive within 4-6 hours and remains so for at least one or two days. The CK-MB fraction rises only with myocardial necrosis, but CK itself may rise with general cardiac trauma or manipulation with resuscitation procedures. Lactate dehydrogenase (LDH) is rarely used as a marker for cardiac damage due to its lack of specificity and sensitivity, but it tends to be positive after 12 hours and remains elevated for up to two weeks. Most centers in the United States now measure serial cardiac enzymes in patients suspected of MI (especially in ruling out MI). Cardiac enzymes including CK-MB, TnT, and CPK are typically collected every 6 hours three consecutive times along with EKG tracings. A patient typically rules out for MI with lack of EKG signs and no increases in cardiac enzymes. TABLE 379 DIAGNOSIS WITH CARDIAC ENZYMES Diagnosis with Cardiac Enzymes Myoglobin

One of the earliest markers within one hour of experiencing an MI; Not very specific.

CPK

Occurs within 6 hours. Not specific.

TnT, TnI

Rise within 3-4 hours, remain elevated for up to a week, and are highly sensitive markers.

CK-MB

Highly specific and sensitive marker for cardiac damage that is positive within 4-6 hours and remains so for at least 1 or 2 days. The CK-MB fraction rises only with myocardial necrosis, but CK itself may rise with general cardiac trauma or manipulation with resuscitation procedures.

TREATMENT Treatment of MI involves a series of medication and procedural orders. Aspirin, or its alternative, clopidogrel, are administered, immediately given evidence that aspirin can reduce mortality from MI by over 25% through decreased clot formation. Betablockers have also been shown to decrease mortality by limiting the extent of damage; metoprolol is typically given three times every 5 minutes and stopped if the heart rate falls below 60. If given in the first few hours after an MI, thrombolytic therapy has also been shown to decrease mortality by breaking up clots; agents include streptokinase and alteplase. Heparin is given to prevent the further development of clots. Oxygen is given to increase oxygen saturation and for patient comfort. Nitroglycerin is also given to relieve symptoms by dilating coronary arteries and reducing cardiac oxygen demand through decreased preload. Morphine is given to reduce pain and anxiety, and thereby further decrease oxygen demand by the damaged myocardium. PTCA can be done on an emergent basis and within an hour or two of MI onset to restore the patency of the coronary arteries, especially if there is multiple vessel disease. Angioplasty is preferred in patients with contraindications to normal thrombolytic therapy, have CHF or poor EF, risk of dangerous arrhythmia development, and patients who fail thrombolytic therapy. Cardiac pacing may be required if arrhythmias develop as a result of damage to the heart. Patients should also be counseled with regard to dietary changes and exercise as discussed above for dyslipidemia and HTN.

www.ClinicalReview.com


Cardiovascular System

311

TABLE 380 MYOCARDIAL INFARCTION (MI) Myocardial Infarction (MI) Etiology

Development of myocardial necrosis following prolonged ischemia. Sudden rupture of a plaque, an embolism from a secondary source, shock, and coronary vasospasm.

Risk factors

Males over 55 years of age, postmenopausal women, smokers, HTN, hyperlipidemia, DM, and atherosclerosis.

Presentation

Severe, retrosternal chest pain for more than 20 minutes. Leads to nausea and vomiting, diaphoresis, weakness, and anxiety. SOB, tachycardia. Cardiogenic shock with hypotension, JVD. Arrhythmias and septal rupture may occur. Inferior MIs may present with bradycardia. Typically, an MI will present as pain that radiates to the left arm or shoulder, or the jaw. It is most likely to occur in the early waking hours. MI may be entirely silent in diabetics, the elderly, patients with HTN, and in postsurgical patients. Psychological symptoms include a feeling of impending doom.

Diagnosis

EKG, myoglobin, CPK, TnT, TnI, CK, CK-MB, LDH.

Treatment

Aspirin or clopidogrel, Beta-blockers, metoprolol, streptokinase and alteplase, heparin oxygen, nitroglycerin, morphine, PTCA, angioplasty, patient counseling, pacemaker.

STAGING MI is broken down into four classes with specific clinical findings and objective signs that are indicative of mortality. Class I MI presents with no pulmonary symptoms, indicating reasonable cardiac function without heart failure. The cardiac index (CI) is greater than 2.2, and the pulmonary capillary wedge pressure (PCWP) less than 15. The hospital mortality is less than 1%. Class II MI presents with pulmonary congestion, CI greater than 2.2, and PCWP more than 15. There is a 10% hospital mortality. Class III MI has systemic hypoperfusion with CI less than 2.2, PCWP less than 15, and a hospital mortality of about 20%. Class IV MI has pulmonary congestion with systemic hypoperfusion, CI less than 2.2, PCWP more than 15, and a hospital mortality exceeding 50%. TABLE 381 STAGING OF MI Staging of MI Class I

Presents with no pulmonary symptoms, CI is greater than 2.2, and the PCWP less than 15.

Class II

Presents with pulmonary congestion, CI greater than 2.2, and PCWP more than 15.

Class III

Systemic hypoperfusion with CI less than 2.2, PCWP less than 15.

Class IV

Pulmonary congestion with systemic hypoperfusion, CI less than 2.2, PCWP more than 15.

ARRHYTHMIA Complications arising from MI include those leading to arrhythmias, dysfunction leading to pump failure, ischemic changes, pericarditis, and sudden cardiac death. Arrhythmias that develop following an MI include bradycardia that may require the use of atropine to prevent ectopic foci from developing and the use of a pacemaker, if the bradycardia is severe enough. Premature atrial contractions (PACs) or premature ventricular contractions (PVCs) can develop, although these tend to be benign, if isolated. Supraventricular tachycardia (SVT) can occur with the development of atrial fibrillation, atrial flutter, or a junctional focus. Ventricular tachycardias (VT) can also occur, which heralds a more dismal prognosis, especially with ventricular fibrillation. Hemiblocks and bundle branch blocks can also occur, in addition to first, second, and third order atrioventricular (AV) blocks.

Â


312

USMLE STEP 2

HEART FAILURE Heart failure is an all too common occurrence following a significant heart attack, and can present with either mechanical disruption due to anatomic failure, or with contractile dysfunction due to failure of normal expansion and contraction from direct muscle damage. In addition, the conduction system of the heart can become dissociated from the targets it innervates.

ISCHEMIA Ischemia can continue and even progress following an MI. Extension of the infarct site can also occur, and is especially common if there is a sustained increase in myocardial oxygen demand but without the concomitant increase in supply. Thrombolytics should be used along with measures to ensure vessel patency prior to discharge.

PERICARDITIS Dressler syndrome is the development of a pericarditis following an MI. Dressler syndrome may occur between a few days to a few months following an MI, and is due to an inflammatory process that runs rampant and leads to damage to otherwise healthy cells. Dressler syndrome may also complicate other heart disorders, especially those that require open surgical management. It is a rare complication of MI, affecting fewer than one in a hundred patients. Dressler syndrome is treated with NSAIDs and steroids.

PRO-THROMBOTIC STATE MI can lend to a pro-thrombotic state due to relative stasis of blood residing in a now dysfunctional portion of the heart. This meets one of the three requirements of Virchow’s triad for vascular thrombosis (the other two being endothelial damage and a hypercoagulable state). These clots can go on to cause pulmonary dysfunction in a right-sided MI, or systemic complications such as stroke and renal destruction in a left-sided MI. Immobilization immediately after an MI can also predispose an individual to deep vein thrombosis (DVT) and subsequent PE.

PAPILLARY MUSCLE RUPTURE Rupture of the papillary muscles can lead to mitral regurgitation and sudden cardiac decompensation following a posteroinferior MI. It tends to occur approximately 3-5 days after an MI and presents with a loud holosystolic murmur. Echocardiography demonstrates a valve leaflet moving around with the flow and regurgitation.

VENTRICULAR SEPTAL RUPTURE Septal rupture is more likely in anterior MI and occurs 3-5 days afterwards. It presents with a loud holosystolic murmur that radiates to many locations, and also tends to have a palpable thrill. Echocardiography is used for diagnosis.

SUDDEN CARDIAC DEATH Sudden cardiac death is occasionally a result of a fatal arrhythmia that develops following an MI, such as ventricular tachycardia and ventricular fibrillation. Repeated myocardial ischemia is the largest positive predictor of risk for arrhythmias leading to sudden cardiac death.

Greatest risk of ventricular septal rupture occurs 3-5 days post MI from macrophageeffect. Presents as loud holosystolic murmur.

www.ClinicalReview.com


Cardiovascular System

313

TABLE 382 COMPLICATIONS OF MI Complications of MI Arrhythmia

Bradycardia, PACs, PVCs, SVT, atrial fibrillation, atrial flutter, or a junctional focus. VT Hemiblocks & BBB can also occur, in addition to first, second, and third order AV blocks.

Heart failure

Mechanical or contractile dysfunction. Conduction system of heart can become dissociated from the targets it innervates.

Ischemia

Ischemia can continue and progress following an MI. Extension of the infarct site can also occur.

Pericarditis

Dressler syndrome.

Prothrombotic state

Relative stasis of blood residing in a dysfunctional portion of the heart. Clots can go on to cause pulmonary dysfunction.

Papillary muscle rupture

Leads to mitral regurgitation and sudden cardiac decompensation following a posteroinferior MI.

Ventricular septal rupture

More likely in anterior MI.

Sudden cardiac death

Occasionally a result of a fatal arrhythmia that develops following an MI.

POSTINFARCT MANAGEMENT Patients who have had an MI should receive a stress echocardiogram prior to discharge to ensure reasonable cardiac function. Left ventricular function may be assessed using a multigated acquisition (MUGA) scan. Patients should be advised to continue ASA and beta-blockers, and patients with poor EF should be started on ACE-inhibitors. Close management of existing risk factors such as HTN, DM, hyperlipidemia, and poor lifestyle habits should be done to minimize these contributing causes and avoid a repeated MI. Patients at particular risk of continued cardiac dysfunction should be evaluated for PTCA and CABG. Post-MI patients should achieve at least 70% of their ideal target rate on a stress test TABLE 383 POSTINFARCT MANAGEMENT Postinfarct Management Stress echocardiogram, MUGA, ASA, beta-blockers, ACE inhibitors, PTCA, CABG, lifestyle change.

CONGESTIVE HEART FAILURE (CHF) ETIOLOGY Congestive heart failure is the loss in the heart’s ability to properly replenish the systemic or pulmonary blood volume. Rightsided CHF (RHF) leads to overflow in the systemic circulation leading to systemic venous congestion. Left-sided CHF (LHF) leads to pulmonary symptoms due to pulmonary venous congestion. The most common cause of CHF is following MI and the resultant ischemic changes. Abnormalities in myocardial cells also contributes to CHF; for example, cardiomyopathy arising from substance abuse, diseases such as sarcoidosis or hemochromatosis, and intrinsic cardiopulmonary diseases such as pulmonary HTN or aortic regurgitation can all lead to symptoms of CHF. Structural abnormalities such as valvular heart disease, congenital heart disease, CAD, constrictive pericarditis, restrictive cardiomyopathy (RCM), and high-output failure (such as wet beriberi) can exacerbate CHF. CHF can also be worsened with a high salt diet, arrhythmias that compromise the proper dynamics of heart function, infection (such as systemic infections or pneumonia), renal failure, high fluid loads, anemia, thyrotoxicosis, PE, and excessive exercise. The most common cause of right-sided CHF is left-sided CHF.

Â


314

USMLE STEP 2

PATHOPHYSIOLOGY DIASTOLIC DYSFUNCTION CHF can present with either a diastolic dysfunction or a systolic dysfunction. Diastolic dysfunction leads to decreased cardiac output (CO) due to elevated systolic pressure (SBP) with normal EF. EF is allowed to remain approximately normal as both SV and EDV decrease proportionately. Recall the formula for EF:

EF =

SV ( EDV − ESV ) = EDV EDV

The decrease in EF due to decreased SV leads to a decrease in CO in the absence of any change in HR. Recall the formula for CO to understand how diastolic dysfunction occurs:

CO = HR × SV The EF in diastolic dysfunction can be measured with echocardiography.

SYSTOLIC DYSFUNCTION In systolic dysfunction, there is decreased contractility leading to decreased EF. The decrease in EF is proportional to changes in SV and inversely proportional to changes in EDV. As a result, decreased EF leads to decreased SV, which in the absence of changes to HR, leads to decreased CO. In both diastolic and systolic dysfunction, congestion in either the pulmonary or systemic circulation is the end result.

CARDIOVASCULAR COMPENSATION As indicated by the formula for CO, the output per beat is directly proportional to HR and SV. With decreases in SV as found in CHF, the first compensatory mechanism is to increase the HR in order to return CO to baseline. The heart can also function along its Frank-Starling curve by increasing stretch, leading to ventricular dilation and thereby increasing EDV. While the heart is along its Frank-Starling curve, it is able to compensate for the increased EDV by increasing its contractility, leading to a return of SV to baseline but at higher pressures. Myocardial hypertrophy results from the increased pressures, and after some time, the heart decompensates, falls off the Frank-Starling curve, and any further dilation only leads to worsening CHF. A systemic compensation can also occur by way of decreased activation of stretch receptors in the heart and carotid artery, leading to increased systemic vascular resistance, increased afterload, and decreased SV. In other words, the systemic compensation is hardly helpful to restoring normal cardiac function.

RENAL COMPENSATION The kidneys also attempt to compensate for the decreased renal perfusion pressures. The renin-angiotensin-aldosterone (RAA) system is activated, leading to vasoconstriction and the retention of sodium chloride and water. This increased volume load leads to an increase in preload, which worsens the congestion as the heart is unable to compensate for the increased stretch by increasing its contractility – the reason is that by the time the kidneys attempt to compensate for the failing heart, the heart has already fallen off its Frank-Starling curve and can no longer adequately compensate for changes in EDV.

www.ClinicalReview.com


Cardiovascular System

315

PRESENTATION – RHF CHF presents differently depending on whether it is right-sided CHF or left-sided CHF. Right-sided CHF presents with hepatic congestion due to backing up of fluid from the right heart to the vena cava. The resultant hepatomegaly can lead to RUQ pain, distention of the jugular veins (JVD), ascites, peripheral edema, cyanosis, and a hepatojugular reflex (HJR). The HJR is increased distention of the jugular veins with gentle upward abdominal pressure due to a fluid wave that travels superiorly.

PRESENTATION – LHF Left-sided CHF presents with signs and symptoms of dyspnea on exertion (DOE), paroxysmal nocturnal dyspnea (PND), orthopnea, rales, nocturia, diaphoresis, an S3 gallop, and tachycardia. Both RHF and LHF can present with ankle edema, white sputum with flecks of blood, and cardiomegaly. Dyspnea is a complaint in CHF, and can present with a restrictive lung defect (RLD) demonstrable by pulmonary function tests (PFTs).

DIAGNOSIS Diagnosis of CHF is made by plain films of the chest demonstrating cardiomegaly, congestion of the pulmonary vasculature, Kerley B lines, and pulmonary effusion. Echocardiography can be used to demonstrate abnormalities in ventricular wall motion and diminished EF. EF can also be evaluated using a MUGA scan. A urinalysis (UA) will demonstrate oliguria (from activation of the RAA axis), increased specific gravity (SG), hyaline casts, and proteinuria. Diastolic dysfunction will present with normal EF and decreased CO; systolic dysfunction will present with decreased EF and decreased CO. SBP is increased in diastolic dysfunction. TABLE 384 CONGESTIVE HEART FAILURE (CHF) Congestive Heart Failure (CHF) Etiology

Most common cause of CHF is following MI and the resultant ischemic changes. Cardiomyopathy, sarcoidosis or hemochromatosis, pulmonary HTN or aortic regurgitation. Structural abnormalities, congenital heart disease, CAD, constrictive pericarditis, RCM,

Diastolic dysfunction

Decreased CO due to elevated SBP with normal EF. Decrease in EF due to decreased SV leads to a decrease in CO in the absence of any change in HR

Systolic dysfunction

Decreased contractility leading to decreased EF. Decreased EF leads to decreased SV, which in the absence of changes to HR, leads to decreased CO.

Pericarditis

Dressler syndrome.

Cardiovascular compensation

Decreases in SV causes increase in the HR. The heart increases stretch, leading to ventricular dilation and thereby increasing EDV. Myocardial hypertrophy results from the increased pressures, and after some time, the heart decompensates, and any further dilation only leads to worsening CHF. A systemic compensation can also occur by way of decreased activation of stretch receptors in the heart and carotid artery, leading to increased systemic vascular resistance, increased afterload, and decreased SV.

Renal compensation

RAA system is activated, leading to vasoconstriction & retention of sodium chloride & water. Increased volume load leads to an increase in preload, which worsens the congestion as the heart is unable to compensate for the increased stretch by increasing its contractility.

Presentation

RHF-- hepatic congestion, hepatomegaly, RUQ pain, JVD, ascites, peripheral edema, cyanosis, HJR. LHF-- DOE, PND, orthopnea, rales, nocturia, diaphoresis, an S3 gallop, and tachycardia Both RHF & LHF--ankle edema, white sputum with flecks of blood, & cardiomegaly. Dyspnea, sudden complaints of SOB.

Diagnosis

CXR demonstrating cardiomegaly, congestion of pulmonary vasculature, Kerley B lines, pulmonary effusion. Echocardiography, EF can also be evaluated using a MUGA scan. A UA will demonstrate oliguria, increased SG, hyaline casts, and proteinuria. Diastolic dysfunction will present with normal EF and decreased CO; systolic dysfunction will present with decreased EF and decreased CO. SBP is often increased in diastolic dysfunction.


316

USMLE STEP 2

TREATMENT OF DIASTOLIC DYSFUNCTION Diastolic dysfunction is typically treated with surgical options as medical options such as diuretics and negative inotropes have limited utility. Diuretics are recommended in situations of volume overload, while beta-blockers and calcium-channel blockers are used decrease the inotropic activity of the heart. Digoxin and vasodilators are not used. The main treatment for CHF is to decrease preload through sodium restriction, furosemide diuresis, and venodilation with nitrates. Afterload reduction is typically accomplished with ACE inhibitors. The Caucasian population responds best to ACE inhibitors and beta blockers, while African Americans respond best to calcium-channel blockers. The elderly respond better to thiazides, and diabetics respond better to ACE inhibitors.

TREATMENT OF SYSTOLIC DYSFUNCTION The cornerstones of treatment involve decreased cardiac oxygen demand, improving cardiac function, and reducing vascular load. Systolic dysfunction is primarily treated by reversing dietary trends such as excessive salt intake, and decreasing major stressors (physical and mental) for the patient. Medical treatment is essential, and is composed of diuretics and vasodilators such as ACEinhibitors and hydralazine or nitrates. Beta-blockers such as carvedilol or metoprolol are used. Other agents that are often added to manage systolic dysfunction include spironolactone (which reduces mortality), digoxin, dobutamine, and amrinone (which decrease symptoms but have no effect on mortality), and ARBs. Surgery and heart transplant are the final options when all other recourses are exhausted. TABLE 385 TREATMENT OF DYSFUNCTIONS Treatment of Dysfunctions Diastolic

Surgery, diuretics, beta-blockers and calcium-channel blockers, decrease preload through sodium restriction, furosemide diuresis, and venodilation with nitrates, ACE inhibitors.

Systolic

Decreased cardiac oxygen demand, improving cardiac function, and reducing vascular load. Decrease salt intake, and decreasing major stressors. Diuretics and vasodilators, beta-blockers. Spironolactone, digoxin, dobutamine, and amrinone, and ARBs. Surgery and heart transplant are the final options.

COMPLICATIONS PULMONARY EDEMA (APE) Acute pulmonary edema is the result of cardiac decompensation leading to fluid collection in the perivascular, peribronchial, and alveolar spaces. It is commonly the result of CHF, arrhythmia, MI, severe HTN, PE, adult respiratory distress syndrome (ARDS), uremia, shock, anaphylaxis, and drug reactions. APE presents with tachypnea, pink sputum discharged with cough, cyanosis, dyspnea, rales, ronchi, wheezing, and crackles. CXR indicates Kerley B lines, pulmonary effusion, Cardiomegaly, and prominent, congested pulmonary vessels. Early ABGs indicate respiratory alkalosis, and later evolve into respiratory acidosis. Treatment of pulmonary edema involves nitroglycerin for preload reduction, oxygen with positive end expiratory pressure (PEEP), morphine, aspirin, and diuretics to further reduce preload. Amrinone may be used to increase inotropy and cause vasodilation, dobutamine can be used to further increase inotropy, and dopamine to cause an inotropic action with vasoconstriction.

www.ClinicalReview.com


Cardiovascular System

317

TABLE 386 PULMONARY EDEMA (APE) Pulmonary Edema (APE) Etiology

Commonly the result of CHF, arrhythmia, MI, severe HTN, PE, ARDS, uremia, shock, anaphylaxis, and drug reactions.

Presentation

Tachypnea, pink sputum discharged with cough, cyanosis, dyspnea, rales, ronchi, wheezing, and crackles.

Diagnosis

CXR indicating Kerley B lines, pulmonary effusion, cardiomegaly. ABG.

Treatment

Nitroglycerin, PEEP, morphine, aspirin, diuretics, amrinone, dobutamine, dopamine.

PAROXYSMAL NOCTURNAL DYSPNEA (PND) PND is the development of difficulty in breathing that awakens a person. During night, the volume of blood that was redistributed to the dependent extremities begins to flow superiorly to the head and chest. This increased fluid load is initially handled by the heart as it stays on the Frank-Starling curve, but the heart in CHF is already at its limits in terms of how much more it can increase contractility in response to increased EDV. The heart falls off the Frank-Starling curve, contractility ceases to keep pace with EDV, and the volume backs up into the lungs. Over a few hours, increasing pulmonary congestion occurs leading to pulmonary edema and impaired oxygen exchange. Hypoxia and dyspnea are the result. PND is treated by having the patient stand up, walk a few paces, and attempt to sleep sitting up. Standard CHF therapies are also used such as vasodilators and volume reducers. TABLE 387 PAROXYSMAL NOCTURNAL DYSPNEA (PND) Paroxysmal Nocturnal Dyspnea (PND) Etiology

At night, blood begins to flow superiorly to the head and chest. Contractility ceases to keep pace with EDV, and the blood backs up into the lungs.

Presentation

Hypoxia, dyspnea wakens patient.

Treatment

Stand up, walk, sleep sitting up, vasodilators, volume reducers.

VALVULAR HEART DISEASE INTRODUCTION Valvular heart disease is a significant cause of morbidity and mortality. However, with advances in the technique and timing of surgical interventions, patients are experiencing improved outcomes after being diagnosed and treated. The standard for diagnosis and assessment is echocardiography. Asymptomatic patients are typically observed on a regular basis, while those with clinical functional abnormalities may receive surgery to correct the defect in cardiac function. Valvular heart disease incorporates deficits in mitral stenosis (MS), mitral regurgitation (MR), mitral valve prolapse (MVP), aortic stenosis (AS), aortic regurgitation (AR), tricuspid stenosis (TS), and tricuspid regurgitation (TR). Pulmonary valve disease is similar to that for aortic valve deficits with the appropriate corrections made for the right side of the heart. Rheumatic heart disease (RHD) is also discussed due to the immunologic effects of rheumatic fever (RF) on the valves of the heart.

Â


318

USMLE STEP 2

MITRAL STENOSIS ETIOLOGY AND PATHOPHYSIOLOGY Mitral stenosis is the development of a narrowing between the left atrium and left ventricle. Abnormalities of the valvular leaflets are the typical cause, and most cases are secondary to RF and subsequent RHD. The majority of patients are female, and congenital causes of MS are rare. Deficits in the valve leaflets and narrowing of the outflow tract lead to impaired LV function (LVF). There is increased pressure in the LA as a result of outflow tract failure, and this causes pulmonary edema and finally RHF.

PRESENTATION MS presents over a period of time with onset of cough, DOE, hemoptysis, RHF with ventricular failure, hoarseness from laryngeal nerve impingement by the expanding LA, and thromboembolic phenomenon in the systemic vasculature due to hemostasis in the LA. Orthopnea, PND, fatigue, hepatomegaly, ascites, and peripheral edema are other symptoms and are especially likely in the later stages of MS. On physical exam, a significant precordial thrust can be palpated with a sternal lift, atrial fibrillation may be present on EKG, and auscultation may yield a loud S1 with an opening snap. Decreased pulse pressure and a typical apical diastolic rumble may also be present.

DIAGNOSIS Physical exam signs of an apical diastolic murmur with an S1 that has an opening snap is indicative of MS. A CXR may indicate a straight left edge of the heart due to LAH, and Kerly B lines may be present as a result of the pulmonary congestion; a large pulmonary artery and pulmonary HTN may also be indicated on CXR. EKG typically indicates RVH, atrial fibrillation, and LAH. Echocardiography demonstrates thickened mitral valve leaflets and LAH.

TREATMENT Treatment of MS starts with prophylaxis for endocarditis by beginning regimens typically used for CHF, controlling any arrhythmia that is present, beginning anticoagulation therapy (especially in the presence of atrial fibrillation), and considering balloon valvuloplasty or open surgical repair with significantly narrowed outflow tracts. Balloon valvuloplasty is more successful in MS than in AS. Specific medications include digitalis, diuretics, and warfarin. TABLE 388 MITRAL STENOSIS (MS) Mitral Stenosis (MS) Etiology

Development of a narrowing between LA and LV. Abnormalities of the valvular leaflets are the typical cause, and most cases are secondary to RF and subsequent RHD.

Presentation

Onset of cough, DOE, hemoptysis, RHF with ventricular failure, hoarseness, and thromboembolic phenomenon in the systemic vasculature. Orthopnea, PND, fatigue, hepatomegaly, ascites, and peripheral edema are other symptoms and are especially likely in the later stages.

Diagnosis

Apical diastolic murmur. CXR, a large pulmonary artery and pulmonary HTN. EKG typically indicates RVH, atrial fibrillation, and LAH. Echocardiography demonstrates thickened mitral valve leaflets and LAH.

Treatment

Prophylaxis for endocarditis, controlling any arrhythmia that is present, beginning anticoagulation therapy, and considering balloon valvuloplasty or open surgical repair.

www.ClinicalReview.com


Cardiovascular System

319

MITRAL REGURGITATION (MR) ETIOLOGY MR is the result of ischemic changes leading to dysfunction of the papillary muscles, sudden rupture of the chordae tendineae, damage secondary to RHD, progressive changes from MVP, endocarditis, HCM, congenital defects, and severe LV dilation. MR leads to the return of blood during systole from the LV to the LA. MR is more common in men.

PATHOPHYSIOLOGY MR leads to a compromise of systolic function by causing the left ventricle to exert more energy than necessary against a greater volume load in order to pump blood throughout the systemic circulation. Due to a relative patency in the mitral valve, part of the SV pumped during systole is transmitted into the LA instead of into the systemic circulation. Preload increases, and with the increase in EDV, there is an increase in myocardial contractility, abnormally high EF to compensate for part of the volume going to the LA, and eventual damage to the myocardium from the abnormally high work load as the MR continues to worsen.

PRESENTATION MR presents with the development of thromboemboli from the relative stasis in the LA, dyspnea, fatigue and weakness, orthopnea, PND, and RHF with pulmonary HTN. A displaced point of maximal impulse (PMI) with a palpable thrill and diminished carotid upstroke, a holosystolic apical murmur that radiates to the axilla, an S3 from rapid LV filling with a split S2 that worsens with inspiration, and JVD.

DIAGNOSIS Diagnosis of MR includes the presence of the physical symptoms, LAH on CXR and EKG, and valvular dysfunction on echocardiogram. LA overload is obvious on catheterization.

TREATMENT Medical treatment is typically of low utility, but is the only option available until surgical correction can be achieved. The goal of treatment is to maintain LV function, which heralds a good prognosis. To this extent, diuretics are used to decrease preload, anticoagulation is used to minimize thromboembolic phenomenon, vasodilators such as ACE inhibitors are used to decrease afterload and attempt to shunt the blood away from the bicuspid valve, and digitalis is occasionally used as an inotropic agent. Replacement of the valve is done as soon as possible. Criteria for surgery include symptomatic heart failure due to severe MR. TABLE 389 MITRAL REGURGITATION (MR) Mitral Regurgitation (MR) Etiology

Ischemic changes leading to dysfunction of the papillary muscles, sudden rupture of the chordae tendineae, damage secondary to RHD, progressive changes from MVP, endocarditis, HCM, congenital defects, and severe LV dilation.

Presentation

Development of thromboemboli, dyspnea, fatigue & weakness, orthopnea, PND, and RHF with pulmonary HTN.

Diagnosis

Presence of physical symptoms, LAH on CXR and EKG, and valvular dysfunction on echocardiogram. LA overload is obvious on catheterization.

Treatment

Medical treatment is the only option available until surgical correction can be achieved. Diuretics, anticoagulation, vasodilators, and replacement of the valve.

Â


320

USMLE STEP 2

MITRAL VALVE PROLAPSE (MVP) ETIOLOGY MVP is most commonly seen in Marfan disease and other connective tissue diseases. It is also common in women, RHD, IHD, and ASD. MVP is a congenital valve defect that leads to mild regurgitation.

PRESENTATION AND DIAGNOSIS MVP is typically asymptomatic, but may occasionally present with atypical chest pain, a mid-systolic click, lightheadedness, syncope, palpitations, fatigue, SOB, and the development of arrhythmias. A late, high-pitched systolic murmur is evident and loudest at the apex, and the S2 is widely splitting. Diagnosis is made with echocardiography, and demonstrates displacement of the bicuspid valve leaflets and subsequent meeting at some point distal to the valve opening.

TREATMENT As most cases are asymptomatic, little treatment is required. Medical management is necessary with increasing MR and includes those agents similar to that of MR. Chest pain and arrhythmias are controlled with beta-blockers, and prophylaxis for endocarditis is started in all individuals. Surgical replacement is rarely required. Close observation is required to avoid complications leading to CHF, infective endocarditis, valvular calcification, transient ischemic attacks (TIAs), and sudden cardiac death. TABLE 390 MITRAL VALVE PROLAPSE (MVP) Mitral Valve Prolapse (MVP) Etiology

Most commonly seen in Marfan disease and other connective tissue diseases. RHD, IHD, and ASD. MVP is a congenital valve defect that leads to mild regurgitation.

Presentation

Typically asymptomatic, but may present with atypical chest pain, a mid-systolic click, lightheadedness, syncope, palpitations, fatigue, SOB, and the development of arrhythmias.

Diagnosis

Echocardiography demonstrates displacement of the bicuspid valve leaflets and subsequent meeting at some point distal to the valve opening.

Treatment

Chest pain and arrhythmias are controlled with beta-blockers, and prophylaxis for endocarditis is started in all individuals. Close observation is required to avoid complications.

AORTIC STENOSIS (AS) ETIOLOGY AS is the result of a congenital disorder, abnormal calcification of the valve leaflets in an otherwise normal adult, presence of a bicuspid valve that predisposes to fibrosis and calcification, or secondary to RHD.

AS presents with syncope, angina, and dyspnea on exertion. Triad leads to CHF Æ 1 year survival.

PATHOPHYSIOLOGY AS leads to increased afterload due to difficulty with transmitted blood through the aortic orifice. In order to avoid the decrease in SV that occurs as a result of the afterload elevation, there is increased contractility of the ventricle in order to maintain CO. www.ClinicalReview.com


Cardiovascular System

321 Hypertrophic cardiomyopathy (HCM) occurs. Until cardiac decompensation takes place, SV is maintained and CO remains normal. Unfortunately, the hypertrophy that occurs in the ventricle leads to increased oxygen demand, but this is confounded by the elevated pressures within the ventricle that decrease oxygen supply. AS can therefore lead to angina through ischemia. Subsequent failure of the LV can lead to backflow congestion into the lungs and cause pulmonary edema.

PRESENTATION The high pressures generated within the LV lead to an S4 gallop. Syncope, angina, and DOE are all symptoms of AS. Angina and syncope are particularly noted for being worse with exertion. This triad eventually leads to CHF. None of these symptoms occur until late in the course of AS – the early presentation is asymptomatic. Survival after development of angina is typically about five years; after syncope it is about 3 years; and after heart failure, it is typically about one year.

DIFFERENTIAL DIAGNOSIS The differential diagnosis of AS includes HCM, MR, and pulmonary stenosis (PS). HCM presents with Q waves on EKG, has a bifid carotid upstroke, and has a murmur that decreases with squatting and increases with the Valsalva maneuver. MR presents as a holosystolic murmur that radiates to the axilla. PS presents as a murmur that worsens with inspiration and is loudest along the left upper sternal border (LUSB). Echocardiography is the definitive procedure to distinguish all of these causes. With regard to maneuvers, the Valsalva maneuver decreases the murmur of AS but increases that of HCM; squatting increases the murmur of AS but decreases that of HCM.

DIAGNOSIS Diagnosis of AS is made with physical signs of an aortic ejection click, harsh systolic ejection murmur (SEM) leading to a palpable thrill, especially at the PMI, and S4 gallop, split S2, narrow pulse pressure, and a carotid thrill. The SEM may radiate to the carotids. On echocardiography, a pulsus tardus et parvus may be present – a specific waveform of blood flow velocity that occurs distal to the AS and has a prolonged early systolic acceleration and flattening of the systolic peak (a late pulse that is wider than normal). EKG changes indicate LV stain, and echocardiography demonstrates the damaged aortic valve. CXR can detect calcification of the aortic valve, along with cardiomegaly and pulmonary HTN.

TREATMENT AS requires endocarditis prophylaxis. The standard therapy, as defined by the American Heart Association, for dental procedures, esophageal procedures, and pulmonary procedures is to administer amoxicillin one hour prior to the procedure. Patients who cannot take oral medications are given ampicillin. Those who are allergic to penicillins are given clindamycin, cephalosporins, or azithromycin. Prior to gastrointestinal or genitourinary (GU) procedures, ampicillin and gentamicin should be given; if the patient is allergic to penicillins, vancomycin plus gentamycin should be administered. Surgery via balloon valvuloplasty to correct the aortic valve defect is done with severe symptoms, but the rate of restenosis is very high and may eventually require replacement of the entire valve. Patients should be advised to avoid exercise.


322

USMLE STEP 2

TABLE 391 AORTIC STENOSIS (AS) Aortic Stenosis (AS) Etiology

A congenital disorder. Abnormal calcification of the valve leaflets in an otherwise normal adult, presence of a bicuspid valve that predisposes to fibrosis and calcification, or secondary to RHD.

Presentation

S4 gallop. Syncope& angina (worse with exertion), and DOE, CHF. Symptoms occur late in the course of AS

Diagnosis

Physical signs of an aortic ejection click, harsh SEM leading to a palpable thrill, narrow pulse pressure, and a carotid thrill. On echocardiography, a pulsus tardus et parvus. EKG changes indicate LV stain, and echocardiography demonstrates the damaged aortic valve. CXR can detect calcification of the aortic valve, along with cardiomegaly and pulmonary HTN.

Treatment

Endocarditis prophylaxis. Prior to GI or GU procedures, ampicillin and gentamicin should be given Surgery via balloon valvuloplasty to correct the aortic valve defect is done with severe symptoms, but the rate of restenosis is very high and may eventually require replacement of the entire valve. Patients should be advised to avoid exercise.

AORTIC REGURGITATION (AR) ETIOLOGY AR is commonly the result of RHD, but may also occur in infective endocarditis, in dilations of the aortic root due to HTN, collagen vascular disease (CVD), or Marfan syndrome, in proximal dissections of the aortic root (as in cystic medial necrosis), syphilis, pregnancy, Turner syndrome, in conditions that affect the ascending aorta (such as aortic dissection), ankylosing spondylitis, and trauma.

PATHOPHYSIOLOGY AR leads to its effects through volume overload of the LV. The increased fluid load leads to increased EDV, which in turn increases preload and causes increased myocardial contractility in order to meet the same EF. This attempt to maintain SV and CO leads to overstretching of the myofibrils, and eventually, decompensation by the LV, as it can no longer stretch enough to have a sufficiently forceful contraction. Acute presentations of AR present with very high LV EDP due to lack of sufficient compensation by the ventricle to generate the necessary contractile force. Sudden AR can even lead to unexpected cardiac death by total heart decompensation and cessation of contractions due to fatal EDP. Acute AR leads to decreased CO and a narrow aortic pulse pressure (PP).

PRESENTATION AR presents with dyspnea, orthopnea, and PND. Angina is common from decreased circulation and increased oxygen demand in the myocardium. There is typically a wide pulse pressure, a bounding pulse known as Corrigan pulse, a bisferiens pulse with a dicrotic pulse (two fluid waves), and a rapid femoral pulse (referred to as a pistol-shot pulse). Compression of the femoral artery leads to a bruit, known as Duroziez sign; the Hill sign is SBP that is higher in the lower extremities (LE) than the upper extremities (UE); Quincke sign is fingernail color that changes with the heartbeat; and De Musset sign is bobbing of the head in synchrony with the HR. Other manifestations include a diastolic decrescendo murmur, a systolic flow murmur, and S3, and an Austin-Flint murmur ( in which blood enters the LV simultaneously from the aorta due to AR),and from the mitral valve (normal flow).

www.ClinicalReview.com


Cardiovascular System

323

DIAGNOSIS Diagnosis of AR is made by physical exam (in which a blowing diastolic murmur is worsened by leaning the patient forward). A2 is accentuated, the PMI is displaced laterally and inferiorly, EKG demonstrates LVH with narrow Q waves in the left precordial leads, and echocardiography demonstrates regurgitation. CXR demonstrates LVH and aortic dilation.

TREATMENT Treatment of AS involves prophylaxis against endocarditis, treating LV failure with preload and afterload reduction, digitalis for positive inotropy, and valve replacement with cardiac decompensation. TABLE 392 AORTIC REGURGITATION (AR) Aortic Regurgitation (AR) Etiology

Commonly the result of RHD, also occurs in infective endocarditis, in dilations of the aortic root due to HTN, CVD, or Marfan syndrome, in proximal dissections of the aortic root, syphilis, HTN, CVD, pregnancy, and Turner syndrome, and in conditions that affect the ascending aorta, ankylosing spondylitis, and trauma.

Presentation

Dyspnea, orthopnea, and PND, angina, a wide pulse pressure, bounding, bisferiens, and rapid femoral pulses. Duroziez sign; Hill sign; Quincke; De Musset sign.

Diagnosis

Physical exam in which a blowing diastolic murmur is worsened by leaning the patient forward. EKG demonstrates LVH. Echocardiography demonstrates regurgitation. CXR demonstrates LVH and aortic dilation.

Treatment

Prophylaxis against endocarditis, treating LV failure with preload and afterload reduction, digitalis for positive inotropy, and valve replacement with cardiac decompensation.

TRICUSPID STENOSIS (TS) ETIOLOGY AND PRESENTATION TS is commonly secondary to RHD, carcinoid, and congenital malformations. It presents with JVD, peripheral edema, and signs of hepatic congestion leading to hepatomegaly, ascites, and jaundice.

DIAGNOSIS AND TREATMENT TS is diagnosed as a low-pitched, rumbling, diastolic murmur. A thrill is palpable at the LLSB, and there is a RV thrust present. Treatment requires surgical repair. TS is distinguished from MS in that TS worsens with inspiration. TABLE 393 TRICUSPID STENOSIS (TS) Tricuspid Stenosis (TS) Etiology

Commonly secondary to RHD, carcinoid, and congenital malformations. It presents with JVD, peripheral edema, and signs of hepatic congestion leading to hepatomegaly, ascites, and jaundice.

Diagnosis

Low-pitched, rumbling, diastolic murmur. A thrill is palpable at the LLSB, and there is a RV thrust present. TS is distinguished from MS in that TS worsens with inspiration.

Treatment

Treatment requires surgical repair.

Â


324

USMLE STEP 2

TRICUSPID REGURGITATION (TR) ETIOLOGY AND PRESENTATION TR is the result of LHF or mitral valve deficits leading to increased pressure from the pulmonary artery. Stretching of the RV, for example from RHF or infarction, can also lead to TR from failure of leaflet coaptation, and direct lesions to the valve leaflets can account for TR. It presents with signs and symptoms of liver congestion, JVD, and RHF.

DIAGNOSIS AND TREATMENT TR is a holosystolic, blowing murmur loudest along the LLSB. Like TS, it worsens with inspiration. EKG signs indicate RVH, and atrial fibrillation tends to occur with TR. As RHF is most commonly due to LHF, treating LHF is usually the treatment of choice. Preload reduction should be attempted and finally surgical repair. Any diseased valve should herald the use of endocarditis prophylaxis to avoid infective endocarditis. TABLE 394 TRICUSPID REGURGITATION Tricuspid Regurgitation Etiology

The result of LHF or mitral valve deficits leading to increased pressure from the pulmonary artery. Stretching of the RV. Presents with signs and symptoms of liver congestion, JVD, & RHF.

Diagnosis

Holosystolic, blowing murmur loudest along the LLSB. Worsens with inspiration. EKG signs indicate RVH, and atrial fibrillation.

Treatment

Treat as LHF. Preload reduction, surgery, endocarditis prophylaxis.

TABLE 395 SUMMARY DIAGNOSES OF MURMURS Summary Diagnoses of Murmurs Diagnosis

Murmur

S1

S2

Clinical Findings

Mitral stenosis

Apical diastolic rumble

Loud with opening snap

Normal

Murmur worsens with exercise

Mitral regurgitation

Holosystolic murmur

Soft

Split S2

S3 with strong carotid upstroke

Mitral valve prolapse

Mid systolic murmur with click

Normal

Normal

Murmur worsens when standing

Aortic stenosis

Mid systolic murmur, soft when severe

Normal

Paradoxical split

S3, S4, and diminished carotid upstrokes

Aortic regurgitation

Blowing diastolic murmur

Soft

Normal

Wide pulse pressure, SBP elevated

Pulmonary stenosis

Systolic

Single

Prominent “a” wave in the JVP, right ventricular lift, ejection click.

Pulmonary regurgitation

Inaudible in the absence of pulmonary HTN

Widened splitting

Usually related to the underlying disease process.

Tricuspid stenosis

Murmur upon inspiration

Tricuspid regurgitation

Systolic

Louder at lower left sternal border

Large jugular “a” wave with a slow Y descent. Often occurs with RHD. Atrial enlargement, ventricular hypertrophy, echocardiography is descriptive.

www.ClinicalReview.com


Cardiovascular System

325

RHEUMATIC FEVER (RF) ETIOLOGY Rheumatic fever (RF) is due to a streptococcal pharyngitis leading to an autoimmune process. RF can develop into rheumatic heart disease (RHD) leading to deposition of immune complexes on valve leaflets and subsequent damage to the anatomy of the heart. The mitral valve is most commonly affected, but the tricuspid and aortic valves are also occasionally damaged.

DIAGNOSIS Rheumatic fever is diagnosed by serology that finds positive antistreptolysin-O (ASLO) titers and an elevated erythrocyte sedimentation rate (ESR). There are major criteria and minor criteria for diagnosis; two major criteria, or one major and two minor criteria must be present for the diagnosis of RF. The major criteria include myocarditis or other inflammation of the heart, migratory arthritis affecting multiple joints, erythema marginatum, Sydenham chorea, and subcutaneous nodules. Minor criteria include fever, elevated ESR, arthralgia, history of streptococcus pharyngitis, and a prolonged PR interval on EKG.

TREATMENT RHD is treated with penicillin to eradicate any continuing infection by streptococcus and resolve existing pharyngitis. ASA is given for relief from arthritis and general pain relief, and steroids are used to emergently reduce the inflammation to the heart tissues. Chronic RHD is treated with continuing penicillin treatment with medications and surgery as indicated. Valve replacement may become necessary with continuing RHD. Anyone with a history of RF requires prophylaxis prior to undergoing any dental or GI procedures. TABLE 396 RHEUMATIC FEVER (RF) Rheumatic Fever (RF) Etiology

Due to a streptococcal pharyngitis leading to an autoimmune process. Can develop into RHD leading to deposition of immune complexes on valve leaflets and damage to the heart anatomy.

Diagnosis

Positive ASLO titers and an elevated ESR. One major and two minor criteria must be present for the diagnosis of RF. Major criteria: myocarditis or other inflammation of the heart, migratory arthritis affecting multiple joints, erythema marginatum, Sydenham chorea, and subcutaneous nodules. Minor criteria: fever, elevated ESR, arthralgia, history of streptococcus pharyngitis, and a prolonged PR interval on EKG.

Treatment

Penicillin, ASA, and steroids. Chronic RHD is treated with medications and surgery. Valve replacement may be necessary.

CARDIOMYOPATHY INTRODUCTION Cardiomyopathy is disease of the myocardium leading to deficits in function. There are numerous distinct causes, but most cases of primary cardiomyopathy can be attributed to HTN, valvular heart disease, CAD, or congenital defects. Cardiomyopathy is divided into three main types, known as dilated cardiomyopathy (DCM), hypertrophic cardiomyopathy (HCM), and restrictive cardiomyopathy (RCM). DCM is sometimes referred to as congestive cardiomyopathy, and is the most common form. Myocarditis is also discussed at the end of this chapter.

Â


326

USMLE STEP 2

DILATED CARDIOMYOPATHY (DCM) ETIOLOGY DCM is the enlargement of the RV or LV with the loss of normal contractility. The end result is CHF, arrhythmia, and a predisposition towards thromboembolic phenomenon. It typically involves both ventricles, and is the most common cause of cardiac transplant. Causes of DCM include viral infections, alcohol abuse, cocaine abuse, heavy metal poisoning, doxorubicin poisoning, endocrine disease (such as hypothyroidism or hyperthyroidism), pheochromocytoma, CTD, glycogen storage disease (GSD), neuromuscular disease (DMD), pregnancy, metabolic disorders (such as hypocalcemia and hypophosphatemia), inherited disorders (such as Fabry disease or Gaucher disease), and genetic predilections.

PATHOPHYSIOLOGY Dilation of the ventricles leads to regurgitation through failure of coaptation of the valve leaflets. The combination of poor cardiac contraction, decreased EF, increased preload, and poor valvular function leads to CHF. Relative stasis of blood predisposes individuals to developing thromboembolisms. Finally, the dilation may impinge upon the conduction system and lead to arrhythmia. DCM is distinguished with decreased CO due to decreased SV and EF, an increase in ventricular filling pressure, increase in ventricular volume, and decreased compliance.

PRESENTATION AND DIAGNOSIS DCM presents as heart failure. Angina is typically present due to increased oxygen demand and poor supply. Diagnosis of DCM is by CXR that indicates cardiomegaly and pulmonary edema, EKG that indicates LVH with LBBB or RVH with RBBB, and confirmation with echocardiography that demonstrates a dilated ventricle, wall motion abnormalities, and valve regurgitation. Physical exam often yields an S3 and S4 murmur, rales, and regurgitation murmurs.

TREATMENT DCM is symptomatically treated as CHF, with preload and afterload reduction and volume reduction through diuretics and vasodilators, and positive inotropic agents such as digoxin. Pacemaker implantation may be necessary with arrhythmia, especially if rhythm suppressants such as procainamide and quinidine fail. The definitive treatment is heart transplant. All patients require lifelong anticoagulation. TABLE 397 DILATED CARDIOMYOPATHY (DCM) Dilated Cardiomyopathy (DCM) Etiology

Viral infections, alcohol abuse, cocaine abuse, heavy metal poisoning, doxorubicin poisoning, endocrine disease, pheochromocytoma, CTD, GSD, DMD, pregnancy, metabolic disorders, inherited disorders, and genetic predilections.

Diagnosis

Presents as heart failure & angina. Diagnosis: CXR that indicates cardiomegaly and pulmonary edema, EKG that indicates LVH with LBBB or RVH with RBBB, echocardiography that demonstrates a dilated ventricle, wall motion abnormalities, and valve regurgitation. Physical exam: often yields murmur, rales, and regurgitation murmurs.

Treatment

Preload, afterload, and volume reduction through diuretics, vasodilators, and positive inotropic agents. Pacemaker (with arrhythmia). Heart transplant. Lifelong anticoagulation therapy.

www.ClinicalReview.com


Cardiovascular System

327

HYPERTROPHIC CARDIOMYOPATHY (HCM) ETIOLOGY HCM is the thickening in the heart wall with narrowing of the intraventricular (IV) septum and sporadic obstruction of the outflow tract. Causes of HCM include generally idiopathic conditions (about half of all cases), and genetic causes (the remainder of all cases). The genetic defects are typically on chromosome 14 with the familial form, and autosomal dominant (AD) with variable penetrance. HCM is sometimes referred to as idiopathic hypertrophic subaortic stenosis (IHSS).

PATHOPHYSIOLOGY HCM leads to decreased ventricular compliance but an increase in CO, as the heart function moves along the Frank-Starling curve due to volume changes. Diastolic dysfunction may occur as the heart is unable to relax. HCM is exacerbated with inotropic agents, tachycardia, preload reduction, and afterload reduction. HCM is improved with B-blockers, calcium-channel blockers, preload increases, afterload increases, and alpha-adrenergic stimulation. HCM is distinguished by normal CO due to increased SV and EF, an increase in EDP, and a decrease in chamber size. HCM has decreased compliance.

PRESENTATION HCM presents with syncope and angina, but may proceed directly to sudden cardiac death. Syncope often occurs following exercise, arrhythmia, and CHF. Angina may occur at rest, is not responsive to nitrates, but may improve by lying down. Angina may be due to outflow obstruction, but the precise cause remains to be elucidated. It is the development of arrhythmia that leads to sudden death in HCM. Clinical findings may include palpitations, paradoxical S2 splitting, bifid carotid pulse, S4 gallop, SEM along the LLSB that decreases with squatting and increases with exercise, decreased LV EDV, and regurgitation murmurs.

DIAGNOSIS HCM diagnosis is by EKG (which indicates arrhythmia including PVCs), atrial fibrillation, Q waves, ST changes, and T wave changes. Echocardiography is definitive with septal hypertrophy, LVH, reduced LV EDV, and midsystolic aortic valve closure. A CXR indicates LVH with a dilated LA.

TREATMENT HCM is treated with arrhythmia suppressants such as amiodarone, beta-blockers decrease HR and permit increased filling time, increasing ventricular space with septal myomectomy, replacement of the mitral valve to reduce obstruction, pacemaker or defibrillator implantation, and avoiding exercise. Heart transplantation is required in most cases. HCM with a resting obstruction is often medically treated with dipyramide, beta-blockers, and calcium-channel blockers; surgical interventions replace the mitral valve or do a ventriculomyotomy. HCM with latent obstruction is treated first with beta-blockers, then by calcium-channel blockers and disopyramide; mitral valve replacement is the surgical procedure of choice. HCM without obstruction is treated with calcium-channel blockers and beta-blockers, and surgery is usually not required.

Â


328

USMLE STEP 2

TABLE 398 HYPERTROPHIC CARDIOMYOPATHY (HCM) Hypertrophic Cardiomyopathy (HCM) Etiology

Idiopathic conditions & genetic causes. The genetic defects are typically on chromosome 14, and AD with variable penetrance.

Presentation

Syncope & angina, but may proceed directly to sudden cardiac death.

Diagnosis

EKG indicates PVCs, atrial fibrillation, Q, ST, and T wave changes. Echocardiography is definitive with septal hypertrophy, LVH, reduced LV EDV, and midsystolic aortic valve closure. A CXR indicates LVH with dilated LA.

Treatment

Arrhythmia suppressants, beta-blockers, septal myomectomy, replacement of the mitral valve and avoiding exercise. Heart transplantation is required in most cases.

RESTRICTIVE CARDIOMYOPATHY (RCM) ETIOLOGY RCM is the result of fibrosis and infiltrative changes to the myocardium, leading to decreased compliance and filling of the ventricles. Causes of RCM include fibroelastosis, hypereosinophilia, amyloidosis, sarcoidosis, hemochromatosis, carcinoid, and cancer. Primary endocardial fibroelastosis is most likely to affect infants and has a thickened aortic and mitral valve with cardiac dilation and hyperplasia. Endomyocardial fibrosis is more common in Africa. Lรถffler endocarditis typically follows inflammation of the arteries and has an eosinophilia associated with it. Scleroderma (and the CREST syndrome), radiation exposure, GSD, and Becker disease are other causes of RCM. Becker disease occurs mostly in southern Africa with fibrosis of the papillary muscle, concomitant necrosis of the endocardium, and dilation of the ventricles.

PATHOPHYSIOLOGY RCM leads to scarring of the ventricles and thereby decreases compliance. The rigid myocardium retards filling and leads to abnormalities in diastole. While systolic function is also compromised to some extent, RCM is primarily a diastolic disorder. It must be differentiated from constrictive pericarditis (discussed below), and biopsy is usually done for confirmation of RCM.

DIAGNOSIS Upon physical exam, an S3 and S4 gallop with mitral valve regurgitation are evident. Low voltage EKG with ST and T wave changes and conduction changes are present. Echocardiography reveals large atria and thickened ventricular walls; some regurgitation is also present. Biopsy clinches the diagnosis.

TREATMENT RCM is treated with heart transplantation. Medical treatment is effective only in reversible causes such as hemochromatosis (treat with phlebotomy and deferoxamine).

www.ClinicalReview.com


Cardiovascular System

329

TABLE 399 RESTRICTIVE CARDIOMYOPATHY (RCM) Restrictive Cardiomyopathy (RCM) Etiology

Fibroelastosis, hypereosinophilia, amyloidosis, sarcoidosis, hemochromatosis, carcinoid, & cancer. Scleroderma (and the CREST syndrome), radiation exposure, GSD, and Becker disease are other causes of RCM.

Diagnosis

Biopsy clinches the diagnosis. S3 and S4 gallop with mitral valve regurgitation. Low voltage EKG with ST and T wave changes and conduction changes are present. Echocardiography reveals large atria and thickened ventricular walls; some regurgitation is also present.

Treatment

Heart transplant.

MYOCARDITIS ETIOLOGY Myocarditis is the inflammation of the myocardium, commonly due to Coxsackie’s B virus infection. Other viruses that can lead to myocarditis include Coxsackie’s A, CMV, Epstein-Barr virus (EBV), hepatitis B virus (HBV), echovirus, HIV, and adenovirus. Bacterial causes include group A streptococcus (GAS) leading to RF, corynebacterium, meningococcus, Borrelia burgdorferi, and Mycoplasma pneumoniae. Parasitic infections include Chagas disease from Trypanosoma cruzi, Toxoplasma, Trichinella, and Echinococcus. Arteritis from Kawasaki disease, general inflammation, sarcoid, SLE, cocaine abuse, and drug allergies to penicillin and sulfonamides can also lead to myocarditis. Idiopathic causes are the most common.

PATHOPHYSIOLOGY Myocarditis is often associated with pericarditis due to the inflammatory changes taking place. Regurgitation and pericardial friction rubs can occur as a result of the infiltrate of PMNs and other mediators of inflammation and infection; this can also lead to conduction abnormalities. Myocarditis is a progressive disease that requires immediate hospitalization and treatment.

PRESENTATION AND DIAGNOSIS Myocarditis presents with fever, fatigue, chest pain, and syncope. CHF may also be present due to diminished myocardial function. A history of URI may be present. Diagnosis of myocarditis is definitively made by biopsy. Serology raises the clinical suspicion with elevated ESR, WBC count, and cardiac enzyme elevations. Echocardiography indicates wall motion abnormalities, a generally dilated heart, and pericardial effusions. CXR is typically normal, and EKG changes sometimes find ST changes, arrhythmia, and may be low voltage. On physical exam, an S3 and S4 murmur is present along with regurgitation murmurs; a pericardial friction rub may also be heard. Presentation of symptoms prior to 30 years of age heralds a poor prognosis and increased risk of sudden death. Severity of symptoms is not correlated with increased risk of sudden death.

TREATMENT Myocarditis is treated with supportive therapy, antibiotics and antivirals as indicated, and treatment of the heart failure and arrhythmia. The patient should be observed in the ICU. Immunosuppressive agents should not be used. Intravenous immunoglobulin (IVIG) may be beneficial.


330

USMLE STEP 2

TABLE 400 MYOCARDITIS Myocarditis

Etiology

Idiopathic causes are the most common. Viral causes: Coxsackie’s A & B, CMV, EBV, HBV, echovirus, HIV, and adenovirus. Bacterial causes: GAS, corynebacterium, meningococcus, Borrelia burgdorferi, and Mycoplasma pneumoniae. Parasitic infections include Chagas disease, Trichinella, and Echinococcus. Other: Arteritis from Kawasaki disease, general inflammation, sarcoid, SLE, cocaine abuse, and drug allergies.

Diagnosis

Diagnosis of myocarditis is definitively made by biopsy. Fever, fatigue, chest pain, and syncope. History of CHF or URI. Serology raises the clinical suspicion with elevated ESR, WBC count, and cardiac enzyme elevations. Echocardiography indicates wall motion abnormalities, a generally dilated heart, and pericardial effusions. CXR is typically normal.

Treatment

Supportive therapy, antibiotics and antivirals, and treatment of the heart failure and arrhythmia.

PERICARDIAL DISEASE INTRODUCTION Nearly all forms of pericardial disease are accompanied by pericardial effusion. Analysis of the fluid helps distinguish the underlying cause of the disease. A transudative effusion has a low specific gravity and is primarily composed of fluid bereft of proteins and cells; transudative effusion is most common in CHF, excessive hydration, and hypoproteinemia. Exudative effusions are more common with direct injury to the pericardium. Serosanguineous effusions are indicative of TB and cancer, while bright red blood should immediately begin a search for aortic dissection and rupture. Other causes of blood within the pericardium include direct trauma, rupture of the heart several days after MI, and coagulopathies. Rapid accumulation of fluid may lead to pericardial tamponade. TABLE 401 PERICARDIAL DISEASE Pericardial Disease Transudative effusion

Has a low specific gravity and is primarily composed of fluid bereft of proteins and cells. Most common in CHF, excessive hydration, and hypoproteinemia.

Exudative effusion

More common with direct injury to the pericardium

Serosanguineous effusions

Indicative of TB and cancer

Aortic dissection

Bright red blood

PERICARDITIS ETIOLOGY Pericarditis is the inflammation of the pericardium leading to chest pain and a friction rub. Pericarditis is commonly a result of various infectious causes, primary tumors from the breast or lung, a complication of MI, uremia, radiation, Gaucher disease, immunologic disorders, allergic reactions to drugs (such as hydralazine), INH, and procainamide, CVD, thyroid disturbances, trauma, and idiopathic in nature. Recent viral infections can grow into pericarditis; bacterial causes include tuberculosis, streptococci, and staphylococci.

www.ClinicalReview.com


Cardiovascular System

331

PRESENTATION AND DIAGNOSIS Pericarditis presents as substernal, pleuritic chest pain relieved by leaning forward. SLN has no effect on ameliorating the symptoms. Upon auscultation, a pericardial friction rub is commonly heard – it is highly specific to pericarditis, but not always present. EKG changes indicate STe; STe in children is most commonly due to pericarditis. CXR may indicate an enlarged cardiac silhouette due to pericardial effusion. Echocardiography may be used to confirm the effusion.

TREATMENT Pericarditis is treated by symptomatic management, including the use of NSAIDs, and steroids for Dressler syndrome. The underlying etiology should be treated to control progression of pericarditis. TABLE 402 PERICARDITIS Pericarditis

Etiology

Inflammation of the pericardium leading to chest pain & a friction rub. Result of various infectious causes, primary tumors from the breast or lung, a complication of MI, uremia, radiation, Gaucher disease, immunologic disorders, allergic reactions to drugs, INH, and procainamide, CVD, thyroid disturbances, trauma, and idiopathic in nature. Recent viral infections can grow into pericarditis.

Diagnosis

Substernal, pleuritic chest pain relieved by leaning forward. Upon auscultation, a pericardial friction rub is commonly heard – it is highly specific to pericarditis, but not always present. EKG changes indicate STe; CXR may indicate an enlarged cardiac silhouette. Echocardiography may be used to confirm the effusion.

Treatment

NSAIDs and steroids. The underlying etiology should be treated.

PERICARDIAL TAMPONADE ETIOLOGY Tamponade is the result of a rapid pericardial effusion leading to fluid constriction around the heart and impeding normal cardiac function. It is typically the result of severe pericarditis, direct trauma, rupture of the heart wall following an MI, and dissection of the aorta with subsequent rupture. Cardiac filling is impaired, CO reduced, and systemic cardiovascular decompensation can lead to death. Numerous other causes also exist, such as uremia, radiation therapy to the chest, CVD and CTD, infection, and cancer.

PATHOPHYSIOLOGY Pericardial tamponade leads to decreased ventricular volume due to increased external cardiac pressure from the pericardial effusion. The decreased filling leads to decreased SV and CO, with a subsequent drop in SBP. The body attempts to compensate for these changes by increased PVR, increasing HR, increased blood volume, and increasing contractility. However, as the effusion increases, the heart is not able to compensate for these changes and cardiac failure ensues with circulatory collapse.

PRESENTATION Pericardial tamponade presents with Beck’s triad, which includes JVD, muffled heart sounds, and hypotension. Other common symptoms include dyspnea, tachycardia, narrow pulse pressure, fatigue, and orthopnea. Pulsus paradoxus is present with a drop in SBP with inspiration (this finding can also occur in other heart failure causes, asthma, and lung disease).


332

USMLE STEP 2

DIAGNOSIS AND TREATMENT Tamponade is diagnosed by auscultation, a low voltage EKG that shifts over time, CXR that shows enlargement of the cardiac shadow, and echocardiogram that demonstrates the effusion. Pericardiocentesis and surgical drainage are the treatments of choice, and must be done on an emergent basis. TABLE 403 PERICARDIAL TAMPONADE Pericardial Tamponade Etiology

Typically the result of severe pericarditis, direct trauma, ruptures of the heart wall following an MI, and dissection of the aorta with subsequent rupture. Numerous other causes also exist, such as uremia, radiation therapy to the chest, CVD and CTD, infection, and cancer.

Presentation

Beck’s triad, dyspnea, tachycardia, narrow pulse pressure, fatigue, and orthopnea. Pulsus paradoxus, drop in SBP with inspiration.

Diagnosis

Auscultation, a low voltage EKG that shifts over time, CXR that shows enlargement of the cardiac shadow, and echocardiogram that demonstrates the effusion.

Treatment

Pericardiocentesis and surgical drainage.

CONSTRICTIVE PERICARDITIS ETIOLOGY Constrictive pericarditis is the result of scarring and granulation tissue within the pericardium that limits CO. The subsequent diffuse thickening of the pericardium leads to abnormal diastole. Most causes are idiopathic, but open-heart procedures, radiation, and viral infections have all been attributed.

PRESENTATION Constrictive pericarditis presents similar to pericardial tamponade but without the rapid circulatory collapse. Orthopnea and dyspnea are the most common signs. Kussmaul sign with continuing JVP during inspiration is a common sign. A pericardial knock manifesting similar to an S3 gallop is typically present, along with distant heart sounds.

DIAGNOSIS Pericarditis is diagnosed through auscultation of distant heart sounds, calcification within the pericardium evident on CXR, low voltage EKG with inverted T waves in V1 and V2 and notched P waves, CT or MRI demonstrating a thickened pericardium, and confirmation of these findings with echocardiography.

TREATMENT Treatment of pericarditis is to remove the pericardium. Sodium restriction and diuretics typically fail with more severe disease.

www.ClinicalReview.com


Cardiovascular System

333

TABLE 404 CONSTRICTIVE PERICARDITIS Constrictive Pericarditis Etiology

Result of scarring and granulation tissue within the pericardium that limits CO. The subsequent diffuse thickening of the pericardium leads to abnormal diastole. Most causes are idiopathic, but open-heart procedures, radiation, and viral infections have all been attributed.

Presentation

Presents similar to pericardial tamponade but without the rapid circulatory collapse. Orthopnea and dyspnea. Kussmaul sign with continuing JVP during inspiration. A pericardial knock along with distant heart sounds.

Diagnosis

Pericardium evident on CXR, low voltage EKG with inverted T waves in V1 and V2 and notched P waves. CT or MRI demonstrating a thickened pericardium, and confirmation of these findings with echocardiography.

Treatment

Remove pericardium.

ENDOCARDIAL DISEASE INTRODUCTION Endocarditis is most commonly the result of infection by bacteria leading to infection of the endocardium and subsequent cardiac dysfunction and systemic phenomenon. It is especially likely in individuals with a pre-existing heart defect, those with a history of RHD, and in the elderly with a history of calcific AS. Non-infectious endocarditis is the result of iatrogenic damage to a heart valve followed by the development of infectious endocarditis. Rarer causes of endocarditis include complications of systemic lupus erythematosus (SLE) leading to Libman-Sacks endocarditis (LSE), and nonbacterial thrombotic endocarditis (which is typically a late-stage finding in numerous autoimmune disorders), chronic infections, and systemic illnesses.

INFECTIVE ENDOCARDITIS ETIOLOGY Endocarditis is typically the result of an infective process leading to vegetation on the leaflets. Both acute bacterial endocarditis (ABE) and subacute bacterial endocarditis (SBE) can occur. Infective endocarditis is rare, but is increasing in incidence in children with congenital heart defects, especially those with tetralogy of Fallot (TOF), VSD, and AS.

PATHOPHYSIOLOGY The deposition of platelets and fibrin in regions of endothelial injury provide a region for bacteria to implant and multiply. Dental procedures, oral manipulation, esophageal procedures, respiratory procedures, GI procedures, and GU procedures all require prophylaxis as they may inadvertently allow bacteria to enter the blood (bacteremia), which may permit seeding of the damaged endocardium. About 80% of all cases are attributable to gram-positive bacteria such as Streptococcus viridans and Staphylococcus aureus. IV drug abusers and those with indwelling catheters are more likely to have fungal causes such as Candida. Patients with ABE are likely to have had normal, healthy valves and are typically infected with S. aureus. Death is likely within a month unless treatment is obtained. Patients with SBE have a history of heart damage, and are most likely to have S. viridans. The mitral valve is most likely to be affected in these patients. Infection by S. bovis should raise the suspicion for colon cancer. Most infection by S. viridans, group D streptococci, nonenterococcal group D streptococci, HACEK (Haemophilus aphrophilus, Actinobacillus actinomycetemcomitans, Cardiobacterium hominis, Eikenella corrodens, and Kingella kinkae) organisms, and fungi tend to cause subacute disease. Group B streptococcus, Staphylococcus aureus, and Pseudomonas tend to cause acute disease.

Â


334

USMLE STEP 2

PRESENTATION Infective endocarditis should be suspected in any patient with a fever of unknown origin (FUO). Common signs and symptoms include fever, anorexia, headache, arthralgia, and a new heart murmur (common finding that strongly raises the clinical suspicion). ABE in particular presents with acute onset of infection, a new murmur, and infections in other parts of the body from bacteremia leading to meningitis and pneumonia. SBE presents with gradual onset of infection and has splenomegaly. Patients with a rightsided endocarditis should be suspected of IV drug abuse, and septic PE may be the result from the tricuspid infestation. However, left-sided endocarditis is the most common type of endocarditis overall.

DIAGNOSIS Objective findings of infective endocarditis include multiple petechiae on the chest and mucous membranes, Osler nodes, Janeway lesions, splinter hemorrhages, Roth spots, and hemorrhage. Osler nodes are tender subcutaneous nodules on the distal extremities. Janeway lesions are hemorrhagic, nontender nodules on the distal extremities. Splinter hemorrhages are found in the nail bed, while Roth spots are points of retinal hemorrhage. Three positive blood cultures (BCx) are required for diagnosis. Vegetation on valve leaflets is pathognomonic and can be demonstrated best by TEE. Elevations in ESR, CRP, and WBCs are common, and the UA may have hematuria. CXR may indicate a water-bottle configuration.

TREATMENT The primary treatment of infective endocarditis is prophylaxis, as discussed above with antibiotic treatment prior to major procedures. Following infection, treatment with ceftriaxone for one month is required for streptococcus infection and oxacillin for a month with staphylococcus infection. Vancomycin is used for resistant strains. TABLE 405 ENDOCARDITIS Endocarditis Etiology

Result of an infective process leading to vegetation on the leaflets. Both ABE and SBE can occur. Rare, but is increasing in incidence in children with congenital heart defects.

Presentation

Fever, anorexia, headache, arthralgia, and a new heart murmur. ABE in particular presents with acute onset of infection, a new murmur, and infections in other parts of the body from bacteremia leading to meningitis and pneumonia. SBE presents with gradual onset of infection and has splenomegaly. Patients with a right-sided endocarditis should be suspected of IV drug abuse, and septic PE may be the result from the tricuspid infestation.

Diagnosis

Three positive BCx are required for diagnosis Multiple petechiae on the chest and mucous membranes, Osler nodes, Janeway lesions, splinter hemorrhages, Roth spots, and hemorrhage. Vegetation on valve leaflets are pathognomonic, TEE. Elevations in ESR, CRP, and WBCs are common, and the UA may have hematuria. CXR may indicate a water-bottle configuration.

Treatment

Prophylactic antibiotic treatment prior to major procedures. Following infection, treatment with ceftriaxone for one month is required for streptococcus infection and oxacillin for a month with staphylococcus infection. Vancomycin is used for resistant strains.

LIBMAN-SACKS ENDOCARDITIS (LSE) ETIOLOGY AND PATHOPHYSIOLOGY Libman-Sacks endocarditis (LSE) is a consequence of SLE and leads to sterile, verrucous vegetations. The mitral and aortic valves are most commonly involved, and the abnormalities are typically clinically silent. Valvular regurgitation can occur, but embolic www.ClinicalReview.com


Cardiovascular System

335 phenomenon is typically reserved for infective endocarditis (which is more likely with LSE). About half of all fatal SLE leads to LSE, and nearly ž of all patients with SLE can be demonstrated, by echocardiography, to have vegetation present on their valve leaflets. The precise pathogenesis is unknown, but in the presence of the antiphospholipid antibody (APA), phospholipids in the endothelial cell membranes may be targeted.

PRESENTATION AND DIAGNOSIS LSE is more common in African American patients and Hispanic women, and young women in particular tend to be affected by cardiac valvular lesions. Patients are typically asymptomatic, but may develop signs and symptoms similar to infective endocarditis. Thromboembolic phenomenon is rare, unless a superinfection leading to infective endocarditis occurs. SLE symptoms are universally present, along with APA. Physical exam is typically normal, and TEE may be required for diagnosis.

TREATMENT Treatment is best done by supportive therapy and treating the underlying SLE. It is unclear what the role of steroid therapy is. Prophylaxis for infective endocarditis is mandatory, and anticoagulation therapy may also be necessary. TABLE 406 LIBMAN-SACKS ENDOCARDITIS (LSE) Libman-Sacks Endocarditis Etiology

Consequence of SLE. The precise pathogenesis is unknown.

Presentation & Diagnosis

Patients are typically asymptomatic, but may develop signs and symptoms similar to infective endocarditis. SLE symptoms are universally present, along with APA. Physical exam is typically normal.

Treatment

Supportive therapy and treating the underlying SLE. Prophylaxis for infective endocarditis, and anticoagulation therapy may also be necessary.

NONBACTERIAL THROMBOTIC ENDOCARDITIS (NBTE) PRESENTATION Nonbacterial thrombotic endocarditis (NBTE) is often the late manifestation of numerous chronic infections, autoimmune disorders, and wasting diseases that lead to a degenerative series of endocardial lesions. Deposition of fibrin is the general underlying cause. NBTE appears to be related to DIC, and the underlying cause may be a variety of neoplasms and systemic infections. Prolonged time in elevated altitudes has been demonstrated to be a causative factor by one study.

DIAGNOSIS AND TREATMENT NBTE is difficult to detect prior to death, but TEE provides the best chance of identifying this disease. There is no known treatment.

Â


336

USMLE STEP 2

TABLE 407 NONBACTERIAL THROMBOTIC ENDOCARDITIS (NBTE) Nonbacterial Thrombotic Endocarditis (NBTE) Etiology

Deposition of fibrin is the general underlying cause. Late manifestation of numerous chronic infections, autoimmune disorders, and wasting diseases that lead to a degenerative series of endocardial lesions. NBTE appears to be related to DIC, and the underlying cause may be a variety of neoplasms and systemic infections.

Diagnosis

TEE provides the best chance of identifying this disease. There is no known treatment.

ARRHYTHMIA INTRODUCTION Normal electrical conduction within the heart begins with the sinoatrial (SA) node, located in the RA. This natural pacemaker of the heart receives innervation from both the parasympathetic nervous system (PNS) and sympathetic nervous system (SNS), and responds to the body’s need for heart rate modulation as in exercise or in combating illnesses. The impulses leave the SA node and travel to the AV node, where they are briefly delayed to permit atrial contraction and ventricular filling. Once the impulses are transmitted from the AV node, they travel via the bundle of His and the Purkinje fibers to reach the ventricles and cause ventricular depolarization and contraction. The intact conduction system is referred to as normal sinus rhythm (NSR). Abnormal heartbeats due to electrical abnormalities are known as arrhythmia. Arrhythmias are classified by where they begin.

PREMATURE CONTRACTIONS ETIOLOGY Premature atrial contractions (PACs) and premature ventricular complexes (PVCs) are early electrical impulses that cause a premature contraction of the heart. They are typically benign and disappear on their own.

SINUS BRADYCARDIA ETIOLOGY Sinus bradycardia is a regular heart beat with normal P waves and regular PR intervals but with a rate less than 60 beats per minute (BPM). It is caused by excessive vagal tone, which in turn may be due to vasovagal syncope, MI, carotid sinus pressure, vomiting, parasympathetic agonists such as edrophonium, cardiac glycosides, and Valsalva maneuvers. Overmedication with beta-blockers and calcium-channel blockers also contribute. Increased intracranial pressure (ICP), hypothyroidism, and hypothermia are other causes. Sinus bradycardia may be an entirely normal finding, especially in athletes. It is typically asymptomatic.

TREATMENT Symptomatic patients are typically treated with atropine. Continuing bradycardia with symptoms requires the use of a pacemaker. A dopamine drip may also be used in an emergent situation.

www.ClinicalReview.com


Cardiovascular System

337

TABLE 408 SINUS BRADYCARDIA Sinus Bradycardia Etiology & Presentation

It is caused by excessive vagal tone, which in turn may be due to vasovagal syncope, MI, carotid sinus pressure, vomiting, parasympathetic agonists, cardiac glycosides, and Valsalva maneuvers. Overmedication with betablockers and calcium-channel blockers also contribute. Increased ICP, hypothyroidism, and hypothermia are other causes. It is typically asymptomatic.

Treatment

Symptomatic patients are typically treated with atropine. Continuing bradycardia with symptoms requires the use of a pacemaker. A dopamine drip may also be used in an emergent situation.

SINUS TACHYCARDIA ETIOLOGY AND TREATMENT Sinus tachycardia is a sustained HR over 100 BPM, commonly due to fever, low BP, stress, medications, and hyperthyroidism. It may also occur for a short period of time following the cessation of beta-blocker therapy. Treatment often involves carotid sinus massage and otherwise increasing vagal tone. TABLE 409 SINUS TACHYCARDIA Sinus Tachycardia Etiology Treatment

Commonly due to fever, low BP, stress, medications, and hyperthyroidism. May occur for a short period of time following cessation of beta-blockers. Carotid sinus massage and increasing vagal tone.

PAROXYSMAL ATRIAL TACHYCARDIA (PAT) ETIOLOGY AND TREATMENT Paroxysmal atrial tachycardia (PAT) is the result of a premature supraventricular beat leading to an AV nodal re-entry rhythm with a rate greater than 130 BPM. It is typically treated by increasing vagal tone, using calcium-channel blockers or adenosine, betablockers, and cardioversion. TABLE 410 PAROXYSMAL ATRIAL TACHYCARDIA Paroxysmal Atrial Tachycardia Etiology

Premature supraventricular beat leading to an AV nodal re-entry rhythm with a rate greater than 130 BPM.

Treatment

Increase vagal tone, calcium-channel blockers or adenosine, beta-blockers, and cardioversion.

Â


338

USMLE STEP 2

ATRIAL FLUTTER ETIOLOGY AND TREATMENT Atrial flutter is a regular atrial rhythm of about 300 BPM and a 2:1 block through the AV node leading to a ventricular rhythm of 150 BPM. It is commonly the result of COPD, PE, MVP, ETOH, and thyrotoxicosis. Treatment involves cardioversion, calciumchannel blockers, and digoxin. Atrial flutter is distinguished from atrial fibrillation by its regular rate and rhythm. TABLE 411 ATRIAL FLUTTER Atrial Flutter Etiology

It is commonly the result of COPD, PE, MVP, ETOH, and thyrotoxicosis.

Treatment

Cardioversion, calcium-channel blockers, and digoxin.

ATRIAL FIBRILLATION ETIOLOGY Atrial fibrillation is the development of impotent atrial contractions as a result of chaotic electrical activity through the conduction system. It is commonly found in patients with dilated atria, CHF, valvular heart disease, elderly patients, CAD, cardiomyopathy, ETOH abuse, sepsis, RHD, and thyrotoxicosis.

PRESENTATION AND DIAGNOSIS Atrial fibrillation presents with palpitations, missed heart beats, fatigue, chest pain, and TIAs due to thromboembolic phenomenon likely in this condition. Atrial fibrillation is diagnosed as an irregularly irregular pulse, and nondistinct P waves on EKG. Atrial fibrillation has been described as having a “bag of worms� appearance.

TREATMENT Atrial fibrillation is treated with thromboembolism prophylaxis using warfarin. Patients with a HR greater than 100 BPM may receive IV beta-blockers or calcium-channel blockers, digoxin, and cardioversion. Unstable patients receive cardioversion followed by maintenance therapy. Quinidine or procainamide may also be used with variable effect. TABLE 412 ATRIAL FIBRILLATION Atrial Fibrillation Etiology

Result of chaotic electrical activity through the conduction system. Commonly in patients with dilated atria, CHF, valvular heart disease, elderly patients, CAD, cardiomyopathy, ETOH abuse, sepsis, RHD, and thyrotoxicosis.

Diagnosis

Presents with palpitations, missed heart beats, fatigue, chest pain, and TIAs, irregularly irregular pulse, and nondistinct P waves on EKG.

Treatment

Warfarin. Patients with a HR greater than 100 BPM may receive IV beta-blockers or calcium-channel blockers, digoxin, and cardioversion. Unstable patients receive cardioversion followed by maintenance therapy.

www.ClinicalReview.com


Cardiovascular System

339

ATRIOVENTRICULAR (AV) BLOCK ETIOLOGY AND PATHOPHYSIOLOGY Atrioventricular (AV) block can be divided into three distinct classes: first-degree heart block, second-degree heart block, and third-degree heart block. First-degree heart block is a PR interval greater than 0.20 s at a normal resting HR. It is commonly due to AV conduction system degeneration with aging, excessive vagal tone, inflammation, ischemia, and digoxin toxicity. Seconddegree heart block is divided into Mobitz I and Mobitz II. Mobitz I second-degree heart block, also known as Wenckebach rhythm, is a progressive increase in the PR interval with shortening of the RR interval until a ventricular beat is dropped. It is typically due to AV nodal block in conduction, and this may occur due to poor perfusion. Mobitz II second-degree heart block is a prolonged but stable PR interval with regularly dropped beats. The site of blockage is usually infranodal. Third-degree AV block arises from discontinuity between the atria and ventricles, and leads to independent activity in the atria compared to the ventricles. Thirddegree heart block is also known as complete heart block, and the source of ventricular rhythm is an ectopic focus distal to the point of conduction blockade. Complete heart block is typically attributed to Lenegre disease, which is age-related degeneration in the conduction system. Other causes include inferior or posterior MI, infection, inflammation, digoxin toxicity, and ankylosing spondylitis. HLA-B27 is linked with the development of complete heart block.

PRESENTATION First-degree and second-degree heart blocks are typically asymptomatic. Third-degree heart block may present with intermittent CHF, transient ventricular arrhythmias leading to circulatory failure (known as Adams-Stoke attacks), and bradycardia which can worsen CHF.

DIAGNOSIS First-degree heart block is diagnosed by EKG with the characteristic prolonged PR interval. Mobitz I second-degree heart block is diagnosed by progressive PR prolongation with intermittent dropped QRS complexes. Mobitz I may worsen with increased vagal tone, while atropine may ameliorate this condition. Mobitz II second-degree heart block has a stable and prolonged PR interval with predictable dropped beats. Atropine has no effect. Third-degree heart block is diagnosed by independent activity of the atria and ventricles.

TREATMENT First-degree heart block is typically asymptomatic and not treated. Mobitz I is treated with atropine and pacing. Mobitz II typically requires a pacemaker. Complete heart block requires epinephrine or isoproterenol to establish a sustainable ventricular rate, then maintenance with a pacemaker.

Â


340

USMLE STEP 2

TABLE 413 ATRIOVENTRICULAR BLOCK Atrioventricular Block

Etiology

First-degree heart block: Commonly due to AV conduction system degeneration with aging, excessive vagal tone, inflammation, ischemia, and digoxin toxicity. Second-degree: is divided into Mobitz I and Mobitz II. Mobitz I is typically due to AV nodal block in conduction, and this may occur due to poor perfusion. Mobitz II site is usually infranodal. Third-degree: Typically attributed to Lenegre disease. Other causes include inferior or posterior MI, infection, inflammation, digoxin toxicity, and ankylosing spondylitis. HLA-B27 is linked with the development of complete heart block.

Presentation

First-degree and second-degree heart blocks are typically asymptomatic. Third-degree heart block may present with intermittent CHF, transient ventricular arrhythmias leading to circulatory failure, and bradycardia which can worsen CHF.

Treatment

First-degree heart block is typically not treated. Mobitz I is treated with atropine and pacing. Mobitz II typically requires a pacemaker. Complete heart block requires epinephrine or isoproterenol, then maintenance with a pacemaker.

VENTRICULAR ARRHYTHMIAS ETIOLOGY Ventricular arrhythmias are composed of ventricular tachycardia (VT), ventricular fibrillation (VF), WPW, and Torsade de Pointes (the latter two are discussed separately below). VT is a serious condition due to its degeneration into VF. VF is incompatible with life and patients with VF lasting more than a few seconds will lose consciousness. VT is an organized depolarization of the ventricles from a focus of ventricular origin that leads to more than 120 BPM. VT is characterized as having bizarre QRS complexes, and typically has some level of AV dissociation. VT is common in patients with MI, cardiomyopathy, metabolic changes, and digoxin toxicity. WPW and antiarrhythmic agents can also lead to VT. VF is the end result of a chaotic electrical activity within the ventricles leading to haphazard depolarization of the ventricles and an impotent contraction.

PRESENTATION VT presents with hypotension, CHF, syncope, and cardiac failure. SBP varies over time, and extra heart sounds may be present. Cannon waves may be present in the JVP due to simultaneous contraction of the chambers of the heart. There is also wide splitting of S1 and S2. VF presents with syncope and leads to death if no emergent interventions are taken.

TREATMENT Increasing voltage cardioversion is used to restore normal heart rhythm, followed by epinephrine, and repeated cardioversion. Amiodarone, lidocaine, magnesium, and procainamide are given with cardioversion repeated between each medication administration. If successful, maintenance therapy includes pacemaker implantation, an implanted cardioverter and defibrillator (ICD), and ablation of any bypass tracts.

www.ClinicalReview.com


Cardiovascular System

341

TABLE 414 VENTRICULAR ARRHYTHMIAS Ventricular Arrhythmias Etiology

VT, VF, WPW, and Torsade de Pointes. VT is common in patients with MI, cardiomyopathy, metabolic changes, and digoxin toxicity. WPW and antiarrhythmic agents can also lead to VT. VF is the end result of a chaotic electrical activity within the ventricles.

Presentation

Hypotension, CHF, syncope, and cardiac failure. SBP varies over time, and extra heart sounds may be present. Cannon waves may be present in the JVP. There is also wide splitting of S1 and S2. VF presents with syncope and leads to death if no emergent interventions are taken.

Treatment

Increasing voltage cardioversion, followed by epinephrine, and repeated cardioversion. Amiodarone, lidocaine, magnesium, and procainamide are given with cardioversion repeated between each medication administration. If successful, maintenance therapy includes pacemaker implantation, an ICD, and ablation of any bypass tracts.

WOLFF-PARKINSON-WHITE SYNDROME (WPW) ETIOLOGY AND PATHOPHYSIOLOGY Wolff-Parkinson-White (WPW) syndrome is a pre-excitation syndrome characterized by an accessory pathway that bypasses the delay in conductance at the AV node. Narrow-complex reentry occurs with anterograde conduction between the bundle of His and Purkinje fibers and back up the accessory pathway. Simultaneously, a wide-complex reentry starts with conduction down the accessory pathway and retrogrades through the Purkinje fibers and His fibers. The end result is premature activation of parts of the ventricle. WPW is especially common in congenital heart defects such as Ebstein anomaly associated with the tricuspid valve and transposition of vessels (ToV). It can also occur in CHF, HCM, SVT, PAT, and atrial fibrillation.

DIAGNOSIS AND TREATMENT EKG typically shows a wide QRS wave with a delta wave indicating early depolarization of the ventricle via the bypass tract. The bypass is identified through electrophysiological tests and cardiac catheterization, and subsequently ablated with radio waves. Cardioversion may be required in emergent cases and with destructive arrhythmias. Maintenance therapy of WPW may be instituted until radioablation can take place; the medications used include amiodarone, flecainide, procainamide, or sotalol. Adenosine, beta-blockers, calcium-channel blockers, and digoxin are absolutely contraindicated due to the potential to block normal AV conduction. TABLE 415 WOLF-PARKINSON-WHITE SYNDROME (WPW) Wolf-Parkinson-White Syndrome (WPW) Etiology

Premature activation of parts of the ventricle. WPW is especially common in congenital heart defects such as Ebstein anomaly, ToV. It can also occur in CHF, HCM, SVT, PAT, and atrial fibrillation.

Diagnosis

EKG typically shows a wide QRS wave with a delta wave indicating early depolarization of the ventricle by the bypass tract. The bypass is identified through electrophysiological tests and cardiac catheterization.

Treatment

Cardioversion may be required and with destructive arrhythmias. Maintenance therapy may be instituted until radioablation can take place; the medications used include amiodarone, flecainide, procainamide, or sotalol.

Â


342

USMLE STEP 2

TORSADE DE POINTES ETIOLOGY AND PATHOPHYSIOLOGY Torsade de Pointes is characterized by a prolonged QT and a QRS complex that rotates 180 degrees about its long axis. It is caused by hypokalemia, hypomagnesemia, tricyclic antidepressants (TCAs), procainamide, disopyramide, psychotropic agents (including lithium and phenothiazines), CVA, congenital QT syndrome, quinidine, bradycardia, complete heart block, and idiopathic causes. The arrhythmia itself may be initiated by a PVC leading to abnormal ventricular repolarization.

PRESENTATION AND TREATMENT Torsade de Pointes presents with syncope and may degenerate into ventricular fibrillation. It may also be initiated by sudden auditory stimuli. Long QT syndrome presents with recurrent lightheadedness and syncope. Treatment of torsade includes treating the underlying etiology, using magnesium to stabilize the heart rhythm, overdriving the heart, and using beta-blockers for maintenance. Pacing may be necessary. TABLE 416 TORSADE DE POINTES Torsade de Pointes Etiology

Caused by hypokalemia, hypomagnesemia, TCAs, procainamide, disopyramide, psychotropic agents, CVA, congenital QT syndrome, quinidine, bradycardia, complete heart block, & idiopathic causes.

Diagnosis

Syncope which may worsen into ventricular fibrillation. It may also be initiated by sudden auditory stimuli. Long QT syndrome presents with recurrent lightheadedness and syncope.

Treatment

Treat the underlying etiology, use magnesium to stabilize the heart rhythm, and using beta-blockers for maintenance. Pacing may be necessary.

AORTIC DISEASES INTRODUCTION The largest artery in the body is an elastic blood vessel composed of three layers of tissue: the innermost tunica intima, the middle tunica media, and the outermost tunica adventitia. The beginning of the aorta is known as the aortic root, followed by the ascending aorta that continues to the brachiocephalic, left common carotid, and left subclavian branches. The part that gives off these three branches is known as the aortic arch. The descending aorta travels through the diaphragm and into the abdomen, where it is referred to as the abdominal aorta. Numerous diseases affect the aorta – the most common and clinically significant ones include aortic dissection, and aortic aneurysm. Mesenteric ischemia is included in this section due to its relation to vascular disorders.

AORTIC DISSECTION ETIOLOGY Aortic dissection is the result of a transverse tear through the intima and media of the aortic wall leading to extravasation of blood, potential compromise of flow to other arterial branches, and the possibility of rupture and hemorrhage. Aortic dissection is attributable to HTN, congenital heart defects, CTD, syphilis, pregnancy, coarctation of the aorta (as in Turner syndrome), abuse of cocaine, and trauma. The Treponema pallidum organisms of syphilis cause aortic dissection through infection of the vasa www.ClinicalReview.com


Cardiovascular System

343

vasorum with subsequent irritation by the inflammatory and immune reaction. Aortic dissection is classified by the DeBakey system into three types: type I involves the ascending aorta and part of the distal aorta, and is the most common of the three types; type II involves only the ascending aorta; and type III involves only the descending aorta. The Stanford system classifies aortic dissections into the type A class if the ascending aorta is affected, and the type B class if the descending aorta is affected.

PRESENTATION AND DIAGNOSIS Aortic dissection presents as a tearing sensation with severe chest pain that radiates to the back. HTN is typically present, discordant pulses between extremities can be measured, and AR is typically present. Diagnosis is made by CXR that indicates loss of the aortic knob and a widened mediastinum, CT with IV contrast or TEE that demonstrates an intimal flap with extravasation of blood, and angiogram as the gold standard. A CXR should be obtained in all patients suspected of MI, as the thrombolytic therapy used in MI is absolutely contraindicated in aortic dissection.

TREATMENT Aortic dissection is treated with BP control, immediate surgical repair for type A / type II dissections, and medical management for type B / type III dissections. Complications of treatment include the development of MI, stroke, and tamponade. TABLE 417 AORTIC DISSECTION Aortic Dissection Etiology

Attributable to HTN, congenital heart defects, CTD, syphilis pregnancy, coarctation of the aorta, cocaine abuse, & trauma.

Classifications DeBakey

Type I involves the ascending aorta and part of the distal aorta Type II involves only the ascending aorta Type III involves only the descending aorta.

Stanford

Type A if the ascending aorta is affected Type B if the descending aorta is affected.

Presentation

A tearing sensation with severe chest pain that radiates to the back. HTN, discordant pulses, and AR.

Diagnosis

CXR indicates loss of the aortic knob & a widened mediastinum, CT with IV contrast or TEE that demonstrates an intimal flap with extravasation of blood, and angiogram.

Treatment

BP control, immediate surgical repair for type A / type II dissections, and medical management for type B / type III dissections.

ABDOMINAL AORTIC ANEURYSM (AAA) ETIOLOGY An abdominal aorta aneurysm is a dilation of the abdominal aorta secondary to atherosclerosis. Other causes include cystic medial necrosis in CTD, syphilis, fungal infections, aortitis, and trauma. Expansion of the AAA over 5 cm heralds a significant risk of rupture, and thus a very high morbidity and mortality. A true aneurysm has a defect in all three layers of the aortic wall, while a pseudoaneurysm affects only the tunica intima and media.

PRESENTATION AND DIAGNOSIS AAA presents with abdominal pain or back pain and a palpable, pulsatile mass in the abdomen. Syncope is sometimes present, as well as hypotension. There is typically a history of CAD. Most patients are asymptomatic. Diagnosis of AAA is by US, but MRI with

Â


USMLE STEP 2 344 IV contrast is preferred to demonstrate the precise dimensions of the mass and whether a leak is present. Angiogram was previously the gold standard. Bloody BM s/p AAA = bowel ischemia. Dx w/ colonoscopy.

TREATMENT

After diagnosis, patients should have two large bore IV access points and a type and cross (TxC). Patients in whom rupture has occurred should receive emergent laparotomy and surgical correction. Unstable patients with a high clinical suspicion of AAA should be taken immediately to the operating room (OR). TABLE 418 ABDOMINAL AORTIC ANEURYSM (AAA) Abdominal Aortic Aneurysm (AAA) Etiology

Dilation of the abdominal aorta secondary to atherosclerosis. Other causes include cystic medial necrosis in CTD, syphilis, fungal infections, aortitis, and trauma.

Presentation

Abdominal pain or back pain and a palpable, pulsatile mass in the abdomen. Syncope is sometimes present as well as HTN.

Diagnosis

US, but MRI with IV contrast is preferred.

Treatment

Two large bore IV access points and a TxC. Patients in whom rupture has occurred should receive emergent laparotomy and surgical correction. Unstable patients with a high clinical suspicion of AAA should be taken immediately to the OR.

COMMON CARDIOVASCULAR MEDICATIONS NITRATES The use of nitrates is a mainstay regimen for the treatment of numerous cardiac disorders. In low doses, nitrates dilate veins and thereby reduce preload. Larger doses lead to a reduction in afterload, and eventually lead to dilation of the coronary arteries to permit increased oxygenation of a heart that now has lower stresses. Side effects of nitrates stem from their vasodilation, causing orthostatic hypotension, reflex tachycardia, headaches, and facial flushing. Nitrates are contraindicated in individuals with low SBP.

BETA-BLOCKERS Beta-blockers are used to decrease oxygen requirements by the myocardium through decreased heart rate, blood pressure, and contractility. They improve morbidity and mortality following an acute MI and in CHF. Side effects of beta-blockers include fatigue, depression, worsening of the lipid profile, bronchoconstriction, sexual dysfunction, and insomnia. Beta-blockers are contraindicated in individuals with severe asthma, and should be used with care in individuals with DM. Relative contraindications include poorly controlled DM, coexisting use of calcium channel blockers, in 1st degree heart block, and severe peripheral vascular disease.

ASPIRIN Aspirin is used to decrease the incidence of clot formation and provide some risk aversion to the development of MI. Aspirin is also one of the first medications given following an MI, and can reduce mortality by one-quarter. The major side effect of aspirin is gastrointestinal bleeding. Aspirin is renally secreted.

www.ClinicalReview.com


Cardiovascular System

345

HEPARIN The use of heparin and low molecular weight heparin is necessary in minimizing further clot formation, especially after MI and in numerous coagulopathies. Heparin administration requires monitoring of PT and PTT; LMWH does not require such monitoring. Contraindications to heparin administration include active bleeding diatheses (such as GI bleeds), tendency towards hemorrhage (as in severe liver disease), immediately following a major surgery, in patients with severe HTN, and in those with infective endocarditis. LMWH is given subcutaneously (SC).

STREPTOKINASE AND ALTEPLASE Among the thrombolytics are streptokinase, urokinase, alteplase, anistreplase, and reteplase. These medications are used to actively break up existing clots and are best if given within a few hours of MI onset. Contraindications to thrombolytic therapy include patients with a history of stroke in the past year, any history of significant hemorrhage, aortic dissection, severe HTN, and an active bleed. Relative contraindications include patients with recent surgical interventions, those with a bleeding disorder or coagulopathy, active PUD, and those who had cardiopulmonary resuscitation (CPR). Streptokinase cannot be used more than once every few months due to its immunogenic potential.

ACE-INHIBITORS Angiotensin-converting enzyme (ACE) inhibitors have been shown to reduce the mortality and morbidity following an MI due to the reduction in the progression of complications. ACE-inhibitors decrease cardiac load and permit improved oxygenation and myocardial function. ACE-inhibitors should be avoided in patients with renal artery stenosis, renal disease, and pregnancy.

DIGOXIN The cardiac glycosides are used for the treatment of CHF, atrial fibrillation, and atrial tachycardia. Although they do not have any effect on mortality in CHF, digoxin does have a role in decreasing hospitalization and making the patient more comfortable. Digoxin works by promoting increased intracellular calcium through decreased calcium-sodium exchange and inhibition of the Na+/K+-ATPase. The increase in calcium leads to the positive inotropic activity of the cardiac glycosides; this in turn leads to increased stroke volume and improved cardiac output. Digoxin administration is confounded by hyperkalemia, which decreases the potency, and hypokalemia, which can lead to toxicity. Digoxin toxicity can occur in other disorders such as ESRD, electrolyte disturbances, hypothyroidism, hypoxemia, interaction with multiple drugs, HCM, WPW, intrinsic heart diseases, arrhythmia, and in the elderly. Medications that potentiate the effect of digoxin include quinidine, calcium-channel blockers, and numerous diuretics. Digoxin effect is decreased with bile acid sequestrants. Digoxin toxicity can lead to nausea and vomiting, blurred vision with a yellow halo around objects, gynecomastia, and arrhythmia (especially paroxysmal atrial tachycardia). Digoxin toxicity is treated by cessation of the medication, administration of potassium to decrease the effect of digoxin through calcium competition, digibind for significant overdoses, and lidocaine and phenytoin.

VASCULAR DISORDERS INTRODUCTION A number of vasculitides affect patients with rheumatic illnesses. Several have already been discussed in the renal section, including polyarteritis nodosa and Wegener granulomatosis. The rest of the major vascular disorders, including Churg-Strauss and Takayasu, are discussed below. Avascular necrosis of the hip is also mentioned for completeness.

Â


346

USMLE STEP 2

CHURG-STRAUSS DISEASE ASSESSMENT Churg-Strauss disease affects medium-sized arteries. Churg-Strauss leads to granulomatosis and systemic necrotizing vasculitis with a number of nonspecific and constitutional symptoms. Overall, bronchospasm leading to asthma is common. Maculopapular rashes also develop throughout the body, along with purpura and nodules.

MANAGEMENT Treatment of Churg-Strauss disease is to use corticosteroids, azathioprine, and cyclophosphamide to control the immune reaction. TABLE 419 CHURG-STRAUSS DISEASE Churg-Strauss Disease Etiology

Systemic vasculitis of medium sized arteries.

Presentation

Systemic necrotizing vasculitis with a number of nonspecific and constitutional symptoms. Overall, bronchospasm leading to asthma is common. Maculopapular rashes also develop throughout the body, along with purpura and nodules.

Diagnosis

Hypereosinophilia

Treatment

Corticosteroids, azathioprine, and cyclophosphamide.

TAKAYASU ARTERITIS ASSESSMENT Takayasu arteritis is an autoimmune complex that affects medium and large-sized arteries. It is especially common in Asian women. Takayasu arteritis is also known as pulseless arteritis because it leads to a loss of a palpable pulse in the upper extremities and carotid vessels. Raynaud’s phenomenon occurs as a result of this decreased flow, along with decreased perfusion of the brain and the resultant sequelae. Mesenteric ischemia leading to abdominal pain also occurs. Diagnosis of Takayasu arteritis is made by arteriography.

MANAGEMENT Takayasu arteritis is treated with corticosteroids and methotrexate to induce remission. TABLE 420 TAKAYASU ARTERITIS Takayasu Arteritis Etiology

Autoimmune disorder of medium and large arteries.

Presentation

Loss of palpable pulse in upper extremities and carotids. Raynaud’s, abdominal pain.

Diagnosis

Arteriography.

Treatment

Corticosteroids, methotrexate.

www.ClinicalReview.com


Cardiovascular System

347

WEGENER GRANULOMATOSIS ETIOLOGY AND PATHOPHYSIOLOGY Wegener granulomatosis is the formation of necrotizing granulomas in the respiratory tract and renal parenchyma. The result is the development of a glomerulonephritis, disseminated vasculitis, and pulmonary disease. Cytoplasmic antineutrophil cytoplasmic antibodies (C-ANCA) have been associated with Wegener granulomatosis, and implicate an autoimmune cause. Leukocyte release of inflammatory cytokines and toxic oxygen radicals leads to organ damage.

PRESENTATION AND DIAGNOSIS Wegener granulomatosis presents with inflammation of the respiratory tract, vasculitis of smaller vessels, and focal glomerulonephritis. Some of the manifestations of this potentially devastating disease are: septal perforation, otitis media, hearing loss, facial paralysis, oral ulcerations, subglottic stenosis, pulmonary symptoms such as cough, hemoptysis, and pleuritis, renal involvement with focal glomerulonephritis and ESRD, conjunctivitis, uveitis, arthralgias, myalgias, petechiae, peripheral neuropathy, and GI symptoms. Diagnosis is by clinical history, elevated C-ANCA, and confirmation by biopsy. Wegener granulomatosis tends to present with the triad of sinusitis, pulmonary infiltrates, and nephritis.

TREATMENT Wegener granulomatosis is treated with glucocorticoids to achieve immunosuppression. Cyclophosphamide and methotrexate are used in more serious disease. TMP-SMX has been used to avoid relapse, but the mechanism of action of this agent is unknown. Survival is dismal without treatment and death is likely after only a few months due to renal failure. TABLE 421 WEGENER GRANULOMATOSIS Wegener Granulomatosis Etiology

Formation of necrotizing granulomas in the respiratory tract and renal parenchyma.

Presentation

Inflammation of the respiratory tract, vasculitis of smaller vessels, and focal glomerulonephritis. Septal perforation, otitis media, hearing loss, facial paralysis, oral ulcerations, subglottic stenosis, pulmonary symptoms such as cough, hemoptysis, and pleuritis, renal involvement with focal glomerulonephritis and ESRD, conjunctivitis, uveitis, arthralgias, myalgias, petechiae, peripheral neuropathy, and GI symptoms. Triad of sinusitis, pulmonary infiltrates, and nephritis.

Diagnosis

Clinical history, elevated C-ANCA, and confirmation by biopsy.

Treatment

Glucocorticoids. Cyclophosphamide and methotrexate are used in more serious disease. TMP-SMX has been used to avoid relapse.

HENOCH-SCHONLEIN PURPURA ETIOLOGY AND PATHOPHYSIOLOGY (HSP) Henoch-Schonlein purpura is aggregation of IgA and deposition of this immunoglobulin within vessel walls and the renal mesangium. Abnormalities with IgA1 have been implicated, with an inflammatory response resulting after abnormal deposition of these antibody complexes. The release of various inflammatory mediators by activated lymphocytes leads to the vasculitis seen in this disease. An allergic reaction to foods, insects, cold exposure, and various medications (penicillins, erythromycin, quinidine)

Â


USMLE STEP 2 348 have been implicated as being the basis for the faulty immune reaction and IgA release. Other causes include infections by EBV, VZV, parvovirus B19, GAS, HCV, and SBE, and vaccinations including those for typhoid, measles, cholera, and yellow fever.

PRESENTATION AND DIAGNOSIS HSP carries high morbidity but mortality is rare. A symmetrical erythematous macular rash begins on the lower extremities followed by the development of purpura. Edema of the scalp and distal extremities occurs. Abdominal pain, bloody diarrhea, arthralgias, and acute renal damage occur. Eosinophilia is often present along with IgA. Ultrasound, plain films, and MRI are used to confirm the findings, and a renal biopsy often clinches the diagnosis. The distinguishing feature of HSP is the presence of arthralgias.

TREATMENT Treatment of HSP is primarily supportive management of renal failure and abdominal complications. Corticosteroids have been used with some benefit. TABLE 422 HENOCH-SCHONLEIN PURPURA (HSP) Henoch-Schonlein Purpura (HSP)

Etiology

Aggregation of IgA and deposition of this immunoglobulin within vessel walls and the renal mesangium. An allergic reaction to foods, insects, cold exposure, and various medications have been implicated as being the basis for the faulty immune reaction and IgA release. Other causes include infections by EBV, VZV, parvovirus B19, GAS, HCV, and SBE, and vaccinations including those for typhoid, measles, cholera, and yellow fever.

Presentation

A symmetrical erythematous macular rash begins on the lower extremities followed by the development of purpura. Edema of the scalp and distal extremities occurs. Abdominal pain, bloody diarrhea, arthralgias, and acute renal damage occur. Eosinophilia is often present along with IgA.

Diagnosis

Ultrasound, plain films, and MRI are used to confirm the findings, and a renal biopsy often clinches the diagnosis. The distinguishing feature of HSP is the presence of arthralgias.

Treatment

Supportive management of renal failure and abdominal complications. Corticosteroids have been used with some benefit.

POLYARTERITIS NODOSA ETIOLOGY AND PATHOPHYSIOLOGY (PAN) PAN is the development of a systemic vasculitis with necrotizing inflammatory changes to medium and small arteries and subsequent infarction of organs. The kidney vasculature is especially affected, along with the intestines, peripheral nerves, joints, and skin. PAN is occasionally a consequence of viral infections such as HBV, but an autoimmune cause has yet to be ascertained.

PRESENTATION AND DIAGNOSIS The complications of PAN include skin ulceration, gangrene of the distal extremities, infarction of the bowel, liver, and kidney, renovascular HTN, and internal hemorrhage. Peripheral neuropathy is often present. PAN may be a complication of RA and SjÜgren’s syndrome. Diagnosis of PAN is difficult as ANCA is rarely present, and biopsy is typically required to detect the focal necrotizing arteritis.

www.ClinicalReview.com


Cardiovascular System

349

TREATMENT Treatment of PAN involves glucosteroids to suppress the immune reaction. Seroconversion in patients with HBV is sometimes done through granulocyte-macrophage colony-stimulating factor (GM-CSF). TABLE 423 POLYARTERITIS NODOSA (PAN) Polyarteritis Nodosa (PAN) Etiology

Development of a systemic vasculitis with necrotizing inflammatory changes to medium and small arteries and subsequent infarction of organs. The kidney vasculature is especially affected, along with the intestines, peripheral nerves, joints, and skin. Occasionally a consequence of viral infections.

Presentation

Skin ulceration, gangrene of the distal extremities, infarction of the bowel, liver, and kidney.

Diagnosis

Biopsy is typically required to detect the focal necrotizing arteritis.

Treatment

Glucosteroids. Seroconversion in patients with HBV is sometimes done through GM-CSF.

THROMBOTIC THROMBOCYTOPENIC PURPURA (TTP) ETIOLOGY AND PATHOPHYSIOLOGY TTP, and its related syndrome, hemolytic uremic syndrome (HUS) are the consequence of vascular injury from inflammatory or toxic damage. Platelet responses, subsequent release of free radicals, and loss of prostacyclin leads to amplified damage. TTP occurs in adults, while HUS occurs in children. TTP presents with widespread capillary thrombosis that primarily affects the kidney and brain. Microinfarction and petechial hemorrhages occur. HUS presents with mostly subendothelial capillary damage and affects the kidneys with necrotizing renal artery thrombosis only in older children. HUS appears to be due more to circulating Escherichia coli type 2 verotoxin. Binding of this verotoxin to the renal parenchyma may result in additional renal damage. HUS is responsible for as much as 1 in 10 cases of HTN in children. The prognosis with TTP and HUS remains guarded, with TTP having a higher mortality. HUS primarily affects children under five, TTP affects adults in their 30s.

PRESENTATION AND DIAGNOSIS TTP has been related to RA, PAN, SLE, and Sjögren’s syndrome. TTP evolves over a week with early fever and skin purpura. Hematuria and hematochezia are present with hemorrhage in the mucosal membranes and retina. Pancreatitis, arthralgia, and CNS changes also evolve. HUS may be preceded by a viral or bacterial prodrome; viruses include echovirus, adenovirus, Coxsackie virus, and bacteria include E. coli, Salmonella, Shigella, Streptococcus, and Yersinia. E. coli O157:H7 is the most common cause of HUS, and poorly cooked hamburger meat is the implicated cause. HUS presents with weeks of abdominal pain, nausea and vomiting, diarrhea, hematochezia, GI bleeding, ARF, uremia, HTN, CNS changes, blindness, high fever, and pallor. A microangiopathic hemolytic anemia with hematuria and proteinuria can be demonstrated in both TTP and HUS.

TREATMENT Treatment of TTP involves plasmapheresis, fresh frozen plasma (FFP), and corticosteroids. Splenectomy is sometimes beneficial in TTP. HUS requires blood transfusions in most patients. Dialysis is necessary in about half of all patients due to the severe ARF. Fluid and electrolyte management is required, and reduction of HTN is necessary. Anticoagulation, thrombolysis, plasmapheresis, and prostacyclin infusion are necessary in some cases. Mesenteric infarction leading to ischemia of the colon in HUS may necessitate bowel resection.


350

USMLE STEP 2

TABLE 424 THROMBOTIC THROMBOCYTOPENIC PURPURA (TTP) Thrombotic Thrombocytopenic Purpura (TTP) Etiology

Consequence of vascular injury from inflammatory or toxic damage.

Presentation

Widespread capillary thrombosis that primarily affects the kidney and brain. Microinfarction and petechial hemorrhages. Evolves over a week with early fever and skin purpura. Hematuria and hematochezia are present with hemorrhage in the mucosal membranes and retina. Pancreatitis, arthralgia, and CNS changes also evolve. HUS may be preceded by a viral or bacterial prodrome (see text). HUS presents with weeks of abdominal pain, nausea and vomiting, diarrhea, hematochezia, GI bleeding, ARF, uremia, HTN, CNS changes, blindness, high fever, and pallor. A microangiopathic hemolytic anemia with hematuria and proteinuria can be demonstrated in both TTP and HUS.

Diagnosis

Biopsy is typically required to detect the focal necrotizing arteritis.

Treatment

Plasmapheresis, FFP, and corticosteroids. Splenectomy. HUS requires blood transfusions in most patients. Dialysis is necessary in about half of all patients due to the severe ARF. Fluid and electrolyte management is required, and reduction of HTN is necessary. Anticoagulation, thrombolysis, plasmapheresis, and prostacyclin infusion are necessary in some cases. Mesenteric infarction leading to ischemia of the colon in HUS may necessitate bowel resection.

AVASCULAR NECROSIS ASSESSMENT The blood supply to the scaphoid bone of the wrist and the head of the femur are particularly limited and therefore susceptible to distal ischemia, if the perfusion is disturbed. Wrist fractures in the anatomical snuff box and fractures of the head of the femur must therefore be managed quickly and efficiently to prevent avascular necrosis (AVN). Other conditions that can lead to avascular necrosis of the hip include use of steroids, radiation therapy, alcoholism, sickle cell anemia, and Gaucher disease. AVN of the hip presents as referred pain to the knee and is worsened with internal rotation of the hip. It is for this reason, and many other related conditions, that all knee pain elicits a full examination of the entire lower extremity, including the hip. Diagnosis of AVN of the hip or scaphoid is made by MRI or bone scans. Plain films are nonspecific early in the disease.

MANAGEMENT If surgical repair cannot be done and the blood supply reestablished to the ischemic region, total replacement of the joint may become necessary. TABLE 425 AVASCULAR NECROSIS (AVN) Avascular Necrosis (AVN) Etiology

Ischemia due to Wrist fractures in the anatomical snuff box and fractures of the head of the femur. Use of steroids, radiation therapy, alcoholism, sickle cell anemia, & Gaucher disease.

Presentation

AVN of the hip presents as referred pain to the knee and is worsened with internal rotation of the hip.

Diagnosis

MRI and bone scans.

Treatment

Surgery, replacement of joint.

www.ClinicalReview.com


Cardiovascular System

351

PRACTICE QUESTIONS Which of the following medications can be used in CABG patients to improve left ventricular function given high pulmonary artery pressure while avoiding tachycardia? A. B. C. D. E.

Dobutamine Dopamine Epinephrine Milrinone Norepinephrine

The best answer is Milrinone. Milrinone is used in CABG patients to improve left ventricular function in patients with an elevation in pulmonary artery pressure without causing any tachycardia. Although long term studies debate the effects on morbidity and mortality, it's short term benefits are helpful.

Which of the following best describes the function of amrinone, an inotrope used following cardiac surgery? A. B. C. D. E.

Activator of adenylate cyclase leading to inhibition of cAMP, vasodilation, and increased cardiac output Activator of guanylate cyclase leading to vasodilation and increased cardiac output Inhibitor of guanylate cyclase leading to vasodilation and increased cardiac output Phosphodiesterase inhibitor that activates cAMP, leads to vasodilation, and increases cardiac output Phosphodiesterase inhibitor that inhibits cAMP, leads to vasoconstriction, and increases cardiac output

The best answer is Phosphodiesterase inhibitor that activates cAMP, leads to vasodilation, and increases cardiac output. Amrinone is a phosphodiesterase inhibitor that activates cAMP, leads to vasodilation, and increases cardiac output. It is one of the inotropes, along with milrinone, that leads to utilization of catecholamine stores. The end result is an increase in myocardial oxygen consumption (typically 60%), noted by the equation: oxygen extraction ratio = (arterial oxygen - venous oxygen) / arterial oxygen.

An 84 year old male who has a myocardial infarction is transferred to the ICU. A Swan-Ganz catheter is floated and a significant elevation in PCWP is noted. What is the underlying cause? A. B. C. D. E.

Decreased CO Increased EDV Increased EDV and increased SVR Increased EDV, increased SVR, and decreased CO Increased SVR

The best answer is Increased EDV, increased SVR, and decreased CO. This patient has an increase in end diastolic volume due to cardiac hypocontractility, leading to an increase in systemic vascular resistance. The hypocontractility directly leads to a drop in cardiac output.

Â


USMLE STEP 2

352

A 76 year old male who has a myocardial infarction is brought to the ICU in cardiogenic shock. Which of the following is the best initial course of therapy? A. B. C. D. E.

Dobutamine Epinephrine Milrinone Norepinephrine Vasopressin

The best answer is Dobutamine. The best initial vasopressor is dobutamine due to its improvement in cardiac contractility without significantly increasing myocardial oxygen consumption. The result is a mitigation in ischemic damage due to failure to increase oxygenation. Hypertrophic subaortic stenosis should be treated with ACE inhibitors, beta blockers, and a possible balloon pump placement to decrease myocardial oxygen demand. Pressors that increase oxygen demand should be avoided in hypertrophic subaortic stenosis.

Which of the following best describes the most common cause of late death in patients with heart transplantation? A. B. C. D. E.

Congestive heart failure Graft failure Papillary muscle rupture Silent myocardial infarction Ventricular septal rupture

The best answer is Silent myocardial infarction. The most common cause of late death following heart transplantation is atherosclerosis leading to silent myocardial infarction. There are few initial warning signs due to denervation of the transplanted heart.

A 53 year old male presents to the ER with crushing substernal chest pain. He is diagnosed with a myocardial infarction and treatment is started immediately. Four days later, he is seen in urgent care complaining of difficulty breathing and additional chest pain. A holosystolic murmur is auscultated. A transthoracic echocardiogram is most likely to reveal which of the following? A. B. C. D. E.

Aortic dissection Atrial septal rupture Dressler syndrome Papillary muscle rupture Ventricular septal rupture

The best answer is Ventricular septal rupture. Septal rupture is more likely in anterior MI and occurs 3-5 days afterwards. It presents with a loud holosystolic murmur that radiates to many locations, and also tends to have a palpable thrill. Echocardiography is used for diagnosis. Rupture of the papillary muscles can lead to mitral regurgitation and sudden cardiac decompensation following a posteroinferior MI. It tends to occur approximately 3-5 days after an MI and presents with a loud holosystolic murmur. Echocardiography demonstrates a valve leaflet moving around with the flow and regurgitation. A ventricular septal rupture is the most common disruption, and occurs in up to 2% of patients who have a myocardial infarction. www.ClinicalReview.com


Cardiovascular System

353

Which of the following is a major contributor to blood oxygen content leading to perfusion of peripheral tissues? A. B. C. D. E.

Heart rate Hemoglobin concentration and %SO2 Hemoglobin concentration and PaO2 PaO2 PAO2

The best answer is Hemoglobin concentration and %SO2. The greatest contributor to oxygenation is hemoglobin concentration and percent of oxygen saturation within the blood. The initial treatment to improve peripheral oxygenation is to increase oxygen saturation to 100%. Further increases lead to an increase in dissolved oxygen within the blood, but this has a negligible effect at low oxygen flow levels. The next step is to give blood transfusions as clinically indicated in order to improve oxygenation. The relevant equation is 1.37 x oxygen saturation x hemoglobin concentration + 0.003 x PaO2.

A 38 year old male s/p CABGx4 is brought to the ICU in stable condition. Due to continued oozing from various surgical sites, the decision is made to reverse heparin with protamine. What is the most common complication of this maneuver? A. B. C. D. E.

Anaphylaxis Fever Hypotension Thrombosis Urticaria

The best answer is Hypotension. The most common complication with reversing heparinization with protamine is hypotension, and occurs in up to 10% of all patients. 1mg of protamine per 100 units of heparin should be given for adequate reversal. Serial PTT should be followed. LMWH can also be reversed with protamine to some extent.

Which of the following is not one of the changes that occur in cardiogenic shock? A. B. C. D. E.

Decreased CO Increased CVP Increased PCWP Increased SVO2 Increased SVR

The best answer is Increased SVO2. Cardiogenic shock includes a drop in cardiac output, an increase in central venous pressure, an increase in systemic vascular resistance to maintain blood pressure, an increase in pulmonary capillary wedge pressure due to backflow, and a decrease in SVO2. In right heart failure, CVP is equal to PCWP due faulty right ventricular contraction.

Â


USMLE STEP 2

354

A 32 year old male involved in a motorcycle accident presents to the ER with pulsus paradoxus and cardiovascular decompensation. He is found to have cardiac tamponade on an echocardiogram. What is the mechanism of his decompensation? A. B. C. D. E.

Collapse of right and left atria Collapse of the thoracic aorta Enlarged left ventricle Hemorrhagic blood loss Pulmonary collapse

The best answer is Collapse of right and left atria. This patient has tamponade, which leads to collapse of the right and left atria and impaired filling of the ventricles. During inspiration, the left ventricle collapses further while the right ventricle enlarges somewhat. Collapse of the great vessels is more likely to occur with tension pneumothorax. Blunt cardiac injuries should be monitored with telemetry. A widened mediastinum on CXR requires a CTA for diagnosis and management.

Which of the following is an effect of an AV fistula graft formed in a diabetic patient on dialysis? A. B. C. D. E.

Bradycardia Decreased SVO2 Distal thrombosis Increased cardiac output Increased peripheral vascular resistance

The best answer is Increased cardiac output. A patient who has an AV fistula has an increase in cardiac output, decreased peripheral vascular resistance, and tachycardia. The preferred choice for fistula formation is a radiocephalic fistula. The presence of high outflow occlusion should be treated with angioplasty through the venous limb.

A 65 year old female presents to surgery clinic with significant pain in her calf after walking 20 feet. She has dopplerable pulses in both extremities and ABIs of 0.5 and 0.6 (right and left). What is the next best step in management? A. B. C. D. E.

Anticoagulation Arteriogram Femoral-popliteal bypass graft MRI Smoking cessation and walking regimen

The best answer is Smoking cessation and walking regimen. This patient has symptomatic claudication with lifestyle limitations. The initial therapy is smoking cessation with an aggressive walking regimen. Control of hyperlipidemia is also important.

www.ClinicalReview.com


Cardiovascular System

355

A patient presents with a 3 cm popliteal aneurysm. What is the next step in management? A. B. C. D. E.

Anticoagulation Femoral-popliteal bypass Observation Resection of the aneurysm Stenting of the aneurysm

The best answer is Resection of the aneurysm. This patient presents with a popliteal aneurysm over 2 cm in size, which leads to a significant risk of thrombosis. The treatment is resection of the aneurysm with bypass. Dislocation of the knee can also lead to popliteal artery injury and thrombosis; therapy is with a saphenous vein graft following angiography.

A 76 year old male is taken to the OR for a 6 cm AAA that has a small leak. Which of the following leads to the formation of an abdominal aortic aneurysm? A. B. C. D. E.

Decreased MMP activity Destruction of the medial layer Ischemic wall disease Plaque rupture Release of nitric oxide

The best answer is Destruction of the medial layer. The formation of an AAA can be attributed to destruction of the medial layer of the vessel wall, secondary to an increase in matrix metalloprotease activity. AAA must be repaired when it is over 5 cm in size, becomes symptomatic or develops a leak, or grows by over ½ cm per year. Endovascular repair can be undertaken with infrarenal aneurysms and must have suitable distance between the renal artery and the beginning of the aneurysm. This is important for stent fixation. Symptomatic carotid disease must be repaired prior to AAA repair. Embolectomy is required if distal cyanosis is present at the end of the surgery. Bloody diarrhea following surgery is likely due to ischemic sigmoid disease and requires reimplantation of the IMA to prevent complications. Treatment once ischemia starts is a colonoscopy and sigmoidectomy if full thickness necrosis is present. Graft infections are most commonly attributed to S. aureus, S. epidermidis, and E. coli. Infection of a graft may present as GI bleeding.

Which of the following is the most common intra-abdominal aneurysm second to a AAA? A. B. C. D. E.

GDA aneurysm Hepatic artery aneurysm Renal artery aneurysm SMA aneurysm Splenic artery aneurysm

The best answer is Splenic artery aneurysm. Splenic artery aneurysms require resection and bypass if they are over 2 cm in size or if they occur in a gestational female. They are the second most common type of abdominal aneurysm.

Â


USMLE STEP 2

356

A 54 patient is found to have 60% stenosis in his right carotid artery. Over the past several months, he has experienced occasional blurry vision and a brief limitation in function of his left hand. What is the next best step in management? A. B. C. D. E.

Anticoagulation Bilateral CEA CABGx4 Observation Right CEA

The best answer is Right CEA. This patient has symptomatic carotid disease with over 50% stenosis, which is treated by carotid endarterectomy. The right side should be corrected first as it is the symptomatic side. Asymptomatic disease with over 70% blockage should be surgically corrected. Simultaneous CABG and CEA can be conducted if there is critical stenosis of the coronary vessels. Angioplasty is the treatment of choice if a younger patient presents with carotid disease, as it is likely secondary to fibromuscular dysplasia. Fibromuscular dysplasia may present as renal artery stenosis, which is treated by stent placement. Recurrent disease may be due to myointimal hyperplasia. The presence of stroke following a CEA requires re-exploration; stroke hours after a CEA may be due to microembolisms and requires a CT of the head for diagnosis and staging. The most commonly injured nerve following a CEA is the vagus, and leads to hoarseness. Dissection of the carotid artery is treated with anticoagulation.

A 62 year old male with longstanding vascular disease presents with claudication of his calf muscles. The patient presents with warm feet but significant erythema. Femoral pulses are 2+, but the dorsalis pedis and posterior tibial arteries are only dopplerable as biphasic signals. What is the next best step in treating this patient? A. B. C. D. E.

Aggressive ambulation therapy Anticoagulation Below knee amputation Femoral arteriography Femoral-popliteal bypass graft

The best answer is Aggressive ambulation therapy. This is a patient who likely has progressive vascular disease leading to femoral-popliteal occlusive disease. This is the most common lower extremity arterial occlusive disease, and occurs in up to 50% of all cases of vascular disease. The major indications for surgery include significant lifestyle limitation in the setting of endangered limb. This patient does not meet these criteria, as the presence of dopplerable pulses indicates that the limb is still being perfused. An aggressive walking regimen and amelioration of other risk factors is the recommended therapy. In the setting of continued compromise, arteriography is a veritable course of action.

www.ClinicalReview.com


Cardiovascular System

357

CHAPTER CONTENTS Basic Science .....................................................................................................358 Esophagus .........................................................................................................359 Stomach ............................................................................................................369 Small Intestine ..................................................................................................378 Large Intestine ..................................................................................................384 Hernias ..............................................................................................................402 Practice Questions ............................................................................................404

Â


USMLE STEP 2

358

GASTROINTESTINAL SYSTEM GASTROINTESTINAL SYSTEM BASIC SCIENCE PHARMACOLOGY ANTIHISTAMINES – H1 DRUG

INDICATIONS Akathisia prophylaxis Sedative Allergic reactions

Diphenhydramine

Antiemetic Sedative

Promethazine

MECHANISM OF ACTION

COMPLICATIONS

H1 receptor blocker; reduces smooth muscle contraction along with CNS effects Also acts as SSRI

Anticholinergic effects leading to drowsiness, psychomotor agitation, ataxia, xerostomia, flushing, tachycardia, cycloplegia, photophobia, urinary retention, constipation, hallucinations, and delirium Severe muscarinic acetylcholine antagonism that may lead to MI in overdose

H1 receptor blocker

Sedation and confusion with anticholinergic effects May lead to EPS with long-term use NMS Fatal respiratory depression in infants

ANTIHISTAMINES – H2 DRUG Cimetidine

INDICATIONS

MECHANISM OF ACTION

GERD PUD

H2 receptor antagonist to prevent stomach acid production

COMPLICATIONS Interaction with numerous drugs via P-450

CONTRAINDICATIONS Hepatic disease

PROTON PUMP INHIBITORS DRUG

INDICATIONS PUD, GERD ZE syndrome H. pylori infection

Omeprazole

MECHANISM OF ACTION Proton pump inhibitor that prevents hydrochloric acid secretion from parietal cells

COATING AGENTS DRUG Bismuth

Sucralfate

INDICATIONS H. pylori PUD

MECHANISM OF ACTION

NOTES

Antagonist to H. pylori

Given with clarithromycin, amoxicillin (or metronidazole) and PPI for H. pylori treatment

Sucrose plus sulfate and aluminum that lines the stomach and protects the epithelium

Not as commonly used as PPIs and H2 blockers due to low long-term effectiveness

www.ClinicalReview.com


GASTROINTESTINAL SYSTEM

359

OTHER AGENTS DRUG

INDICATIONS

MECHANISM OF ACTION

Infliximab

Crohn disease RA

Decreases TNF-α and reduces its effect

Leukopenia, thrombocytopenia, neutropenia

Avoid in active infection

IBD RA

Reduces inflammation through 5-ASA metabolite

Diarrhea, HA

Avoid in sulfa hypersensitivity

PUD Induce labor Abortifacient

PGE1 analog

N/V/D/F

Sulfasalazine

Misoprostol

COMPLICATIONS

CONTRAINDICATIONS

Salicylic Acid

ESOPHAGUS ESOPHAGEAL DISORDERS DYSPHAGIA AND ODYNOPHAGIA Esophageal disorders can present with symptoms of dysphagia, indicating that the patient has difficulty swallowing foods and / or liquids. Dysphagia is often a symptom of esophageal diseases, and is often an element of an underlying defect in normal esophageal transport. Odynophagia is painful swallowing; the presence of pain often indicates an underlying inflammatory disorder of the esophagus such as those that occur in certain viral etiologies.

ACHALASIA EPIDEMIOLOGY Achalasia is due to increased LES tone and failure of normal peristalsis of the esophagus, leading to severe dysphagia with solid foods and liquids. Achalasia affects males and females equally. A study in England rated the incidence of achalasia as affecting about 1 in 100,000 patients.

PATHOPHYSIOLOGY Achalasia stems from a loss of conducting neurons in the esophagus. These neurons are responsible for maintaining the normal progression of peristalsis. Loss of these neurons within Auerbach’s plexus is due to scarring, and the primary cause of the scarring


USMLE STEP 2 360 is presently being explored. Secondary causes of this scarring can be attributed to invasive lymphoma or gastric carcinoma, scleroderma, and Chagas disease (note on right: Chagas leads to DCM, toxic megacolon, and achalasia).

PRESENTATION Many of the signs and symptoms of achalasia are secondary to the severe dysphagia that accompanies this disease. Weight loss often results, along with dysphagia with both solids and liquids, and regurgitation of food. The denervation of the esophagus combined with difficulty with normal movements can also exacerbate a cough, and lead to a diffuse chest pain, especially after the consumption of food. Contractions of the esophagus often resemble simultaneous small waves from superior to inferior.

DIAGNOSIS Diagnosis of achalasia is made by barium swallow, which identifies a dilated esophagus and a bird-beak narrowing at the inferior aspect. The narrowing at the inferior aspect occurs as a result of failure of the LES to relax. Manometry is also used to diagnose achalasia, and findings include a normal to high pressure at the LES with no change after swallowing. Esophagogastroduodenoscopy (EGD) is required to rule out gastric carcinoma and lymphoma, two infrequent causes of achalasia (about 1% of patients with gastric carcinoma have achalasia). The typical gastric bubble is also missing on plain films of the abdomen.

TREATMENT Medical treatments for achalasia include: the use of nitroglycerin to relieve symptoms; injection of botulinum toxin into the LES, to block acetylcholine-induced muscle contraction; and calcium-channel blockers, to treat on a symptomatic basis. Botulinum toxin injections are successful in about 2/3 patients, but must be repeated over time. Achalasia is surgically treated with pneumatic dilation at the LES that tears the muscle fibers (and thereby decreases LES pressure), and permits the semi-normal transport of food and liquid to the stomach. Balloon dilation of the LES can cause perforation in about 1-2% of patients. TABLE 426 ACHALASIA Achalasia Pathophysiology

Loss of conducting neurons with increased LES tone; failure of normal peristalsis; severe dysphagia; weight loss; cough; diffuse chest pain.

Diagnosis

Barium swallow (bird-beak narrowing in inferior esophagus); manometry (normal to high pressure at LES), EGD to rule out gastric carcinoma; gastric bubble absent on plain film.

Treatment

Nitroglycerin to relieve symptoms, calcium channel blockers, botulinum into LES, pneumatic dilation of LES.

DIFFUSE ESOPHAGEAL SPASM (DES) PATHOPHYSIOLOGY AND PRESENTATION DES is a generalized spasm of the esophagus secondary to failure of activity of the inhibitory neurons within Auerbach’s plexus. Spontaneous contractions of the esophagus occur that do not resemble peristalsis. The diffuse contractions in DES lead to dysphagia and chest pain.

www.ClinicalReview.com


GASTROINTESTINAL SYSTEM

361

DIAGNOSIS Diagnosis of DES is made by barium swallow and manometry. Barium swallow indicates a corkscrew-like pattern due to the uncoordinated activity of various parts of the esophagus. Large waves are seen after swallowing. Manometry confirms the uncoordinated activity of the esophagus and detects high amplitude contractions that may extend into the upper 1/3 of the esophagus. The LES is typically at normal or slightly low pressure.

TREATMENT Fewer treatments are available for DES than for achalasia due to the lack of ameliorative effect of changing the behavior of the LES. DES sometimes responds to calcium-channel blockers, nitroglycerin, and anticholinergics. TABLE 427 DIFFUSE ESOPHAGEAL SPASM Diffuse Esophageal Spasm (DES) Pathophysiology

Spasm of esophagus, dysphagia, chest pain.

Diagnosis

Barium swallow (corkscrew-like pattern) & manometry (high amplitude contractions in upper 1/3 of esophagus).

Treatment

Calcium channel blockers, nitroglycerin, and anticholinergics.

NUTCRACKER ESOPHAGUS PATHOPHYSIOLOGY AND PRESENTATION Nutcracker esophagus is due to high-amplitude contractions that arise from increased activity of the neurons within the esophageal plexuses. It presents similarly to DES, with dysphagia, diffuse chest pain, and large waves of contraction.

DIAGNOSIS AND TREATMENT Nutcracker esophagus is diagnosed by manometry, which indicates high-amplitude peristaltic contractions. Barium swallow distinguishes between nutcracker esophagus and DES, as the latter has a corkscrew appearance. Treatment is primarily symptomatic and similar to that of DES with the use of nitroglycerin, calcium-channel blockers, and anticholinergic. TABLE 428 NUTCRACKER ESOPHAGUS Nutcracker Esophagus Pathophysiology

High-amplitude peristaltic contractions, dysphagia, chest pain.

Diagnosis

Manometry (high-amplitude contractions). Barium swallow (differentiates from DES in that DES has corkscrew appearance Nutcracker does not).

Treatment

Same as with DES.

INFECTIOUS ESOPHAGITIS ETIOLOGY AND EPIDEMIOLOGY

Â


USMLE STEP 2 362 Esophagitis is often accompanied by odynophagia, or pain upon swallowing. This is due to irritation of the surrounding tissue in conjunction with swelling and abrasion of the superficial layers of the epithelium. Esophagitis is often secondary to infection by viral, bacterial, or fungal causes, and also to a number of environmental causes. Esophagitis is most common in immunocompromised patients, especially those with HIV.

PATHOPHYSIOLOGY Viral causes of esophagitis include herpes simplex virus (HSV), cytomegalovirus (CMV), varicella-zoster virus (VZV), and human immunodeficiency virus (HIV). Bacterial causes include Streptococcus, Cryptosporidium, Lactobacillus, Pneumocystis carinii, and Mycobacterium tuberculosis. Fungal causes are chiefly due to Candida. Esophagitis may also be secondary to radiation exposure, corrosion from certain medications taken without water (e.g. acetylsalicylic acid), nonsteroidal anti-inflammatory drugs, doxycycline, alendronate, iron sulfate, quinidine, and corrosion from toxic substances such as strong acids or bases. In the latter situation, strictures and webs often form secondary to the intense damage that often occurs.

PRESENTATION Signs and symptoms of esophagitis include odynophagia, dysphagia, nausea and vomiting (often secondary to infection), bleeding (often secondary to corrosion and destruction of the epithelium), and chest pain. Esophagitis may also be entirely asymptomatic.

DIAGNOSIS Diagnosis of esophagitis requires a careful history to help elucidate the offending cause. Infection by Candida often presents in HIV patients and with a nodular filling defect observed on a barium swallow. HSV and VZV present with vesicles and erosions when examined through endoscopy. CMV presents with intranuclear inclusions on examination of biopsied specimens. The best method of diagnosing the offending cause is through response to therapy – this is often the case with HIV patients complaining of dysphagia and odynophagia who are treated with fluconazole for putative Candida infection.

TREATMENT Treatment of esophagitis depends on the underlying cause. Candida is treated with oral fluconazole. HSV is treated with acyclovir, and CMV is treated with gancyclovir. Additional antibiotics, antivirals, and antifungals are used as indicated. Prevention is usually the best method to treat medication-induced esophagitis by flushing those medications with water during consumption. TABLE 429 INFECTIOUS ESOPHAGITIS Infectious Esophagitis Pathophysiology

Viral, bacterial, fungal, radiation, medications, toxic substances.

Presentation

Odynophagia, dysphagia, nausea & vomiting, bleeding, chest pain.

Diagnosis & treatment

Identify underlying cause and treat with appropriate antiviral, antibiotic, and/or antifungal.

GERD EPIDEMIOLOGY According to a journal article in “Gastroenterology”, gastroesophageal reflux disease (GERD) affects nearly 20% of the population on a weekly basis, and nearly 40% of people on a monthly basis. The American Gastroenterological Association estimates that the www.ClinicalReview.com


GASTROINTESTINAL SYSTEM

363

public health burden of GERD is over $10 billion dollars a year in direct and indirect costs. The direct cost to the patient is also serious. GERD increases the risk of developing Barrett’s esophagus and subsequently, esophageal adenocarcinoma.

PATHOPHYSIOLOGY Gastroesophageal reflux disease is predominantly due to incompetence of the lower esophageal sphincter (LES), leading to reflux of stomach contents into the esophagus. Other causes include the presence of a hiatal hernia, delayed emptying of the stomach, and decreased motility of the esophagus.

LES INCOMPETENCE Failure of the LES to properly function may be attributed to one or more causes. The high level of progesterone in pregnancy contributes a great deal to the symptoms of heartburn and substernal burning that causes many pregnant women suffering. Consumption of acidic foods and fatty foods, in addition to chocolate, peppermint, alcohol, and coffee can exacerbate LES dysfunction and potentiate the symptoms of GERD. Smoking has also been tied to a decrease in LES tone. Finally, a variety of medications that have effects on muscle tone, such as calcium-channel blockers, β-blockers, nitrates, anticholinergics, and theophylline have been implicated as having a negative effect on maintaining the tone of the LES.

PRESENTATION The signs and symptoms of GERD include heartburn, dysphagia, increased salivation, cough, and asthma-like symptoms. The hydrochloric acid secreted by the stomach travels up the esophagus due to LES incompetence, and leads to a burning feeling in the epigastric or substernal region, known as heartburn. This pain may occasionally present as chest pain, and so GERD should be on the differential diagnosis of angina and chest pain. Dysphagia in GERD is often a result of the formation of anatomic defects in the esophagus, such as webs and strictures. Increased salivation in GERD leads to a water brash. Cough in GERD is often dry and non-productive. GERD may also exacerbate underlying asthma or may present as symptoms of asthma in certain individuals.

DIAGNOSIS Diagnosis of GERD is made by a careful consideration of the history and confirmed through the use of a 24-hour pH probe, barium swallow, esophagoscopy, and biopsy. The latter tests are more commonly used in long-standing cases of GERD to rule out histologic changes that may lead to the formation of adenocarcinoma and to detect any underlying anatomical defects.

TREATMENT GERD is treated by inducing lifestyle changes in the patient in conjunction with the use of medications. In patients highly compliant with lifestyle changes, the use of medications may not be necessary. Lifestyle modifications include obese patients losing weight, promoting the consumption of smaller meals to avoid overburdening the stomach and thereby minimizing reflux, elevating the head of the bed to use gravity to keep food in the stomach, discontinuing the foods mentioned above, cessation of smoking, and eating several hours before bedtime to allow food to travel to the small intestine.

MEDICATIONS The first line of medications that is often tried by patients is oral antacids, such as Mylanta and Tums. In GERD that is refractory to lifestyle changes and over-the-counter medications, H2 blockers and/or proton pump inhibitors are commonly used. H2 blockers tend to be curative in about half of all patients, while proton pump inhibitors are even more successful. In patients with an


USMLE STEP 2 364 identified anatomical defect, or those who are refractory to medical management, Nissen fundoplication can be used to position the stomach around the lower esophageal sphincter to allow the normal contractions of the stomach and to keep the LES closed.

COMPLICATIONS GERD is commonly thought of as a relatively benign complaint, but serious complications can develop over a long period of time, if GERD is not appropriately managed. GERD can lead to esophageal damage, which can lead to bleeding and friability. These inflammatory changes are known as esophagitis. In about 10% of patients, peptic strictures can form that further heighten the symptoms of dysphagia. Columnar cell metaplasia of the lower 2/3’s of the esophagus can also occur in a condition known as Barrett’s esophagus. This transformation from the normal squamous cell epithelium predisposes individuals to developing adenocarcinoma of the esophagus. Finally, continuing smoking in the face of GERD, in conjunction with damage to the upper 1/3 of the esophagus, can lead to the development of squamous cell carcinoma. The use of certain medications for the treatment of GERD can also lead to decreased B12 absorption and impaired absorption of other medications. TABLE 430 GASTROESOPHAGEAL REFLUX DISEASE Gastroesophageal Reflux Disease (GERD) Pathophysiology

Incompetence of LES leading to gastric reflux into esophagus (pregnancy, acidic and fatty food, smoking, medications); hiatal hernia; decreased motility of esophagus, delayed gastric emptying.

Presentation

Heartburn, dysphagia, increased salivation, cough, asthma-like symptoms.

Diagnosis

History, pH probe, barium swallow, esophagoscopy, biopsy.

Treatment

Weight loss in the obese, smaller meals, elevating the head of the bed, avoiding certain foods, medications (H2 blockers, proton pump inhibitors, oral antacids).

ANATOMIC ESOPHAGEAL DEFECTS ESOPHAGEAL OBSTRUCTIONS EPIDEMIOLOGY AND PATHOPHYSIOLOGY Plummer-Vinson syndrome and Schatzki rings are two of the most common anatomic esophageal obstructions that can occur. Plummer-Vinson syndrome is characterized by hypopharyngeal webs in conjunction with iron-deficiency anemia. It is most common in middle-aged women. Plummer-Vinson syndrome presents with complaints of dysphagia immediately after swallowing food. Schatzki rings are narrow mucosal rings located in the lower esophagus just proximal to the LES with concomitant dysphagia. Schatzki rings affect both males and females, and appear to target individuals with pre-existing symptoms of intermittent dysphagia. Nearly 10% of Schatzki rings are asymptomatic.

DIAGNOSIS AND TREATMENT Both Plummer-Vinson syndrome and Schatzki rings are diagnosed with barium swallow. Treatment is through balloon dilation of the rings, which is the preferred method of treating Schatzki rings. Plummer-Vinson syndrome often requires more involved surgical management. Complications of Plummer-Vinson syndrome include an elevated risk of squamous cell carcinoma.

TABLE 431 ESOPHAGEAL OBSTRUCTIONS www.ClinicalReview.com


GASTROINTESTINAL SYSTEM

365

Scleroderma Etiology

Plummer-Vinson-- hypopharyngeal webs in conjunction with iron-deficiency anemia Schatzki rings-- narrow mucosal rings located in the lower esophagus

Diagnosis/ Treatment

Barium swallow, balloon dilation, surgery

Complications

Plummer-Vinson—elevated risk of squamous cell carcinoma

ZENKER DIVERTICULUM ETIOLOGY AND PATHOPHYSIOLOGY Zenker diverticulum is the presence of a pouch extending outside of the esophagus due to a defect in the muscular layer below the epithelium. It is most common in the posterior hypopharynx where the muscular layer is thinner and in the upper 1/3 of the esophagus.

PRESENTATION AND DIAGNOSIS Zenker diverticulum presents with halitosis, aspiration of ingested contents through regurgitation of food, and esophageal obstruction leading to dysphagia. Diagnosis is through barium swallow which identifies the outpouching as a contrast-filled region. Endoscopy and intubation should not be done due to the risk of perforation.

TREATMENT Treatment of Zenker diverticulum is through surgical removal, including procedures such as cricopharyngeal myotomy or diverticulectomy. TABLE 432 ZENKER DIVERTICULUM Zenker Diverticulum Pathophysiology

Pouch extending outside of the esophagus due to a defect in the muscular layer of the epithelium. Typically in posterior hypopharynx.

Diagnosis

Halitosis, aspiration of food, dysphagia. Diagnose through barium swallow.

Treatment

Surgical removal.

ESOPHAGEAL PERFORATION OVERALL ETIOLOGY AND EPIDEMIOLOGY Perforation or rupture of the esophagus is commonly due to trauma to the esophagus, whether through medical procedures or following severe vomiting. Common iatrogenic causes include endoscopy, esophageal dilation, the use of Blakemore tubes, intubation of the esophagus instead of the trachea, and the use of nasogastric (NG) tubes. Leakage of air and ingested contents may occur into the mediastinum, causing mediastinitis and rampant infection. Mortality following perforation or rupture occurs in up to 50% of patients, and is commonly due to uncontrolled hemorrhage.

Â


366

USMLE STEP 2

TABLE 433 ESOPHAGEAL PERFORATION Esophageal Perforation Etiology

Trauma to the esophagus (endoscopy, vomiting, dilation, NG tubes, improper intubation).

Presentation

Uncontrolled hemorrhage.

BOERHAAVE SYNDROME Boerhaave syndrome is a full-thickness tear of the esophagus typically located in the left posterolateral portion of the distal 1/3 of the esophagus. Full-thickness tears are commonly due to forceful vomiting and retching, strong and chronic cough, heavy lifting or physical labor, and direct trauma. Presentation of Boerhaave syndrome is typically with unremitting bleeding and hematemesis that quickly leads to death. TABLE 434 BOERHAAVE SYNDROME

Boerhaave Syndrome Pathophysiology

Full thickness tear of esophagus located in left, posterolateral, distal 1/3.

Etiology

Forceful vomiting, strong cough, heavy lifting, direct trauma.

Presentation

Bleeding, hematemesis. Severe pain that radiates to the chest, back, and abdomen.

Diagnosis

CXR (identifies pleural effusion and emphysema); esophagogram; endoscopy.

Treatment

Pressure to stop hemorrhage, surgical repair.

MALLORY-WEISS SYNDROME The most common site of the perforation in Mallory-Weiss syndrome is through the squamocolumnar junction near the LES. This is a partial-thickness tear that is commonly on the right posterolateral portion of the distal 1/3 esophagus. Presentation of Mallory-Weiss syndrome occurs with transient bleeding and is often secondary to forceful vomiting and retching.

GENERAL PRESENTATION Boerhaave and Mallory-Weiss syndromes generally present with the sudden onset of severe pain that radiates to the chest, back, and abdomen. Dyspnea sometimes occurs, especially with bleeding into the pleural cavity. Subcutaneous emphysema and mediastinal emphysema are often present in Boerhaave syndrome due to the leakage of air from the esophagus into those cavities. Mediastinal emphysema presents with a rapid crunching sound known as Hammon’s crunch, and is due to expansion of the pericardium with every heart beat against air trapped in the region. Mallory-Weiss syndrome may present with dysphagia.

DIAGNOSIS AND TREATMENT Diagnosis of esophageal perforation or rupture is often done with a chest x-ray (CXR) that identifies a pleural effusion with mediastinal, pleural, or subcutaneous emphysema. An esophagogram with water-soluble contrast (Gastrograffin) can be done to identify movement of the contrast out of the esophagus and into a cavity in the thorax. Barium cannot be used due to the risk of chemical mediastinitis. Endoscopy and other studies may also be done. Endoscopy is sometimes preferred due to the ability to exert pressure on the bleeding in an attempt to stop the hemorrhage. Emergent surgical repair is required for all episodes of Boerhaave syndrome.

www.ClinicalReview.com


GASTROINTESTINAL SYSTEM

367

TABLE 435 MALLORY-WEISS SYNDROME Mallory-Weiss Syndrome Pathophysiology

Perforation through the squamocolumnar junction near the LES. Transient bleeding, vomiting.

Presentation

Sever pain that radiates to chest, back and abdomen, dysphagia.

Diagnosis

CXR (identifies pleural effusion and emphysema); esophagogram; endoscopy.

Treatment

Pressure to stop hemorrhage, surgical repair.

ESOPHAGEAL CANCER BARRETT ESOPHAGUS PATHOPHYSIOLOGY Barrett’s esophagus is the transformation of squamous cell epithelium to mucus-producing columnar cell epithelium. This transformation is likely secondary to an attempt by the body to protect the lower esophagus from continuing damage due to acid reflux from an incompetent LES. Barrett’s esophagus occurs due to chronic reflux and over a period of time of continuing symptoms. These metaplastic changes are most common in patients with GERD, affecting some 20% of these patients. Barrett’s metaplasia is a strong positive predictor of adenocarcinoma, carrying a relative risk of 30 times that of normal.

DIAGNOSIS AND TREATMENT Diagnosis of Barrett’s esophagus is made by examining biopsied specimens for columnar cell epithelium superior to the LES. Treatment is to limit the symptoms of GERD, with medications if possible but surgery if the reflux is refractory to medical management. Repeated EGDs should be done after diagnosis. If no dysplastic changes are present, EGD should be repeated every 5 years. If low-grade dysplasia is present, EGD should be repeated twice a year. High-grade dysplasia is a strong predictor of adenocarcinoma, and prophylactic esophageal resection should be done. The risk of adenocarcinoma is worsened with concomitant Barrett’s esophagus with metaplasia extending about six inches, dysplastic changes, and a history of smoking. TABLE 436 BARRETT ESOPHAGUS Barrett Esophagus Pathophysiology

Metaplasia (squamous cell epithelium to mucous producing columnar epithelium).

Diagnosis

Biopsy for columnar epithelium. Limit symptoms of GERD with medications or surgery. Multiple EGDs.

ESOPHAGEAL CARCINOMA EPIDEMIOLOGY AND ETIOLOGY The most common type of esophageal carcinoma is adenocarcinoma, which is typically associated with Barrett’s esophagus and occurs more frequently in Caucasian males. Squamous cell carcinoma is more common in African American males over the age of 40 and with a history of smoking. Adenocarcinoma is more likely to occur in the distal 1/3 of the esophagus, while squamous cell carcinoma is more likely in the proximal 1/3 of the esophagus. Adenocarcinoma and squamous cell carcinoma have approximately equal incidence in the middle 1/3 of the esophagus.


368

USMLE STEP 2

PATHOPHYSIOLOGY Squamous cell carcinoma has a number of risk factors, including a history of alcohol consumption, a long-standing history of smoking, esophageal motility disorders such as achalasia, anatomic defects such as Plummer-Vinson syndrome, and consumption of carcinogens such as foods rich in nitrates and certain spices. The largest positive predictive value of squamous cell carcinoma risk is smoking. Adenocarcinoma has been discussed in previous sections of this chapter.

PRESENTATION With squamous cell carcinoma and adenocarcinoma, the esophageal cancer spreads inward circumferentially to cause progressive dysphagia. Initially, the dysphagia is with solid foods, but it steadily progresses to affect the transport of liquids in the esophagus. The decrease in food consumption, in addition to the effects of the cancer, leads to weight loss, poor nutritional status, and referred pain substernally or to the back. Due to the nature of squamous cell carcinoma, hypercalcemia is sometimes present and can be diagnosed through laboratory tests.

DIAGNOSIS Diagnosis of esophageal cancer is initially made through barium swallow studies, which will detect jagged edges inside the esophagus representing rows of cancerous cells. Biopsy through EGD is required to establish the diagnosis.

TREATMENT The primary treatment for esophageal cancer is with surgical resection, a procedure that carries a high mortality rate. Chemotherapy is standard with a platinum-agent such as cisplatin, and 5-fluorouracil (5-FU). Radiation therapy is also used to help prevent recurrence. Even with all of these measures, esophageal cancer has a particularly poor prognosis and very few people still survive after 5 years. TABLE 437 ESOPHAGEAL CARCINOMA Esophageal Carcinoma Pathophysiology

Squamous cell carcinoma risk factors: smoking, alcohol, achalasia, carcinogens.

Diagnosis

Barium swallow (detects jagged edges); biopsy through EGD.

Treatment

Surgical resection, chemotherapy (5-FU, cisplatin), radiation.

SCLERODERMA PATHOPHYSIOLOGY AND PRESENTATION Scleroderma occasionally presents with the CREST syndrome, including calcinosis, Raynaud phenomenon, esophageal dysmotility, sclerodactyly, and telangiectasis. The esophageal dysmotility that may occur with scleroderma and the CREST syndrome is similar to achalasia in that fibrotic changes of the esophagus lead to dysphagia and defects in peristalsis. Unlike achalasia, the LES is incompetent. Esophageal dysmotility in scleroderma presents with progressive dysphagia from solids to liquids and GERD from LES incompetence.

www.ClinicalReview.com


GASTROINTESTINAL SYSTEM

369

DIAGNOSIS AND TREATMENT Esophageal dysmotility in scleroderma is diagnosed with a barium swallow, which indicates decreased peristalsis in the lower 2/3’s of the esophagus, and decreased smooth muscle contraction on manometry and motility studies. Treatment consists of minimizing the symptoms of GERD, but there is no known treatment for avoiding the esophageal dysmotility. Scleroderma is discussed in more detail in a later chapter. TABLE 438 SCLERODERMA Scleroderma Pathophysiology

CREST syndrome.

Diagnosis

Barium swallow (decreased peristalsis); manometry (decreased motility). Symptomatic treatment as in GERD.

STOMACH ANATOMY STOMACH The stomach is composed of three smooth muscle layers. The outer longitudinal muscle, the middle circular muscle, and inner oblique muscle layers work in tandem to assist with digestion. The longitudinal layers are found predominantly along the lesser and greater curvatures. The circular muscle is found throughout the stomach, but is especially found at the pylorus. Like the remainder of the gastrointestinal tract, stimulation is primarily by the parasympathetic fibers. At the stomach, these fibers emanate from the vagus nerve. These fibers initiate normal stomach contractions via a pacemaker located in the greater curvature at the body. The phases of contraction are similar to that of the MMC, discussed below. Relaxation of the stomach occurs with CCK, distention of the duodenum, and the presence of glucose in the duodenum. Atonic gastritis and abnormal distention and failure to empty can occur in the postoperative patient due to electrolyte disturbances, hyperglycemia, and uremia.

VASCULAR The blood supply to the gastric pouch following esophagogastrectomy is carried by the right gastroepiploic artery. The SMA crosses at the junction of D3 and D4. The branches of the gastroduodenal artery (GDA) are the superior pancreaticoduodenal artery (which separates into an anterior and posterior branch), the supraduodenal artery, the right gastroepiploic artery, and retroduodenal artery. The right gastric artery arises from the hepatic artery, but is sometimes a branch of the GDA. The most common anomalous artery related to the liver is the common hepatic artery, which may arise from the SMA. The right hepatic artery also sometimes arises from the SMA. The left hepatic artery may arise from the left gastric artery. The blood supply to the duodenum consists of the superior and inferior pancreaticoduodenal arteries (from GDA and SMA, respectively), and part of the gastroduodenal artery and its branches. The right gastroepiploic and right gastric artery also send collaterals to the duodenum.

PHYSIOLOGY Acid secretion in the stomach is a result of stimulation of parietal cells to produce hydrochloric acid. To offset the production of hydrochloric acid, bicarbonate is also produced by the parietal cells and secreted into the venous circulation of the stomach, leading to the formation of an alkaline tide with every meal. Bicarbonate is also secreted into the lining of the stomach to aid in the formation of the protective barrier. Acid production is stimulated by gastrin, histamine, and acetylcholine from the vagus nerve, and the production of gastrin itself is stimulated by gastrin-releasing peptide (GRP) and inhibited by somatostatin. The

Â


USMLE STEP 2 370 protective barrier is enhanced through elevated levels of prostaglandin E (PGE), and reduced through the action of non-steroidal anti-inflammatory agents (NSAIDs) and steroids. Bicarbonate production is also negatively affected by NSAIDs, and also by acetazolamide, alcohol, and alpha-adrenergic blockers.

SALIVA Saliva, gastric acid, and small intestine fluid have very high levels of potassium. Overall, saliva has the highest amount of potassium at 1170mg, followed by gastric acid at 1120mg.

MUCUS Epithelial cells located in the gastric mucosa secrete mucus. This secretion, along with the secretion of bicarbonate, are inhibited by aspirin. Therefore, the mechanism by which aspirin could potentially contribute to damage to the gastric lining is through inhibition of production of protective factors.

GASTRIN The release of gastrin is inhibited by antral and duodenal acidification as part of a feedback reaction.

SECRETIN Secretin leads to the release of bicarbonate-rich and chloride-poor fluid from the pancreas. This is due to the exchange of bicarbonate for chloride; the effect is neutralization of the acidic contents entering into the duodenum from the stomach. Secretin also stimulates the release of pepsin and the inhibition of gastrin release, and mediates its effects through the second messenger cAMP. Without stimulation, the normal constituents of pancreatic fluid are sodium and potassium at a concentration iso-osmolar to that found in the serum.

CCK CCK is a 33 amino acid peptide hormone that stimulates fat and protein digestion. It is produced by the I-cells of the duodenum, and also leads to the secretion of Secretin: Bicarbonate-rich pancreatic fluid and enzymes. digestive enzymes by the pancreas and bile through gallbladder contraction. CCK CCK: GB contraction, pancreatic suppresses hunger, and it has recently been implicated as playing a significant enzyme secretion. role in drug tolerance to opioids. CCK may be involved in the hypersensitivity to pain experienced during opioid withdrawal. CCK also relaxes the sphincter of Oddi, stimulates secretin release, and stimulates intestinal motility. CCK production is stimulated by long-chain lipids entering the duodenum. Trypsin inactivates CCK.

SUBSTANCE P Substance P has been implicated in playing a role in inhibiting small bowel motility.

www.ClinicalReview.com


GASTROINTESTINAL SYSTEM

371

ACID PRODUCTION Gastric acid is produced by the parietal cells in the stomach. Parietal cells release bicarbonate into the blood stream, known as the alkaline tide, during this process. Carbonate anhydrase plays a role in catalyzing the reaction between CO2 and H2O, forming carbonic acid that later dissociates into hydrogen ions and hydrogen carbonate ions. Gastric acid secretion has three phases - the cephalic phase that releases 30% of the total acid in anticipation of eating, the gastric phase that releases 60% of the total acid during digestion, and the intestinal phase that releases 10% of the acid during the secretion of chyme into the small intestine.

INTRINSIC FACTOR Parietal cells produce intrinsic factor, which plays a role in binding to vitamin B12 and promoting its absorption in the ileum.

GASTRITIS AND ULCER DISEASE GASTRITIS ETIOLOGY Gastritis is divided into chronic gastritis and erosive gastritis. Chronic gastritis is further divided into type A chronic gastritis and type B chronic gastritis. Type A chronic gastritis is caused by autoimmune disorders, atrophic gastritis, pernicious anemia, and achlorhydria. Type B chronic gastritis is caused by Helicobacter pylori infection. Type B chronic gastritis is the most common type of gastritis (see below). Erosive gastritis is typically a result of NSAID use, alcohol abuse, and is commonly found in very sick patients, typically in an intensive care setting.

PATHOPHYSIOLOGY Type A chronic gastritis can be due to the presence of antibodies against parietal cells, leading to the disruption of normal acid production and secretion by the stomach in addition to low levels of intrinsic factor. This leads to very high levels of gastrin. If intrinsic factor is no longer produced, pernicious anemia can result in which vitamin B12 absorption ceases to be efficient. This leads to a decrease in vitamin B12 levels in the body, leading to megaloblastic anemia. Pernicious anemia is tied to gastric adenocarcinoma. Type B chronic gastritis is distinct from type A chronic gastritis in that there is increased acid secretion by the stomach. The effects of erosive gastritis stem from disruption of the normal protective barrier of the stomach.

PRESENTATION Gastritis presents with abdominal pain that commonly worsens immediately after the ingestion of food. This leads to weight loss, early satiety, nausea, and vomiting. If there is serious erosion of the stomach lining, gastrointestinal bleeding can also develop. Combined with any deleterious effects from destruction of parietal cells, signs and symptoms of anemia may also develop.

Â


372

USMLE STEP 2

DIAGNOSIS A good history is typically the first step towards diagnosis of gastritis and for differentiating between the various types. Upper endoscopy can then be used to confirm the clinical suspicion. Gastritis caused by H. pylori (type B chronic gastritis) requires either histologic, serologic, or laboratory testing to confirm the findings. The best test is with biopsy of the antrum of the stomach. Highly sensitive and specific tests are ELISA serologic tests that use IgG and urease laboratory tests that use either culture or the urea breath test.

TREATMENT Treatment of type A chronic gastritis requires the use of vitamin B12 injections to maintain stable levels and avoid pernicious anemia. Type B chronic gastritis is treated with a triple therapy including bismuth salts, amoxicillin, and metronidazole. If the patient is allergic to penicillin, amoxicillin can be substituted with clarithromycin. Erosive gastritis is treated similarly to peptic ulcer disease, which is discussed separately. TABLE 439 GASTRITIS Gastritis Etiology

Chronic (types A & B) Type A due to Auto immune disorders and others. Type B due to H. pylori Erosive due to NSAIDs, & alcohol abuse.

Differential

Type A-Antibodies against parietal cells, low levels of acid. Type B-H. pylori, increased acid secretion. Erosive-disruption of protective barrier of stomach.

Diagnosis

Confirm H. pylori and endoscopy.

Treatment

Type A-B12 injections; Type B- triple therapy; Erosive—treat like PUD.

PEPTIC ULCER DISEASE (PUD) ETIOLOGY AND EPIDEMIOLOGY PUD is divided into gastric ulcers and duodenal ulcers. Generally, PUD is best thought of as a disruption in the protective barrier of the stomach or duodenum, leading to erosion and subsequent damage to the underlying epithelium. PUD is twice as common in men and has been tied to more stressful personality types (“Type A” personality). The incidence of PUD increases with age, and the largest positive predictors of risk for ulcer development are alcohol use and smoking.

PATHOPHYSIOLOGY PUD is a consequence of the loss of integrity of the normal mucosal barrier that protects the stomach and duodenum from the hydrochloric acid secreted by parietal cells in the antrum of the stomach. Overall, the most common cause of PUD is H. pylori, the same bacterium responsible for type B chronic gastritis.

PRESENTATION PUD can present as either a gastric ulcer, or a duodenal ulcer. The pathophysiology, presentation, diagnosis, treatment, and complications of each are presented below. www.ClinicalReview.com


GASTROINTESTINAL SYSTEM

373

GASTRIC ULCER PATHOPHYSIOLOGY Gastric ulcer formation is most commonly associated with H. pylori infection and overuse of NSAIDs or steroidal medications. However, H. pylori infection remains a higher predictor of duodenal ulcer formation. Gastric ulcers occur due to a diminished protective barrier against stomach acid, including a thinner mucosal gel and decreased secretion of bicarbonate into the mucosal gel.

PRESENTATION Gastric ulcers present with a sharp, burning pain in the epigastrium shortly following the consumption of food. The lag time is typically between 10-30 minutes, during this time the stomach ramps up acid production and additional insult occurs to the protective barrier of the stomach and the underlying epithelium. Gastric ulcers often lead to nausea and vomiting, and the pain may lead to weight loss and anorexia. Due to possible susceptibility from protein similarities, individuals with type A blood are more susceptible to gastric ulcers. Nearly one-quarter of all individuals with gastric ulcers will experience significant hemorrhaging. As a result, gastric ulcers carry a significantly higher risk of complications and death than do duodenal ulcers. TABLE 440 PEPTIC ULCER DISEASE Peptic Ulcer disease (PUD) Epidemiology

Twice as common in men and related to Type A personality. PUD increases with age. Predictors--alcohol use and smoking.

Pathophysiology

Normal mucosal barrier is broken down.

DIAGNOSIS Diagnosis of gastric ulcer is by endoscopy and biopsy. Biopsy is mandatory for all endoscopal explorations of gastric ulcer due to the increased association of gastric ulcer with stomach cancer. The presence of H. pylori can also be detected using noninvasive tests such as an urea breath test and serology, while invasive tests such as a rapid urease test, biopsy, and culture can be utilized as confirmatory tests.

TREATMENT Treatment options for gastric ulcer primarily involve the eradication of H. pylori through a triple therapy, including bismuth salts, metronidazole, and amoxicillin. For individuals allergic to penicillins, amoxicillin can be substituted with clarithromycin. These agents are given for a period of several weeks to ensure eradication, and a repeat biopsy afterwards is often done. A proton pump inhibitor such as omeprazole and lanzoprazole is also standard of care, along with antacids and H2 blockers. Examples of antacids include aluminum hydroxide and magnesium hydroxide; examples of H2 blockers include cimetidine, famotidine, nizatidine, and ranitidine. Prostaglandin E in the form of misoprostol is given to buttress the mucosal barrier, and artificial substrates are occasionally given to coat the stomach, such as sucralfate. Symptoms may also be reduced through lifestyle changes such as discontinuing alcohol use and cessation of smoking, and reducing the use of offending medications such as NSAIDs and steroids.

Â


374

USMLE STEP 2

FIRST LINE TREATMENT The first line treatment for PUD is to eradicate H. pylori, followed by the administration of H2 receptor blockers and / or proton pump inhibitors. Prevention is best done by instituting misoprostol therapy for PUD prophylaxis.

Treat H. pylori with bismuth salts, metronidazole, and amoxicillin. Eradication is sufficient for MALToma Rx.

COMPLICATIONS The complications of gastric ulcer disease are numerous, with the most serious including a risk of hemorrhage leading to death. The presence of H. pylori has also been tied to the development of lymphoproliferative disease in the form of mucosa-associated lymphoid tissue (MALT) lymphoma. Chronic superficial gastritis can also occur, along with chronic atrophic gastritis. The latter condition can lead to the development of gastric adenocarcinoma. Complications may also arise as side effects of treatment. Cimetidine in particular is the only H2 blocker that can cause testicular degeneration and gynecomastia. Cimetidine is not recommended for use in patients on warfarin, phenytoin, or theophylline due to its inhibition of liver enzymes such as cytochrome P450. TABLE 441 GASTRIC PEPTIC ULCER Gastric Peptic Ulcer Pathophysiology

H. pylori, overuse of NSAIDs and steroids.

Presentation

Sharp, burning pain in epigastrium shortly after eating. Nausea, vomiting, anorexia.

Diagnosis

Endoscopy and biopsy, presence of H. pylori.

Treatment

Triple therapy. Proton pump inhibitors, antacids, H2 blockers.

CURLING AND CUSHING ULCERS Curling ulcers and Cushing ulcers are two special types of gastric stress ulcers that develop in certain types of patients. Following severe burn injury, a Curling ulcer is common. Following severe brain damage, Cushing ulcers may develop. TABLE 442 CURLING AND CUSHING ULCERS Curling and Cushing Ulcers Curling Ulcers

Peptic ulcers formed after sever burn injury.

Cushing Ulcers

Peptic ulcers formed after severe brain damage.

DUODENAL ULCER PATHOPHYSIOLOGY Duodenal ulcers are distinct from gastric ulcers in that they are commonly due to increased gastric acid production. Combined with the insult to the protective mucosal barrier from H. pylori infection found in nearly all cases of duodenal ulcers, the two factors combine to cause epithelium destruction and subsequent ulcer formation. H. pylori is able to exert its negative effects on the protective lining by degrading the mucous barrier through the production of urease. Other complicating factors include degradation of the mucosal lining through overuse of NSAIDs and steroids, and elevated gastric acid production through high

www.ClinicalReview.com


GASTROINTESTINAL SYSTEM

375 levels of gastrin found in Zollinger-Ellison (ZE) syndrome. ZE syndrome is discussed in the section below. 20% of patients with H. pylori develop symptomatic PUD, and 90% of patients with ZE syndrome develop PUD.

PRESENTATION Duodenal ulcers typically become symptomatic several hours after ingesting a meal. Late dinners often lead to symptoms around midnight, forcing the patient to wake up with severe epigastric pain with a strong burning sensation. Duodenal ulcers are relieved by ingesting additional food, which commonly causes the acidic stomach contents to be retained within the stomach and thereby give the bicarbonate secretion within the proximal duodenum a chance to neutralize the high level of acid. Duodenal ulcers are more common in patients with type O blood.

DIAGNOSIS Diagnosis of duodenal ulcer is made through endoscopy. Due to the significantly diminished risk of malignancy, biopsy is usually not performed, and the entire diagnosis may even be made following history and physical exam. H. pylori infection in duodenal ulcers is diagnosed in the same manner as for gastric ulcer, including through serology, a urease breath test that detects carbon isotopes after ingesting radiolabeled urea.

TREATMENT Duodenal ulcers require treatment with the triple therapy, including bismuth salts, metronidazole, and amoxicillin, with clarithromycin substituted for amoxicillin in patients with allergies to penicillins. The treatment is otherwise similar to that for gastric ulcers, including cessation of smoking and alcohol, avoiding NSAIDs and steroid medications, using antacids, H2 blockers, proton pump inhibitors, enhancers of the mucosal protective barrier such as prostaglandin E and sucralfate, and surgery when indicated. Surgery becomes the best option for the treatment of duodenal ulcers when the ulcer is refractory to medical management after three months, or there is increasing hemorrhage, obstruction, or even perforation. Perforation of a duodenal ulcer requires prompt surgical attention. The presence of duodenal ulcers does not imply the same risk of a concomitant cancer as it does for gastric ulcers. TABLE 443 DUODENAL ULCER Duodenal Ulcer Pathophysiology

Due to increased acid production & H. pylori

Presentation

Severe epigastric pain several hours after eating.

Diagnosis

Endoscopy, history, physical exam. Test for H. pylori

Treatment

Triple therapy, stop smoking & alcohol consumption, no NSAIDs or steroids, surgery.

ZOLLINGER-ELLISON SYNDROME PATHOPHYSIOLOGY Zollinger-Ellison (ZE) syndrome is due to the uncontrolled production of gastrin, leading to excessive production of hydrochloric acid by the parietal cells of the stomach antrum and thereby exacerbating PUD. Gastrinomas are typically found in the head of the pancreas, but may be located almost anywhere in the gastrointestinal (GI) tract, including the stomach, duodenum, and even spleen.

Â


376

USMLE STEP 2

PRESENTATION Gastrinoma often presents with PUD, diarrhea with steatorrhea, and other symptoms of mild malabsorption. The PUD is often refractory to medical and surgical interventions as the continuing elevations in hydrochloric acid from the gastrin secretion lead to repeated ulcer formation. Gastrinomas are one of the tumors that sometimes appear in the multiple endocrine neoplasia (MEN) type I disorder, which includes pancreatic tumors (the gastrinoma), pituitary tumors, and parathyroid tumors; approximately one-quarter of all affected individuals will have the full-fledged MEN type I disorder. In those individuals with MEN type I, hypercalcemia may also be part of the presentation. The majority of gastrinomas are malignant tumors.

DIAGNOSIS Diagnosis of ZE syndrome is made through blood tests that find fasting elevations of gastrin of nearly 1 mg/L. The diagnosis is confirmed through the secretin stimulation test. In this test, secretin, the natural inhibitor to gastrin, is administered IV to determine the change in gastrin levels. There is a paradoxical rise in gastrin levels to very high levels following the administration of secretin. Gastrinoma: Positive secretin stimulation test. Presents with TREATMENT multiple ulcers. Resect if not part of MEN I (i.e. sporadic). Else Treatment of ZE syndrome is best made through surgical resection of the affected PTHectomy and monitor. region. Medical management is then typically used to control the symptoms, such as the use of proton pump inhibitors. TABLE 444 ZOLLINGER-ELLISON SYNDROME Zollinger-Ellison Syndrome Pathophysiology

Uncontrolled production of gastrin. Gastrinomas often found in head of pancreas.

Presentation

Gastrinomas often with PUD, diarrhea, steatorrhea, malabsorption symptoms.

Diagnosis

Fasting blood tests revealing elevated gastrin titers, secretin stimulation test.

Treatment

Surgical resection of affected area. Proton pump inhibitors.

OTHER GASTRIC DISORDERS GASTROPARESIS PATHOPHYSIOLOGY AND PRESENTATION Gastroparesis is the delay in the normal emptying rate of the stomach, leading to food and liquids being present in the stomach for prolonged periods of time and diminished gastric digestive function. Gastroparesis is most commonly due to degeneration of the innervation to the stomach, chiefly the vagus nerve. The most likely cause of this degeneration is due to diabetic neuropathy in patients with longstanding diabetes. Gastroparesis presents with early satiety, nausea, and vomiting due to defects in gastric emptying.

www.ClinicalReview.com


GASTROINTESTINAL SYSTEM

377

DIAGNOSIS AND TREATMENT Diagnosis of gastroparesis is made through radiolabeled studies that gauge the digestive function of the gastrointestinal tract. Metoclopramide to minimize the symptoms of nausea and vomiting is often the treatment of choice. TABLE 445 GASTROPARESIS Gastroparesis Pathophysiology

Delay in normal emptying rate of stomach due to degeneration of stomach nerves.

Presentation

Early satiety, nausea, vomiting.

Diagnosis

Radiolabeled studies that gauge digestive function of the GI tract.

Treatment

Metoclopramide.

DUMPING SYNDROME PATHOPHYSIOLOGY Dumping syndrome occurs following gastrointestinal surgeries such as bariatric surgery and PUD repair that destroy the normal digestive function. In dumping syndrome, digested food and liquids accumulate in the small intestine, causing circumferential expansion, additional accumulation of fluids emptying from the stomach into the duodenum, and sudden expulsion of the food through the gastrointestinal system.

PRESENTATION AND DIAGNOSIS Dumping syndrome presents most commonly after ingesting a fatty or carbohydrate-laden meal. The sudden shift in electrolytes and fluids, and increased sudden blood flow to the small intestine leads to dizziness, lightheadedness, diaphoresis, nausea, and vomiting. Fatty meals tend to present with symptoms within about half-an-hour, while meals rich in carbohydrates present after an hour. Diagnosis is made on clinical presentation.

TREATMENT Treatment of dumping syndrome requires decreasing fluid intake to small, but frequent amounts that typically should not immediately follow a meal. Fatty foods and simple sugars should be avoided, and meals should be smaller in size. TABLE 446 DUMPING SYNDROME Dumping Syndrome Pathophysiology

Occurs after GI surgery & PUD repair.

Presentation

Usually occurs after eating fatty meals. Dizziness, nausea, vomiting, sudden expulsion of food through GI system.

Treatment

Decrease fluid intake to small, frequent amounts. Small meals. Avoid fatty foods, simple sugars.

Â


378

USMLE STEP 2

SMALL INTESTINE INTRODUCTION The small intestine is anatomically separated into the duodenum (25 cm), jejunum (2.5 m), and ileum (3.5 m). Transverse semilunar folds of submucosa extend throughout the small intestine, known as the plicae circularis. Finger-like projections extend into the mucosa of the intestinal lumen, known as villi, and smaller striated projections extend from the villi, known as microvilli. The function of the villi and microvilli is to dramatically increase the surface area available for absorption of nutrients. The small intestine functions primarily to digest food and liquids. Enzymes in the glycocalyx of the microvilli function to breakdown protein and sugar, and aid in the absorption of monosaccharides, amino acids, electrolytes, vitamins and minerals, and fluids.

TYPES OF CELLS The small intestine contains enterocytes, which function to transport substances from the lumen into the circulatory system. Goblet cells secrete mucin to help aid the transport of ingested contents. Paneth cells control bacterial populations through the secretion of lysozyme and zinc. Enteroendocrine cells release cholecystokinin, secretin, and gastric inhibitory peptide.

MALABSORPTION DISORDERS MALABSORPTION SYNDROMES ETIOLOGY Malabsorption syndromes are defined as the alterations in intestinal absorption of vital fluids, vitamins, and nutrients. The small bowel is the site of many of these malabsorption problems, which may include selective deficiencies in monosaccharides, lipids, lipid-soluble vitamins, amino acids, nucleic acids, fluids, water-soluble vitamins, bile salts, and electrolytes. Malabsorption syndromes commonly occur due to defects in digestion, poor distribution of emulsifying bile acids, changes to the anatomy of the bowel, obstruction to the lymphatic drainage of the bowel, decreased function or surface area available for absorption, alterations in hormones, and infection.

PATHOPHYSIOLOGY – MONOSACCHARIDE Defects in absorption related to monosaccharides involve the entire small bowel. In this scenario, only monosaccharides are absorbed due to defects in the degradation of complex carbohydrates, poor transport with the sodium cotransporter due to low amounts of sodium in meals, or excess fructose in the diet that overwhelms the fructose transporters and causes an osmotic diarrhea.

PATHOPHYSIOLOGY – LACTOSE INTOLERANCE Lactose is a major sugar digested by many people who consume dairy products, but some patients have an inherent deficiency in lactase that usually worsens with age. It is especially common in adult patients from Africa, Asians, and a minority of Caucasians. This congenital defect in lactase leads to symptoms including abdominal cramps, flatus, and diarrhea.

www.ClinicalReview.com


GASTROINTESTINAL SYSTEM

379

PATHOPHYSIOLOGY – LIPIDS The absorption of lipids and lipid-soluble vitamins such as vitamins D, E, K, and A typically affects the duodenum more often than in the other parts of the small intestine. For proper absorption to occur, pancreatic lipase must be present in appropriate amounts to hydrolyze triglycerides, bile salts must be present to emulsify fats and form micelles, and digestive products must be reabsorbed in the ileum and not sooner. In addition, sufficient micelles must be present to permit proper absorption of the fatsoluble vitamins.

PATHOPHYSIOLOGY – AMINO ACIDS AND NUCLEIC ACIDS The absorption of amino acids occurs more often in the jejunum than in other parts of the small intestine. Protein digestion starts in the stomach and must continue without problems in the duodenum, with the proper functioning of pancreatic enzymes such as trypsin, chymotrypsin, and elastase. Cotransport of most amino acids requires sodium, but there are also various sodiumindependent transporters available for certain amino acids. Nucleic acid transport involves the entire small intestine and requires pancreatic nucleases to cleave nucleic acids into pentoses for passive transport through diffusion.

PATHOPHYSIOLOGY – FLUIDS Fluids are absorbed mostly in the jejunum, followed by the ileum and then colon. The small intestine secretes an additional 7 L to the 2 L ingested daily, with only 200 mL remaining for defecation after absorption by the intestines. Water moves parallel to the osmotic gradients, and thus any defect in fluid absorption requires a search for shifts in these gradients.

PATHOPHYSIOLOGY – VITAMINS AND BILE SALTS The transport of water-soluble vitamins primarily occurs in the duodenum through passive diffusion. Vitamin B12 is an exception to this rule because it requires intrinsic factor to be produced by the parietal cells in the stomach for absorption in the terminal ileum. Bile salts are reabsorbed through an active transport process in the ileum for recycling.

PATHOPHYSIOLOGY – ELECTROLYTES Sodium, potassium, chloride, calcium, and iron ions are absorbed and secreted throughout much of the small intestine. The majority of chloride, calcium, and iron transport occur in the duodenum. Sodium ions are absorbed through an active and a passive mechanism, while secretion of sodium ions into the intestinal lumen occurs through the Na+-K+-ATPase transport system. Potassium ions are absorbed in the small intestine, and secreted in the colon. Chloride ions are secreted in small amounts, but overall, they are absorbed, especially in the duodenum. Calcium ions are actively transported with the assistance of 1,2,5dihydroxycholecalciferol and further assisted by protein absorption. A diet high in phosphates or oxalates inhibits calcium ion transport. Finally, iron ion transport is generally poor and is best done when iron is in the Fe2+ form, as this ferrous form freely diffuses through the mucosa and binds to the iron storage molecule, apoferritin. The iron then moves from apoferritin to the transport protein known as transferrin. Proper iron ion absorption thereby requires the reduction of the dietary form of iron (Fe3+, or ferric iron) to ferrous iron (Fe2+) by stomach acid, binding to adequate amounts of apoferritin, and subsequent binding to proper amounts of transferrin.

PRESENTATION The signs and symptoms of malabsorption syndromes are numerous and depend on the particular etiology of the disorder. Generally, malabsorption syndromes present with steatorrhea (which is in especially large amounts with pancreatic defects),


USMLE STEP 2 380 deficiencies in one or more vitamins and/or minerals, weight loss, bloating and abdominal pain, osteoporosis with poor calcium absorption, diarrhea, flatus, amenorrhea, anemia and platelet defects leading to easy bruising, and neuropathy.

DIAGNOSIS AND TREATMENT Diagnosis of malabsorption syndromes is made using a Sudan stain to confirm the presence of steatorrhea, confirming the nature of the malabsorption through a challenge test, and finally doing a biopsy of the small bowel to determine the presence of certain malabsorptive diseases. Treatment for malabsorptive disorders is accomplished by treating diseases that are causing the defect or correcting for deficits in the body through replacement of particular nutrients or replacing pancreatic enzymes that are not functioning properly. All malabsorptive syndromes require vitamin replacement due to normal losses experienced with the diarrhea. Lactose intolerance is treated by avoiding dairy products and administering lactase supplements. TABLE 447 MALABSORPTION SYNDROMES Malabsorption Syndromes Etiology

Alteration in intestinal absorption of vital fluids, vitamins, and nutrients.

Monosaccharides

Involve entire SI. Defect in degradation of complex carbohydrates, excess fructose.

Lactose intolerance

Inherited deficiency in lactase.

Lipids & Vitamins A, D, E, K

Occurs more often in the duodenum. May be lipase, or bile salt deficient.

Amino & nucleic acids

Occurs more often in jejunum. May be deficient in trypsin, chymotrypsin, and elastase.

Water soluble vitamins & bile salts

Vitamin transport occurs in the duodenum. Bile salts are reabsorbed in the ileum.

Electrolytes

Na, K, Ca, and Fe ions are absorbed and secreted through much of the SI.

Presentation

Generally present with steatorrhea, deficiencies in one or more vitamins/minerals, weight loss, bloating, abdominal pain, neuropathy, easy bruising.

Diagnosis & treatment

Sudan stain, malabsorption challenge tests, biopsy. Replace missing nutrients & vitamins.

CELIAC SPRUE ETIOLOGY AND EPIDEMIOLOGY Celiac sprue is a malabsorptive syndrome that affects the small intestine due to an inability of the body to process a gluten-rich diet. These large proteins are commonly found in grains such as wheat and barley, and consumption of sufficient quantities leads to an immune reaction causing an enteropathy that impairs absorption of other needed nutrients and minerals. Celiac sprue disease is more common in Caucasian women of European descent, and is associated with HLA DR3 and HLA DQw2 antigen groups.

PATHOPHYSIOLOGY Celiac sprue disease, also known as gluten-induced enteropathy, occurs due to the infiltration of lymphocytes as part of an immune-mediated sensitivity to gluten proteins. Antibodies to gliadin and endomysium form, and the end result is anatomic malformations of the crypts of Lieberkuhn and villous atrophy. These changes dramatically decrease the surface area available for food absorption, leading to the signs and symptoms of malabsorption discussed below.

www.ClinicalReview.com


GASTROINTESTINAL SYSTEM

381

PRESENTATION Celiac sprue presents like other malabsorptive syndromes in that there is diarrhea and steatorrhea, bloating and abdominal pain, and weight loss. Unlike other malabsorptive syndromes, there may also be seizures and ataxia. Further, celiac sprue may present with a pruritic rash known as dermatitis herpetiformis.

DIAGNOSIS Diagnosis of celiac sprue is made by analysis of the feces for high fat content, the presence of IgG and IgA antibodies against gliadin, antibodies against endomysium and reticulin, and the presence of a normal biopsy after cessation of foods that contain gluten. Due to the malabsorptive syndrome, decreases in water soluble vitamins, iron, calcium, and protein may occur. The protein losing enteropathy may also lead to a decrease in prothrombin time due to the inability of the liver to make adequate amounts of clotting factors. Tthe diagnosis of celiac sprue disease can be made with barium swallow that indicates the presence of clumps of barium and decrease in the number of plicae circulares. Failure to treat celiac sprue may result in lymphoma.

TREATMENT Treatment of celiac sprue disease is to remove all grains from the diet other than corn and rice. Severe cases may be treated with corticosteroids to control the immune reaction. The dermatitis herpetiformis that occasionally accompanies celiac sprue disease may be treated with topical sulfone until it too resolves with a gluten-free diet. TABLE 448 CELIAC SPRUE Celiac Sprue Pathophysiology

Gluten- induced, lymphocytic invasion infiltration, antibody production.

Presentation

Diarrhea and steatorrhea, bloating and abdominal pain, weight loss, rash, seizures, ataxia

Diagnosis

Fecal analysis for fat, IgG and IgA to gliadin, antibodies against reticulin and endomysium, barium swallow.

Treatment

Remove all grains except rice and corn from diet, corticosteroids, gluten-free diet.

PROTEIN-LOSING ENTEROPATHY Protein-losing enteropathies are similar to celiac sprue disease in that they present with diarrhea, steatorrhea, and edema. Protein-losing enteropathies may be caused by a number of different etiologies and do not necessarily directly involve the gastrointestinal system. A protein-losing enteropathy occurs when the normal protein turnover in the gastrointestinal system is no longer adequately replaced, especially albumin. Diagnosis of a protein-losing enteropathy is made by comparing the level of alpha-1-antitrypsin in the serum to the stool, as alpha-1-antitrypsin is not broken down in the intestines. Treatment for proteinlosing enteropathies is to correct the underlying cause and to also treat as per other malabsorptive syndromes. TABLE 449 PROTEIN LOSING ENTEROPATHY Protein Losing Enteropathy Presentation

Diarrhea, steatorrhea, edema.

Pathophysiology

GI system is no longer able to adequately replace protein (especially albumin).

Diagnosis

Comparing alpha-1-trypsin levels in serum and stool.

Treatment

Correct underlying cause and treat other malabsorptive syndromes.

Â


382

USMLE STEP 2

TROPICAL SPRUE PATHOPHYSIOLOGY Tropical sprue is a malabsorptive disorder that primarily affects the jejunum. The etiology is unknown, but tropical sprue predominantly affects those who have visited tropical regions. Tropical sprue develops some time after exposure to a postulated infectious agent and leads to poor absorption of proteins and folic acid. Some sort of immune reaction is likely based on the presence of a monocyte infiltrate into the jejunum and mild villous atrophy.

DIAGNOSIS AND TREATMENT Tropical sprue presents like other malabsorptive syndromes with the addition of cheilosis, glossitis, and stomatitis – all symptoms of folic acid deficiency. Diagnosis is made by the presence of a megaloblastic anemia from the folic acid deficiency, malabsorption of at least two nutrients including calcium, cholesterol, folic acid, iron, magnesium, vitamin B12, and various proteins. A travel history to tropical regions is also important. Treatment of tropical sprue is to give tetracycline or TMP-SMX for several months. Supplements for the vitamin deficiencies should also be given. TABLE 450 TROPICAL SPRUE Tropical Sprue Pathophysiology

Poor absorption of proteins and folic acid.

Presentation

Primarily affects jejunum.

Diagnosis

Cheilosis, glossitis, stomatitis, folic acid deficiency, travel to tropical regions.

Treatment

TMP-SMX, supplemental vitamins.

WHIPPLE DISEASE ETIOLOGY AND EPIDEMIOLOGY Whipple disease is a rare, deadly, but curable malabsorptive disorder that destroys the lamina propria of the intestine, then spreads to cause systemic symptoms. Whipple disease is caused by the gram-negative rod Trophermyma whippleii. Whipple disease is significantly more common in older Caucasian males.

PRESENTATION Whipple disease presents with signs and symptoms of abdominal pain similar to that of other malabsorptive syndromes. Diarrhea, steatorrhea, weight loss, and decreased amino acid absorption ensue. With continuing infection, symptoms of arthralgia, low-grade fever, uveitis, heart failure or endocarditis, CNS defects including palsy, memory loss, or confusion, and increased skin pigmentation occur. The latter symptoms occur within a relatively short amount of time – within a month is typical of presentation.

DIAGNOSIS AND TREATMENT Diagnosis is made by doing a PCR of peripheral blood to demonstrate infection by Trophermyma whippleii. Replacement of damaged intestinal epithelium is also made to show the presence of foamy macrophages that are PAS-positive. Whipple disease is cured with treatment by TMP-SMX or tetracycline for at least six months. www.ClinicalReview.com


GASTROINTESTINAL SYSTEM

383

TABLE 451 WHIPPLE DISEASE Whipple Disease Pathophysiology

Caused by Trophermyma whippleii.

Presentation

Abdominal pain, steatorrhea, weight loss, arthralgia, low fever, endocarditis.

Diagnosis

PCR of peripheral blood to demonstrate T. whippleii

Treatment

TMP-SMX or tetracycline for at least 6 months.

MÉNÉTRIER DISEASE PATHOPHYSIOLOGY Ménétrier disease is a specific type of protein-losing enteropathy that primarily affects the gastric mucosa. A low-grade inflammation occurs with replacement of the normal chief cells and parietal cells by glandular cells.

PRESENTATION AND DIAGNOSIS The thickening of the mucosal layer in Ménétrier disease leads to poor absorption of proteins. This disorder presents with epigastric pain, nausea and vomiting, weight loss, and decreased secretion of gastric acid. Diagnosis is made by endoscopy and biopsy that identifies the tissue changes, and a barium swallow that demonstrates enlarged rugae. A barium swallow may also appear similar to that of gastric cancer.

TREATMENT Treatment of Ménétrier disease is to enrich the protein nutrition of the patient, reduce protein loss with anticholinergics and H2 blockers, and treat any gastric complications of the disease, including ulcers and cancer. TABLE 452 MÉNÉTRIER DISEASE Ménétrier disease Pathophysiology

Protein-losing enteropathy of gastric mucosa. Metaplasia of chief and parietal cells.

Presentation

Epigastric pain, nausea/vomiting, weight loss, decreased gastric acid.

Diagnosis

Endoscopy and biopsy (identifies metaplasia), barium swallow (enlarged rugae).

Treatment

Enhance protein nutrition, anticholinergics, H2 blockers, treat complications.

APPENDICITIS ETIOLOGY Appendicitis is attributed to the formation of an appendicolith in adults, leading to bacterial overgrowth with subsequent development of pain, fever, chills, and overlying abdominal tenderness in the right lower quadrant. In children, the most common cause of appendicitis is lymphoid hyperplasia. Tumors such as carcinoid and adenocarcinoma can also lead to appendicitis.


384

USMLE STEP 2

PATHOPHYSIOLOGY The most common mechanisms are fecalith for adults and lymphoid hyperplasia for children. Carcinoid is the most common tumor causing appendicitis (90%), followed by adenocarcinoma.

DIAGNOSIS A clinical diagnosis is sufficient to take a patient to the OR for an appendectomy. However, many times, a CT scan of the abdomen is typically completed. Findings on CT scan include periappendiceal inflammation including stranding, incomplete contrast filling of the appendix, a diameter greater than 6mm, wall thickening, and enhancement with IV contrast.

TREATMENT Treatment of appendicitis is by appendectomy. The presence of perforation and a Carcinoid < 2cm at tip of walled-off abscess can be treated with interval drainage of the abscess followed appendix = appendectomy. by scheduled appendectomy. Carcinoid less than 2cm in size at the tip of the Carcinoid > 2cm or at base of appendix can be treated by appendectomy alone. A carcinoid that is larger than appendix = right hemicolectomy. 2cm or at the base of the appendix must be treated by a right hemicolectomy. The presence of an ovarian cyst that is likely leading to the signs and symptoms of what was originally thought to be appendicitis should be treated by removing the appendix and pursuing treatment for the cyst afterwards. The presence of terminal ileitis but a normal cecum and appendix should also prompt the surgeon to remove the appendix. The presence of a cystic mass at the base of the appendix is likely a cystadenocarcinoma and must be treated by a right hemicolectomy. Due diligence is required to avoid spilling the contents as seeding of the tumor will occur. The presence of a walled-off abscess can be treated with a CT-guided drain placement followed by interval appendectomy.

LARGE INTESTINE INTRODUCTION The large intestine consists of several anatomical segments, including the cecum, ascending colon, transverse colon, descending colon, sigmoid colon, rectum, and anal canal. The inner surface of the colon is known as the mucosa; it is surrounded by three equally spaced muscular bands known as the teniae coli in the outer longitudinal layer. Straight tubular glands extending through the entire mucosa of the colon are known as the crypts of Lieberkuhn. The mucosa, with the crypts of Lieberkuhn, is responsible for the absorption of fluids and electrolytes while secreting large amounts of mucus to help smoothly propel the contents of the large intestine.

www.ClinicalReview.com


GASTROINTESTINAL SYSTEM

385

DIARRHEA AND CONSTIPATION DIARRHEA ETIOLOGY Diarrhea may be due to a number of disparate causes, including infectious, medication-induced, inflammatory, osmotic, secretory, and altered intestinal motility etiologies. The most common causes of infectious diarrhea include E. coli (ETEC), Vibrio cholerae, E. coli (O157:H7), Giardia lamblia, rotavirus, Norwalk agent, Salmonella, and Clostridium difficile. The most common causes of bloody diarrhea include Campylobacter, E. coli (EHEC), E. histolytica, Salmonella, and Shigella. Inflammatory diarrhea is commonly secondary to inflammatory bowel disease and infections from AIDS. Osmotic diarrhea is most commonly due to lactase deficiency and celiac sprue disease. Secretory diarrhea may occur from carcinoid syndrome and gastrinoma due to ZE syndrome. Altered intestinal motility leading to diarrhea may be secondary to irritable bowel syndrome.

PATHOPHYSIOLOGY Diarrhea is defined as increase in stool weight over 200 grams per day or abnormally frequent passage of stools. Eight of the 9 liters of fluids that enter the gastrointestinal tract are absorbed in the small intestine, and only about 1 liter passes into the large intestine. Of that liter, about 4/5s of it are reabsorbed and the remainder goes towards the formation of stool. Any alteration in that absorption either through poor absorption, increased secretion, or increased production may lead to diarrhea. Most cases of diarrhea lasting less than two weeks are infectious in etiology, while chronic diarrhea lasting more than a few weeks is typically due to lactase deficiency. TABLE 453 DIARRHEA Diarrhea Epidemiology

Medication induced, pathogens, inflammatory, osmotic and secretory causes. Chronic diarrhea lasts more than a few weeks, infectious diarrhea less than two weeks.

Pathophysiology

Stool weight over 200 g/day or abnormally frequent defecation.

Diagnosis

Fecal analysis for ova and parasites.

Treatment

When infectious agents ascertained, Bactrim. Hydration therapy.

INFECTIOUS DIARRHEA Infectious diarrhea from E. coli may be enterotoxigenic (ETEC) such as traveler’s diarrhea, or enteroinvasive (EIEC) or enterohemorrhagic (EHEC) that presents with fever and bloody diarrhea. The most common bacterial diarrhea in North America is Campylobacter, which may also lead to a seronegative arthropathy known as reactive arthritis. Diarrhea in day care centers that is commonly transmitted through impure food or water is commonly due to Shigella. Salmonella is a cause of diarrhea in patients who consumed improperly prepared poultry or dairy products. Diarrhea that presents similar to appendicitis but with joint pain and a skin rash should lead to a clinical suspicion for Yersinea. Poorly cooked beef and unpasteurized dairy products may lead to poisoning with E. coli O157:H7 and concomitant thrombotic thrombocytopenic purpura (TTP) or hemolytic uremic syndrome (HUS). Diarrhea from viral agents are very common, especially Norwalk agent on cruise ships and rotavirus through oral-fecal infection. Shigella infection leads to diarrhea after several hours or days. Staphylococcus aureus infection leads to rapid diarrhea within a few hours. Most infectious diarrhea occurs within one day. Common microorganisms found in diarrhea in immunocompromised individuals include Mycobacterium avium, Cryptosporidium, and Isosporium.

Â


386

USMLE STEP 2

TABLE 454 INFECTIOUS DIARRHEA Infectious Diarrhea Types

ETEC, EIEC, EHEC (presents with fever and bloody diarrhea).

Causes

Campylobacter, Shigella (in impure water or food), Salmonella (improperly prepared poultry or dairy products), E. coli (improperly prepared beef and dairy ). Viral agents (Norwalk, Rota virus, etc.).

MEDICATION-INDUCED DIARRHEA Diarrhea may be induced through the consumption of a number of medications. These include laxatives, antibiotics such as penicillins, cephalosporins, clindamycin, and tetracycline, antacids that contain magnesium, colchicine, beta blockers such as propranolol, quinidine, theophylline, diuretics such as furosemide or thiazide diuretics, angiotensin-converting enzyme (ACE) inhibitors, and antidepressants such as fluoxetine and sertraline. Diarrhea following digoxin and lithium is typically an early sign of toxicity, and most diarrhea following consumption of medications is typically a sign that the offending drug should be stopped. TABLE 455 MEDICATION-INDUCED DIARRHEA Medication-Induced Diarrhea Causes

Laxatives, antibiotics, antacids with magnesium, beta blockers, diuretics.

Treatment

Cease offending medication(s).

INFLAMMATORY DIARRHEA Inflammatory diarrhea is associated with intestinal changes in inflammatory bowel disease such as Crohn disease and ulcerative colitis, and in infections associated with AIDS. Ulcerative colitis tends to present with a bloody diarrhea, while Crohn disease tends to present with a diarrhea that contains more mucus and pus, and smaller amounts of blood. TABLE 456 INFLAMMATORY DIARRHEA Inflammatory Diarrhea Causes

Crohn disease (mucus and pus filled diarrhea), ulcerative colitis (bloody diarrhea), infections with AIDS.

Pathophysiology

Intestinal changes in inflammatory bowel disease.

Treatment

Treat underlying cause.

OSMOTIC DIARRHEA Osmotic diarrhea is due to the presence of indigestible solutes such as lactose in patients with lactase-deficiency. High carbohydrate content such as fructose, sorbitol, and mannitol may also cause osmotic diarrhea. Osmotic diarrhea leads to the transport of fluids and electrolytes from the circulation to the lumen – the opposite of normal transport. The increase in fluid volume in the small bowel quickly overwhelms the absorptive capacity of the large bowel, and a large volume of watery diarrhea results. The diarrhea resolves after passage of the offending solutes, and cessation of oral intake of the offending nutrient typically leads to rapid relief of symptoms.

www.ClinicalReview.com


GASTROINTESTINAL SYSTEM

387

TABLE 457 OSMOTIC DIARRHEA Osmotic Diarrhea Cause

Indigestible solutes and high carbohydrate content (mannitol, sorbitol, fructose).

Presentation

Watery diarrhea which abates after expulsion of offending solute.

Treatment

Eliminate offending nutrient(s).

SECRETORY DIARRHEA Secretory diarrhea is often a result of gastrinoma, as in ZE syndrome and in carcinoid syndrome. High levels of vasoactive intestinal peptide (VIP) from pancreatic adenomas and irritation from certain laxatives (such as phenolphthalein) may also result in secretory diarrhea. Secretory diarrhea tends to be very high in volume, and resolves with treatment of the underlying disorder or cessation of the offending medication.

PRESENTATION AND DIAGNOSIS Diarrhea from virtually any cause often presents with signs of dehydration, if symptoms do not remit after a few days. Fever often accompanies most cases of infectious diarrhea. Diagnosis of diarrhea involves testing the stool for ova and parasites, identifying the presence of leukocytes, and having the presence of hypokalemic metabolic alkalosis. Treatment may be instituted only when the infectious agent is ascertained; trimethoprim-sulfamethoxazole (TMP-SMX) is used for most cases of infectious, invasive diarrhea. Metronidazole is the agent of choice for Giardia. Hydration through intravenous fluids is often mandated, depending on the level of dehydration. Antidiarrheals are rarely used due to the risk of toxic megacolon.

CONSTIPATION PRESENTATION AND DIAGNOSIS Constipation is defined as the passage of stools less than 3 times every 7 days unless this is a normal bowel habit. Constipation is often secondary to a diet low in fiber with poor oral fluid intake. Constipation may also occur due to complications associated with diabetes, secondary to medications, obstruction, pregnancy, Hirschsprung disease, Chagas disease, or hypothyroidism. Medications such as narcotics, anticholinergics, iron, and calcium-channel blockers can also cause constipation.

TREATMENT Treatment of constipation is by increasing fluid intake, either orally or intravenously. A diet high in fiber, up to 30 grams per day, often ameliorates most cases of constipation. Laxatives that lead to increased bulk and emollient laxatives may be prescribed as needed. TABLE 458 CONSTIPATION Constipation Causes

Low fiber and fluid diets. Complications of diabetes, medications (iron, narcotics, anticholinergics, calcium channel-blockers) hypothyroidism, pregnancy.

Treatment

High fiber and liquid diets, laxatives (emollients and/or bulk increasers).

Â


388

USMLE STEP 2

OGILVIE SYNDROME ASSESSMENT Ogilvie syndrome is a distention of the bowel leading to obstruction. It tends to occur following non-abdominal procedures, especially following cardiac surgery. It is thought to be due to neurologic dysfunction, abnormalities in electrolytes, and related to age.

Ogilvie Rx = neostigmine. Have atropine for bradycardia. Tends to perforate at cecum.

MANAGEMENT Ogilvie syndrome is treated with nasogastric tube decompression and intravenous neostigmine to stimulate the parasympathetic system. Atropine should be on hand in the event of significant bradycardia. Decompression with colonoscopy has a 90% success rate but a 40% recurrence rate. In the worst circumstance, an exploratory laparotomy with bowel resection or cecostomy is required. The cecum is at the greatest risk of perforation.

PSEUDOMEMBRANOUS COLITIS ETIOLOGY AND PATHOPHYSIOLOGY Pseudomembranous colitis is diarrhea caused by Clostridium difficile that overgrows within the colon and disrupts the normal bacterial flora, leading to impaired absorption. C. difficile also releases a toxin that may lead to damage to the mucosa and further contributes to the diarrhea. Pseudomembranous colitis commonly occurs between a week and a month after stopping antibiotics, and the most commonly implicated antibiotics are clindamycin, vancomycin, metronidazole, and cephalosporins. TABLE 459 PSEUDOMEMBRANOUS COLITIS Constipation Causes

Low fiber and fluid diets. Complications of diabetes, medications (iron, narcotics, anticholinergics, calcium channel-blockers, antibiotics), hypothyroidism, pregnancy.

PRESENTATION AND DIAGNOSIS Pseudomembranous colitis presents with watery diarrhea that contains scant, if any blood, abdominal cramping, fever, and leukocytosis. Diagnosis is made by searching for the C. difficile toxin in stool samples. The majority of patients with actual C. difficile infection will have a positive test after testing three separate stool samples. Sigmoidoscopy or colonoscopy clinches the diagnosis with the visualization of yellow membranous plaques on the mucosa, but this test is rarely used.

TREATMENT Treatment of C. difficile requires cessation of the antibiotics leading to the demise of the bowel flora, and the use of metronidazole or vancomycin to treat the C. difficile. Oral metronidazole is preferred unless the strain is resistant, then oral vancomycin becomes the preferred regimen. Treatment for up to two weeks is necessary. If treatment is not started in a timely manner, severe inflammation and infection can occur throughout all layers of the colonic epithelium, leading to severe diarrhea, sepsis, and toxic megacolon. Plain films of the abdomen demonstrate long loops of colon with signs of edema in the bowel wall (known as a positive “thumb printing� sign). The standard of care, if these complications arise, is colectomy. www.ClinicalReview.com


GASTROINTESTINAL SYSTEM

389

TABLE 460 PSEUDOMEMBRANOUS COLITIS Pseudomembranous Colitis Etiology

Clostridium difficile. May occur a week to a month after stopping antibiotics.

Presentation

Watery diarrhea that contains little or no blood, abdominal cramps, fever.

Diagnosis

Positive C difficile test, sigmoidoscopy or colonoscopy showing yellow plaques.

Treatment

Metronidazole or vancomycin; if severe, colectomy.

IRRITABLE AND INFLAMMATORY BOWEL DISEASE IRRITABLE BOWEL SYNDROME ETIOLOGY AND EPIDEMIOLOGY Irritable bowel syndrome is the most common outpatient complaint with regard to the gastrointestinal system. Irritable bowel syndrome is defined as a change in bowel habits due to a change in intestinal motility, which itself may be secondary to a number of causes. Irritable bowel syndrome is most common in young adults up to the age of 40, and affects females twice as much as males.

PRESENTATION AND DIAGNOSIS Irritable bowel syndrome presents as continuous transitions between diarrhea and constipation with frequent abdominal pain and a sense of bloating and distention relieved by defecation. Diagnosis is made by clinical history, and requires the presence of intermittent symptoms for more than three months with no night time symptoms. The lack of systemic symptoms and the presence of stress typically clinch the diagnosis. The symptoms must include two elements from the Rome criteria, which are altered passage of stool, abnormal frequency, change in consistency, mucus in stool, and abdominal distention. The differential diagnosis of irritable bowel syndrome includes Giardia infection, inflammatory bowel disease, lactase deficiency, hypothyroidism, and colon cancer. Irritable bowel syndrome is associated with fibromyalgia, interstitial cystitis, fatigue, and depression.

TREATMENT Irritable bowel syndrome is treated with a high-fiber, low-fat diet to promote proper passage of intestinal contents. All gasforming foods should be eliminated. One of the following medications may also be used in severe cases: psyllium, antispasmodics, anticholinergics, antidiarrheals, tricyclic antidepressants, anxiolytics, or osmotic laxatives. TABLE 461 IRRITABLE BOWEL SYNDROME Irritable Bowel syndrome Presentation

Fluctuation between diarrhea and constipation, frequent abdominal pain for at least 3 months.

Diagnosis

Two elements of Rome criteria. Differential diagnosis--Eliminate Giardia, inflammatory bowel disease, lactase deficiency, hypothyroidism, colon cancer.

Treatment

High fiber, low fat diet. Eliminate gas-forming foods. Psyllium, antispasmodics, anticholinergics, antidiarrheals, osmotic laxatives, anxiolytics.

Â


390

USMLE STEP 2

CROHN DISEASE ETIOLOGY AND EPIDEMIOLOGY Crohn disease is one of two major inflammatory bowel diseases. Inflammatory bowel disease is significantly more common in the Ashkenazi Jewish and Caucasian populations, and tends to affect young adults up to the age of 35 but may also affect individuals over 60 years of age. There is a strong pattern of family inheritance. Crohn disease has an incidence of approximately 5 in 100,000 individuals, and is more common in men. Crohn disease is three times more likely to cause colon cancer, with the risk increasing over time. Although the precise cause remains unknown, some researchers have implicated the presence of antibodies to alpha-Saccharyomyces cerevesiae as prognostic.

Crohn disease: Transmural, fistula, spares rectum but any location within GI tract, aphthous ulcers, cobblestoning, skip lesions, targets terminal ileum, nonbloody diarrhea, calcium oxalate stones. Rx: remicade, gastrojejunostomy

PATHOPHYSIOLOGY Crohn disease occurs from an unknown source and leads to inflammation of the intestines. It affects all of the layers of the bowel, leading to the formation of fistulas and abscesses. Crohn disease spares the rectum in about half of all patients, but may be found in any location of the gastrointestinal tract. Discontinuous, skipping lesions are present with normal bowel located between stretches of diseased bowel. Thickening of the submucosal layer in the diseased region also leads to a cobblestone appearance that alternates with regions of ulceration of the submucosal layer. The most commonly affected regions of the gastrointestinal tract are the terminal ileum (1/3 of all cases), the colon (1/3 of all cases), or both regions (1/3 of all cases).

PRESENTATION Crohn disease presents with nonbloody diarrhea, unless there is significantly involvement of the rectum. Crampy abdominal pain, weight loss, fever, malaise, and a tender right lower quadrant are typically the signs and symptoms. Crohn disease, like all inflammatory bowel diseases, may also have a number of extraintestinal manifestations such as arthritis, uveitis, iritis, erythema nodosum, and pyoderma gangrenosum. Nearly 1 in 5 patients may be afflicted with such extraintestinal complications. Occasionally, calcium oxalate stones may be present in the kidney. These extraintestinal complications are discussed in their own sections below.

DIAGNOSIS Crohn disease is diagnosed through colonoscopy (skip lesions, cobblestoning, abscess formation, and fistulas may be pathognomonic). Histology often demonstrates granulomas. Laboratory tests that may raise the suspicion of Crohn disease include iron-deficiency anemia, vitamin B12 deficiency, and high erythrocyte sedimentation rate (ESR). www.ClinicalReview.com


GASTROINTESTINAL SYSTEM

391

TREATMENT The cornerstone of treatment is to reduce inflammation, which is achieved with sulfasalazine and corticosteroids. Sulfasalazine is a combination of sulfapyridine and 5-aminosalicylic acid (5-ASA). Immunosuppressive therapy is occasionally used, typically in lieu of corticosteroids, and may include the use of 6-mercaptopurine, infliximab, methotrexate, and azathioprine. If Crohn disease primarily affects the small intestine, sulfasalazine is replaced with 5-ASA. Should fistulas complicate the presentation of Crohn disease, administration of metronidazole and ciprofloxacin for three weeks for bowel flora control is the standard of care. Experimental therapies with antibodies or cytokines against tumor necrosis factor (TNF) have also shown some success. TABLE 462 CROHN DISEASE Crohn Disease Pathophysiology

Affects all layers of bowel leading to fistulas and abscesses. Discontinuous lesions.

Presentation

Nonbloody diarrhea, crampy abdominal pain, fever, malaise, tender RLQ.

Diagnosis

Colonoscopy (skip lesions, cobblestoning, abscesses, fistula). Histology (granulomas), high ESR.

Treatment

Sulfasalazine, corticosteroids, metronidazole for bowel flora control, antidiarrheals.

SUPPORTIVE THERAPY Supportive therapy for Crohn disease includes the use of antidiarrheals such as loperamide or diphenoxylate for those with diarrhea due to fatty acids, or cholestyramine for those with other causes of diarrhea. Antidiarrheals are absolutely contraindicated in individuals with toxic megacolon or those with severe colitis that may lead to toxic megacolon. Anticholinergics are used to reduce the symptoms of inflammatory bowel disease, including abdominal cramping and urgency.

SULFASALAZINE The method of action for sulfasalazine is unknown, since other NSAIDs do not have the same beneficial effects in Crohn disease. Sulfasalazine functions as a free radical scavenger and inhibits the synthesis of prostaglandins and leukotrienes. Further, by inhibiting leukocyte motility, it may achieve its anti-inflammatory actions through a combination of effects. Sulfasalazine has the added benefit of controlling bacterial populations in the intestine, and is safe in all populations except those allergic to sulfa drugs. Side effects of sulfasalazine administration include diminished absorption of folic acid, reversible male impotence, and gastrointestinal distress, unless given in the enteric-coated form.

CORTICOSTEROIDS Corticosteroids function to inhibit vasodilation, decrease vascular permeability and proliferation, inhibit neutrophil motility, and decrease collagen deposition and fibroblast activation.

Â


392

USMLE STEP 2

ULCERATIVE COLITIS ETIOLOGY AND EPIDEMIOLOGY Ulcerative colitis is the second major inflammatory bowel disease, and is approximately equal to Crohn disease in incidence. Ulcerative colitis affects the same populations and age groups, but is more Ulcerative colitis: continuous common in women. In addition, the risk of colon cancer with ulcerative colitis is lesions in colorectal mucosa, significantly greater than with Crohn disease: there is a 30 fold increase in risk bloody diarrhea w/ rectal with ulcerative colitis. In 1 out of 10 persons, ulcerative colitis spontaneously discharge, lead pipe colon. remits. In 他, symptoms are intermittent. In 1/10, the symptoms are continuous, Rx: TPC w/ IPAA. and in 1/20, death results due to complications. A positive ANCA is associated with ulcerative colitis.

PATHOPHYSIOLOGY Ulcerative colitis tends to have continuous lesions that are restricted to the mucosa. This tends to lead to a rectal discharge of mostly mucus, blood, and pus. Ulcerative colitis begins with rectal involvement, and lesions in the majority of patients extend from the rectum into adjacent gastrointestinal structures.

PRESENTATION AND DIAGNOSIS Signs and symptoms of ulcerative colitis include bloody diarrhea and rectal pain. Ulcerative colitis is otherwise similar in presentation to Crohn disease. Diagnosis of ulcerative colitis is made through a colonoscopy that demonstrates continuous lesions emanating from the rectum, and a lead pipe colon that is secondary to chronic damage leading to scarring. Granulomas are not present. Extraintestinal complications of inflammatory bowel disease are discussed below.

TREATMENT Treatment of ulcerative colitis is similar to that of Crohn disease, including the use of sulfasalazine and corticosteroids for management of the inflammation. Colectomy is mandatory after 15 years of symptoms to minimize the risk of colon cancer; this is typically curative. TABLE 463 ULCERATIVE COLITIS Ulcerative Colitis Pathophysiology

Continuous lesions restricted to the mucosa. Begins in rectum.

Presentation

Bloody diarrhea and rectal pain.

Diagnosis

Colonoscopy shows continuous lesions starting at rectum. No granulomas.

Treatment

Similar to Crohn disease. Colectomy after 15 years of symptoms. www.ClinicalReview.com


GASTROINTESTINAL SYSTEM

393

INTESTINAL COMPLICATIONS Complications of ulcerative colitis include perforation, stricture formation, and toxic megacolon. Hemorrhage can also occur if there is significant damage to blood vessels. Toxic megacolon is especially likely to occur in the face of severe inflammation that leads to dilation of the bowel. Hypotension and septic changes may also occur in long-standing ulcerative colitis.

EXTRAINTESTINAL COMPLICATIONS Both Crohn disease and ulcerative colitis may be complicated with a number of extraintestinal symptoms. Those that affect the eye include episcleritis and uveitis. Uveitis is more common in Crohn disease. Episcleritis typically presents with the conglomeration of uveitis, erythema nodosum, and colitic arthritis. Dermatologic symptoms include erythema nodosum (especially in Crohn disease), pyoderma gangrenosum (ulcerative colitis), and aphthous ulcers (Crohn disease). Colitis arthritis is more common in Crohn disease, and ankylosing spondylitis is often a progressive feature of ulcerative colitis. Primary sclerosing cholangitis is more common in ulcerative colitis, and renal failure due to amyloidosis is more common in Crohn disease. All forms of inflammatory bowel disease may be complicated with anemia, thromboembolic disorders, steatohepatitis, and cholelithiasis. TABLE 464 COMPLICATIONS Complications Ulcerative colitis

Perforation, stricture formation, and toxic megacolon. Hemorrhage, if there is significant damage to blood vessels. Toxic megacolon is likely to occur, if severe inflammation leads to dilation of the bowel. Hypotension and septic changes may also occur in long-standing ulcerative colitis.

Extraintestinal (Ulcerative colitis & Crohn)

Episcleritis, uveitis, erythema nodosum, colic arthritis, pyoderma gangrenosum, aphthous ulcers, ankylosing spondylitis, primary sclerosing cholangitis, colitis, and renal failure, due to amyloidosis. Note: All forms of inflammatory bowel disease may be complicated with anemia, thromboembolic disorders, steatohepatitis, and cholelithiasis. (see text for complications that occur more often in one disease than another).

DIVERTICULAR DISEASE DIVERTICULOSIS ETIOLOGY Diverticulosis is the development of a saclike outpouching of the colon that herniates in between the teniae coli. Diverticulosis is the most common cause of significant bleeding from the lower gastrointestinal tract in elderly patients. The outpouchings of diverticulosis generally occur in the sigmoid colon, likely due to the higher pressures experienced in this section of the gastrointestinal tract. Diverticulosis is more common in patients who have a diet poor in fiber, and generally affects a majority of the elderly population.

PATHOPHYSIOLOGY AND PRESENTATION Diverticulosis can lead to a lower gastrointestinal bleed due to inflammation of the mucosa that eventually affects an intestinal artery. Profuse bleeding can occur with this erosion, leading to painless hemorrhage and bright red blood per rectum (BRBPR).

Â


USMLE STEP 2 394 The majority of bleeds occur with right-sided diverticula, and tend to resolve spontaneously. Most cases of diverticulosis tend to be painless, but abdominal pain relieved by defecation can occur.

DIAGNOSIS Diagnosis of diverticulosis is made through colonoscopy that detects the outpouching. A more sensitive test is barium enema, which is also a safer test as the colonoscopy can lead to perforation of the sac(s). Tagged red blood cells (RBCs) or angiography can be used to assess the cause of the lower gastrointestinal bleed and potentially correct it.

TREATMENT Treatment of diverticulosis is to correct the cause of the lower gastrointestinal bleeding, as discussed in the section on gastrointestinal bleeds. A high fiber diet is recommended in all patients with existing diverticulosis (and generally, in all patients). A complication of diverticulosis is the inflammation of the outpouching, leading to diverticulitis. TABLE 465 DIVERTICULOSIS Diverticulosis Etiology

Sac-like outpouching of the colon that herniates between the teniae coli.

Presentation

Lower GI bleeding (BRBPR), most cases painless.

Diagnosis

Colonoscopy, (shows outpouching), barium enema, tagged RBCs, angiography.

Treatment

Correct GI bleeding, high fiber diet.

DIVERTICULITIS PATHOPHYSIOLOGY AND PRESENTATION Diverticulitis is the inflammation of the outpouching of the colon due to the formation of a fecalith and subsequent proliferation of bacteria within the diverticulum. Diverticulitis presents with lower abdominal pain, typically in the lower left quadrant (the site of most diverticulitis; a right-sided diverticulitis is typically due to Meckel diverticulitis). There is generally constipation and increased flatus. Irritation from the diverticulum and the inflammatory process can lead to urinary urgency. Further, as this is an infectious process, there is typically fever and peritonitis can occur with perforation of the diverticulum. A sigmoid mass may be present upon physical examination.

DIAGNOSIS Diagnosis of diverticulitis requires blood cultures positive for infection, leukocytosis indicating an inflammatory and infectious process underway, and a computed tomographic (CT) scan can be done to confirm the presence of a diverticulum. In diverticulitis, invasive tests such as barium enema and colonoscopy are contraindicated due to the increased risk of perforation and subsequent peritonitis and sepsis, especially in the acute phase of diverticulitis.

TREATMENT Treatment of diverticulitis is to prescribe nothing per oral (NPO), hydration to assist in the propulsion of intestinal contents, antibiotics to cover bowel flora, and to control pain symptoms. Sigmoid resection should be considered to minimize the Âź chance www.ClinicalReview.com


GASTROINTESTINAL SYSTEM

395 of recurrence, especially in older adults. Finally, testing to rule out colon cancer should be undertaken once the acute phase has resolved. TABLE 466 DIVERTICULITIS Diverticulitis Presentation

Inflammation of outpouching of colon due to fecalith and bacterial proliferation. Left lower abdominal pain, constipation, fever.

Diagnosis

Blood cultures positive for infection, leukocytosis, CT scan (confirm diverticulum).

Treatment

NPO, hydration, antibiotics, sigmoid resection, testing to rule out colon cancer.

GASTROINTESTINAL BLEEDING UPPER GASTROINTESTINAL BLEEDING PATHOPHYSIOLOGY Upper gastrointestinal (GI) bleeds are arbitrarily defined as those located proximal to the ligament of Treitz, also known as the suspensory ligament of the duodenum. Upper GI bleeds can occur at any level between the oral mucosa and the proximal portion of the duodenum, and include causes such as epistaxis, esophageal varices, tears, or ruptures, gastric erosion, PUD, arteriovenous malformations (AVMs) and tumors.

PRESENTATION Upper GI bleeds present with a variety of symptoms, depending on the severity of the blood loss. Minor bleeding is often asymptomatic, but may lead to nausea and subsequent hematemesis with the appearance of bright red blood or coffee grounds, if the blood has been exposed to stomach acid for a long time. More severe bleeding leads to melena with black, tar-like stools, hypotension, and tachycardia from the decrease in blood volume. Very brisk bleeds may lead to hypovolemic shock and bright red blood per rectum (BRBPR). Orthostatic hypotension indicates that more than 20% of the intravascular volume has been lost due to hemorrhage. Breakdown of blood by intestinal flora may lead to elevations in blood urea nitrogen (BUN), leading to false positive kidney function tests. Finally, chronic stomach bleeding may lead to iron deficiency anemia.

DIAGNOSIS Diagnosis of upper GI bleeds is made by directly visualizing the blood via a nasogastric (NG) lavage. A rectal exam with hemoccult testing is always performed. Bleeding sites can often be directly assessed and sometimes immediately corrected via endoscopy. Active bleeding is often identified using tagged red blood cells (RBCs) or arteriography. Finally, laboratory tests include a complete blood count (CBC) to help determine the severity of the hemorrhage. Endoscopy is the standard of care in all upper GI bleeds.

TREATMENT Treatment for upper GI bleeds is often carried out using an esophagogastroduodenoscopy (EGD), which permits the clinician to carry out a number of procedures to ameliorate the hemorrhage. EGD permits outpatient procedures such as electrocoagulation, sclerotherapy, ligation of ruptured varices, and tamponade if more severe. Severe bleeds also require intravenous (IV) fluids and replacement of lost blood as needed (PRN). Total bowel immobilization to minimize blood flow to the gastrointestinal system may

Â


USMLE STEP 2 396 be required through the use of somatostatin. Finally, the risk of recurrent bleeds due to PUD may be reduced through the use of proton pump inhibitors such as omeprazole. TABLE 467 UPPER GASTROINTESTINAL BLEEDING Upper Gastrointestinal Bleeding Pathophysiology

Bleed proximal to the ligament of Treitz (epistaxis, esophageal varices, ruptures, PUD, gastric erosion, AVMs, tumors).

Presentation

Minor bleeding-asymptomatic, hematemesis (bright red or coffee ground); Severe bleeds (tar-like stools, hypotension, tachycardia, shock, BRBPR, elevated BUN.

Diagnosis

Visualized blood via NG lavage, rectal exam, FBOT, tagged RBCs, CBC, endoscopy.

Treatment

EGD with electrocoagulation, sclerotherapy, ligation of varices, tamponade, Ivs.

LOWER GASTROINTESTINAL BLEEDING PATHOPHYSIOLOGY Lower GI bleeds occur distal to the suspensory ligament of the duodenum, and may occur at any location between the duodenum and the anus. Although the most common cause of lower GI bleed in the elderly is due to diverticulosis, all GI bleeds in this population should be suspected as heralding gastrointestinal cancer until this etiology is ruled out. Younger patients are more likely to have lower GI bleeding from hemorrhoids, but this particular etiology is commonly managed by the patient and not seen in a clinical setting unless very severe or troublesome. Other causes of lower GI bleeds include inflammatory bowel disease, mesenteric ischemia, arteriovenous malformation (AVM) such as angiodysplasia, and infections. AVMs may present as part of a hereditary conditions that predispose to angiodysplasias, such as Osler-Rendu-Weber syndrome. Congenital anatomic abnormalities such as Meckel diverticulum may also lead to GI bleeds with a presentation common in teenagers and young adults.

PRESENTATION Lower GI bleeds present with BRBPR, and such bleeding should begin an immediate investigation of lower GI causes. However, not all incidents of BRBPR are due to lower GI bleeds – a particularly brisk upper GI bleed may also lead to fresh blood expressed from the rectum. Laboratory tests for occult bleeding are positive, and other systemic symptoms such as tachycardia and hypovolemia may present in a manner similar to upper GI bleeds.

DIAGNOSIS Diagnosis of lower GI bleeds is made after ruling out causes of upper GI bleed (through NG lavage). A digital rectal exam is mandatory to test for occult bleeding, followed by a colonoscopy in an attempt to directly visualize and potentially correct obvious sources of bleeding. As per upper GI bleeds, a tagged RBC scan and arteriography are often done. A CBC is done. The most common cause of a lower GI bleed is an upper GI bleed.

TREATMENT Treatment for a lower GI bleed is similar to that for upper GI bleed. All patients with GI bleeds should receive two large bore IV lines in preparation for significant volume resuscitation. A type and cross should be ordered should emergent use of blood products become necessary. Severe cases of bleeding from the lower GI tract are sometimes treated with colectomy.

www.ClinicalReview.com


GASTROINTESTINAL SYSTEM

397

TABLE 468 LOWER GASTROINTESTINAL BLEEDING Lower Gastrointestinal Bleeding Pathophysiology

Occur distal to the duodenal suspensory ligament. May be diverticulosis (elderly), hemorrhoids (young), cancer, inflammatory bowel disease, AVM, Meckel diverticulum.

Presentation

BRBPR.

Diagnosis

Rule out upper GI bleed. Digital rectal exam, colonoscopy, tagged RBCs, CBC.

Treatment

IVs, colectomy.

MESENTERIC ISCHEMIA ETIOLOGY Mesenteric ischemia is the compromise of GI blood flow. It is typically caused by CAD leading to celiac artery, superior mesenteric artery (SMA) or inferior mesenteric artery (IMA) obstruction. Atrial fibrillation leading to thromboembolic phenomenon, low flow states contributing to hemostasis, mesenteric thrombi, and hypercoagulable states also contribute to mesenteric ischemia.

PRESENTATION AND DIAGNOSIS Mesenteric ischemia presents with severe abdominal pain that worsens with ingestion. Late signs include metabolic lactic acidosis with BRBPR. Diagnosis is confirmed by angiography, but a spiral CT with contrast can also be used.

TREATMENT Mesenteric ischemia is treated supportively with tissue perfusion. Surgery is required. TABLE 469 MESENTERIC ISCHEMIA Mesenteric Ischemia Etiology

It is typically caused by CAD leading to celiac artery, SMA or IMA obstruction. Atrial fibrillation, low flow states, mesenteric thrombi, & hypercoagulable states also contribute to mesenteric ischemia.

Presentation

Severe abdominal pain that worsens with ingestion. Late signs include metabolic lactic acidosis with BRBPR.

Diagnosis

Angiography, but a spiral CT with contrast can also be used.

Treatment

Surgery is required. Supportively with tissue perfusion.

CANCERS CARCINOID SYNDROME ETIOLOGY AND EPIDEMIOLOGY Carcinoid syndrome is the result of a neuroendocrine tumor that secretes hormones or neurotransmitters that have an effect on the gastrointestinal system. These active compounds can include serotonin (5-HT), adrenocorticotrophic hormone (ACTH), histamine, dopamine, tryptophan, substance P, and bradykinin. Carcinoid is an idiopathic disorder that has no known familial

Â


USMLE STEP 2 398 inheritance pattern. It is part of the MEN type I disorder, and the incidence of carcinoid syndrome is greater in patients who have Gardner syndrome or Crohn disease.

PATHOPHYSIOLOGY The majority of carcinoids are in the appendix, but the endocrine cells in this location tend not to have any effect. The majority of carcinoids in the ileum are secreting, as are those found in the lung. Carcinoid typically causes most of its effects through the conversion of tryptophan to serotonin, and the unchecked production of serotonin may lead to symptoms of niacin deficiency and subsequent pellagra.

PRESENTATION The majority of carcinoids are asymptomatic and are discovered only incidentally. Those located in the appendix may create symptoms of appendicitis, and any carcinoid in the small intestine may lead to a small bowel obstruction or diarrhea. Carcinoid, like any tumor, may lead to weight loss. The classic triad of carcinoid rarely presents with the tripartite symptoms, but it includes flushing due to excessive bradykinin production, diarrhea from excessive serotonergic effects, and valvular heart disease that primarily affects the mitral valve from excess serotonin. Other classic symptoms include hypotension, tachycardia, and alcohol intolerance. Carcinoid may metastasize to the liver, leading to right upper quadrant (RUQ) pain and elevated liver function tests (LFTs) and the lung, leading to wheezing, obstructed bronchus, and pneumonia.

DIAGNOSIS Diagnosis of carcinoid syndrome is made with a 24 hour 5-hydroxyindolacetic acid (5-HIAA) collection, with elevated titers 100% specific for carcinoid. Point collections through serum or urine samples may also be done. 5-HIAA is the metabolite of serotonin when it is broken down by monoamine oxidase (MAO) and then by aldehyde dehydrogenase. CT of the lung and abdomen are compulsory to detect metastasis.

TREATMENT Carcinoid syndrome is best treated with surgical resection and radiation therapy. Appendiceal Carcinoid: <2cm at With metastasis to the liver, embolization and alpha interferon therapy are often distal appendix = appendectomy used, in addition to surgical resection. Symptomatic control of carcinoid can >2cm or at base = right sometimes be achieved with the somatostatin analog octreotide. Survival after a hemicolectomy diagnosis of active carcinoid syndrome is typically 3 years. Carcinoid primarily located in the appendix carries a very good prognosis, while primary carcinoid outside of the appendix has a 50% 5 year survival. 3 years/50% contradictory

www.ClinicalReview.com


GASTROINTESTINAL SYSTEM

399

TABLE 470 CARCINOID SYNDROME Ulcerative Colitis Etiology

Idiopathic.

Pathophysiology

Majority in appendix, those in ileum are secreting, niacin deficiency.

Presentation

Most asymptomatic, possible symptoms of appendicitis, SI blockage, diarrhea.

Diagnosis

24 hour 5-HIAA collection (elevated titers specific for carcinoid) CT lung & abdomen.

Treatment

Surgical resection and radiation therapy.

COLORECTAL CANCER EPIDEMIOLOGY The second most common cause of death due to cancer is from colon cancer, affecting both males and females equally. The most common underlying etiology is the presence of sessile, villous, adenomatous polyps greater than 2 cm in size. Risk factors for colon cancer include a positive family history, the presence of APC gene or p53 gene defects, ulcerative colitis (more than Crohn disease), and smoking. Longstanding infection with Streptococcus bovis can also predispose individuals to colon cancer, while chronic aspirin use over a long period of time somewhat reduces the risk of colon cancer.

FAMILIAL ADENOMATOUS POLYPOSIS Colon cancer develops over approximately a decade of having large villous or tubulovillous adenomatous polyps. The presence of familial adenomatous polyposis (FAP) syndromes, due to an autosomal dominant gene, is a certain predictor of malignancy. FAP is due to a deletion on chromosome 5q, and it requires total colectomy to avoid colon cancer from the numerous polyps that inevitably form by the age of 25. Without treatment, colon cancer is likely to have occurred in nearly all individuals by the age of 40.

HEREDITARY NONPOLYPOSIS COLON CANCER Hereditary nonpolyposis colon cancer (HNPCC) is an autosomal dominant disorder with a strong family inheritance. In most cases, early colon cancer develops in several relatives of the patient, and there is a strong risk of the development of other cancers within the family. HNPCC is a strong predictor of future colon cancer development in the patient. In this disorder, also known as Lynch syndrome, the cancer arises spontaneously from the mucosa, and it is a strong predictor of future ovarian or endometrial cancer. TABLE 471 COLORECTAL CANCER Colorectal Cancer Etiology

Genetics, ulcerative colitis, smoking, S. bovis.

Pathophysiology

FAP, HNPCC—autosomal dominant gene.

Treatment

FAP—Total colectomy.


400

USMLE STEP 2

GARDNER SYNDROME Gardner syndrome is the development of numerous polyps due to an autosomal dominant disorder. Gardner’s syndrome is further complicated by the presence of fibrous dysplasia of the skull, osteomas, and extra teeth. Gardner syndrome typically evolves into colon cancer.

TURCOT SYNDROME Turcot syndrome is the development of polyps and tumors within the central nervous system (CNS). It presents a significant risk for colon cancer.

PEUTZ-JEGHER SYNDROME Peutz-Jegher syndrome presents a relatively low risk of developing colon cancer. Many polyps develop within the small and large intestine, but they are typically hamartomas and therefore benign. Peutz-Jegher syndrome typically leads to pigmentation of the cutaneous and mucosa, especially with the development of macules within the oral mucosa. While the risk of colon cancer is low, Peutz-Jegher should raise the suspicion of cancers that occur in women.

JUVENILE POLYPOSIS SYNDROME Juvenile polyposis syndrome is an insignificant cause of colon cancer, with polyps often occurring as a result of benign hamartomas within the intestines. Peutz-Jegher and juvenile polyposis syndrome are worth keeping on the differential diagnosis of any polyposis syndrome as rapid diagnosis of these two varieties may avoid costly and invasive tests and obviate the need for prophylactic colon resection. TABLE 472 OTHER COLONIC SYNDROMES Other Colonic Syndromes Gardner syndrome

Numerous polyps due to autosomal dominant disorder. Evolves into colon cancer.

Turcot syndrome

Polyps and tumors in CNS. High risk for colon cancer.

Peutz-Jegher synd.

Many polyps in SI and LI, low cancer risk.

Juvenile Polyposis

Benign hamartomas in intestines.

PRESENTATION Presentation of colon cancer is with weight loss leading to anorexia and wasting, and generalized malaise. Colon cancer emanating from the right colon presents with anemia and pallor. Left sided colon cancer presents with thin-caliber stools, obvious rectal bleeding, and a mass on rectal exam. Nearly half of all colon cancers will be on the left side and within reach of a digital www.ClinicalReview.com


GASTROINTESTINAL SYSTEM

401

rectal exam. Left sided colon cancer occasionally presents with constipation, nausea, and vomiting. Colon cancer that has metastasized to the liver will present with hepatomegaly and other sites will have masses palpable on physical examination.

DIAGNOSIS Diagnosis of colon cancer is typically done with barium enema that indicates a filling defect in the shape of an apple core. This particular pattern occurs due to the circumferential limitation of the lumen due to inward growth of the cancer. CT scan with contrast is done, and plain films of other parts of the body are often taken to rule out metastasis. Laboratory tests often indicate a hypochromic, microcytic anemia. The response of colon cancer to treatment is sometimes gauged by tracking the level of carcinoembryonic antigen (CEA), but CEA cannot be used for routine tests to rule out colon cancer as it is a nonspecific marker. TABLE 473 COLON CANCER Colon Cancer Presentation

Anorexia, wasting, malaise. Right colon (anemia, pallor), left colon (rectal blooding, thin stools, mass on rectal exam).

Diagnosis

Barium enema (apple core shape), CT scan (with contrast).

Treatment

Stages A & B-surgery; Stage C-surgery and chemotherapy; Stage D-palliation.

CLASSIFICATION Following the diagnosis of colon cancer, classification into one of four Duke’s stages is done to determine the prognosis. In stage A disease, the cancer is limited to the mucosa and submucosa, and carries a greater than 90% 5 year survival. In stage B disease, the cancer has invaded the muscularis propria, with a 70% 5 year survival. Stage C disease has invasion into local lymph nodes, with a 30% 5 year survival. Stage D disease has distant metastases with limited survival after a few years. TABLE 474 STAGING OF COLON CANCER—DUKE’S STAGES Staging of Colon Cancer—Duke’s Stages Stage A

Cancer limited to mucosa and submucosa. 90% 5 year survival.

Stage B

Cancer invades the muscularis propria. 70% 5 year survival.

Stage C

Invasion of local lymph nodes. 30% 5 year survival rate.

Stage D

Distant metastases. Limited survival.

TREATMENT Colon cancer that falls into Duke’s stage A or B is highly amenable to surgery, and wide resection of the colon is typically done along with sampling of nearby lymph nodes to rule out higher stages of disease. Stage C disease is often treated with a combination of surgery and chemotherapy, including agents such as 5-fluorouracil (5-FU) and leucovorin. Stage D disease is treated only with palliation. Radiation therapy is often used for stage B and C disease, and chemotherapy is sometimes employed in stage B disease.

Stage A: Resection Stage B: Resected Stage C: Chemoradiation, resection Stage D: Palliation


402

USMLE STEP 2

TABLE 475 TREATMENT OF COLON CANCER Treatment of Colon Cancer Stage A & B

Surgery and wide resection of colon, sampling of lymph nodes (radiation and chemo are sometimes used in B).

Stage C

Surgery and chemotherapy (5-FU, leucovorin), radiation may also be utilized.

Stage D

Palliation.

SCREENING Colon cancer screening guidelines are proposed by the American Cancer Society, and recommends annual digital rectal exam in all individuals age 40 and over. Occult blood tests with the stool (FOBT) are recommended in individuals over the age of 50. In asymptomatic individuals with no risk factors for colon cancer, flexible sigmoidoscopy is done every 5 years starting at the age of 50. Finally, colonoscopy is recommended in any individual with positive stool guaiac (positive FOBT), those with a high risk assessment following sigmoidoscopy, and for those with inflammatory bowel disease and hereditary risk factors. Many practitioners now recommend the use of colonoscopy in lieu of sigmoidoscopy due to the increased sensitivity and specificity, the reach of colonoscopy to the ascending and transverse colon (where half of all colon cancers may lie), and the ability to biopsy polyps with the colonoscope. Digital rectal exam may find up to half of all colon cancers, and many practitioners urge that a digital rectal exam be performed with any abdominal exam. TABLE 476 SCREENING OF COLON CANCER Screening of Colon Cancer Annual digital rectal exam in all over 40. FOBT in all over 50. Flexible sigmoidoscopy every 5 years over 50. Colonoscopy for those with risk factors.

HERNIAS FEMORAL HERNIA The femoral canal is located below the inguinal ligament on the lateral aspect of the pubic tubercle. Inguinal hernias are above and medial to the pubic tubercle, while femoral hernias are inferior and lateral to the pubic tubercle. The femoral canal is bounded by the inguinal ligament anteriorly, the pectineal ligament posteriorly, the lacunar ligament medially, and the femoral vein laterally. The canal typically consists of empty space or rare lymphatics. A node is occasionally located within the femoral canal, known as Cloquet's node. Femoral hernias occur when abdominal contents pass through the femoral canal. Femoral hernias are located medial to the femoral vein.

www.ClinicalReview.com


GASTROINTESTINAL SYSTEM

403

INGUINAL HERNIA An indirect inguinal hernia protrudes through the inguinal ring and is typically congenital in nature due to failure to close the internal inguinal ring after the testicle passes through it during development. Indirect inguinal hernias are lateral to the inferior epigastric vessels and are covered by the internal spermatic fascia. Direct inguinal hernias enter through a weakness in the abdominal floor and are acquired. They are located medial to the inferior epigastric vessels and are not covered by the internal spermatic fascia. A reduced incarcerated hernia that presents with a small bowel obstruction requires surgical exploration. MDs (direct hernia, medial to inferior epigastric vessels) don’t Lie (indirect hernia, lateral to vessels). Also, Indirect = Infantile.

COMPLICATIONS OF HERNIA REPAIR The presence of oliguria following laparoscopic hernia repair may be due to ureteral injury. The most common nerve injured in the TAPP (transabdominal preperitoneal) approach to hernia repair is the genitofemoral nerve (2%) of the time. This is closely followed by the ilioinguinal nerve (1.1%) and lateral cutaneous nerve of the thigh (1.1%). The ilioinguinal, iliohypogastric, and genitofemoral nerves also go through the region and are often transected intentionally or accidentally. Damage to the genital branch of the genitofemoral nerve leads to loss of the cremaster reflex and loss of sensation to the penis and scrotum. The femoral branch of the genitofemoral nerve provides sensation to the proximal medial thigh. The ilioinguinal provides redundant sensation to this region. Another complication includes hemorrhage from the obturator artery, deep circumflex iliac artery, external iliac artery, cremaster artery, or inferior epigastric vessels. Constriction of the femoral vein is possible in certain repairs, leading to DVT and possible PE postoperatively. Intentional or accidental transection of the spermatic cord leads to swelling in the testis followed by atrophy in about 1/3 of patients. Orchiectomy is not required following transection of the spermatic cord. Damage to the vas deferens should be repaired primarily. Damage to the intestines is also possible during a high ligation of the hernia sac, and care must be taken to open the hernia sac and deliberately reduce the contents of the sac prior to ligation. This is more likely to occur with an indirect hernia. Damage to the bladder wall can occasionally occur with a direct hernia, and closure in two layers is required along with foley catheter drainage. Recurrence following indirect or direct inguinal hernia repair is about 5% per year.

Â


USMLE STEP 2

404

PRACTICE QUESTIONS What is the source of the gastroduodenal artery? A. B. C. D. E.

Celiac artery Common hepatic artery Left gastric artery Proper hepatic artery Superior mesenteric artery

The best answer is Common hepatic artery. The gastroduodenal artery is a direct branch of the common hepatic artery before it becomes the proper hepatic artery. Damage to the common hepatic artery typically does not compromise the gastroduodenal artery due to its anastomosis with the superior mesenteric artery via the pancreaticoduodenal branches, and liver function is typically preserved. However, damage to the proper hepatic arteries often leads to liver ischemia.

A 36 year old female presents to the surgery clinic with a right lower quadrant bulge that is difficult to reduce. She has signs and symptoms of a bowel obstruction and is taken to the OR for reduction and repair. She is diagnosed with a Spigelian hernia. Which of the following best describes the particular anatomy of this hernia? A. B. C. D. E.

Hernia is anterior to the external oblique and travels between the internal oblique and transversus abdominus Hernia is anterior to the external oblique and travels medial to the internal oblique and transversus abdominus Hernia is posterior to the external oblique and travels between the internal oblique and transversus abdominus Hernia is posterior to the external oblique and travels lateral to the internal oblique and transversus abdominus Hernia is posterior to the external oblique and travels medial to the internal oblique and transversus abdominus

The best answer is Hernia is posterior to the external oblique and travels between the internal oblique and transversus abdominus. A Spigelian hernia commonly leads to an incarcerated hernia that usually requires urgent surgical management for signs and symptoms of bowel obstruction. It travels between the attachment of the internal oblique and transversus abdominus to the rectus sheath and lies posterior to the external oblique muscle.

Which of the following organs utilizes glutamine as its primary nutritional resource? A. B. C. D. E.

Esophagus Large intestine Rectum Small intestine Stomach

The best answer Is Small intestine. The primary source of energy for the small intestine is glutamine. Neoplastic cells also utilize this resource. The large intestine uses butyrate and other small chain fatty acids.

www.ClinicalReview.com


GASTROINTESTINAL SYSTEM

405

Which of the following is a non-mechanical cause leading to bowel obstruction? It is receptive to therapy with neostigmine. A. B. C. D. E.

Colorectal cancer Crohn disease Diverticulitis Ogilvie syndrome Ulcerative colitis

The best answer is Ogilvie syndrome. Ogilvie syndrome is the acute pseudoobstruction and dilation of the colon in the absence of any mechanical obstruction in severely ill patients. Colonic pseudo-obstruction is characterized by massive dilatation of the cecum (diameter > 10 cm) and right colon on abdominal X-ray. Recent surgery (most common following coronary artery bypass surgery),neurologic disorders, serious infections, cardiorespiratory insufficiency, metabolic disturbances, and drugs that disturb colonic motility (e.g. anticholinergics or narcotics) contribute to the development of this condition. The exact mechanism behind the acute colonic pseudo-obstruction is not fully elucidated. The probable explanation is imbalance in the regulation of colonic motor activity by the autonomic nervous system. It usually resolves with conservative therapy stopping oral ingestions, i.e. nil per os and a nasogastric tube, but may require colonoscopic decompression which is successful in 70% of the cases. A study published in the New England Journal of Medicine showed that neostigmine is a potent pharmacological way of decompressing the colon. According to the American Society for Gastrointestinal Endoscopy (ASGE), it should be considered prior to colonoscopic decompression. The use of neostigmine is not without risk since it can induce bradyarrhythmia and bronchospasms. Therefore atropine should be within immediate reach when this therapy is used. Mortality can be as high as 30% in Ogilvie syndrome.

A 28 year old female has a significant family history of colorectal cancer, with numerous first-degree and seconddegree relatives diagnosed and treated early for cancer. Which of the following would you consider for this patient? A. B. C. D. E.

Colonoscopy to search for proximal tumors in FAP Genetic testing to determine whether she has autosomal recessive HNPCC Prophylactic chemotherapy Prophylactic subtotal colectomy with mucous fistula for Peutz-Jegher syndrome Prophylactic total abdominal hysterectomy with bilateral salpingo-oopherectomy if she has HNPCC type 2.

The best answer is Prophylactic total abdominal hysterectomy with bilateral salpingo-oopherectomy if she has HNPCC type 2. Hereditary nonpolyposis colon cancer (HNPCC) is an autosomal dominant disorder with a strong family inheritance. In most cases, early colon cancer develops in several relatives of the patient, and there is a strong risk of the development of other cancers within the family. HNPCC is a strong predictor of future colon cancer development in the patient. In this disorder, also known as Lynch syndrome, the cancer arises spontaneously from the mucosa, and it is a strong predictor of future ovarian or endometrial cancer. Lynch II syndromes include right sided colon cancer due to mismatch repair. It is typically autosomal dominant. Colon cancer develops over approximately a decade of having large villous or tubulovillous adenomatous polyps. The presence of familial adenomatous polyposis (FAP) syndromes, due to an autosomal dominant gene, is a certain predictor of malignancy. Prophylactic chemotherapy has no role in therapy.

Â


USMLE STEP 2

406

A 62 year old male undergoes a low anterior resection for colorectal cancer. The specimen is sent to pathology, which indicates invasion of the muscularis propria but 0 out of 17 lymph nodes come back as positive. What is the correct staging, therapy, and expected survival? A. B. C. D. E.

T1N0M0, no chemotherapy, 5 year survival over 50% T1N0M0, no chemotherapy, 5 year survival over 90% T1N1M0, no chemotherapy, 5 year survival over 70% T2N0M0, chemotherapy, 5 year survival over 70% T3N0M0, chemotherapy, 5 year survival over 50%

The best answer is T1N0M0, no chemotherapy, 5 year survival over 90%. This patient has a T1N0M0 tumor, and therefore does not require chemotherapy. Such tumors have an expected 90% five year survival. In stage A disease, the cancer is limited to the mucosa and submucosa, and carries a greater than 90% 5 year survival. In stage B disease, the cancer has invaded the muscularis propria, with a 70% 5 year survival. Stage C disease has invasion into local lymph nodes, with a 30% 5 year survival. Stage D disease has distant metastases with limited survival after a few years. Colon cancer that falls into Duke’s stage A or B is highly amenable to surgery, and wide resection of the colon is typically done along with sampling of nearby lymph nodes to rule out higher stages of disease. Stage C disease is often treated with a combination of surgery and chemotherapy, including agents such as 5-fluorouracil (5-FU) and leucovorin. Stage D disease is treated only with palliation.

A diagnosis of carcinoid is made after a one centimeter mass is discovered at the distal end of the appendix following an appendectomy. What is the next best step? A. B. C. D.

Cecectomy with primary anastomosis No further surgical procedure Right hemicolectomy with ileostomy Right hemicolectomy with primary reanastomosis

The best answer is No further surgical procedure. Carcinoid less than 2 cm in size at the tip of the appendix can be treated by appendectomy alone. A carcinoid that is larger than 2 cm or at the base of the appendix must be treated by a right hemicolectomy. Metastasis is rare, and nearly ¾ of tumors are located at the distal extent.

Infliximab (remicade) is used as a therapy for Crohn disease. Which of the following best characterizes the nature of infliximab? A. B. C. D. E.

Chimeric anti-TGF antibody Monoclonal antibody Protein C inhibitor Recombinant DNAase Tumor marker

The best answer is Monoclonal antibody. Infliximab (brand name Remicade) is a drug used to treat autoimmune disorders. Infliximab is a chimeric monoclonal antibody. The drug blocks the action of TNFα (tumor necrosis factor alpha) by binding to it and preventing it from signaling the receptors for TNFα on the surface of cells. TNFα is one of the key cytokines that triggers and sustains the inflammation response. www.ClinicalReview.com


GASTROINTESTINAL SYSTEM

407

Which of the following organisms is tied to the development of a lymphoma? A. B. C. D. E.

Difficile E. coli fragilis H. pylori S. bovis

The best answer is H. pylori. The presence of H. pylori has also been tied to the development of lymphoproliferative disease in the form of mucosa-associated lymphoid tissue (MALT) lymphoma. Most lymphomas of the stomach tend to be of the non-Hodgkin lymphoma variant. Chronic superficial gastritis can also occur, along with chronic atrophic gastritis. The latter condition can lead to the development of gastric adenocarcinoma. C. difficile leads to colitis. S. bovis has been tied to colorectal cancer.

Which of the following tumor suppressor genes is the first to be mutated in the pathway leading to colorectal cancer? A. B. C. D. E.

APC DCC hMSH2 K-ras p53

The best answer is APC. APC is the earliest change that occurs in the pathway to colorectal cancer. A further dysplastic change occurs in the oncogene K-ras. Later changes leading to carcinoma occur in p53 and DCC.

Which of the following patients requires a mandatory Hartmann’s procedure? A. B. C. D. E.

23 year old female with gunshot wound to rectum, 3 hours old 37 year old female with stab wound to left colon, 2 hours old 44 year old male with gunshot wound to transverse colon, 1 hour old 53 year old male with gunshot wound to right colon, 4 hours old 71 year old female with stab wound to transverse colon, 4 hours old

The best answer is 23 year old female with gunshot wound to rectum, 3 hours old. Low velocity or stab wounds to the colon less than 4-6 hours old may be primarily repaired after sufficient washout. The patient must be stable and minimal soilage must be the case. However, all extraperitoneal rectal wounds require washout and a diverting colostomy.

Â


USMLE STEP 2

408

Following an emergent thoracotomy for several stab wounds sustained to the abdomen, an 18 year old teenager has a drop off in urine output and sudden difficulty with ventilation. What is the next best step in management? A. B. C. D. E.

Cisatracuronium Exploratory laparotomy Increase ventilation settings Succinylcholine Thoracotomy

The best answer is Exploratory laparotomy. This patient has abdominal compartment syndrome until proven otherwise. Signs include the decrease in urine output due to compression of the renal vessels and ureter, and difficulty with ventilation due to the high abdominal pressures precluding ventilation of the lungs. Management includes going to the OR for exploratory laparotomy and decompression. On a related note, a positive DPL includes the presence of over 100,000 RBCs per mm3, over 500 WBCs per mm3, amylase over 175, bile, or urine. These are all indications for an exploratory laparotomy.

A 36 year old male presents with an incarcerated inguinal hernia in his left groin. The hernia cannot be reduced, is exquisitely tender, and appears to have overlying skin changes. What is the next best step in management? A. B. C. D. E.

Inguinal hernia repair Inguinal hernia repair with small bowel resection NPO + NGT and IVF Reduction of hernia in the OR Small bowel resection

The best answer is Inguinal hernia repair. This patient has an incarcerated inguinal hernia with signs of strangulation. This patient requires immediate operative intervention with primary reduction of the hernia and repair of the underlying defect. If ischemic bowel is found, a small bowel resection may also be necessary. Whenever possible, elective repair is preferred due to less morbidity and mortality. Recurrence of a hernia following mesh repair is most likely due to migration of the mesh. Repair of hernias via laparoscopy may lead to damage of the lateral femoral cutaneous nerve, presenting as numbness in the lateral proximal lower extremity. Related to this discussion, a Spigelian hernia is due to a defect in the linea semilunaris between the rectus and transversalis muscles. Repair is required. In the event of infection following hernia repair, the most common offending organism is group A Staphylococcus. Obturator hernias require immediate repair (positive Howship-Romberg test), as do Grynfelt hernia’s through the L12 vertebral column. The lateral border of a femoral hernia is the femoral vein; medially is the lacunar ligament; posteriorly the pectineal ligament; anteriorly the inguinal ligament.

www.ClinicalReview.com


GASTROINTESTINAL SYSTEM

409 A 64 year old male is diagnosed with a 2 cm squamous cell carcinoma of the anus. The tumor is located just 4 cm from the anal verge and can be palpated with a finger. The patient presents with itching and bleeding. What is the first step in management? A. B. C. D. E.

APR APR followed by chemoradiation Chemoradiation Chemoradiation followed by APR LAR

The best answer is Chemoradiation. The initial step in management is chemotherapy with radiation therapy. Chemotherapeutic agents include 5-FU and mitomycin. If this approach fails, the next step is APR.

A 46 year old alcoholic presents with worsening ascites and a new umbilical hernia. There is evidence of some skin breakdown around the umbilicus. Which of the following is the next best step in management? A. B. C. D. E.

Antibiotics and observation Liver transplantation Peritovenous shunt with umbilical hernia repair Spironolactone, lasix, and salt restriction Umbilical hernia repair

The best answer is Peritovenous shunt with umbilical hernia repair. This patient has an umbilical hernia due to worsening ascites with new evidence of superficial skin breakdown. This requires an immediate surgical intervention due to the elevated risk of bacterial peritonitis with seeding through the broken skin. Repair of the umbilical hernia alone is insufficient; this patient also requires diversion of his ascites, which is accomplished via a peritovenous shunt. If the patient did not have skin breakdown, treatment with spironolactone, lasix, and salt restriction would be sufficient. The cause of the patient’s worsening ascites is likely retention of fluid secondary to elevated aldosterone.

Â


410

USMLE STEP 2

www.ClinicalReview.com


HEPATOPANCREATOBILIARY

GASTROINTESTINAL SYSTEM

411

CHAPTER CONTENTS Basic Science .....................................................................................................412 Hepatic Disease .................................................................................................420 Pancreatic Disorders .........................................................................................428 Spleen ...............................................................................................................431 Biliary Disease ...................................................................................................431 Cancer and Transplantation..............................................................................434 Practice Questions ............................................................................................436


412

USMLE STEP 2

HEPATOPANCREATOBILIARY SYSTEM HEPATOPANCREATOBILIARY SYSTEM BASIC SCIENCE EPIDEMIOLOGY It is estimated that over 25,000 patients die yearly from liver disease and cirrhosis. This makes liver disease the eighth leading cause of death in the United States, and nearly ¾ of the cases of cirrhosis can be prevented by better treatment of alcohol abuse. Hepatitis C contributes to nearly half of all liver disease deaths, and infects nearly 3 million people in the US. Hepatitis B contributes nearly one quarter to all liver disease deaths, with close to 100,000 new cases diagnosed yearly.

ANATOMY LIVER LOBES AND DIVISIONS The liver is a three pound, cone-shaped organ that lies in the RUQ. It is divided into two main lobes, each composed of thousands of lobules. The ducts within each of the lobules coalesce to eventually form the hepatic duct, which transports bile produced by the liver to the gallbladder, and then to the duodenum. The liver is supplied by the hepatic artery, which provides oxygenated blood. The hepatic portal vein supplies minerals and nutrients from digestion to the liver. At any particular time, the liver contains over 10% of the body’s total blood volume. Nearly ¾ of the liver can be destroyed before it is unable to carry out its functions. The liver is divided into right and left lobes using an artificial demarcation between the inferior vena cava and the gallbladder fossa. As indicated in the above diagram, the liver segment lateral to the gallbladder fossa is segment 5. The common bile duct is located parallel, anterior, and lateral to the hepatic artery (i.e. to the right of the hepatic artery). The portal vein is located posteriorly.

HISTOLOGY The liver is composed of hepatocytes (50% by number, 80% by volume), Kupffer cells (macrophages), and endothelium. Hepatocytes are divided into three zones. Zone I cells are nearest the periportal blood supply and process incoming nutrients.

www.ClinicalReview.com


Hepatopancreatobiliary System

413

These cells primarily produce protein. Zone III cells are far from arterial blood and take part in glycogenolysis and Lipogenesis. Formation of urea occurs in zones II and III. Gluconeogenesis predominantly occurs in zone I.

BILIARY TREE The biliary system is composed of the gallbladder, bile ducts, and related transport systems to deliver bile from the liver to the duodenum. The right and left hepatic ducts collect bile produced by the thousands of lobules of the liver and transport it to the common hepatic duct and then the gallbladder for bile storage. The common hepatic duct joins with the cystic duct from the gallbladder to form the common bile duct, which transports bile to the duodenum for emulsification of fats and release of waste products from the liver.

PANCREAS The pancreas is the abdominal organ located dorsal to the stomach and extends between the right and left upper quadrants. The widest part of the pancreas is known as the head, and is located within the first section of the intestines – the duodenum. The left side tapers and travels somewhat superiorly and is known as the body of the pancreas. The pancreas terminates as the tail near the spleen. The pancreas is composed of exocrine tissue that secretes digestive enzymes into the small intestine and endocrine tissue that secretes hormones into the bloodstream. The blood supply to the head of the pancreas is from the superior and inferior branches emanating from the gastroduodenal artery and superior mesenteric artery, respectively. In the event of a transection of the pancreatic neck, a distal pancreatectomy should be done. The dorsal pancreatic artery is a branch of the splenic artery, and supplies the body and tail of the pancreas along with a bit of the head. It anastomosis with the posterior arcades and left gastroepiploic artery. The transverse pancreatic artery comes from the left gastroepiploic artery. Venous drainage of the pancreas terminates in the portal vein. As a result, splenic vein occlusion leading to gastric varices is possible with pancreatic cancer. Gastric varices occur due to retrograde venous drainage from flow diversion into the short gastric and left gastroepiploic veins.

PHYSIOLOGY HORMONAL MEDIATORS GALLBLADDER REGULATION The liver serves to regulate the concentration of toxic chemicals and byproducts within the body. It also processes ingested minerals and nutrients and converts most ingested substances into a more easily used form. The liver serves to create proteins necessary for coagulation, breaks down waste products and uses bile to carry them to the intestines, serves as a storage organ for glucose in the form of glycogen, controls the concentration of amino acids, forms urea from toxic ammonia, stores iron for use with hemoglobin, and produces a number of factors present in inflammatory and immune reactions for control of infection. Bile itself is composed of waste products, cholesterol, and bile salts and is responsible for the dark color of feces. Impaired bile secretion leads to clay-colored stools. Bile is released from the gallbladder through stimulation by CCK. Emptying of the gallbladder is inhibited by parasympathetic blockade. The gallbladder absorbs a significant amount of water and sodium through sodium diffusion created by a sodium-potassium ATPase. Water absorption is linked to this active sodium transport.

PANCREAS The exocrine pancreas serves a distinct function from the endocrine pancreas, even though the two tissues are juxtaposed within the same organ. The exocrine pancreas releases a number of enzymes into the pancreatic duct in an inactive form. These

Â


USMLE STEP 2 414 digestive enzymes are activated by the acidic environment within the duodenum and break down carbohydrates, proteins, and fats. The exocrine pancreas also secretes bicarbonate to neutralize the hydrochloric acid produced by the parietal cells of the stomach. Nearly 3L of bicarbonate and enzyme-rich fluid are produced every day by the pancreas, and the release of these compounds is facilitated by cholecystokinin (CCK), secretin, and bile salts. The endocrine pancreas secretes insulin and glucagon to regulate the titer of sugar within the bloodstream, and also somatostatin to control the function of the intestines and modulate the concentration of insulin and glucagon. The amount of bicarbonate secretion from the pancreas is inversely related to the amount of chloride secretion.

SERUM-ASCITES ALBUMIN GRADIENT The SAAG is calculated by comparing the albumin concentration in the serum to that of the ascites fluid. If the difference between the serum and the ascites is greater than 1.1 and portal hypertension is present, it indicates the underlying etiology to likely be due to liver disease such as hepatitis, cirrhosis, liver failure, or HCC; it may alternatively be due to congestion caused by hepatic failure, heart failure, constrictive pericarditis, tricuspid insufficiency, or Budd-Chiari syndrome. A SAAG less than 1.1 with hypoalbuminemia typically indicates that nephrotic syndrome, severe malabsorption with protein loss, or malnutrition with anasarca are the cause. Other conditions that may lead to a SAAG less than 1.1 include ascites of pancreatic, bile, stomach, kidney, bladder, or ovarian origin. A diseased peritoneum can also present with a SAAG less than 1.1 with causes including infection, malignancy, and rarer conditions such as familial Mediterranean fever (FMF), vasculitis, granulomatous peritonitis, and eosinophilic peritonitis.

HEPATIC BIOSYNTHESIS Proteins created by the liver include albumin, which has a halflife of 3 weeks, prealbumin, which has a half-life of 2 days, and transferrin, which has a half-life of 10 days. Factor VII has the shortest half-life at 5 hours. The cytochrome p450 system in the liver plays a role in toxin metabolism. Phase I reactions lead to oxidation and reduction, while phase II reactions lead to conjugation. Hepatic biotransformation is dependent on the cytochrome P450 system.

BIOCHEMISTRY BILE SALT METABOLISM The most important bile acids are cholic acid, deoxycholic acid, and chenodeoxycholic acid. Bile acids are conjugated with either the amino acid glycine or taurine prior to secretion by the liver. Conjugation increases water solubility and thereby prevents passive reabsorption once secreted into the small intestine. The concentration of bile acids in the small intestine can thereby stay high enough to form micelles and solubilize lipids. Bile acid salts are reabsorbed in the terminal ileum. The primary bile acids are cholic acid and chenodeoxycholic acid. The secondary bile acids are deoxycholic acid and lithocholic acid formed by anaerobic bacteria in the intestine. Biliverdin is the bile pigment formed from the breakdown of hemoglobin. Urobilinogen is produced by bacteria in the intestines. It can be converted to stercobilin, which gives feces their brown color. It may be converted to urobilin, which is absorbed and secreted in urine. Conjugation of bile acids is required for absorption. www.ClinicalReview.com


Hepatopancreatobiliary System

415 The bile salt pool is approximately 2 grams and is recycled half a dozen times a day. Cholesterol synthesis by the liver replenishes this pool. The cholesterol content in bile is entirely derived from that created by the liver. The rate of this production is inhibited by high cholesterol intake. Cholesterol gallstones are the most common type of stone in the gallbladder.

GLUCONEOGENESIS During an overnight fast, 75gm of liver glycogen is depleted rapidly through glycogenolysis and serum glucose begins to fall. The result is a decrease in insulin and an increase in glucagon, growth hormone, catecholamines, and cortisol. The net effect is stimulation of hepatic gluconeogenesis and glycogenolysis. Gluconeogenesis relies on amino acids as the main carbon source, especially glutamine and alanine. Other sources include lactate, pyruvate, propionate, and glycerol. The primary source of substrate is from proteins, where nearly 75gm of proteins are broken down every day leading to excretion of 10gm of nitrosurea products. This protein breakdown stops about a week after starvation as the body begins to shift to ketoacid use. The fuel source to the body during periods of prolonged starvation (i.e. > 2 weeks) comes from ketone bodies. These ketone bodies are generated by the oxidation of fatty acids. The primary tissues that relay upon glucose are the kidney and liver. The brain, red blood cells, and testes also require glucose.

PHARMACOLOGY BILE ACID RESINS DRUG

INDICATIONS

Cholestyramine

Elevated LDL

MECHANISM OF ACTION Bile acid sequestration by binding and forcing excretion

COMPLICATIONS GI discomfort

NOTES Lead to an increase in the LDL receptor

Prior to administration of these lipid-lowering agents, the first line is to change the diet and lifestyle of the patient whenever possible Continued indefinitely once started (any lipid lowering agent)

HMG-COA REDUCTASE INHIBITORS DRUG

INDICATIONS

MECHANISM OF ACTION

Simvastatin

Elevated LDL, TG

Increase LDL receptor expression to decrease LDLs

COMPLICATIONS

Myositis, SLE

CONTRAINDICATIONS

Hepatic disease

NOTES Some increase in HDL, increase in LDL receptor, thought to decrease MI risk

Expensive drugs, but commonly used due to potency of effects.

FIBRATES DRUG

INDICATIONS

Gemfibrozil

Elevated TG

MECHANISM OF ACTION Increase lipoprotein lipase to decrease LDL, increase HDL, and significantly decrease TGs

COMPLICATIONS Increase LFTs, myositis, gallstones

NOTES Some increase in HDL

Â


USMLE STEP 2

416

OTHER AGENTS DRUG Ezetimibe

INDICATIONS

COMPLICATIONS

Prevents absorption of cholesterol by binding to it in the GI tract to decrease LDL

Elevated LDL

Niacin (nicotinic acid)

MECHANISM OF ACTION

Elevated LDL, TG Improve HDL

Directly inhibits VLDL synthesis to cause lipolysis, decrease TGs, and decrease LDL with a subsequent rise in HDLs

Dumping syndrome Facial flushing

OSMOTIC AGENTS DRUG

Mannitol

Lactulose

Sorbitol

INDICATIONS

MECHANISM OF ACTION

COMPLICATIONS

CONTRAINDICATIONS

Reduce ICP Oliguric renal failure Sweetener for diabetes

Hypertonic solution to increase water and salt excretion in distal tubule Also opens BBB through shrinking the endothelial cells

Rare

Few

Constipation Hepatic encephalopathy

Produces ammonia by bacterial flora to help reduce body ammonia levels Osmotic reaction in GI system to help draw fluids into the lumen and lead to bowel movements

Diarrhea Æ dehydration Hypernatremia

Those requiring low galactose diets

Artificial sweetener Constipation

Osmotic reactions in GI system

Diabetes

NOTES

Sometimes used as an adulterant for opioid abuse Also given for tropical fish poisoning

Elevated in diabetic retinopathy and neuropathy due to action of aldose B pathway converting excess sugar into sorbitol Not readily used by body in PO form

Mannitol www.ClinicalReview.com


Hepatopancreatobiliary System

417

PATHOLOGY JAUNDICE Jaundice is the deposition of excess bilirubin into the tissues of the body leading to a yellow discoloration of the eyes and skin. Elevated bilirubin also darkens the color of the urine due to high levels of excretion from the kidney. Causes of elevated bilirubin include damage to the liver, inflammation, blockade of bile drainage from the bile ducts, or hemolytic anemia.

CHOLESTASIS Cholestasis is the cessation of bile flow at some point in the hepatobiliary system. Cholestasis presents with symptoms of jaundice and darkening of the urine, clay-colored stools, osteoporosis, telangiectasia with platelet abnormalities, pruritus, splenomegaly, biliary pain, and an enlarged, palpable gallbladder.

PORTAL HYPERTENSION Portal hypertension is the elevation of blood pressure (BP) within the portal vein due to elevated back pressure from the flow of blood through the liver. Portal hypertension may present with splenomegaly, hemorrhoids, ascites, esophageal varices, and caput medusa. Collateral vessel development can occur as a measure to alleviate this elevated BP, and this can lead to the development of ascites and varices. The formation of ascites can lead to the build-up of fluid within the abdominal cavity due to fluid leaks. Ascites can present with a significantly distended abdomen and SOB. Portal hypertension is treated with the surgical formation of a transjugular intrahepatic portacaval shunt (TIPS) to bypass the liver, and also via portasystemic shunts. Decreased portal pressure can also be done with beta-blockade. The most common cause of portal hypertension is schistosomiasis infection.

VARICES Esophageal varices are the result of dilated blood vessels within the distal portion of the esophagus that can become abraded and subsequently hemorrhage. Rupture of varices can lead to significant blood loss, hematemesis, and anemia, and these large bleeds can be fatal in over half of all patients. BP is maintained during acute variceal hemorrhage through normal saline and transfusions, fresh frozen plasma (FFP) to replace clotting factors, vasopressin or somatostatin to decrease mesenteric blood flow, beta blockers to decrease blood flow, sclerotherapy, and band ligation of bleeding vessels.

HEPATIC ENCEPHALOPATHY Encephalopathy from liver disease is commonly the result of elevated levels of ammonia leading to central nervous system (CNS) damage. The elevated amino acid content of the blood also leads to an increase in GABA levels and endogenous benzodiazepines. Hepatic encephalopathy is often heralded by signs of impaired consciousness such as drowsiness, confusion, disorientation, or coma, changes in personality, behavior, mood or judgment, changes in speech, asterixis, fetor hepaticus, and cerebral edema. Hepatic encephalopathy is worsened by acute GI bleeds, increased protein intake, and active infection. The differential diagnosis of hepatic encephalopathy includes alcohol intoxication, delirium tremens, dementia, infection, and various systemic disorders.

HEPATIC FAILURE Liver failure occurs when the liver is no longer able to carry out its necessary functions due to excessive damage. It presents with jaundice, easy bruising, ascites, encephalopathy, malaise, weakness and fatigue, nausea and vomiting, and loss of appetite.

Â


418

USMLE STEP 2

SPONTANEOUS BACTERIAL PERITONITIS Spontaneous bacterial peritonitis occurs in a subset of patients with longstanding ascites, and is due to infection by enteric gram negative bacteria from the intestinal flora. Spontaneous bacterial peritonitis presents with fever and chills, rebound abdominal tenderness, and sepsis. It is diagnosed with gram stain, culture, and sensitivity following paracentesis of the ascites. The presence of over 250 polymorphonuclear lymphocytes (PMNs) is diagnostic. Spontaneous bacterial peritonitis is treated with cefuroxime to cover the intestinal flora and electrolyte replacement for the ascites.

HEPATORENAL SYNDROME Hepatorenal syndrome is the development of acute renal failure with worsening oliguria, azotemia, and sodium retention. It is diagnosed by very low urine sodium. It is especially common in later stages of severe hepatic failure. There is no known treatment other than supportive therapy and palliative care.

DUPUYTREN CONTRACTURE Dupuytren contracture is the result of contractions within the palmar fascia leading to flexion of the fourth digit. It is due to the proliferation of myofibroblasts leading to replacement of collagen II by collagen III. Dupuytren contracture may be a result of microvascular ischemic injury. TABLE 477 PRESENTATION OF HEPATOBILIARY DISEASE Presentation of Hepatobiliary Disease Jaundice

Deposition of bilirubin leading to yellow discoloration of eyes and skin.

Cholestasis

Stoppage of the flow of bile, jaundice, dark urine, clay stools.

Portal hypertension

High BP within portal vein, ascites, varices.

Varices

Dilated blood vessels.

Hepatic encephalopathy

Elevated levels of ammonia leading to CNS damage, impaired consciousness, and personality, mood, and/or behavior changes.

Hepatic failure

Jaundice, easy bruising, ascites, encephalopathy, malaise, nausea, fatigue.

Hepatorenal syndrome

Acute renal failure, low sodium in urine, late stages of hepatic failure.

Bacterial peritonitis

Hepatic patients with ascites, fever, chills, sepsis, many PMNs, + bacterial cultures.

Dupuytren contracture

Contraction of the palmar fascia leading to flexion of the fourth digit.

LIVER FUNCTION TESTS (LFTS) There are over a dozen tests used to gauge the function of various aspects of the liver. The most relevant tests include measurements of bilirubin, albumin, alkaline phosphatase, transaminases, prothrombin time (PT), alanine transaminase (ALT), aspartate transaminase (AST), gamma-glutamyl transpeptidase (GGT), lactate dehydrogenase (LDH), 5’-nucleotidase, alphafetoprotein (AFP), and antibodies to mitochondria (AMA). Serum bilirubin tests measure the amount of bilirubin excreted in the blood, which indicates a pathology that includes normal excretion of bile through the bile ducts. Serum albumin tests measure the ability of the liver to form proteins. Alkaline phosphatase is an indicator for biliary obstruction, but may also be elevated with liver or bone disease. Transaminases are enzymes released from damaged liver cells, and are either ALTs or ASTs. Elevated ALT indicates acute liver cell damage, and is elevated especially in hepatitis. Elevated AST is less specific to the liver, as it may also www.ClinicalReview.com


Hepatopancreatobiliary System

419

indicate damage to the heart, skeletal muscle, kidney, pancreas, and RBCs. Increases in PT indicate a coagulopathy due to the failure of the liver to manufacture proteins necessary for proper blood clotting (including vitamin K). Elevated GGT is typically increased in alcohol liver disease, but may also be due to damage to the pancreas or biliary tract; elevations in GGT are sensitive for recent alcohol binging. Elevated LDH indicates a derangement in metabolism due to liver damage. The 5’-nucleotidase test is specific to the liver and is especially elevated in cholestasis. The AFP test is used to monitor the benefits of therapy for primary liver cancers (but may also be elevated in various embryonic cell tumors). Finally, the AMA test indicates primary biliary cirrhosis (PBC), chronic hepatitis, and various other autoimmune diseases.

DIAGNOSTIC STUDIES A number of tests exist to detect the presence of gallstones and related biliary disease. A series of X-rays taken after swallowing a radiopaque dye permits a cholecystography to be done to map out the bile ducts and gallbladder and potentially detect obstruction. Computed tomography (CT) is often the procedure of choice to provide a more detailed view of the abdomen and detect disease. A more detailed picture can sometimes be produced through magnetic resonance imaging (MRI), but at a significantly greater cost. Ultrasound is a relatively inexpensive test best suited for detecting gallstones but also visualizing various other internal organs. Plain films are also often used. Scintigraphy can be used to image the hepatobiliary tree and the duodenum through the use of radiolabeled compounds. A hepatobiliary iminodiacetic acid (HIDA) scan is the gold standard for the diagnosis of cholecystitis; this test is done by IV injection of radiolabeled iminodiacetic acid and observing the pattern of uptake within the hepatobiliary tree.

OUTPATIENT PROCEDURES Endoscopic retrograde cholangiopancreatography (ERCP) is used to directly visualize the luminal surfaces of the bile ducts, gallbladder, liver, and pancreas. Esophagogastroduodenoscopy (EGD), also known as endoscopy, is used to directly visualize the luminal surfaces of the upper gastrointestinal tract; this instrument permits the clinician to perform basic outpatient procedures as well. Cholangiography through a percutaneous transhepatic (PTC) procedure permits injection of a radiopaque dye into the liver and visualization of the bile ducts through x-ray. Finally, a liver biopsy can be done to collect a specimen for histology.

SERUM-ASCITES ALBUMIN GRADIENT (SAAG) The SAAG is calculated by comparing the albumin concentration in the serum to that of the ascites fluid. If the difference between the serum and the ascites is greater than 1.1 and portal hypertension is present, it indicates the underlying etiology to likely be due to liver disease such as hepatitis, cirrhosis, liver failure, or HCC; it may alternatively be due to congestion caused by hepatic failure, heart failure, constrictive pericarditis, tricuspid insufficiency, or Budd-Chiari syndrome. A SAAG less than 1.1 with hypoalbuminemia typically indicates that nephrotic syndrome, severe malabsorption with protein loss, or malnutrition with anasarca are the cause. Other conditions that may lead to a SAAG less than 1.1 include ascites of pancreatic, bile, stomach, kidney, bladder, or ovarian origin. A diseased peritoneum can also present with a SAAG less than 1.1 with causes including infection, malignancy, and rarer conditions such as familial Mediterranean fever (FMF), vasculitis, granulomatous peritonitis, and eosinophilic peritonitis.

Â


420

USMLE STEP 2

TABLE 478 DIAGNOSTIC STUDIES Diagnostic Studies LFT

Bilirubin, albumin, alkaline phosphatase, transaminases, PT, ALT, AST, GGT, LDH, AFP, AMA.

Cholecystography

X-rays to map out bile ducts and gall bladder.

CT/ MRI

Check out abdomen.

HIDA

Diagnosis of cholecystitis.

ERCP

Used to visualize luminal surfaces of ducts, gall bladder, liver, pancreas.

EGD

Used to visualize upper GI.

Cholangiography

Visualize bile ducts through x-ray.

SAAG

Compares albumin in serum to ascites fluid albumin.

HEPATIC DISEASE CIRRHOSIS ETIOLOGY Cirrhosis is the development of repeated liver damage leading to cycles of fibrosis, necrosis, regeneration, and eventually, HCC. Causes of cirrhosis include alcohol, viral infection, primary biliary cirrhosis (PBC), Wilson disease, alpha-1-antitrypsin deficiency, and hemochromatosis. The most common cause of cirrhosis in the United States is alcohol abuse leading to fatty liver, and eventually hepatitis and cirrhosis with continued insult.

PRESENTATION Cirrhosis presents with many of the signs discussed above for liver damage, including jaundice, nausea and vomiting, hepatomegaly, loss of appetite, angiomas, ascites, encephalopathy, palmar erythema, Dupuytren contracture, and portal hypertension. Overall, 15% of alcoholics will develop cirrhosis. There is a 10% mortality from hepatitis, and half of all cases are due to progression to cirrhosis and the other half to fibrosis.

DIAGNOSIS Diagnosis of cirrhosis is made with abnormal LFTs including an elevated AST to ALT ratio (viral disorders raise the ALT to AST ratio), elevated direct and indirect bilirubin, elevated GGT, prolonged PT, low albumin, and anemia.

TREATMENT Cirrhosis is treated by avoiding risk factors that can further exacerbate liver injury. Failure to do so, and continued use of alcohol, will likely lead to fulminant hepatic failure or HCC. Unlike some other hepatobiliary conditions, a high protein diet is recommended with vitamin supplements to permit the liver to form the necessary proteins required for normal body function. Medications that can further damage the liver are avoided, including acetaminophen and isoniazid (INH). Inflammatory changes can often be halted with the use of colchicine and glucocorticoids. Other complications of cirrhosis such as ascites should be treated as necessary. www.ClinicalReview.com


Hepatopancreatobiliary System

421

PROGNOSIS The prognosis of cirrhosis is made by the Child-Pugh score, which examines the levels of bilirubin, albumin, PT, the presence of hepatic encephalopathy, and the grade of ascites. The score for each category is summed, and a Child class assigned based on the range of the scores. Child class A is a score between 5 and 6, class B with a score between 7 and 9, and class C with a score greater than 9. With serum bilirubin less than 2, a score of 1 is given; bilirubin greater than 3 has a score of 3. Serum albumin more than 3.5 has a score of 1; less than 2.8 has a score of 3. PT of 4 or less is given a score of 1; PT more than 6 has a score of 3. No hepatic encephalopathy has a score of 1, while severe symptoms have a score of 3. No ascites is given a score of 1, while severe ascites has a score of 3. A Child score of A has a 15-20 year survival; Child class C survival has a survival of only a few years. TABLE 479 CIRRHOSIS Cirrhosis Pathophysiology

Repeated liver damage leading to fibrosis, necrosis, regeneration, and HCC.

Cause

Alcohol, viral infection, PBC, Wilson disease, hemochromatosis.

Presentation

Jaundice, nausea/vomiting, hepatomegaly, ascites, encephalopathy, portal hypertension, Dupuytren contracture, palmar erythema.

Diagnosis

Abnormal LFTs, elevated AST to ALT ratio, high bilirubin and GGT, long PT.

Treatment

Avoiding risk factors, high protein diet, vitamins.

Prognosis

Made by Child-Pugh score (see above).

PRIMARY BILIARY CIRRHOSIS ETIOLOGY AND EPIDEMIOLOGY PBC is an autoimmune disorder that causes destruction of the intrahepatic bile ducts. It is more common in women, and is predicted by the presence of AMA and a demonstrable autoimmune response to the epithelium of the biliary tree. PBC leads to the accumulation of waste products commonly dissipated with bile secretion, namely, cholesterol, bile acids, and bilirubin.

PRESENTATION PBC presents with symptoms similar to other liver diseases, including jaundice, fatigue and weakness and pruritus. In addition, PBC presents with xanthomas from the elevated cholesterol. There is typically an additional autoimmune disorder present with PBC, such as rheumatoid arthritis (RA). Nearly 1/3 of patients are asymptomatic, and the most common initial complaint is fatigue.

DIAGNOSIS AND TREATMENT Diagnosis of PBC is made by elevated alkaline phosophatase and GGT. AST and ALT are typically normal. There is also a positive AMA. Ultrasound is typically unremarkable. Hyperlipidemia is another feature of PBC. Treatment largely consists of surgical intervention due to the failure of medical management. Liver transplantation is necessary.

Â


422

USMLE STEP 2

TABLE 480 PRIMARY BILIARY CIRRHOSIS (PBC) Primary Biliary Cirrhosis (PBC) Epidemiology

Autoimmune disorder which destroy intrahepatic bile ducts. More common in women.

Presentation

Jaundice, fatigue, weakness, pruritus, Xanthomas, RA.

Diagnosis

Elevated alkaline phosphatase and GGT. AST and ALT usually normal. Positive AMA.

Treatment

Surgery, liver transplant.

GILBERT DISEASE BACKGROUND Hyperbilirubinemia is divided into three categories: 1) defective bile outflow due to intrahepatic or extrahepatic bile obstruction (discussed above with examples such as cholelithiasis), 2) impaired handling of bilirubin due to liver disease (i.e. cirrhosis, hepatitis), and 3) excessive bilirubin production (such as with hemolytic anemia). The inherited syndromes subsequently discussed, including Gilbert disease, Crigler-Najjar syndrome, Dubin-Johnson syndrome, and Rotor disease are disorders leading to defects in bilirubin processing. TABLE 481 INHERITED HYPERBILIRUBINEMIA OVERVIEW Inherited Hyperbilirubinemia Overview Defective bile flow.

Due to intra- or extra- hepatic bile obstruction.

Impaired handling of bilirubin.

Cirrhosis, hepatitis.

Excessive bilirubin.

Hemolytic anemia.

PATHOPHYSIOLOGY AND MANAGEMENT Gilbert disease is an autosomal disease leading to excessive unconjugated hyperbilirubinemia due to a defect in diphosphate glucuronosyltransferase (UGT). Gilbert disease has a significant prevalence and affects up to 1 in 10 persons. Jaundice is the most common presentation, and becomes clinical only in periods of high stress, infection, alcohol abuse, and following surgery. Bilirubin rarely exceeds 5 mg / dL with this disorder. There is no treatment.

CRIGLER-NAJJAR DISEASE PATHOPHYSIOLOGY AND MANAGEMENT Crigler-Najjar syndrome is a rarer autosomal recessive disorder than Gilbert disease. This syndrome comes in two types: the first type has high bilirubin levels with neonatal jaundice, kernicterus, and serious CNS deficits; the second type has lower bilirubin levels with survival into adulthood. Unconjugated bilirubin in type I Crigler-Najjar syndrome may approach 50 mg / dL. Inability to convert bilirubin to a polar derivative in a process known as glucuronidation is at fault, and involves a serious defect with diphosphate glycosyltransferase (UGT). Crigler-Najjar requires prompt medical attention and treatment to avoid serious psychomotor defects leading to death.

www.ClinicalReview.com


Hepatopancreatobiliary System

423

DUBIN-JOHNSON DISEASE PATHOPHYSIOLOGY AND MANAGEMENT Dubin-Johnson syndrome is a familial conjugated hyperbilirubinemia caused by an autosomal recessive defect leading to impaired transport of conjugated bilirubin. It is a relatively benign disorder that leads to jaundice without pruritus. The presentation may be exacerbated by periods of stress such as pregnancy, infection, and oral contraceptive medications. As the conjugated bilirubin remains in the hepatocytes, these cells appear intensely dark on histologic examination. Hepatobiliary scans of the liver are unique in this disorder in that they are intensely stained for a long period of time with little visualization of the gallbladder. There is no treatment.

ROTOR DISEASE PATHOPHYSIOLOGY AND MANAGEMENT Rotor syndrome is similar to Dubin-Johnson syndrome with a similar presentation. It is also an autosomal recessive condition leading to defective bilirubin excretion. Unlike Dubin-Johnson syndrome, the gallbladder is usually visualized on hepatobiliary scans. This is also a relatively benign condition with no treatment.

HEMOCHROMATOSIS PATHOPHYSIOLOGY AND MANAGEMENT Hemochromatosis is the result of an autosomal recessive defect leading to excessive iron absorption. This large iron load precipitates in various tissues leading to bronze discoloration of the skin, hepatomegaly, diabetes, restrictive heart failure, arthropathy, and generalized organ failure. The bronze discoloration in conjunction with glucose control problems leads this disease to have a secondary name of bronze diabetes. Diagnosis is made by high iron levels and iron-processing molecules such as ferritin and transferrin. Liver biopsy confirms the clinical suspicion. Treatment is to bleed the patient to remove the extraneous iron; deferoxamine is also occasionally used to chelate the excess iron.

WILSON DISEASE PATHOPHYSIOLOGY AND MANAGEMENT Wilson disease is an autosomal recessive disorder leading to excess accumulation of copper due to poor removal as waste from the liver. This copper precipitates in the cornea to form pathognomonic Kayser-Fleischer rings, in the brain to cause psychiatric disturbances such as schizophrenia or psychosis, and in the liver to cause cirrhosis. Diagnosis is made by elevated copper in the serum and low levels of ceruloplasmin. Liver biopsy is typically done to clinch the diagnosis. Copper chelation therapy with penicillamine is typically successful, but liver transplantation may be needed in some cases.

ALPHA-1-ANTITRYPSIN DEFICIENCY PATHOPHYSIOLOGY AND MANAGEMENT

Â


USMLE STEP 2 424 Alpha-1-antitrypsin deficiency is an autosomal recessive disorder that presents with defects in this enzyme leading to emphysema and cirrhosis. It is a relatively common reason for liver disease in infants. The presentation is typically asymptomatic other than the emphysema, and diagnosis is made by demonstrating lack of the enzyme. TABLE 482 INHERITED SYNDROMES Inherited syndromes Gilbert disease

Autosomal dominant. Excessive unconjugated hyperbilirubinemia due to a defect in UGT. Presents with jaundice. Clinical in periods of high stress, infection, alcohol abuse, postsurgery. No treatment.

Crigler-Najjar disease

Autosomal recessive. Type I has high bilirubin with neonatal jaundice & serious CNS deficits. Type II lower bilirubin levels with survival into adulthood.

Dubin-Johnson disease

Autosomal recessive. Impaired transport of conjugated bilirubin. Jaundice without pruritus. Hepatobiliary scans unique.

Rotor disease

Similar to Dubin-Johnson. Unlike in that gall bladder is visualized in scans. No treatment.

Wilson disease

Autosomal recessive. Excess accumulation of copper. Causes Kayser-Fleischer rings, cirrhosis, and psychiatric disturbances. Diagnosed by elevated serum copper and low ceruloplasmin, liver biopsy. Treat by chelation therapy.

Hemochromatosis

Autosomal recessive. Excessive iron absorption. Bronze discoloration of skin, hepatomegaly, diabetes, restrictive heart failure, arthropathy, organ failure. Diagnosed by high iron levels and high ferritin and transferrin, biopsy. Treatment is to bleed patients and iron chelation therapy.

Alpha-1-antitrypsin deficiency

Autosomal recessive. Defect in this enzyme leading to emphysema and cirrhosis. Asymptomatic, tested by lack of this enzyme.

HEPATITIS VIRAL HEPATITIS ETIOLOGY Viral hepatitis is an infectious process due to hepatitis A (HAV), B (HBV), C (HCV), D (HDV), E (HEV), or G (HGV). Longstanding viral hepatitis leads to symptoms of liver damage, including jaundice and dark urine, right upper quadrant (RUQ) pain, weakness and fatigue, anorexia, nausea and vomiting, low-grade fever, and general malaise. Generally, viral hepatitis presents with elevations in ALT. The increases are significant – viral hepatitis is one of three serious causes of transaminases increasing above 1,000; the other two causes are acetaminophen toxicity and septic shock leading to fulminant liver failure. Fulminant liver failure can also occur with infection by any of the hepatitis viruses except HGV. Cirrhosis is most commonly associated with chronic hepatitis, which can occur with HBV and HCV. Finally, all of the causes of viral hepatitis are due to RNA viruses except HBV, which is a DNA virus. The specific hepatitis viruses are discussed below. TABLE 483 VIRAL HEPATITIS Viral Hepatitis Cause

HAV, HBV, HCV, HDV, HEV, HGV

Presentation

Jaundice, dark urine, RUQ pain, weakness, fatigue, anorexia, nausea/vomiting, low grade fever, general malaise. Typically has a high ALT

www.ClinicalReview.com


Hepatopancreatobiliary System

425

HEPATITIS A VIRUS PATHOPHYSIOLOGY HAV is a single-stranded RNA virus in the hepatovirus family. It is predominantly spread by the fecal-oral route and is associated with poor hygiene and contaminated food. Contaminated shellfish in particular have been implicated. HAV is not transmitted vertically from mother to neonate, and there is no chronic carrier state. HAV has a 30 day incubation. A minority of HAV infections lead to fulminant hepatic failure.

DIAGNOSIS The assessment of HAV infection is made by serology that is positive for IgM antibodies to HAV, which indicates recent exposure, and IgG antibodies to HAV, which indicates exposure in the past and current immunity. In general, the presence of IgM antibodies indicates acute exposure as the immunoglobulin has not yet had a chance to shift to IgG expression.

TREATMENT Symptomatic management is often the treatment of choice for HAV infection. This is due to HAV often being a self-limited disorder with little chance of progression to a more serious disease. HAV prophylaxis is given to people who are planning to travel to developing nations by way of a vaccine that takes three weeks to take effect. Immunoglobulins against HAV can also be administered in susceptible patients if given within a couple of weeks of exposure. The vaccine and immunoglobulin therapy are typically administered to patients with chronic liver disease to avoid further insult and injury. TABLE 484 HEPATITIS A (HAV) Hepatitis A (HAV) Pathophysiology

Single-stranded RNA. Spread by fecal-oral route, contaminated food.

Diagnosis

Positive for IgM and IgG to HAV in serum.

Treatment

Symptomatic management. Prophylaxis available.

HEPATITIS B VIRUS PATHOPHYSIOLOGY HBV is a DNA virus in the hepadnavirus family. HBV is spread predominantly through sex, blood products, and saliva. It is found in all body fluids. HBV has a 1/10 chance of progressing to chronic hepatitis in adults, and a 90% risk in neonates. Chronic infection significantly increases the risk of hepatocellular carcinoma (HCC) due to repeated cycles of damage and healing. The risk of fulminant liver failure is approximately 1 in 100 patients, and half of all cases of fulminant hepatitis are due to HBV. HBV has a two-six month incubation period.

DIAGNOSIS A number of markers for HBV infection are used to determine the infectivity and prognosis of the disease. Hepatitis B core antigen (HBcAg) is expressed in infected hepatocytes, but not detectable in the serum. HBcAg is a marker for early infection. IgM antibodies to hepatitis B core (HBcAb IgM) indicates the onset of the window period, and is the time between disappearance of hepatitis B surface antigen (HBsAg) and appearance of antibodies to HBs (HBsAb) in the serum. HBcAb IgM is detectable during

Â


USMLE STEP 2 426 the acute phase of infection, which typically occurs 1-6 months after infection. HBsAg is the antigen present on the protein coat of the virus and is a marker of active hepatitis and a carrier state. Ongoing viral replication with a high chance of infectivity is indicated by the hepatitis B “e” antigen. Both HBsAg and HBeAg are expressed between 1-6 months after initial infection. A significantly decreased risk of infection is indicated by the presence of antibodies to HBe (HBeAb), which can take several months to years and are present in the acute phase of resolution. Immunity is signified by antibodies to HBs (HBsAb). Signs of infection in the distant past are indicated by IgG antibodies to HBc (HBcAb IgG), and are detectable for the longest period of time. Essentially, if HBsAg is positive, infection is ongoing. If HBcAb IgM is present, the infection is in the window period. The presence of only HBsAb indicates immunity via vaccination.

TREATMENT Treatment of HBV is instituted with immunoglobulins to HBV (HBIG), use of alpha-interferon or lamivudine for chronic infections, and liver transplantation in the case of fulminant hepatic failure. Prophylaxis to HBV is the best way to avoid infection, and it is done with a series of three HBV IgG injections and standard precautions to body fluids should be taken. TABLE 485 HEPATITIS B (HBV) Hepatitis B (HBV) Pathophysiology

DNA virus spread through sex, blood, saliva.

Diagnosis

HbcAG in hepatocytes but not in serum IgM to HBV. Also HBsAg, HBcAb, HBeAg, HBeAb, HBsAb at various stages (see text).

Treatment

Immunoglobulins to HBV, lamivudine, alpha-interferon, transplant.

HEPATITIS C VIRUS PATHOPHYSIOLOGY HCV is a single-stranded RNA virus in the flavivirus family, and is spread in a manner similar to HBV. HCV is now the most common cause of viral hepatitis in the United States due to precautions against blood-borne transmission of HBV instituted earlier than HCV precautions. HCV transmission is especially common in IV drug users and in prisons. Vertical transmission is possible but minimal (5%), as is sexual transmission (5%) and via needles (5%). Like HBV, the incubation period is typically on the order of 1-6 months. About ¾ of HCV infections become chronic and there is up to a 5% risk of HCC. HCV infection can present with symptoms of hepatitis and also polyarteritis nodosa (PAN) and cryoglobulinemias. Only ¼ of patients are symptomatic.

DIAGNOSIS Diagnosis is made by positive serum titers for antibodies to HCV (HCVAb). HCVAb indicates chronic infection. Repeated infections over time are possible due to poor antibody development.

TREATMENT HCV is treated with alpha-interferon and ribavirin with a 50% chance of remission. Patients who do not respond to this therapy can be given a course of amantadine or rimantadine. As with HBV, the best treatment is prevention.

www.ClinicalReview.com


Hepatopancreatobiliary System

427

TABLE 486 HEPATITIS C VIRUS (HCV) Hepatitis C Virus (HCV) Pathophysiology

Single-stranded RNA. Spread through sex, saliva, blood.

Presentation

Symptoms of hepatitis, PAN and cryoglobulinemias.

Diagnosis

Positive serum levels of HCVAb.

Treatment

Alpha-interferon and ribavirin, amantidine, rimantadine.

HEPATITIS D VIRUS PATHOPHYSIOLOGY AND DIAGNOSIS HDV is the delta agent, and requires the presence of HBV in order to be infective. Elements from HBV are required in order for HDV to propagate and retain its infectivity. In patients with pre-existing HBV, a super-infection may occur with HDV; both HBV and HDV can be simultaneously transmitted to cause a co-infection. HDV infection dramatically increases the risk of liver damage and cirrhosis. HDV is an RNA virus. Superinfection carries the greatest risk of fulminant liver failure. Treatment is to eradicate the HBV infection; the presence of HBsAb indicates immunity to HBV and HDV. TABLE 487 HEPATITIS D VIRUS (HDV) Hepatitis D Virus (HDV) Pathophysiology

RNA virus. Requires HBV to be infective.

Treatment

Treat the concurrent HBV infection.

HEPATITIS E VIRUS PATHOPHYSIOLOGY AND DIAGNOSIS HEV is a RNA virus with a 1-2 month incubation period. Like HAV, it is transmitted through the fecal-oral route. It is most common in Asia and Africa, and carries a relatively high rate of fulminant liver failure. Pregnant patients are especially susceptible for unknown reasons, and the risk of fulminant liver failure and death is especially worrisome in this population. There is no carrier state with HEV, and no marker for diagnosis.

TREATMENT Treatment of HEV is primarily supportive therapy and avoidance with good hygienic practices. TABLE 488 HEPATITIS E VIRUS (HEV) Hepatitis E Virus (HEV) Pathophysiology

RNA virus. Transmitted by fecal-oral route. Most common in Asia and Africa.

Diagnosis

No marker for diagnosis.

Treatment

Supportive therapy.

Â


428

USMLE STEP 2

HEPATITIS G VIRUS PATHOPHYSIOLOGY HGV is mentioned for completeness. No active infection in humans has yet been demonstrated with this virus. It is otherwise similar to HCV in structure and form. TABLE 489 HEPATITIS G VIRUS (HGV) Hepatitis G Virus (HGV) Epidemiology

No active infections found in humans. Similar to HCV.

PANCREATIC DISORDERS INTRODUCTION ANATOMY The pancreas is the abdominal organ located dorsal to the stomach and extends between the right and left upper quadrants. The widest part of the pancreas is known as the head, and is located within the first section of the intestines – the duodenum. The left side tapers and travels somewhat superiorly and is known as the body of the pancreas. The pancreas terminates as the tail near the spleen. The pancreas is composed of exocrine tissue that secretes digestive enzymes into the small intestine and endocrine tissue that secretes hormones into the bloodstream.

PHYSIOLOGY The exocrine pancreas serves a distinct function from the endocrine pancreas, even though the two tissues are juxtaposed within the same organ. The exocrine pancreas releases a number of enzymes into the pancreatic duct in an inactive form. These digestive enzymes are activated by the acidic environment within the duodenum and break down carbohydrates, proteins, and fats. The exocrine pancreas also secretes bicarbonate to neutralize the hydrochloric acid produced by the parietal cells of the stomach. Nearly 3L of bicarbonate and enzyme-rich fluid are produced every day by the pancreas, and the release of these compounds is facilitated by cholecystokinin (CCK), secretin, and bile salts. The endocrine pancreas secretes insulin and glucagon to regulate the titer of sugar within the bloodstream, and also somatostatin to control the function of the intestines and modulate the concentration of insulin and glucagon.

ACUTE PANCREATITIS ETIOLOGY AND PATHOPHYSIOLOGY Acute pancreatitis is the result of direct damage from prematurely activated pancreatic enzymes digesting the parenchyma of the organ. Severe disease can lead to systemic inflammatory response syndrome (SIRS) and subsequently progress to severe septic shock, multi-organ system failure, and adult respiratory distress syndrome (ARDS). Causes of acute pancreatitis include perforation of a peptic ulcer, alcoholism, neoplastic disorders, cholelithiasis, end stage renal disease (ESRD), endoscopic retrograde cholangiopancreatography (ERCP), malnutrition leading to anorexia, direct trauma to the pancreas, certain infections, use of drugs that lead to toxic byproducts, burn injuries, surgery, and scorpion bites. The most common causes are alcohol abuse and gallstones. Metabolic conditions that predispose to pancreatitis include very high hypertriglyceridemia, and hypercalcemia. www.ClinicalReview.com


Hepatopancreatobiliary System

429

Medications that can lead to pancreatitis include oral hypoglycemic agents, thiazide diuretics, furosemide, pentamidine, TMPSMX, and DDI.

PRESENTATION Acute pancreatitis presents as severe epigastric pain that radiates to the back. The pain worsens after consumption of food and may improve if the patient leans forward. Nausea and vomiting are common, as is fever, tachypnea (to avoid deep breaths that may lead to further irritation of the already inflamed pancreas), a positive Cullen sign, and a positive Turner sign. Cullen sign is the presence of a bluish hue to the umbilicus due to hemorrhaging within the peritoneum – it is most commonly found in necrotizing pancreatitis. Turner sign is a bluish discoloration in the flanks due to hemoglobin deposition and decomposition in the soft tissue. Abdominal exam typically reveals tenderness and involuntary guarding, but no rebound tenderness.

DIAGNOSIS Acute pancreatitis is diagnosed primarily by blood tests that indicate elevations in amylase and lipase. Neither of these two enzymes are 100% specific to the pancreas, and they may be elevated in a number of other conditions. Perforation of the gastrointestinal system may lead to elevations in amylase; this enzyme is also found in the salivary glands, small intestine, reproductive organs such as the testes and ovaries, and in striated muscle. Renal disease and hypertriglyceridemia also lead to elevations in amylase. Lipase is found throughout the gastrointestinal system and the liver. Notwithstanding, very high elevations in both amylase and lipase are often pathognomonic for acute pancreatitis. Overall, the most specific test for pancreatitis is lipase levels, while the most sensitive test is amylase titers.

DIAGNOSTIC IMAGING TESTS The diagnosis of acute pancreatitis can be confirmed through various imaging tests. As acute pancreatitis progresses, various anatomical changes can occur including the development of a pseudocyst, abscess, phlegmon, splenic vein thrombosis, and clear evidence of pancreatic necrosis. These changes are detectable on a CT scan. The earliest changes that can be detected are the development of a phlegmon, which is typically obvious within a couple of days. Pancreatic necrosis and pseudocyst formation are typically apparent within a couple of weeks. Abscess formation is clear within a month or two. Plain films are also of some value in diagnosis. Chest X-rays (CXR) are taken to determine pancreatitis-induced pleural effusions or elevations in part of the diaphragm. Abdominal X-rays (AXR) are taken to detect calcification within the pancreas, which is actually a better predictor of chronic pancreatitis instead of acute pancreatitis. Ultrasound is used to identify gallstones as the causative agent of pancreatitis, and once identified; they may be removed with ERCP.

TREATMENT Treatment of acute pancreatitis is to cease the progression of reversible causes such as alcoholism and gallstones (treat with ERCP). Complications of acute pancreatitis such as pseudocyst or abscess formation may be drained with CT-guided aspiration. Most cases are self-limited, and so the standard of care is to provide supportive therapy, IV fluids, pain control with meperidine, nothing by mouth (NPO), and bowel rest. A nasogastric tube is used to decompress the GI system. Morphine is not used with pancreatitis due to the risk of sphincter of Oddi spasms and needless worsening of the pain. Surgical debridement is used in cases refractory to standard management.

PROGNOSIS The prognosis of acute pancreatitis is gauged by Ranson criteria. Mortality depends on the number of risk factors present. The risk factors are divided into those present on admission and those that develop after two days. Risk factors that increase the risk

Â


USMLE STEP 2 430 of mortality and may present on admission include age over 55, elevations in blood sugar over 200, elevated white blood cell (WBC) counts over 16,000, elevated aspartate transaminase (AST) over 250, and elevated lactate dehydrogenase (LDH) over 350. Risk factors present after two days include a decrease of more than 10% in the hematocrit (HCT), increase in BUN over 5, calcium less than 8, PO2 less than 60 mmHg, a base deficit greater than 4, and a fluid deficit of more than 6 L. The risk of mortality is low if less than three risk factors are present. Three or 4 risk factors carries a 1/6 risk of death. Five or 6 risk factors increase the mortality to 2/5, and more than 6 risk factors have a mortality that approaches 100%. TABLE 490 ACUTE PANCREATITIS Acute Pancreatitis Etiology

Perforation form PUD, alcoholism, cancer, cholelithiasis, ESRD, ERCP, direct trauma, infections, toxic drugs, burn injuries, surgery.

Presentation

Severe epigastric pain radiating to back. Pain is worse after eating. Nausea/vomiting, fever, tachypnea, Cullen and Turner signs, tender abdomen.

Diagnosis

Blood tests (elevated amylase & lipase), CT scan, CXR, AXR, ultrasound.

Treatment

Treatment for alcoholism, ERCP, surgical debridement.

Prognosis

Ranson criteria scores: Mortality low if under 3 factors; 1/6, 3-4 risk factors; 2/5, 5-6 factors, more than 6, 100%.

CHRONIC PANCREATITIS ETIOLOGY AND PATHOPHYSIOLOGY Repeated episodes of acute pancreatitis can lead to chronic injury and scarring to the pancreas, thereby inducing the development of chronic pancreatitis. The most common cause of chronic pancreatitis is continued abuse of alcohol. Approximately 他 of patients with chronic pancreatitis have their disease from alcohol abuse, the remainder tend to be generally idiopathic in nature. Presentation of chronic pancreatitis in younger patients may be a result of cystic fibrosis.

PRESENTATION Chronic pancreatitis presents with signs and symptoms similar to acute pancreatitis, including epigastric pain that radiates to the back, diarrhea with steatorrhea, signs and symptoms of malabsorption including vitamin deficiencies, elevated blood sugars and polyuria due to damage to the endocrine pancreas and subsequent diabetes, and chronic liver disease.

DIAGNOSIS Diagnosis of chronic pancreatitis is made by history and blood tests that indicate normal amylase and lipase levels but decreased pancreatic enzyme levels such as trypsin deficiencies. Calcifications are seen on abdominal plain films, which strongly raise the clinical suspicion for this disorder. Finally, the secretin stimulation test is used to stimulate bicarbonate production by the pancreas.

TREATMENT Treatment for chronic pancreatitis is primarily supportive with modification of risk factors and avoidance of exacerbating activities. Replacement of pancreatic enzymes is often required in addition to dietary changes such as decreased fat intake, vitamin supplements, and use of spontaneously absorbed medium-chain fatty acids. Again, use meperidine instead of morphine

www.ClinicalReview.com


Hepatopancreatobiliary System

431 for pain control. Diabetes induced by chronic pancreatitis should be controlled carefully due to the decrease in both insulin and glucagon. TABLE 491 CHRONIC PANCREATITIS Chronic Pancreatitis Cause

Alcohol abuse (3/4), idiopathic (1/4), cystic fibrosis.

Presentation

Similar to acute pancreatitis.

Diagnosis

History, blood tests (amylase, lipase levels), calcification on plain films, secretin stimulation test.

Treatment

Primarily supportive, modification of risk factors, replacement of enzymes.

SPLEEN SPLENECTOMY Splenectomy can lead to overwhelming sepsis (OPSI) especially with encapsulated organisms due to inability to opsonize organisms and permit complement-mediated destruction. Splenectomy in patients with ITP is advisable as a means to reduce the destruction of platelets due to the presence of antibodies against platelets and their subsequent destruction by macrophages in the spleen.

BILIARY DISEASE CHOLELITHIASIS PATHOPHYSIOLOGY Cholelithiasis is the formation of gallstones that may lead to a cessation in bile transport. Cholesterol stones are the most common and are typically located in the gallbladder. Stones in the common bile duct tend to be pigmented stones or calcium bilirubinate stones. Pigmented stones are more common in liver disease, alcohol abuse, and hemolytic anemia. It is several times more common in women, and has a number of risk factors including patients who are in their 40s, fertile, female, overweight, currently fasting, rapid weight loss, Crohn disease, hypertriglyceridemia, a history of cystic fibrosis, a familial tendency, sickle cell disease, diabetes, or use of oral contraceptive drugs (OCDs). These risk factors generally tend to cause an increase in cholesterol content in bile, decreased secretion of bile by the liver, and lead to pooling of cholesterol-laden gallstones within the gallbladder. The sludge that is created has difficulty passing through the ducts and may become lodged within the lumen, leading to obstruction and the signs and symptoms of cholelithiasis.

PRESENTATION Cholelithiasis presents with colicky, RUQ pain that lasts several hours at a time. The pain is severe and made worse with eating. Nausea and vomiting accompany this relentless pain, and there is some tenderness to RUQ palpation but no rebound tenderness.

DIAGNOSIS Diagnosis of cholelithiasis is made by serum blood tests that demonstrate elevated alkaline phosphatase, direct bilirubin, and signs and symptoms of concomitant hepatitis and pancreatitis. As cholesterol stones tend to be radiolucent and only the minority of

Â


USMLE STEP 2 432 stones are radiopaque (pigment stones), ultrasound is the best way to detect gallstones and diagnose cholelithiasis. Stones within the common bile duct are diagnosed and treated with ERCP.

TREATMENT Treatment of cholelithiasis is primarily symptomatic, and includes ERCP with sphincterotomy, lithotripsy, and the use of ursodeoxycholic acid (UDCA) and other oral bile acids to dissolve cholesterol stones. Supportive treatment with pain control and nausea and vomiting control are often used. Meperidine is preferred over morphine to avoid spasms at the sphincter of Oddi. TABLE 492 CHOLELITHIASIS Cholelithiasis Pathophysiology

Formation of gall stones (cholesterol or calcium bilirubinate) leads to stoppage of bile flow.

Presentation

RUQ pain, pain worse with eating. Nausea/vomiting, tenderness in RUQ.

Diagnosis

Elevated serum alkaline phosphatase, radiolucent cholesterol stones, ERCP.

Treatment

ERCP with sphincterotomy, lithotripsy, UDCA, pain control.

CHOLEDOCHOLITHIASIS Development or diagnosis of choledocholithiasis following laparoscopic cholecystectomy requires an ERCP to attempt stone removal. The success rate of this procedure is 90% with a 1% mortality. A sphincterotomy is routinely done as part of this procedure. Stones larger than 1.5cm or the presence of multiple stones may require a takeback to the operating room for a choledocholithotomy with choledochoduodenostomy. A contraindication to ERCP is the presence of duodenal diverticula. ESWL is a possible alternative procedure. The development of shock following ERCP may be due to a retroduodenal perforation. A CXR, KUB, and gastrograffin swallow study are mandatory as part of the diagnostic workup.

GALLSTONE ILEUS In gallstone ileus, the gallbladder can be left behind if significantly inflamed. The presence of small bowel obstruction and pneumobilia typically indicates the onset of gallstone ileus.

CHOLECYSTITIS ETIOLOGY AND PATHOPHYSIOLOGY Cholecystitis is inflammation and infection that results following cholelithiasis. Nearly ¾ of all presentations of cholecystitis have concurrent infection by enteric bacteria, including E. coli, Klebsiella, Enterococcus, and Bacteroides species. Other causes of cholecystitis include abscess and tumors.

PRESENTATION Charcot’s triad is positive in cholecystitis, including fever, jaundice, and RUQ pain. Guarding and rebound tenderness are common, and a positive Murphy’s sign is often present (inspiration is suddenly arrested during palpation of the RUQ). Murphy’s sign is very sensitive for cholecystitis.

www.ClinicalReview.com


Hepatopancreatobiliary System

433

DIAGNOSIS AND TREATMENT Diagnosis of cholecystitis is best made with a HIDA scan. Elevated PMNs and Charcot Triad: fever, jaundice, positive ultrasound findings increase the clinical suspicion of cholecystitis. RUQ pain Cholecystitis is treated with antibiotic administration, pain control with meperidine, and cholecystectomy for refractory cases. Complications of cholecystitis include the formation of abscesses and fistulas, gallstone ileus, and pancreatitis. One variant of cholecystitis is with acalculus cholecystitis, which occurs in about 10% of all cases. In this situation, there is increased morbidity and mortality. Risk factors for acalculus cholecystitis include increased age, systemic disorders such as diabetes, infections such as HIV, surgery, labor of pregnancy, gallbladder torsion, and vasculitis. Another variant of cholecystitis is emphysematous cholecystitis caused by Clostridium infection leading to gangrene and perforation. Immediate antibiotic treatment and surgery is required. TABLE 493 CHOLECYSTITIS Cholecystitis Pathophysiology

Infection & inflammation after cholelithiasis, infections with enteric bacteria.

Presentation

Charcot’s triad, rebound tenderness, Murphy’s sign.

Diagnosis

HIDA scan, high PMN, ultrasound findings.

Treatment

Antibiotics, meperidine, cholecystectomy in refractory cases.

COMPLICATIONS The morbidity due to a laparoscopic cholecystectomy is about 3%, and the mortality about 1%. Hemorrhage, infection, disability from anesthesia, biliary tree injuries, vascular injury, and organ injury are the most common problems. The use of carbon dioxide insufflation can lead to gas embolism, which may present as a sudden decrease in end tidal CO2. Treatment is to place the patient in the left lateral decubitus position to facilitate gas bubble absorption by the right atrium. An intraoperative cholangiogram performed following a laparoscopic cholecystectomy with no proximal ducts found should elicit open exploration. If a common bile duct injury occurs during a cholecystectomy, the proper treatment is returning the patient to the operating room and to permit an experienced surgeon perform a hepaticojejunostomy. Indications for converting to an open procedure include poor visualization of the anatomy, bleeding, anomalous anatomy, and the presence of adhesions. The presence of a cystic duct leak can be initially treated with ERCP for stenting. Otherwise, a cystic duct leak with a biloma is treated with a sphincterotomy followed by percutaneous drainage. A patient who undergoes an uncomplicated laparoscopic cholecystectomy but then develops a stricture at the left and right biliary ducts years later has cholangiocarcinoma until otherwise proven.

ASCENDING CHOLANGITIS ETIOLOGY AND PATHOPHYSIOLOGY Ascending cholangitis is due to obstruction of the common bile duct leading to obstruction of the biliary tree. This condition is also referred to as choledocholithiasis. Other causes include tumors and strictures that lead to the blockade of the biliary tree.

PRESENTATION Choledocholithiasis presents with Charcot’s triad, including fever, RUQ pain, and jaundice. It is not specific to choledocholithiasis and is present in only some cases. Reynold’s pentad may also develop, which includes the tripartite Charcot’s triad in addition to altered mental status and shock. Reynold’s pentad is an indicator of poor outcome.


434

USMLE STEP 2

DIAGNOSIS AND TREATMENT Diagnosis of choledocholithiasis is made by ultrasound. It is treated with IV fluids and vasopressors to maintain BP, antibiotics to cover enteric flora to prevent cholecystitis, use of ERCP to remove the biliary tree blockade, and surgery to permit the resumption of bile flow. Ascending cholangitis is an emergency. TABLE 494 ASCENDING CHOLANGITIS Ascending Cholangitis Pathophysiology

Obstruction of common bile duct.

Presentation

Charcot’s triad, fever, RUQ pain, jaundice.

Diagnosis

Ultrasound.

Treatment

IVs, vasopressors, antibiotics, ERCP to remove blockage, surgery.

PRIMARY SCLEROSING CHOLANGITIS (PSC) PATHOPHYSIOLOGY PSC is common in patients with ulcerative colitis, and presents as chronic, progressive inflammation and scarring of the bile ducts. It is thought that PSC is due to a hypersensitivity reaction.

DIAGNOSIS AND TREATMENT Diagnosis of PSC is made by ultrasound (US), ERCP, and PTC. PSC presents with elevated alkaline phosphatase, GGT, bilirubin, and symptoms of cholestasis and liver disease. Treatment is through surgical intervention to decompress the biliary tree. TABLE 495 PRIMARY SCLEROSING CHOLANGITIS (PSC) Primary Sclerosing Cholangitis (PSC) Pathophysiology

Chronic, progressing, inflammation and scarring of bile ducts.

Presentation

Elevated alkaline phosphatase, GGT, bilirubin.

Diagnosis

US, ERCP, PTC.

Treatment

Surgery.

CANCER AND TRANSPLANTATION PANCREATIC ADENOCARCINOMA ETIOLOGY AND PATHOPHYSIOLOGY Pancreatic cancer has a dismal prognosis, with over 90% of the patients dead within 1 year. The majority of adenocarcinomas are located at the head of the pancreas, with a concomitant rise in CA 19-9 and CA-50 levels. More advanced disease metastasizes to the liver. A point mutation in K-ras is attributed as the underlying cause for most pancreatic adenocarcinomas.

www.ClinicalReview.com


Hepatopancreatobiliary System

435

TREATMENT OF RESECTABLE CANCER For cancer located in the head of the pancreas without extensive metastasis, a Whipple procedure is the surgery of choice. A Whipple procedure consists of an antrectomy, cholecystectomy, choledochectomy, pancreaticoduodenectomy, proximal jejunectomy, excision of all regional lymph nodes, followed by reconstruction by a gastrojejunostomy, pancreaticojejunostomy, and choledochojejunostomy. Mortality following a Whipple surgery is about 5-10%. A fully resected tumor still requires 5-FU and radiation.

TREATMENT OF UNRESECTABLE CANCER The criteria for unresectable pancreatic cancer include the presence of distal metastasis (i.e. liver, pelvis, ligament of Treitz), regional metastasis to celiac nodes or lymph nodes posterior to the common bile duct, and locally invasive tumor into the portal vein or superior mesenteric vein. The presence of unresectable pancreatic cancer in the setting of worsening pain may be treated with a double pancreatic bypass and injection of the splanchnic bed. Chemoradiation with 5-FU is typically used as a palliative measure. Treatment is centered on reducing the clinically-significant symptoms, including jaundice, gastric obstruction, and pain. A cholecystojejunostomy or choledochojejunostomy can both be done for palliation. Gastrojejunostomy can be done for bowel obstruction. Pain can be treated through destruction of the celiac plexus with ethanol.

HEPATIC ADENOMA An adenoma of the liver is most likely to present with shock due to the high risk of bleeding, especially in pregnancy. About 33% are likely to hemorrhage. Treatment is to stop all oral contraceptives; if the adenoma does not improve, resection is necessary.

HEPATIC HEMANGIOMA No intervention is necessary unless the hemangioma is growing in size and becomes symptomatic. Hemangiomas are not likely to spontaneously rupture.

HEPATIC ANGIOSARCOMA Angiosarcoma of the liver may develop following exposure to vinyl chloride. This carcinogen also leads to cancer in the brain and lung. Vinyl chloride is used in the manufacture of plastics.

HEPATOCELLULAR CARCINOMA ETIOLOGY AND PATHOPHYSIOLOGY Primary hepatocellular carcinoma is a highly morbid and lethal cancer that commonly occurs following chronic liver injury such as cirrhosis. However, up to one quarter of all patients may develop HCC spontaneously. Metastasis from the liver to the bones, brain, and lungs is common in late stage HCC. HCC composes 2% of all cancers and affects some 10,000 new persons annually. Males are more affected than females. The most common causes include chronic alcoholism, HBV, HCV, hemochromatosis, and aflatoxin poisoning.

PRESENTATION AND DIAGNOSIS

Â


USMLE STEP 2 436 HCC presents with signs and symptoms of liver failure, including jaundice, pruritus, hepatosplenomegaly (often with hepatic nodules as in cirrhosis), bleeding diatheses, cachexia, encephalopathy, asterixis, ascites, and varices. LFTs are elevated along with AFP. Biopsy confirms the diagnosis, while imaging studies are used to determine the extent of disease and whether a surgical approach could be beneficial. The presence of a liver mass in a patient with a history of cirrhosis is hepatocellular carcinoma unless otherwise proven. Over 90% of patients will have an elevation in alpha-fetoprotein. All liver metastases must have an arterial supply and so are susceptible to chemoembolization. A liver tumor with a central, stellate scar is focal nodular hyperplasia.

TREATMENT Vaccination for HBV and HCV is protective, and avoiding alcohol is important. Complications of hepatic failure are treated to minimize patient discomfort. Paracentesis and diuretics are often used to treat ascites. Lactulose is used to reduce the symptoms of hepatic encephalopathy. Ursodiol is used for the treatment of significant pruritus. Band ligation is used to cease variceal hemorrhage. Prophylactic antibiotics to avoid SBP may also be used. Chemotherapy is used but has poor response. Use of chemoembolization techniques to provide targeted tumor therapy is beneficial in some patients. Surgical resection with clear margins is the definitive care, but this approach can only be done in only a small percentage of patients; due to missed metastases, there is a high relapse rate even with this aggressive procedure. Transplantation is also an option for localized disease. The most common site of metastasis is the lung. Long term survival from a single colorectal liver metastasis has a 25-50% 5 year survival. TABLE 496 HEPATOCELLULAR CARCINOMA Hepatocellular Carcinoma Etiology

Chronic alcoholism, HBV, HCV, hemochromatosis, and aflatoxin poisoning.

Presentation

Jaundice, pruritus, hepatosplenomegaly (often with hepatic nodules as in cirrhosis), bleeding diatheses, cachexia, encephalopathy, asterixis, ascites, and varices.

Diagnosis

Biopsy confirms the diagnosis. LFTs are elevated along with AFP.

Treatment

Avoiding alcohol, vaccination is important. Complications are treated to minimize patient discomfort. Paracentesis and diuretics, lactulose, ursodiol, band ligation, and, prophylactic antibiotics. Surgical resection with clear margins is the definitive care. Transplantation is also an option for localized disease.

PRACTICE QUESTIONS What is the source of the superior anterior pancreaticoduodenal artery? A. B. C. D. E.

Celiac artery Common hepatic artery Gastroduodenal artery Proper hepatic artery Superior mesenteric artery

The best answer is Gastroduodenal artery. The superior anterior and posterior pancreaticoduodenal arteries are a branch of the gastroduodenal artery. The inferior anterior and posterior pancreaticoduodenal arteries are a branch of the superior mesenteric artery. As the gastroduodenal artery is also removed during a Whipple procedure, the duodenum is also removed due to the loss of its blood supply.

www.ClinicalReview.com


Hepatopancreatobiliary System

437

What is the source of the gastroduodenal artery? A. B. C. D. E.

Celiac artery Common hepatic artery Left gastric artery Proper hepatic artery Superior mesenteric artery

The best answer is Common hepatic artery. The gastroduodenal artery is a direct branch of the common hepatic artery before it becomes the proper hepatic artery. Damage to the common hepatic artery typically does not compromise the gastroduodenal artery due to its anastomosis with the superior mesenteric artery via the pancreaticoduodenal branches, and liver function is typically preserved. However, damage to the proper hepatic arteries often leads to liver ischemia.

The right hepatic artery is a branch of which of the following? A. B. C. D. E.

Celiac trunk Common hepatic artery Gastroduodenal artery Proper hepatic artery Superior mesenteric artery

The best answer is Proper hepatic artery. The right hepatic artery is a branch of the proper hepatic artery, which itself comes off the common hepatic artery after it gives rise to the cystic artery and gastroduodenal artery. The common hepatic artery is a branch of the celiac artery.

Which of the following best describes the anatomy of the common bile duct? A. B. C. D. E.

Formed by the cystic duct and hepatic duct in the gastroduodenal ligament, and lies posterior to the portal vein Formed by the cystic duct and hepatic duct in the hepatoduodenal ligament, and lies anterior to the portal vein Formed by the cystic duct and hepatic duct in the hepatoduodenal ligament, and lies posterior to the portal vein Formed by the cystic duct and hepatic duct in the hepatoduodenal ligament, and lies lateral to the portal vein Formed by the cystic duct and hepatic duct, and is supplied by the cystic artery

The best answer is The CBD is formed by the cystic duct and hepatic duct in the hepatoduodenal ligament, and lies anterior to the portal vein. The common bile duct is formed by the cystic duct and hepatic duct, lies within the hepatoduodenal ligament, lies anterior to the portal vein, and is supplied by the right hepatic artery. Damage to this structure, along with the right hepatic artery, is an unfortunate complication during a laparoscopic cholecystectomy and necessitates a hepaticojejunostomy as the drainage procedure for bile from the liver.

Â


USMLE STEP 2

438

Which of the following is a benign hepatic tumor with significant potential for bleeding and malignancy? A. B. C. D. E.

Adenoma Cavernous hemangioma Focal nodular hyperplasia Hamartoma Nodular regenerative hyperplasia

The best answer is Adenoma. Adenomas of the liver have significant potential to transform into a malignant tumor. They also have a significant risk of bleeding, and are the only one of the five listed that may require surgical intervention for resection.

A 36 year old male is diagnosed with hepatocellular carcinoma. Which of the following markers should be tested first? A. B. C. D. E.

AFP CA19-9 CEA hCG p53

The best answer is AFP. Alpha-fetoprotein is associated with 80% of all hepatocellular carcinomas, and should be tested first as a prognostic indicator for disease.

Which of the following laboratory tests is not used as part of Ranson’s criteria for predicting mortality in acute pancreatitis? A. B. C. D. E.

Amylase AST Glucose LDH WBC

The best answer is Amylase. The prognosis of acute pancreatitis is gauged by Ranson criteria. Mortality depends on the number of risk factors present. The risk factors are divided into those present on admission and those that develop after two days. Risk factors that increase the risk of mortality and may present on admission include age over 55, elevations in blood sugar over 200, elevated white blood cell (WBC) counts over 16,000, elevated aspartate transaminase (AST) over 250, and elevated lactate dehydrogenase (LDH) over 350. Risk factors present after two days include a decrease of more than 10% in the hematocrit (HCT), increase in BUN over 5, calcium less than 8, PO2 less than 60 mmHg, a base deficit greater than 4, and a fluid deficit of more than 6 L.

www.ClinicalReview.com


Hepatopancreatobiliary System

439

A 46 year old male comes to you for painless jaundice nearly a decade after having his gallbladder removed laparoscopically. Delta-bilirubin is elevated. You make the diagnosis of cholangiocarcinoma. Which of the following is the next best step? A. B. C. D. E.

Common bile duct resection with roux-en-y hepaticojejunostomy ERCP Percutaneous transhepatic cholangiography Right hepatic lobectomy Whipple procedure

The best answer is Whipple procedure. This patient has cholangiocarcinoma, which leads to biliary disease, obstruction, and presentation as painless jaundice. It can happen to people who have had their gallbladders taken out. Diagnosis is made by ERCP and PTC, and definitive treatment is excision via a Whipple procedure.

Which of the following is the most common cause of a bacterial liver abscess? A. B. C. D. E.

Bacteroides E. coli Klebsiella Pseudomonas Streptococcus

The best answer is E. coli. E. coli is the leading cause of bacterial liver abscess formation. Treatment with IV antibiotics and percutaneous drainage is needed.

A 26 year old female presents with significant right upper quadrant pain, fever, chills, nausea, and elevated liver function tests. A diagnosis of acute cholangitis is made. The patient is admitted to a stepdown unit, started on fluids and antibiotics, and has an ERCP the following day after an ultrasound shows an obstructing common duct stone. The ERCP is unsuccessful. What is the next step in management? A. B. C. D. E.

Common duct exploration in the OR HIDA scan Laparoscopic cholecystectomy Percutaneous cholecystostomy tube Percutaneous transhepatic catheter

The best answer is Common duct exploration in the OR. This patient has failed an ERCP and will likely not improve on her own at this point. She requires a trip to the operating room for a common duct exploration. It is possible that she may require a hepaticojejunostomy and cholecystectomy. The findings of black stones is due to calcium bilirubinate secondary to Crohn disease, hemolytic disease, or resection of the terminal ileum; brown stones consist of calcium palmitate and are due to bacterial infection; yellow stones are composed primarily of cholesterol.

Â


USMLE STEP 2

440

A 56 year old woman with acute on chronic cholecystitis goes to the OR for a laparoscopic cholecystectomy. She is found to have frank pus within her gallbladder and goes to the ICU after the case for some intraoperative hypotension. Which of the following is the most appropriate antibiotic regimen? A. B. C. D. E.

Ampicillin and gentamicin Linezolid Piperacillin / Tazobactam Vancomycin, Piperacillin / Tazobactam Vancomycin, Piperacillin / Tazobactam, Fluconazole

The best answer is Piperacillin / Tazobactam. This patient likely has a gallbladder infection from one of the PECKS bacteria: Proteus, Enterobacter or E. coli (most common), Clostridium, Klebsiella, or Streptococci. The best antibiotic to begin initial therapy is piperacillin / tazobactam. Additional agents can be added based on culture data and patient status.

Which of the following hormones stimulates the flow of bile the most? A. B. C. D. E.

CCK Gastrin Secretin Somatostatin VIP

The best answer is Secretin. Secretin leads to the greatest increase in the flow of bile, while CCK predominantly leads to gallbladder contraction.

A 51 year old female who had a diverting loop ileostomy at a young age after resection of much of her large intestine secondary to Crohn disease is found to have an incidental parastomal hernia during a laparoscopic cholecystectomy. Which of the following is the best step in management? A. B. C. D. E.

Abort the cholecystectomy and repair the parastomal hernia Abort the entire operation to obtain informed consent Complete the laparoscopic cholecystectomy only Do both the cholecystectomy and parastomal hernia repair Redo the colostomy at another site after the cholecystectomy

The best answer is Complete the laparoscopic cholecystectomy only. This patient has an asymptomatic parastomal hernia. She should have her laparoscopic cholecystectomy at this time and be monitored as an outpatient for signs and symptoms of obstruction related to her parastomal hernia. An immediate repair at this time is not indicated, and there is no reason to abort the cholecystectomy. www.ClinicalReview.com


Hepatopancreatobiliary System

441

A 38 year old longtime alcoholic begins to develop cirrhosis and ascites. Which of the following is the earliest mediator of the ascites? A. B. C. D. E.

Aldosterone Angiotensin II Nitric oxide Renin Substance P

The best answer is Nitric oxide. Nitric oxide release from vasodilation leads to the release of renin and activation of the RAA axis. The effect is an increase in aldosterone, leading to salt and fluid retention. A normal portal pressure is between 3-5; when this level rises to over 12, variceal bleeding can ensue. Treatment is with a TIPS procedure, but the common side effect of this procedure is encephalopathy. The ultimate therapy is liver transplantation. The best measure of synthetic function by the liver is protime.

Which of the following pancreatic enzymes are secreted in their active form? A. B. C. D. E.

Alpha-reductase Chymotrypsin Enterokinase Lipase Trypsin

The best answer is Lipase. Lipase and amylase are two pancreatic enzymes secreted in their active forms. All other pancreatic enzymes are secreted in their inactive form until they are activated by enterokinase in the duodenum, trypsin in the duodenum, or the acidic pH of the duodenum. Secretion of pancreatic enzymes is stimulated by CCK and acetylcholine. Secretin leads to stimulation of pancreatic fluid laden with bicarbonate.

Â


USMLE STEP 2

442

During an intraoperative cholangiogram performed as part of a laparoscopic cholecystectomy, a stricture in the right hepatic duct is noted. Of note, the patient has had multiple bouts of pancreatitis, which is believed to be the underlying cause of this stricture. What is the next best step? A. B. C. D. E.

ERCP and brush biopsy with metal stent placement Liver transplantation Monitoring with CT scan Percutaneous transhepatic catheter placement Roux-en-y hepaticojejunostomy

The best answer is ERCP and brush biopsy with metal stent placement. While it is likely that this hepatic duct stricture is benign, a definitive diagnosis is required prior to an operative intervention. Therefore, the best step is to complete the cholecystectomy, close the patient, and obtain an ERCP with brush biopsy. A metal stent may be required if the stricture cannot be dilated safely. If the pathology returns as negative, this patient is eligible for a roux-en-y hepaticojejunostomy. A PTC can also be completed, especially if the ERCP is not successful.

A 38 year old female who has taken birth control pills for many years develops sudden hypotension. She is taken to the ICU, where her hematocrit is 14. After stabilizing her, an exploratory laparoscopy is done, which indicates evidence of bleeding from a rupture located in her liver. What is the underlying etiology? A. B. C. D. E.

Amebic abscess Focal nodular hyperplasia Hemangioma Hepatic adenoma Hepatocellular carcinoma

The best answer is Hepatic adenoma. This patient has a hepatic adenoma which developed due to long-term birth control medication. The most significant and dangerous risk of hepatic adenoma is rupture leading to bleeding and death. Hepatic adenoma can also lead to carcinogenesis. Once detected, birth control pills should be stopped and the adenoma closely monitored. Failure of the adenoma to regress requires resection. A hemangioma does not require operative intervention due to the low risk of rupture. FNH also does not require operative intervention unless it is changing size; it can be evaluated via a technetium 99 scan. Amebic liver abscesses require metronidazole and percutaneous drainage. Hepatocellular carcinoma can be resected if it is the fibrolamellar type, but typically requires chemoembolization, RFA, and possible liver transplant for definitive cure.

www.ClinicalReview.com


Turn static files into dynamic content formats.

Create a flipbook
Issuu converts static files into: digital portfolios, online yearbooks, online catalogs, digital photo albums and more. Sign up and create your flipbook.